You are on page 1of 237

ACP IM KSAP® �

Medical Knowledge Self-Assessment Program®

Cardiovascular Medicine

A

��
1c p American College of Physicians®
Leading Internal Medicine, Improving Lives
Welcome to the Cardiovascular
Medicine Section of MKSAP 17!
In these pages, you will find updated information on risk assessment in cardiovascular disease, diagnostic testing, coronary
artery disease, heart failure, arrhythmias, pericardia! and myocardial disease, valvular heart disease, and other clinical chal­
lenges. All of these topics are uniquely focused on the needs of generalists and subspecialists outside of cardiovascular medicine.

The publication of the 17th edition of Medical Knowledge Self-Assessment Program (MKSAP) represents nearly a half-century
of serving as the gold-standard resource for internal medicine education. It also marks its evolution into an innovative learn­
ing system to better meet the changing educational needs and learning styles of all internists.

The core content of MKSAP has been developed as in previous editions-newly generated, essential information in 11 topic
areas of internal medicine created by dozens of leading generalists and subspecialists and guided by certification and recer­
tification requirements, emerging knowledge in the field, and user feedback. MKSAP 17 also contains 1200 all-new, psycho­
metrically validated, and peer-reviewed multiple-choice questions (MCQs) for self-assessment and study, including 120 in
Cardiovascular Medicine. MKSAP 17 continues to include High Value Care (HVC) recommendations, based on the concept
of balancing clinical benefit with costs and harms, with links to MCQs that illustrate these principles. In addition, HVC Key
Points are highlighted in the text. Also highlighted, with blue text, are Hospitalist-focused content and MCQs that directly
address the learning needs of internists who work in the hospital setting.

MKSAP 17 Digital provides access to additional tools allowing you to customize your learning experience, including regular
text updates with practice-changing, new information and 200 new self-assessment questions; a board-style pretest to help
direct your learning; and enhanced custom-quiz options. And, with MKSAP Complete, learners can access 1200 electronic
flashcards for quick review of important concepts or review the updated and enhanced version of Virtual Dx, an image-based
self-assessment tool.

As before, MKSAP 17 is optimized for use on your mobile devices, with iOS- and Android-based apps allowing you to sync
your work between your apps and online account and submit for CME credits and MOC points online.

Please visit us at the MKSAP Resource Site (rnksap.acponline.org) to find out how we can help you study, earn CME credit
and MOC points, and stay up to date.

Whether you prefer to use the traditional print version or take advantage of the features available through the digital version,
we hope you enjoy MKSAP 17 and that it meets and exceeds your personal learning needs.

On behalf of the many internists who have offered their time and expertise to create the content for MKSAP 17 and the editorial staff
who work to bring this material to you in the best possible way, we are honored that you have chosen to use MKSAP 17 and appreci­
ate any feedback about the program you may have. Please feel free to send us any comments to mksap_editors@acponline.org.

Sincerely,

Philip A. Masters, MD, FACP


Editor-in-Chief
Senior Physician Educator
Director, Content Development
Medical Education Division
American College of Physicians

ii
Cardiovascular Medicine

Committee Donna Poll<, MD, MPH2


Program Director, Cardiovascular Medicine Fellowship
Andrew Wang, MD, Section Editor2
Brigham and Women's Hospital
Professor of Medicine
Boston, Massachusetts
Director, Cardiovascular Disease Fellowship Program
Division of Cardiology Stuart D. Russell, MD2
Duke University Medical Center Associate Professor of Medicine
Durham, North Carolina Chief, Heart Failure and Transplantation
Johns Hopkins School of Medicine
Howard H. Weitz, MD, MACP, Associate Editor1
Baltimore, Maryland
Professor of Medicine
Director, Jefferson Heart Institute Paul Sorajja, MD1
Director, Division of Cardiology Director, Center for Valve and Structural Heart Disease
Vice-Chairman, Department of Medicine Senior Consulting Cardiologist
Sidney Kimmel Medical College at Thomas Jefferson Program Director, Structural Heart Interventional
University Fellowship
Philadelphia, Pennsylvania Minneapolis Heart Institute at Abbott Northwestern Hospital
Minneapolis, Minnesota
Jeffrey S. Berger, MD, MS2
Assistant Professor of Medicine (Cardiology and
Hematology) Editor-in-Chief
Assistant Professor of Surgery (Vascular Surgery)
Philip A. Masters, MD, FACP1
Director of Cardiovascular Thrombosis
Director, Clinical Content Development
New York University School of Medicine
American College of Physicians
New York, New York
Philadelphia, Pennsylvania
Heidi M. Connolly, MD1
Professor of Medicine
Director, Clinical Program Development
Mayo Clinic College of Medicine
Cynthia D. Smith, MD, FACP2
Rochester, Minnesota
American College of Physicians
W. Schuyler Jones, MD2 Philadelphia, Pennsylvania
Assistant Professor of Medicine
Division of Cardiology
Duke University Medical Center and Durham Veterans Cardiovascular Medicine Reviewers
Affairs Medical Center Joel S. Karliner, MD1
Durham, North Carolina Kent A. Kirchner, MD, FACP1
Mark E. Pasanen, MD, FACP1
Andrew M. Kates, MD2
Michael W. Peterson, MD, FACP1
Associate Professor of Medicine
Ileana L. Pina, MD, MPH2
Director, Cardiovascular Fellowship Program
Mark D. Siegel, MD, FACP1
Washington University School of Medicine
St. Louis, Missouri
Cardiovascular Medicine
Jonathan P. Piccini, MD, MHS2
Associate Professor of Medicine
ACP Editorial Staff
Division of Cardiology Jackie Twomey1, Staff Editor
Duke University Medical Center Margaret Wells1, Director, Self-Assessment and Educational
Duke Clinical Research Institute Programs
Durham, North Carolina Becky Krumm1, Managing Editor

iii
Jeffrey S. Berger, MD, MS
ACP Principal Staff
Research Grants/Contracts
Patrick C. Alguire, MD, FACP2
AstraZeneca, American Heart Association,
Senior Vice President, Medical Education
Doris Duke Charitable Foundation,
Sean McKinney1 National Institutes of Health
Vice President, Medical Education Consultantship
Bristol-Myers Squibb, Takeda Pharmaceuticals
Margaret Wells1
Other
Director, Self-Assessment and Educational Programs
Maintenance of Certification: Pri-Med; Planning
Becky Krumm1 Committee: American College of Cardiology
Managing Editor
W. Schuyler Jones, MD
Valerie A. Dangovetsky1 Other
Administrator American Physician Institute
Research Grants/Contracts
Ellen McDonald, PhD1
AstraZeneca, Bristol-Myers Squibb, American Heart
Senior Staff Editor
Association, Boston Scientific
Katie ldell1
Andrew M. Kates, MD
Digital Content Associate/Editor
Speakers Bureau
Megan Zborowski1 Pfizer, American College of Cardiology, MCE Medical
Senior Staff Editor
Jonathan P. Piccini, MD, MHS
Randy Hendrickson1 Consultantship
Production Administrator/Editor Medtronic, Forest Laboratories, Pfizer /Bristol-Myers Squibb,
Linnea Donnarumma1
Spectranetics, Johnson & Johnson
Staff Editor Research Grants/Contracts
Janssen Pharmaceuticals, GE Healthcare, Boston Scientific,
Susan Galeone1 ARCA Biopharma, ResMed
Staff Editor
Ileana L. Piiia, MD, MPH
Jackie 1\vomey1
Employment
Staff Editor
Montefiore-Einstein Medical Center
Kimberly Kems1 Research Grants/Contracts
Administrative Coordinator Duke University/National Institutes of Health
Royalties

1 . Has no relationships with any entity producing, marketing, reselling, or


UpToDate
distributing health care goods or services consumed by, or used on, patients. Consultantship
2. Has disclosed relationship(s) with any entity producing, marketing, reselling, or Novartis
distributing health care goods or services consumed by, or used on, patients.

Donna Polk, MD, MPH


Disclosure of Relationships with any entity producing, Board Member
marketing, reselling, or distributing health care goods American Society of Nuclear Cardiology
or services consumed by, or used on, patients: Honoraria
American College of Cardiology
Patrick C. Alguire, MD, FACP
Board Member
Stuart D. Russell, MD
Teva Pharmaceuticals Consultantship
Consultantship
Thoratec, Amgen, SulfaGENIX
National Board of Medical Examiners
Speakers Bureau
Royalties
American College of Cardiology, Heart Failure
UpToDate Society of America
Stock Options/Holdings
Amgen Inc, Bristol-Myers Squibb, Covidien, GlaxoSmithKline, Cynthia D. Smith, MD, FACP
Stryker Corporation, Zimmer Orthopedics, Teva Stock Options/Holdings
Pharmaceuticals, Express Scripts, Medtronic Merck and Co.; spousal employment at Merck

iv
Andrew Wang, MD used to apply to the American Board of Internal Medicine
Research Grants/Contracts for Maintenance of Certification (MOC) points.
Abbott Vascular, Edwards Lifesciences, Gilead Sciences,
American Heart Association
Consultantship Royal College Maintenance
American College of Cardiology Foundation of Certification
Other
In Canada, MKSAP 17 is an Accredited Self-Assessment
Expert reviewer for legal case of infective endocarditis
Program (Section 3) as defined by the Maintenance of
Certification (MOC) Program of The Royal College of
Acknowledgments Physicians and Surgeons of Canada and approved by
the Canadian Society of Internal Medicine on December
The American College of Physicians (ACP) gratefully
9, 2014. Approval extends from July 31, 2015 until July
acknowledges the special contributions to the develop­
31, 2018 for the Part A sections. Approval extends from
ment and production of the 17th edition of the Medical
December 31, 2015 to December 31, 2018 for the Part B
Knowledge Self-Assessment Program® (MKSAP® 17) made
sections.
by the following people:
Fellows of the Royal College may earn three credits per
Graphic Design: Michael Ripca (Graphics Technical
hour for participating in MKSAP 17 under Section 3.
Administrator) and WFGD Studio (Graphic Designers).
MKSAP 17 also meets multiple CanMEDS Roles, includ­
Production/Systems: Dan Hoffmann (Director, Web ing that of Medical Expert, Communicator, Collaborator,
Services & Systems Development), Neil Kohl (Senior Manager, Health Advocate, Scholar, and Professional.
Architect), Chris Patterson (Senior Architect), and Scott For information on how to apply MKSAP 17 Continuing
Hurd (Manager, Web Projects & CMS Services). Medical Education (CME) credits to the Royal College
MOC Program, visit the MKSAP Resource Site at
MKSAP 17 Digital: Under the direction of Steven Spadt,
mksap.acponline.org.
V ice President, Digital Products & Services, the digital ver­
sion of MKSAP 17 was developed within the ACP's Digital
Product Development Department, led by Brian Sweigard
(Director). Other members of the team included Dan The Royal Australasian College
Barron (Senior Web Application Developer/Architect), of Physicians CPD Program
Chris Forrest (Senior Software Developer/Design Lead), In Australia, MKSAP 17 is a Category 3 program that may
Kara Kronenwetter (Senior Web Developer), Brad Lord be used by Fellows of The Royal Australasian College
(Senior Web Application Developer), John McKnight of Physicians (RACP) to meet mandatory Continuing
(Senior Web Developer), and Nate Pershall (Senior Web Professional Development (CPD) points. Two CPD cred­
Developer). its are awarded for each of the 200 AMA PRA Category 1
Credits"' available in MKSAP 17. More information about
The College also wishes to acknowledge that many other
using MKSAP 17 for this purpose is available at the MKSAP
persons, too numerous to mention, have contributed to
Resource Site at mksap.acponline.org and at www.racp.
the production of this program. Without their dedicated
edu.au. CPD credits earned through MKSAP 17 should be
efforts, this program would not have been possible.
reported at the MyCPD site at www.racp.edu.au/mycpd.

MKSAP Resource Site


Continuing Medical Education
(mksap.acponline.org)
The American College of Physicians (ACP) is accredited
The MKSAP Resource Site (mksap.acponline.org) is a
by the Accreditation Council for Continuing Medical
continually updated site that provides links to MKSAP 17
Education (ACCME) to provide continuing medical educa­
online answer sheets for print subscribers; the latest
tion for physicians.
details on Continuing Medical Education (CME) and
Maintenance of Certification (MOC) in the United States, The ACP designates this enduring material, MKSAP 17,
Canada, and Australia; errata; and other new information. for a maximum of 200 AMA PRA Category I Creditsrn.
Physicians should claim only the credit commensurate
with the extent of their participation in the activity.
ABIM Maintenance of Certification Up to 21 AMA PRA Category 1 Credits"' are available
Check the MKSAP Resource Site (mksap.acponline.org) from July 31, 2015, to July 31, 2018, for the MKSAP 17
for the latest information on how MKSAP tests can be Cardiovascular Medicine section.

v
Learning Objectives your MKSAP 17 slipcase. You will need your 10-digit
The learning objectives of MKSAP 17 are to: order number and 8-digit ACP ID number, which are
• Close gaps between actual care in your practice and
printed on your packing slip. Please allow 4 to 6 weeks
preferred standards of care, based on best evidence for your score report to be emailed back to you. Be sure
• Diagnose disease states that are less common and
to include your email address for a response.
sometimes overlooked or confusing If you do not have a 10-digit order number and 8-digit
• Improve management of comorbid conditions that can
ACP ID number or if you need help creating a user name
complicate patient care and password to access the MKSAP 17 online answer
• Determine when to refer patients for surgery or care by sheets, go to mksap.acponline.org or email custserv@
subspecialists acponline.org.
• Pass the ABIM Certification Examination
• Pass the ABIM Maintenance of Certification Examination
Disclosure Policy
Target Audience It is the policy of the American College of Physicians
(ACP) to ensure balance, independence, objectivity, and
• General internists and primary care physicians
scientific rigor in all of its educational activities. To this
• Subspecialists who need to remain up-to-date in internal
end, and consistent with the policies of the ACP and the
medicine and in areas outside of their own subspecialty
Accreditation Council for Continuing Medical Education
area
(ACCME), contributors to all ACP continuing medical
• Residents preparing for the certification examination in
education activities are required to disclose all relevant
internal medicine
financial relationships with any entity producing, mar­
• Physicians preparing for maintenance of certification in
keting, re-selling, or distributing health care goods or
internal medicine (recertification)
services consumed by, or used on, patients. Contributors
are required to use generic names in the discussion of
Earn "Instantaneous" CME therapeutic options and are required to identify any unap­
proved, off-label, or investigative use of commercial prod­
Credits Online
ucts or devices.Where a trade name is used, all available
Print subscribers can enter their answers online to earn
trade names for the same product type are also included.
instantaneous Continuing Medical Education (CME) cred­
If trade-name products manufactured by companies with
its. You can submit your answers using online answer
whom contributors have relationships are discussed, con­
sheets that are provided at mksap.acponline.org, where a
tributors are asked to provide evidence-based citations
record of your MKSAP 17 credits will be available. To earn
in support of the discussion. The information is reviewed
CME credits, you need to answer all of the questions in
by the committee responsible for producing this text. If
a test and earn a score of at least 50% correct (number of
necessary, adjustments to topics or contributors' roles in
correct answers divided by the total number of questions).
content development are made to balance the discussion.
Take any of the following approaches:
Further, all readers of this text are asked to evaluate the
1. Use the printed answer sheet at the back of this book to content for evidence of commercial bias and send any rel­
record your answers. Go to mksap.acponline.org, access evant comments to mksap_editors@acponline.org so that
the appropriate online answer sheet, transcribe your future decisions about content and contributors can be
answers, and submit your test for instantaneous CME made in light of this information.
credits. There is no additional fee for this service.

2. Go to mksap.acponline.org, access the appropriate


Resolution of Conflicts
online answer sheet, directly enter your answers, and
To resolve all conflicts of interest and influences of vested
submit your test for instantaneous CME credits. There is
interests, the American College of Physicians (ACP) pre­
no additional fee for this s ervice .
cluded members of the content-creation committee from
3. Pay a $15 processing fee per answer sheet and submit deciding on any content issues that involved generic or
the printed answer sheet at the back of this book by trade-name products associated with proprietary entities
mail or fax, as instructed on the answer sheet. Make with which these committee members had relationships. In
sure you calculate your score and fax the answer sheet addition, content was based on best evidence and updated
to 215-351-2799 or mail the answer sheet to Member clinical care guidelines, when such evidence and guidelines
and Customer Service, American College of Physicians, were available. Contributors' disclosure information can be
190 N. Independence Mall West, Philadelphia, PA found with the list of contributors' names and those of ACP
19106-1572, using the courtesy envelope provided in principal staff listed in the beginning of this book.

vi
Hospital-Based Medicine Publisher's Information
For the convenience of subscribers who provide care in Copyright© 2015 American College of Physicians. All
hospital settings, content that is specific to the hospital rights reserved.
setting has been highlighted in blue. Hospital icons (CJ)
This publication is protected by copyright. No part of
highlight where the hospital-based content begins,
this publication may be reproduced, stored in a retrieval
continues over more than one page, and ends.
system, or transmitted in any form or by any means, elec­
tronic or mechanical, including photocopy, without the
express consent of the American College of Physicians.
High Value Care Key Points
MKSAP 17 is for individual use only. Only one account
Key Points in the text that relate to High Value Care
per subscription will be permitted for the purpose of
concepts (that is, concepts that discuss balancing clinical
earning Continuing Medical Education (CME) credits and
benefit with costs and harms) are designated by the
Maintenance of Certification (MOC) points/credits and for
HVC icon (HVC).
other authorized uses of MKSAP 17.

Educational Disclaimer
The editors and publisher of MKSAP 17 recognize that Unauthorized Use of This Book
the development of new material offers many opportu­ Is Against the Law
nities for error. Despite our best efforts, some errors may Unauthorized reproduction of this publication is unlaw­
persist in print. Drug dosage schedules are, we believe, ful.The American College of Physicians (ACP) prohibits
accurate and in accordance with current standards. reproduction of this publication or any of its parts in any
Readers are advised, however, to ensure that the recom­ form either for individual use or for distribution.
mended dosages in MKSAP 17 concur with the informa­
The ACP will consider granting an individual permission
tion provided in the product information material. This is
to reproduce only limited portions of this publication for
especially important in cases of new, infrequently used,
his or her own exclusive use. Send requests in writing to
or highly toxic drugs. Application of the information in
MKSAP"' Permissions, American College of Physicians, 190
MKSAP 17 remains the professional responsibility of
N. Independence Mall West, Philadelphia, PA 19106-1572,
the practitioner.
or email your request to mksap_editors@acponline.org.
The primary purpose of MKSAP 17 is educational.
MKSAP 17 ISBN: 978-1-938245-18-3
Information presented, as well as publications, technol­
(Cardiovascular Medicine) ISBN: 978-1-938245-19-0
ogies, products, and/or services discussed, is intended
to inform subscribers about the knowledge, techniques, Printed in the United States of America.
and experiences of the contributors. A diversity of pro­
For order information in the United States or Canada
fessional opinion exists, and the views of the contributors
call 800-523-1546, extension 2600. All other countries
are their own and not those of the American College of
call 215-351-2600, (M-F, 9 AM 5 PM ET). Fax inquiries to
-

Physicians (ACP). Inclusion of any material in the pro­


215-351-2799 or email to custserv@acponline.org.
gram does not constitute endorsement or recommenda­
tion by the ACP. The ACP does not warrant the safety,
reliability, accuracy, completeness, or usefulness of and
disclaims any and all liability for damages and claims Errata
that may result from the use of information, publica­ Errata for MKSAP 17 will be available through the MKSAP
tions, technologies, products, and/or services discussed Resource Site at mksap.acponline.org as new information
in this program. becomes known to the editors.

vii
Table of Contents

Epidemiology and Risk Factors Management of Coronary Artery Disease in


Overview . . . . . . . . . . . . . . . . . . . . . . . . . . . . . . . . . . . . . . . 1 Patients with Diabetes Mellitus . . . . . . . . . . . . . . . . . . . . 31

Cardiovascular Disease in Women . . . . . . . . . . . . . . . . . 1 .


Risk and Evaluation . . . . . . . . . . . . . . . . . . . . . . . . . 31

Ethnicity and Cardiovascular Disease . . . . . . . . . .. . .. . . 1 Invasive Treatment . . . . . .. . . . . . . . . .. . . . . . . . . . 31

Genetics in Cardiovascular Disease . . . . . . . . . . . . . . . . . . 2 Medical Therapy and Secondary Prevention . . . . . 31

Lifestyle Risk Factors . . . . . . . . . . . . . . . . .. . . . . . . . . . . . 2


Specific Risk Groups . . . . . . . . . . . . . . . . . . . . . . . . . . . . . 3 .
Heart Failure
Diabetes Mellitus . . . . . . . . . . . . . . . . . . . . . . . . . . . .. 3 Pathophysiology of Heart Failure ... . . ...... . .. . ... 32
Chronic Kidney Disease . . . . . . . . . . . . . . . . . . . . . .. 3 Diagnosis and Evaluation of Heart Failure . . . . . . . . . . . 32
Systemic Inflammatory Conditions . . . . . . . . . . . . . . 3 Clinical Evaluation . . . . . . . . . . . . . . . . . . . . .. . . . . 32
Calculating Cardiovascular Risk . . . . . . . . . . . . . . . . . . . . 3 Diagnosis . . . . . . . . . . . . . . . . . . . . . . . . . . . . . . . . . . 32
Emerging Risk Factors . . . . . . . . . . . . . . . . . . . . . . . . . . . . 3 Evaluation for Ischemia . . . . . . . . . . . . . . . . . . . . . . 33
Aspirin for Primary Prevention . . . . . . . . . . . . . . . . . . . .. 4 Medical Therapy for Systolic
Heart Failure . . . . . . . . . . . . . . . . . . . . . . . . . . . . . . . . . . . 34
ACE Inhibitors and Angiotensin
Diagnostic Testing in Cardiology
Receptor Blockers .. . . . . . . . . . . .. . . . . . . . .. ... . 34
Clinical History and Physical Examination . . . . . . . . . . . 5 B-Blockers . . . . . . . . . . . . . . . . . . . . . . . . . . . . . . . . . 35
Diagnostic Testing for Atherosclerotic Initiating and Managing ACE Inhibitor
Coronary Disease .. .. . . . . . . . . . . . . . . . . . . . . . . . . . . . . 5 and B-Blocker Therapy . . . . ... . .. . . . . . . . . . . . . . 35
Cardiac Stress Testing . . . . . . . . . . . . . . . . . . . . . . . . 5 .
Diuretics . . . . . . . . . . . . . . . . . . . . . . . . . . . . . . . . . . . 35
Visualization of the Coronary Anatomy . . . . . . . . . 10 Digoxin . . . . . . . . . . . . . . . . . . . . . . . . . . . . . . . . .. 35
. .

Coronary Artery Calcium Scoring . . . . . . . . . . . . . 11 .


Aldosterone Antagonists . . . . . . . . . . . . . . . . . . . . . . 35
Risks of Coronary Diagnostic Testing . . . . . . . . . . 11 .
Isosorbide Dinitrate and Hydralazine .. . . . . . . . . . 36
Diagnostic Testing for Structural Heart Disease . . . . . . 12 Calcium Channel Blockers . . . . . . . . . . . . . . . . . . . . 36
Diagnostic Testing for Cardiac Arrhythmias . . . . . . . . . 12 Management of Heart Failure
with Preserved Ejection Fraction . . . . . . . . . . . . . . . . . . 36
Device Therapy . . . . . . . . . . . . . . . . . . . . . . . . . . . . . . . . . 36
Coronary Artery Disease
Implantable Cardioverter-Defibrillator
Stable Angina Pectoris . . . . . . . . . . . . . . . . . . . . . . . . . . . 14
for Prevention of Sudden Cardiac Death . . .. . . . . 36
Diagnosis and Evaluation . . . . . . . . . . . . . . . . . . . . . 14
Cardiac Resynchronization Therapy . . . . . .. . . . .. 37
General Approach to Treatment
Assessment of Chronic Heart Failure . . . . . . . . . . . . . . . 37
of Stable Angina Pectoris . . . . . . . . . . . . . . . . . . . . . 15
Serial B-Type Natriuretic Peptide Assessment .. . . 38
Coronary Revascularization . . . . . . . . . . . . . . . . . . . 18
Echocardiography in Chronic Heart Failure . . . . . . 38
Acute Coronary Syndromes . . . . . . . . . . . . . . . . . . . . . . . 20
Assessing Prognosis . . . . . . . . . . . . . . . . . . . . . . . . . 38
General Considerations . . . . . . . . . . . . . . . .. . . . . . . 20
Inpatient Management of Heart Failure . . . . . . . . . . . . . 38
ST-Elevation Myocardial Infarction . . . . . . . . . . . . . 20
Acute Decompensated Heart Failure . . . . . . . . . . . . 38
Non-ST-Elevation Acute Coronary
Syndromes . . . . . . . . . . . . . . . . . . . . . . . . . . . . . . . . . 25 Cardiogenic Shock . . . . . . . . . . . . . . . . . . . . . . . . . . 39

Acute Coronary Syndromes Not Associated Strategies to Prevent Readmission. . . . . . . . . . .. . . 40


with Obstructive Coronary Disease . . . . . . . . . . . . 30 Advanced Refractory Heart Failure . . . . .. . . . . . . . .. . . 40
Long-Term Care After an Acute Mechanical Circulatory Support .. 40 . . . . . . . . . . . . .

Coronary Syndrome . . . . . . . . . . . . . . . . . . . . . . . . . 30 Management of Post-Transplant Patients . . . . . . . . 40


Management of Coronary Artery Disease in Women . . 30 ' Specific Cardiomyopathies . . . . . . . . . . . . . . . . . . . . . . . . 41
Clinical Presentation . . . . . . . . . . . . . . . . . . . . . . . . . 30 Takotsubo Cardiomyopathy . . . . ... . . . . . . . . .. . . 41
Evaluation and Treatment . . . . . .. . . . . . . . . . . . . . 31 Acute Myocarditis . .. . . . . . . ... ... . . . . ... . ... . 41

ix
Giant Cell Myocarditis . . . . . . . . . . . . . . . . . . . . . . . . 41 Pericardia! Disease
Tachycardia-Mediated Cardiomyopathy . . . . . . . . . 42 Acute Pericarditis . . . . . . .. . . . . . . . . . . . . . . . . . . . . . . . 64
Clinical Presentation . . . . . . . . . . . . . . . . . . . . . . . . . 64
Myocardial Disease Evaluation . . . . . . . . . . . . . . .. . . . . . . . . . . . . . . . . . 64
Hypertrophic Cardiomyopathy . . . . . . . . . . . . . . . . . . . . 42 Management . . . . . . . . . . . . . . . . . . . . . . . . . . . . . . . 6S
Clinical Presentation and Evaluation . . . . . . . . . . . 42 Constrictive Pericarditis . . . . . . . . . . . . . . . . . . . . . . . . . . 66
Clinical Course and Risk Stratification . . . . . . . . . . 43 Clinical Presentation and Evaluation . . . . . . . . . . . 66
Management . . . . . . . . . . . . . . . . . . . . . . . . . . . . . . . 44 Management . . . . . . . . . . . . . . . . . . . . . . . . . . . . . . . 68

Restrictive Cardiomyopathy . . . . . . . . . . . . . . . . . . . . . . . 47 Cardiac Tamponade . . . . . . . . . . . . . . . . . . . . . . . . . . . . . 69


Clinical Presentation and Evaluation . . . . . . . . . . . 47 Clinical Presentation and Evaluation . . . . . . . . . . . 69
Management . . . . . . . . . . . . . . . . . . . . . . . . . . . . . . . 48 Management . . . . . . . . . . . . . . . . . . . . . . . . . . . . . . . 69
Cardiac Tumors . . . . . . . . . . . . . . . . . . . . . . . . ... . . . . . . 48
Tumor Types . . . . . . . . . . . . . . . . . . . . . . . . . . . . . . . 48 Valvular Heart Disease
Clinical Presentation and Evaluation . . . . . . . . . . . 48
Pathophysiology of Valvular Heart Disease . . . . . . . . . . 71
Management . . . . . . . . . . . . . . . . . . . . . . . . . . . . . . . 49
Diagnostic Evaluation of Valvular Heart Disease . . . . . . 71
History and Physical Examination . . . . . . . . . . . . . 71
Arrhythmias
Laboratory and Imaging Tests . . . . . . . . . . . . . . . . . 72
Antiarrhythmic Medications . . . . . . . . . . . . . . . . . . . . . . SO
General Principles of Management of Valvular Heart
Approach to the Patient with Bradycardia . . . . . . . . . . . Sl Disease . . . . . . . . . . . . . . . . . . . . . . . . . . . . . . . . . . . . . . . . 7S
Clinical Presentation . . . . . . . . . . . . . . . . . . . . . . . . . Sl Aortic Stenosis . . . . . . . . . . . . . . . . . . . . . . . . . . . . . . . . . . 77
Sinus Bradycardia . . . . . . . . . . . . . . . . . . . . . . . . . . . Sl Clinical Presentation and Evaluation . . . . . . . . . . . 77
Atrioventricular Block . . . . . . . . . . . . . . . . . . . . . . . Sl Management . . . . . . . . . . . . . . . . . . . . . . . . . . . . . . . 78
Pacemakers . . . . . . . . . . .. . . . . . . . . . . . . .. . . . . . . S2 Aortic Regurgitation . . . . . . . . . . . . . . . . . . . . . . . . . . . . . 79
Approach to the Patient with Tachycardia . . . . . . . . . . . S3 Clinical Presentation and Evaluation . . . . . . . . . . . 79
Supraventricular Tachycardias . . . . . . . . . . . . . . . . . . . . . S3 Management . . . . . . . . . . . . . . . . . . . . . . . . . . . . . . . 79
Clinical Presentation . . . . . . . . . . . . . . . . . .. . . . . . . S3 Bicuspid Aortic Valve . . . . . . . . . . . . . . . . . . . . .. . . . . . . 80
Premature Atrial Contractions Mitral Stenosis . . . . . . . . .. . . . . . . . . . . . . . . . . . . . . . . . . 80
and Atrial Tachycardia . . . . . . . . . . . . . . . . . . . . . . . S3
Clinical Presentation and Evaluation . . . . . . . . . . . 80
Atrioventricular Nodal Reentrant
Management . . . . . . . . . . . . . . . . . . . . . . . . . . . . . . . 81
Tachycardia . . . . . . . . . . . . . . . . . . . . . . . . . . . . . . . . S4
Mitral Regurgitation . . . . . . . . . . . . . . . . . . . . . . . . . . . . . 81
Atrioventricular Reciprocating
Clinical Presentation and Evaluation . . . . . . . . . . . 81
Tachycardia . . . . . . . . . . . . . . . . . . .. . . . . . . . . . . . . S4
Atrial Fibrillation . . . . . . . . . . . . . . . . . . . . . . . . . . . . . . . . SS Management . . . . . . . . . . . . . . . . . . . . . . . . . . . . . . . 82

Clinical Presentation . . . . . . . . . . . . . . . . . . . . . . . . . SS Tricuspid Valve Disease . . . . . . . . . . . . . . . . . . . . . . . . . . 82

Acute Management . . . . . . . . . . . . . . . . . . . . . . . . . . SS Tricuspid Regurgitation . . . . . . . . . . . . . . . . . . . . . . 82

Long-Term Management . . . . . . . . . . . . . . . . . . . . . S6 Tricuspid Stenosis . . . . . . . . . . . . . . . . . . . . . . . . . . . 82


Infective Endocarditis . . . . . . . . . . . . . . . . . . . . . . . . . . . . 83
Atrial Flutter . . . . . . . . . . . . . . . . . . . . . . . . . . . . . . . . . . . S8
Wide-Complex Tachycardias . . . . . . . . . . . . . . . . . . . . . . S9 Diagnosis and Management . . . . . . . . . . . . . . . . . . . 83

Ventricular Arrhythmias . . . . . . . . . . . . . . . . . . . . . . . . . S9 Prophylaxis . . . . . . . . . . . . . . . . . . . . . . . . . . . . . . . . 84


Prosthetic Valves . . . . . . . . . . . . . . . . . . . . . . . . . . . . . . . . 8S
Premature Ventricular Contractions . . . . . . . . . . . . S9
Ventricular Tachycardia with
Structural Heart Disease ... 60
. . . . . . . . . . . . . . . . . . .
Adult Congenital Heart Disease
Idiopathic Ventricular Tachycardia . . . . . . . . . . . . . 60 Introduction . . . . . . . . . . . . . . . . . . . . . . . . .. . . . . . . . . . . 86
Inherited Arrhythmia Syndromes . . . . . . . . . . . . . . . . . . 61
Patent Foramen Ovale . . . . . . . . . . . . . . . . . . . . . . . . . . . . 86
Sudden Cardiac Arrest . . . . . . . . . . . . . . . . . . . . . . . . . . . 62
Atrial Septa! Defect . . . . . . . . . . . . . . . . . . . . . . . . . . . . . . 87
Epidemiology and Risk Factors . . . . . . . . . . . . . . . . 62
Pathophysiology and Genetics . . . . . . . . . . . . . . . . . 87
Acute Management . . . . . . . . . . . . . . . . . . . . . . . . . . 62
Clinical Presentation . . . . . . . . . . . . . . . . . . . . . . . . . 87
Device Therapy for Prevention
Diagnostic Evaluation . . . . . . . . . . . . . . . . . . . . . . . . 87
of Sudden Cardiac Death . . . . . . . . . . . . . . . . . . . . . 63
Treatment........... . ....... . .... . ......... 89
Device Infection . . . . . . . . . . . . . . . . . . . . . . . . . . . . . . . . 63
Follow-up After Atrial Septal Defect Closure . . . . . 89

x
Ventricular Septal Defect . . . . . . . . . . . . . . . . . . . . . . . . . 89 Aortic Atheroma . . . . . . . . . . . . . . . . . . . . . . . . . . . . . . . . 99
Pathophysiology . . . . . . . . . . . . . . . . . . . . . . . . . . . . 89 Abdominal Aortic Aneurysm . . . . . . . . . . . . . . . . . . . . . . 99
Clinical Presentation . . . . . . . . . . . . . . . . . . . . . . . . . 90 Screening and Surveillance . . . . . . . . . . . . . . . . . . . 99
Diagnostic Evaluation . . . . . . . . . . . . . . . . . . . . . . . . 90 Treatment . . . . . . . . . . . . . . . . . . . . . . . . . . . . . . . . . 100
Treatment. . . . . . . . . . . . . . . . . . . . . . . . . . . . . . . . . . 90
Follow-up After Ventricular Septal Peripheral Arterial Disease
Defect Closure . . . . . . . . . . . . . . . . . . . . . . . . . . . . . . 90
Epidemiology and Screening . . . . . . . . . . . . . . . . . . . . . 100
Patent Ductus Arteriosus . . . . . . . . . . . . . . . . . . . . . . . . . 91
Evaluation . . . . . . . . . . . . . . . . . . . . . . . . . . . . . . . . . . . . 101
Pathophysiology . . . . . . . . . . . . . . . . . . . . . . . . . . . . 91
History and Physical Examination . . . . . . . . . . . . 101
Clinical Presentation . . . . . . . . . . . . . . . . . . . . . . . . . 91
Diagnostic Testing . . . . . . . . . . . . . . . . . . . . . . . . . 101
.

Diagnostic Evaluation . . . . . . . . . . . . . . . . . . . . . . . . 91
Medical Therapy . . . . . . . . . . . . . . . . . . . . . . . . . . . . . . . 102
Treatment . . . . . . . . . . . . . . . . . . . . . . . . . . . . . . . . . . 91
Cardiovascular Risk Reduction . . . . . . . . . . . . . . . 103
Pulmonary Valve Stenosis . . . . . . . . . . . . . . . . . . . . . . . . 91
Symptom Relief . . . . . . . . . . . . . . . . . . . . . . . . . . . . 103
Pathophysiology . . . . . . . . . . . . . . . . . . . . . . . . . . . . 91 Interventional Therapy . . . . . . . . . . . . . . . . . . . . . . . . . . 104
Clinical Presentation . . . . . . . . . . . . . . . . . . . . . . . . . 91 Acute Limb Ischemia . . . . . . . . . . . . . . . . . . . . . . . . . . . 104
Diagnostic Evaluation . . . . . . . . . . . . . . . . . . . . . . . . 91
Treatment . . . . . . . . . . . . . . . . . . . . . . . . . . . . . . . . . . 92
Cardiovascular Disease in Cancer Survivors
Aortic Coarctation . . . . . . . . . . . . . . . . . . . . . . . . . . . . . . . 92
Cardiotoxicity of Radiation Therapy
Pathophysiology . . . . . . . . . . . . . . . . . . . . . . . . . . . . 92
to the Thorax . . . . . . . . . . . . . . . . . . . . . . . . . . . . . . . . . . 105
Clinical Presentation . . . . . . . . . . . . . . . . . . . . . . . . . 92
Cardiotoxicity of Chemotherapy . . . . . . . . . . . . . . . . . . 106
Diagnostic Evaluation . . . . . . . . . . . . . . . . . . . . . . . . 92
Treatment. . . . . . . . . . . . . . . . . . . . . . . . . . . . . . . . . . 92
Follow-up After Aortic Coarctation Repair. . . . . . . 93 Pregnancy and Cardiovascular Disease
Tetralogy of Fallot . . . . . . . . . . . . . . . . . . . . . . . . . . . . . . . 93 Cardiovascular Changes During Pregnancy . . . . . . . . . 108
Diagnostic Evaluation After Repair Prepregnancy Evaluation . . . . . . . . . . . . . . . . . . . . . . . . 108
of Tetralogy of Fallot . . . . . . . . . . . . . . . . . . . . . . . . . 93 Management of Cardiovascular Disease
Treatment of Tetralogy of Fallot Residua . . . . . . . . 94 During Pregnancy . . . . . . . . . . . . . . . . . . . . . . . . . . . . . . 109
Adults with Cyanotic Congenital Heart Disease . . . . . . 94 Peripartum Cardiomyopathy . . . . . . . . . . . . . . . . . 110
General Management . . . . . . . . . . . . . . . . . . . . . . . . 94 Other Cardiovascular Disorders . . . . . . . . . . . . . . 110
Eisenmenger Syndrome . . . . . . . . . . . . . . . . . . . . . . 94 Cardiovascular Medication Use
During Pregnancy . . . . . . . . . . . . . . . . . . . . . . . . . . 110

Diseases of the Aorta Anticoagulation Therapy


During Pregnancy . . . . . . . . . . . . . . . . . . . . . . . . . . . 112
Introduction . . . . . . . . . . . . . . . . . . . . . . . . . . . . . . . . . . . . 95
Imaging of the Thoracic Aorta . . . . . . . . . . . . . . . . . . . . . 95
Bibliography . . . . . . . . . . . . . . . . . . . . . . . . . . . . . . . . . 113
Thoracic Aortic Aneurysm . . . . . . . . . . . . . . . . . . . . . . . . 96
Acute Aortic Syndromes . . . . . . . . . . . . . . . . . . . . . . . . . . 98
Pathophysiology . . . . . . . . . . . . . . . . . . . . . . . . . . . . 98 Self-Assessment Test . . . . . . . . . . . . . . . . . . . . . . . . . . . 119

Diagnosis and Evaluation . . . . . . . . . . . . . . . . . . . . . 98


Treatment . . . . . . . . . . . . . . . . . . . . . . . . . . . . . . . . . . 99 Index . . . . . . . . . . . . . . . . . . . . . . . . . . . . . . . . . . . . . . . . 217

xi
Cardiovascular Medicine High Value
Care Recommendations

The American College of Physicians, in collaboration with without other major risk factors such as hypertension or
multiple other organizations, is engaged in a worldwide tobacco use) (see Item 88).
initiative to promote the practice of High Value Care • Stress testing is most efficacious in patients with an
(HVC). The goals of the HVC initiative are to improve intermediate pretest probability of coronary artery dis­
health care outcomes by providing care of proven benefit ease (10%-90%), because it is these patients who, by the
and reducing costs by avoiding unnecessary and even result of their stress test, can be reclassified into higher
harmful interventions. The initiative comprises several or lower risk categories.
programs that integrate the important concept of health • Exercise electrocardiographic testing is recommended
care value (balancing clinical benefit with costs and as the initial test of choice in patients who are able to
harms) for a given intervention into a broad range of edu­ exercise with a normal baseline electrocardiogram and
cational materials to address the needs of trainees, prac­ an intem1ediate pretest probability of coronary artery
ticing physicians, and patients. disease based on age, sex, and symptoms (see Item 87).
• Measurement of coronary artery calcium should be lim­
HVC content has been integrated into MKSAP 17 in several
ited to a select group of asymptomatic patients with an
important ways. MKSAP 17 now includes HVC-identified
intermediate Framingham risk score (10%-20%) in whom
key points in the text, HVC-focused multiple choice
results will influence treatment strategy because of its
questions, and, for subscribers to MKSAP Digital, an HVC
associated cost and radiation exposure.
custom quiz. From the text and questions, we have gen­
• Routine yearly imaging evaluation of structural heart dis­
erated the following list of HVC recommendations that
ease in asymptomatic patients is usually not indicated.
meet the definition below of high value care and bring us
• Patients with grade 1 or 2 midsystolic murmurs who are
closer to our goal of improving patient outcomes while
asymptomatic with no associated findings and those with
conserving finite resources.
continuous murmurs suggestive of a venous hum or mam­
High Value Care Recommendation: A recommendation to mary souffle do not warrant echocardiographic evaluation.
choose diagnostic and management strategies for patients • For stable angina pectoris, percutaneous coronary inter­
in specific clinical situations that balance clinical benefit vention is reserved for patients with refractory symptoms
with cost and harms with the goal of improving patient while on optimal medical therapy, those who are unable
outcomes. to tolerate optimal medical therapy owing to side effects,
or those with high-risk features on noninvasive imaging.
Below are the High Value Care Recommendations for the
• Clinical practice guidelines do not recommend the rou­
Cardiovascular Medicine section of MKSAP 17.
tine use of ECG monitoring, stress testing, or anatomic
• Current guidelines do not support the use of high-sensi­ testing (coronary CT angiography or invasive angiogra­
tivity C-reactive protein (hsCRP) evaluation in the general phy) in asymptomatic patients after percutaneous coro­
population, but hsCRP testing may be used in interme­ nary intervention or coronary artery bypass graft surgery.
diate-risk patients in whom choice of therapy may be • Routine stress testing is not currently recommended
affected by reclassification of risk. for asymptomatic patients following an acute coronary
• There is currently no role for the routine measurement of syndrome who are not entering a cardiac rehabilitation
Lp(a) lipoprotein levels, homocysteine levels, or evalua­ program.
tion of lipid particle size as these tests are expensive and • Evaluation of unusual causes of heart failure should not
no studies to date have shown that treatment targeted to be performed routinely but should be performed when
these levels affects outcomes. there are suggestions of specific diseases by history or
• The evaluation of subclinical disease with coronary physical examination findings (see Item 116).
artery calcium scoring may be appropriate to further risk • In patients with chronic heart failure who are clinically
stratify intermediate-risk patients but is not a component stable, annual or more frequent follow-up echocardiogra­
of routine risk assessment. phy rarely provides therapeutic or diagnostic benefit and
• Aspirin should not routinely be given to patients with is not recommended.
diabetes mellitus who are at low cardiovascular risk (men • Patients hospitalized for heai1 failure who are scheduled
younger than 50 years and women younger than 60 years for a follow-up appointment within 1 week after discharge

xiii
have a reduced risk of future heart failure hospitalization • Screening of asymptomatic patients for abnormalities
(see Item 34). of the thoracic aorta should be reserved for patients
• Patients with asymptomatic, mild aortic stenosis or with underlying vascular pathology (such as Marfan or
regurgitation require echocardiography every 3 to Ehlers-Danlos syndrome), a bicuspid aortic valve, or a
5 years. family history of aortic disease.
• Asymptomatic patients with moderate aortic regurgi­ • Invasive imaging of the aorta by angiography is rarely
tation should be evaluated clinically on a yearly basis necessary for the diagnosis of acute disease; it should
and have echocardiography performed every 1 to be reserved for patients in whom a percutaneous inter­
2 years, but they do not require medical or surgical vention is planned.
intervention (see Item 59). • Diuretics, particularly in high doses, will exacerbate
• Patients with asymptomatic, severe aortic stenosis with the propensity towards dynamic left ventricular out­
preserved left ventricular function may be managed flow tract obstruction and, therefore, should be
with close clinical follow-up and echocardiography avoided in patients with hypertrophic cardiomyopathy
every 6 to 12 months and should not undergo valve (see Item 5).
replacement (see Item 20). • When an atrial septal aneurysm is identified inciden­
• Patients with an asymptomatic bicuspid aortic valve tally, no further evaluation, medical treatment, or inter­
should undergo surveillance transthoracic echocardiog­ vention is needed (see Item 10).
raphy yearly if the aortic root or ascending aortic diam­ • Influenza vaccine should be administered to patients
eter is greater than 4 cm (see Item 97). with established cardiovascular disease to reduce the
• Infective endocarditis prophylaxis should be limited risk of future cardiovascular events (see Item 13).
to those with a prosthetic cardiac valve; a history of • Accelerated idioventricular rhythm is a common com­
infective endocarditis; unrepaired cyanotic congen­ plication following coronary reperfusion and does not
ital heart disease or repaired congenital heart defect require intervention when it occurs within 24 hours of
with prosthesis or shunt (::;6 months post-procedure) reperfusion (see Item 42).
or residual defect; or valvulopathy following cardiac • Routine screening for coronary artery disease in
transplantation. asymptomatic patients with diabetes mellitus does not
• Antibiotic prophylaxis to prevent bacterial endocarditis reduce mortality (see Item 65).
is not recommended for nondental procedures, includ­ • Supervised exercise therapy can effectively treat clau­
ing transesophageal echocardiography and genitouri­ dication, with increases in pain-free walking time and
nary or gastrointestinal procedures, in the absence of maximal walking time, and is recommended as part of
active infection (see Item 25). the initial treatment regimen for intermittent claudica­
• There is no indication for patent foramen ovate closure tion (see Item 84).
or for antiplatelet therapy in asymptomatic patients, and • Asymptomatic first-degree atrioventricular block with
randomized trials do not support patent foramen ovale bifascicular block does not require pacemaker implan­
closure to reduce risk of recurrent stroke or migraine. tation (see Item 109).
• A small atrial septa! defect (pulmonary-to-systemic blood • Patients with acute pericarditis who do not have
flow ratio [Qp:Qs] <l.5:1) with no associated symptoms or high-risk features (fever, leukocytosis, acute trauma,
right heart enlargement can be followed clinically. abnormal cardiac biomarkers, immunocompromise,
• A small membranous ventricular septal defect with­ oral anticoagulant use, large pericardial effusions, or
out left heart enlargement, pulmonary hypertension, evidence of cardiac tamponade) can be managed medi­
recurrent endocarditis, or valve regurgitation can be cally on an outpatient basis with close clinical
observed clinically (see Item 45). follow-up (see Item 114).

xiv
Cardiovascular Medicine

Epidemiology and Women have a higher prevalence of risk factors for CVD,
including elevated cholesterol levels, diabetes mellitus,
Risk Factors hypertension, and inactivity. Only tobacco use is higher
among men.
Overview More women present with angina than men, but women
In the United States, the mortality rate from cardiovascular dis­ often have other symptoms in addition to chest pain. Women
ease (CVD), including heart disease, stroke, peripheral vascular have "atypical" symptoms more frequently than men, includ­
disease, hypertension, and heart failure has steadily declined ing nausea, shortness of breath, and unusual fatigue. More
over the past decade-33% from 1999 to 2009, likely as a result than two thirds of women who die suddenly from coronary
of better prevention and acute care efforts. Nonetheless, CVD is heart disease either did not recognize the symptoms or had no
the leading killer of both men and women, and although mortal­ previous symptoms. Women undergo fewer revascularization
ity of CVD is decreasing, CVD prevalence is increasing. By 2030, procedures than men, with 25% of coronary artery bypass
according to the American Heart Association's Heart Disease surgeries and nearly 33% of percutaneous coronary interven­
and Stroke Statistics, more than 40% of the U.S. population is tions occurring in women.
projected to have some form of CVD. More than one in three
American adults currently have some form of CVD, and the
prevalence increases from more than 10% in those aged 20 to
Eth n icity a n d
39 years to more than 70% in those aged 60 to 79 years. Based on Ca rd iovascu lar Disease
data from the Framingham Heart Study, two out of three men and The prevalence of CVD and risk factors in the United States
one out of two women will develop CVD in their lifetime. Despite vary by ethnicity. American Indians and Alaska Natives
the decreasing mortality, hospitalizations for cardiovascular­ have the highest rate of heart disease (12.7%), followed by
related diseases have steadily continued to rise. There were whites (11.1%), blacks or African Americans (10.7%),
nearly 6 million hospital discharges for cardiovascular-related Hispanics or Latinos (8. 6%), and Asians (7.4%). Peripheral
diseases in 2009, with an estimated cost of $312.6 billion. arterial disease affects nearly 8.5% of Americans older than
The prevalence of heart failure continues to rise, with a 40 years, and prevalence is highest among older persons,
predicted prevalence in the United States of 25% by 2030. It is non-Hispanic blacks, and women. The population most
estimated that 5.1 million Americans older than 20 years have affected by heart failure is African Americans, at a rate of
a diagnosis of heart failure. Currently, the incidence is 1/100 4.6/1000 person-years, followed by Hispanic, white, and
annually in those older than 65 years. Most of these patients Chinese Americans.
have a history of hypertension. Both systolic dysfunction and Cardiovascular risk factors also vary among ethnicities.
diastolic dysfunction are associated with the development of Blacks have the highest rate of hypertension, at 33.4%
symptomatic heart failure, and the prevalence of heart failure (higher in black women), followed by American Indians or
with preserved ejection fraction (diastolic dysfunction) is Alaska Natives (25.8%), whites (23.3%), Hispanics or Latinos
increasing. Mortality in heart failure is quite high-nearly 50% (22. 2%), and Asians (18.7%). Blacks have the highest preva­
mortality at 5 years. lence of two or more cardiovascular risk factors (48.7%).
The prevalence of risk factors is increased with decreasing
levels of education and income. Obesity and lack of physical
Cardiovascu lar D isease i n Women activity are highest among Hispanic/Latino adults and non­
Since 1984, the number o f deaths from CVD has been greater Hispanic blacks.
for women than men and highest among black women. More Environmental influences on cardiovascular risk factors
than 400,000 women died of CVD in 2009, 51% of all CVD are changing the prevalence of CVD in certain populations. In
deaths. Women have a higher mortality rate after myocardial countries with previously low rates of CVD, rates of disease are
infarction: 26% in women versus 19% in men older than increasing with the adoption of Western eating habits and
45 years. The death rate for women with heart failure is higher increasing tobacco use. With declining rates of infant mortality
than among men, although women are often older. Incidence and death from infectious diseases, the influence of urbaniza­
of and mortality from stroke is highest among women, with tion and change in traditional lifestyles are resulting in increas­
the highest among black women. ing rates of CVD.

1
Epidemio logy a n d Risk Factors

for CVD, increasing the risk by 25% to 30%. Smoking cessation


Genetics i n Card iovascu l a r Disease
substantially reduces cardiovascular risk within 2 years, and
Family history of premature (male <45 years; female <55 years)
this risk returns to the level of a nonsmoker within 5 years.
coronary artery disease (CAD) significantly increases risk of CVD.
Efforts to assess smoking status and provide assistance with
Having a parent with premature CAD doubles risk of myocardial
cessation should be made at every encounter (see MKSAP 17
infarction in men and increases risk in women by 70%. CAD in a
General Internal Medicine, Routine Care of the Healthy Patient).
sibling increases risk by 50%. Genetic predisposition as well as
Nearly one in three adults in the United States older than
shared environment may contribute to increased risk in family
20 years has hypertension, and the rates are equal among men
members. Although prediction models based on the genetics of
and women. Nearly 30% of adults older than 20 years have pre­
CVD are not yet available, research continues at a rapid pace.
hypertension (systolic blood pressure 120-139 mm Hg; diastolic
blood pressure 80-89 mm Hg) . The rates increase with age, with

Lifestyle Risk Factors a prevalence greater than 70% in persons older than 65 years.

As much as 90% of the risk for myocardial infarction has been Treatment of hypertension reduces risk for cardiovascular events,

attributed to modifiable risk factors, with elevated cholesterol including stroke, and reduces end-organ damage such as heart

levels, smoking, and psychosocial stressors accounting for a failure and kidney disease. Although the prevalence of blood

significant portion of the attributable risk. The attributable risk pressure control (that is, blood pressure within recommended

for myocardial infarction is highest for cholesterol levels, fol­ ranges) has improved in the United States from less than 30% two

lowed by current smoking, psychosocial stressors, diabetes, decades ago (1988-1994) , it still is only 50% (2007-2008) .

hypertension, abdominal obesity, no alcohol intake, inadequate Sedentary lifestyle, poor diet, and obesity contribute to
exercise, and irregular consumption of fruits and vegetables. increased cardiovascular risk and increased risk for diabetes.
Elevated cholesterol levels increase the risk of CVD, and Nearly one third of all U.S. adults report no leisure time activity,
multiple studies have shown that reductions in cholesterol and less than 30% of high school students engaged in 60 min­
levels, particularly LDL cholesterol, reduce risk. Nearly 14% of utes of daily physical activity; this rate was lowest among girls.
adults older than 20 years have total cholesterol levels greater Between 1971 and 2004, total energy consumption increased by
than 240 mg/dL (6.21 mmol/L}; approximately 6% of adults are 22% in women and 10% in men. Average fruit and vegetable
estimated to have undiagnosed hypercholesterolemia. Elevated consumption was 2.4 to 4 servings daily (recommended,
LDL cholesterol and low HDL cholesterol levels are independ­ >5 daily) and was lowest among blacks. The increased caloric
ent risk factors for CVD. For every 1 % decrease in LDL choles­ intake coupled with decreased physical activity has led to an
terol level, there is a corresponding 1 % decrease in risk for
increased incidence of obesity. More than two thirds of the
coronary artery disease. The risk reduction is even greater with
American population older than 20 years are overweight
changes in HDL cholesterol, with a risk reduction of 2% to 3%
(BM! 25-29.9) vvith more than one third obese (BMl>30). In chil­
for every 1% increase in HDL cholesterol level. However, rand­
dren and adolescents between the ages of 2 and 19 years, nearly
omized clinical trials evaluating pharmacologic therapies that
33% are obese or overweight and 17% of these children are obese.
raise HDL cholesterol levels in patients with well-treated LDL
Psychosocial stressors are an important contributor to
cholesterol levels have not shown reduction in clinical end­
cardiovascular risk. These include depression, anger, and anx­
points. Long-standing guidelines (Adult Treatment Panel III
iety, and are associated with worse outcomes. Depression has
[ATP III]) have provided treatment goals for LDL and non-HDL
been associated with higher risk for cardiovascular events,
cholesterol levels based on cardiovascular risk factors and
and psychosocial stressors also affect the course of treatment
Framingham risk score. In 2013, the American College of
and adherence to healthy lifestyles after an event. Awareness
Cardiology and the American Heart Association (ACC/AHA)
of these factors and appropriate therapies may improve out­
published revised guidelines that treat lipid blood levels
comes in these individuals.
according to cardiovascular risk, rather than LDL cholesterol
Impaired glucose control is a significant component of
targets (see MKSAP 17 General Internal Medicine, Dyslipidemia).
the metabolic syndrome, which is characterized by elevated
The use of tobacco has declined over the past few decades,
but despite this decline, in 2011, more than 21.3% of men, 16.7% glucose, central obesity, low HDL cholesterol, elevated triglyc­

of women, and 18% of high school students were smokers. The erides, and high blood pressure. More than 34 % of adults older

rates were highest among American Indian/ Alaska Natives and than 20 years meet the criteria for metabolic syndrome (three

non-Hispanic black males and lowest among Hispanic females. of the five components). The presence of metabolic syndrome

Tobacco use increases the risk of CVD, including coronary is associated with an increased risk of CVD. This risk increases

heart disease, stroke, and peripheral vascular disease, for which with an increased number of components and also appears to

smoking is a major risk factor, and increases CVD mortality by be higher among women. The National Diabetes Prevention
2 to 3 times. The risk of coronary artery disease is increased by Program found that in persons at high risk for diabetes,
25% in women who smoke. Smoking increases the risk of improved food choices and at least 150 minutes of exercise
stroke by 2 to 4 times. Secondhand smoke is also a risk factor weekly led to 5% to 7% weight loss and reduced the risk of

2
Epidemiology a n d Risk Factors

developing diabetes by 58%, but no interventions have shown burden is likely a result of both the inflammatory process of the
a reduction in CVD events to date. systemic disease, including a prothrombotic state, as well as
KEY POINT the presence of traditional cardiovascular risk factors.

• Elevated cholesterol levels, smoking, and psychosocial


stressors are the greatest modifiable risk factors for car­
diovascular disease.
Ca lcu latin g Ca rd iovascu l a r Risk
Cardiovascular risk scores should be utilized to stratify patients
for appropriate prevention targets. Traditionally, the
Framingham risk score has been used to estimate the 10-year
Specific Risk Gro u ps
risk of a major cardiovascular event (myocardial infarction or
Diabetes Mellitus coronary death). An online Framingham risk calculator is avail­
The presence of diabetes mellitus is associated with increased able at http: //cvdrisk.nhlbi.nih.gov/calculator.asp. Using this
cardiovascular risk, particularly among women. Persons with method, a 10-year risk of ASCVD of less than 10% is considered
diabetes have a 2 to 4 times increased risk of CVD, with more low risk, 10% to 20% is classified as intermediate risk, and above
than two thirds of those with diabetes eventually dying of heart 20% is designated as high risk. Age is the component that drives
disease. The risk of stroke is increased 1.8- to 6-fold in persons most of the risk, with increasing age reflected in increased risk.
with diabetes. The presence of diabetes in those older than The Framingham risk score underestimates risk in women and
65 years is nearly 27%. In those aged 12-19 years, the prevalence minority populations. In an effort to account for the underesti­
of diabetes and prediabetes is increasing, from 9% to 23% from mation in women, the Reynolds risk score was developed,
1999-2007. Diabetes is often undiagnosed, and is frequently which is a sex-specific score for both men and women that
diagnosed at the time of an acute event such as myocardial includes family history and high-sensitivity C-reactive protein
infarction. Appropriate treatment of cardiovascular risk factors (hsCRP) levels (www.reynoldsriskscore.org).
in persons with diabetes is associated with reduced cardiovas­ The Pooled Cohort Equations are a new risk assessment
cular risk. The most recent cholesterol guidelines recommend instrument developed from multiple community-based
moderate- or high-intensity statin therapy in patients aged 40 to cohorts (including the Framingham study) that includes a
75 years with diabetes. Patients with diabetes aged 40 to 75 years broader range of variables than the Framingham score when
with a 10-year atherosclerotic cardiovascular disease (ASCVD) evaluating 10-year ASCVD risk. Its use as a primary risk assess­
risk greater than or equal to 7.5% should receive high-intensity ment tool was recommended in the 2013 ACC/AHA Guideline
statin therapy because of their increased risk. In patients with on Assessment of Cardiovascular Risk. The ACC/AHA CV risk
diabetes in this age group with a 10-year risk below 7.5%, calculator includes age, sex, race, total and HDL cholesterol
moderate-intensity statin therapy is recommended. levels, systolic blood pressure, blood pressure-lowering medi­
cation use, diabetes status, and smoking status. Using this
Chronic Kidney Disease method, a 10-year risk of ASCVD of below 5% is considered
Chronic kidney disease (CKD) is associated with higher car­ low risk, 5% to below 7.5% is classified as intermediate risk,
diovascular mortality, and more patients with kidney disease and 7.5% or above is designated as high risk. The new risk
will die of CVD than will go on to have end-stage kidney dis­ calculator can be accessed at http: //my.americanheart.org/
ease requiring dialysis. Chronic kidney disease shares many of professional/StatementsGuidelines/PreventionGuidelines/
the same risk factors for CVD such as hypertension, diabetes, Prevention-Guidelines_ DCM_ 457698_SubHomePage.jsp.
and smoking. The exact etiology of the high death rate in KEY POINT
patients with CKD is uncertain and may be related to a higher
• Cardiovascular risk scores should be utilized to stratify
incidence of fatal arrhythmias, lack of adequate therapies at
patients for appropriate prevention targets; risk assess­
the time of an acute cardiovascular event, or multi-organ
ment tools include the Framingham risk score, the
changes related to kidney failure.
Reynolds risk score, and the American College of
CardiologyIAmerican Heart Association's cardiovascular
Systemic I nflammatory Conditions
risk calculator based on the Pooled Cohort Equations.
Patients with systemic inflammatory conditions, such as sys­
temic lupus erythematosus (SLE) and rheumatoid arthritis,
have an increased risk of CVD. Most deaths in persons with SLE Emerg i n g Risk Factors
and nearly 40% of deaths in those with rheumatoid arthritis are Because atherosclerotic disease is thought to be in part an
cardiovascular and, in particular, heart failure related. The risk inflammatory process, hsCRP measurement has been investi­
of CVD increases with the duration of the underlying inflam­ gated for enhancing risk prediction. Current guidelines do not
matory condition. The risk of CVD increases from two times support the use of hsCRP evaluation in the general population.
that of the general population to three times after 10 years' However, hsCRP testing may be used in intermediate-risk
duration ofrheumatoid arthritis. The increased atherosclerotic patients (Framingham 10-year risk score of10%-20%) in whom

3
Epidemio logy a n d Risk Factors

choice of therapy may be affected by reclassification of risk. Asp i ri n for Pri m a ry Prevention
Elevated hsCRP levels should be rechecked within 2 weeks, and
Aspirin is a powerful agent for both primary and secondary
other potential causes of infection or inflammation should be
prevention of coronary artery disease. Aspirin for second­
ruled out. Although statin therapy has been shown to lower
ary prevention is discussed under Coronary Artery Disease.
hsCRP levels, therapy targeting hsCRP alone is not appropriate
For primary prevention of myocardial infarction, data sug­
as patients should be treated according to cardiovascular risk.
gest that there is greater benefit in men, particularly those
Although elevated levels of Lp(a) lipoprotein and homo­
older than 45 years. For women, benefit outweighs risk of
cysteine have been associated with elevated cardiovascular
aspirin therapy after the age of 65 years. Between the ages
risk, these tests should not be routinely performed.
of 55 and 65 years, the risk of stroke is reduced in women
Interventions to reduce homocysteine levels with folic acid
on aspirin therapy. Guidance for using aspirin for primary
supplementation have not been shown to reduce cardiovascu­
prevention of myocardial infarction and stroke is provided
lar events. Although epidemiologic evidence supports the
in Table 1 and Table 2 .
association between elevated levels of Lp(a) lipoprotein and
I t i s important to balance the benefits o f aspirin therapy
cardiovascular events, to date no trials have shown that treat­
with the risks of gastrointestinal (GI) bleeding. The risk of seri­
ment to lower Lp(a) lipoprotein levels lowers risk. There is
ous bleeding is greatly increased in patients with a history of
currently no role for the evaluation of lipid particle size and
GI ulcers and who use NSAIDs, and these factors should be
number. No studies to date have shown that treatment targeted
considered when assessing the benefits and harms of using
to particle size and number affects outcomes.
aspirin in the individual patient.
The evaluation of subclinical disease with coronary artery
Aspirin should not be routinely given to patients with
calcium (CAC) scoring may be appropriate to further risk
diabetes who are at low risk for CVD (men <50 years and
stratify intermediate-risk patients but is not a component of
women <60 years with no major additional CVD risk fac­
routine risk assessment. Evidence of calcification of coronary
tors; 10 -year CVD risk <5%). It is reasonable to give low­
vessels is indicative of atherosclerotic disease, but the absence
dose aspirin to adults with diabetes and no previous
of calcification does not rule out the presence of soft plaque.
history of vascular disease who are at increased CVD risk
K EY P O I N T S (10-year Framingham risk >10%) and without increased
HVC • Current guidelines do not support the use of high- risk for bleeding.
sensitivity C-reactive protein (hsCRP) evaluation in the KEY P O I NTS
general population, but hsCRP testing may be used in
• In men ages 45 to 79 years, aspirin for primary preven­
intermediate-risk patients in whom choice of therapy
tion of myocardial infarction is recommended if the
may be affected by reclassification of risk.
benefit of treatment outweighs the risk of gastrointesti­
HVC • There is currently no role for the routine measure-
nal bleeding.
ment of Lp(a) lipoprotein levels or homocysteine lev­
• In women ages 55 to 79 years, aspirin for primary pre­
els or evaluation of lipid particle size as these tests
vention of stroke is recommended if the benefit of treat­
are expensive and no studies to date have shown that
ment outweighs the risk of gastrointestinal bleeding.
treatment targeted to these levels affects outcomes.
HVC • Aspirin should not be routinely given to patients with HVC
• The evaluation of subclinical disease with coronary
diabetes who are at low risk; that is, men younger than
artery calcium scoring may be appropriate to further
50 years and women younger than 60 years with no
risk stratify intermediate-risk patients but is not a com­
major additional cardiovascular risk factors.
ponent of routine risk assessment.

TABLE 1 . Use of Aspirin for Primary Prevention of Cardiovascular Disease


Patient Category Recommendation
Men age 45-79 years Aspirin recommended when potential benefit of reduction in M l outwei g hs risk of GI bleed i ng
Women age 55-79 years Aspiri n recommended when potential benefit of reduction in isch e m ic stroke outweighs risk of
GI bleeding

M e n/women age >80 years I nsufficient evidence for primary prevention


Men age <45 years, women age Aspirin not reco mmended for prevention of Ml i n men or stroke in women
<55 years

GI gastrointesti nal; Ml myocardial infarction.

· . · ·
Adapted with permission from U.S. Preventive Services Task Force Aspirin for the prevention of card iovascular disease: U.S. Preventive Services Task Force recommendation
statement. Ann I ntern Med. 2009 Mar 1 7; 1 50(6):396-404. [PMID: i9293072]

4
D i a g n ostic Testing in Cardio lo g y

TABLE 2. Risk Level at Which CHO Events Prevented Exceed GI Harms in Patients Taking Daily Aspirin for Primary Prevention
Men Women
1 0-Year CHO Risk• 1 0-Year Stroke Riskb

Age 45-59 years �4% Age 55-59 years �3%


Age 60-69 years �9% Age 60-69 years �8%

Age 70-79 years � 1 2% Age 70-79 years �1 1 %

CHO = coronary heart disease; G I = gastrointestinal.

'Risk factors for CHO include age, diabetes mellitus, total cholesterol level, HOL cholesterol level, blood pressure, and smoking. CHO risk estimation tool: http://cvdrisk.nhlbi.nih.
gov/calculator.asp. (Note: This is the Framingham risk score.)

bRisk factors for ischemic stroke include age, high blood pressure, diabetes mellitus, smoking, history of cardiovascular disease, atrial fibrillation, and left ventricular hypertrophy.

NOTE: This table applies to adults who are not taking NSAIOs and who do not have upper GI pain or a history of GI ulcers.

Adapted with permission from U.S. Preventive Services Task Force. Aspirin for the prevention of cardiovascular disease: U.S. Preventive Services Task Force recommendation
statement.Ann Intern Med. 2009 Mar 1 7; 1 S0(6):396-404. [PMIO: 1 9293072]

st ress test, can be reclassified i n to h igher or lower risk cate­


Diag nostic Testing gories. Stress testing also has important prognostic value for
i n Ca rdiology predict i ng the risk of myocardi a l i n fa rction and death i n
selected patients. For example, i n patients with a previous
Clin ical H istory a n d Physica l history o f CAD and worsening cardiac symptoms, stress test­
Exam i n ation ing i s helpfu l to assess for possible recu rrent or progressive
d isease. However, a l t hough t he lead i ng cause of death i n
The initial step in evaluating fo r heart disease i s a thorough
patients w i t h d i abetes mellitus is card i ovascular disease,
history and physical examination. Specifically, a careful
rout i ne stress tes t i ng in asympLOmatic patients with d iabetes
exploration of changes in functional status, associated
has not been shown to reduce mortal i ty.
symptoms, and the timing and nature of symptoms will
The modalities able to detect cardiac ischemia are h ighly
help focus the assessment and guide selection of appropri­
dependent on the degree of i mpairment of coronary blood
ate testing, if indicated.
flow. The earliest changes with mild stenosis are perfusion
Cardiovascular testing provides both diagnostic and prog­
changes detectable o n ly with h ighly sensitive modal ities, such
nostic information and its use should be guided by symptoms,
as nuclear or cardiac magnetic resonance (CMR) imagi ng.
the level of risk for heart disease, and whether outcomes may
With progressive coronary occlusion. d i astoLic dysfu nction
be altered by interventions based on testing results.
followed by systolic dysfu nction may be seen by i maging stud­
ies such as echocardiography. O n ly when there is sig n i ficant
coronary stenosis will electrocardiographic (ECG) changes be
Diagnostic Testin g for
seen and symptoms occur.
Atheroscle rotic Coro n a ry Disease The many d i fferent types of tests to d i agnose CAD can
c:J Cardiac Stress Testing be b road ly categorized as assessi ng e i t her functional or
Pat ients are referred for stress testi ng to establish the d i agno­ a nato m ic evidence of ischem ia. Functional studies evaluate
sis of coronary artery d isease (CAD) most often because of for obstructive CAD from evidence of ECG changes, myocar­
new onset of or a change in symptoms. The u t i l ity of stress d i a l perfusion abnorma l i t ies, or wal l motion abnorma l i t ies.
testing should be i n terpreted in t he con text of the pretest usu a l ly u nder stress con d i t ions. Anatomic studies assess
l i ke l i hood of disease. Those w i t h low probab i lity of disease, percentage stenosis o f the coronary vessels a t rest, which
such as younger patients, have a h igher i ncidence of false­ can be visual ized by singl e- photo n e m ission CT (SPECT) .
positive tests and may u ndergo unnecessary testing without PET/CT sca n , or C M R i maging study. These i maging modal­
changing patient ou tcomes. Those with a h igh proba b i l i ty of i ties may a lso be used to q u a n t i fy i n farction size and assess
d isease should proceed d i rectly to a n i nvasive d i agnostic myocard i a l viab il ity. More specifi c signs of ischern i a such as
strategy, such as cardiac catheteriza t io n , because the risk of a reduced regional c o n t racti l i ty can be assessed by echocardi ­

false-negative result and m i ssed d iagnosis is too h igh. ography or M R I .


Furthermore, a negative test i n a h igh-risk patient would not Testing modalities for suspected CAD are summarized i n
signi ficantly change the post-test probabi l i ty o f CA D, and Table 3 . I n i ntermediate-risk pat ients who are able t o exercise
t herefore wou l d not change managemen t. Stress testing is and have a normal baseline ECG, the initial type of stress test­
most clinically appropriate in patients with a n i ntermediate ing should be exercise stress testing. The add i t ional prognostic
risk of CAD. It is t hese patients who. by t he resul t of thei r i nformation ava ilable with exercise, i ncluding functional

5
D i a g nostic Testi n g i n Cardiology

TABLE 3 . Diagnostic Testing for Coronary Artery Disease


Diagnostic Test Utility Advantages Limitations
Exercise Stress Testing

Exercise ECG I n itial diagnostic test in m ost Data acq u i red on exercise Not useful when baseline ECG is
patients with suspected CAD capacity, blood pressure and abnormal (LVH, LBBB, paced
heart rate response, and rhythm, WPW syndrome, >1 mm
p rovoked symptoms ST-segment depression)

Stress echocard iography Recommended when baseline Exercise data acq u i red along Image quality is suboptimal i n
ECG is abnormal or when with i mag ing for wal l motion some patients but can be
informatio n on a particular area abnormalities to i ndicate improved with microbubble
of myocard i u m at risk is ischemia transpu l monary co ntrast
needed
Allows evaluation of valve Image interpretation is difficult
function and pulmonary when basel ine wall motion
pressures abnormalities are present

Relatively portable and less Diagnostic accu racy decreases


costly than nuclear p rotocols with single-vessel disease or
de layed stress image acq uisition
Entire study is completed in <1 h

Nuclear SPECT perfusion Recommended when basel i ne Gating ( i mage acq uisition Attenuation artifacts can be
ECG i s abnormal o r when coordinated with the cardiac caused by breast tissue or
information on a particular area cycle), use of higher energy diaphragm i nterference
of myocard i u m at risk is agents such as technetium,
Radiatio n exposure
needed and tech niques used to correct
f o r attenuation provide
With LBBB, conduction delay i n
improved specificity
the septum may cause false-
positive abnormal ity; this can Late reperfusion imaging
be i m proved with the use of allows evaluation of myocardial
vasodilator stress viability if thal l i u m is used

Pharmacologic Stress Testing

Dobutami n e Recommended i n patients w h o Because t h e patient is supi ne, Dobutamine contraindications are
echocardiography cannot exercise i mages are acquired severe baseline hypertension,
continuously, allowing the test unstable angina, and arrhythmias
Recommended when
to be stopped as soon as
i nformatio n on an area of �-Blockers m ust be withheld
ischemia is evident
myocardi u m at risk i s needed before the test
Vasod ilator nuclear Recommended in patients who Vasodi lator stress testing may Contraind ications are
perfusion (adenosine, cannot exercise m i n i mize effect of �-blockade bronchospastic airway disease,
dipyridamole, on perfusion defect size theophyl line use, sick sinus
May minimize septa!
regadenoson) syndrome, and high-degree AV
abnormalities frequently seen Can image sooner after
block
with nuclear perfusion myocardial i nfarction with
scan n i ng in patients with LBBB vasodilator stress Caffeine must be with held 24
hours before the test
Adenosine or d i pyridamole may
cause chest pain, dyspnea, or
fl ushing

Radiation exposure
Dobutami n e nuclear Recommended in patients who Has comparabl e sensitivity and Dobutam ine contraindications are
perfusion cannot exercise and have specificity to exercise or severe baseline hypertension,
contraindications to vasodilators vasodilator perfusion i mag i ng unstable ang i na, and arrhythmias
for diagnosis of myocard ial
Recommended when �-Blockers should be withheld
ischemia
information on an area of before the test
myocard i u m at risk is needed
Radiation exposure
PET/CT Provides best perfusion i mages Study d u ration is shorter and Not widely avai lable
i n larger patients rad iation dose is lower than
More expensive than other
conventional nuclear perfusion
Provides data on myocard ial i mag ing modalities
i magi n g
perfusion, function, and
Used with pharmaco logic stress
viability Absol ute myocardial blood
only (no exercise protocol)
flow can be measured
Radiation exposure
Can be combi ned with CAC
scoring

(Continued on the next page)

6
D i a g n ostic Testing i n Cardi o logy

TABLE 3 . Diagnostic Testing for Coronary Artery Disease (Continued)


Diagnostic Test Utility Advantages Limitations
Other Tests

Coronary angiography Provides anatomic diagnosis of Percutaneous revascularization Invasive


the presence and severity of can be performed following
Risks of vascular access and
CAD diagnostic study
radiocontrast exposure (kid ney
dysfu nction, a l le rgy, b l eeding)

Radiation exposure
CAC testing CAC testi ng is reasonable i n CAC scores a r e predictive of Does not provide data on
asymptomatic patients at cardiovascular risk in selected coronary l u m i n a l na rrowing
intermed iate risk for CAD patients
Radiation exposure
Coronary CT angiogra phy Identifies anomalous coronary Coronary a rtery vessel l u men Req u i res high-resolution (64-slice)
arteries and atherosclerotic lesions can CT instruments
be visualized i n detail
Useful for selected patients Does not provide d eta iled images
with intermed iate risk for CAD of distal vessel anatomy

Catheterization will be needed if


intervention is planned

Abil ity to quantify lesion severity


can be l imited by sign ificant
calcification

Radiation and radiocontrast


exposure

C M R imaging Gadoliniu m-enha nced images Accu rate test for myocard ial Some patients experience
identify viable and i nfarcted via bility claustrophobia
myocard i u m
M a y b e contra indicated i n patients
Id entifies anomalous coronary with pacemaker, ICD, or other
arteries implanted device

Gadol i n i u m is contra ind icated in


kidney fa i l u re
Sinus rhythm and a slower heart
rate are needed for i m p roved
image quality

Lim ited availabil ity a n d expertise

AV = atrioventricular; CAC = coronary artery calcium; CAD = coronary artery disease; CMR = cardiac magnetic resonance; ECG = electrocardiography; ICD = implantable
cardioverter-defibrillator; LBBB = left bundle branch block; LVH = left ventricular hypertrophy; SPECT = single-photon emission CT; WPW = Wolff-Parkinson-White.

Cl capacity and heart rate and blood pressure response, can be of myocardial oxygen demand, patients should continue to
u t i lized in prediction models such as the D u ke t readmill score. exercise until l i m i ted by symptoms. Achiev i ng a rate pressure
CONT
which factors development of symptoms. degree of ST-segment product (heart rate x systol ic blood pressure) of at least
depression, and exercise duration to provide incremental 2 5 , 0 0 0 is also considered an adeq uate workload. as t h is
prognostic i n formation for 5-year mortality risk. Heart rate measure renects left ven t ricular myocardi a l performance. A
recovery is another powerfu l predictor; patients wit h a heart standard Bruce protocol i ncreases t he speed and grade of t he
rate drop of less than 12/m i n in the first m in u te after cessation t rea d m i l l every 3 minutes. and patients who have poor func­
of exercise have a h igher mortality rate. tional capacity and ca n not achieve at least the first s tage of
lschemia is iden t i fied on the basis or t he development of the Bruce protocol (5 metabol i c equivalents [ M ETs]) have
1 m m or greater o f horizo n tal o r downsloping ST depression significantly h igher a l l -cause mortal i ty. Stress tests s hould be
with exercise (Figure I) . but the coronary territory i nvolved terminated when the patient has exerted maximal effort and
cam1ot be localized based on the ECG changes alone. Ideally, achieved at least 85'1o PM H R , the patient requests to stop or
patients should exercise for 6 to 12 m i n u tes to provide ade­ experiences sign i ficant angina! or other physical symptoms,
q uate t i me for development of maximal metabolic demand. or when other adverse markers develop. such as exertional
Although achieving 85% of t h e age-predicted maxi mal heart hypotension, signi ficant hypertension, ST-segment e levation
rate ( P M H R) is considered adequate for d iagnosis of i schemia. o r signifi ca n t ST-segmen t depressi o n . o r ven t ri c u l a r or
as heart rate a n d blood pressure are t he major determina nts supraventricular a rr hy th mias.

7
D i a g n ostic Testing in Card i ol ogy

BASEl.INE 2'-L>..
X ST
L••d Ltnd
EXERCISE STAGE 1 49 bpm ST @ l OmmlmV ST EXERCISE STAGE 4 136 bpm ST @ lOuunlwV ST
00:00 1.0 SOms poslJ Slopt 9:29 10.S SOu1> pa.tJ Slop<

''k
���;,jr-- � ��;.:1r-Jr--
�: I · ·:� l �: I "

�� �·1.1 r--��
•·· ··· ..

;; '1i

���� ��;;:t-�
0.5
0.1
0.4
-0.1
1.1
0. 1
0.1
-0.2
-1.9
-0.S
-2.4
-0.9
-2. 1
-0. l
-2.0
-1.0

F I G U R E 1 . El ectrocard iogram recorded d u ring exercise stress testi ng. The presence of 2-mm downsloping ST-segment depressions i n leads I, II, I l l, and aVF, and leads V3
through v6 ind icates ischemia.

CJ
The decision about whether to keep a patient on card iac t he pat ient is able to exercise because of t h e potential fr)r a
medications during stress tes t i ng should be i ndividual ized false- posit ive test owing to a septa! perfusion abnorma l i ty t hat
CONT.
based on the c l i nical question being addressed. I f the stress test may occur with exercise. The choice o f imaging moda l i ty
is being performed to establish the diagnosis of CAD, medica­ should be based on local expertise a nd patient characterist ics.
t ions such as �-blockers and n i t rates should be wit hheld for at I n stress tes t i ng w i t h adjunct ive imaging, basel i n e i m ages
least 24 hours before the test. If the stress test is being performed are obt a i ned and compared w i t h i m ages obtained after ei ther
to evaluate symptoms or to define prognosis in a patient with exercise or pharmacologic stress (Table 4) . Exercise invokes
known d isease. the patient should remai n on current therapy to ischemia as t he epicard i a l vessels become unable to m a i n t a i n
determ ine i f ischemia is presen t on the current regimen . adeq u a te flow related t o myocardial oxygen d e m a n d via
There are several indications for stress testing w i t h add i­ autoregu lation, and ischemia develops d i stal to t he obstruc­
ti onal imaging with either echocardi ography, CM R imaging, or t ion . Dobuta m i n e. l i ke exercise, i ncreases myocard i a l oxygen
perfusion imaging w i t h S PECT or PET/CT. These include i nabil ­ demand and e l i c i ts ischemia because of i nsuff1cient perfusion
ity to exercise. base l i ne ECG abnormali ties, and cond it i o n s i n to the a ffected myocardium. Vasodilators. such as regadenoson
which exercise i s contrai nd icated . Patients w i t h abnormal or adenosine, produce hypere m i a and a flow d isparity between
base l i n e ECGs that i n terfere with the i nterpreta t i o n of t he exer­ myoca rdium suppl ied by the stenotic vessel ( i n which the d i s­
cise ECG ( for example, left bundle branch block [LBBB] . left tal vasculature is a l ready max i m a l ly d i lated) as compared w i t h
ventricular hypertrophy w i t h ST-segment abnorma l i ties. or a the myocard i u m supp l i ed b y unobstructed vessels. I n add i t ion
paced rhyt h m ) should u ndergo s t ress i m ag i ng lo iden t i fy to ide n t i fy i ng t h e presence o f d i sease, perfusion imagi ng can
obstruct ive CA D. I n addition, stress testing w i t h imaging may define the loca t ion and extent o f reduced perfusion and pro­
be helpful lo elucidate a d i agnosis i n patients w i t h i ndeterm i­ vide add i t i o nal prognostic i n fo rmat io n compared with ECG
nate resu lts on t readm i l l test i ng. Patients with right bundle s t ress tes t i ng alone. These i m aging mod a li t ies may a lso be
branch block ( RBBB) . b i fascicular block. or who are o n d igoxin used to q ua n t i fy i n farction and assess myocard i a l viab i l i ty. The
can undergo exercise stress test i ng. but ST segments may be additional i n formation and impact on patient care obtai ned
more d i fficu l t to interpret or may produce false-posi tive resu l ts. with i maging must be bala nced with the add i t ional costs,
Patients w i t h severe aort ic stenosis, abdomi na l aortic aneu­ t i me, and exposure to rad i a t i o n or contrast agents incurred .
rysm, severe hypertension. or uncontrolled a rrhyth m i as should Exercise stress echocardiography is performed w i t h either
not exercise : rather, t hese pat ients should undergo pharmaco­ supi n e ergometry or t read m i l l test i ng. Supine ergometry allows
logic stress test i ng with vasod i lators. Pa t ients who are unable to for contin uous i maging during exercise. whereas with t read­
exercise should u ndergo pharmacologic stress testing w i t h m i l l tes t i ng. i mages need to be obtained i mmediately after
i magi ng. I n addit ion . i n patients with LBBB u ndergoing nuclear exercise. and any delay can reduce t he accuracy of the i n forma­
stress testing, a pharmacologic stressor should be used even i f t ion obtai ned. New regional wal l motion abnormalities seen on
8
D i a g n ostic Testing in Card i o logy

TABLE 4. Interpretation of Stress Testing with Imaging


Stress SPECT
At Rest After Stressor Interpretation
Normal Normal Normal, n o ischemia
Normal Perfusion d efect Normal fu nction at rest, ischemia after stress
Perfusion defect Perfusion d efect I nfarct
Normal LV dilation Small o r no distinct zone of ischemia, possible bala nced ischemia or
m u ltivessel CAD

Stress Echocardiography
At Rest After Stressor I nterpretation
Normal Normal Normal, no ischemia
Normal Wa l l motion abnorma lity Normal function at rest, ischemia with stress
Abnormal Abnormal I nfarct
Normal LV d i l ation Sma l l or no distinct zone of ischemia, possi ble bala nced isch emia or
m u ltivessel CAD

CAD = coronary artery disease; LV = left ventricle; SPECT = single-photon emission CT.

Cl
t h e echocardiogram fo l l owing exercise i n d i cate areas o f stress echocardi ography a l l ows assessment of wall motion at
ischemia (see Tab l e 4) . Wal l m o t i o n abnorma l i t i es at rest t ha t rest and at peak or i m mediately following imaging to assess for
CONT.
do not change w i t h exercise usually ind icate i n farct ion. obstructive CA D. I f the examination is performed to assess
I mprovement i n regional wall motion w i t h low-dose exercise dyspnea on exertion or valvular function w i t h exercise, t hese
or dobutamine that worsens at higher levels suggests viable but should be specifically requested in order to be sure t ha t ade­
h ibernating myocardi u m . A s wit h perfusion i m aging, t h e quate echocardiographic i n formation is o b t a i ned . Rout ine
extent o f w a l l motion abnormali t ies provides prognostic i n for­ t ranst horacic echoca rdiography (TTE) evaluates left and right
mation regarding risk of future cardiovascul a r events. ventricular size. t h ickness, and function; valvular morphology
The sensitivity of stress echocardiography is reduced with and fu nction; diastolic fu nction ; and t h e pericardium. These
s i ngle-vessel disease and i s dependent on timely i maging. In a re not necessarily rout i nely performed in a st ress echocardio­
addition, i n terpretation can be more subjective t h a n with gram so if' t h is i n forma tion is clinica l ly i mportant, it m ay be
o t h e r m od a l i t ies, parti c u l a rly w i t h base l i ne wa l l motion necessary to obtain both a TTE and a s t ress echocardi ograrn.
abnormali ties o r systolic dysfunction . A major advantage of Dobutam i n e stress echocardiography is used for patients who
s t ress echocard i ography i s t h e abi l i ty to obtain add i t ional cannot exercise and can be particularly useful for evaluation of
i n format i o n , such as changes i n p u l monary pressures o r myocardial viab i l ity (Table 5) and to evaluate aortic stenosis i n
cha nges i n valvular fu nction w i t h exercise. At m i n i m u m . patients w i t h a low ejection frac tion.

TAB LE 5. Interpretation of Myocardial Viability Studies


SPECT Viability Testing
Initial Study (at rest) Rest Study Repeated After 4-24 h (with thallium) Interpretation
Perfusion d efect Perfusion defect Fixed d efect: i nfa rct, no viability

Perfusion d efect Reperfusion of area Viable myocard i u m

PET Viability Testing


Baseline Metabolism Interpretation
Perfusion defect Metabo l ically active M ismatch indicative of via ble myoca rdi u m

Echocardiography Viability Testing


Baseline Response to Dobutamine Interpretation
Wal l motion abnormality Low dose: i m p rovement of fu nction Biphasic response i nd i cative of viable
myocardium
Highe r dose: worsening of function

SPECT = single-photon emission CT.

9
D i a g nostic Testing i n Card i ol ogy

c::J in blood flow with stress. Radioactive tracer is injected and


SPECT imaging takes advantage of the relative differences Like SPECT imaging, cardiac PET can be used to detect
ischemia. PET imaging provides improved diagnostic accuracy
CONT.
taken up by the myocardium with blood flow. I mages are compared with SPECT and can be particularly useful in
obtained at rest. Then. with exercise or vasodilator stress, a patients with nondiagnostic imaging stress tests, obese
second injection is given. Tracer is again distributed with patients, and patients with diabetes rnellitus. Cardiac PET also
blood flow and, therefore, less tracer is taken up in the left allows for assessment of peak stress ejection fraction, quanti­
ventricular region supplied by a stenotic vessel. This relative tation of myocardial blood flow, and evaluation of myocardial
difference in flow bern1een stress and rest tomographic images metabolism. Because some PET radiotracers identify meta­
is seen as a perfusion defect and is indicative of CAD (Figure 2) . bolically active myocytes, it is an excellent tool to evaluate for
Most commonly, technetium-based radiotracers are used. viabilily. Because cardiac PET uses CT for attenuation correc­
These have a higher energy and provide good image quality. tion. limited anatomic information about coronary calcifica­
This is particularly useful when there is potential for soft­ tion is also obtained. Because of the short half-life of PET
tissue attenuation that can interfere with interpretation. such radiotracers, all studies must be performed with vasodilators.
as with breast attenuation in women. Thallium is a potassium The utility of PET/CT scanning is limited by its availability.
analogue and can only be taken up by active myocytes. Like CMR imaging can be used for evaluation of myocardial
technetium. thallium can be used for myocardial perfusion and pericardia! disease processes and can be particularly use­
imaging, but because it requires active metabolism, thallium f·ul for evaluation of infiltrative and inflammatory diseases. It
also can be used to assess viability. Myocytes that on initial can be utilized to detect the extent and severity of myocardial
stress testing appear to be infarcted may slowly take up thal­ infarction and viability. Because CMR imaging can be gated,
lium tracer. identifying them as viable. The benefits of thal­ measures of right and left ventricular function can be obtained.
lium are balanced against higher radiation exposure because Although not widely performed, stress testing with dobu­
of its long half-li fe. tamine to assess wall motion and vasodilators such as adeno­
Myocardial perfusion imaging can quantify the extent sine to assess perfusion can be used with CMR imaging to
and severity of disease and help direct treatment strategies. detect ischemia. Limitations of the use ofCMR imaging include
High-risk features that may be seen on myocardial perfusion the length of time needed to acquire images and magnetic
imaging include lack of augmentation of post-stress ejection interference with cardiac implanted electronic devices. Cl
fraction. cavity dilatation. and new wall motion abnormalities.
K E Y P O I N TS
Unlike stress echocardiography. images are not obtained
immediately post stress and are o ften delayed. If there is evi­ • Stress testing is most efficacious in patients with an HVC
dence of a new wall motion abnormality in these delayed intermediate pretest probability of coronary artery dis-

images, it signifies a high degree of stenos is. ease, because it is these patients who, by the result of
their stress test, can be reclassified into higher or lower
risk categories.

�UPlliE • In patients who are able to exercise and have a normal HVC
_t.flT
baseline electrocardiogram, the initial type of stress
SEP l4T \ testing should be exercise stress testing.
STIUSS_fBP(G} • Patients with abnormal baseline electrocardiograms

t) l)
SUPlflE
(ECGs) that interfere with the interpretation of the exer­
;.rn

)
cise ECG (for example, left bundle branch block, left
SEP , L.r
' ( ventricular hypertrophy with ST-segment abnormali­
HIF
FUST 1-IJP(G} A ties, or a paced rhythm) should undergo stress imaging
to identify obstructive coronary artery disease.
·
. ''"H<E
AllT
......
f.;:.5E • ll..F' EX

CJ
Visualization of the Coronary Anatomy
l�JF
STnFSS FRP(G) Coronary angiography and coronary CT angiography (CTA)

,,.
·;.1.w·11;i::;
AtlT provide anatomic information regarding the coronary vessels
.... ....
. ..,

�·
(Figure 3) . Both procedures require iodinated contrast and
f:&.SE
.., ,,, J.:. - expose the patient to radiation. Coronary angiography pro­
tiff
R£ST FBP(GJ B vides a two-dimensional image of the lumen of the vessel filled
with contrast. Assessment of the stenotic lesions is made from
F I G U R E 2 . Selected images from a nuclear perfusion single-photon emission
multiple views of the vessel. Coronary CTA can provide addi­
CT (SPECT) stress study. Short axis views (panel A ) of the heart with stress (top row)
and at rest (bottom row) show a radiotracer defect in the septum and inferior wall tional information about some of the characteristics of the
that is filled on the rest images. Long axis views (panel 8) demonstrate an apical plaque. I f. however. culprit lesions are visualized on coronary
filling defect with stress ( top row) that is perfused on rest images (bottom row). CTA. the patient typically requires coronary angiography for

10
Diag nostic Testing i n Cardiology

associated with a higher incidence o f abnormal perfusion o n


SPECT imaging. Because o f its cost and associated radiation
exposure, measurement of coronary artery calcium should be
limited to a select group of asymptomatic patients with an
intermediate Framingham risk score (10%-20%) in whom
results will influence treatment strategy. CAC scoring may be
useful particularly if the results will influence treatment strat­
egy, such as initiation of lipid-lowering therapy.

KEY POINT
• Measurement of coronary artery calcium should be lim- HVC
ited to a select group of asymptomatic patients with an
intermediate Framingham risk score (10%-20%) in
whom results will influence treatment strategy because
of its associated cost and radiation exposure.

Risks of Coronary Diag nostic Testing c::J


l n addition to t h e physical a n d societal costs of downstream
testing t hat result from inappropriate testing, each o f t he
rnodaUties used for dfagnosis and risk stratification carry spe­
cific risks. It is important to determine the pretest probab i l i ty
F I G U R E 3 . Coronary angiogram. Radiocontrast dye is injected via catheter
of disease and to focus additional testing appropriately. For
(arrow) d u ring coronary angiography, demonstrating 85% to 90% occlusion of the
mid right coronary a rtery (asterisk). example, ordering an exercise stress test in a patient with a
low pretest probab i l i ty of disease may resul t in a false- positive

c::J
stress test and additional downstream testing. Obtaining a
better definition of the degree of coronary stenosis. Coronary CAC score in a low- risk pat ient may lead to additional tests or
angiography is also required if coronary revascularization is to procedures for an incidental finding on CT.
CONT.
be performed. If percutaneous coronary i n terven tion is indi- Exercise testing is associated with a small risk of myocar­
cated. it may be performed a t the t i me of a patient's diagnostic dial infarction or death (1 1 2500 tests) . Exercise stress testing is
catheterization. Coronary a ngiography may be useful as a contraindicated in patients with unstable cardiac conditions,
diagnostic test in patients who, despite maximal medical ther­ such as uncontrolled cardiac arrhythmias, severe symptomatic
apy, have intolerable ischemic symptoms as long as they are aortic stenosis. uncontrolled heart failure. and unstable angina.
candidates for coronary revascularization. Coronary CTA may Pharmacologic stress agents. including dipyridamole, adeno­
be used to rule o u t CAD in symptomatic patients with a n sine, and regadenoson. are associated with development o f
intermediate r i s k of coronary disease. O t her l i m i tations of high-degree atrioventricular b l o c k a n d bronchospasm .
coronary CTA include poor visualization of distal vessels and Nuclear stress testing, CAC scoring. coronary CTA. and
a rtifact from calcification that may l i mit interpretation. coronary angiography expose the patient to radiat ion . The
Suspected coronary anomalies, such as anomalous coro­ amount of exposure is dependent on factors such as the radi­
nary origins, can be evaluated by coronary CTA. CMR imaging. otracer used. equipment, operator technique, complexity of
or coronary angiography. These imaging modali t ies can help procedure performed. and patient characteristics (such as
identify those abnormalities that are associated with a higher body size) .
risk of sudden cardiac death. Cl Various contrast agents are used for CMR imaging (gado­
l i n i u m ) . echocardiography (microbubble contrast agents used
Coronary Artery Calcium Scoring for enhancement of endocardial borders ) . coronary CTA. a nd
Coronary artery calcium (CAC) scoring provides information coronary a ngiography (nonionic contrast) . Nonionic contrast
regarding the burden of atherosclerotic disease but does not may be associated with hypersensitivity reactions and acute
provide information regarding the degree of obstruction it kid ney i nj u ry whereas gadolinium is associated w i t h the
may be causing. CAC scoring can be performed with either deve lopment of nephrogenic fibrosing dermatopathy in
electron beam or multi-detector CT, and newer technologic patients with chronic kidney disease.
advances limit radiation exposure to the patient. It detects the Cardiac catheterization can resu l t in complications from
presence of calcification in the walls of the coronary arteries, vascular access, injury to the coronary arteries. dissection of
which is directly proportional to the degree of plaque burden the aorta. or disruption o f plaque res u l t i ng in peripheral
present. CAC scores are categorized as follows: 0, no disease; embo l i and possible stroke. Vascular access complications
1-99, mild disease; 100-399, moderate disease; and above 400, include retroperitoneal hematoma from bleeding at the groin
severe disease. Coronary calcium scores greater than 400 are access site as well as pseudoaneurysm at the arterial puncture

11
D i a g n ostic Testing in Card i o logy

CJ si te. Both o f these complications req u i re prompt recognition


and t reatment.
CONT.

Diag nostic Testi n g fo r


Structura l Heart Disease
Diagnostic Lesting for sLructural heart disease should be based on
a t horough history and physical examinat ion . New murmurs or
a change in examination findi ngs or symptoms in a patient with
known structural heart disease should prompt further eva lua ­
tion. Routine yearly imaging eval uation of structural heart d i s­
ease in asymptomatic patients is usually not i ndicated . Benign
murmurs. such as grade 116 or 2/6 midsystolic murmurs. are
common with pregnancy. anemia. and other hjgh-flow states,
and do not routinely need echocardiographic evaluation .
I maging modalit ies used to evaluate s t ructural heart d i s­ F I G U R E 4 . Transesophageal echocardiogram. The transducer is posteri or to the
ease are l isted in Table 6. Eva l uation of structural heart disease heart, and the left atrium (LA) and left atrial appendage (LAA, arrow) are more easily
typically begins with a TTE. TTE provides i n formation about seen than with transthoracic echocardiography, showing absence of thrombus in
the appendage. LV left ventricle.
=
left a n d right ven t ricu lar cavity size. th ickness, and fu nction.
as we l l as quantitat ive i n formation regard i ng valvular func­
t ion . diastolic fu nct ion . and fi l ling pressures. TTE can be used Diag nostic Testi n g fo r CJ
w i t h i n L ravenous agitaLed saline contrast, normally cleared by
Cardiac Arrhyth m ias
t h e pul monary circu la t io n . Lo document t h e presence o f a n
I n addition to a careful history and physical examination. t h e
i n l racardiac s h u n t or a patent foramen ovale. Atrial septa!
evaluation of a patient w i t h a history of palpitations, presyncope.
defect i s suggested by shu n t i ng o f m i crobubbles fro m the righ t
or syncope in which an arrhythmfa is suspected begins with a
atrium to L h e left atrium . TTE is a noni nvasive procedure and
12 - lead resting ECG. Evidence of preexcitation. ectopic rhythms.
i s t he preferred i maging mod a l i ty for evalmH ing an terior
atrioventricular block. or intraventricular conduction delay may
s t ructures o r t he heart. such as the aort ic valve.
give insight into the etiology of the symptoms. I n patients in
Transesophageal echocardiography (TEE) takes advan tage
whom t he presence of structural heart disease is suspected, echo­
of t he prox i m i ty of t he heart to L he esophagus for better i mage
cardiography may also be i ndicated. Because of the intermittent
qua l ity. The procedure req u i res sedation and is con t rai nd i­
nature of arrhythmias, theiJ· diagnosis and documentation can be
cated in pa tients w i t h esophageal strictures or act ive esop ha­
chal lenging. Monitoring and diagnostic strategies are based on
geal varices or bleeding. Complications incl ude esophageal
the frequency of the patient's episodes (Table 7. on page 14).
i nj u ry and bleed i ng. TEE is commonly used to evaluate for
Patients with daily symptoms can be evaluated with a 24- or
endocard i t i s in pa tients with a high pretest probab i l i ty : to
48-hour ambu latory ECG monitor ( Holter monitor). vvhereas
assess for diagnostic findi ngs or compl ications of endocard i t is
patients with less frequent episodes require other monitoring
(such as abscess) : Lo better visual ize valvular pathol ogy. pa r­
s trategies. For i n frequent symptomatic events. an external
ticularly when pla n n i ng repai r: to eval uate specific st ructures
patient -t riggered event recorder can capture t he arrhyt hmia.
that cannot be we l l visual ized on TTE (such as prosthet i c heart
provided the event lasts long enough for the patient to record il.
valves) : t o evaluate acute aortic pathologies : and to ru le out left
A looping event recorder captures several seconds of t l1e ECG
a t rial t h rombus prior to cardioversion (Figure 4) . CJ
signal prior to tl1e device being t1iggered and is useful when epi­
KEY POINTS sodes are accompanjed by syncope or presyncope. For very i n fre­

HVC • Routine yearly imaging evaluation of structural h eart quent events, an implanted loop recorder may be wananted.
disease in asymptomatic patients is usually not indi­ Exercise testing is also frequently employed in patients with
cated; benign murmurs, su c h as gra de 1/6 or 2/6 a suspected or known arrhythmja. Treadm i l l exercise testing is
midsystolic murmurs, are conm1on with pregnancy, an important tool for evaluating chrnnotropic response. ischenua,

anemia, and other high-flow states and do no t ro u t i n ely and exercise-i nduced or adrenergically induced arrhytlU1Ua.
need echocardiographic evaluation . O nce a p a t i e n t is d i agnosed w i t h an arrhy t h m i a or
arrhy t h m ia- prone cardiovascu lar condition. d iagnostic elec­
• Evaluation of structural heart disease typically begins
trophysio logy testing can be helpful for both risk strati fication
with transthoracic echocardiography, which provides
a n d treatment (such as catheter ablat io n ) . Selection o f t hese
information about ventricular cavity size, thickness, and
d iagnostic tests is dependent upon the particular pat ient and
function, as well as quantitative information regarding
the d iagnostic concerns, and most patients w i t h arrhy t h m ias
valvular function, diastolic function, and filling pressures.
do not requ i re a n electrophysiology study. CJ

12
D i a g n ostic Testi n g in Card i o logy

TABLE 6. Diagnostic Testing for Structural Heart Disease


Diagnostic Test Major I ndications Advantages Limitations

Tra nsthoracic Heart fai l u re Accurate d iagnosis of presence Operator-dependent data


echocardiography and severity of structural heart acqu isition. Interpretation
Cardiomyopathy
disease req u i res expertise.
Valve disease
Quantitation of LV size and Varia b i l ity i n i nstru mentation
Congenital heart disease function, pul monary pressu res,
I mage qual ity l i m its d iagnosis i n
valve function, and i ntracardiac
Pu lmonary hypertension some patients, m a y requ i re
shu nts
microbubble contrast agents
Aortic disease Widely available, portab l e, fast
Pericardia! disease

Transesophageal Endocarditis High-quality i mages, especia l ly of Req uires esophageal intubation,


echocardiogra phy posterior cardiac structures typica l ly with conscious sedation
Prosthetic valve dysfu nction
Most accu rate test for
Aortic disease
endocarditis evaluation,
Left atrial thrombus prosthetic valves, and left atrial
thrombus

Three-d imensional Mitra! valve disease Improved tomographic imaging Adjunct to two-d imensional
echocardiography i maging
ASD (percutaneous ASD Used d u ring cardiac procedures
closu re) for device placement Limited ava i l a b i l ity a n d expertise

Improved assessment of LV
g l o bal/reg ional systol i c function

Radionuclide a n g iography Evaluation of LV systolic Qua ntitative EF measurem ents Radiation exposure
( M UGA) function
Accu rate for serial LVEF No data on other cardiac
measurements with card iotoxic structures
drugs

Cardiac catheterization Congenital heart disease Direct measurement of Invasive


(left and rig ht) intracardiac pressures, gradients,
Coronary a rtery disease Radiation and radiocontrast
and shu nts
exposure
Valve assessment
Contrast a ngiography provides
I mages not tomographic, l i m iting
S h u nt assessment visualization of complex cardiac
evaluation of complex three­
a n atomy
d imensional anatomy
Allows percutaneous i ntervention
for structural heart disease

Coronary CT a n g iography Coronary artery disease Visualization of complex cardiac Invasive


anatomy
Co ngenital heart disease Radiation and radiocontrast
High-reso l ution tomographic exposure
images
Image acquisition i m p roved with
sinus rhythm and slower heart
rate

CMR imaging Congenital heart disease Hig h-resolution tomogra phic Lim ited avai lability a n d expertise
imaging a n d blood-flow data
Aortic disease Some patients experience
Qua ntitative RV volumes and EF cla ustrophobia
Myocardial disease ( i nfiltrative
disease, myocard itis, No i o n izing radiation o r contrast May be contra i n d icated in
hypertrophic cardiomyopathy) patients with pacemaker, !CD, or
Enab les three-dimensional
other i mplanted devices
RV cardiomyopathy (ARVC) reconstruction of aortic and
coronary anatomy Gadolinium is contra i n d icated i n
Qua ntitation of LV mass a n d
kidney fa i l u re
fu n ction
Sinus rhyth m and slower heart
rate are needed for improved
image qual ity
Chest CT Aortic disease High-resolution tomogra phic Radiation and radiocontrast
images exposure
Coronary disease
Enab les th ree-d imensional
Cardiac masses
reconstructi on of vascu l a r
Perica rd ia! disease structures

ARVC = arrhythmogenic right ventricular cardiomyopathy; ASD = atrial septal defect; C MR = cardiac magnetic resonance; EF = ejection fraction; ICD = implantable cardioverter­
defibrillator; LV = left ventricle; LVEF = left ventricular ejection fraction; MUGA = multi-gated acquisition; RV = right ventricle.

13
Coro n a ry Artery D i sease

TABLE 7. Diagnostic Tests for Suspected or Known Cardiac Arrhythmias


Diagnostic Test Utility Advantages Limitations

Resting ECG I n itial d i agnostic test i n a l l 1 2-lead ECG recorded d u ring Most arrhythm ias are
patients the arrhyth mia often identifies interm ittent and not recorded
the specific arrhyth m ia on a resting ECG

Ambulatory ECG ( H o lter Frequent (at least dai ly) Records every heart beat Not helpful when arrhyth mia
monitor) asymptomatic or symptomatic d u ring a 24- or 48-hour period occurs less frequently
arrhythmias for later analysis
ECG leads l i mit patient
Patient log al lows correlation a ctivities
with symptoms

Exercise ECG Arrhyth mias p rovoked by A llows d ia gnosis of exercise- Physician supervision needed
exercise related a rrhyth mias i n case a serious a rrhyth m ia
occurs
Al lows assessment of i m pact of
arrhyth mia on blood pressure Most arrhythmias are not
exercise related

Patient-triggered event I nfrequent symptomatic S m a l l , pocket-sized recorder is Only useful for symptomatic
recorder a rrhyth mias that last more than held to the chest when arrhyth mias that persist long
1 -2 min utes symptoms are present enough for patient to activate
the device
Recorded data are transm itted
to central monitoring service Arrhythmia on set not recorded

Not useful for syncope or


extremely brief a rrhyth mias
Looping event recorder Infrequent symptomatic brief Conti nuous ECG signal is ECG leads l i m it patient
(wearable) arrhyth mias recorded with the previous 30 a ctivities
seconds to 2 m i n utes saved
Syncope Device records only when
when the patient a ctivates the
a ctivated by patient
record ing mode

Arrhythmia onset is recorded


I m plantable loop recorder Very i nfreq uent asymptomatic Long-term conti nuous ECG Invasive procedure with m i nor
or symptomatic a rrhyth mias monitoring with patient- risks
triggered or heart rate-
Device must be explanted later
triggered episode storage

Specific heart rate or ORS


parameters can be set to
i n itiate record ing of data
Mobile cardiac outpatient Continuous outpatient ECG Auto-triggered a n d patient- ECG leads l i mit patient
telemetry record ing for precise triggered capture of a ctivities
quantification or capture of arrhythmia events
rare arrhyth mia Resource intensive
U p to 96 hours of retrieva ble
memory
E l ectrophysiology study Used for both indu cing, The origin and mechanism of I nvasive proced u re with some
identifying, a n d clarifying a n arrhyth mia can be precisely risk
mechanism of arrhyth mia as defined
Some arrhyth mias may not be
well as potential treatment
inducible, particularly if the
(catheter ablation)
patient is sedated

ECG = electrocardiogram.

KEY POINT
Corona ry Artery Disease
• Patients with a suspected arrhythmia who experience
daily symptoms can be evaluated with a 24- or 48-hour Sta ble Ang i n a Pectoris
ambulatory electrocardiographic monitor (Holter moni­
Diagnosis and Evaluation
tor) , whereas patients with less frequent episodes
The most common manifestation of coronary artery disease
require other monitoring strategies, including various
(CAD) is stable angina pectoris: chest pain, pressure, or dis­
types of longer-term event recorders.
comfort that develops with exertion and is relieved with rest.

14
Coronary Artery D isease

Symptoms often occur when the burden of atheromatous optimal medical therapy, measure exercise capacity, and
plaque results in fixed coronary stenosis and limitation of evaluate the extent and severity of ischemia.
blood flow, leading to an imbalance between myocardial The development of coronary CTA is an emerging
oxygen supply and demand. When patients with cardiovascu­ alternative to stress testing, but coronary CTA does not
lar risk factors present with chest pain, the location of the provide important functional information, such as extent of
pain, quality of symptoms (sharp/dull, transient/persistent, ischemia, reproduction of symptoms, or exercise capacity.
occurring at rest/with exertion) , and the age and sex of the Coronary CTA is useful for diagnostic purposes in patients at
patient can help to differentiate stable angina pectoris from intermediate risk for CAD if stress testing is contraindicated
other causes of chest pain, such as gastrointestinal, musculo­ or revascularization is unlikely to be performed or change
skeletal, or pulmonary causes. These factors are also used to management.
determine a patient's pretest likelihood of CAD (Table 8) . The use of invasive coronary angiography in patients with
The decision to perform exercise or pharmacologic stable angina pectoris is generally limited to those with persis­
stress testing or coronary CT angiography (CTA) is based on tent or progressive life-limiting symptoms while on optimal
the pretest likelihood of CAD, the patient's baseline electro­ medical therapy or those with high-risk criteria on noninva­
cardiogram (ECG) , the patient's ability to exercise, and the sive stress testing or coronary CTA (see Figure 5) .
patient's comorbid illnesses, such as asthma or emphysema,
KEY POINTS
that would limit pharmacologic testing (Figure 5) .The selec­
tion of tests for evaluating patients with chest pain is dis­ • When patients with cardiovascular risk factors present
cussed in Diagnostic Testing in Cardiology. Stress testing is with chest pain, the quality of symptoms, the age, and
most useful in patients at intermediate pretest likelihood of the sex of the patient can help to differentiate stable
CAD (10% to 90%) . In patients with low pretest probability, a angina pectoris from other causes of chest pain.
normal test result only confirms that the patient is low risk, • Stress testing is most useful in patients at intermediate HVC
and an abnormal stress test result is most likely a false­ pretest likelihood of coronary artery disease (10% to 90%) .
positive, possibly leading to more testing (additional stress
testing or invasive angiography) . In patients with a high pre­ General Approach to Treatment
test likelihood, the use of stress testing for diagnostic pur­ of Stable Angina Pectoris
poses is not indicated, as an abnormal test result only All patients with ischemic heart disease should be counseled
confirms the presence of disease and a normal test result is on the importance of risk factor modification, including life­
most likely to indicate a false-negative result. style changes, such as smoking cessation, weight manage­
Stress testing in patients with high pretest likelihood can ment, daily physical activity, and diet modification; as well as
be used to obtain prognostic information, but the results control of modifiable risk factors, such as diabetes mellitus,
should not affect the initiation of optimal medical therapy. In hype1iension, and hyperlipidemia. Medical therapy should be
patients who have been started on medical therapy for CAD, initiated in all patients with ischemic heart disease. The com­
stress testing can be used to determine a patient's response to bination of risk factor modification and medical therapy is

TABLE 8. Pretest Likelihoods of Coronary Artery Disease in Low-Risk and High-Risk Symptomatic Patients
Pretest Likelihood
Nonanginal Chest Pain• Atypical Anginab Typical Angina<
Age (y) Men Women Men Women Men Women

35 3-35 1-19 8-59 2-39 30-88 1 0-78

45 9-47 2-22 2 1 -7 0 5-43 5 1 -92 20-79

55 23-59 4-25 45-79 1 0-47 80-95 38-82

65 49-69 9-29 7 1 -86 20-5 1 93-97 56-84

aNonanginal chest pain has one or none of the components for typical chest pain.

bAtypical angina has two of the three components for typical angina.

C"fypical angina has three components: ( 1 ) substernal chest pain or discomfort, (2) provoked by exertion or emotional stress, (3) reli eved by rest and/or n itrog l yce r i n .

NOTE: Each value represents the percentage with significant coronary artery disease. The first is the percentage for a low-risk, mid-decade patient without diabetes mellitus,
smoking, or hyperlipidemia. The second is that of the same-age patient with diabetes mellitus, smoking, and hyperlipidemia. Both high- and low-risk patients have normal results
on resting electrocardiography. If ST-T wave changes or 0 waves had been present, the likelihood of coronary artery disease would be higher in each entry of the table.

Adapted with permission of Elsevier Science and Technology Journals, from Gibbons RJ, Abrams J, Chatterjee K. ACC/AHA 2002 guideline update for the management of
patients with chronic stable angina--summary article: a report of the American College of Cardiology/American Heart Association Task Force on practice guidelines (Committee
on the Management of Patients With Chronic Stable Angina). J Am Coll Cardiel. 2003 Jan 1 ;4 1 ( 1 ): 1 S9-68. [PMID: 1 2S 70960]; permission conveyed through Copyright Clearance
Center, Inc.

15
Coro n a ry Artery Disease

Cardiovascular symptoms

Pretest proba b i l ity of CAD

I
Intermediate

No additional ECG normal and Medical therapy


diagnostic testing able to exercise? for CAD

Yes

ECG abnormal and No


Exercise tread m i l l stress test
able to exercise?
• No response to therapy
Markedly
Yes
positive test • Lifestyle-limiting symptoms

• Progressi o n to unstable angina

Exercise nuclear Exercise Pharmacologic


stress test echocardiography stress test

Markedly Markedly Markedly


positive test positive test positive test

Coronary angiography

Stress Test Criteria for Markedly Positive Test


Exercise treadmill Significant ST segment .J, at low work load, ST segment i, hypotension
Exercise nuclear TID or lung uptake of thallium, ischemia in >2 vascular distributions, EF <35%
Exercise echocardiography EF <35% at rest, ischemia i n >2 vascular distributions, fall i n EF with stress

F I G U R E 5 . Diagnosis of coronary artery d isease. CAD = coronary artery d isease; ECG = electrocard iogram; EF = ejection fraction; T I D = transient ischemic d i lation.

referred to as guideline-directed medical therapy (Figure 6) . currently recommended only following percutaneous coro­
Medical therapy is divided into two categories: cardioprotec­ nary intervention (PCI) or an acute coronary event.
tive medications and antianginal medications. Cardioprotective Owing to the protective effects of p-blockers, these agents
medications improve survival, reduce the occurrence of car­ are considered first-line therapy in patients with stable angina
diovascular events, and reduce the progression of systemic pectoris. Dose titration of P-blockers is recommended until
atherosclerosis. Antianginal medications vasodilate the coro­ the resting heart rate is between SS/min and 6 0 /min.
nary vasculature or decrease myocardial oxygen demand, thus P-Blockers have been associated with fatigue, reduced exercise
reducing the frequency and severity of angina pectoris and capacity, symptomatic bradycardia, mood disturbance
improving quality of life. (depression) , and erectile dysfunction. P-Blockers are con­
traindicated in patients with symptomatic bradycardia, high­
Cardioprotective Medications grade atrioventricular block, acute decompensated heart
The use of aspirin is associated with a decreased risk of myo­ failure, and severe reactive airways disease.
cardial infarction, stroke, and cardiovascular death in patients ACE inhibitors are indicated in the treatment of patients
with CAD. Aspirin doses of 81 mg to 162 mg daily are recom­ with stable angina pectoris to reduce cardiovascular and all­
mended in all patients with established CAD unless contrain­ cause mortality. This effect on mortality is more profound in
dicated. In patients allergic to aspirin, clopidogrel is recom­ patients with concomitant diabetes mellitus and left ven­
mended as an alternative. The use of newer antiplatelet agents tricular systolic dysfunction. Additionally, ACE inhibitors are
(prasugrel, ticagrelor) as monotherapy has not been tested in indicated in patients with concomitant systemic hyperten­
patients with stable angina pectoris. Dual antiplatelet therapy sion and proteinuric chronic kidney disease. ACE inhibitors
(aspirin plus either clopidogrel, prasugrel, or ticagrelor) is are contraindicated in pregnant women and caution is

16
Coro n a ry Artery D isease

l Stable angina pectoris


J
Guideline-directed medical therapy (to include
lifestyle and risk factor modification)
• Aspirin (at least 75 m g/d)
• B-B l ocker
• Long-acting n itrate
• S u b l i n g u a l n itroglyceri n as needed for a n g i n a
• Moderate- t o h ig h-i ntensity stati n

Continued symptoms

y
• I ncrease dose of B-blocker
• I ncrease dose of long-acting n itrate
• Add ca lci u m channel bl ocker (if not a l ready receivi ng)

Continued symptoms and


i m paired q u a l ity of l ife
y
• Opti m ize long-acting n itrate, B-blocker, and
ca lci u m channel b locker
• Consider ra nolazine

Persistent symptoms despite adequate trial


of g u i d e l ine-d irected medical therapy
y
Referral for coronary a n g i og ra phy

Medically retractory angina


Coronary a natomy suita b l e for No _i..
-
,. • Externa l enhanced cou nterpu lsation a
PCI or surgical revascularization?
• Spinal cord sti m u lation a
Yes

y
Revascu la rize

F I G U R E 6 . Management of stable angina pectoris. PCI = percutaneous coronary i ntervention.


•Not recommended b y current guidelines.

Recommendations based on Oaseem A, fihn SD, Dallas P, Williams S, Owens DK, Shekel le P; Clinical Guidelines Committee of the American College of Physicians. Management of stable ischemic heart disease: summary of a clinical prac·
tice guideline from the American College of Physicians/American College of Cardiology foundation/American HeartAssociation/American Association for Thoracic Surgery/ Preventive Cardiovascular Nurses Association/Society oflhoracic
Surgeons.Ann Intern Med. 2012 Nov 20;157(1 0):735-43. [PMID: 23165665]

advised in patients with advanced chronic kidney disease. is proportional to the degree of LDL cholesterol reduction;
Angiotensin receptor blockers are considered acceptable however, statins have been proved to be beneficial in patients
alternatives in patients who are allergic to or intolerant of regardless of cholesterol level. New cholesterol management
ACE inhibitors. guidelines recommend the use of moderate- to high-intensity
Statins have been shown to reduce the risk of myocardial statins for all patients with (1) LDL cholesterol level of190 mg/
infarction and death in patients with chronic ischemic heart dL (4.92 mmol/L) or greater; (2) diabetes mellitus; or (3)
disease by 25% to 30%. The reduction in cardiovascular events greater than 7.5% estimated 10-year risk of atherosclerotic

17
Coro n a ry Artery Disease

cardiovascular disease (ASCVD) . Management of statin ther­ constipation. Ranolazine should be used with caution in
apy is discussed in MKSAP 17 General Internal Medicine. patients with advanced kidney or liver disease and in those
According to Advisory Committee on Immunization taking medications that are potent inhibitors of the CYP3A4
Practices (ACIP) guidelines, all persons aged 6 months or older pathway. Examples of strong inhibitors of the CYP3A4 path­
should have an annual influenza vaccination. In addition, way include ketoconazole, clarithromycin, tacrolimus, and
patients at risk for cardiovascular disease warrant influenza cyclosporine.
vaccination as a preventive measure for cardiovascular disease.
KEY POINTS
The use of selenium, chromium, �-carotene, vitamin C,
• Aspirin or clopidogrel (if aspirin-allergic) is recom­
vitamin E, and estrogen has not been associated with improved
mended in all patients with established coronary artery
cardiovascular outcomes and is not recommended in patients
disease unless contraindicated; the use of newer anti­
with ischemic heart disease.
platelet agents (prasugrel, ticagrelor) as monotherapy has
Antianginal Medications not been tested in patients with stable angina pectoris.
Medications to reduce the frequency and severity of angina • All patients with stable angina pectoris should receive
pectoris comprise �-blockers, nitrates. calcium channel block­ a statin and a �-blocker.
ers, and ranolazine.
• ACE inhibitors are indicated in the treatment of stable
�-Blockers and nitrates are first-line antianginal agents. In
angina pectoris, particularly in patients with concomitant
addition to their cardioprotective effects, �-blockers improve
diabetes mellitus and left ventricular systolic dysfunction.
angina pectoris by reducing heart rate, myocardial contractility,
and blood pressure, resulting in reduced myocardial oxygen
Coronary Revascularization
demand. Nitrates improve myocardial oxygen supply and
reduce myocardial oxygen demand by their effects on coronary Decision to Revascularize
and systemic vasodilation, respectively. Nitrates have not been In patients with stable angina pectoris whose symptoms are
proved to reduce the frequency of cardiovascular events (myo­ not improved with optimal medical therapy, invasive angiog­
cardial infarction, death) . Two categories of nitrates are indi­ raphy is warranted to define coronary artery anatomy and
cated for patients with stable angina pectoris: sublingual or prepare for revascularization via PC! or coronary artery bypass
spray nitroglycerin (for emergency use) and topical or oral graft surgery (CABG) . All patients should be counseled on the
nitroglycerin (for chronic, daily use) . The use of daily nitrates risks, benefits, and alternatives to angiography and revascu­
requires periodic nitrate-free intervals (typically at night) to larization before diagnostic angiography is pursued.
avoid the development of tolerance. The most frequent adverse In patients found to have significant CAD on angiography
effect of nitrates is headache. The use of either short- or long­ that would benefit from revascularization, multiple factors are
acting nitrates is contraindicated in patients who take phos­ considered in deciding which technique (PC! or CABG) would
phodiesterase 5 (PDE-5) inhibitors for erectile dysfunction be best for the patient. These include the degree of left ven­
(sildenafil, vardenafil, tadalafil) owing to the potentiation of tricular systolic dysfunction, whether the patient has had a
hypotension when these drugs are used together. prior CABG, and the patient's ability to adhere to a medication
Calcium channel blockers are second-line therapy i n treatment regimen. The SYNTAX score is an anatomic scoring
patients with stable angina pectoris who are intolerant o f system based on the results of angiography that quantifies
�-blockers or who have continued symptoms on �-blockers lesion complexity in patients vvith multi-vessel and/or left
and nitrates. All calcium channel blockers cause systemic main coronary artery disease and is useful in helping predict
and coronary vasodilation, and nondihydropyridine calcium the outcome of different revascularization strategies. The
channel blockers (diltiazem, verapamil) reduce the heart development of appropriate use criteria (AUC), a collection of
rate. Because of their vasodilatory properties, calcium chan­ clinical scenarios that mimic frequently encountered patient
nel blockers are first-line agents for the management of presentations, has assisted clinicians in making treatment
patients with Prinzmetal (variant) angina pectoris. The most decisions for patients with all forms of ischemic heart disease.
common adverse effects of calcium channel blockers are
peripheral edema, dizziness, constipation, and bradycardia. Percutaneous Coronary Intervention
Calcium channel blockers are contraindicated in patients PC! has not been shown to be superior to optimal medical
with left ventricular systolic dysfunction or advanced atrio­ therapy in patients with stable angina pectoris for reduction of
ventricular block. cardiovascular endpoints such as m011ality and myocardial
Ranolazine is a selective inhibitor of the late inward infarction. However, PC! has been associated with improve­
sodium channel in the myocardium. It is generally reserved for ment in quality of life by reducing the severity and frequency
patients who remain symptomatic with the use of �-blockers, of angina. Current guidelines recommend that diagnostic
nitrates, and calcium channel blockers. Ranolazine is an effec­ angiography and PCI be reserved for patients with refractory
tive antianginal medication; however, its use is limited by cost symptoms while on optimal medical therapy, those who are
and adverse effects such as dizziness, headache, nausea, and unable to tolerate optimal medical therapy owing to side

18
Coron a ry Artery D isease

effects, or those with high-risk features on noninvasive exercise ischemic risk is high and bleeding risk is low (Table 9) . A major
and imaging tests. risk with premature discontinuation of dual antiplatelet ther­
apy is the occurrence of stent thrombosis, a complication with
Coronary Artery Bypass Graft Surgery high morbidity and mortality.
The use of CABG in patients with stable angina is generally In patients who undergo CABG, preoperative cardioprotec­
indicated only for those who remain symptomatic with opti­ tive and antianginal medications should be continued indefinitely.
mal medical therapy and have specific angiographic findings The benefit of cardioprotective medications (aspirin, �-blockers,
(either left main disease or multivessel disease with involve­ ACE inhibitors, statins) is greatest in patients with high-risk fea­
ment of the proximal left anterior descending artery) , con­ tures such as reduced left ventricular systolic function, prior
comitant reduced systolic function, or diabetes mellitus. CABG myocardial infarction, chronic kidney disease, or diabetes.
is typically performed via median sternotomy incision and
KEY P O I NTS
institution of cardiopulmonary bypass ; however, recent
advances with off-pump CABG allow patients to avoid the • Percutaneous coronary intervention improves angina
need for cardiopulmonary bypass. Off-pump CABG is associ­ symptoms and quality of life in patients with stable
ated with adverse event and graft patency rates similar to tra­ angina pectoris but does not increase survival or reduce
ditional CABG. This less invasive procedure may be more suit­ future cardiovascular events.
able for patients with significant comorbid medical conditions • For stable angina pectoris, percutaneous coronary HVC
as it may reduce operative risk and shorten hospital and recov­ intervention is reserved for patients with refractory
ery times, but definitive proof is lacking. symptoms while on optimal medical therapy, those who
are unable to tolerate optimal medical therapy owing
After Revascularization to side effects, or those with high-risk features on non­
The long-term goals of therapy for ischemic heart disease are invasive imaging.
to maximize quality of life and exercise function and minimize • Clinical practice guidelines do not recommend the HVC
morbidity and mortality. Clinical practice guidelines do not routine use of ECG monitoring, stress testing, or ana-
recommend the routine use of ECG monitoring, stress testing, tomic testing (coronary CT angiography or invasive
or anatomic testing (coronary CTA or invasive angiography) in angiography) in asymptomatic patients after percuta-
asymptomatic patients after PC! or CABG. neous coronary intervention or coronary artery bypass
All patients with stable angina pectoris who undergo PC! graft surgery.
or CABG should be treated with aspirin (81-162 mg/d) indefi­
• In patients with stable angina pectoris who undergo
nitely. In patients who undergo PCJ, dual antiplatelet therapy
percutaneous coronary intervention, dual antiplatelet
(aspirin plus clopidogrel} is recommended for at least 1 month
therapy (aspirin plus clopidogrel) is recommended for
after bare metal stent (BMS) implantation and at least 1 year
at least 1 month after bare metal stent implantation and
after drug-eluting stent (DES) implantation, although extended
at least 1 year after drug-eluting stent implantation.
use can be considered on an individual basis if a patient's

TABLE 9. Duration of Dual Antiplatelet Therapy'


Condition No Stent Bare Metal Stent Drug-Eluting Stent CABG

Stable angina pectoris Clopidogrel, only Clopidogrel Clopidogrel Not i n d icatedb


if aspirin is
1 month 1 year
contrai n d i cated
UA/NSTEMI Clopidogrel or Clopidogrel, prasugrel, Clopidogrel, prasugrel, Clopidogrel or
ticagrelor or ticagrelor or ticagrelor ticagrelor
1 year At least 4 we eks , u p to 1 year 1 year
1 year

STEM! Clopidogrel or Clopidogrel, prasugrel, Clopidogrel, prasugrel, Clopidogrel or


ticagrelor or ticagrelor or ticagrelor ticagrelor

1 year At least 4 weeks, u p to 1 year 1 year


1 year

CABG = coronary artery bypass grafting; NSTEMI = non-ST-elevation myocardial infarction; STEMI = ST-elevation myocardial infarction; UA = unstable angina.

•Dual anti platelet therapy consists of aspirin and another antiplatelet agent, with aspirin taken indefinitely unless contraindicated.

bPreliminary data suggest clopidogrel improves patency of bypass grafts after CABG.

NOTE: Extended dual anti platelet therapy can be considered if the risk-benefit ratio is favorable.

19
Coro n a ry Artery D i sease

Acute Co ro n a ry Synd romes ST-Elevation Myocardial I nfarction


Recognition
l"l"'I General Considerations
O p t i m a l manage ment and i mproved outcomes in p a t i ents
LI.I Acute coronary syndrome (ACS) encompasses ST-elevat ion myo­
with STEM I depend on early recogn i t i o n a n d inst i tution o r
cardial infarction (STE M ! ) and non -ST-elevation acute corona1y
reperfusion therapy w i t h e i t her t h rombolysis or pri m a ry P C I
syndromes ( N STE-ACSs) . which comp1ise non-ST-elevalion myo­
(Figure 8) . Over t h e past decade. various systems- based i n ter­
cardial i n farction (NSTE M I ) and tmstable angina (UA) (Figure 7) .
ven t i o ns have proved e ffect ive in i ncreasing t h e percentage of
The pathophysiology of ACS is most commonly characterized by
pat ients d iagnosed a t the t i m e o f hospital presen tation. reduc­
plaque rupture (75% or cases) and plaque erosion (25'Y., of cases) .
i ng t h e t i m e req u i red lo reperfuse an occluded blood vessel
STEM! is caused by a complete occlusion of' an epicardial corona1y
w i t h e i t her t h rombolysis or PCI and reducing transfer t i m e to
arte1y by thrombus at the site of plaque disruption and is defined
a fac i l i ty capable o f t h rombolysis or PCI .
by the presence of ischernic chest pain (or an equivalent) and the
STE M ! is diagnosed c l i n ical ly based on the i n i tial ECG and
presence of greater than l - mm ST-segment elevalion in two or
the presence of ischemic chest pai n. Several conditions should be
more consecutive leads or new left bundle branch block on ECG.
considered i n the d i fferential diagnosis at the t i me of presen ta­
ST-segment depression in tvvo or more precordial leads (V1 t hrough
tion . particularly acute aortic dissection. pulmonary embol ism.
V) may indicate transrnural posterior i njury. I n UA and N STE M I .
and pericard i t is. A focused h istory should be taken to determine
t h e occlusion i s incomplete. NSTE-ACSs are characterized by the
the patient's qual ity and duration of symptoms. risk factors for
presence of ischemic chest pain (or an equivalent) . the notable
CAD. prior h istory of PCI or CABG. and bleeding risk as it pertains
absence of ST-segment elevation on ECG, and the presence of' eit her
to reperfusion therapy. Physical exa m i na t ion is imperat ive to
ST-segment depression or T-wave inversion on ECG. In NSTE M I .
evaluate for conditions that mimic STEM ! . i ncluding acute aortic
cardiac biomarkers (serum crealine kinase M B a n d tropon in) are
dissection (asymmetric arm pressures) and peticarcLitis (pericar­
abnormal, whereas in UA. cardiac bioma rkers are no1111a l .
dia! rub) . Physical examination should also look for signs of
Because the a tlected artery is completely occluded i n STEM!.
STEM I complications, such as hypo tension (ventricular wall rup­
ils diagnosis and treatment are markedly d i fferen t from t hose of
ture, cardiogenic shock) and heart murmur (acute m i t ra l regu r­
UA or NSTEM I. For this reason . an i nitial ECG is imperat ive in a l l
gitation) . Final ly. the physica l examination is important to assess
patients presenting with symptoms consistent with ischemic
for factors that may i n lluence treatment options (gastroi ntesti n a l
chest pai n, and once diagnosis of STEM I occurs. emphasis is
bleeding. neurologic deficits. hea1t fail ure. coagu lopalhy ) .
placed on immediate reperf'usion of the vessel via l h rornbolytic
t herapy or PCI . Patients with ischernic chest pa in but without Reperfusion
ST-segment e l evation on initial ECG are typica l ly classi fied as Reperfusion for patients w i t h STE M ! occurs pri marily via
having NSTE-ACS and then undergo laborato1y test i ng for cardiac thrombolytic t herapy and primary PCI . Despite early success
biomarkers. Because these patients do not have an ECG consistent with L h rombolytic t herapy. some studies suggest t hat 30'Y., lo
with complete occlusion of a coronary artery a nd because this 50% of patients receiving t h rombolysis do not achieve complete
group of patients is heterogeneous. risk strati fication should reperfusion . Factors that must be considered when deci d i ng to
occur prior to consideration of corona1y angiography and subse­ a d m i n ister t h rombolysis or perform primary PCI include the
quent corona1y revascularization with PCI or CABG. ava i l ab i l ity of a PCl-capable fac i l i ty. t i m e from o nset of

Acute coronary syn d rome


(ST E M ! , NSTE M I a n d u nsta ble a n g i na)

ST-segment e l eva tion ST-seg m ent depression


T-wave i nversion
l Nonspecific ECG changes J
Normal troponin
l Troponin Normal CK-M B
l Troponin
l CK-MB
l CK-MB
... y
STE M ! NSTE M I U nsta ble a n g i n a

F I G u R E 7 . Diagnosis o f acute coronary syndromes. CK-M B = creatine kinase M B ; ECG = electroca rdiographic; NSTEMI = non-ST-elevation myocardial infarction;
STEMI = ST-elevation myocard ial infarction.
20
Coro n a ry Artery D i sease

Eva l uate: 1 . Time from onset of sym ptomsa


2. H i g h-risk featu resb
3. Time to device therapy with PCI
4. Risk of throm bolytic therapy (contra ind icati ons)

T
I n itial medical thera py: aspirin, &blocker, n itrates,

I
heparin (u nfractionated or l ow-mo lecu lar-weight)

Transfer for p r i m a ry PCI C


I ---____-�
N o n-PC I-ca p a b le fa ci I i ty

PO-ca pa b l e facil ity Selection of


reperfusion strategy

F a i l u re of thrombolytic therapy Thrombo lytic thera py


Ad m i n ister P2Y,2 i n h i bitord (goa l <30 m i n fro m arriva l)e
Ad m i n ister clopidogrel
(300-mg loa d i n g dose)
Rescue PCI
Successful reperfusion
Pri mary PCI
F M C-to-device g oa l :
::; 9 0 m i n Risk stratification

Long-term medical therapy: No


aspirin, &bl ocker, ACE inhi bitor, Clinica l ly sign ificant ischemia? E F <40 % 7
n itrates, statin, P2Y12 i n h i bitor
Yes
Yes
Coronary angiogra phy
No

F I G U R E 8 . Management of ST-elevation myocardial infarction. EF = ejection fraction; FMC= first medical contact; PCI = percutaneous coronary intervention;
STEMI = ST-elevation myocardial infarction.
�1f 4 or more hours have elapsed since symptom onset, PCI is preferred.

' High-risk features such as cardiogenic shock and heart failure favor PCI.

'FMC-to·device ("door·to·balloon·) goal for patients being transferred for primary PCI is as soon as possible and � 1 20 minutes.

dP2Y1 2 inhibitors: clopidogrel, prasugrel, ticagrelor.

fSTEMI patients presenting to a hospital without PCI capability and who cannot be transferred to a PCI center and undergo PO within 1 20 minutes of first medical contact (·door-to-balloon time·) should be treated with thrombolytic therapy
within 30 minutes of hospital presentation (•door-to-needle time·) as a systems goal unless thrombolytic therapy is contraindicated.

O"Gara PT, Kushner FG, Ascheim DD, et al; American College of Cardiology Foundation/American Heart Association Task Force on Practice Guidelines. 2013 ACCF/AHA guideline for the management of ST·elevation myocardial infarction: a
repon of the American College of Cardiology Foundation/American Heart Association Task Force on Practice Guidelines. Circulation. 201 3 Jan 29;127(4):e362·425. Erratum in: Circulation. 2013 Dec 24;1 28(25):e481. (PMID: 23247304)

21
Coro n a ry Artery Disease

CJ symptoms to presentation. the presence of high -risk features. TAB LE 1 0. Contraindications to Thrombolytic Therapy for
and relative or absolute contraindications to thrombolytic ther­ ST-Elevation Myocardial Infarction
CONT.
apy (Table 10) . The preferred method of reperfusion is pri mary Absolute Contraindications
PC!, especially for patients presenting to hospitals with onsite
Any previous intracerebral hemorrhage
PC! faci lities. Because many patients with STE M ! present Lo
hospitals without onsite PCI faci lities, a t reatment algorithm is Known cerebrova scu lar lesion (e.g., a rteriovenous
ma lformation)
typically in place to emergen t ly transfer pat ients to a PCI­
capable facil ity or admin ister full-dose thrombolytic therapy. lschemic stroke within 3 months

Even when thrombolytic t herapy is administered. treatment Suspected aortic dissection


guidelines recorrm1end that patients be transferred to a PCJ­ Active bleed i n g or bleed ing diathesis (excl uding menses)
capable facility because of t he potential for thrombolytic failure. Sign ificant closed head or facial trauma within 3 months
In patients who undergo primary PC!. the time from
Relative Contraindications
patient presentation to device delivery (balloon i n flation or
thrombectomy catheter activation) is a major determi nant of H istory of chronic, severe, poorly control led hypertension
improved outcomes observed in primary PCI when compared Severe u ncontro l led hypertension on presentation
with thrombolysis. Several studies have demonstrated superior (SBP > 1 80 mm Hg or DBP > 1 1 0 m m Hg)•

efficacy of transfer for primary PC! versus thrombolytic ther­ H i story of ischemic stroke (>3 months), dementia, o r known
i ntracra nial pathology
apy: however. observational studies have reported that patients
oflen experience delays in t ransfer for primary PCI that exceed Tra umatic or prolonged (> 1 0 m i n utes) CPR or major surgery
(<3 weeks)
l hour. When PC! cannot be readi ly ach ieved within 120 min­
utes. thrornbolytic therapy is recommended i n t hose patients Recent (within 2-4 weeks) internal bleed i ng

without contraindications. Regional and national efforts have Noncompressible vascu lar puncture site
improved t hese t ransfer procedures to include the abil i ty or For streptokinase/a nistreplase: previous exposure (>5 days) o r
emergency medical services (EMS) to perform ECG in the field. previous al lergic reaction t o these agents

i m mediate transfer of patients to PCl-capable facil i t ies when Pregnancy


STE M I is recognized. and i mproved communication between Active peptic u l cer disease
non- PC I facilities and PCI facilities. Cu rrent use of a nticoag u l a nts: the higher the I N R, the higher
Table 11 shows the characteristics of the most commonly the bleeding risk
used lhrombolytic agents that a re currently ava ilable. A lthough CPR = cardiopulmonary resuscitation; DBP = diastolic blood pressure; SBP = systolic
l i fe-t h reateni ng bleedi ng events, such as intracerebral hemor­ blood pressure.

rhage. are rare. t hey carry an extremely h jgh mortal i ty rate •Thrombolytic therapy can be considered if SBP can be reduced to < 1 40 mm Hg
and DBP to <90 mm Hg with initial medical therapy.
(50%-60%) . Patients treated with t h rombolytic t herapy should

TABLE 1 1 . Characteristics of Thrombolytic Agents Used in the Treatment of STEMI


Characteristic Streptokinase Alteplase Reteplase Tenecteplase

Dose 1 .5 m i l l io n u n its over Up to 1 00 mg in 90 m i n• 1 0 u n its x 2 (30 m i n 30-50 mg•


30-60 m i n apart) each over 2 m i n
B o l u s admin istration No No Yes Yes
Allergic reaction Yes No No No
possible on repeat
exposure
TIMI flow grade 2/3b -55% - 7 5% -83% - 83%
Rate of intracerebral -0.4% -0.4-0.7% -0.8% -0.9%
hemorrhage
Fibri n specificity None +++ + ++++

STEMI = ST-elevation myocardial infarction.

'Based on body weight.

bTJMI flow grade 213 refers to mildly impaired flow through the coronary artery involved in the myocardial i nfarction. The higher the percentage of TIMI 213 flow, the more effective
the thrombolytic agent.

Adapted from Boden WE, Eagle K. Granger CB. Reperfusion strategies i n acute ST-segment elevation myocardial infarction: a comprehensive review of contemporary manage­
ment options. J Am Coll Cardiel. 2007;50( 1 0):91 7·929. IPMID: 1 77651 1 7]

22
Coro n a ry Artery D isease

f1FR be closely observed c l i n ically fo r symptom resol u t ion and rep­ agents include clopidogrel. t icagrelor. a n d prasugrel. Clopidogrel
W erfusion arrhythm ias. especially an accelerated iclioventricu l a r has been the mosl widely studied. and its use with concomitant
CONT.
rhythm (AIVR) : A f V R is consiclerecl a benign rhythm when it t h rombolytic therapy and primary PC! is associated w i t h
occurs within 24 hours of reperfusion. A repeat ECG should be i mproved outcomes a n d no apparent increase in the r i s k of
obtai ned 60 m i nutes a fter thrombolysis to determine i f bleeding. For patients for whom primary PC! for treatment of
ST-segment resolutio n h a s occurred . STEM! is planned, both ticagrelor and prasugrel demonstrated
Thrombolytic failure occurs in approximately 30% of' superior e fficacy when compared with clopidogre l ; however,
patients. and typically presents with fai l u re to fu l ly resolve very tew patients in these studies were t reated with t h rornbo­
chest pain or improve ST- segment elevation by 50%; some lytic therapy, and l i ttle evidence exists to recommend t he use of
patients may show hemoclynamic instabil ity or ventricular either ticagrelor or prasugrel in patients receiving thrombolytic
arrhyt h m ias. I n these circu mstances, rescue PC! is inclicatecl . t herapy. Dual antiplatelet therapy should be continued in STEM!
Rescue PCI is associated with i mproved cardiovascular out­ patients for a fu l l year, regardless of intervention or stent used;
comes when compared w i t h conservative medical t herapy in however, i f dual antiplatelet t herapy cannot be maintained for a
patients with fa ilure o f t hrombolytic therapy. Faci litated PC! is fu l l year (for example, because of bleeding, need for surgery. or
t h e administrat ion of fu l l - or h a l f-close t h rombolytic therapy problems with adherence) and the patient has a bare metal stent
(with or without a glycoprotein l l b/l l la i n hi bitor) followed by implanted. a minimum of 4 weeks of dual antiplatelet therapy
pla nned. immediate PCI. Adverse events (especi a l ly bleedi ng) is advised.
have l i m ited the safety and overa l l use of fac i l itated PCI . P l atelet glycoprotein l ib/Il la i n h i b itors (abciximab,
tirofiban. eptifibatide) further i nh i b i t platelet aggregation and
Medical Therapy impair platelet activation. They are useful i n patients with
At the t i me of initial presentation. a l l patients w i t h STEM I STE M ! who undergo prima1y PC] ; however. the use of glyco­
should be given a 325 - mg loading close of aspirin. supplemen­ protein l i b / I l l a inhibitors should be reserved for adm ir1istra­
tal oxygen . t herapy to improve symptoms (nitrates. analge­ tion in the catheterization laboratory rather t h a n u p- fro n t in
sics) , therapy to reduce i n farct size ( � - b l ockers, ACE t he emergency department owing to the i ncreased risk of
inh ibitors) . and an tit hrombotic therapy (anti platelet agen ts. bleeding and no clear benefit when administered prior to pri­
ant icoagu l a nts) . Aspirin should be administered immediately; mary PCI. Rou t i ne glycoprotein l l b / l l la i n h i bitor use i n
however, the admi nistration of ot her agents should not delay patients who receive thrombolytic treatment without P C I i s
t h e plan to reperfuse t he i n farct-related artery. controversial ancl n o t currently recom mended.
In many patients wi th STEM!, control of' chest pain can be The choice of anticoagu lant for treatment of STEM! is
achieved with sublingual or intravenous ni trates. Morphine and dependent on t he reperfusion st rategy ava ilable for the patient.
other opioid analgesics are also effective for reducing chest pain U n fractionated heparin (UFH) has been t horoughly studied i n
by decreasing the body's sympat hetic response to STEM ! . patients receiving t h rombolytic agents. a n d its use i s associ­
Caution should b e used i n patients with inferior STEM! and evi­ ated w i t h a reduced incidence of reocclusion of the i n farct­
dence of right ventricular infarction because nitrates and anal­ related artery. Low-molecu lar-weight heparin (LMWH) has
gesics can lead to reduced preload and significant hypotension. also been associated with i mproved outcomes in patients who
In the treatment of STEM ! , �-blockers are recommended receive thrnmbolytic t herapy. In patients undergoing primary
at the time of initial presentation except in patients with evi­ PCI. the use of U F H is favored over LMWH owing to the abil i ty
dence of heart fai l u re, hypotension. bradycardia, advanced to monitor t h e degree of anticoagu lation (by measurement of
atrioventricul a r block. or o t her contraindications to �­ activated clotting t i mes) . When a heparin -based strategy is
b lockers. I n t ravenous metoprolol is t he most widely used u t i l i zed for pri mary PC!. guidelines recommend the concomi­
� - b locker for STEM ! trea t men t : it is dosed in 5 -rng increments tant administration of a glycoprotein l i b / I l la inhibitor. Recent
every 5 minutes. for a total dose of 15 mg. Fol l owing reperfu­ studies of bivalirudin. a direct t h rombi.Ji i n h ibitor. have shown
sion, an oral �-b l ocker is recommended to reduce myocard ial that its use at the t i me of primary PCI is associated with a
oxygen demand and reduce mortality. similar rate of ischemic events (death, myocardial infarction.
ACE i11hibitors should be admin istered after reperfusion stroke. stent t h rombosis) and fewer b l eeding events when
in all patients w i t hout contraindications (systolic blood pres­ compared with a heparin p l us glycoprotein J i b / I l la i n h i b itor.
sure <90 mm Hg, advanced k i dney dysfu nc t i o n , hyper­ In general. t h erapies that reduce bleeding complications may
kalemia) . Angiotensin receptor blockers may be substituted i n improve survival but with concern for greater risk of nonfatal
patients w h o are a l l ergic or i n tolerant to ACE in h ibitors. ischemic events. such as early stent t h rombosis.
The use of antiplatelet agents in the setti11g of STEM ! has In patients with diabetes mell itus who present with
cha nged over the past decade with the ava i l ab i l i ty of several STEM ! , plasma glucose levels should be maintained below
new agents. Aspirin remains a mainstay in the treatment of ACS 1 80 mg/ c! L ( 1 0 . 0 mmol / L) w h i l e avoiding hypoglycem i a .
and should be administered to a l l patients unless a l lergic or I ntravenous insulin has been tested in m u l tiple trials, b u t
intolerant. Platelet P2Y receptor i n h i b i tors impair platelet n e i t h e r i n t ravenous insu l i n nor glucose- insul i n- potassium
12
aggregation, and t h is effect is additive to aspiri n. Available infusions are recommended curren t ly.

23
Co ro n a ry Artery Dise a se

CJ Complications of STEMI be needed to deter m i ne revers i b i l ity or benefit of perma­


The most com mon complications during the early manage­ nent pacemaker i mplantation.
CONT.
ment period a fter STE M ! a re a rrhyth m i as . heart fai l u re. a n d The seve r i ty of heart fai lu re (rangi ng from asympto­
vascu l a r acce s issues i n patients w h o undergo pri mary P C I . matic left ven t ricu l a r systolic dysfunct ion to ca rd i ogen ic
A s many a s 75°/r, of patients w i t h STEM ! have a n arrhy t h m i a shock) i s dependent on the extent of myocard ial i n fa rction.
d ur i ng hosp i t a l i za t i o n . i nc l u d i ng a t r i a l a n d ven t ri c u l a r severity of obstruct ive CA D. t i me from symptom onset to
a rrhy t h m ias. s i n u s bradycardia ( a fter i n ferior wa l l myo­ reperfusion, and pat ie n t-speci fic factors (age. comorbid
card ia l i n fa rc t io n ) , at rioventricular b lock, a n d sinus tach ­ cond it ions) . Pat ie n ts who develop cardiogenic shock a fter
yca rd i a ( a rter a nterior wa l l myoca rd i a l i n fa rc t i o n ) . STE M ! often have more extensive left ven t ricular i n farction
Approx i ma te ly 1 0 % to 1 5 % of patients have a t ri a l fibri l l a t i o n and an elevated i npatient mort a l i ty rate greater t ha n 6 0 % ,
o r flu t ter d ur i ng hospital izat ion. a n d t h i s is associated w i t h t h us prompting aggressive medical t herapy and hemody­
poorer long- term outcomes. T h e presence 0 1· a t r i a l fibri l la­ namic support w i t h i n t ra-aortic balloon cou n terpu lsation .
tion or heart block is often t ransient i n patients w i t h i n fe­ The i n i t iation of therapy to reduce preload (diuretics) and
rior wa l l myocardi a l i n fa rc t i on a nd suggestive o f more a fterloacl ( n i t rates, ACE i n h i b i tors) i s i n dicated i n all patients
extensive i n fa rct ion i n patients w i t h a n terior wa l l myoca r­ with symptoms o r heart fai lure : hovvever. caution is advised
d ia l i nfarct ion . Ven t ricu l a r arrhy t h m ias (ve n tricul a r tachy­ if systolic blood pressure is less t h a n 90 m m Hg or kid ney
card ia, ventricular fibri l lation) that occur i n the first 24 dysfunction exists.
hours a fter STE M ! diagnosis do not typica l ly a ffect progno­ Vascu l a r access com p l ica t ions i nclude hematoma.
sis. req u i re a n t ia rrhyt h m ic medications. or require defi bril ­ pseudoaneu rysm . a rteriovenous fistula, and retroperi tonea I
lator i mplanta t ion . The occurrence of recurre n t ven t ricular hemorrhage.
a rrhy t hmias later i n hospitalization is associated with a Mechanical complications i n STE M ! patients (Table 12)
larger i n farct a n d higher s hort- a n d long-term morb i d i ty a re m uch less frequent i n the reperfusion era . R ight ven­
a n d morta l i ty. I n patients w i t h persistent h igh -degree a t rio­ tricular i n fa rct i o n . which most com m o n ly results from
ven t ricular block or symptomatic bradycard ia. placement of occlusion of the proxi m a l right coronary artery. should be
a temporary t ranscutaneous o r t ra nsvenous pacemaker may considered in all patients w i t h i n ferior wall myocardi a l

TABLE 1 2. Potential Mechanical Complications of Myocardial Infarction


Complication Physical Examination Electrocardiography Echocardiography Pulmonary Artery
Findings Findings Findings Catheter Findings

Right ventricular Hypotension, j u g u l a r > 1 m m ST-segment Dilated right ventricle Elevated right atria l
i nfarction venous distention, clear elevation in leads V3 R with reduced systolic and right ventricular
lung fields and V4 R function pressures, low wedg e
pressure

Extensive left ventricular Systolic blood pressure Extensive ST-segment Severe left ventricular Cl <2.0 Umin/m2,
i nfarcti on <90 m m Hg elevation, usua l ly in systolic dysfunction wedge pressure
a nterior leads > 1 8 m m Hg

Ventricular septa I defect Holosysto l ic murmur Nonspecific; H ig h-velocity left-to- Pro m inent, large v
along left sternal border, a pproximately 50% right systol i c jet within waves in wedge
often with thri l l o f ventricular septal ventricular septum, pressure tracing; step-
defects occur in anterior systol i c turbulence on u p i n oxygen saturation
wall M l right ventricle s i d e of from right atri u m to
ventricular septum right ventricle

Pa pillary muscle rupture Holosystolic murmur at Usua l ly associated with Flail mitral valve leaflet Prominent, large v
left sternal border and inferior and inferior- with attached mass waves i n wedge
apex, may radiate to poste rior wall M l ( pa p i l l a ry m uscle pressure tracing
axil lae; pulmonary hea d ), severe mitral
edema regurg itation
Left ventricular free wall Hypotension, j u g u lar Nonspecific; pulseless Diffuse or localized Eq ualization of diastolic
ru pture venous distention, electrical a ctivity pericardia! effusion pressures, C l <2.0 U
distant heart sounds with tamponade; min/m2
d iscrete wall motion
a bnormality; defect in
myocard i u m may be
seen

Cl = cardiac index; M l = myocardial infarction.


'----- ----- ----- ------- --------'

24
Coron a ry Artery D isease

Cl leads to decreased p u l mo n a ry blood flovv and left atrial


i n farc t i o n a nd hypoten s ion . Right ven t ricular i n f'a rction echocardi ography is the most common di agnost ic modality.
and t h rombus is detected as an echo-dense stru c t u re, o ften
CONT.
ret u rn , decreased preload, and i 111pa i red ri l l i ng of' t he lel't a t t h e apex of' t he left ven t ricle (Figure 9 ) . Trea t m e n t
ven t ricle. This resul ts in the t riad of' hypotension . clear l u ng i nvolves t he u s e o f t herapeu t ic warfa r i n for 3 t o 6 m o n t h s
exa m i na t i o n , a n d elevated j ug u l a r venous p ressure. following myocard i a l i n fa rc t i on t o reduce t he r i s k o f' s t roke
D i agnosis is often made c l i n i cally and can be con fi rmed by or systemic embolization . Cl
e i t her ECG ( l eads v.1 R t hrough V 6 R ) or echocarcl iography
KEY POI NTS
(often used to exclude other causes of cardiogenic shock) .
• I n patients with ST-elevation myocardial infarction,
Treatment consists of reperfusion. aggressive volume resus­
when percutaneous coronary intervention cannot
citation. and t he use of i notropes (dopa m i ne or dobu ta m i ne)
be readily achieved within 120 minutes, thrombo­
u n t i l right ven t ricular fu nction i mproves (often 2 to 3 days
lytic therapy is recommended in the absence of
a fter myocardial i n fa rct ion) .
contraindications.
A ven t ric u l a r septa I d e fect is an i n frequent compl ica­
t io n of STEM I . It man i fes ts as hemodyna m ic compromise • Patients with ST-elevation myocardial infarction
i n t h e set t i ng o f' a new loud holosystolic murmur a n d o ften who receive thrombolytic therapy should be trans­
a palpable t h r i l l 3 to 7 days a fter t he i n i t i a l myoca rd i a l ferred to a percutaneous coronary intervention­
capable facility because o f the potential for throm­
i n fa rc t i o n . D iagnosis i s 111ost com m o n ly m a d e by transtho­
bolytic failure.
rac i c echocard i ogra p hy. Medical stab i l i za t i o n genera l ly
requ i res t h e ad m i n i s t ra t ion o f vasopressor age n t s a n d
p l acement o f a n i n t ra - aort i c bal loon p u 111p. A l t hough sur­

Cl
gical mortal i ty is h igh . i npatient 111ort a l i ty for p<l l ie n t s Non-ST-Elevation Acute Coronary Syndromes
who d o not undergo surgery i s nearly I O O'Y. . Percuta neous
. The 111ost common pat hophysiology of' NSTE-ACS is nonoc­
ven t ri c u l a r septa! defect closu re devices a re so111 e t i 111es clusive coronary atherosclerosis with or without t hrombus
u sed in nonsurgical patients. but t he i r use is l i m ited by formation . The t rearment o l' UA and NSTEM I patients is
a nato111y a n d operator expert i se. focused on improvement i n epicardial blood flow w i t h medi­
M i t ra ! regurgit a t i o n occurs co111 m o n ly a f'ler STE M ! . cations and revascu larization . Because the l i n k between revas­
M e ch a n isms i nclude severe lef'l ven t ricular dysfu n c t ion cuiarizat ion a nd clin ical outcomes is less clear than i n STE M !
with a n n u l us d i latation. worsening of pre-exist i ng 111 i t ral patient's. NSTE-ACS patients should undergo r i s k stra t i fication
regu rgi tati o n . a n d comprom ise o f t h e m i t ra l apparatus prior to invasive t rea l menl.
( ru p t u re o f pap i l l a ry 111uscle or chordae l e n d i neae) .
Papil lary muscle rupture o f'ten presen t s 3 to 7 clays a fter Risk Stratification
i n i t ia l myocardi a l i n fa rction with hemodyna111ic compro ­ The TI M I risk score is t he most com mo n ly used rool for
m i se. p u i 111onary edema . a n d a loud systolic 111 u r m u r. estimating t he s hort- term risk for dea t h a n d n o n fatal myo­
D iagnosis is 111ost of'len made by t ransthoracic echocarci iog­ card i a l i n fa rction i n pa t i ents w i t h a N STE-ACS (Tab l e 13) .
ra phy. a n d t ra nsesophagea l echocarci i ogra p hy 111ay be The T I M I risk score is most usefu l to assist in dec i d i ng
requ i red to plan surgical reconstruct ion . Trea t ment consists whether p a t i e n t s w i l l benefi t rrom a n early i nvasive
o f the a d m i n is t n.J t io n o f vasod i l ators to reduce a fterload and
d iuretics to decrease prel oad . If patients become hemociy­
namicaliy co111pro111 ised . t h e a d m i n i s t ration of vasopres­
sors. placement of an i n t ra -aort i c bal l oon pump. a n d /o r
s u rgical i nterven t io n a re req u i red .
Left ven t ricular free wall rupture is the most ominous
mechanica l co111plication of STEM I and has a h igh 111ortality
rate. It often occurs 3 to 7 days after i n i t ial 111yocarciial i n farc­
t ion. Risk factors !or left ven t ricular rupture i nclude advanced
age, fe111ale sex. anterior myocard ial infarction . and i nco111-
plete reperfusion of STEM ! . Pat ients most co111111only present
with pericard ia! tamponade (due to hemopericard i u m) . pulse­
less electrical activity. and death. Early recogni t ion. emergent
pericardiocen tesis. and subsequent surgical reconstruction
can improve survival.
Left ventricular t h rombus occurs in approximately L O 'Y..
lo 20% of pa t i en ts a fter a n terior 111yoca rcl ic1 I i n fa rct ion F I G U R E 9 . Echoca rd iographic image showing a left ventricular thrombus in a
despite reperfusion a n d aggressive t rea t m e n t . Transt horacic patient with a recent myocardial infa rction. LV = left ventricle; RV = right ventricle.

25
Coro n a ry Artery Disease

TABLE 1 3. TIMI Risk Score for Non-ST-Elevation Acute D u a l antiplatelet t herapy (aspirin p l u s clopidogrel, pras­
Coronary Syndromes ugrel. or t icagrelor) is recom mended in all patients w i t h
Prognostic Variables NSTE-ACS. regardless of T I M I risk score. u n less an increased
risk or bleedi ng exists (see Table 9) . The use of clopidogrel. in
( 1 ) Age �65 years addition to aspi rin, is t h e best-studied combination .
(2) �3 Trad itional CAD risk factors• Clopidogrel should be given as a loadi ng dose (300 mg or
(3) Documented CAD with �50% d i a meter stenosis 600 mg) at hospi tal adm ission and ad m i nistered as a 75-mg
(4) ST-segment deviation daily dose for at least l year regard less of t he need for PC! or
CA BG. Pat ients with a bare metal sLent who cannot tolerate
(5) �2 Angina! episodes in the past 24 hours
dual ant iplatelet t herapy for t h e fu l l year ( for example.
(6) Aspirin use in the past week
because of bleeding. need for su rgery. or problems w i t h
(7) Elevated cardiac biomarkers ( creatine kinase MB or troponin)
adherence) should remain on the t herapy for at least 4 weeks.
TIMI Risk Score (Sum of Prognostic Variables) l f CABG is u l t i mately required. clopiclogrel should be d iscon ­
tinued and CABG should be postponed for 5 to 7 clays in order
0-2 Low risk
to avoid perioperative bleedi ng.
3-4 Intermed iate risk
Two oral P2Y 12 i n h ib i tors . prasugrel and t icagrelor. have
5-7 High risk been developed, a n d when tested . were superior to clopi ­
CAD = coronary artery disease. clogrel in UA and NSTEl'vl l pat ients. Ticagrelor and prnsugrel
.iHypertension, hypercholesterolemia, diabetes mellitus, being a current smoker, do not require hepati c m etabolism. are more poten t . and
family history of CAD.
have a faster onset of action when compared with clop i ­
Adapted from Antman EM, Cohen M, Bernink PJ, et al. The TIMI risk score for dogrel. These agents also should be discont inued 5 t o 7 days
unstable angina/non-ST elevation Ml: a method for prognostication and therapeu-
tic decision making. JAMA. 2000;284(7):835-842. (PMID: 1 0938 1 721 or more prior to CABG.
Ad m i n istration of glycoprotein l ib/ I I la i n h i b i tors in
patients with NSTE-ACS does not appear to be of net clin ical
benefi t un less h igh-risk features. such as ongoing angina or
l'PI t rea t rnenl strategy (Figure 10) . The est imated rates of death evidence of ischemia after the i n i t iation of sta nda rel an ti plate­
LI.I
CONT.
and non fa Lal myocardial i n farction also a re useful lo cou nsel let and antianginal medications. rein farction . or heart failure.
palients rega rd ing their risk. ln patients at low risk (Tl M I a re present. However. L hese agents, in combinalion wi l h U FI-1
score o f 0 - 2 ) , practice guide l i nes recom mend a n ischem i a­ or bivalirudin, are indicated at the time of PC! in pat ients with
gu idecl st rategy t ha t u t i l izes invasive t reatment only i f medi­ NSTE-ACS who u l t imately require revascularizal ion . Because
cal t herapy is ineffec t ive. Patients a t h igher risk (TI M I score of t heir potent antiplatelet activity, t he main adverse effe ct of'
�3) are more l i kely to benefi t f'rom an early i nvasive approach. glycoprotein l l b/l l la i n h ibitors is i ncreased risk of major a nd
m i nor bleeding events.
Medical Therapy
A l l patients who present with ischemic chest pain should be A 11 t icoagu la n t Medica t i o n s
t reated i ni t i a l ly w i t h asp i r i n , �-blockers, and n i t rates. The use of ant icoagulant medications ( U FI-1. LMWH . foncla­
However. compared w i t h STE M ! . in which reperfusion is the parinux, and bivalirudin) has been a cornerstone of t herapy
primary goal of therapy, once the diagnosis of UA or NSTEM I for NSTE-ACSs for more than t h ree decades. The choice of a
has been established ( through the ECG and biomarkers). risk panicular agent is based on t he patient"s bleeding risk. TI M I
stra t i fication can be used to guide t he clinical use of additional risk score. comorbid conditions (such as chronic kidney d is­
t herapies (Table 14 . on pages 28-29) . All NSTE-ACS patients ease ) . plan for an early invasive versus a conservat ive st rategy.
should receive a stati n and a P2Y 12 i n h ibitor (such as clopi­ timing of coronary angiography. and physician preference.
dogrel ) . I n patients at i n termediate or h igh risk (TI M I score U F H and L M W H are the most widely used anticoagu ­
�3) . additional t herapies. such as anticoagu lant agents or a lants for NSTE-ACSs. In patients in whom an early i nvasive
glycoprotein l ib/ I l la inhibitor, should be considered. approach is planned and i n patients vvi t h chronic kid ney
d isease. U FH is preferred over L M W H . I n patients in whom a
A n t ipla te let Medica t io n s conservative strategy is p la n ned . bot h L M W H and fonda­
The i n itial aspirin dose should be 325 mg at t h e t i me of pres­ parinux have been proved w be safe and effect ive. Advantages
entat ion for ischem ic chest pain . Patients who are al lergic to of fondapari nux and LMWI-1 i nclude the abil ity to close once
aspirin should be administered clopidogrel at the time of pres­ or twice daily rather than con t i nuously and no req u i rement
entation. Alt hough t here remains debate about subsequent to monitor t herapeutic levels. Because of a sign i fi c a n t ly
aspirin closing based on patient risk and whether revasculari­ i ncreased bleedi ng risk. the use of the a n ticoagu lant bivaliru­
zation with PC! or CABG occurs. most patients can be treated din is curre n t ly n o t recom mended by clinical guidelines
with a dose of 81 mg daily inde fi n i tely (especially when dual other than d u ri ng PC! or in patients who a re allergic to
antiplatelet t herapy is being used) . hepari n-based produ cts.

26
Coro n a ry Artery Disease

N on-ST-elevation acute
coronary syndrome
(UA/NST E M I )

1
I n iti ate aspirin, B-blocker, n itrates, stati n

R i s k stratificati on (TI M I risk score)

H i g h risk I ntermed iate risk Low risk


(TI M I 5-7) (TI M I 3-4) (TI M I 0-2)

• Anti platelet therapy: • Anti platelet therapy:


Ticagrelor or clopidog rel•,b Ticag rel or or clopidog rel•.b
or • Anticoag u l a nt therapy:
GP l i b/I lla i n h i b itor U F H, LM WH, or fondaparinux
• Anticoa g u l a nt therapy: • For patie nts with continued
U F H or LMWH; alternatively, sympto ms despite i n itial
fondaparinux medica l thera py, consider G P
• For patients with continued l i b/I l l a in hi bitor a n d emergent
symptoms despite i n itial coronary ang i og raphy
medical thera py, consider G P
I l b/I l la i n h i bitor a nd emergent
Only if recurrent symptoms,
coronary ang iog raphy a b norm a l stress test, or LVE F <40%

I I
... '

D i ag nostic coronary a n g iography<

F I G U R E 1 0 . I n itial management of n on-ST-elevation acute coronary syndromes. G P = glycoprotei n ; LMWH = low-molecular-weight heparin; LVEF = left ventricular
ejection fraction; NSTEMI = non-ST-elevation myocard ial infarction; UA = unstable angi na; U FH = unfractionated heparin.
aclopidogrel o r ticagrelor m a y b e dosed at t h e time of hospital admission a n d acute coronary syndrome diagnosis.

'If coronary artery bypass grafting is required, clopidogrel or ticagrelor should be stopped and surgery delayed for at least 5 days.

elf the decision is made to withhold a P2Y1 2 inhibitor until time of angiography and a P2Y 12 inhibitor is desired, clopidogrel, ticagrelor, or prasugrel can be initiated.

Recommendations based on Amsterdam EA, Wenger NK, Brindis RG, et al; ACC/AHA lask Force Members. 2014 AHAJACC guideline for the management of patients with non-ST-elevation acute coronary syndromes: executive summary: a
report of the American College al Cardiology/American Heart Association Task force on Practice Guidelines. Circulation. 2014 Dec 23;1 30(25):2354-94. IPMID: 25249586]

CJ A n tia ngi n a / Med ications t ions. I n travenous n i t rates are ty pic a l ly used in patients with
Un less con tra i nd i cated. o ra l � - b l o cke rs should be i n i tiated i n ongoing chest pai n or borderline blood pressure or hemody­
CONT.
a l l patients who present w i t h a N STE-ACS . I n travenous narn ics. In patients without act ive symptoms. topical and oral
P-blockers can also be adm i n istered; however, caution should formulations should be prescribed in order to prevent recu r­
be used in patients with heart fa i l u re . advanced age. advanced rent symptoms. Subl i ngual n i t roglyce rin is often p res c ri b e d on
atrioventricular block, and hypotension . an as- needed basis for new or worse n i ng angina_ All forms of
N i t rates that can be a d m i nistered to NSTE-ACS patients ni t rntes should be avoided in patients who have t a ke n a PDE- 5
include i n t rave n o u s . topica l . o ra l , and sublingual formula- i n h ibitor (such as silclen<Jfi l or varclenaf11) w i t h i n 24 hours.

27
Coro n a ry Artery Disease

TABLE 1 4. Long-Term Medical Therapy for Coronary Artery Disease


Medication Drugs in This Class Dosage Indication• Comment
Antiplatelet Medications

Aspirin N/A 7 5 - 1 62 mg/d All patients, u n less


intolerant or a l l ergic

Clopidogrel N/A 75 mg/d P2Y1 i n h i bitor in Clopidogrel is


2
com b i n ation with aspiri n is reco m mended as an
i n d icated i n all patients altern ative for patients
after ACS or PCI with i ntolerance or
a l l ergy to aspirinb

Prasugrel N/A 1 0 mg/d P2Y1 2 i n h i b itor i n Contra indicated i n


combination with aspirin is patients > 7 5 years of
ind icated in a l l patients age, low body weight
after ACS o r PCI (<60 kg [ 1 32 lb]), or
history of stroke!TIA

Ticagrelor N/A 90 mg/d P2Y1 2 i n h i b itor i n More rapid onset of


co mbination with aspirin is action; does not require
indicated in a l l patients first-pass he patic
after ACS or PCI meta bolism; no known
genetic polymorphisms.
Decreased effectiveness
with aspirin dosages
�1 00 mg

'
Cardioprotective Medications
�- B l ockers Atenolol, metoprolol, Vari a b l e All patients with prior M l or Caution is advised i n
carvedilol, nebivolol LV systolic dysfunction patients with significant
obstructive lung disease
(e.g., COPD) or advanced
atrioventricu lar block

ACE i n h i bitors Benazepril, captopri l , Va riable All patients with LV systo lic
enal a pril, fosino pril, dysfu ncti on, hypertension,
peri ndopril, tra ndola pril, dia betes, or proteinuric
l i sinopril, ra mipril, chronic kid ney disease
q u i n a pril

Angiotensin receptor Losartan, valsartan, Varia b l e ACE i n h i b itor-intolerant The use of ang iotensin
blockers o l mesartan, candesartan, patients receptor blockers in
irbesartan, tel m isartan combination with ACE
i n h i b itors is not well
esta b l ished (class l i b
recommendation)
Aldosterone blockade Spironolactone, Varia ble ACS patients with LV Caution is advised i n
eplerenone systo l i c dysfu nction on patients with sign ificant
therapeutic doses of kidney dysfu nction or
�-bl ockers and ACE hyperka lemia
i n h i b itors

H igh-i ntensity statin Atorvastatin 40-80 mg/d For a l l patients with


therapy evidence of corona ry
Rosuvastatin 20-40 mg/d
artery d i sease and age
�75 years
Moderate- intensity Atorvastati n 1 0-20 mg/d For a l l patients with
statin therapy evidence of coronary
Rosuva stati n 5-1 0 mg/d
artery d isease a nd age
Si mvastatin 20-40 mg/d > 75 years or otherwise
i nto lerant of h ig h-i ntensity
Prava stati n 40-80 mg/d statin therapy
Lovastatin 40 mg/d
Fl uvastatin 40 mg B I D

(Continued on the next page)

28
Coro n a ry Artery Disease

TABLE 1 4. Long-Term Medical Therapy for Coronary Artery Disease (Continued)


Medication Drugs in This Class Dosage Indication• Comment
Antianginal Medications

P-blockers Atenolol, metoprolol, Va riable All patients with prior M l or Caution is advised i n
carved ilol, nebivo l o l LV systolic dysfunction patients with s i g n ificant
obstructive l ung
d i sease o r advanced
atrioventricular b lock
Long-acting nitrates lsosorbide, transdermal Va riable Useful for rel i ef of N itrates are
patch, ointment symptoms when maximal, contraindi cated i n
tolerated dose of patients taking PDE-5
P-blocker has been i n h i bitors for erectile
achievedb dysfu nction
Short-acting nitrates S u bling ual, spray, o r Variable Useful for relief of N itrates are
aerosol symptoms, usually contra indicated i n
prescribed on a n patients ta king PDE-5
as-needed basis i n h i b itors for erectile
dysfunction
Ca lcium channel Amlod i pine, d i ltiazem, Va riable Useful for relief of
blockers vera pa m i l , nifed i pine, symptoms when maximal,
nicard i pine tolerated dose of
P-blocker has been
ach ievedb

Ranolazi ne N/A 500- 1 000 mg B I D Useful for rel i ef of Caution is advised i n


symptoms when maximal, patients with advanced
tolerated dose of kid ney disease, liver
P-blocker has been d i sease, and i n those
achievedc ta king potent i n h ibitors
of CYP3A4 pathway

ACS = acute coronary syndrome; BID = twice daily; LV = left ventricular; M l = myocardial infarction; N/A = not applicable; PCI = percutaneous coronary intervention;
PDE-5 = phosphodiesterase S; TIA = transient ischemic attack.

"Level of evidence: All are class I, level A, except as noted.

blevel of evidence: class l, level B.

'Level of evidence: class Ila, level B.

l"l"'I Calcium channel blockers and ranolazine do not have considered for early invasive treat ment. Patients treated w i t h
LI.I clear i nd ications for use i n pat ients presenting with an ACS. an ischemia-gu i ded strategy should undergo noninvasive
CONT.
Opioids, such as morph i ne. may be administered for sympto- stress testing before hospital discharge; coronary angiography
matic relief o f pat ients w i t h ongoing chest pai n. although cau­ and subsequent PCI or CABG should be reserved for t hose
tion is advised because of their blood pressure-loweri ng with recurrent symptoms or h igh-risk features on stress test­
effects. i ng. H igh - risk patients assigned to an early i nvasive strategy
(coronary angiography and subsequent PCI or CABG within
L ipid-Lowering Medica tions 24 hours of i n i t ial hospitalization) have been shown to have
The benefits of statin therapy i n ACS patients a re wel l estab­ i mproved outcomes when compared w i t h high -risk patients
l ished; however, the timing or i nitiation (hospital adm ission treated conservatively. For patients who are appropriate for an
versus at the t i me of· PCl versus hospital d ischarge) is less clear. early i nvasive strategy. t here is no evidence t hat very early
Recent studies suggest t hat h igh-intensity dosing of atorvasta­ angiography (<6 hours or at hospital admission) offers incre­
l" i n or rosuvastat i n is associated with i mproved 2-year survival mental benefit.
after ACS. Current treatment guideli nes recom mend the i n i­ Clinical guideli nes recom mend the use of early i nvasive
t ia tion of a h igh- i ntensity statin in patients at very high car­ treatment i n NSTE-ACS patients regardless or risk stra t i fica­
d iovascular risk, i ncluding t hose patients w i t h an ACS. tion who have recurrent symptoms. hemodynamic i nstability,
or elect rical i nstabi l i ty.
Invasive Versus Ischemia-Guided Treatment CABG is often recommended tor ACS patients w i t h spe­
Once medical t herapy has been i n i t iated and risk stra t i ficat ion c i fic angiographic criteria ( that is. le!'t main disease and multi­
has occurred, guideli nes recom mend that pa tients with a low­ vessel CAD w i t h or w i t hout proximal left anterior descending
risk N STE-ACS be treated conservatively with an ischemia­ stenosis) with concomitant lert ventricular systolic dysfu nc­
guided strategy and that intermedi ate- to h igh- risk patients be tion and /or d iabetes mel l i tus. Cl

29
Coro n a ry Artery Disease

K EY P O I N T S and rhabdomyolysis) a nd cardiac condi t ions (such as heart


fa i lu re. a t rial fib ri l la t ion . and myocard i t is) can lead to a d iag­
• All patients who present with ischemic chest pain should
nosis or ACS w i t hout obstruct ive coronary a rtery stenosis on
be treated initially with aspirin, �-blockers, and nitrates;
coronary angiography. I n most cases. t he e t iology of ACS i s
once the diagnosis of a non-ST-elevation acute coronary
presumed to b e demand ischemia (myocard ial oxygen demand
syndrome has been established, risk stratification can be
surpasses oxygen supply) : il i s termed secondary myocarcl i<ll
used to guide the clinical use of additional therapies.
i n fa rc t ion when card i a c biorna rkers a re abnorm<1 l .
• All patients with a non-ST-elevation acute coronary Ma nagement of t hese pat ients typically i ncludes l reatment o r
syndrome should receive a statin and dual antiplatelet
the underlying condi t ion in addition t o medical t herapy (aspi­
therapy with aspirin and a P2Y12 inhibitor; in patients at ri n . sla t i n) and preven tive measures for heart disease. Cl
intermediate or high risk, additional therapies (antico­
agulant agents, glycoprotein Ilb/IIla inhibitor) should Long-Term Care After an Acute
be considered. Coronary Syndrome
• Dual antiplatelet therapy is recommended for at least All patients who are diagnosed with an ACS should continue
1 year in all patients with a non-ST-elevation acute cor­ aspirin (81 mg/d) indefinitely. Patients treated medically with­
onary syndrome, unless an increased risk of bleeding out PC! should receive an oral P2Y12 inhibitor for up to 12
exists. months. The optimal duration of dual antiplatelet therapy in
• Once medical therapy has been initiated and risk strati­ patients with an ACS who undergo PC! with drug-eluting stent
fication has occurred, guidelines recommend that low­ implantation is at least 12 months (see Table 9) . In patients
risk patients with a non-ST-elevation acute coronary who undergo PC! with bare metal stent implantation, the
syndrome be treated conservatively and that intermediate­ duration of dual antiplatelet therapy is at least 4 weeks but up
to high-risk patients be considered for early invasive to 12 months as tolerated. In the absence of contraindications,
treatment. ACE inhibitors and statins should be continued indefinitely in
all patients with an ACS. There is evidence of benefit for treat­

c:J Acute Coronary Syndromes Not Associated ment with a �-blocker for at least 3 years following an ACS,
although many clinicians choose to continue these medica­
with Obstructive Coronary Disease
tions indefinitely if they are well tolerated.
A l though plaque rupture al the s i te or al herosclerolic p laque Current guidelines recommend that patients who are
depos i t ion is t he most com mon cause or ACS. ot her d isease entering cardiac rehabilitation programs after ACS should
e n t i t ies may cause patients to presen t w i t h chest pa i n . tran­ undergo routine exercise ECG testing. Routine stress testing is
sient ECG changes. and elevated cardiac biomarkers. The diag­ not currently recommended for asymptomatic patients who
nosis of t hese d i seases is genera l ly made once diagnos t ic are not entering a cardiac rehabilitation program. For patients
coronary a ngiography has confirmed t he absence or obstruc­ with an ACS who are symptomatic after hospital discharge, the
t ive coronary stenoses.
use of stress testing is acceptable and should be determined by
Coronary \·asospasm of'ten occurs in normal or nea r­
the patient's symptoms (for example, unstable angina, stable
normal coronary arteries. and spasm can be t riggered by the angina) , ability to exercise, and interpretability of ECG.
use of i l l icit drugs. such as coca ine or met hampheta rn i ne. In
patients who presenr with ischern ic chest pa i n at rest and KEY POINT
t ransient ST-segment elevat ion or depression. t he d iagnosis of • Routine stress testing is not currently recommended for HVC
vasospasm is often one of exclusion once angiography con ­ asymptomatic patients following an acute coronary
firms t h e absence or obst ruc t ive CAD. Vasospasm is most fre­ syndrome who are not entering a cardiac rehabilitation
quen t ly t reated w i t h long-term n i t rates and calcium channel program.
b lockers and avoidance of t riggers such as i l l ic i t drugs.
Takotsubo cardiomyopathy. onen temporally associated
w i t h a stressful event. can m i m i c an ACS with t he presence of M a n a g ement of Coronary
chest pa i n . ECG changes. and ele·ated cardiac biomarkers (see
Artery Disease i n Wo men
Heart Fai lure) . H owever. co ro na ry angiography is usu a l ly nor­
mal or shows o n ly m i n i mal a therosclero l ic d isease. and left Clinical Presentation
ven tricu lography is classical ly defined by t he presence of mid­ Women typically develop ischemic heart disease at a later age
wa ll <1l1d apical wa l l motion abnorma l i t ies w i t h sparing of the in life than men. Unlike men, women present more frequently
basal segments. Treat ment is support ive. and more than 95% with stable angina pectoris rather than an ACS or sudden car­
of pat ients have resol u tion of symptoms and recovery of l e ft diac death. More than 50% of women who present with typical
ventricular function within 7 days. angina are diagnosed with nonobstructive coronary stenoses,
Owi ng to the i mproved sensitivity of troponin assays. and the presence of microvascular disease is thought to be
other systemic d iseases (such as chronic kidney d isease. sepsis. significantly higher in women than in men.

30
Coro n a ry Artery Disease
------- ----- ---·------

In women presenting wilh acute myocardial infarction. significantly more frequently in persons with diabetes com­
chest pain remains the predom inant symptom; however. the pared with those without the disease. The diagnostic accuracy
Likelihood of atypical symptoms . such as fatigue, dyspnea. of noninvasive stress testing in symptomatic patients with dia­
nausea. and abdominal complai nts. is significantly h igher betes is similar to that in patients without diabetes; however, the
than in men . In women wi thout sign i ficant obstruction on assessment of CAD in asymptomatic persons with diabetes is
invasive angiography. microvascular dysfunction (either controversial. Currently, the American Heart Association rec­
endothelium-dependent or endothelium-independent) is ommends stress testing in patients with diabetes who are
t hought to be the cause of symptoms. Cl (1) symptomatic, (2) initiating an exercise program, or (3) known
to have CAD and have not had a recent (>2 years) stress test.
Evaluation and Treatment

l:::J
The sensitivity and specificity of noninvasive stress testing for I nvasive Treatment
the evaluation of chest pain are lower in women than in men. Jn patients with dh:1betes who are candidates for revasculariza-
ST-segment deviation is less accurate in women than in men. tion. the decision to pursue PC! or CABG remains controversial
Clinical guidelines report an improved diagnostic test accu­ and depends on a mul titude of factors, i ncluding severity and
racy with the use of stress testing with imaging (see Diagnostic extent of CAD. the presence of comorbid conditions, and the
Testing in Cardiology) . However, no specific diagnostic evalu­ degree of atherosclerotic narrowing of' smal l . distal vessels.
ation guidelines exist for women, and the same guidelines Multiple studies have analyzed outcomes of patients with diabe-
apply to men and women. tes undergoing PC! or CABG. Although CABG is general ly asso­
In the COURAGE trial, women with stable angina pectoris ciated with fewer repeat revascularization procedures, mortality
had reduced overall mortality or nonfatal myocardial infarction is similar between the t\.vo procedures. When a decision is made
with revascularization therapy as compared with men. However, to pursue PC!. the use of a drug-eluting stent is recommended
women have a higher complication rate, particularly bleeding and to reduce the occurrence of target vessel revascularization
vascular complications, surrounding revascularization proce­ because of the more extensive coronary a11ery disease and
dures. For these reasons, it is recommended that guideline­ higher rate of' restenosis in patients with diabetes. CJ
directed medical therapy be initiated in women prior to
consideration for revascularization. In women undergoing PCI, a
Medical Therapy and Secondary Prevention
trend toward fewer such complications has been found with the
Aggressive risk factor reduction, control of plasma glucose
use of radial rather than femoral arterial access. Overall, treatment
levels, and medical therapy are essential in patients with dia­
guidelines do not currently differ between men and women. The
betes. In most patients with diabetes and CAD, high-intensity
use of estrogen is not recommended to reduce the occurrence of
statin therapy and antihypertensive treatment with a target
future cardiovascular events in post-menopausal women.
blood pressure below 140 /90 mm Hg are recommended.
K EY P O I NTS Because of the protective renal effects of ACE inhibitors and
• In women presenting with acute myocardial infarction, angiotensin receptor blockers in patients with proteinuric
chest pain remains the predominant symptom; however, nephropathy, these agents are preferred over other antihyper­
atypical symptoms, such as fatigue, dyspnea, nausea, tensive agents, such as thiazide diuretics.
and abdominal complaints, are more likely than in men. Common medications used for the treatment of diabetes
are of special concern in patients with CAD. In a meta-analysis,
• In women with coronary artery disease, it is recom­
thiazolidinediones, specifically rosiglitazone, had been asso­
mended that guideline-directed medical therapy be
ciated with an elevated risk of cardiovascular events, espe­
initiated prior to consideration for revascularization;
cially myocardial ischemia. However, a more recent clinical
otherwise, treatment guidelines do not currently differ
trial demonstrated no elevated risk of myocardial infarction
between men and women.
or death in patients being treated with rosiglitazone when
compared with standard-of-care diabetes drugs. The use of
M a n ag ement of Coro n a ry metformin at the time of coronary angiography, after myo­
cardial infarction, and in patients with heart failure should
Artery Disease i n Patients
be avoided because of a rare but potentially fatal risk of lactic
with Diabetes M e l l itus acidosis.
Risk and Evaluation KEY POINT
Patients with diabetes mellitus are at increased risk of develop­
• In patients with diabetes, stress testing is recommended
ing CAD, and cardiovascular mortality is significantly higher in
by the American Heart Association for those who are
this population. Additionally, patients with diabetes mellitus
(1) symptomatic, (2) initiating an exercise program, or
often do not experience classic angina pectoris and can present
(3) known to have coronary artery disease and have not
with atypical cardiac symptoms, such as dyspnea, nausea, or
had a recent (>2 years) stress test.
hyperglycemic symptoms. Sudden cardiac death occurs

31
H e a rt Fa i l ure

H ea rt Fai l u re there i s about a 5 0 % survival rate at 3 years after presenting


with symptoms of heart failure.
Classic symptoms of acute heart failure include exertional
Pathophysiol ogy of Heart Fa i l u re
dyspnea, paroxysmal nocturnal dyspnea, orthopnea, and
Heart failure is a complex clinical syndrome in which cardiac
peripheral edema. In patients with stable chronic heart failure,
output is insufficient for meeting the demands of the body,
symptoms are typically similar to those of new-onset heart
causing symptoms of exertional dyspnea and fatigue.
failure but less intense. A patient may normally sleep on two
Approximately one half of patients with heart failure have left
or three pillows owing to nocturnal dyspnea, but with decom­
ventricular systolic dysfunction, or heart failure with reduced pensation will sleep on more pillows or move to sleeping in a
ejection fraction (HFrEF); the other half have normal systolic recliner. Similarly, a patient at baseline may become short of
function, or heart failure with preserved ejection fraction breath walking up one flight of stairs, but with decompensa­
(HFpEF) . Regardless of whether left ventricular ejection frac­ tion may have dyspnea putting on his or her clothes. Patients
tion is reduced or preserved, symptoms of exertional dyspnea with previously diagnosed heart failure often do not present
and fatigue are the same, and assessment of ejection fraction with classic findings. These patients may present with
is required to differentiate between these two entities. increased abdominal girth rather than peripheral edema, or
Common causes of HFrEF include hypertension, coro­ with nausea and anorexia caused by gut edema rather than
nary artery disease, myocarditis, certain drugs (for example, with exertional dyspnea. Educating patients to weigh them­
doxorubicin, trastuzumab, cyclophosphamide) , and toxins, selves daily and report changes in weight or baseline symp­
including alcohol, cocaine, amphetamines, cobalt, and lead. toms may reduce heart failure hospital admissions if acted on
Systemic diseases that may cause HFrEF include hypo- and quickly enough.
hyperthyroidism, HIV infection, systemic lupus erythemato­ The first step in the evaluation ofa patient with signs and
sus, scleroderma, and neuromuscular diseases such as symptoms of heart failure is a thorough history and physical
Duchenne and Becker muscular dystrophy. A high percentage examination, which should be performed to evaluate for pos­
of HFrEF cases are idiopathic. The most common causes of sible causes of heart failure and assess cardiovascular risk
HFpEF are hypertension and coronary artery disease; less factors. A detailed history, including alcohol and illicit drug
common causes include infiltrative diseases such as amyloido­ use, alternative therapies, family history, and any history of
chemotherapy, should be obtained. Features that increase the
sis and hemochromatosis. In general, patients with HFpEF
likelihood of heart failure include the presence of paroxysmal
tend to be older, heavier women who have a history of hyper­
nocturnal dyspnea (>2-fold likelihood) and the presence of an
tension, coronary disease, and diabetes mellitus.
S 3 (11 times greater likelihood) . The likelihood of heart failure
Although the symptoms of exertional dyspnea and fatigue
is decreased 50% by the absence of dyspnea on exertion and
and hemodynamic abnormalities of reduced stroke volume
by the absence of crackles on pulmonary auscultation.
and elevated ventricular filling pressures are similar between
Elevated jugular venous pressure and an S3 are independently
HFpEF and HFrEF, there are significant differences in the
associated with adverse outcomes, including progression of
pathophysiology between the two disease processes. In
heart failure.
patients with HFrEF, the common defect is an abnormality of

Cl
myocardial contraction. Reduced systolic function results in Diagnosis
progressive ventricular dilation. In contrast, patients with I n p at ie n ts who presen t with acute dyspnea of undetermined
HFpEF have similar symptoms but normal systolic contraction e ti ol ogy B-type natri uret i c peptide (BNP) levels can be used to
.

and an abnormality in diastolic relaxation. This results in quickly d ifferentiate between dyspnea se con da ry to heart fai l ­
restricted filling and high filling pressures. To maintain a nor­ u re (elevated B N P) a n d dysp nea related to pulmonary disease
mal cardiac output (heart rate x stroke volume), patients with ( low to normal B N P) . The B rea th i ng Not Properly stud y evalu­
HFpEF tend to have a higher heart rate. Clinically, because ated patients who presented to t he e m ergen cy de p a r t m e n t
these patients have a very small left ventricular size, they are with dyspnea. Patients who had heart fai l u re had a mean BNP
usually much more sensitive to volume loading than patients level greater than 600 pg/ml (600 ng/ L) whereas those with
,

with HFrEF. noncardiac causes of dysp n e a had levels o f approximately


50 pg/ m l (50 ng/ L) . Pa tie n ts with a h ist ory of left ven t ricular
dysfu nc t ion but not an acute exacerbation had a B N P level of
Diagnosis and Eva l uation approximate ly 200 pg/m l (200 ng/L) . B N P levels increase
of Heart Fa i l u re with age and wo rse n i ng ld d n ey function and are reduced in
patients with an elevated BM!.
Clinical Evaluation B N P leve l s alone are not d iagnostic for heart fai l u re but
Approximately half of all heart failure hospital admissions should be used o n ly as a n i n i tial test to guide t he diagnostic
result from HFpEF. There is no difference in mortality between eva luation in patients with dyspnea o f u ncertain etiology.
patients with HFpEF and HFrEF. For both groups of patients, B N P l eve ls should not be used to fol low a p a tien t s clin ical
'

32
Heart Fa i l u re

Cl hospi talization to determ ine i f diuresis has been adequate.


course. There is no benefit to fol lowing BNP levels d u ri ng a vent ricle. Also, a combination of findi ngs on echocardiogra­
phy may provide a clue to the cause of heart fai l ure. For exam­
CONT.
Additional ly, rout i n e use of B N P measurement in outpatients ple, left ven tricular hypertrophy and bi-atrial enlargement in
with heart fai l u re is not helpfu l for determ i ning i f a patient is a patient with reduced ejection f'raction suggests a restrictive
fluid overloaded. card iomyopat hy. The left ven tricular hypertrophy of H FpEF is
A 1 2-lead electrocardiogram ( ECG) should be obtained for usually secondary to hypertension and should not be con fused
all patients with heart fai lure. A n ECG can be helpfu l to evalu­ with the severe hypertrophy characteristic of hypertrophic
ate for possible myocardial i n farction, tachyarrhythmia. or left cardiomyopathy (see Myoca rdial Disease) . Pat ients with
ven t ricular hypertrophy. A chest radiograph should be H FpEF often have m i ld to moderate left ven tricular hypertro­
obtai ned to evaluate for concomitant pulmonary disease. phy (<15 mm in a ny region) .
Chest radiography can also be helpfu l for ident i fy i ng vascular Cardiac magnetic resonance (CM R) i maging is used
congestion indicati ng volume overload or pleural effusions. increasingly in the evaluation of' patients with heart fai lure.
The i n i tial laboratory assessment of heart fai lure should C M R imaging can be used to assess wall motion abnormalities.
i nclude electrolyte levels, urinalysis and kid ney function. global wall function. and viabi lity. Additionally. i t can be used lo
glucose and lipid levels. l iver chemistry tests, a nd t hyroid­ assess t issue perfusion, tissue injury (inflammation or necro­
stimulating hormone levels. A l though t hyroid disease is an sis) , fibrosis, infiltration (sarcoid or a my lo id). or iron deposition .
uncommon cause of heart failure, hypothyroidism and hyper­ Endomyocardial biopsy is rarely indicated for the evalua­
thyroidism are potentially reversible causes of heart failure t ion of acute heart failure. Pat ients with progressive heart
with appropriate trea t ment. Coronary disease causes approxi­ failure on medical t herapy who have malignan t arrhythmias
mately two t h i rds of cases of heart failure. and an acute coro­ should undergo biopsy to evaluate for gian t cel l myocarditis.
nary syndrome should be suspected as a cause of new heart Biopsy is also re<1sonable for patients with new-onset heart
fai lure or an exacerbating factor in pat ients with preexist i ng ra i l u re u nresponsive to standard medica l t herapy.
Endomyocardial biopsy can assist i n the d iagnosis of amyloi­
heart fai lure. I f an acute coronary syndrome is suspected as
dosis and hemochromatosis, which are diffuse processes a me­
precipitating heart fai l u re, measurement of troponin levels
nable to diagnosis by biopsy tech niques; sarcoidosis. on the
may be usefu l . However. tropon i n levels are occasiona l ly
other hand, can be quite patchy and is less l i kely to be discov­
mildly elevated i n patients wi t h an exacerbat ion of heart fail­
ered on endomyocardial biopsy.
ure owing to wal l stress a nd subendocardial ischemia or with
There is n o role for rou tine right heart catheterizalion
acute myocarditis. A n elevated tropon i n level does not guide
for the diagnosis or ma nagemen t of patients w i t h heart fa i l ­
t herapy in this setting but can be used as a ma rker of more
ure. I n patients a d m i t ted f O the hospital w i t h heart fai lu re.
progressive heart fai l u re a nd worse prognosis. Routine evalu­
rou tine right heart catheterization has not been demon­
ation for unusual causes of heart fa ilure. i ncluding hemochro­
strated to decrease either morta l i ty or re hosp i ta lization rates
matosis, Wilson d isease. mul tiple myeloma. and myocarditis,
compared with usual care. Right heart monitoring can be
should not be performed. An evaluation for other u nusual
helpfu l in patients with advanced heart fai lure who are
causes of heart failure should not be pedormed routi nely in a l l
refractory to medical t herapy. Symptomatic hypotension and
patients b u t should on ly b e performed when there are sugges­
worse n i ng kid ney fu nction may be suggestive of low cardiac
tions of specific diseases by h istory or physical examination.
output but cou ld also be caused by i n fection or progression
The most i mportant diagnoslic test i n the evaluation of
of disease. A right heart cat heteriza tion direct ly measuring
heart fai lure is transthoracic echocardiography. An echocar­
cardiac output a n d fi l l i ng pressures can guide t herapy
diogram will give an assessment of ejection fraction as well as
toward i m proving hemodyna m ics ( h igher stroke volume
i n formation about possible causes. For example. ident i fication
and lower n l l i ng pressures) with inotropic agents and/or
of wal l motion abnormalities increases suspicion for coronary more aggressive diuresis i f' the fi l l i ng pressures are high.
artery disease and myocardial ischemia. Echocard iography Right heart cat heterization is also ind icated i n patients bei ng
also allows assessment for aortic and m i t ral valve d isease. with eva luated for heart t ranspla ntation. Pulmonary hyperten­
the caveat that m i t ral regurgi tation is often caused by the sion is a risk factor for poor ou tcomes fo l lowin g heart t ran s­
remodel ing process of heart failure and is therefore secondary plantation because the right vent ricle of the donor heart is
to the heart failure rather than the pri mary cause ( functional not accustomed to pumping aga inst high p u lmonary pres­
mitral regurgitation) . Additionally, the lef't ventricular end­ sures and may fai I .
d i astolic d i mension can be helpful for eva l u a t i n g the chronic­

i ty of t he disease process as wel l as prognosis. Palierns with Eva luation for lschemia
acute heart failure syndromes and a di lated left ventricle likely Alt hough coronary artery d isease is the most com mon cause
have a chronic disease process with delayed onset or recogni­ of heart failure, owing lo expense and radiation exposure, the
t ion of symptoms. A small left ventricle (particularly without routine i nvestigation for coronary disease by stress testing or
wall thin ning) is associated with a grea ter chance of recovery cardiac catheterization or other i maging modalities (such as
of' ejection fraction compared with a markedly dilated left C M R i maging. P ET, or CT) is no longer considered part of the

33
H e a rt Fa i l u re

Cl rou t i ne evaluation or all patients with newly d iagnosed heart TABLE 1 5. New York Heart Association (NYHA)
fa i lure. Card iac catheterizat i on should be performed i n Functional Class
CONT. ,
patients presen t i ng with angina or sign i ricant ischernia. Class Description
Addit ional ly. cardiac cat heteri za t ion is recom mended !or
patients presenting with chest pai n t ha t may or may not be ol' No l i m itations of physical activity

cardiac origin a nd t hose with previous ly d iagnosed coronary S l ig ht l i mitation of physical activity
artery d isease w i thout chest pain if they are e l igible !or revas­ Ill M a rked l i m itation of physical activity
culariza tion. Non i nvasive stress test i ng i s reasonable i n lllA Symptoms with less than ordi nary activity
pat ients with a history or coronary artery disease L o evaluate
lllB Symptoms with minimal exertion
for reversible ischernia. as revascula riza t ion can dramatica lly
IV U n a ble to carry on any physical activity without
improve lert ventricular funct ion . Add i t iona l ly. patients with symptoms
m u ltiple risk !'actors for coronary disease should undergo non­
invasive testing to eva luate for signs of i schemia. I r st ress test ­
i ng identi fies signi fican t ischemic myocard i u m , coronary
TABLE 1 6. Medical Therapy for Heart Failure with
a ngiography shoul d be consi dered . Cl Reduced Ejection Fraction
KEY POINTS , Therapies that Decrease Mortality
• B-type natriuretic peptide levels can be useful to distin­ ACE i n h i bitors/a ng iotensin receptor blockers
guish cardiac from noncardiac causes of dyspnea in the
�-Bl ockers
urgent care setting.
Aldoste rone antagonists (if NYHA class II to IV)
HVC • In patients with heart failure, with the exception of thy-
Hydralazine/isosorbide d i n itrate (b lack patients with NYHA
roid disease, an extensive evaluation of µnusual causes class 1 1 1/IV symptoms)
of heart failure should not be performed unless there
Therapies that Improve Symptoms
are suggestions of specific diseases by history or physi­
cal examination. Digoxin

• The most important diagnostic test in the evaluation of Diuretics


heart failure is transthoracic echocardiography, which l notro pic agents
allows assessment of left ventricular ejection fraction as Vasodilators
well as information regarding potential causes, clinical
NYHA = New York Heart Association.
course, and prognosis.

CJ Med ica l Thera py fo r Systol i c ACE I n h ibitors and Angiotensin


Receptor Blockers
H e a rt Fa i l u re ACE i n h i b itors have been shown to decrease mortal i ty.
The t reatment of patients with H FrEF (systolic heart fai l ure) decrease symptoms of heart fai l u re, improve functional capac­
comprises treat ment or t he acute exacerbation followed by ity. and improve left ventricular ejection f'ract ion . All patients
institution of long- term therapy to decrease morbidity and w i t h H FrEF should be started on an ACE i n h ibitor. The patient
morta l ity and improve symptoms i n pa tients with chronic should be started on a low dose. which can be up-titrated as
heart fai l ure. The initial therapy !or patien ts present ing with tolerated based on blood pressure and symptoms. H igher
acute heart fai lure and volume overload is a diu retic. An ACE doses of ACE i n h i b i tors compared with lower doses have been
inh i b i tor or angiotensin receptor blocker ( A R B) should also be shown to decrease heart fai l u re hospital adm issions but not
sta rted unless the patient has symptomat i c hypotension. Once mortality. I n patients with baseline hypotension. i t is i mpor­
the acute heart f a i l ure episode has stabi l ized. all patients tant to i n itiate ACE inhibitor t herapy, but t he dose should not
should be treated with a �-blocker. be maxim ized pri.or to i n i ti H
< i ng p-blockade because the com­
The Jong- term therapy of h e,irt fo i l u re is b;ised on t h e b i na tion of both age n t s is superior to t hernpy with ei ther one
patient's functional status as measured by New York Hemt <1 lone. Caution should be used when i n i t iating ACE inh ibitor
Associat ion (N \'HA) functional class (Table 15) and signs and t herapy in pat ients with chronic kidney disease. and the
symptoms of vol ume overload. ln addition to ACE inhibitors patient's kidney function should be monitored closely.
and �-blockers. other treatments for heart fa ilure t hat have Hovvever. the presence or kidney dysfunction should not be
been shown to decrease mort a lity <llld future hospital izat ions considered a con t ra indication to t he initiation of these agents:
i nclude aldosterone a ntagoni sts and. speci l1cal ly for black recent guidelines suggest i n i tiati ng t herapy i n pat ients w i t h a
patients. hydralazine-isosorbide d i n itrate (Table 16) . Several serum crea t i n i ne level belovv 3 . 0 mg/d L (26S . 2 p mol/L) .
additional medic;1t ions have been shown to improve symp­ A common adverse effect of ACE inhibitors is a d ry. non­
toms but have no effect on mortal i ty. productive cough. which occurs in up to 20% of patients. For

34
H e a rt Fa i l u re

c::J t h ese patients. it is reasonable to switch to an A R B instead. Less TABLE 1 7 . Therapeutic Doses of �-Blockers for Treatment
information regarding mortality is avai lable for ARBs so they
• of Heart Failure with Reduced Ejection Fraction
OOITT
should not be used as first-line t herapy. O t her common adverse Agent Target Dosage
effects of both cl rugs include hyperkalemia and. occasionally.
worsening kidney fu nction. I n patien ts with angioedema while Carved ilol 25 mg BID (SO mg BID if >85 kg ( 1 87 lb] )

taking ACE inhibitors, ARBs should not be used as an alterna­ Metoprolol succi nate 200 mg daily
tive because there are reports of angioeclema also occurring Bisoprolol 1 0 mg daily
with t hese agents.
BID = twice daily.

�-Blockers
�-Blockers shoul d be started in a l l patients with H FrEF a fter
acute clecompensation is t reated and the patient is hemocly­ D i u retics
namically stable. These d rugs block t h e adverse effects of Diuret ics are the mainstay of t herapy for symptoms of heart
chronic neurohormonal activation on cardiac funct ion . Three fai l u re associated with vol u me overload. To avoid hypo­
�-blockers have been shown to decrease mortal i ty. reduce volemia. the lowest close of diuretic necessary should be used.
heart fa ilure symptoms. and improve left ventricular ejection Loop diuretics are the most commonly used agents. I n patients
fraction in patients with H FrE F: metoprolol succinate, carve­ w i t h refractory heart failure. t he addition of a t hiazide diuretic
cli lol. and bisoprolol. I t is important to use one of' t hese t h ree is occasionally used to augment t he effects of t he loop diuretic.
agents because they are t he only ones t hat have a demon­ There is no advantage to a conti nuous intravenous i n fusion
strated benefit in patients with heart fai l u re. O t her �- blockers. versus bolus therapy in decompensatecl heart fail ure. Adverse
including short- acting metoprolol (metoprolol tartrate) . have effects of diuret ics include hypokalemia . hypomagnesemia.
not shown similar benefit. Some pat ients experience i ncreased and worsening kidney functio n. As electrolyte abnormal ities
fatigue on �- blockade. but the vast majority experience a n can lead to malignant arrhythmias, electrolytes should be fre­
improvement in heart fai lu re symptoms. q u e n t ly measured i n patients receiving h igh doses.
Addi t ionally. patients should be counseled to restrict their
Initiating and Managing ACE I n h i b itor sodi u m and fluid i ntake.
and �-Blocker Therapy
Pa tients with acute heart failure a nd volume overload should Digoxin
ini tially be started on an ACE i nh ibitor. Typical ly. a short­ Digoxin has been used for decades for t he treatment of heart
acti ng agent such as captopril should be used in divided daily fai lure. Digoxin has not been shown to reduce mortality but
closes so t ha t if the patient experiences symptomatic hypoten­ does decrease hospi tal izations for H FrEF i n comparison with
sion, t he effect w i l l be transient. ACE inhibi tors shoul d be placebo. In short- term t rials. cl igox i n has been shown to
t i t rated based on blood pressure and the presence or absence i mprove heart fai lure symptoms, quality of l i fe, and exercise
of adverse effects. For patients w i t h new-onset heart failure tolerance. The withdrawal of digoxin in patients is associated
and volume overload. a �-blocker should not be initiated until with increasing heart failure symptoms.
t he patient is euvolemic or close to euvolemic. Therapy w i t h d igox i n should be c losely fol lowed . I t i s
I n contrast to ACE inhibitors. in which the close can be reasonable t o c h e c k a serum digo x i n level when a p a t i e n t i s
rapidly t i t rated upward. �-blockers should be started at a very stabl e. Pa t i e n ts w i t h kid ney impairment, low body mass.
low close once pa t ients a re euvolemic because these agents a n d o l de r age have reduced metabolism of d igox i n and can
h ave a negative i notropic effect. I nstead of up- titrat ing the q u ickly develop a toxic leve l . I t is important to check a
drug on a daily basis. t i t ra tion of a �-blocker should be per­ d igox i n level in patients w i t h worsen i ng kid ney fu nct io n .
formed slowly at 1 - to 2 -week i n tervals, on an outpatient basis. Ret rospect ive a n alyses h ave s h ow n t h a t serum leve l s
A number of studies have demonstrated a dose-response effect greater t ha n I ng/m L ( 1 .28 nmol / L) a re associated w i t h
with �- blockers. H igh doses compared with low closes of i ncreased r i s k o f morta l i ty, most comm o n ly re l a ted t o
�-blockers have been shown to be more beneficial for both arrhy t h m ias .
morta l i ty reduction and t he reduction of heart failure symp­
toms. Although patients are often discharged on low doses. Aldosterone Antagonists
t hese agents should be up- t i t rated to t he maximal tolerated A ldosterone a n t agon ists (spironolactone. eplerenone) have
doses a fter the patient has been d ischarged (Table 17) . been studied in patients w i t h heart fai lure and N Y H A func­
Limitations lo maximal up-tit ra t ion i nclude symptomatic tional class I I to I V symptoms and have been s hown to
hypotension and bradycardia . Once t he heart rate is below reduce mortal i ty and morb i d i ty. For patients w i t h class I I
60/m i n , the current close can be mai n tained. A history of symptoms, t he benefi t has been shown o n ly i n t hose w it h a
COPD is not a con t raindication to in itiating �-blocker therapy. history of prior hospitalization or an elevated B N P leve l .
a nd t here is no evidence that the nonselective �-blockers are The principal s i d e effec t of t hese agents is hyperka lemia .
not tolerated i n these patients. Spironolactone and eplerenone have not been compared

35
H e a rt Fai l u re

c:::J w i t h one another, but in c l inical trials. gynecomasti a occurs KEY PO I NTS
spec i fically w i t h spi ronolactone. Because of the risk of kid-
CONT. • Initial therapy for all patients with heart failure with
ney dysfunction a n d hyperkal e m i a , t hese d rugs should be
reduced ejection fraction should include an ACE inhibi­
used o n ly i n patients w i t h a seru m crea t i n i n e level below
tor; those with volume overload should be given a diu­
2 . 5 mg/d L (221 µ mol / L) i n men or below 2 . 0 mg/d L
retic, and once the acute heart failure episode has stabi­
( 1 76 . 8 µ mo l / L) i n women . a n d with a serum potassi u m level
lized, all patients should be placed on a �-blocker.
below 5 . 0 m Eq / L (5 . 0 m mol / L) . Add i t ionally, i f' the patient
• �-Blockers in the treatment of heart failure should be
is on potassium supplementation. this should be disco n t i n ­
started at a very low dose and up-titrated slowly, at 1- to
ued when therapy is i n it iated. Electrolytes and kid ney func­
2-week intervals.
t ion should be c hecked l week a fter i n i tiation of' therapy
a n cl be closely m o n itored over t ime. Aldosterone a n tago­ • Aldosterone antagonists should be started in patients
n i sts should be used very caut iously in elderly pa t ien ts . with New York Heart Association class II to IV symp­
These d rugs a re not effect ive as d i u ret ics at the doses used toms with appropriate kidney function and a potassium
in heart fai l ure t herapy ( 1 2 .5 -25 mg/cl for spironolactone. level below 5 . 0 mEq/L (5.0 mmol/L).
25-50 mg/d for eplerenon e) . • The addition of isosorbide dinitrate and hydralazine to
standard heart failure therapy is associated with
lsosorbide Dinitrate and Hydra lazine improvements in quality of life and a mortality benefit
The combination of isosorbide d i n i t rate and hydralazine is in black patients.
an a lternative therapy for patients with heart fa i l u re who
have kid ney dysfu n c t io n that l i m its therapy w i t h e i t her ACE
i n h i b i tors or A R Bs . In t h is sett i ng. this comb i n a t io n is used M a n a g ement of Heart Fai l u re c:::J
for its vasod i la t i ng propert ies. t-.'1ore recen t ly, based on retro­
spective clata from earl ier c l i n ical trials. a c l i n ical t rial i n
with Preserved Ejection Fraction
black patients w i t h heart fai l u re and reduced ejection frac­ ACE i n hibitors. A R Bs, �-blockers, and a ldosterone antagon ists
tion and NYHA class I l l and IV symptoms was performed have been studied i n patients with H FpEF. U n fortunately, none
t hat demonstrated a reduction i n mortality w i t h a speci fic of these agen ts have demonstrated any clinical benefit com­
for m u l a t ion of t h i s combination compared w i t h pl acebo. pared with placebo. At this t i me, no medications have demon­
There was a high incidence of adverse effects (pri mari ly strated a reduction i n mortality i n this pat ien t population.
peripheral edema. dizzi ness. gastro i n testi n a l sy mptoms. and Therapy for H FpEF should i nstead be based on treating the
headaches) and d rug w i t h d rawa l . For black pat ients. studies causes and symptoms of the heart failure. Hypertension is a
have demonstrated i mp rovements i n qual i ty of l ife i n addi ­ common cause of H FpEF. and aggressive control of blood pres­
tion to a mort a l i ty benefi t . Note t ha t t h i s combination was sure is necessary. Additional ly, controll i ng tachycardia can be
studied as an additional t herapy for patients already on a n helpfu l in pa tients with atrial arrhythmias.
A C E i n h ibitor or a n A R B and a �-blocker, n o t a s a replace­ Patients with H FpEF are often quite vol u me sensitive. with
m e n t t herapy. and should o n ly be i ns t i t u ted after t hese a small t herapeut ic window between hypovolemia and hyper­
agents have been max i m ized. volemia. Judicious use of diuretics to maintain euvolemia is
importan t . These patients should be encouraged ro monitor
Calcium Cha nnel Blockers t heir weight closely. as small d i fferences in volume can quickly
Because of t he i r vasod i l a t i ng effects, ca lcium channel block­ cause volume overload and subsequent hospital admissions.
ers have been closely studied for their potential role in the
management o f heart fai l u re. U n fortunately, t he non­
d i hydropyridi ne calc i u m channel b lockers ( for exa m ple , Device Th era py
d i l t i azem or verapa m i l ) also h ave myocard ial depression Sudden cardiac death is t he cause of deat h i n approximately
activity and have been demonstrated to either have no ben­ 50% of patients with heart fai lure. The only reliable predictor
efit or worse outcomes i n patients w i t h heart fa i lure. Patients of an arrhythmic event is left ven tricular ejection fraction. For
who have been t reated for hypertension with cl i l t iazem or this reason. i mplantable cardioverter-defibril lators ( ! CDs) are
verapa m i l should have t h ose agents d isco n t i n ued once a used for primary prevention of sudden cardiac death i n
d iagnosis of heart fai l u re has been made. The second­ patients w i t h heart failure a n d low ejection fraction .
generation d i hydropyrid i ne calcium channel blockers, such
as amlodipine and felod i p i ne. have been shown to be safe i n I mplantable Cardioverter-Defibrillator
patients w i t h heart fai l u re. but d o not reduce morb i d i ty or for Prevention of Sudden Cardiac Death
morta l i ty. For patients who are s t i l l hypertensive o n h ig h In patients with m i l d to moderate heart fai l u re symptoms and
doses o f A C E i n h i bi tors and 0-blockers. a periphera l ly acting left ven t ricular eject ion fraction less t han or equal to 35%.
d i hydropyridine calcium channel blocker can be used as an placement of an !CD reduces mortal ity compared with medi­
anti hypertensive agent. CJ cal t herapy or placebo i n patients with both ischemic and

36
H e a rt Fa i l u re

Cl patients with NYHA functional class I I and I I I symptoms, ejec-


nonischemic etiologies (Table 18) . !CDs are indicated for electromechanical coordination) manifested by prolongation
of the QRS duration or a left bundle branch block. Biventricular
CONT.
tion fraction less than or equal to 35% on guideline-directed pacing. or cardiac resynchronization therapy (CRT) , involves
medical therapy. and a l i fe expectancy of at least 1 year. As pacing the right and left ventricles simultaneously. In addition
patients with class IV symptoms have a reduced life expec­ to the usual placement of a pacer lead i n the apex of the right
tancy, !CDs are not indicated in this population except in ventricle, an additional lead is placed through the coronary
patients who are awaiting transplantation or undergo place­ sinus down a coronary vein on the lateral wall of the left ven­
ment of a mechanical circulatory device. t ricle. This simultaneous pacing has been demonstrated to
!CDs should only be placed after patients are on guideline­ increase ejection fraction, decrease heart failure symptoms.
directed medical t herapy. For patients with recent onset of and reduce mortality in patients with HFrEF and ECG evi­
heart failure who have a reasonable chance of recovery of dence of dyssynchrony. The 2013 American Col lege of
function, one should wait upwards of 6 months with the Cardiology Foundation/ American Heart Association heart fail­
patient on adequate medical therapy and then reassess ven­ ure management guideline makes a strong recommendation
tricular function to determine if the ejection fraction is still with strong supporting evidence for CRT therapy in patients
less than 35% prior to implantation. Specifically. patients with an ejection fraction less than or equal to 35%. NYHA
treated with revascularization for coronary artery disease may functional class I l l to IV symptoms on guideline-directed
have improvement in ejection fraction. which should be medical therapy, and left bundle branch block with QRS dura­
re-measured after revascularization. tion greater than or equal to 150 msec (see Table 18) . A strong
recommendation with weaker evidence is provided for
Cardiac Resynchronization Therapy patients with NYHA functional class II symptoms.
In approximately 30% of patients, heart failure progression is Patients with NYHA class I symptoms caused by ischemia
accompanied by dyssynchrony (dysfunctional ventricular and a left bundle branch block with a QRS duration greater
than 150 msec have shown some benefit with CRT therapy, but
the risk-benefit ratio is high owing to adverse effects of the
therapy, and it is currently a class Ifb recommendation. Adverse
TAB LE 1 8. Indications for Device Therapy in Heart Failure
effects include infection at the site of the device, inappropriate
Implantable Cardioverter-Defibrillator firings. and occasional tricuspid valve regurgitation. Cl
(for primary prevention)
KEY POI NTS
NYHA class II or I l l while ta king guideline-directed medical
thera py• and • An implantable cardioverter-defibrillator is recom­
Expectation of survival > 1 year and mended for patients with New York Heart Association
class II or III heart failure, a left ventricular ejection
Either of the fo l l owi n g :
fraction less than or equal to 35% after treatment with
lschemic cardiomyopathy �40 days post M l or nonischemic
guideline-directed medical therapy, and a life expec­
cardiomyopathy with ejection fraction ;<;35% (primary
prevention) tancy of at least 1 year.

History of hemodynamically sign ificant ventricu l a r • Cardiac resynchronization therapy is recommended for
arrhythmia or cardiac a rrest (secondary prevention) patients with New York Heart Association class II to IV
Biventricular Pacemaker heart failure, a left ventricular ejection fraction less
(cardiac resynchronization therapy) than or equal to 35% on guideline-directed medical
therapy, and a left bundle branch block with QRS
All of the fo l l owi n g :
duration greater than or equal to 150 msec.
NYHA class I I t o IV

Ejection fraction ;<;35%

On guideli ne-d i rected medical therapy


Assessment of Chronic
Ventricular dyssynchrony (LBBB with a ORS duration �1 50 msec)
Heart Fai l u re
LBBB = left bundle branch block; M l = myocardial infarction; NYHA = New York
Heart Association. Patients with chronic heart failure should be serially assessed
3Also NYHA class I in patients with ischemic cardiomyopathy and ejection fraction for progression of disease in the outpatient setting. At each
<30% (MADIT-11 criteria: Moss AJ, Zareba W, Hall WJ, et al; M ulticenter Automatic
Defibrillator Implantation Trial II I nvestigators. Prophylactic implantation of a defi­
visit, it is important to assess current symptoms and func­
brillator in patients with myocardial infarction and reduced ejection fraction. N tional capacity, volume status, and the adequacy of the medi­
Engl J Med. 2002;346( 1 2):877-883. [PMID: 1 1 907286])
cal therapy (both appropriate doses and the appropriate med­
Recommendations from Yancy CW, Jessup M, Bozkurt 8, et al; American College of
Cardiology Foundation; American Heart Association Task Force on Practice ications as heart failure progresses) . Of equal or greater
Guidelines. 20 1 3 ACCF/AHA guideline for the management of heart failure: a
importance is repeated patient education, including remind­
report of the American College of Cardiology Foundation/American Heart
Association Task Force on Practice Guidelines. J Am Coll Cardiel. 20 1 3 Oct ing patients to take their medications as prescribed, measure
1 5;62(1 6):e1 47-239. [PMID: 23747642]
their weight daily, avoid dietary sodium, watch their fluid

37
H e a rt Fai l u re

intake, and exercise regularly. Patients who appropriately take <14 mL/kg/min) or a high ratio o f ventilation-to-carbon dioxide
their medications and avoid sodium and excess fluid intake production (VE/VC02 >34) have a poor 1-year prognosis.
can greatly improve their functional status. The Seattle Heart Failure model (www. SeattleHeart
Despite multiple studies demonstrating the benefit of FailureModel.org) is an online program that uses clinical char­
medical therapies in heart failure, fewer than 60% of patients acteristics to predict outcomes in patients with heart failure.
are discharged from the hospital on ACE inhibitor and �-blocker This model can be used to help assess prognosis based on
therapy. It is important to review medications at every visit to clinical characteristics and can be used to guide patients as
ensure that patients are on the appropriate therapy. they ask questions about their prognosis.

K EY P O I NTS
Serial B-Type Natriuretic Peptide Assessment
• Patients with chronic heart failure should be seen regularly
Serial assessment of BNP levels in patients with chronic heart
for assessment of clinical status as well as ongoing patient
failure have been evaluated in a number of studies. Although
education regarding taking medications as prescribed,
higher BNP levels are associated with increased mortality,
measuring their weight daily; reducing dietary sodium and
change in level in an individual patient does not predict pro­
avoiding excess fluid intake, and exercising regularly.
gression of disease. Additionally, there is no evidence of
benefit to using BNP for serially following patients to assess • In patients with chronic heart failure who are clinically HVC
volume status or for dose adjustment of medications. stable, armual or more frequent follow-up echocardiog-
raphy rarely provides therapeutic or diagnostic benefit
Echocardiography in Chronic Heart Failure and is not recommended.
Echocardiography should be performed in patients with severe
left ventricular dysfunction after optimization of medical
therapy to determine if the left ventricular ejection fraction
has improved to above 35% before consideration of !CD
I n patient Management CJ
of H ea rt Fa i l u re
implantation. For patients with chronic heart failure who are
clinically stable, echocardiography rarely provides diagnostic Acute Decompensated Heart Fai l u re
benefit, and obtaining annual echocardiograms is not likely to Patients with heart failure admitted to the hospital usually have
change therapy or outcome. For patients hospitalized with symptoms of volume overload as the primary concern. Reasons
acute heart failure, obtaining a repeat echocardiogram to eval­ to admit patients include progressive heart failure symptoms
uate left ventricular function or for worsening valvular abnor­ with dyspnea at rest, an inability to respond to oral diuretics.
malities is reasonable. If a patient has progressive heart failure recurrent ICD firings, symptoms of ischemia. worsening kid­
symptoms as an outpatient, a repeat echocardiogram can be ney function. and signs of poor perfusion (such as cool extrem­
helpful to evaluate for progressive valvular abnormalities, new ities, a low pulse pressure, or pulsus alternans) . Therapy i
wall motion abnormalities, or an increase in left ventricular primarily focused on diuresis. Additional evaluation should be
size that may alter treatment and affect prognosis. performed to determine the reasons for the decompensation,
including a review of medications and whether the patient was
Assessing Prognosis taking his or her medications properly, an echocardiogram to
Multiple retrospective studies have been performed looking at look for reversible causes of worsening function. and, i f appro­
methods to evaluate prognosis in patients with heart failure. priate. an evaluation for ischemia . Generally. the initial dose of
Current 1-year survival rates for patients undergoing heart intravenous diuretic should be at least equivalent to the total
transplantation or placement of a left ventricular assist device daily oral dose. l f the patient does not respond appropriately to
are between 85% and 90%. Patients with a higher risk of death that dose, rapid up-titration should be performed to assist in
should be considered for these therapies. Important risk factors fluid removal. The patient's usual outpatient medications (for
for death include NYHA class IV symptoms, repeat hospitaliza­ example, ACE inhibitor, �-blocker) should be continued unless
tions, hyponatrernia (serum sodium <133 mEq/L [133 mmol/L]) , the patient is hypotensive or demonstrates signs of poor perfu­
worsening kidney function, higher doses of diuretics, intoler­ sion, in which case dose reduction or discontinuation of both
ance of ACE inhibitors or �-blockers, and arrhythmias result­ the ACE i n hi b i t o r and �-blocker should be considered . I n
ing in !CD firings. Patients with multiple risk factors should be patients with signs o f low-output heart failure (hypotension,
referred to a heart failure cardiologist for further evaluation. worsening kidney or liver function. cool extremities) . the
Additionally, occasional discussions with patients regarding �-blocker should be discontinued.
end-of-life issues and their wishes for advanced heart failure Patients should be adequately diuresed during the hospi­
therapy should be initiated while the patient is still stable. talization . Orthopnea and an elevated central venous pressure
Patients' advanced care plans often change over time. are suggestive of elevated fJlling pressures. Patients often have
Cardiopulmonary exercise testing is routinely performed to symptomatic improvement before they are euvolemic. and
assess prognosis in patients being evaluated for transplantation. striving to achieve euvolernia may result in a reduction in read­
Patients with a low oxygen consumption (peak 02 consumption mission rates. Volume status can be difficult to assess and a

38
H e a rt Fa i l u re

Cl careful physical examination is required. Patients should be Card iogenic S hock


assessed for signs of volume overload even if they have reached Cardiogenic shock is defined by persistent, symptomatic hypo­
CONT.
a goal or "dry" weight, as this is not a reliable measure of euv- tension and end-organ dysfunction. Patients have acute kidney
olemia. Findings supportive of euvolemia include an estimated failure, evidence of liver dysfunction with elevated arrunotrans­
central venous pressure below 10 cm H20, no orthopnea, ferase levels, poor peripheral perfusion wHh cool extrerruties,
absence of peripheral edema, and reduced dyspnea. Despite the and decreased mental status. Cardiogenic shock requires inten­
use of high-dose intravenous diuretic therapy and improve­ sive therapy with intravenous vasopressors. Patients who remain
ment in volume status during hospitalization, many patients in shock despite intravenous therapy and with worsening organ
are discharged before achieving euvolemic or decongested sta­ function should be considered for mechanical support.
tus. Objective markers of decongestion include net urine out­ It is important to quickly rule out reversible causes in
put, weight loss, evidence of hemoconcentration (increase in patients with cardiogenic shock. Reversible causes include
hematocrit and hemoglobin levels) , and reduction in BNP level. acute myocardial infarction; ventricular septa! or free wall
These markers have been associated with lower rates of rehos­ rupture; and acute valvular regurgitation, possibly related to
pitalization for heart failure in short-term follow-up. papillary muscle rupture, infection, or ascending aortic arch
Ultrafiltration can occasionally be used in patients admit­ aneurysm with dissection of the aortic valve. Bedside echocar­
ted with volume overload when routine diuresis fails or diography can be helpful in identifying structural causes of
patients are unresponsive to diuretic therapy. A recent rand­ cardiogenic shock.
omized trial of ultrafiltration versus intravenous diuretics in The irutial therapy for cardiogenic shock includes vasoac­
patients with decompensated heart failure and worsening tive medications to increase cardiac output and raise blood pres­
kidney function showed no difference in weight loss but wors­ sure through peripheral vasoconstriction (Table 19) . Patients
ened kidney function in patients treated with ultrafiltration . with cardiogenic shock secondaty to progressive heart failure
Worsening kidney function, typically defined as a semm are generally given an inotropic agent, such as dobutamine or
creatinjne increase of 0 .3 mg/dL (26 .52 µmol/L) or greater milrinone (cleared by the kidneys) . Patients with peripheral
during hospitalization for heart failure, is a common compli­ vasoconstriction (increased systemic vascular resistance) often
cation, occurring in up to 25% of patients. This event often benefit from the addition of a pure vasoililator such as sodium
triggers changes in medical therapy, including reduction or nitroprusside. Placement of a right heart catheter can be helpfi.Ll
cessation of diuretics, ACE inhibitors or ARBs, or aldosterone to assess filling pressures, cardiac output, and systemic vascular
antagonists. Retrospective studies have found that this compli­ resistance to help choose the appropriate medical regimen.
cation is associated with worse survival long-term, but no Although the routine placement of a right heart catheter for
therapies have been shown to prevent or reduce worsening patients admitted with heart failure has not been shown to
kidney function in patients with heart failure. improve outcomes. it should be considered to assist in therapeu­
Vasopressin antagonists can be used for the treatment of tic decision-making in patients with cariliogenic shock.
patients with hypervolemic or euvolemic hyponatremia. They For patients with symptomatic hypotension and end­
are beneficial for correcting hyponatremia but have not dem­ organ dysfunction despite vasopressor therapy, mechanical
onstrated any mortality benefits. therapy should be considered. Mechanical support options

TABLE 1 9. Intravenous Vasoactive Medications Used for Treatment of Cardiogenic Shock


Medication Mechanism lnotropy Vasodilation

M ilrinone Phosphodiesterase i n hibitor ++ +

Dobutamine 13 1 , 132 receptors ++ (+) (at low dose)

- (vasoconstriction, at high dose)


Nesiritide Natriuretic peptide receptors 0 ++

Sod i u m nitroprusside Nitric oxide 0 ++

Nitroglycerin N itric oxide 0 + + (mainly venous)

Vasopressin V receptor - (vasoconstriction)

Dopamine D receptor + - (vasoconstriction, at high dose)

13 1 receptors at i ntermediate dose


ex, receptor at high dose
Norepinephrine Affinity for ex1 , ex receptors g reater than for 131 + - (vasoconstriction)
2
receptors

Strength o f effect: ++ indicates very strong; + indicates strong; ( + ) indicates weak; 0 indicates neutral; - indicates opposite effect.

39
H e a rt Fa i l u re

CJ include intra-aortic balloon pumps and percutaneous or sur­ transplant is 65 to 70 years. Patients with kidney dysfunction,
gically implanted short-term mechanical ventricular assi.st diabetes with end-organ manifestations. malignancy. chronic
CONT.
devices (VADs) . These assist devices have catheters that are infection, or other comorbidities are often denied transplant.
placed into the vascular system (left atrium or ventricle) ; they Options tor patients who are not candidates for heart transplan­
then pump the blood into the aorta, essentially assisting the tation include mechanical circulatory support as destination
failing left ventricle. These devices augment cardiac output therapy and inotropic therapy. However, inotropic therapy does
and improve end-organ perfusion . Because all of these devices not decrease mortality and may actually increase it. The survival
require large catheters, a common complication is vascular of inotropic-dependent patients is less than 10% at 1 year.
compromise at the point of insertion.
Mechanica l Circulatory Support
Strategies to Prevent Readmission With the development of continuous-flow left ventricular assist
At many U.S. hospitals. heart failure is the most common dis­ devices (LVADs) , the survival of patients vvith advanced heart
charge diagnosis. Currently. 30-day readmission rates are greater failure after implantation of these devices has dramatically
than 20%, and reducing these admissions is a major focus of improved. The pumps are smgically inse11ed into the left ventri­
study and resources. Studies evaluating dimesis have shown that cle, and the blood is pumped through the device from the left
greater fluid removal during the hospitalization is associated ventTicle to tJ1e aorta. The patients have a line (driveline) coming
with a lengthening of the time to readmission. Additionally, it is through the skin through which the power is transmitted to tlle
imp011ant that patients are discharged on appropriate medical pump. Ninety percent of patients receiving an LVAD as a b ridge to
therapy, including an ACE inhibitor and a �-blocker. Early physi­ heart transplantation are alive at 1 year. For patients who are not
cian fol low-up, ideally within 7 days of discharge, has also been candidates for transplant and have an LVAD placed as destination
associated with a reduction in readmissions. therapy, the survival at 2 years is more tJ1an 60% and improving.
Because it is often difficult to schedule patients for an CompUcations related to these devices include ischemic and
office visit within 1 week of discharge, multidisciplinary heart hemoIThagic stroke (>10% o f patients), driveline-related infec­
failure clinics have been created by a number of hospitals. tions (approximately 30%) . and gastrointestinal bleeding related
These programs often include telephone monitoring of signs to arteriovenous malformations (20% in some reports) . Cl
and symptoms. evaluating whether patients are actually tak­
Management of Post-Transplant Patients
ing their medications, educating patients on salt and fluid
restriction, and providing a mechanism for early fol low-up The prognosis of heart transplant recipients has improved
after hospitalization. Cl greatly in recent years. Most patients have no functional limita­
tions and return to a normal quality of life. Patients typically
KEY POINTS begin on a three-drug immunosuppression regimen early after
• In hospitalized patients with acute volume overload, the transplantation that includes a calcineurin inhibitor (cyclo­
initial dose of intravenous diuretic should be at least sporine or tacrolimus) , an antiproliferative agent (mycopheno­
equivalent to the total daily oral dose; rapid up-titration late mofetil, sirolimus, or everolimus) , and prednisone. Most
should be performed if needed to assist in fluid removal. centers try to wean patients off of prednisone by 1 year.
• Reversible causes of cardiogenic shock include acute lmmunosuppressive medications are associated with a number
myocardial infarction, ventricular septa! or free wall of adverse effects, including hypertension (>90% of patients)
rupture, and acute valvular regurgitation. and new-onset diabetes (20% of patients) . During the first year
post-transplant, while doses of immunosuppressants are high,
HVC • In patients discharged with a diagnosis of heart failure,
patients have an increased risk for infection. Rejection occurs
early physician follow-up, ideally within 7 days of dis­
in approximately 20% of patients in the first year but is almost
charge, has been associated with a reduction in hospital
nonexistent after the first year unless a patient stops taking
readmissions.
immunosuppressants. Signs of rejection include heart failure
and atrial arrhythmias (typically atrial flutter) . However, most
patients with rejection manifest no clinical symptoms, neces­
Adva nced Refractory Heart Fa i l u re sitating routine surveillance with endomyocardial biopsy. The

Cl therapeutic options are limited to inotropic therapy, heart trans­


Once patients have progressed to advanced heart failure, the surveillance interval varies; however, most centers perform
biopsies between 1 and 5 years after transplant.
plantation, mechanical circulatory suppm1, and palliative care. The long-term complications of heart transplant include
Hea11 transplantation is the best option for patients with end­ cardiac allograft vasculopathy and malignancy. Cardiac allo­
stage heart failure, with 50% smvival rates approaching 13 years, graft vasculopathy occurs in more than 50% of the patients
but is limited by donor availability such that only 2000 trans­ by the fifth year after transplant. It is characterized by diffuse
plants are performed in the United States annually. Candidates intimal thickening of the coronary arteries that starts distally
for transplant are therefore thoroughly evaluated for comor­ and progresses proximally. For this reason, the usual thera­
bidities that may limit survival The upper age Limit for heart pies for coronary disease, such as percutaneous coronary

40
H e a rt Fa i l u re

intervention and coronary artery bypass grafting, are usually Characteristic electrocardiographic changes include ST-segment
not beneficial. Lymphoproliferative disorders and skin can­ elevation and diffuse deep T-wave inversions with some pro­
cer are the most common malignancies. longation of the QTc interval. Takotsubo cardiomyopathy is
Because the heart in transplant patients is denervated, they usually associated with recovery of systolic function in the acute
usually do not experience typical ischemic chest pain, leading to period. Nevertheless, these patients should be treated with
atypical presentations of coronary artery disease and acute coro­ ACE inhibitors and �-blockers acutely. There is no accepted
nary syndromes. Additionally, without vagal innervation, heart length of time to continue this therapy in patients whose left
rates tend to run between 90/min and 110/min. Heart transplant ventricular function returns to normal. For the rare patient who
patients have a marked response to adenosine but are not does not recover, this therapy should be continued.
responsive to digoxin or atropine. For transplant patients pre­
senting with atrial arrhythmias, caution should be used before Acute Myocarditis
giving adenosine to diagnose the arrhythmia because it may Myocarditis usually presents with heart failure symptoms over
cause prolonged atrioventricular conduction block. a few days to weeks. Occasionally, patients have symptoms for
several months before heart failure is discovered. The classic
KEY POI NTS
presentation of viral myocarditis includes a viral prodrome
• Cardiac allograft vasculopathy occurs i n more than 50% with fever, myalgia, and upper respiratory symptoms, but a
of heart transplant recipients by the fifth year after prodrome is not required for the diagnosis. Patients present
transplant; because of its diffuse nature, revasculariza­ with dyspnea, chest pain, and arrhythmias. ECG abnormalities
tion is usually not beneficial. are often present, along with evidence of myocardial damage
• Because the heart in transplant patients is denervated, with elevated troponin levels.
they usually do not experience typical ischemic chest Various infectious pathogens can cause myocarditis. The
pain, leading to atypical presentations of coronary most common causes are adenovirus, coxsackievirus, and
artery disease and acute coronary syndromes. enterovirus. The pathogenesis of myocarditis is unclear and
may involve direct infection of the myocardium with the virus
or an immune system response to the infection.
S pecific Ca rdiomyopathies Endomyocardial biopsy can define myocarditis with evi­
The most common cause of heart failure is coronary artery dis­ dence of myocardial necrosis, degeneration, or both, with an
ease. Other common causes include hypertension and idiopathic adjacent inflammatory infiltrate. Indications for endomyocar­
cardiomyopathy. Approximately 10% of patients with heart fail­ dial biopsy include ventricular arrhythmia, high-grade con­
ure have heart failure related to a specific etiology. This includes duction block (type II or III) or lack of response to usual heart
medication-induced cardiomyopathies (primarily chemothera­ failure therapy.
peutic agents) , myocarditis, amyloidosis, sarcoidosis, infectious Therapy for acute myocarditis is supportive and con­
etiologies such as HIV, periparturn cardiomyopathies, and alco­ sists of usual heart failure therapy. Placebo-controlled
hol or other drug-induced cardiomyopathies. Others will be immunosuppressive trials have not demonstrated improve­
discussed later. Restrictive cardiomyopathies with such causes as ments in mortality or ejection fraction. Patients often take
chemotherapeutic agents, amyloidosis, and sarcoidosis are dis­ months (6 -12) to recover left ventricular function.
cussed in Myocardial Disease. Peripartum cardiomyopathy is Approximately 50% of patients eventually recover cardiac
discussed in Pregnancy and Cardiovascular Disease. function; therefore, it is important to wait and not place an
!CD for the usual indications (ejection fraction <35% and
Takotsubo Cardiomyopathy NYHA class II or III symptoms) until at least 6 months of
Takotsubo, or stress-induced, cardiomyopathy is a syndrome of heart failure therapy.
reversible ventricular systolic dysfunction usually precipitated
by an acute emotional or physiologic stress. Although takotsubo Giant Cell Myocarditis
cardiomyopathy was initially described in elderly women fol­ Giant cell myocarditis is an acute, rapidly progressive form of
lowing intense emotional stress, the syndrome may occur in myocarditis associated with ventricular arrhythmias and pro­
men and in some patients an antecedent stress may not be gressive cardiac dysfunction despite medical therapy. For
identifiable. It is believed to be caused by sympathetic-mediated unclear reasons, this process usually occurs in persons younger
myocyte injury, but the precise pathogenesis is unknown. It than 40 years. The underlying mechanism is unknown but is
often mimics an acute myocardial infarction with elevated tro­ thought to be autoimmune. On endomyocardial biopsy, the
ponin levels and electrocardiographic changes, but it is usually pathognomonic "giant cell" is seen. Unlike other forms of
associated with normal coronary arteries. The hallmark is wall myocarditis, aggressive immunosuppressive therapy has been
motion abnormalities that extend beyond a single coronary ter­ shown to improve survival but this process is still often fatal.
ritory, identified by echocardiography or other imaging study. These patients should be considered for cardiac transplantation
For example, on left ventriculogram, the apex of the heart will but often need to be bridged with VADs. There are case reports
be hypokinetic and the mid heart will contract normally. of giant cell myocarditis recurring post-transplantation.

41
M yo ca rdial Disease

Tachycardia-Mediated Cardiomyopathy sarcomeric mutations involving at least 11 different genes have


Sustained tachyarrhythmia for weeks to months can produce been identified. The mutations demonstrate an autosomal
cardiomyopathy. The most common arrhythmias implicated dominant pattern of inheritance with high disease penetrance.
are atrial fibrillation, atrial flutter, and tachycardia. Patients Sporadic cases may also occur.
with very frequent premature ventricular contractions The major pathophysiologic mechanisms contributing to
(�10,000/d) can develop cardiomyopathy. Controlling heart signs and symptoms of HCM are diastolic dysfunction and
rate with medications or ablation often results in improve­ dynamic left ventricular outflow tract (LVOT) obstruction.
ment and resolution of heart failure for these patients. Patients
Diastolic dysfunction arises from abnormal relaxation and
with tachycardia should be evaluated for hyperthyroidism.
poor myocardial compliance, leading to an increase in left
ventricular filling pressures. Dynamic LVOT obstruction, pre­
sent in approximately two thirds of patients, results from

Myocardial Disease septa! hypertrophy and anterior displacement of the rnitral


valve apparatus; this displacement also is associated with
Hypertro p h i c Ca rdiomyopathy decreased coaptation of the valve leaflets and resultant mitral
Clinical Presentation and Evaluation regurgitation (Figure 11, Figure U) . Dynamic LVOT obstruction
Hypertrophic cardiomyopathy (HCM) is an inheritable cardiac is exacerbated by any drug therapy that increases contractility,
disorder with a prevalence ofl in 500 persons. More than 1400 such as digoxin or other inotropic agents. Obstruction is also

F I G U R E 1 1 A patient with hypertrophic cardiomyopathy. Resting 1 2-lead electrocardiogram ( top left panel) demonstrating findings of left ventricular hypertrophy and second·
.

ary ST-seg ment changes. Echocardiography (parasternal view) (bottom left panel) demonstrating severe myocardial hypertrophy of the ventricu lar septum (asterisk). Systolic ante·
rior motion of the mitral valve is present (arrow) (top right panel). Apical long axis view of the left ventricle (bottom right panel). Systolic anterior motion of the mitral valve (arrow­
head) leads to decreased coaptation of the mitral valve and secondary mitral regurgitation (arrow). Ao = ascending aorta; LA = left atrium; LV = left ventricle; RV = right ventricle.

42
Myocard ial D isease

etiology (Figure 13) . The pattern of myocardial hypertrophy


.. 200. .. .. classically is described as asymmetric and involving princi­
Expiration-+ Inspiration--+
pally the ventricular septum, but concentric, apical, and
eccentric forms also are common. Cardiac magnetic reso­
nance (CMR) imaging also may be used for diagnosis, par­
ticularly when there is eccentric or apical hypertrophy that
may be difficult to visualize on echocardiography. Doppler
echocardiography can accurately diagnose and quantify
LVOT obstruction in most instances, with examinations
performed at rest, with provocation (such as with amyl
nitrite or dobutamine), or during exercise. LVOT obstruc­
tion is considered significant when the calculated pressure
4 . gradient is either 30 mm Hg or greater at rest, or 50 mm Hg
LV
or greater with provocation. In 80% to 90% of patients with
HCM, electrocardiography (ECG) demonstrates findings of
left ventricular hypertrophy.
F I G U R E 1 2 . Dynamic changes i n the left ventricular outflow tract ( LVOT) gradi­
ent from obstructive hypertrophic cardiomyopathy d u ri n g respiration. These hemo­ Dynamic LVOT obstruction is present in approximately
dynamic tracings were taken d u ring q u iet respiration. Note that the LVOT gradient 25% of HCM patients at rest, with another 30% to 40% demon­
(shaded) i ncreases sign ificantly with expiration. These changes are mediated by strating obstruction only after provocative maneuvers. Patients
respiratory-induced variation i n ventricular afterload and have been described as can suffer from debilitating symptoms regardless of the pres­
reversed pulsus paradoxus. Ao = ascending aorta; LV = left ventricle.
ence of dynamic LVOT obstruction. However, because of the
high prevalence of dynamic obstruction in patients with HCM
and the availability of specific strategies that can reduce symp­
toms by alleviating it, clinicians should thoroughly evaluate
worsened by reduced ventricular preload, such as from dehy­ for the presence and degree of obstruction.
dration, or afterload, such as from vasodilator therapy. HCM must be distinguished from other disorders that are
Most patients with HCM have minimal or no symptoms. associated with myocardial hypertrophy or increased wall
However, a subset develop exertional dyspnea, angina, pre­ thickness. These disorders include hypertensive heart disease,
syncope, or syncope. Sudden cardiac arrest may be the initial aortic stenosis, cardiac amyloidosis, Noonan syndrome, Fabry
manifestation of the disorder. HCM accounts for approxi­ disease, and mitochondrial cardiomyopathy. Athlete's heart is
mately 35% of sudden deaths among all persons younger a syndrome of myocardial hypertrophy that can be difficult to
than 35 years and is the leading cause of sudden death in this differentiate from HCM. Several clinical features are suggestive
age group. of athlete's heart, although a period of deconditioning
Several physical signs are helpful in diagnosing HCM in (6 months or more) may be required to diagnose the presence
patients in whom it is suspected. The carotid upstroke typi­ of physiologic hypertrophy (Table 20, on page 45) .
cally is rapid and may be bifid (pulsus bisferiens- "twice
beating") in those with LVOT obstruction. The jugular venous
K EY P O I N T
pulse may demonstrate a heightened a wave when there is • The systolic murmur of obstructive hypertrophic cardi­
significant infundibular hypertrophy. The apical impulse typ­ omyopathy is augmented by maneuvers that decrease
ically is sustained, localized, and may be bifid or trifid. An S4 preload (squat-to-stand, Valsalva strain) ; in contrast, the
is frequently present, especially in younger patients. systolic murmurs of aortic stenosis and mitral regurgi­
Paradoxical splitting of the S 2 also may occur with significant tation diminish with maneuvers that reduce preload.
LVOT obstruction.
The murmur of dynamic LVOT obstruction is systolic with Clinical Course and Risk Stratification
an ejection quality, frequently heard best at the left lower ster­ Although symptoms of HCM can be debilitating and sudden
nal border. Augmentation of the murmur occurs during cardiac death is a devastating complication, symptoms are
maneuvers that decrease preload (squat-to-stand, Valsalva minimal or absent and lifespan is normal in approximately
strain) and afterload (amyl nitrite inhalation) . These maneu­ 90% of patients with HCM. Atrial fibrillation occurs in 20% to
vers can help distinguish HCM from other causes of systolic 25% of patients with or without symptoms of obstruction. In
murmurs, such as aortic stenosis and mitral regurgitation. some patients, owing to the underlying diastolic dysfunction,
Contrary to HCM, the murmurs of aortic stenosis and mitral the loss of atrial systole from these arrhythmias can precipitate
regurgitation diminish with maneuvers that reduce preload. significant hemodynamic deterioration in addition to embolic
The diagnosis of HCM typically is made by two-dimen­ risk Contemporary investigations of unselected HCM popula­
sional echocardiography and visualization of severe myo­ tions demonstrate an incidence of sudden cardiac death of
cardial hypertrophy in the absence of a local or systemic 0.5% to 0.8% per year. All patients with HCM should undergo

43
Myocard i a l Disease
�������-

F I G U R E 1 3 . Pathology of hypertrophic cardiomyopathy. Gross specimens demonstrate severe myocardial hypertrophy due to hypertrophic cardiomyopathy with concen­
tric hypertrophy (A and B) and apical hypertrophy (C). M icroscopy demonstrates myocyte disa rray, the histologic hal l mark of hypertrophic cardiomyopathy (0). Ao = ascending
aorta; LA = left atrium; LV = left ventricle; RA= rig ht atri um; RV = right ventricle.
Images courtesy of Dr. William D. Edwards, Department of Pathology, Mayo Clinic

risk stratification for sudden cardiac death (Table 21) . Therapy Because a significant number of sudden deaths occur during or
with an implantable cardioverter-defibrillator (!CD) should be following exercise, abstention from competitive sports or stren­
considered in patients with one or more risk factors. The uous aerobic activities is advised for all patients. Patients with
clinical efficacy of !CDs for prevention of sudden cardiac death HCM also should be counseled on the need to avoid dehydration
in HCM has been demonstrated in several large registries. and states of severe peripheral vasodilatation, such as from hot
baths or saunas. Pregnancy generally is well tolerated, although
KEY POINT
precautions should be taken to minimize peripheral vasodilata­
• Placement of an implantable cardioverter-defibrillator tion as for all patients with HCM.
should be considered in patients with hypertrophic car­
diomyopathy with one or more major risk factors for Phannacologic Treatment
sudden cardiac death: prior cardiac arrest, massive Negative inotropic agents are the cornerstone of medical ther­
myocardial hypertrophy, family history of sudden car­ apy for patients with symptomatic obstructive HCM. Negative
diac death, ventricular tachycardia, blunted blood pres­ inotropes (�-blockers, nondihydropyridine calcium channel
sure response to exercise, unexplained syncope.
blockers [verapamil], and disopyramide) depress contractility,
thereby reducing the intraventricular flow velocities that pre­
Management dispose to LVOT obstruction. These agents also lengthen dias­
The major management goals for patients with HCM are allevia­ tole, facilitating more time for ventricular filling. Appropriate
tion of symptoms, risk stratification for sudden cardiac death, drug therapy suffices for most patients with symptomatic
ICD implantation for those at high risk, and family counseling. obstructive HCM, although high doses may be required.

44
Myocard i a l Disease

TABLE 20. Clinical Features Distinguishing Hypertrophic Cardiomyopathy from Athlete's Heart
Feature Hypertrophic Cardiomyopathy Athlete's Heart
Fa m i ly history Positive Negative
El ectroca rdiography Patho l ogic Q waves, T-wave inversions, Absence of these features
conduction defects
Doppler echocardiography Diastolic fil l ing abnormalities Normal d iastol ic fil l i n g
Extent of hypertrophy > 1 5 mm Often :'> 1 2 m m
Pattern of hypertrophy Asymmetric, concentric, or eccentric Concentric
Left ventricu l a r end-d iastolic d i mension <45 m m >55 mm
Gadolinium hyperenhancement o n Present Absent
card iac magnetic resonance i m a g i n g

Objective exercise testing % Predicted peak Vo2 < 1 00% % Predicted peak Vo2 > 1 20% or
>50 m Ukg/min
Genetic testi ng Positive Negative
Eva l uation after period of deconditioning No regression in hypertrophy Regression >2 mm

Vo2 = oxygen consumption.

TABLE 2 1 . Risk Factors for Sudden Death in Patients with Hypertrophic Cardiomyopathy
Risk Factors Comments
Major"
Cardiac arrest (ventricular fi brillation) Portends high rate of recurrence or death ( 1 1 % per year)
Spontaneous susta i n ed VT

Fam i ly h istory of premature sudden death Most predictive if occurs i n a close relative or m u ltiple relatives
U n explained syncope Most predictive if occurs i n young patients, is exertional, or is
recurrent

Left ventricu lar d iasto l i c wall thickness �30 mm

B l u nted increase (<20 mm Hg) or decrease i n systo lic blood More predictive in patients <SO years of age
pressure on exercise

Nonsusta i n ed spontaneous VT Less predictive if very brief (:'>3 beats) a n d asym ptomatic
Heart fai l u re that has progressed to dilated cardiomyopathy with Occurs in 5% to 1 0% of patients with hypertrophic
ejection fraction :'>35% and NYHA class I I or Ill symptoms cardiomyopathy

Possible Risk Factors in Individual Patients•


Myocardial ischemia May be a trigger for sustained ventricular arrhyth mia i n sel ect
patients

Left ventricular outflow obstruction Gradient is modifiable with therapy


Delayed hyperenhancement with gadolinium o n CMR imaging Is com mon and i s not a n independent risk factor. However, severe
extent is l i kely i m portant and ca n be considered to be a n
arbitrating risk factor

Factors Not Predictive


Ventricular arrhyth m ias inducible by electrophysiologic Generally not considered to be of incremental value above
sti mulation noninvasive risk factors

CMR = cardiac magnetic resonance; NYHA = New York Heart Association; VT = ventricular tachycardia.

a Risk factors from Gersh BJ, Maron BJ, Bonow RO. et at; American College of Cardiology Foundation/American Heart Association Task Force on Practice Guidelines. 20 1 1 ACCF/
AHA Guideline for the Diagnosis and Treatment of Hypertrophic Cardiomyopathy: a report of the American College of Cardiology Foundation/American Heart Association Task
Force on Practice Guidelines. Developed in collaboration with the American Association for Thoracic Surgery, American Society of Echocardiography, American Society of Nuclear
Cardiology, Heart Failure Society of America, Heart Rhythm Society, Society for Cardiovascular Angiography and Interventions, and Society of Thoracic Surgeons. J Am Coll
Cardiel. 201 1 Dec 1 3;58(25):e21 2-60. IPMID: 220754691

45
M yo ca rdial Disease

Positive inotropes (such as digoxin), vasodilators, and high­


dose diuretics should be avoided, as these drugs exacerbate
LVOT obstruction. Patients with atrial fibrillation and HCM
should receive anticoagulation therapy with warfarin for
stroke prevention, independent of cardiovascular risk factors.
Although �-blockers are commonly prescribed in these
patients, their use in the absence of symptoms or for preven­
tion of sudden cardiac death is generally not recommended.

Septal Reduction Therapy


For patients with persistent, severe, drug-refractory symptoms
caused by obstructive HCM, septa! reduction therapy with
surgical septa! myectomy or percutaneous alcohol septa! abla­
tion should be considered.
Surgical myectomy is the gold standard therapy for relief
ofLVOT obstruction. In this procedure, a transaortic approach
is typically used to resect the ventricular septum, often
extending the resection to the base of the papillary muscles
(Figure 14 ) . In experienced centers, symptom relief with sur­
gical myectomy occurs in more than 90% of patients with a
low perioperative mortality rate (<1%) . Long-term survival
following myectomy has been found to be comparable to the
expected survival of the general population.
Percutaneous alcohol septa! ablation is an alternative
therapy for septa! reduction and may be particularly consid­
ered as an option for patients with high surgical risk. In this
technique, a slightly oversized angioplasty balloon is placed in
a septa! perforating artery. The perfusion bed is delineated
with contrast, followed by injection of desiccated alcohol. The
alcohol results in a controlled infarction, which decreases
ventricular systole and thereby ameliorates LVOT obstruction.
A major limitation of alcohol ablation is the risk of pacemaker
dependency. Pacemaker dependency occurs more frequently F I G U R E 1 4 . Cardiac magnetic resonance (CMR) imaging o f hypertrophic cardio·
in patients with preexistent conduction disease. Incidence is myopathy before and after surgical myectomy. Prior to surgery, there is systolic anterior
motion of the mitral valve owing to ventricular septaI hypertrophy (arrow) (top panel).
10% to 15% in patients with a normal ECG but up to 50% in
Surgery facilitates direct resection of the basal ventricular septum (arrowheads), lead·
patients with preprocedural left bundle branch block or a
ing to reconstruction of the left ventricular outflow tract and relief of obstruction
wide QRS interval. The decision to implant a pacemaker or (bottom panel). Ao = ascending aorta; lA= left atrium; LV = left ventricle.
!CD in the event of heart block with septal reduction therapy
is based primarily on risk factors for sudden death.
In comparison to surgical myectomy, alcohol septal ablation
has been associated with similar rates of clinical improvement, as based on the inheritable nature of HCM, the potential for
measured by symptoms of heart failure and objective measures of sudden cardiac death, and the demonstrated efficacy of !CDs.
peak myocardial oxygen consumption on treadmill testing. Few Screening consists of physical examination, ECG, and two­
data on long-term follow-up after alcohol septal ablation are dimensional echocardiography (or CMR imaging) , with
available, leading to recommendations that this procedure be screening intervals recommended according to age, symp­
restricted to relatively older patients as the long-term conse­ toms, and family history (Table 22) . Ongoing screening of
quences of the therapeutic infarction of septal ablation are rela­ first-degree relatives who are initially free of the disease is
tively unknown. Current national guidelines recommend that recommended throughout adulthood because of the possi­
surgical myectomy and alcohol septal ablation should only be bility of disease expression at any age.
performed in centers of expertise, in the context of a longitudinal, Genetic testing of patients with HCM can identify patho­
comprehensive care program dedicated to patients with HCM. logic mutations, which can then be used to screen family
members for HCM. Genetic counseling is an important facet of
Role of Genetic Testing and Screening the care of patients with HCM, regardless of whether genetic
All first-degree relatives of patients with HCM should testing is performed. Counseling enables informed decision­
undergo screening for the disease. This recommendation is making about the risks and benefits of testing and facilitates

46
Myocard i a l Disease

TABLE 22. Recommended Screening Intervals for Evaluation of First-Degree Relatives of Patients
with Hypertrophic Cardiomyopathy
Age Group Recommendation

< 1 2 years Screening optional u n l ess:

Presence of symptoms

Fa m ily history of malignant ventricular tachyarrhythmias

Patient is com petitive athlete i n a n intense tra i n i n g program

Other clinical suspicion of early LV hypertrophy


1 2 to 1 8-21 years Every 1 2 to 1 8 months
> 1 8-2 1 years At symptom o nset or at least every 5 years (more frequently in fa m i lies with malignant tachyarrhyth mias or
late onset)

LV = left ventricular.

NOTE: These recommendations are for relatives of patients with hypertrophic cardiomyopathy in whom genetic testing is negative, inconclusive, or not performed.

Recommendations from Gersh BJ, Maron BJ, Bonow RO, et al; American College of Cardiology Foundation/American Heart Association Task Force on Practice Guidelines. 20 1 1
ACCF/AHA Guideline for the Diagnosis and Treatment of Hypertrophic Cardiomyopathy: a report of the American College of Cardiology Foundation/American Heart Association
Task Force on Practice Guidelines. Developed in collaboration with the American Association for Thoracic Surgery, American Society of Echocardiography, American Society of
Nuclear Cardiology, Heart Failure Society of America, Heart Rhythm Society, Society for Cardiovascular Angiography and Interventions, and Society of Thoracic Surgeons. J Am
Coll Cardiol. 201 1 Dec 1 3;58(25):e21 2-60. [PMID: 22075469]

the interpretation of the results, which can include known and heart failure. In rare cases, restrictive cardiomyopathy is
pathologic mutations and likely pathogenic mutations, as well familial, with an autosomal dominant pattern of inheritance.
as variants of unknown significance. Restrictive cardiomyopathy is a diagnosis of exclusion and
Genetic testing is performed as a panel, with the mutations must be distinguished from constrictive pericarditis (see later) ,
associated with the greatest likelihood of pathologic conse­ eosinophilic syndromes, radiation-induced disease, storage
quences being �-myosin heavy chain, myosin-binding protein diseases (such as Fabry disease, hemochromatosis) , as well as
C, troponin T, troponin I, o:-tropomyosin, actin, regulatory light infiltrative cardiomyopathies, such as amyloidosis and sar­
chain, and essential light chain. A negative genetic test result coidosis. Patients with other forms of heart failure can also
does not rule out HCM in patients with phenotypic evidence of have restrictive patterns of diastolic ventricular filling but are
disease, so their immediate family members still should undergo not defined as having a primary restrictive cardiomyopathy.
interval screening. Whereas all patients with HCM can be con­ Patients with restrictive cardiomyopathy may present at
sidered for genetic testing to facilitate diagnosis in family mem­ any age, with symptoms and signs of pulmonary and systemic
bers, the decision to pursue testing is individualized based on congestion. The diagnosis can be considered when there is
likelihood of detecting mutations, patient and family desire, and severe diastolic dysfunction and dilated atria in the absence of
reimbursement concerns. The likelihood of detecting mutations ventricular hypertrophy or cavity dilatation. Systolic function
is increased with a positive family history and a reversed curva­ is preserved in most patients or, at least, is disproportionately
ture morphology of the ventricular septum. In reversed curva­ high given the degree of diastolic dysfunction. Pulmonary
ture, the hypertrophy of the septum is maximal in the hypertension, secondary to diastolic dysfunction, is common.
mid-portion, with relatively less hypertrophy in the apical and Endomyocardial biopsy can be performed for patients
basal segments. To date, genetic test results have not been with suspected restrictive cardiomyopathy to evaluate for infil­
strongly linked to risk of sudden cardiac death, so they should trative disease, such as storage diseases or amyloidosis, when
not be used for risk stratification. clinical assessment or less invasive testing (such as protein
electrophoresis or fat biopsy) is inconclusive. The yield of
KEY POINT
biopsy is low in disorders with patchy myocardial involvement
• All first-degree relatives of patients with hypertrophic such as sarcoidosis.
cardiomyopathy should undergo screening for the dis­
ease, with screening intervals recommended according
Cl
Differentiating Restrictive Cardiomyopathy
to age, symptoms, and family history. from Constrictive Pericarditis
Both restri c t ive cardiomyopat hy and constrict ive pericarditis
present with e levation of d iasto lic pressures and hea rt fai lu re
Restrictive Ca rd i omyopathy that is disproportionate to the degree of systo l i c dysfunction .
Clinical Presentation and Evaluation Distinction of the two di sorders is i mportan t as pericardiec­
,

Primary restrictive cardiomyopathy is an idiopathic disorder tomy w i l l result in symptom relief and i mprovemen t in lon­
characterized by nondilated, poorly compliant ventricles, lead­ gev ity i n patients with constriction . I maging and hemody na mi c
ing to severe diastolic dysfunction, elevated filling pressures, eva l ua tion a re most usef'ul for di ffe re n t iat ing t he se en ti ti es.

47
Myocard i a l Disease

Cl cardiomyopathy with physical exa m i nation can be chal leng­


D istingu ishing constrictive pericard i t is a n d restrictive not be used in patients with restrictive cardiomyopathy. Atrial
fibrillation, with loss of atrial contractile function, may lead to
CONT.
ing. as t hese conditions have s i m ilar signs. In both conditions, decompensated heart failure in patients with restrictive cardio­
peripheral edema with elevation of the jugu lar venous pulse myopathy, but rhythm control may be challenging because of
can be evident. The contour of the j ugular venous pulse dem­ chronic left atrial pressure elevation and dilation.
onstra tes pro m i nent y descents owing to accentuation of The overall prognosis is generally poor in patients with
early ventricu lar fi l l i ng. and may rise (or fail to fal l ) with idiopathic restrictive cardiomyopathy, with a 5-year survival of
inspira t ion because of poor effective operative compliance approximately 64%. Prognosis is affected by functional status,
( Kussmaul sign) . I n both d isorders. i f systol ic function is pre­ and cardiac transplantation should be considered in patients
served, the apical impulse cha ra c teristically is not displaced with severe refractory symptoms.
nor d i ffusely enlarged. In constriction. t he apical impulse
KEY POI NTS
may be d i m i nished. An early d iastolic fil l i ng sound may be
presen t in both disorders. attributable to either a ''k nock" in • Restrictive cardiomyopathy i s characterized by severe
constrictive pericarditis or an S 3 (right- or left -sided) in diastolic dysfunction and, frequently, pulmonary

restrictive cardiomyopathy. Because of severe diastolic abnor­ hypertension.


malities, physical fi ndings of pulmonary hypertension are • Restrictive cardiomyopathy is a diagnosis of exclusion
relatively more com mon in restrictive cardiomyopat hy. and must be distinguished from constrictive pericardi­
Measurement of B type natriuretic peptide may be usefu l ,
- tis, eosinophilic syndromes, and infiltrative diseases.
a s this biomarker is released at high levels i n response to
wal l tension with restrictive cardiomyopathy (2:400 pg/m L
[400 ng/ L]) but is usua l ly normal o r only mi ldly elevated i n
Ca rdiac Tu mors
constriction (<100 pg/m L [ 1 0 0 ng/L]) . CT imaging may dem ­
Tumor Types
onstrate pericardia! t hickening and possibly calcification in
constrict ive pericard itis, although t he pericardium can also be Cardiac tumors are usually metastatic, arising from carcinoma
normal in t h ickness i n a small subset of patients. of the lung, breast, kidney, esophagus, or liver, or from blood
A hemodyna m ic evaluation is frequently required to dis­ dyscrasias, such as leukemia or lymphoma. Malignant mela­
t i nguish restrictive cardiomyopat hy from constriction. As a noma is commonly associated with cardiac metastases. I n
myocardial disease, restrictive cardiomyopat hy may predomi­ autopsy series, cardiac involvement occurs in 1 0 % t o 1 5 % of all
nantly affect left ventricular fil l i ng. resul ting in elevated left patients with metastatic malignancy.
ventricular end-diastolic pressure and resultant pulmonary Primary cardiac tumors are exceedingly rare (0.02% to
hypertension. In patients with constriction the total cardiac
.
0.06%) and are mostly benign. The most common primary car­
volume is fixed by the rigid pericardium. As a resu lt. there is diac tumors in adults are myxomas, followed by lipomas and
enhancement of vent ricular interdependence and resultant papillary fibroelastomas. Myxomas are connective tissue tumors
equalization of rigbt and left ventricular diastolic pressures and with cells encompassed within mucopolysaccharide stroma,
reciprocal changes i n the fi l l i ng and stroke volume of the right with an appearance that may be villous, smooth, or friable.
and left ventricles during i nspiration (see Pericard ia! Disease) . Myxomas may arise as part of the Camey complex, an autosomal
These reciprocal changes during inspiration can be detected by dominant disorder associated with pigmentation abnormalities
either echocardiography or cardiac catheterization and are not (blue nevi), cardiac and extracardiac myxomas, schwannomas,
present in patients with restrictive cardiomyopathy. Cl and endocrine tumors. Lipomas are fatty cell tumors that typi­
cally arise in the subendocardial layer. Papillary fibroelastomas
Management are pedunculated tumors with frond-like arms, usually present
The primary goals in the management of restrictive cardiomyo­ on the left-sided valves of the heart. Malignant sarcomas of vari­
pathy are optimization of diastolic filling and relief of pulmo­ ous types (angiosarcoma, fibroma, rhabdomyosarcoma, leio­
nary and systemic congestion. Negative chronotropic agents, myosarcoma) also have been described with cardiac involvement,
such as calcium channel blockers and �-blockers, are benefi­ accounting for approximately 15% of primary cardiac tumors.
cial, as they lengthen the diastolic filling period and improve
myocardial relaxation. Loop diuretics may be utilized; however, Clinical Presentation and Evaluation
these agents should be used judiciously owing to the underly­ Manifestations of cardiac tumors depend on their location
ing poor ventricular compliance, which leads to the need for a and size. Cardiac tumors that arise near or involve valves may
relatively high ventricular filling pressure to maintain forward obstruct blood flow or cause regurgitation, leading to pulmo­
cardiac output. ACE inhibitors (or angiotensin receptor block­ nary or systemic congestion. The auscultatory findings asso­
ers) may improve diastolic function, but the benefit of these ciated with left atrial myxoma are classically described as
drugs in patients with heart failure due to restrictive cardio­ resembling those of a mitral stenosis murmur with an
myopathy is less certain than in patients with systolic dysfunc­ accompanying sound of a "tumor plop"; this finding, how­
tion. Digoxin, which increases intracellular calcium, should ever, occurs in the minority of patients (10% to 15%) . Patients

48
Myocard i a l Disease

with a myxoma may report constitutional symptoms, such as


fever, anorexia, and weight loss, which are likely related to
tumor cytokine production. Embolization, from either tumor
fragments or associated thrombi, may cause neurologic
symptoms or other systemic sequelae. Such embolization is
relatively more common with villous or friable myxomas and
papillary fibroelastomas. In a series of 74 patients with left
atrial myxoma, systemic embolization occurred in 12%.
Cardiac tumors, particularly lipomas and malignant sarco­
mas, may either directly invade or be located within myocar­
dium, causing heart block or impairment of contractility.
Pericardial effusion with or without tamponade also is a
common manifestation.
Imaging (echocardiography, CT, or CMR imaging) is needed
for the detection and evaluation of cardiac tumors. Myxomas
characteristically are pedunculated and gelatinous, arising most
commonly in the left atrium with the stalk adherent to the fossa
ovalis (85% of cases) (Figure 15 ) . Papillary fibroelastomas, whose
origin may be related to trauma, typically involve left-sided
cardiac valves or the left ventricular outflow tract. Papillary
fibroelastomas have a central core stalk with fronds and often
F I G U R E 1 6 . Gross pathologic specimen o f a papill ary fibroelastoma.
are mobile, with an appearance resembling a sea anemone
Image courtesy of Dr. William D. Edwards, Department of Pathology, Mayo Clinic.
(Figure 16) . Lipomas typically are found within the subendocar­
dium, although involvement of valves has been reported.
Sarcomas frequently infiltrate the myocardium. Although sar­
to symptoms and risk of embolization. Myxomas should be
comas may occur in the right atrium, myxomas predominate as
resected after diagnosis owing to the risk of embolization and
the cause of primary cardiac tumors in both atria.
cardiovascular complications, including the potential for sud­
den death. Survival for patients after surgical resection of
Management
myxoma is comparable to that of healthy individuals. Myxomas
The need for surgical resection is dependent on the size, loca­
may recur, with a reported incidence varying between 5% and
tion, and malignant nature of the cardiac tumor, in addition
13%, with recurrence being more common in younger patients
and those with Carney complex or a positive family history.
Routine surveillance with imaging is indicated to monitor for
recurrence, which usually occurs at the original site of
the tumor.
Surgical resection is often considered for papillary fibroe­
lastoma, particularly if the lesion is large (>10 mm) , highly
mobile, or associated with systemic embolization.
Treatment of tumors with myocardial involvement is
challenging. I n particular, sarcomas are difficult to treat
owing to their widespread, invasive nature and the high
recurrence rates despite surgical resection. Survival is
improved when there is complete resection and no metastatic
involvement (median survival, 15 to 18 months versus 2 to
5 months) . Cardiac transplantation using an allograft or auto­
graft may be a therapeutic option in selected patients. In the
autograft procedure, the tumor is resected from an excised
heart, followed by reconstruction and auto-implantation of
the heart into the patient.

KEY POINT
F I G U R E 1 5 . Transesophageal echocardiography across the right atrium (RA) • Myxomas should be resected after diagnosis owing to
and left atriu m (LA) demonstrates a large mass (arrow) attached to the atrial sep· the risk of embolization and cardiovascular complica­
tum i n proximity of the Iossa ovalis (asterisk); the mass was surgically removed tions, including the potential for sudden death.
and proved to be a left atrial myxoma.

49
Arrhyth mias

although they are helpful in specific situations (see Table 23) ,


Arrhythmias including the use of procainamide in patients with preexcited

Antia rrhyth mic Medications atrial fibrillation. Class IC agents are avoided in patients with
coronary artery disease and structural heart disease as they
Antiarrhythmic medications are used t o prevent recur­
have been shown to cause proarrhythmic activity (ventricular
rent arrhythmias and maintain sinus rhythm. Although anti­
arrhythmias) and increase mortality. Class II agents
arrhythmic medications have historically been organized
(�-blockers) and class IV agents (nondihydropyridine calcium
according to their predominant mechanism of action using
channel blockers) are frequently used to slow heart rates in
the Vaughan-Williams classification system (Table 23) , it is
increasingly recognized that this nomenclature system has patients with supraventricular or atrial arrhythmias; however,

limitations because most antiarrhythmic drugs have several they should be avoided in patients who have atrial fibrillation

mechanistic actions. with preexcitation. Class III agents are used to treat atrial and
The membrane-active antiarrhythmic agents (class I and ventricular arrhythmias. These agents are cleared by the kid­
class III) principally affect ion channels. Class I agents decrease neys and should be avoided in patients with significant chronic
impulse formation and speed of depolarization and are often kidney disease owing to increased toxicity and proarrhythmia.
used in patients with atrial arrhythmias and no structural Because class III agents lead to QTc-interval prolongation, ini­
heart disease. Several class IA agents are used less frequently, tiation of this therapy is usually done on an inpatient basis

TABLE 23. Antiarrhythmic Medications


Classification Mechanism of Action Individual Agents/ Effects Use
Examples

Class IA Sodium channel Quinid ine, Decreases speed of Pre-excited atrial


blockade with some procainamide, depola rization and fibril lation
potassium channel disopyramide prolongs repola rization (procainamide),
blockade Brugada syndrome
(quinidine), SVT, atrial
fibrillation, and
ventricular arrhythmias
Class I B Sod i u m channel Lidocaine, mexiletine, Decreases speed of Ventricular arrhyth mias
blockade phenytoin depola rization

Class IC Sod i u m channel Flecainide, Decreases speed of Atrial fi bri l lation, SVT,
blockade propafenone depola rization and ventricular a rrhyth mias;
shortens repola rization avoid with CAD or
structural heart disease.
Class I I �-adrenergic blockade Metoprolol, Decreases sympathetic Rate control of atrial
propranolol, carved ilol, tone; suppresses arrhythmias, SVT,
atenolol, bisoprolol automaticity, sinoatrial ventricular arrhythmias;
conduction, and AV avoid if pre-excitation is
cond uction present.
Class I l l Potassi u m channel Sotalol, dofetilide Prolongs action Atrial fibrillation, atria l
blockade potential d u ration flutter, ventricular
arrhythmias; avoid in
CKD.
Class IV Ca lcium channel Verapamil, diltiazem Sup presses sinoatrial SVT, rate control of atrial
blockade and AV conduction arrhythmias, triggered
(nondihydropyrid i nes) arrhythmias (e.g.,
outflow tract VT); avoid
if pre-excitation is
present.
Multichannel blockers Severa l, incl u d i ng Amiodarone, Multi ple mechanisms, Atrial arrhyth mias,
potassi um, sod i u m , and d ronedarone although they a ct ventricular arrhythmias
calcium channel principally by extending
blockade repola rization
Adenosine receptor A1-receptor agonist Adenosine Slows or blocks Termination of SVT
agonists sinoatrial and AV nodal
conduction
Cardiac glycoside Increasing vagal a ctivity Digoxin Slows AV nodal Rate control of atrial
conduction fibrillation

AV = atrioventricular; CAD = coronary artery disease; CKD = chronic kidney disease; SVT = supraventricular tachycardia; VT = ventricular tachycardia.

50
Arrhyth m i a s

with regular assessment of the QTc interval. Patients taking malities Lhal require urgent in tervention . Evaluation includes
class I I I agents should avoid other QT-prolonging medications, a carefu l history. a focused laboratory evaluation (including an
and serum potassium and magnesium levels should be assessment of thyroid function) . resting 12-lead electrocardio­
checked regularly. gram (ECG) . exercise treadmill testing to assess the heart rate
Amiodarone, a multichannel blocker, is among the most response to exercise (chronotropic competence) , and ambula­
commonly used antiarrhythmjc medications. It is frequently tory ECG monitoring based on the nature and frequency of the
used to treat atrial fibrillation in older persons and to prevent patient's episodes or symptoms (see Diagnostic Testing in
recurrent ventricular tachycardia. Amjodarone is the preferred Cardiology) . Rarely. electrophysiologic testing can be used to
antiarrhythmic agent in patients with structural heart disease help ascertain i f sinus node dysfunction is present. Cl
and heart failure. Although hlghly effective, amiodarone has
multiple toxicities. Amiodarone therapy is associated with Sinus Bradycardia
risks for thyroid toxicity, hepatotoxicity, lung toxicity, photo­
Sinus bradycardia (sinus rhythm with a heart rate <60/min)
sensitivity, corneal and lenticular deposits, optic neuropathy,
may be appropriate in several situations, i ncluding i n
and other neurologic adverse effects. Patients on amiodarone
trained athletes or during sleep, when t h e heart rate may
require routine monitoring of thyroid and liver function, pul­
fall as low as 30/min. The most common intrinsic cause of
monary function testing at baseline and with symptoms, and
inappropriate or pathologic sinus bradycardia (sinus node
periodic ophthalmologic evaluation. Amiodarone interacts
dysfunction) is age-related myocard ia l fibrosis in the vicin­
with several medications. Patients on amiodarone require
ity of the sinus node. The most common extrinsic cause o f
lower doses of warfarin, statins, and digoxin. Dronedarone is a
sinus bradycardia i s medication effect. S i nus node dysfunc­
multichannel blocker used to treat atrial fibrillation. Owing to
tion can also present with chronotropic i ncompetence, and
increased mortality i n patients with heart failure or permanent
this is frequently overlooked. Other, less common, causes of
atrial fibrillation, its use should be restricted to patients with
sinus node dysfunction include right coronary ischemia,
i ntermittent atrial fibrillation and no overt heart failure.
i ntracranial hypertension, postsurgical scarring a fter car­
Digoxi n is an oral positive inotropic agent that acts on the
diothoracic surgery, and infiltrative or inflammatory d isorders
sodium-potassium exchanger and has vagal properties that
(such as sarcoidosis ) .
lead to decreased atrioventricular (AV) nodal conduction. As a
result of its vagal mechanism, it primarily controls the heart
rate at rest and is less effective during activity. Adenosine is an Atrioventricular Block
A1-receptor blocker that can inhlbit AV conduction. Adenosine AV block is classified as first degree, second degree, or third
is frequently used as a therapeutic agent to terminate supraven­ degree. First-degree AV block is characterized by prolonged AV
tricular tachycardia. conduction, which manifests on the ECG as a PR interval
greater than 200 msec. First-degree AV block is not a true
KEY POINT
block because all P waves conduct to the ventricles. It has been
• Calcium channel blockers and �-blockers are often used associated with an increased risk of atrial fibrillation, pace­
to treat supraventricular and atrial arrhythmias; how­ maker implantation, and all-cause mortality in long- term
ever, these agents should be avoided i n patients who follow-up.
have atrial fibrillation with preexcitation. ln second-degree AV block, some P waves conduct to the
ventricle and some do not. There are two forms of second­
degree AV block. When progressive PR prolongation is
observed prior to a blocked beat, second-degree Mobitz type 1
Approach to the Patient
(Wenckebach block) is present. Second-degree Mobitz type 1
with Bradyca rd ia
c::J Clinica l Presentation
block is characterized by grouped beating and progressive
shortening of the R-R intervals. Mobitz type 1 block is almost
Symptoms ofbradycardia (heart rate less than 60 /min) include always localized to the AV node. It generally carries a benign
fatigue, exertional intolerance, dyspnea, light headedness. and prognosis and frequently i mproves with exercise or i ncreased
syncope. Bradycardia can result from pathology in the sinus sympathetic tone.
node. the AV node, or the His-Purkinje system. Physicians When the PR i nterval is constant prior to nonconducted P
should maintain a high suspicion for reversible causes of waves, the second-degree block is termed Mobitz type 2 block.
bradycardia. including elevated i ntracranial pressure. hypo­ When 2:1 block is present, Mobitz typ e 1 versus type 2 block
thyroidism, hyperkalemia. Lyme disease. and medication cannot be di fferentiated. Mobitz type 2 block usually repre­
effects (most common ly AV nodal blockers, especi a l ly sents block lower in the conduction system and has a higher
�-blockers and digoxin) . risk of progression to complete heart block.
The diagnostic evaluation of bradycardia i ncludes ( 1 ) Third-degree AV block, or complete heart block, is defined
establishing a correlation between the rhythm (bradycardia) as the failure of any P waves to conduct to the ventricles, and
and symptoms and (2) excluding severe conduction abnor- it i s characterized by AV dissociation on the ECG.

51
Arrhyth m i a s

Cl Pacemakers
P<.1 t ienls w i t h i n t ravc n t ri c u l a r co n d u c t i o n delays have 8

Pace m a kers a re i n d i ca Lcd i n pal ien Is wi I h sy m p l o m a l ic brady low risk or p rogression to com p l c l e hearl block ( I %-3'Y., a n n u
carcl i a in t h e �1bscnce or 8 reversi b l e cause, hence l h c i m ror a l ly) a n d clo nol req u i re perma n c n l pa c i ng. W h e n a pa t i e n l

l <J nce o r cst abl i s h i ng sy m p toms when cva l u <JL i ng pal icn ts w i t h develops new-on set cond u c l i o n d i srnsc i n L h e set t i ng o r a n

braclyca rd i a . In pa l ien ls w i t h m i n i ma l sy m pl o m s, a pcrs islc n t aculc coronary sy n d ro me. t<.:m porary pac i ng 1rn1y be req u i red.

rcs l i ng hc<Jrl rn l c below 40 m i n is a lso co nsidered a n i nd ica but deci s ion s on permanent pac i ng s h o u l d be delayed u n i i i a

L i on for per m a n e n t paci ng. Pacemakers a l so a rc i ncl i c a l cd i n pa l i c n l has been revase u la r i i'.cd a n d s t ab i l i zed to determ i ne

pa l i c n t s w i l h evidence or AV cond uct i o n d i s t urb<Jnccs t ha l whet her t h e a rrhy t h m i a persists.

have a h igh I i kcl i hood o r p rogress i ng to com p l e l e he�1 rL block Pa t ie n l s w i t h pacema kers who req u i re su rgery s h o u l d

or Ii le L h rea t c n ing sudden a sysl olc. I llC.I ical ions lcir perm a ncnt have a preopera L ive device eva l ua t ion lo determ i n e whet her

pacemaker i m p l a n l a t ion <Jrc s hown i n Table 24. preopera t ive reprogra m m i ng o r the dev i ce i s necessa ry.
/\ I L hough "MRI con d i t ional" pacemakers a rc now ava i l a b l e. L hc
TABLE 24. Selected Indications for Permanent Pacing presence or a pace m a ker re m a i n s a co n l ra i ncl ic a l i o n to MRI
sca n n i ng lor most pa t i e n ts.
Symptomatic bradycardia without reversible cause
There a rc severa l Lypes or i m p l a n lecl card i a c devices, w i l h
Asymptomatic bradycardia with significant pauses (>3 seconds
va rious ca p a b i I i t i cs . I m p l a n tecl c a rd iac e l ec t ro n i c devices
i n sinus rhythm) or persistent heart rate <40/min
i nc l u d e i m pl<i n ted loop mon i to rs. p8<.:c m a kers. i m p l a n table
Atrial fibri l l ation with 5-secon d pauses
ca rcl i overtcr- de libri l l a t o rs ( I CDs ) . a n d card i<Jc resy n c h ro n i i'.a
Asymptomatic complete heart block or Mobitz type 2 second­
l io n dev i ces. Wi l h L h c exce p l i o n or subcula neous JCDs. w h i c h
degree atriove ntricular block
do not u t i l ize i n t ra c a rd i ac l e a d s . a l l I C Ds a l so have pacemaker
Alternating bundle branch block
fu nct ions. Table 25 reviews l h e V<J rious types or i m p l a n t ed

TABLE 25. Cardiac Implantable Electronic Devices for Treatment of Cardiac Rhythm Disorders
Functions
Device Components Indications Pacemaker Antitachycardia Defibrillation
Function Pacing

Pacemaker Pulse generator and Sinus node Yes No No


i ntravascu lar leads dysfu nction,
(single- or dual- atrioventricular
chamber) block, or other
causes of non-
reversible
symptomatic
bradycardia
I m p l a nta ble Defi b ril lator and To provide Yes Yes Yes
cardioverter- intravascu lar leads continuous
defi b ri l l ator (si ngle- or dual- monitoring and
cha m ber) treatment of
ventricular
a rrhyth mias
Subcutaneous Defi brillator and a To provide No No Yes
i m plantable single lead that are conti n u ous
cardioverter- entirely under the skin monitoring and
defibri l lator (extravascular); no treatment of
transvenous leads ventricu lar
a rrhyth mias
Cardiac Pulse gen erator and To restore electrical Yes No No
resynchronization intravascu lar leads, syn chrony between
therapy-pacing (CRT- P) including a pacing lead the ventricles in
i n the coronary sinus to patie nts with heart
pace the left ventricle fa i l u re
Cardiac Defi bril lator and To restore el ectrica l Yes Yes Yes
resynchronization intravascu lar leads, synchrony between
thera py-defi bril lator including a pacing lead the ventricles in
(CRT-D) in the coronary sinus to patients with heart
pace the left ventricle fa i l u re and to
mon itor and treat
ventricular
arrhythmias

52
Arrhyth m i a s

CJ i n d icat ions. Cl
C<l!Tl i a c elcc l ro n i c devices. l h e i r !'u n c t i o n s . a n cl L h e i r general
Supraventricu lar Tachyca rd ias
CONT.
KEY POINT
Clinical Presentation
Supravc n t ri c u l a r tachycard ias ( SVTs) a rc a group or a rr hy l h -
CJ
• A pacemaker is indicated for symptomatic bradycardia 111 ias t h a t a rise i n a t ri a I t i ssue or t h e /\V nocle. Because conduc­
without a reversible cause as well as for atrioventricular t i o n o l' su p ravc n t ri c u l a r i m p u l ses below t he /\V node i s
conduction abnormalities that are likely to progress to conducted normal ly. t he l�CG i n S V T usu<1 l ly revea l s a n a 1-row­
complete heart block. complcx t;1chyca rd i a . a l t hough t he QRS complexes ca n be
w i de ( > 1 20 msec) i n t h e presence o l' b u n d l e bra nch b l ock,
abe rra ncy. paci ng. or a n t e rograde accessory pat hway conduc­

CJ Ap proach to the Patient


t i o n ( a n t i d ro m i c tachycard i a ) .
SVTs include abnormal elect rical act i v i ty a rising i n t h e
with Tachyca rd ia a t r i u m (pre m a t u re a t ria l co n t ra c t i o n s . t a c hyca rd i a . a t ri a l
Pa l i e n t s w i t h sy m p l o1ml l i c lachyc a rc l i a ol'tcn report pa l p i t a 11bri l l a t io n a n d lhi l t cr. m u l l i local a t ri a l lachycarclia) or AV
l io ns . l i g h t h eaded ncss or d izzi ness. chest d iscom lon . clyspne<l. node (j u n c t i o n a l t achycard i a , AVN RT. a l rioven t ri c u l a r reci pro ­
exert i o n a l i n to l erance. or syncope. Some pa t i e n t s a re asy m p ca t i ng tachyccm i i a [ i\VRT I ) . Because t hey a re so com mon a nd
Loma t ic a n d a rc l o u n d t o have a rrhy t h lll i <l s incide n ta l ly d u ri ng i()r t h e pu rpose or t h i s rev iew. a t ri a l f'ibril l <l l io n a nd a t rial l'l u t
m o n i toring i n t he sel l i ng or hosp i t a l iz<1 i i o n or ol her medical ter a re d i scussed sep a rate ly : L h e rest o r t h i s sec t i o n w i l l locus
c a re. The most i m po rt a n t part o r t h e eva l u a t i o n i s t h e docu ­ exclusively o n t h e ot her S\/Ts.
men ta t io n or tachycard ia a n d corre l a t i o n w i t h sy m p t o ms (see SVT can occur in a l l age groups bul is l'req u en t ly e nco u n ­
D i agnos t i c Test i ng in C1rd i o l ogy) . In add i t i o n to a h i story a n d tered i n younger pa t i e n t s . SVT i s m o re common i n wo men
p hys ical exa m i n <1 l io11. a l l p a t i e n t s w i t h l<lchycarcl i a shou lcl t h a n men <illd usu<1 l ly occurs w i t hout st ru c t u ra l heart d i sease,
have a res t i n g 1 2- lead ECG. Most pa t ie n t s w i t h lnchyca rd ia a l t hough t h i s s h o u l d be evalu<llecl w i t h an ech ocarcl i ogrn m .
should undergo echoca rd i ography l o exclude t he presence o r Pal i c n l s w i t h SVT o rt e n have repea ted episodes or t achyca rd i a .
sl ru c l u ra l heart d i sease a n d t hy ro i d !'u n c t i o n eva l u a t i o n . Pa t ie n t s may have pa l p i t a t ions. a sensat i o n o l' pou n d i ng i n t he
I n bo t h hosp i t a l a n d a m b u l a t o ry set t i ngs. s i n u s ta chyca r­ neck. l�1 t iguc. I igh t headed n css. chesl cl iscom fort. clyspne<J,
d i a (si n u s rhy t h m w i t h heart ra t e > 1 00 / m i n ) is t he mosl com ­ prcsy n copc, a n d . l ess common ly. syncope.
m o n ly encoun tered t achyca rd i a . S i nus tachyccml ia is usu a l ly The l ·:CG cl ass i li ca l i on ol' SVT i s usu;1 l ly based o n t he rela ­
caused by physio logi c d ist ress. i nclud i ng pa i n . lever. <J ncm i a . or t io n s h i p 0 1 · t he P wave a n d t h e QRS com plex . I n s h o r t- R P
a n x ie ty. The eva l u a t i o n a nd l rea l m e n t o f sinus tachyca rd ia a re t achyca rd ias ( R P i n l erv<l l < P R i n t e rva l ) . t h e P wave closely
c l l rectccl at t he u n derly i ng et i o l ogy. Sign i fi ca n t s i n us t ac hyca r­ I(> I lows t he QRS complex. I n long R P tac hyca rd ias ( R P i n terva l >
d i a i n a cri t ica l ly i l l p a t i e n t is a worrisome li n d i ng as it usua l ly PR i n terva l ) . t h e P wave is m o re t h a n h a l l ' t h e d i s t a nce between
i net ic:llcs <lciva need physiologic compro m i se. i nc l u d i n g res pi ra­ t he QRS co m p l exes. Sh ort R P lachycarcl ias i nclude typical
tory l� 1 i l u re. insu l'llcicnt card i ac o u t p u t . or severe i n lcc t i on . 1\V N RT. i\V RT. a n cl j u ncl ional t a c hycard i a . J u n c l ional t a c hycar­
Older pa t i e n t s w i t h pa l p i l a t i o n s a rc m o re l i ke ly l o have d i as a rc less common i n acl u l l s . b u t L iley can occ u r i n pat ien t s
a t ri a l libri l l a t io n . at ri<J l l'l u t tcr. or ve n t ri c u l a r ta chycard i a (VT) . w i t h d i goxi n i n t o x i cat ion a ncl ot her con d i t i o ns . Lo ng- R P Lach
A l t hough VT is o f'tcn assoc i a t ed w i t h h c m ody n a m i c com pro ycarcl ias i nclude a typical AV N RT, s i n us tachycard i a . a l rial lach ­
m i sc, VT is o rten we l l tolera ted. whereas m a ny 1x1 t i e n l s h <we ycard i a . a n d t h e perm a n e n t lorm or j u 11 c t i o n�11 reciproca t i ng
he mody n a m ically s ign i l1ca n t supraven t ri c u l a r tachycard i a or t <Jchyca rd i a .
a t ri a l a rrhy t h m i as. Therefo re. vital signs a rc n o t hel p l'u l in [pi sod cs o l ' SVT e<l l l o l'len be t e r m i n a ted w i t h V;1 lsalva
d e te rm i n i ng t he n a t u re o f' a n a rrhyt h m i a . ma ncuvers (bca ring dow n ) . ca rot id s i n u s massage. or !�Kial
I n younger persons w i t h tachyca rdic sy mptoms. suprnven i m mersion i n cold wa ler. Adenos i n c can be used t o t erm i n a t e
t ri c u l a r t achycard i as a re more co m m o n . i n c l u d i ng i\V nodal S V T a n d to h e l p d i agnose L h e et io logy. Te rm i na t i o n w i l h aden ­
ree n t ra n t tachyca rd ia (AV N RT) a nd accessory pat hway med i osi n e orten suggcsls /\\/ noclc clcpcndence (AV N RT a ncl AVRT) .
a l ed tachyc a rd i a . Pa t i e n t s w i l h an accessory pa t hway orten w h e reas con t i nued a t ri a l act i v i ty ( P w<ives) d u ring AV block
have ev idence o l' a n t erograde concluc t io n a n d �1 d c l t :.1 wave ca n help iden t i l'y a t ri a l l l u t ter a nd a t r i a l t a c hyca rcl i<l. Cl
on [CG .
i\L rial a ncl vcn l ri c u l a r ectopy a rc present in m a ny i i' n o t Premature Atrial Contractions
most persons. The l'requency ol' ectopy and symptoms usua l ly and Atrial Tachycardia
d i c t a t e bot h t he work-u p ;1ncl subseq u e n t m a n age m e n t . Cl Atrial ectopy can be isolated (premature atrial contractions
K EY P O I N T [PACs] ) , occur in salvos, or be sustained (atrial tachycardia) .
PACs are extremely common, and the frequency increases
• I n addition to a resting electrocardiogram, diagnostic
with age. Only 1 % of persons in the general population have no
testing for most patients with tachycardia should include
PACs during ambulatory ECG monitoring. However, PAC bur­
an echocardiogram and evaluation of thyroid function.
den is associated with increased risk of atrial fibrillation.

53
Arrhyth m i a s

Symptomatic PACs are typically treated w i t h P-blockers or atrium over the fast pat hway (slow-fast) . This leads to a short
calcium channel blockers. RP interval with a ret rograde P wave inscribed very close to
Atrial tachycardia can occur in patients with or w i thout the QRS complex. The closely coupled retrograde P waves may
structura l heart disease; when symptomatic, first-l i n e treat­ be buried in t he QRS complexes and may not be visible. or
ment i s a P-blocker or nondihydropyridine calcium channel t hey may appear as a pseudo R' wave in lead V 1 and a pseudo
blocker ( d i l t iazem or verapam i l ) . Second- l i n e treatment S wave in the i n ferior leads. In atypical AVN RT. conduction
i ncludes catheter ablation or antiarrhythrnic drug therapy. I n goes clown the fast pathway and returns to the atrium via t h e
genera l , success rates for ablation o f atri a l tachycardia are slow pat hway ( fast-slow) : t his leads to a l o n g R P i nterval.
lower t h a n t hose for other SVTs. Rare ly. AVN RT can involve conduction over more t han one
Multifocal atri a l tachycardia, characterized by multiple slow pa t hway (slow-slow AV N RT) .
(2'.3) P-wave morphologies and a heart rate greater than 1 00/ Beyond acute term ination with physical maneuvers or
m i n , i s frequently seen in patients w i t h end-stage COPD. adenosine. treat ment to prevent recurrent AVN RT incl udes AV
Treatment is usually directed at t h e u nderlying etiology and nodal blocking t herapy w i t h P- blockers or nondi hyclropyri­
electrolyte d isturbances, a l though P-blockers and calcium dine calcium channel blockers. Patients who have recurre n t
nonclihyclropyri cl i n e calcium channel blockers can be AVN RT or do not tolerate or prefer to avoid long-term medical
used cautiously. therapy are usually referred for catheter ablation, which has a
h igh success ra te. The major risk of' ablation is a 1 % risk of'
c::J Atrioventricular Nodal Reentrant Tachycardia injury to the AV node requiring pacemaker i mplantat ion .
AVN RT is t he most com mon type of SVT. account ing for two
t h i rds of a l l patients w i t h SVT (excl uding a t rial fibr i l lat ion Atrioventricu lar Reciprocating Tachycardia
and a t rial nu tter) . AV N RT is caused by reentrant conduct ion AVRT is an accessory pathway (bypass tract ) -mediated tachy­
w i t h i n t he AV node, util izing both the fast and slow pat hways cardia . Accessory pat hway conduction is often observed as
(Figure 17) . In typical AVN RT, t he electrical conduction goes preexcita tion on ECG. Because of' early ven t ricu lar activa t ion
clown the slow pat hway and conducts back up toward the over the accessory pat hway. t he P R interval is shortened and
the i n itial part of the QRS complex is sl urred (del ta wave)
because of ventricular depolarization adjacent to the pat hway.
In AVRT, conduction is anterograde over t h e AV node (ortho­
clromic AVRT) or anterogracle over the accessory pathway
(ant idromic AVRT) . Orthocl romic AVN RT. the most common
(To the atrium) type of' AVRT (more than 90% to 95'X, of cases) is char2cterized
by a na rrow QRS complex resu l t i ng from conduct ion over t he
AV node 2nd t he H is-Purkinje system. A n t i d romic AVRT is
characterized by a wide. sl urred QRS complex resu l t i ng from
conduction over t he bypass trnct and act ivation of the ven t ri­
cle w i t hout use of t h e specia lized conduction system .
Adenosine c2n be given to termi nate ort hodromic AV RT:
however. adenosine or other AV nodal blockers are con t rn i n­
clicated in preexcited atrial fibri l lat ion and a n t iclrom ic AV RT.
AV nodal blockade i n patients w i t h t hese rhythms can pro­
mote rapid conduct ion down t h e bypass tract and induction
of ventricu lar fi bri l lation (VF) .
Pat ients w i t h evidence of preexci tation on t heir resting
ECG (delta wave) and symptomatic SVT have Wolff-Parkinson­
White (WPW) syndrome. Up to one third of patients with WPW
syndrome have or w i l l develop atrial fibri l lation. Rapid con ­
duction over an accessory pa t hway i n atrial fibri l l a t ion can lead
to VF and sudden cardiac death (SCD). a l t hough t h is is a rela­
(To the ventric le)
t ively rare event. Risk factors for VF in WPW syndrome include
documented AVRT. multiple bypass tracts. Ebstein anomaly
F I G U R E 1 7 . Mechanism of typical atrioventricu lar nodal reentrant tachycar· (right heart enlargement and severe tricuspid v;1Jve regurgita­
d ia. The slow pathway has a short refractory period, a n d the fast pathway has a tion) . and a rapidly conducting accessory pathway. WPW syn­
long refractory period. The blue line represents anterograde conduction down the
drome is often seen in patients with Ebstein anomaly.
slow pathway; conduction does not occ u r down the fast pathway because it is
refracto ry. The yellow line represents impu lse conduction into the ventricle and In genera l . eva lu ation of a pa tien t with preexcitation
retrog rade u p the fast pathway, which is no longer refracto ry. The red line repre· i ncl udes a 12- lead ECG. echocardiogra m , ambula tory ECG
sents completion of the circuit with activation of the atria and ventricles. moni toring. and an exercise stress test . Stress testing is an

54
Arrhyth m i a s

Cl e ffective means or noninvasive risk strat i ficat ion for patients At rial 11bri l lation i s classified a s first-detected. paroxys­
with preexci tat ion. Loss of preexcitation duri ng exercise gen­ mal. persistent, or long-standing persistent atrial fibrillation.
CONT.
erally indicates low risk. Electrophysiology ( EP) test i ng can Paroxysm<1 l atrial fibrillation starts and stops spontaneously.
help determine rapidity of conduction and risk for sudden Persistent atrial fibril lation lasts for 7 clays or more and requires
death : it also can help local ize t he pat hway and facili tate cath­ elect rical or pharmacologic carc!ioversion. Long-stand ing per­
eter ablation. which has a h igh success rate (alt hough success sistent atrial fibri llation is persistent atrial fibril lation t hat is
depends on the locat ion of the bypass t ract) . In general. cath­ more than J year i n duration.
eter ablation is first- li ne t herapy for patients with preexcita­
t ion a nd symptoms. Antiarrhyth mic agents are reserved for Acute Management
second-line t herapy. pa rticularly i n patients with accessory Bot h acute and chronic management or atrial fibri llation are
pat hways in close vicinity to t h e AV node. based on t h ree t herapeutic goals: ( L ) preve n t i ng stroke.
Management of asymptomatic preexcitat ion on ECG is (2) con t roll i ng the heart rate (preventing tachycardia/rapid
con t roversial. I n t he absence of symptoms, however. i nvasive vent ricular rates). and (3) symp to m relief". Once a diagnosis of
testi ng is genera l ly not required. u nless the patient has a h igh­ at rial fibri l lat ion is made. a search for reversible causes should
risk occupation , such as an airl i ne pilot or bus d rive r. Cl be completed. including an evaluation of t hyroid function .
Pat ients with atrial fibri llation should undergo screening for
KEV POI NTS
sleep apnea with more extensive testing if t he clin ical h istory
• Therapeutic options for prevention o f recurrence o f
is suggest ive. An echocardiogram should be obtai ned to inves­
atrioventricular nodal reentrant tachycardia include
t igate potential valvular or other structural heart disease.
atrioventricular nodal blocking drugs and catheter
ablation.
Acute Anticoagulation
• First-line therapy for Wolff-Parkinson-White syndrome In patients wi l h newly discovered atrial fi b r i l lation in whom
(preexcitation with symptoms) is catheter ablation. cardioversion will not be perlormed. i nst itutio n of i n t rave­
nous a n t icoagulation is usua l ly not necessary. I n t hese
pat ien ts. oral a n ticoagulat ion can be started based on risk
factors (see Long-Term M a nagement) . I f cardioversion is
Atria l Fibril latio n
pJ,1 11 ned. ant icoagulat ion therapy is based on t h e duration of
Atrial fibrillation i s the most common sustained cardiac at rial l"i bri l lat ion. For patients who a re known to have been i n
arrhythmia. The diagnosis of atrial fibrillation is based upon atrial 11brillat ion fo r less than 4 8 hours, preprocedural a n tico­
the demonstration of disorganized atrial activity, seen as an agulation is not necessary as the risk or t h rombus formation
irregularly irregular ventricular response on ECG. Fibrillation is low. Pat ients with atrial 11brillation of unclear duration or
of the atrial myocardium can lead to stasis and intracardiac t hose with atrial 11bri l la l ion for more t han 48 hours require
thrombus formation. In patients older than 40 years, the life­ preprocedural ant icoagu lat ion . These patients should receive
time risk of atrial fibrillation is 1 in 4. The incidence of atrial 3 weeks of t herapeutic ant icoagulation prior to cardioversion.
fibrillation is age-related, and more than 10% of persons aged Alternat ive ly. transesophageal echocardiography (TEE) can be
80 years and older have atrial fibrillation. Atrial fibrillation is performed to look f()r a n i n t racardiac t h rombus. If TEE is
associated with a fivefold increased risk of stroke as well as an negative for t h rombus, acute cardioversion can be performed
increased risk of heart failure and dementia. Atrial fibrillation i m mediately. All patients (regard less of t he duration of atrial
can occur secondary to reversible or acute physiologic insults, fibri l lation) must be ant icoagu lated at t he time of cardiover­
including hyperthyroidism, cardiac surgery, and pulmonary sion and after carclioversion for a m i n i mu m o f 4 weeks owing
embolism. More commonly, atrial fibrillation is the result of to an i ncreased risk of t h romboembolic events a fter restora­
long-standing disease affecting the heart, particularly hyper­ tion of' sinus rhythm .
tension, structural heart disease, and obstructive sleep apnea.

Cl Clinical Presentation
Cardioversion and Acute Rate Control
Many pat ients who present with an i n it ial episode of atrial
As with most arrhy t h mias, patients wit h atrial libri l lation can fibri l lation convert spontaneously. often w i t h i n hours.
experience a wide range of symptoms. i nclud i ng palpitations, However. the presence or hypotension, myocard ial ischemia,
l ight headedness or dizziness. dyspnea. exercise i n tolerance, or heart lailure is an i ndication for i mmediate cardioversion
chest pain. and syncope. Some patients are asymptomatic a nd rega rdless or the d uration or at rial fibri l lation . Acute cardio­
are found to have atrial 11bri l lation as an i ncidental 11nding on version of atrial libril lalion should be synchron ized to the R
ECG. I n its most severe forms. part icularly in patients with wave so as to avoid an " R-on-T" event and provoca tion of V F.
advanced diastolic dysfunction or restrict ive card iomyopathy. Pat ie n t s w i t h rapid ven t ri c u l a r co nduct ion req u i re
atrial fibrillation can resu l t in hemodynamic compromise. heart rate con t rol i n order to i mprove cardiac fu n c t i o n a n d
Some patients i n i t ia l ly present with heart failure caused by symptoms. Ta rget heart rates should be between 6 0 /m i n
tachycardia-i nduced ca rdiomyopathy. a nd l t O / m i n i n t h e acute sett i n g. A c u t e ra te cont ro l is most

55
Arrhyt h m i a s

Cl c i u m c h a n n e l b l oc kers . l nt nivenous medica t i o n s . i nc l u d i n g


o ft e n ach ieved w i t h �- bl ockers or n o n d i hyd ropyrid i ne cal ­ Long-Term Management

CONT.
Anticoagulation
m e t o p ro l o l . e s m o l o l . d i l t ia ze m , a n d verapa m i l . c a n b e Stroke is the most concerning consequence of atrial fibrillation.
used , w i t h su bseq u e n t t ra n s i t i o n ro o ra l f'o rm u l,1 l io n s. I n The absolute risk of stroke is 4% per year among patients with
pat i e n t s w i t h m i l d sym p to m s , ora l age n t s can be consid ­
nonvalvular atrial fibrillation, but comorbiclities can increase the
ered w i t hout i n i t i a l i n t rave nous t herapy. C a l c i u m c h a n n e l
risk 15 to 20 times. Hypertension is an important risk factor for
b l ockers s h o u l d be avo i d e d i n pa t i e n t s w i t h l e f't ven t ricu l a r
both atrial fibrillation and stroke; therefore, aggressive blood pres­
dysf'u n c t ion . O i goxi n ca n b e a d ded to i m prove ra te con t ro l ,
sure control is paramount in the management of atrial fibrillation.
espe c i a l l y i n p a t i e n t s w i t h h e a r t fa i l u re. Pa t i e n t s w i t h
Stroke prevention with an ti thrombotic therapies is predi­
e v i d e n ce of p reexc i ta t i o n s h o u l d not b e t reated w i t h
cated on a patient's aggregate risk profile. Several risk stratifi­
� - b l oc ke rs o r c a l c i u m c h a n n e l b l o c ke rs . I n pa t i e n t s
cation scores are available to clinicians. In patients with
w i t h preexc i t ed a t r i a l fi b r i l l a t ion , p roca i na m i d e i s t h e
t re a t m e n t of cho ice. nonvalvular atrial fibrillation, the CHADS 2 score was, until
I f' cardioversion i s favored because or sign i ficant symp­ recently, the basis for most guideline and consensus docu­
toms d espite rate con t ro l , pharmacologic or e lectrical ca r­ ments. Owing to the limited ability of the CHADS 2 score to
d i oversion can be p ur s ued . Class IC age n ts ( fleca i n i d e , discern between low and intermediate risk, the CHA2 DS 2 -
propa fenone) or i bu l i l ide (an i n t ravenous class I l l m e d i ca­ VASc risk score was developed and now is the recommended
t io n ) can be considered for pharmacologic cardioversion i n score to assess risk of stroke in patients with nonvalvular atrial
pa t ie n t s without structural hea rt d i sease. Cl fibrillation (Table 26) . The CHA2 DS 2 -VASc score is particularly

TABLE 26. Risk Stratification Scores, Adjusted Stroke Rates, and Antithrombotic Therapy Recommendations
Score Incidence of lschemic Stroke Antithrombotic Therapyb
(per 1 00 patient-years)•
CHADS2 Score<
0 0.6 Aspirin or no therapy
1 3.0 Aspirin or OAC
2 4.2 OAC
3 7.1 OAC
4 1 1 .1 OAC
5 1 2 .5 OAC
6 1 3 .0 OAC

0 0.2 None
1 0.6 None or aspirin or OAC
2 2.2 OAC
3 3.2 OAC
4 4.8 OAC
5 7.2 OAC
6+ 1 0. 3 OAC

OAC = oral anticoagulation.

aoata from Friberg L, Rosenqvist M, Lip GY. Evaluation of risk stratification schemes for ischaemic stroke and bleeding in 1 82 678 patients with atrial fibrillation: the Swedish Atrial
J. 2 0 1 2 Jun;33( 1 2) o 1 500· 1 0. [PMIDo 222464431
F;br;llat;on cohort study. Eur Heart

bCHADS2 recommendations from Fuster V, Ryden LE, Cannom DS, et al; American College of Cardiology; American Heart Association Task Force; European Society of Cardiology
Committee for Practice Guidelines; European Heart Rhythm Association; Heart Rhythm Society. ACC/AHA/ESC 2006 guidelines for the management of patients with atrial fibrilla­
tion: full text: a report of the American College of Cardiology/American Heart Association Task Force on practice guidelines and the European Society of Cardiology Committee
for Practice Guidelines (Writing Committee to Revise the 2001 guidelines for the management of patients with atrial fibrillation) developed in collaboration with the European
Heart Rhythm Association and the Heart Rhythm So6ety. Europace. 2006 Sep;8(9):651 ·745. Erratum in: Europace. 2007 Sep;9(9):856. [PMID: 1 69879061. CHA DS VASc recom·
2
mendations from January CT, Wann LS, Alpert JS, et al; ACC/AHA Task Force Members. 201 4 AHA/ACC/HRS guideline for the management of patients with atrial fribrillation: a
report of the American College of Cardiology/American Heart Association Task Force on practice guidelines and the Heart Rhythm Society. Circulation. 20 1 4 Dec 2; 1 30(23):e199-
267. [PMIDo 246823471

'CHADS2 scoring {maximum 6 points): One point each is given for heart failure, hypertension, age �75 years. and diabetes mellitus. Two points are given for previous stroke/
transient ischemic attack.

d(HA2DS2 -VASc scoring (maximum 9 points): One point each is given for heart failure, hypertension, d iabetes mellitus, vascular disease (prior myocardial infarction, peripheral a rte·
rial disease, aortic plaque), female sex, and age 65 to 74 years. Two points each are given for previous stroke/transient ischemic attack/thromboembolic disease and age 2:75 years.

56
Arrhyt h m ia s

helpful in patients with O or 1 CHADS2 risk factors. In addition bleeding compared with warfarin. Apixaban also i s superior
to the CHADS 2 points, this score gives an additional point for to warfarin for the prevention of stroke and is associated
age 65 to 74 years, female sex, and the presence of atheroscle­ with less bleeding overall, including intracranial bleeding,
rotic disease, and gives 2 points for age 75 or older. Patients but similar rates of gastrointestinal bleeding. All of the novel
with a CHADS 2 score of 0 or 1 who have a CHA 2 DS 2 -VASc score oral anticoagulants are cleared by the kidneys. Thus, dose
of 2 or more may benefit from oral anticoagulation. Certain adjustment is required based on estimated glomerular filtra­
high-risk features, such as mitral stenosis or rheumatic heart tion rate (eGFR) , and these agents are contraindicated in
disease, prior systemic embolism, a prosthetic heart valve, left patients with end-stage kidney disease. For this reason,
atrial appendage t hrombus, and hypertrophic cardiomyopathy annual measurement of serum creatinine level is recom­
require oral anticoagulation regardless of risk score. mended for patients treated with these drugs. All of the
For patients who are treated with aspirin, the recom­ novel oral anticoagulants have shorter half-lives relative to
mended dose is 81 to 325 mg daily. For patients who require warfarin; however, there are no quick, readily available
oral anticoagulation, several agents are now available. Dose­ serum assays to accurately determine anticoagulant activity.
adjusted warfarin (a vitamin K antagonist) remains an effec­ Furthermore, currently there is no antidote for these agents
tive low-cost alternative for stroke prevention in patients with in patients with severe hemorrhage.
a higher risk of stroke. The efficacy and safety of warfarin In patients with concomitant coronary artery disease and
therapy are closely associated with the amount of time in the atrial fibrillation, antithrombotic therapy presents significant
therapeutic range (INR 2-3) . The chief limitations of warfarin challenges. For most patients with stable coronary artery dis­
are its need for frequent INR monitoring and adjustment and ease, single-agent therapy with an oral anticoagulant is suffi­
its numerous food and drug interactions. Recently, several cient for prevention of both acute coronary syndromes and
new oral anticoagulants have been approved by the FDA for stroke events. Combination antiplatelet and oral anticoagulant
the prevention of stroke in patients with nonvalvular atrial therapy increases the risk of bleeding, including intracranial
fibrillation, including dabigatran, rivaroxaban, and apixaban hemorrhage. However, patients with an acute coronary syn­
(Table 27) . Warfarin remains the agent of choice in patients drome or revascularization in the previous 12 months are
with valvular atrial fibrillation, generally defined as atrial thought to benefit from combination therapy with low-dose
fibrillation with mitral stenosis or mitral valve replacement. aspirin (<100 mg/d) and oral anticoagulation. In patients who
Dabigatran is superior to warfarin for the prevention of receive a coronary stent, triple therapy with low-dose aspirin
stroke and is associated with less intracranial bleeding, but (<100 mg/d), a thienopyridine (such as clopidogrel) , and war­
carries a higher risk of gastrointestinal bleeding. Rivaroxaban farin is indicated for as short a period as possible. In patients
is noninferior to warfarin for the prevention of stroke or with a drug-eluting stent, this period may extend to 6 months
systemic embolism and is associated with less intracranial or a year. Ongoing clinical trials are evaluating the combina­
and fatal bleeding. Similar to patients receiving dabigatran, tion of anticoagulant therapy for atrial fibrillation and anti­
patients on rivaroxaban have a higher risk of gastrointestinal platelet agents for coronary artery disease.

TABLE 27. Anticoagulants FDA-Approved for Stroke Prevention in Atrial Fibrillation


Medication Reversibility Frequency Type of AF Cautions

Warfa rin (vita m i n K Yes Dosing adjusted to I N R Va lvular or nonvalvu l a r Avoid i n pregna ncy. Caution with
antagonist) id iopathic throm bocytopenic
pu rpura, H IT, hepatic disease,
protein C or S deficiency. M a ny
d rug interactions.

Dabigatran (direct No Twice daily Nonvalvular Decrease dose i f CrCI 1 S-30.


throm bin i n h i bitor) Caution with P-glycoprotein
i n h i bition.

Rivaroxaban (factor Xa No• Once daily Nonvalvu lar Avoid with CrCI <30, moderate
i n h i bitor) hepatic disease. Caution with
m i l d hepatic d i sease

Apixa ban (factor Xa No• Twice daily Nonvalvu l a r Avoid with severe hepatic
i n h i b itor) disease, strong dual i n h i b itors
or inducers of CYP3A4 and
P-glycoprote i n . Caution with
moderate hepatic disease.

AF = atrial fibrillation; CrCI = creatinine clearance {mUmin/1 .73 m2); HIT = heparin�induced thrombocytopenia.

"Early data suggest that factor Xa inhibitors may be able to be reversed with prothrombin complex concentrates. Additionally, several "antidotes" are in development for the factor
Xa inhibitors.

57
Arrhyth m i a s

Rate Versus Rhythm Control guided by risk factors. I n patients with symptomatic atrial
There is no evidence of a survival advantage or reduction in fibrillation who are undergoing cardiac surgery for other rea­
stroke with restoration and maintenance of sinus rhythm in sons, the maze procedure can be performed as a means of
patients with atrial fibrillation, including those with heart maintaining sinus rhythm.
failure. Therefore, the decision to institute a rate or rhythm Patients with refractory symptomatic tachycardia despite
control strategy largely depends on symptoms and patient attempts at rate and rhythm control may be candidates for AV
preference. Patients who are asymptomatic can be managed node ablation. In this approach, patients receive a pacemaker
with rate control only, with a resting heart rate goal of less than and undergo therapeutic ablation of the AV node, rendering
110/min. Patients with tachycardia-induced cardiomyopathy, them pacemaker-dependent but no longer tachycardic. These
heart failure, or left ventricular ejection fraction of less than patients remain in atrial fibrillation and still require stroke
40% may require more stringent rate control (heart rate 60-80/ prevention therapy.
min at rest) . AV nodal blockers, including �-blockers and
KEY POI NTS
nondihydropyridine calcium channel blockers, can be used to
• All patients with atrial fibrillation who u ndergo cardio­
control the heart rate. Combination therapy is often required
version require anticoagulation therapy for a minimwn
to adequately control the heart rate. In addition to assessing
for 4 weeks following the procedure.
the resting heart rate, assessment of the heart rate with activ­
ity should be considered, e ither with ambulatory ECG moni­ • The CHA2DS2-VASc score for estimating stroke risk in
toring, a stress test, or a 6-minute walk test. atrial fibrillation is similar to the CHADS 2 score but bet­
In patients who continue to have symptoms despite ter differentiates low- and intermediate-risk patients; in
adequate rate control, a rhythm control strategy should be addition to heart failure, hypertension, age, diabetes
considered to i mprove quality of lite. Rhythm control may mellitus, and previous stroke, the CHA2DS 2-VASc score
require cardioversion followed by antiarrhythmic therapy. incorporates lower age (65-74 years) , sex, and the pres­
Antiarrhythmic drug selection is based on patient comorbidi­ ence of atherosclerotic disease.
ties and the safely profile of the antiarrhythmic drugs. Some • Options for long-term anticoagulation in patients with
patients with infrequent symptomatic atrial fibrillation may atrial fibrillation include warfarin, dabigatran, rivaroxa­
not require daily therapy. Patients with infrequent atrial fibril­ ban, and apixaban; the latter three agents do not
lation and neither structural heart disease nor conduction require blood monitoring and lack the food and drug
disease may benefit from a "pill-in-the pocket" approach, interactions of warfarin, but they are substantially more
whereby patients take a class IC drug (flecainide or expensive.
propafenone) only when they develop an episode of atrial
fibrillation. Patients who follow this approach should be tak­
ing an AV nodal blocker or should take one before taking their
"pill in the pocket." The first time this approach is used, it
Atria l F l utter
should take place in a monitored setting to ensure that the U nlike atrial fibrillation, atrial flutter is an organized
patient can safely tolerate the therapy without development of macro-reentrant rhythm with discrete and organized atrial
proarrhythmia or conduction disturbance (for example, post­ activity on the ECG, usually with an atrial rate of 250/min to
termination pause) . Regardless of the rate or rhythm control 300/min. Although they are distinct rhythms, atrial fibril­
strategy used, stroke prevention should be guided by patient lation and atrial flutter are often found in the same patients
risk (CHA 2 DS 2 -VASc score) . because of similar risk factors and pathophysiology.
Episodes of atrial flutter can induce atrial fibrillation
Nonpharmacologic Strategies and vice-versa.
In patients who have refractory symptomatic atrial fibrillation Typical atrial flutter has a sawtooth appearance on ECG,
despite antiarrhythmic drug therapy, catheter ablation with with negative flutter waves in the inferior leads and positive
pulmonary vein isolation is an etlective rhythm control ther­ flutter waves in lead V1 (Figure 18) . Typical atrial flutter is
apy. Atrial fibrillation ablation is best reserved for patients caused by counterclockwise reentry around the tricuspid
with early atrial fibrillation without evidence of significant left annulus. Atypical flutter can be clockwise or can occur i n
atrial enlargement and those without multiple comorbidities. other locations in the atria, including the left atrium after
The success rates for atrial fibrillation ablation are variable, but atrial fibrillation ablation.
in patients with paroxysmal atrial fibrillation, between 70% In many respects, atrial flutter is managed similar to atrial
and 90% are symptom-free at 1 year. Complications can fibrillation, including stroke prevention. However, owing to the
include intraprocedural or late tamponade, vascular complica­ atrial rate and the ratio of conduction through the AV node (for
tions, and a 0.5% to 1 % risk of stroke. Patients who develop example, 2:1 or 4:1), rate control of atrial flutter can be difficult
dyspnea months to years after an atrial fibrillation ablation and often requires large doses of AV nodal blockers. Therefore,
may have pulmonary vein stenosis. Anticoagulation is manda­ atrial flutter is usually managed with a rhythm control strategy.
tory for lhe first 2 to 3 months after ablation, and thereafter is Catheter ablation oflypical atrial flutter is often preferred owing

58
Arrhyt h m i a s

I I
I _l I� !I • ij

Ir-
, J I,

I
1
rr

F I G U R E 1 8 . In this electrocard iogram demonstrating typical atrial flutter, negative sawtooth waves are seen in the inferior leads and positive waves are seen in lead V 1 . I n
the bottom rhythm strip, 2 : 1 and 4 : 1 conduction patterns are seen.

to a high success rate and lower complication rate relative to


Ventricu lar Arrhythmias
other ablation procedures. In asymptomatic patients in whom
Premature Ventricular Contractions
rate control can be achieved, a medical rate control strategy
Premature ventricular contractions (PVCs) are common and
is acceptable.
can occur in up to 75% of healthy persons. Patients with PVCs

CJ Wide-Co m p lex Tachyca rdias


generally report palpitations and a sensation of skipped beats.
Forceful palpitations with PVCs are usually caused by exagger­
A wide-complex tachycard ia is a ny tachycard ia ( heart rate ated cardiac filling during the pause after the PVC. PVCs are
�1 0 0 /m i n ) with a QRS comp l ex of 120 msec or greater. The more common in patients with hypertension, left ventricular
d i ffere n t ia l d iagnosis incl udes suprnventricular rhyt h m s hypertrophy, prior myocardial infarction, and other forms of
w i t h aberran t conduct ion (such as underlying b u n d l e structural heart disease. For patients with bothersome palpita­
bra nch block) , preexci tation, paced rhyt hms, a n d ven t ricu­ tions, the first diagnostic test is an ECG. I f the diagnosis is not
lar ta chyca rd ia . established, 24- to 48-hour ambulatory monitoring is used to
Often, pat ients presen t w i t h a wide-complex tachyca rdia diagnose and quantify the frequency of PVCs and determine if
of unknown etiology. In adults with structural heart d isease, they are monomorphic or polymorphic. Frequent PVCs (>10%
95% of wide-complex tachycardias are VT. Wide- complex of all beats or �10,000 PVCs in a 24-hour period) can lead to
tachycardias that are positive in lead a VR, have a QRS mor­ tachycardia-induced myopathy. Patients with frequent PVCs or
phology t ha t is concordant i n t he precordial leads (monopha­ polymorphic PVCs should undergo echocardiography or other
sic with t he same polarity) , have a QRS morphology other than cardiovascular imaging (such as cardiac magnetic resonance
typical right or left bundle branch block, and exhibit extreme [CMR] imaging) to evaluate for the presence of structural
axis deviation ( - 90° to ±180°. someti mes cal led a nort hwest heart disease.
axis) . are usual ly VT. The presence of AV dissociation. l"u sion In patients without high-risk features (such as syncope, a
beats (QRS co mpl ex created by rusion between a sinus capture family history of premature SCD, coronary artery disease, or
beat and a VT bea t ) , and capture beats (sinus beat that cap­ structural heart disease) , PVCs (including ventricular bigem­
t u res t he myocard i u m in between VT beats) are all high ly sug­ iny and trigeminy) are generally benign and do not require
gestive of VT. additional testing or treatment. Treatment should be limited to
When t he origi n of a wide-complex tachycard ia cannot those with symptoms or a high burden of PVCs (�10,000 in a
be determi ned, VT should be assumed until expert consu l ta­ 24-hour period) . Treatment for PVCs usually begins with
t ion can be obtai ned Cl . P-blocker or nondihydropyridine calcium channel blocker

59
Arrhyt h m i a s

therapy. Antiarrhythmic drug t herapy c a n also be used when and susta ined VT/V F should u n dergo !CD implan tation for
PVCs persist despite �-blockade or calcium channel blockade. secondary preve n t ion . In patients wi t h an !CD. if VT recurs
EP study and catheter ablation can be considered in patients desp i te � - blocker therapy. a n t ia rrhy t h m i c d rug therapy
who cannot tolerate medical therapy or i f medical t herapy fails should be consi dered . In most patients wi t h s t ructural
to suppress the PVCs. heart disease. a m iodarone is fi rst - l i ne a n l i a rrhy t h m i c d rug

Cl Ventricular Tachycardia with


t herapy. Pat i e n ts with recu rrent VT despi te medical t her­
apy should be considered for EP study <111 d catheter abla­
Structural Heart Disease t io n . which has been shown to reduce ! C D shocks and t h us
In structural heart disease. including both ischemic and non­ i m p rove q u a l i ty of l i fe.
ischemic cardiomyopathy. the presence of myocardial scar
t issue focilitates reentry and the development of VT. VT can Idiopathic Ventricular Tachycardia
present as nonsustained or sustained VT (>30 seconds) . I n VT in patients without structura l heart d isease is considered
pa tients w i t h ven tricular scarring. V T is usual ly regular and id iopathic. Patients often present with palpitations in early
monomorphic. Figure 19 shows ECG findings of monomor­ adulthood (20-40 years of age) . Episodes are often provoked by
phic VT in a patient with cardiac sarcoidosis. I n patients wi th stress, emotion, or exercise. Syncope is uncommon. I d iopathic
structural heart disease, VT may lead to hypotension, syncope. VT usually arises from the outflovv tracts. the fascicles. or t he
degenera tion into VF. and cardiac arrest. A l ternatively, short papillary muscles. Outflow t ract tachycardias, the most com­
runs of VT or slow sustai ned VT may be wel l tolerated mon type, are triggered arrhythmias that can arise from the
or asymptomatic. right or left ven t ricular outflow tracts. They a re adenosine­
A l l patients w i t h VT should undergo rest i ng ECG. exer­ sensitive and often exhibit repetitive salvos. Right ventricular
cise t rea d m i l l testing to provoke t he a rrhyt h m i a , a n d car­ outflow tract tachycardia has a left bundle branch block
d i a c imaging to eva l u a te for st ru c tu ra l heart d isease. appearance wi t h tall R waves i n the i n ferior leads.
Pa t i e n ts w i t h i schem i c card i omyopathy who present w i t h Pharmacologic t herapy for idiopathic VT i ncludes calcium
V T should u ndergo a n i schemia evaluation and revascu ­ channel blockers (especially verapami l) or �-blockers. \Vhen
la rization i f i nd icated . Pat ie n ts w i t h cardiomyopathy and symptoms continue despite these measures, catheter ablation
heart fa i l u re should receive optimal medical therapy in can be considered. !CDs are rarely i ndicated i n patients with
order t o red uce their r i s k 01· ven t r i c u l a r arrhy t h m i a . idiopathic VT owing to the benign prognosis and efficacy of
Pat ie n ts w i t h s t ructural hea rt d i sease or card i o myopat hy other t herapies. Cl

F I G U R E 1 9 . An electroca rdiogram demonstrating episodes of monomorphic ventricular tachycardia in a patient with cardiac sarcoidosis. Note that the repeated wide·
complex beats do not resemble a typical bundle branch block pattern. There are several sinus beats (na rrow com plexes) that help ascerta i n the etiology of the wide beats as
ventricular tachyca rdia.

60
Arrhyt h m i a s

KEY POINT heart disease; and electrolyte abnormalities. Patients with a


QTc interval greater than 500 msec are at greatest risk for SCD.
• In patients with premature ventricular contractions
First-line therapy for long QT syndrome is �-blocker therapy.
without high-risk features, reassurance is usually suffi­
Patients with cardiac arrest or those who have recurrent events
cient; treatment should be limited to those with symp­
(syncope or VT) despite �-blocker therapy should undergo !CD
toms or frequent episodes.
implantation. Patients with documented long QT syndrome
should avoid participation in competitive athletics.
I n herited Arrhythmia Syn d romes Short QT syndrome is a rare and genetically heterogene­
The diagnosis and management of inherited arrhythmia syn­ ous disorder characterized by a short QT interval, usually less
dromes are complicated by the variable penetrance and vari­ than 340 msec (or QTc <350 msec) . It is inherited in an auto­
able expressivity often observed. Characteristics and treat­ somal dominant pattern. Patients can present with atrial and
ment of the most important inherited syndromes are reviewed ventricular arrhythmias and syncope. Short QT syndrome
in Table 28. The presence of unexplained premature (younger carries a high risk for SCD, and !CD placement is recom­
than 35 years) death or sudden death in a first-degree family mended for all patients.
member should raise suspicion for the possible presence of an Brugada syndrome, an autosomal dominant disorder
inherited arrhythmia syndrome and referral to a cardiovascu­ associated with mutations in the sodium channel gene, is
lar specialist. Genetic testing has facilitated the diagnostic characterized by right precordial ECG abnormalities, including
evaluation of these disorders, particularly when an affected ST-segment coving (ST-segment elevation that descends into
family member has a known pathogenic mutation. Patients an inverted T wave) in leads V 1 through V3 with or without
with a family history of SCD and unexplained syncope are right bundle branch block (Figure 20) , VF, and cardiac arrest.
particularly at high risk and merit aggressive evaluation. Brugada syndrome is more common in men and in persons of
Long QT syndrome is one of the most common inherited Asian descent. Arrhythmic events often occur at night during
arrhythmias and is defined by the presence of a prolonged QTc sleep. The ECG abnormalities can be variable and may be
interval (>440 msec in men and >460 msec i n women) accom­ unmasked by fever or pharmacologic challenge with sodium
panied by unexplained syncope or ventricular arrhythmia. channel blockade (for example, procainamide infusion) . Risk
The presence of a prolonged QTc interval alone is not sufficient stratification in patients with Brugada syndrome is principally
for a diagnosis of long QT syndrome. The diagnostic criteria based upon the presence or absence of syncope; those with
include ECG findings, symptoms, and in some cases, results of syncope or ventricular arrhythmia should undergo !CD place­
genetic testing. There are many causes of a prolonged QTc ment. Patients with recurrent ventricular arrhythmias and/or
interval, most of them are acquired, including medications !CD shocks often benefit from quinidine antiarrhythmic
such as antiarrhythmic agents, antibiotics (macrolides and drug therapy.
fluoroquinolones), anti psychotic drugs, and antidepressants (a Catecholaminergic polymorphic VT is a rare disorder
list can be accessed at http: //crediblemeds.org/) ; structural characterized by polymorphic ventricular arrhythmias and

TABLE 28. Inherited Arrhythmia Syndromes


Disorder Characteristic Findings Treatment•

Long QT syndrome Syncope, QTc interval usually >460 msec, torsades �-Bl ockers, ICD, exercise restriction
de poi ntes

Short QT syndrome Syncope, QT interval <340 msec, atrial fi bril lation, ICD i n all patients
VT, VF

Brugada syndrome Syncope, VF, coved ST-segment elevation in early ICD


p record ial leads (V1 -V3 )

Catecholam i n ergic polymorphic VT Syncope, polymorphic or bidirecti onal VT d u ring �-Blockers, ICD, exercise abstinence
exercise or emotional d i stress

Early repola rization syndrome Syncope, inferior and lateral early repolarization on ICD
ECG, VF

ARVC/D Syncope, T-wave i nversions in leads V1 to at least V3, ICD, �- b l ockers, antiarrhythmic
monomorphic VT, a b n o rm a l signa l-averaged ECG, medications, exercise abstinence
frequent PVCs, and abnormal right ventricular size
and function on echocardiography or CMR imaging

ARVC/D = arrhythmogenic right ventricular cardiomyopathy/dysplasia; CMR = cardiac magnetic resonance; ECG = electrocardiography; ICD = implantable cardioverter·
defibrillator; PVCs = premature ventricular contractions; QTc = corrected OT interval; VF = ventricular fibrillation; VT = ventricular tachycardia .

.,Treatment recommendations for ICDs in inherited arrhythmia syndromes are guided by risk stratification with criteria that are often disease specific. Additionally, antiarrhythmic
drugs are often required in several syndromes for recurrent ventricular arrhythmias.

61
Arrhyt h m i a s

n� :v;J� v---J
1 r rr
J �r.,-v-1.f � ;-v-1
�--r �� r\/'1 i v 1 v 11 �:
_ _ __

F I G U R E 2 0 . In this electrocardiogram demonstrating a type 1 Brugada pattern, �2 111 111 J-point elevation, coved ST-segment elevation (arrowheads), and T-wave inversions
are seen i n leadsV1 to Vr

cardiac arrest usually provoked by high-adrenergic states, accelerate disease progression and arrhythmogenesis. Patients
including strong emotion and exercise. Patients with this dis­ with ARVC/D and cardiac arrest or risk factors (nonsustained
order usually have provocable arrhythmias with exercise or VT, inducible VT) are offered !CD placement. �-Blockers are
epinephrine infusion. Treatment includes �-blocker therapy first- line therapy for ventricular arrhythmia; however, antiar­
and often !CD placement. Patients with the disorder should rhythmic therapy with sotalol or amiodarone or catheter
abstain from exercise. ablation is often required for recurrent VT.
In patients with unexplained VF arrest, particularly when
provoked during exercise, early repolarization syndrome
should be considered. Whereas early repolarization (J-point Sudden Ca rdiac Arrest
elevation) is a common and benign finding on ECG, the pres­ Epidemiology and Risk Factors
ence of inferior and lateral early repolarization more than 1 SCD is defined as instantaneous death or sudden collapse
mm in a patient with VF or cardiac arrest should be consid­ within 1 hour of symptoms. Unwitnessed death is considered
ered early repolarization syndrome. Jn patients with VF or SCD if the patient was known to be well within 24 hours of the
cardiac arrest, !CD implantation is indicated. event. Most episodes ofSCD are caused by ventricular arrhyth­
Hereditary structural heart disease, such as hypertrophic mias (VT/VF arrest) . In the general population, the risk of SCD
cardiomyopathy (see Myocardial Disease) or arrhythmogenic is 1 / 1000 per year. The incidence is greatest in patients with
right ventricular cardiomyopathy/dysplasia (ARVC/D) , often preexisting structural heart disease; however, most episodes of
manifests as sudden cardiac arrest in a young person. ARVC/D SCD occur in patients with normal left ventricular function.
is characterized by fibrous and flbro-fatty changes of the right Risk factors for SCD include (but are not limited to) heart fail­
ventricle and subsequent ventricular arrhythmias. Penetrance ure. diminished left ventricular function. prior myocardial
is variable and age-related, with many patients presenting infarction, unexplained syncope, left ventricular hypertrophy,
between puberty and young adulthood. Patients with ARVC/D nonsustained ventricular arrhythmia, chronic kidney disease,
usually have ventricular ectopy or monomorphic VT, although and obstructive sleep apnea.
patients with severe disease may have heart failure. The diag­

Cl
nosis of ARVC/D is guided by diagnostic criteria that include Acute Management
ECG abnormalities, family history, the presence of arrhyth­ Patients with cardiac arrest require immediate cardiopulmo­
mias, and structural abnormalities of the right ventricle as nary resuscitation (CPR) and advanced cardiac lite support.
seen on cardiac imaging_ A RVC/D is usually progressive, and The two most impo11ant i n terventions for patients in cardiac
those with ARVC/D should abstain from exercise. as it may arrest are h igh-quality CPR chest compressions and rapid

62
Arrhyth m i a s

Cl clefibri l lation in pat i ents with VT/VF arrest. Basic l i fe support Patients w i t h modern ICDs have few li m itations. I n gen­
guideli nes now emphasize t he acronym CAB (Chest compres­ era l , l ight to moderate exercise. i ncluding sexual intercourse, is
CONT.
sions. A i rway. Breat h i ng) lo high l ight the importance of permi sible and is associated with i mprovement in cardiovas­
i m mediate, rapid. and sustained chest compressions and de­ cular healt h and q ua l i ty or l i fe. However. some d isorders carry
emphasizi ng assisted breath i ng. Once a code has been cal led or speci fic restrictions (see Table 28) . Patients with ICDs should
t he emergency medical system has been activated and a n au to­ avoid st renuous upper extremity exercises. i ncluding weight
mated external defibri l lator has been requested. t h e pat ient's l i ft i ng. because t hese activities can damage the leads coursing
pulse should be checked i m mediately. I f no defi nite pulse is t h rough t he chest. Electromagnetic i n terference can lead to
detected w i t h i n l O seconds, chest compressions should begin inappropriate detection of VT/VF and shocks: t herefore, patients
without delay. In pat ients with VT/VF. time to defibri l la t ion is an should avoid l a rge sources of electromagnetic i n terference,
i mportant determinant of t he l i ke l ihood of survival to hospital including arc welding and h igh -voltage machi nery. During sur­
discharge. Therefore. when a shockable rhy t h m is present. defi­ gery. ICDs may need to be reprogram med or have a magnet
bri l lat ion should be performed as rapid ly as possible. applied to avoid false detection of VT/VF clue to electrocaute1y.
Once CPR has been started. the 2 0 1 0 American Heart For this reason . patients with ICDs should have an evaluat ion or
Association guidel i nes on CPR and emergency cardiovascular device program ming recom mendation from t heir electrophysi­
care dictate management based upon the presence or absence of ologist before u ndergoing i nvasive procedures or surgery.
a shockable rhythm. In patients with asystole or pulse less electri­ Patients who experience shocks need to contact their ICD
cal activity (PEA ) . CPR is continued with reassessment of rhythm physician . Patients who experience more than one shock in
status for a shockable rhythm every 2 mjnutes. Epinephrine (I mg 24 hours or any shock accompanied by dyspnea. chest pain. syn­
intravenously) should be given eve1y 3 to 5 mi nutes. alt hough cope, or heart failure �ymptoms require emergency medical care. Cl
vasopressin (40 units i ntravenously) can replace the fi rst or sec­
KEY POINT
ond close of epinephrine. Atropine is not recommended for the
• Implantable carclioverter-defibrillator placement is
t reatment of asystole or PEA arrest. Further management or PEA
arrest should include ascertainment and t reatment of a ny cor­ inclicated for secondaiy prevention in patients with sus­
rectable etiology (for example. tamponacle) . In patients with VT/ tained ventricular arrhythmias (>30 sec) or cardiac
VF, a shock is advised with i mmediate resumption or CPR and arrest without a reversible etiology.
reassessment of t he rhythm in 2 m i n u tes. Epinephrine should be
given after t he second shock and every 3 to 5 m i nutes thereafter.
Device I nfection CJ
I f VT/VF continues despite three shocks and epinephrine. am i­ Betvveen 1993 and 2008. the use of cardiac i mplanted electronic
odarone should be given as a bolus. devices i ncreased by 96%. As a resu l t . the number of patients
Patients w i t h symptomatic bradycard i a and hemocly­ suscept ible to device in fecl ion seen in c l i n ical practice has
namic distress should first be t reated w i t h atropine. I f a t ropine increased dramat ical ly. Device i n fect ions range from i n fections
is i neffect ive. dopa m i n e or epinephrine i n rusions can be i nvolving t he site of device placement (pocket in fection) to
a ttempted u n t i l transcutaneous pacing or a tempora ry pacing in fective enclocard i tis. Most device in fections are clue to staphy­
w i re (pre ferred) can be implemented. lococcal i n fect ions. part icularly Staphylococcus epidermidis
Post-resusci tation care i ncludes t herapeut i c hypot herm ia and S. a u reus. When caring for patients with cardiac implan ted

i n patients who remai n comatose. Compl ica t ions o f t herapeu ­ electronic devices who present with symptoms of i n fection .

t i c hypothermia i nclude ven t r i c u l a r a rrhy t h mias during c l i n icians must have a h igh suspicion f o r device i n fection.

rewar m i ng and i n fectious com p li c a t ions. i nclud i ng sepsis. Patients with cardiac device i n fect ion can presen t w i t h

Hemoclynamics and oxygenation should be opt i m ized in the fever. ch i l ls. a n d malaise. T h e physical exam i nation may reveal
post-a rrest sett i ng. Moderate glycemic control is also recom­ eryt hema. pocket swel l i ng. and d ra i n age from t h e pocket .

mended. Pat ients w i t h evidence of acute coronary syndrome Laboratory findi ngs frequen t ly include anemia, leu kocytosis.
should undergo i m mediate cat heterization and revascu la riza­ and an elevated eryth rocyte sed i mentation rate. In pat ients
tion provided t here are no con t ra i ndications. with suspected device i n fection, m u l t iple blood cultures should
be d rawn . Echocardiography ( most often with t ransesophageal
Device Therapy for Prevention echoca rcl iography) should be performed to ident i fy i n t racar­
of Sudden Cardiac Death d iac or lead vegetat ions. The device pocket should never be
Pat ients w i t h sustai ned ven t ricu l a r arrhy t h m ias or cardiac asp i rated lor d i agnostic purposes because puncturing the
arrest w i t hout a reversi ble etiology have a class I recom menda­ pocket can damage the leads or i n t roduce i n fection.
tion tor secondary prevent ion ICD placement. I n patients w i t h Once a cardiac device in fection is d i agnosed. t reatment
st ructural heart d i sease w h o meet specific criteria, ICDs a rc includes complete removal of a l I hardware. debridement of t he
i ndicated for prima1y prevent ion (see Heart Fa i lu re) . ICD bat pocket, sustai ned a n t i biot ic t herapy, and rei mplantation at a
tery l i fe is approx i mately 7 lo 1 0 years but is variable. A l t hough new site ( i f and when appropriate) . Suppressive a n tibiotic
ICD malfunction is rare. when i t occurs. it is often clue to a t herapy w i t hout complete removal of' t he device is not curative
problem with the i n t racarcliac leads. and is associated w i t h a h igh fata l i ty rate. Cl

63
Pe r i c a rd i a ( D i sease

related to t he u n derly i ng cause. hcst pa i n c l ue to acute pcrirnrdi


KEY P O I NTS
t i s typica l ly is sharp. a n terior. a nd pos i l iona l . :incl it is aggrav<1 l ccl
• I n a patient with suspected implanted cardiac device
by ma neuvers t ha t i ncrease pericard ia! pressu re (cough . i nspi1�1
infection, the device pocket should never be aspirated
l io n ) <lnt l orl hostasis (decreased pa i n in t he upright posi t ion) .
for diagnostic purposes because puncturing the pocket
These charnctcristics may be uscf'ul in c l isl i nguishing perirnrd i t is
can damage the leads or introduce infection. ·
rrom acute myocardia I i schemia : however. t hese lea t u res also <l rc
• Treatment of implanted cardiac device infection com­ freq uen t ly present in ot her che ·t pa i n syndromes. such as aort ic
prises complete hardware removal and pocket debride­ d issec t io n . costochomlrit is. and pul monary embol ism . Fever may
ment, sustained antibiotic therapy, and reimplantation also be present w i t h acute pcricard i t is.
at a new site if appropriate. /\ perica rd i a I fri c t i o n rub i s considered l o be a h a l l m a rk of'
acu te pcrica rd i t i s . /\ perirnrcl i a l rub occurs in 3S'Y.. t o SO'Y., of'
p<t l icnts. w i t h or wit h o u l a pericard ia I effus i o n . T h ree com po
Pericardial Disease ncn t s may be ausc u l ta t e d . a t t ribu table lo a t ri a l systole . vcn

CJ Acute Pe rica rditis


t ri c u l a r syst ole . a ncl early ra pid ven t ri c u l a r d i astolic f i l l i ng.
l 'cricm l i a l rub i n te n s i t y a n d loca l ion cu n va ry. b u t it is c h a rac
Cli nical Presentation tcrist ica l ly hca rcl a t t he lcl't lowcr sternal border. It i s best he<ml

Acu t e perica rd i t i s m<1y occur as part or a s 1stc111i c d i sorder or d u ri n g a held end e x p i ra t i o n w i t h t he pa t ie n t lea n i ng ifJ rwa rd.

in i so l a t i o n . /\ I t hough t here a re m a ny potcn t i<1 l causes o r acute This m a neuver a l lows d ist i nct ion f'rom a pleuropericarc l i a l or

pe rica rcl i t is . most cases a rc id iopa t h ic o r presum ed to be v i ra l pleu ra l rub. \Nh ich arc p resen t o n ly d u ri n g resp i ra t ion .

o r a u t o i m m u ne i n o r i g i n . I n clcwloping cou n t ries a n d su sccp


t i ble i n d iv i d u a l s . t u be rcu losis a nd 1 1 1 \1 i n fect io n a rc com mon Eva luation
causes o r acute pcricard i t i s . O t hn causes i n clude neo p l a s m . l·:CG cha nges f'req u c n t ly occ u r in pa t i e n t s w i t h acute pcrica r
t ra u m a . u re m i a . t horacic i rrncl i<t l io n . a n d ccrt •l i n med ica t ions d i t i s owi ng to i n l l a m m a t ion of' t he visceral pericard i u m (or
( hyd ra lazi n c. pen ic i l l i n . chemot her•lpc u t ic age n t s ) . cpi c<ml i u m ) . Ty p i c a l ECG c h a nges a rc con cave u pward
Acu te perica rcl i t is may be d iagnosed Oil t he b<lSis or typical ST segme n t e l eva t i on a nd JJR-scgm e n l abnorma l i t ies. w i t h
sy mp toms or chest pa i n . a periGm l i a l rrict ion rnh. d ist incl iw eleva t i o n i n lead a V R :rncl depression i n a l l ot her leads.
clcct roca rtl iogrnphic (!-:CG) ab11nrm:1 l i t ics. a nd supportive d<tt<l IJR scg1 1 1 c 1 1 l a b n o r m a l i t ies a rc d u e l o a t ri a l i nvolve m e n t
rrom noninvasive test i ng. Chest pa i n clmractcrizcs t he cl i n ic:1I (Figure 21 ) . /\ I t h o u g h t he E , G abnorm a l i t ies typical ly a rc d i f'
presentat ion in 90'Y., t o 9S'�. of' p•t t ients. wit h acid i t ional sy mptoms ruse. cert a i n causes (such as card i ac pcrforat i o n . t ra u m a . or

F I G U R E 2 1 . El ectrocardiographic changes of acute pericarditis. Con cave ST-segment elevation is present in most of the leads (arrowheads). The PR segment is depressed
i n all leads except aVR (arrows).

64
Pe ricard i a ! D i sease

Cl segme n t a l myocard i a l i n f'a rc l i o n ) m ay n::s u l t in l ocal ized sy m p w m re l ief' w i t h a n t i i n fl a m m a t o ry age n t s . a n d t h e recog­


c h a nges . S i n us tachycard i a a nd elect rical a l l e rn <J n s a l so a re n i t io n a n d t reat m e n t of' pcric;1rd i a l c lTusions t ha t a re hemody­
CONT.
f'req uen t ly seen . Severa l ECG fea t u res s h o u l d be ro u t i ne ly co n - n a m i ca l ly sign i fica n t . Most pa t i e n t s wi t h acute pericarcl i t i s
s id ered to h e l p lo d i s t i nguish t h e cha ngcs of' acu t e pcricard i t i s c a n b e m<rnaged as ou l p�ll ic n t s . Pa t ie n t s w i t h acute peri card i ­
f'rom myoca rd i a l ischemia a n d ea rly repo l a ri 1.a t io n (Table 29) . t i s a rc al i n creased risk for deve l o p i n g a t ri a l f1bri l la t i o n and
O t h er n o n i nvasive t es t s ca n be used lo support t h e d i ag­ fl u t ter, which 111ay req u i re t rea t me n t for sym p t om ma n<1ge
nosis o f ' a c u t e perica rd i l i s b u t a rc l i m i l ccl i n t h e i r se ns i t i v i ty rnent a n d reversion lo s i n u s rhy t h m . Li fe- t h rea t e n i n g co m p l i ­
a n d spec i l'ic i t y. I n fl a m m a to ry m a rke rs . such as e ry t h rocy t e c a t i o n s d u e l o e lTu s i o n :l l'c rare but m ay deve lop rapid ly.
sed i m e n t a t i o n rat e. l e u kocy t e cou n t . a n d C- rea c l ive protei n Cl i n i cal lea t u rcs i n d i cl l ivc of' i n c reased r i s k a ncl a poten t i a l
leve l . may be e l eva tec l . I nc reases i n b i o m a rkers of' c::i rcl i a c need f ( ir hosp i t<1 l iz<1 l ion a 1-c t h e prescnce of' fever. leu kocytosis,
i nj u ry (card i a c t ropo n i n T o r t ropon i n I ) i n d i r n t c conco m i ­ <J cute t ra u m a . abnormal card i ac b ioma rkers. i m m u nocom p ro­
t a n t myoca rcl i l i s ( t h a t i s . myo pcrica rd i t i s) . Ca rd i o m ega ly m i sc. ora l a n t icoagu l a n t use (which may i n c rease t he risk of'
may be seen on chest racl iogra p h s i f' a l a rge perica rd i a ! c fTu b l eed i ng a n d t a m pon<1de) . a nd l a rge or l1cmody n a m ica l ly s ig­
s i o n i n p rese n t . n i f'ica n l pericard i a ! cfTusi o ns .
E c h oca rcl i ogrnphy s h o u l d b e pcrf'o r rn ccl i n a l l p a t i e n t s For :rnalgcsia a n d t rea t me n t o f ' perica rd i a ! i n ll ::i m m a t i o n .
w i t h su spec ted c1 c u t e periea rcl i t i s lo eva l u <J l e i C J r t h e p res NS/\ I Ds prov ide cffC:c t i vc re l i e f ' i n 70'Y., lo 80'Y., of' p a t i e n t s . The
en cc o f ' a h e m ocly n a m ica l ly s i g n i f'ic a n t perica rd ial e f'f'u s i o n . ef'f'icacy of' 111 cd ical l herapy va ri es accord i ng lo t he u n derly i ng
A l t h o u g h a p e r i c a rcl i a l e ffu s i o n occu rs i n S O 'Y., to 6 0 'Y., o f ' ca use. w i t h response rn t es grea test a mong t hose w i t h i d io­
pa t i e n t s . i ts a bs e n ce d o e s n o t r u l e o u t t h e d i a g n o s i s o f' pa t h i c or presumed v i ra l causes. Re l a t ively h igh closes of' t hese
a c u t e p e r ic a rd i t i s . med ica t i ons a re req u i red (aspi r i n , 65 0- 1 0 0 0 mg every 6 to
A l t hough cva l u a l i on f c i r a cause is usua l ly warra n ted i n 8 h o u rs ; i burirotc n . 4 00 - 800 mg every 8 h o u rs ; i ndom e t ha c i n ,
pa t i en t s w i t h acu te pericarcl i t is. t he d i agnos t i c y i e l d o f stand ­ 5 0 m g every 8 h o u rs) . S l ow t a pe r i ng over a period o f ' 2 t o
a rd t es t i ng is l ow. w i t h an et iology iden t i fied i n fewer t h a n 20'Y., 4 weeks i s reco m mended t o reduce t h e r i s k of' recu rren t
of' cases. I m port a n t causes lo consider frir fu r t h e r eva l u a t i o n i n l'la111 m a l ion . I n pa t i e n t s w i t h per icard i t i s assoc i a ted w i t h
are t h ose t ha t req u i re speci l'ic t h erapy. such as n co ri l a s t ic myocard i a l i n fa rc t i o n , N S/\ ! D s o t he r t ha n asp i r i n s h o u l d n o t
d i sorde rs. a u t o i m m u ne d i sease. t rauma ( f c J r example, postsur­ be u s e d . These age n t s ca n i m pa i r myocard i a l hea l i ng a nd
gical periGird i t i s ) . a n d i n fec t i o n (such as t u be rcu l os i s) . i n crease t he risk of' mech::i n ical com p l i ca t i ons i n t hese pa t ien t s.
l�va l u a t i o n for spcc i f1 c causes o f' acu t e ricrica rd i t i s s h o u l d be Colc h i e i n e ( 0 . 5 to 1 . 2 111g 1d for 3 m o n t h s ) . i n add i t i on t o
considered accord i ng lo t he degree of' c l i n i cal suspicion for N S A ! Ds. i s a n ef'kcl i vc adj u nct ive t herapy f o r acute pericard i­
d i sorders t h a t wo u l d req u i re t herapy beyo nd l rcal 111 e n t o f t h e l is. The ef'f'icacy of' colchici ne has been demonst ra ted i n several
acute ep i sode of' pericard it is ( for example. m a ! igna ncy or a u t o­ ra n d o m i zed a n d rel rospect ive s t u d ies. which have shown s ig­
i 111 m u ne d i sease) . Most cases of' acute pericard i t i s arc v i ra l or n i f'ica n t ly lower rates of' t rea t me n t fa i l u re a nd rec urre n t peri ­
i d iopa t h i c :rnd responsive lo a n t i i n ll a m m a t o ry d rugs . c a rcl i t i s when used i n conj u n c t i o n w i t h s t a n c l a rcl NSA I D t her­
Pericard iocc n tesis ca n assist i n d i agnosis when o t her test apy. Co lch icinc i s ge nera l ly we l l t o le ra ted : side e f'fecls i nclude
ing i s i n conclu sive and is a lso i n d icated for ohl<1 i n i ng a t i ssue gast roi n test i n a l d i s t ress a n d . l ess com mon ly, bone m a rrow
sample i f' t u bercu losis i s suspect ed . su priress i o n . myosi t i s , a nd I iver t ox i c i ty.
G I ucocorl i coid s a re n o t considered fi rst - 1 i ne t h e rapy fbr
Ma nagement acute perica rd i t i s and a re rese rved for pa t ie n t s w i t h con t ra i n ­
M a nage m e n t goa l s f o r a pa t i e n t w i t h acute pericard i l i s a re t h e d i c a t i o n s lo NSA I Ds and t h ose w i t h refractory pe r i ca rd i t i s .
i d e n l i fka t i o n a nd t rea t m e n t ol' poten t i a l u n d e rlying cau ses . G l ucoco r t icoids a l so mny be consid ered in spec i fi c d i sease

TAB LE 29. Electrocardiographic Features for Differentiating Acute Pericarditis from Myocardial lschemia
Feature Acute Pericarditis Myocardial lschemia

ST-segment conto ur Concave u pwards Convex upwards

ST-segment lead involvement Diffuse Loca l ized

Reciprocal ST-T wave changes No Yes


PR-segment abnorma lities Yes No
Hyperacute T waves No Yes

Pathologic Q waves No Yes

Evo l ution ST-segment c h a n g e initially, then T-wave change i n itial ly, then
T-wave change ST-segment change

QT prolongation No Yes

65
Peri card i a ! Disease

Cl states that a re pote n t i a l ly a menable to such t herapy ( for The jugu lar venous pressure is elevated i n nearly all patients.
example. autoi m m u ne d i sorders. u remic pericardi tis) . A with prominent x and y descents. Physical findings t hat also
CONT.
3-month course o f' predn i sone ( 0 . 25 to 0 . 50 mg/kg daily may be present include a Kussmaul sign (jugular vein engorge­
starti ng dose) may be used in t hese circumstances. with a ment with i nspiration) . pericardia! knock. pulsus pa radoxus.
slow taper beginning at 2 to 4 weeks. Recurrent pericarditis pleura l e ffusion. congestive hepalomegaly. and peripheral
has been reported to be more com mon among pa tients pre­ edema or asciles. In patients with long-standing constrict ive
scribed glucocorticoids. but t hese reports have been ham­ pericarditis. hepatic fai lure a nd cirrhosis may be present.
pered by t he t,e ndency to use t hese agents i n patients whose The diagnosis of constri c tive pericard i t is can be made
pericarditis has been refractory to other t herapies. Thus. glu­ with a deta i led hemodynamic evaluation using e i t her
cocort icoids are sti l l considered to be an effect ive al ternal ive Doppler echocardiography or cardi ac cat heterizat ion . The
t he rapy for these patients. basis for the d iagnosti c hemodynamic fi ndi ngs i n constric­
A fter an episode of'acule pericard it is. as many as 30% o r t ive pericarditis i s the concept o f en hanced ventricu l a r i nter­
patients develop recurre n t pericard i t is. There is a low (<l 'Y.. ) depen dence. which classical ly resul ts i n equalization o f' d ias­
risk of' developing constrict ive pericardi tis. Recurre n t peri­ tol ic pressures i n all heart chambers. The nonco m p l i a n t
card it i s can be t reated w i t h a n t i - i n fl a m m a tory agents wi t h pericardium also prevents the complete t ransmission of res­
a s lower. longer taper ( for example, 4 m on t hs) . t he add i t ion piratory changes i n t horacic pressure to the cardiac cham­
of col c h ici ne. or the use o f glucocorticoids. Etiologies of bers. Fi l l ing of t he right and left ven tricles varies sign i fi ca n t ly
acute perica rd i t is ot her t han v i ral or idiopa t h i c causes with respiration owing to ma rked changes i n t he early dias­
should be suspected . Cl tolic gradient emptying i n to t hese chambers ( t hat is. disso­
ciation of l horacic-cavitary pressures) . D uring i nspira t ion,
K EY P O I NTS
t he decrease i n thoracic pressure leads to relat ively less left
• Echocardiography should be performed i n patients with
ven t ricular f1 1 l i ng. while the i ncrease i n caval blood flow aug­
suspected acute pericarditis to evaluate for the presence ments right ven t ricular preload. Reciprocal cha nges i n ven­
of a hemodynamically significant pericardia! effusion; t ricular loading occur during expiration (Figure 22) . Because
however, the absence of an effusion does not rule out
the diagnosis.
• First-line therapy for acute pericarditis is high-dose
NSAIDs; colchicine can be used in conjunction with
standard NSAID therapy. 200

• Glucocorticoids in the treatment of acute pericarditis


are reserved for patients with contraindications to
NSAIDs and those with refractory pericarditis.

CJ Constrictive Perica rditis


Clinica l Presentation and Evaluation
Constrictive pericarditis is a chronic disorder that resu lts f'rom
pericardia! i n flammation . fibrosis. and possibly calcification.
leading to a loss of elast icity. Although many cases are idio­
pathic, causes of constrict ive pericard i t is i ncl ude chest rad ia­
t ion t herapy. cardiac su rgery. t rauma . post- myocardia l
i n farction syndromes. and systemic diseases t hat a rfect the
pericardium. such as connect ive t issue disease. mal ignancy.
tuberculosis. and other i n fect ions.
The pericard i u m i n co n s t r i c t ive perica rd i t i s is rigid a n d
noncompliant. res u l t i ng i n a total cardiac volume t ha t is
largely fixed. Ven t ricular fi l l ing occurs rapidly i n early diastole
a nd term inates abruptly near m id-diastole owing lo lhe peri­ F I G U R E 2 2 . Comparison of hemodynamics of constrictive pericarditis versus
cardia! restraint. With disease progression . the impairment i n restrictive cardiomyopathy. In constrictive pericarditis (top panel), there is sign ifi·
diastolic ti l l i ng and i ncreased intracardiac fi l l i ng pressures cant enhancement of ventricular interdependence leading to discordance of the
leads to reduced end-diastolic ven t ricular volume and stroke left ventricular and right ventricular pressures during respiration. Arrows indicate
onset of expiration and subsequent respective changes in left ventricular and right
volume. As a resu l t . symptoms and signs of climi n ished cardiac
ventricular systolic pressures. I n restrictive cardiomyopathy (bottom panel), there is
output (such as fat igue) and evidence of volume overload char­ evidence of early rapid ventricular fi l l i ng, but the ventricular pressures concord·
acterize t he clinical presentation o f' constrict ive pericardilis. antly rise and fall during respiration. LV = left ventricle; RV = right ventricle.

66
Pericard i a ! Disease

Cl constrictive pericard i t i s does not i nvolve the ventricular sep­ D ifferentiation o f constrictive pericard i tis from restrictive
tum, bu lging of t he septum towards t he left occu rs d u ri ng cardiomyopa t hy is c l i n i ca l ly i mport a n t because surgical
CONT. . . . . . .
111sp1rat10n a n d returns toward s t h e ng h t d u ri ng exp1rat1on, t reatment of' constrictive pericard i t is may be curative of t he
leading lo marked e n hancement of ven tricular i n terdepend ­ condition , but t here are no therapies t h a t sign i fi c a n t ly
ence. The dissociation of thoracic and i n t racavitary pressu res improve the natural h istory or restrict ive carcliomyopat hy.
and t h e enhanced ventricular i nterdependence lead to recip­ Other echocardiographic fi ndings supportive of t he diag­
rocal changes in fil l i ng and emptying of the right and left nosis of constrictive pericarcl i t is a re i ncreased pericardia!
ventricles, which manifest as a ltera tions in right- and left­ t hickness and plet hora of t he i n ferior vena cava. Pericardia!
sided forward stroke volumes during respiration (see Figure t h i ckening. with or without calcification. can also be detected
22) . These fi ndings a re d istinct from restrict ive cardiomyopa­ with cardiac CT or cardiac magnetic resonance ( C M R ) imaging
t hy, in which t here is not signi ficant enhancement of ven­ (Figure 23) .
tricular i n terdependence or d issociation of i n t racavitary­ A subset of patients w i t h chronic constrictive pericarcli­
intrathoracic pressures (Table 30) . Restrictive cardiomyopathy tis develop effusive constrict ive pericard itis. There typical ly
is c h a racterized by severe l e ft ven t r i c u l a r diastolic dys­ is m i ni m a l pericard ia! fluid present in const rictive pericardi­
fu nction due to myocardial abnorma l i ties or i nfil trat ion. t is : in some patients. however. perica rd i a ! i n flammation

TABLE 30. Characteristics of Constrictive Pericarditis, Restrictive Cardiomyopathy, and Cardiac Tamponade
Characteristic Constrictive Pericarditis Restrictive Cardiomyopathy Cardiac Tamponade

Mechanism Rigid, inelastic pericardium Myocardial d isease with severe I ncreased i ntra pericard i a l
resulting in fixed card iac d iastolic dysfunction pressure compromises
volume ventricular fil l i n g

Hemodynamic i m p l ications Rapid a n d l i m ited diasto lic Diastolic abnormalities lead to Ventricular fi lling is impaired
fi lling d u e to pericardia! elevated ventricular fi lling with a reduction i n cardiac
restra int; i n creased intracardiac pressures and pu l monary output; jugular ven ous and
fi l l i ng pressures, d i m i n ished hypertension pul monary venous pressures
cardiac output, and vo l u me increased
overload

Physical examination i JVP, Kussmaul sign, i JVP, Kussmaul sign, i JVP, pulsus paradoxus,
pericardia! knock, d i m i nished pericard i a ! knock, d i m i nished hypotension, reflex tachycardia
apica l i m p u lse, congestion a pical i m p u l se, congestion
(peripheral edema, pleural (peripheral edema, pleural
effusion, congestive effusion, congestive
hepatopathy) he patopathy)

El ectrocard iography No characteristic fi ndings No cha racteristic fi n d i ngs Decreased voltage, sinus
tachycard ia, electrical a lternans

Chest radiography Perica rd ia! calcification Atrial enlargement Enlarged cardiac silhouette
("water-bottle" heart)

Echocardiography Pericard i a ! thickening/ M a rkedly d i l ated atria with Pericardia I effu sion, d iastolic
cal cification; respiratory preserved systolic function; collapse of right atrium and
variation in fi l l ing of right a n d severe i m pairment of ventricle, enlargement of
left ventricles; ventricular septa I myocardial relaxation i nferior vena cava, ventricular
sh ift d u ri ng respiration; septal shifting d u ring
plethora of i nferior vena cava respiration, respi ratory
changes in mitral inflow

CMR i magi ng/CT Pericard ia! thickeni ng/ Normal pericard i u m Pericardia! effusion
calcification

Hemodynamic right and left Prom i n entx and y desce nts; Promi nent x and y descents; Promi nent x descents and
heart catheterization elevated atrial pressures; el evated atrial pressures; b l u nted y descents; elevated
elevated and equa lized concordance of LV and RV atrial pressures; blunting/loss
d ia stolic LV and RV pressures pressures d u ring respiration of early ventricular diasto lic
(within 5 mm H g ); decreased fi l l ing wave
transm ission of intrathoracic
pressure causes d i scordance of
LV and RV pressures and stroke
vol u me d u ring respiration
(enha nced ventricular
interdependence)

BNP Normal o r m i n imally elevated Elevated Normal or low

BNP = B-type natriuretic peptide; CMR = cardiac magnetic resonance; JVP = j ugular venOlJS pressure; LV = left ventricular; RV= right ventricular.

67
Perica rd i a ! Disease

F I G U R E 2 3. Constrictive pericarditis. Chest radiograph (top left panel) showing


pericardia! calcification (arrows). CT scan (top right panel) demonstrating pericardia I
th ickening (arrows). Gross pathology (bottom left panel) demonstrating severe
pericardia! calcification.
Pathology image courtesy of Dr.William D. Edwa<ds. Department of Pathology, Mayo Clinic.

Cl resu l t s i n a n effusion L hat i s placed under pressure by the Management


inelastic pericard i u m . A l t hough L h i s does not typically cause Some patients w i t h constrict ive pericard i t i s can have reversi­
CONT.
t he hemodynamic changes seen w i t h ca rd iac t a rn ponade. i t ble or t ransient i n flammation . such as post-pericardiotomy
rnay change t he c l i nical presenLa t i on . Symptoms of low car­ syndrome after cardiac su rgery, that responds to an t i-i n flam­
d iac output . syst e rn i c congesl ion, and <l ll e!evaled j ugular matory agents. Thus. i t may be reasonable to perform a t rial of
venous pulse occur. as i n con s L r i c t ive pc ri card i t is : however, a med i c a l t herapy (such as NSAJDs) before p c r i c a rd i ect omy
pericardia! knock may be absent and t he y desce n l of the surgery in some patients presenting w i t h const rictive pericar­
jugular venous pu lse may be less pro m i nent. The etiology is d i t is. part icula rly t hose w i l h m i ld symptoms. a potentially
s i m i l a r Lo t hat of' chronic constrict ion. a l t hough tu bercu lous reversible cause of acute i nfla m mation . and no evidence of
pericard i t i s i s particularly prone to Lhis cl i n ical presentat ion . c h ronic constri ctio n. Diuretic t herapy may reduce vol u me
Effusive con s l r i c l ive pericard i l i s s hould be suspecled in a overload and i m prove symptoms t ransiently.
patient wi t h residual symptoms a n d signs of persislent atrial Cardiac surgery with pericardiectomy is the defi n i t ive
pressure e levation after successful pericard iocentesi s. The treatment for relief of' heart fai lu re i n patients w i t h constric­
d iagnosis can be rnade w i t h e i t h e r echocard iography or car­ t ive pericardi t is. Because of t he compl ex i ty of t h e procedure.
diac catheteriza l i o n . t h i s surgery is best performed in experienced centers where a

68
Perica rd i a ! Disease

Cl total pericardiectomy, with removal of as much of the pericar­ pulsus paradoxus ( a decrease i n the systolic pressure of >10
dium as is technically possible, may be achieved. Patients mm Hg with i nspiration) in these patients. As compensation
CONT.
treated with total pericardiectomy have been shown to have for the reduced stroke volume and hypotension. tachycardia
i mproved outcomes relative to those who undergo more l i m ­ is usual ly present.
i ted procedures. Predictors of poor outcome after surgical Less common presentations of cardiac tamponade also
pericardiectomy i nclude advanced age. severe symptoms. need to be considered i n some patients. A loculated pericardia!
chronic kidney disease, pul monary hypertension . left ven­ effu sion may occur after cardiac surgery or in other postopera­
tricu lar dysfunction. and radiation t herapy as t he underlying t ive settings. It may occur in the posterior pericardiaJ space
cause of constrictive pericarditis. In one study, the 7-year sur­ adjacent to the atria. posing challenges for detection by echo­
vival after pericardiectorny was 27%, 66%, and 88%, respectively. ca rdiography. Posterior loculated effu sions should be suspected
for patients with constrictive pericarditis due to radiation. prior i n postoperative patients with hemodynamic instability. Low­
cardiac surgery, and an idiopathic cause. Medical therapy with pressure cardiac tamponade occurs without elevated jugular
diuretics can i mprove symptoms or be pal liative in patients who venous pressure because the intracardiac fil l i ng pressures are
are not surgical candidates, but the chronic nature of the disor­ low. Examples include patients with malignancy or tuberculo­
der can prove to be drng-refractory. Cl sis complicated by severe dehydration. Pneumopericardium
with cardiac tamponade may resu l t from gas-forming bacterial
KEY POI NTS
pericarditis after penetrating chest trauma.
• I n some patients with constrictive pericarditis, particu­ Cardiac tamponacle should be suspected when there is a
larly those with mild symptoms, a potentially reversible compatible h istory, hypotension. and an elevated j ugular
cause of acute inflammation, and no evidence of venous pressure and pulsus paradoxus. An enlarged cardiac
chronic constriction, it may be reasonable to perform a silhouette may be seen on chest racliograph ( "water-bottle
trial of medical therapy before surgery. heart") . The ECG typically demonstrates sinus tachycardi a
• Cardiac surgery with pericardiectomy is the definitive and electrical a l ternans. Echocardiography readily detects
treatment for relief of heart failure in patients with con­ pericardia! effusions and i s the primary modal i ty for diagnos­
strictive pericarditis. ing cardiac tamponade (Figure 24) . Signs of cardiac tarnpon­
ade i nclude diastolic col lapse of t h e right atrium and right
ventricle, ven tricular septa! s h i ft i n g w i t h respiration, and

CJ Ca rdiac Ta mponade enlargement of t he i n ferior vena cava . With Doppler echocar­


d iography, respi ratory variation in m i t ra l i n flow can be
Clinica l Presentation a n d Evaluation detected early in the evolution of tamponade. Moreover. t h e
The normal pericardium contains 15 to 50 m l o f plasma u l t ra­ changes i n m i tral i n flow are highly sensitive, and may pre­
filtrate. The rate of accumulation and the absolute amount of cede changes in cardiac output. b lood pressure, and other
pericardia! fluid surrounding the heart above the normal base­ ecbocardiographic evidence o f tamponade. Respiratory
l i ne level determine t he hernodynamic effects of an effusion. changes in m i t ral i n flow resolve after pericardiocentesis
Cardiac tamponade occurs when i n t rapericardial pressure un less an e ffusive-constrictive physiology is present.
exceeds intracardiac pressure. leading to i mpairment of ven­ The signs of cardiac tamponade also can be evident with
tricular fil l i ng t hroughout t he entire diastolic period. i nvasive hemodynamic catheterization. Characteristic findings
Virtually a ny disorder that causes a pericardia! effusion include prominent x descents and blunted y descents on the
can result in cardiac tamponade. Malignancy is t he most com­ atrial tracings (Figure 25) and elevated ventricular diastolic
mon atraumatic etiology, w i t h breast and l u ng cancer being pressures with blunting or loss of t he early rapid ventricular
t he most frequent. O t her i mporta n t causes are idiopathic or fil ling wave. which is the hal l mark of cardiac tamponade that
viral pericard i t is, complications of invasive cardiac proce­ distinguishes it from other diastolic disorders. O ther hernody­
dures. aortic dissection with disruption of the aortic valve namic findings i nclude equalization of left- and right-sided
annulus, tuberculosis, uremia, and pericarditis or ventricular end-diastolic pressures, reduced cardiac output. and altera­
rupture from a myocardial i nfarction. t ions in the systolic ejection period or pulse pressure that
During cardiac tamponade, venous return and ventricu­ resul t from decreased stroke volume (analogous to pulsus
lar fil ling become i mpaired owing to excess i n t rapericard i a l pa radoxus) .
pressure. T h i s impa irment precipitates elevation a nd equali­
zation o f d iasrolic pressu res in the heart, i ncl u d i ng i nc reased Management
pulmonary venous and right atrial pressures. a nd u l t i mately. I ntravenous normal sali ne and. if needed. vasopressors should
a reduction i n cardiac output. With inspiration. fill i ng pres­ be administered to stabil ize the hernodynamics i n a patient
sure to the left ventricle fal l s , leadi ng to a reduction in t h e with cardiac tamponade. Although t he presentation includes
stroke volume. T h e fal l i n left ventricular stroke volume dur­ symptoms and signs of heart failure, reduction of i ntravascular
i ng i nspiration manifests as a relative decrease in pulse pres­ volume by diuresis may exacerbate the hemodynarnic abnor­
sure or peak systolic pressure and is the hal lmark finding of malities of tamponade.

69
Pericard i a ! Disease

F I G U R E 2 5 . I nvasive hemodynamic features of card iac tamponade. Y-axis


i n d icates pressure (mm Hg). Hypotension and pulsus paradoxus (asterisk) i n the
femoral artery (FA) pressure tracing a n d loss of the y descent i n the right atrial
(RA) pressure tracing are evi dent ( top panel). Following pericardiocentesis, there
is a rise in arterial pressure a n d return of the y descent i n the RA pressure tracing
(bottom panel).

Pericardiocentesis with echocardiographic guidance is CJ


the most expeditious treatment for cardiac tarnponade, with a
CONT.
lower complication rate than blind or ECG-guided pericardio-
centesis. Pericardiocentesis should be avoided when treating
tamponade associated with aortic dissection. In these patients,
abrupt return of ventricular ejection may exacerbate the
dissection and precipitate acute decompensation. During
attempts at needle passage with pericardiocentesis, the tense
pericardium may discharge fluid from the pericardia! effusion
into the pleural space. This effect can be immediately recog­
nized on echocardiography and may obviate further attempts
at pericardiocentesis, as acute relief of tamponade may occur.
Although the vast majority of pericardia! effusions can be
treated percutaneously, some still require surgical drainage. A
surgical approach also may be required for viscous or loculated
effusions, those resulting from bacterial infections, posterior
effusions that can be difficult to approach from any thoracic
approach, and malignant effusions with a high likelihood of
recurrence. I n addition, recent hemorrhage into the pericar­
diw11 may resul t in pericardia! clot formation that can be dif­
ficult to remove with a catheter. Pericardiocentesis in the set­
F I G U R E 2 4 . Cardiac tamponade. Chest radiograph (top panel) showing "water­
ting of aortic dissection may exacerbate the intimal tear owing
bottle heart." Transthoracic echocardiogram (middle panel) showing a large pericar­
to the abrupt increase in stroke volume that occurs with relief
dia I effusion (arrows). Gross pathology ( bottom panel) of cardiac tamponade.
LV = left ventricle; RV = right ventricle. of cardiac tamponade. Surgical therapy most commonly con­
Pathology image courtesy of Dr. William D. Edwards, Department of Pathology, Mayo Clinic. sists of a pericardia! window to allow drainage of fluid: some

70
Va l v u l a r H e a rt Disease

Cl tomy can be beneficial when there is a diagnostic need for


patients require an open pericardiectorny. Open pericardiec­ intervention compared with medical therapy or expectant
management, patient desi res, and local resources for sur­
CONT.
pericardia! tissue. Cl gery or intervention.
KEY POINT In many patients with chronic VHD, the development of
symptoms is insidious and their onset may go unnoticed.
• Pericardiocentesis with echocardiographic guidance is
Typical symptoms include exertional dyspnea, edema, and
the most expeditious treatment for cardiac tamponade.
orthopnea. A thorough history provides the opportunity to
assess daily activity and lifestyle changes that may effectively
"mask" symptoms. This is important as the presence or
Valvu la r Heart Disease absence of symptoms may play a pivotal role in deciding tim­
ing of intervention.
Pathophysiology of Va lvu lar The physical examination, including cardiac auscultation,

Heart Disease is the key first step in identifying the presence ofVHD and the
likely valve or valves involved and in helping to guide an
Valvular heart disease (VHD) comprises cardiac dysfunction
appropriate diagnostic and therapeutic approach ( Figure 26 ) .
due to structural or functional abnormalities of the cardiac
A thorough cardiac examination entails determining the
valves. Dysfunction results from either failure of the valves to
intensity of the murmur as well as specific characteristics
competently close (regurgitation) or to effectively open (steno­
that help define whether it is a clinically significant murmur.
sis) . Causes of VHD have changed markedly in developed
Not all murmurs require further evaluation (specifically
countries over the past 50 years, with a shift away from rheu­
echocardiography) .
matic disease and toward calcific, degenerative, and congenital
The clinical grading of both systolic and diastolic mur­
causes. Rheumatic heart disease continues to be the primary
murs is based on a scale of 1 to 6 (Table 31 ) . Diastolic mur­
cause of VHD in the developing world.
murs are always considered abnormal and require further
The effect of VHD on the heart and on the patient varies
evaluation. Systolic murmurs may be either innocent or
based on the affected valve and the mechanism of dysfunction:
indicative of significant valve disease. Some murmurs are
• Aortic stenosis causes chronic pressure overload that typi­
associated with normal valves but abnormal systemic pro­
cally leads to concentric left ventricular (LV) hypertrophy
cesses (for example, anemia and other high-output states,
with increased wall thickness and normal chamber size to
such as hyperthyroidism) or increased flow (for example, a
compensate for increased LV afterload.
systolic murmur in the setting of aortic regurgitation) .
• Chronic aortic regurgitation causes increased LV preload I nnocent murmurs are characteristically brief and are asso­
and afterload, leading to increased LV volume and mass. ciated with normal heart sounds and no hemodynamic
• Mitra! stenosis causes increased pressure within the left abnormalities. Patients with grade 1 or 2 midsystolic mur­
atrium (LA), leading to increased pulmonary venous pres­ murs who are asymptomatic with no associated findings
sure, pulmonary hypertension, and atrial dilation. and those with continuous murmurs suggestive of a venous
• Chronic mitral regurgitation causes volume overload (in­ hum or mammary souffle (a continuous murmur heard
creased preload) of the LV and LA, leading to increased LA over the breast in lactating women) do not warrant echo­
size and pressure and LV dilation. cardiographic evaluation.
The character of a murmur in response to clinical
Many of these changes in cardiac structure occur gradu­
maneuvers is helpful in localizing the flow disturbance to a
ally because of physiologic mechanisms that compensate for
specific valve. For example, in general, right-sided murmurs
the increased loading conditions. For this reason, patients
increase with inspiration and left-sided murmurs increase
remain asymptomatic for some time even in the setting of
with expiration. A Valsalva maneuver decreases the length
significant valve disease. The exception to this is acute valve
and intensity of most murmurs, except for systolic murmurs
lesions, in which the compensatory changes do not have suf­
associated with hypertrophic obstructive cardiomyopathy
ficient time to develop.
and mitral valve prolapse. Exercise causes the murmur of
mitral stenosis to get louder, whereas isometric maneuvers,
such as handgrip, increase the intensity of regurgitant mur­
Diag nostic Eva l uation
murs such as aortic and mitral regurgitation. Postural
of Va lvu l a r Heart Disease maneuvers are ideal for differentiating hypertrophic obstruc­
History and Physical Examination tive cardiomyopathy and mitral valve prolapse from other
The initial evaluation of the patient with possible VHD murmurs as they are louder with standing and softer with
serves to diagnose, quantify, and assess mechanisms of squatting. Physical examination findings, such as an enlarged
valve dysfunction. The evaluation should focus on disease or displaced apical impulse, abnormal peripheral pulses, and
severity, symptoms, quality of life, expected benefits of timing and intensity of heart sounds (including extra heart

71
Va l v u l a r H e a rt D isease

Physical examination
Systolic m u rm u r <". grade 3/6
Diasto l i c or conti nuous m u rm u r
Symptoms

Transthoracic echocardiography
Ventric u l a r size and function
Pulmonary pressu res
Measures of disease severity

If poor image quality on I f symptoms are


transthoracic echocardiogram equivocal

_f
Transesophageal echocardiography Plasma B-type natriuretic peptide level
Measures of disease severity and/or stress echocardiography
Exercise tolerance
Exercise p u l m on a ry pressures

If not well seen


by echocardiography

CT or MRI
Aorta assessment: ascending, arch, descending
Cardiac chamber size and function: ventricular vo l u me

F I G U R E 2 6 . Diag nostic evaluation of valvu lar heart disease.

TABLE 3 1 . Clinical Grading of Murmurs clues (such as a dilated left ventricle in the setting of chronic
aortic regurgitation) , but its utility in the evaluation of VHD is
Grade Description
less than other modalities, especially echocardiography.
Fai ntest murmur that can be heard (with difficulty) Transthoracic echocardiography (TIE) with color flow
2 Faint m u rmur but ca n be ide ntified i m mediate l y and spectral Doppler imaging is the primary noninvasive
3 Moderately loud m u rmur means of evaluating cardiac murmurs as well as the primary
means for following the course of VHD. TIE is indicated for
4 Loud murmur associated with a palpable thrill
patients with the following (see Figure 26) :
5 Very loud murmur but cannot be heard without
the stethoscope • Systolic murmurs <':grade 3/6 or late or holosystolic

6 Loudest m u rm u r; ca n be heard without a murmurs


stethoscope
• Diastolic or continuous murmurs

• Murmurs with accompanying symptoms

sounds such as an S 3 or S4 or a systolic ejection click) may TTE permits assessment of LV size and function (systolic
help identify specific valve lesions as well. Common valve and diastolic) , estimation of pulmonary artery pressure (an
lesions and heart murmurs are further described in Table 32. indirect estimate based on the tricuspid regurgitant jet veloc­
ity) , and determination of disease severity. Echocardiography
Laboratory and I maging Tests also helps establish a reference point for future comparisons if
Electrocardiography (ECG) and chest radiography are often significant abnormalities are detected. Transesophageal echo­
performed during the initial evaluation. ECG provides valuable cardiography (TEE) provides improved image quality over TIE
insight into rhythm (such as the presence of atrial fibrillation) as the probe has only the thin tissue of the esophagus through
and cardiac chamber enlargement. Chest radiography is essen­ which to image. TEE is indicated for patients with severe dis­
tial when interpreting dyspnea or clinical signs of heart failure ease who need further quantification, better identification of
with evidence of cardiac enlargement and pulmonary vascular leaflet involvement (particularly for patients with mitral
redistribution. Chest radiography may also provide secondary regurgitation) , or when poor image quality on TTE limits

72
Va lvu l a r H e a rt D isease

TABLE 32. Valvular and Other Cardiac Lesions aiid'Their Associated Examin�fion Findings
Cardiac Condition Characteristic Location Radiation Associated Severity and
Murmur Findings Pitfalls

Aortic stenosis Mid-systol ic; RUSB Right clavicle, Enlarged, Severe aortic
crescendo- carotid, apex nondispl aced stenosis may
decrescendo a p ical impu lse; S4 ; i nclude decreased
bicuspid valve A2 ; h i g h-pitched,
without cal cification late peaking
will have systol i c murmur;
ejection click d i m i n ished and
fol lowed by delayed carotid
m u rmur upstroke

Rad iation of
m u rm u r down the
descending
thoracic aorta may
mimic mitral
regurgitation

Aortic regurgitation Diastolic; LLSB (valvular) or None Enlarged, Acute, severe


decresce ndo RLSB (dilated aorta) displaced apical regurg itation
i m pulse; S3 or S4; murmur may be
increased pulse masked by
pressure; bounding tachyca rdia, short
ca rotid and d u ration of murmur
peripheral pu lse
Severity i n chronic
regurg itation is
difficult to assess by
auscultation

Mitral stenosis Diasto l ic; low Apex (heard best in None Opening snap after Interval between S2
pitched, left lateral s 2 if leaflets mobi le; and opening snap
decrescendo decubitus position) irregular pulse if is short i n severe
atria l fi bril lation mitral stenosis
present
Intensity of murmur
correlates with
transva lvular
gradient

P2 may be loud if
pulmonary
hypertension
present

Mitral regurgitation Systo l ic; holo- or Apex To axilla or back; Systolic click in Acute, severe
late systolic occasiona l ly mitral valve regurgitation may
anteriorly to prolapse; S3; have soft or no
precord i u m apical i m pulse holosystolic
hyperdynamic and murmur, mitral
may be displaced i nflow rumble, S3
if dilated left
ventricle; i n m itral
valve prolapse,
Va lsalva ma neuver
moves onset of
clicks and murmur
closer to S 1 ;
handgrip increases
murmur intensity

Tricuspid stenosis Diastolic; low LLSB Nonradiating El evated central Low-pitched


pitched, venous pressure frequency may be
decrescendo; with prominent a difficult to
increased intensity wave, signs of auscu ltate,
d u ri ng inspiration venous congestion especially at higher
(hepatomegaly, heart rate
ascites, edema)

(Continued on the next page)

73
Va l v u l a r H e a rt Disease

TAB LE 32. Valvular and Other Cardiac Lesions and Their Associated Examination Findings (Continued)
Cardiac Condition Characteristic Location Radiation Associated Severity and
Murmur Findings Pitfalls
Tricuspid H olosysto l ic LLSB LUSB Merged and Right ventricular
reg u rgitation prominent c and v i m pulse below
waves i n jugular stern u m
venous pulse;
Pulsatile, enlarged
murmur i ncreases
liver with possible
d u ring i ns piration
ascites

May be higher
pitched if
associated with
severe pulmonary
hypertension

Pu l m onary stenosis Systolic; crescendo- LUSB Left clavicle Pu lmonic ejection Increased i ntensity
decrescendo click after S1 of murmur with late
( d i m i nishes with pea king
inspiration)

Pulmo nary Diastolic; LLSB None Loud P2 if Murmur may be


regurgitation decrescendo pul monary m i n i m a l or absent if
hypertension severe due to
present m i n i m a l difference
i n pul monary
a rtery and right
ventricular di astolic
pressures
I n nocent flow Midsysto l ic; grade RUSB None N ormal intensity of May be present i n
m u rm u r 1 / 6 or 2/6 i n {
A normal splitting conditions with
intensity o S2 ; no radiation increased flow
(e.g., pregnancy,
fever, a n emia,
hyperthyroidism)

Hypertrophic Systol ic; crescendo- LLSB None Enlarged, Murmur may not be
obstructive decrescendo hyperdynamic present in
cardiomyopathy apical impu lse; nonobstructive
bifid carotid hypertro phic
i m pu lse with delay; cardiomyopathy
increased intensity
d u ring Va lsalva
maneuver or with
squatti ng to
standing
Atrial septa I d efect Systol i c; crescendo- RUSB None Fixed, split S2 ; right May be associated
decrescendo ventricular heave; with p u l monary
rarely, tricuspid hypertension,
inflow murmur including increased
intensity of P2 ,
pulmonary valve
regurgitation
Ventricular septa I Holosysto lic LLSB None Pa lpable thri l l ; M u rm u r intensity
defect murmur increases and d u ration
with ha nd-grip, decrease as
decreases with pul monary
amyl n itrite hypertension
develops
(Eisenmenger
syndrome)

Cyanosis if
Eisenmenger
syndrome develops

A2 = aortic valve component of 52; LLSB = left lower sternal border; LUSS = left upper sternal border; P2 = pulmonary valve component of 52; RLSB = right lower sternal border;
RUSB = right upper sternal border.

74
Va l v u l a r H e a rt Disease

image interpretation. The disadvantage compared with TIE is and a history of coronary artery disease, suspected myocardial
that it is invasive, requiring sedation and technical expertise ischemia, or LV systolic dysfunction; in men older than 40
distinct from TIE. years and postmenopausal women; in patients with one or
For stenotic lesions, Doppler-derived velocities across the more cardiovascular risk factors; and in patients i n whom
affected valve allow calculation of pressure gradients and valve mitral regurgitation is thought to be caused by LV dysfunction.
area. Based on the conservation of mass principle, the meas­ These patients may require concomitant revascularization at
ured flow through one heart chamber region and measured the time of surgery.
velocity of blood across the region (stenotic valve) of interest For patients in whom symptoms are equivocal, adjunctive
can be used to calculate the area of the steno tic valve. evaluation (such as B-type natriuretic peptide measurement,
Assessment of severity for regurgitant valves includes exercise evaluation for tolerance/symptoms, and blood pres­
color Doppler jet size and the width of the narrowest segment sure response to exercise) , assessment of pulmonary artery
of the regurgitant jet (vena contracta). Additional quantitative pressures, and LV outflow tract gradient (by TIE or cardiac
measures for regurgitant lesions, such as effective regurgitant catheterization) may be helpful.
orifice area, regurgitant volume, and proximal isovolumic
KEY PO I NTS
surface area, although technically more demanding to acquire,
are strongly associated with prognosis and should be meas­ • Patients with grade 1 o r 2 midsystolic murmurs who are HVC
ured routinely in patients with moderate or severe regurgitation asymptomatic with no associated findings and those
by color Doppler jet size. Three-dimensional echocardiography, with continuous murmurs suggestive of a venous hum
both transthoracic and transesophageal, provides additional or mammary souffle do not warrant echocardiographic
information such as feasibility of repair (rather than replace­ evaluation. •

ment) and location of paravalvular leaks. Additional imaging, • Transthoracic echocardiography is indicated for the
such as cardiac magnetic resonance (CMR) imaging or multi­ evaluation of valve disease i n patients with systolic
detector CT (MDCT) , may provide information on LV func­ murmurs grade 3/6 intensity or greater; late or holosys­
tion, aortic dimensions, chamber sizes and volumes, and tolic murmurs; diastolic or continuous murmurs; or
coronary anatomy. Consistency of quantitative evaluation is any murmur with accompanying symptoms.
important. Simple measurements, such as LV outflow tract
diameter, may vary greatly in the same patient when obtained
at different times and by different operators. These differ­
Genera l Pri nciples of Ma nagement CJ
ences may affect valve area calculations and influence timing of Va lvu l a r Heart Disease
of surgery. Treatment is guided by valve lesion severity, which can be
Cardiac catheterization can provide important hemody­ classified into four stages (A to D) in a system similar to that
namic information, especially when the severity of the valve used for patients with heart failure (Table 33) .The mainstay of
disease as measured by noninvasive testing is not consistent treatment for VHD. both stenosis and regurgitation, is cen­
with symptoms or physical examination severity, or when tered on interventions to treat the mechanical problem. The
accurate assessment of pulmonary pressures or detection of goals of t reatment are both to improve symptoms and reduce
intracardiac shunts is needed. Coronary angiography is rec­ the risk of complications such as i rreversible ventricular dys­
ommended before valve surgery in patients with severe VHD function, pulmonary hypertension, and death. The most

TABLE 33. Stages of Progression of Valvular Heart '.Disease


Stage Definition Description

A At risk Patients with risk factors for development of VHD


B Progressive Patients with progressive VHD ( m i ld-to-moderate severity and asymptomatic)
c Asymptomatic Asymptomatic patients who meet criteria for severe VH D:
severe
C 1 : Asymptomatic patients with severe VHD i n whom the left o r right ventricle remains
compensated

C2: Asymptomatic patients with severe VHD, with decompensation of the left or right ventricle
D Symptomatic Patients who have developed symptoms as a result of VHD
severe

VHD = valvular heart disease.

Reprinted with permission of Elsevier. Science a n d Technology Journals, from Nishimura RA, Otto C M , Bonow RO, e t al. 2 0 1 4 AHA/ACC guideline for the management o f patients
with valvular heart disease: executive summary: a report of the American College of Cardiology/American Heart Association Task Force on Practice Guidelines. J Am Coll Cardiel.
2 0 1 4 Jun 1 0;63(22):2438-88. (PMID: 246031 92]; permission conveyed through Copyright Clearance Center, Inc.

75
Va lvu l a r H e a rt Disease

Cl other
com mo n i ndica t i o n for t reatment is sy mptoms (Table 34) but preve n t i ng progression of aortic stenosis. have not been
fac tors. i ncluding chamber clys f'u nct ion . chamber shown to be usef'u l .
CONT.
enlargemen t . a ncl pulmonary hypertension may be i nd ica­ T h e pri mary therapy f·o r patients with severe V J-I D is valve
tions lor t reat men t. I n genera l . for bot h stenotic and regu rgi­ repa i r or valve replacement w i t h ei t h er bioprosthelic or
t a n t lesions. symptoms occur before t h e development of' mechan ical valves. For patients with h igh surgical risks, per­
ven t ricular dysfunction : however. some pat ients may have cutaneous techni ques for aortic valve replacement and m i t rnl
asy mptomatic ventricular dysf'unction . Medical t herapy does valve repair a re now ava i lable.
not stall the progression of' disease but is indi cated lor patients r:ollow-u p is essen tial in asymptomatic patients. as dis­
with symptoms and LV dysf'unction who are awa i t i ng valve ease progression may occur over t i me (Table 35) . After valve
repai r or replacement, as wel l as for those who are not opera­ repair or replacement. pat ients should ideally be followed i n
t ive cand idates. Preve n t ive measures, such as s t a t i n s lor conj u nction with a cardiovascular special ist. Cl

TABLE 34. Indications for Interventions for Valvular Heart Conditions


Valve Lesion Indications to Intervene• Interventions

Aortic stenosis Symptoms (class I ) Aortic valve replacement

LVEF < S O % (class I )

Moderate (class I l a ) or severe (class I) aortic


stenosis at time of other cardiac surgery

Abnormal blood pressure response (decrease i n


systolic blood pressure) during exercise (class Ila)

Asymptomatic patients with very severe aortic


stenosis (class Ila)

Aortic regurgitation Symptoms (class I) Aortic valve replacement

LVEF <SO% (class I)


Moderate (class Ila) or severe (class I) aortic
regurg itation at time of other cardiac surgery

LV d i l atation (end-systolic d i mension >SO mm)


(class Ila)

M itra I stenosis Symptoms (class I ) Percuta neous bal loon valvotomy (if anatomy
2 favora ble by echocardiography with less than
Very severe mitral stenosis ( MVA < 1 .0 cm ) a n d
moderate mitral regurg itation and no left atrial
no symptoms i f valve morphology favora ble for
thrombus)b
bal loon va lvotomy (class Ila)
M itral valve replacement
Severe mitral stenosis attime of other cardiac
surgery (class I)

Mitral regurgitation Symptoms (class I) M itra I valve repa ir if anatomy favora ble (presence
of a n n u l a r d i l ation, mitral leaflet prolapse, or
LVEF 30%-60% (class I)
myxomatous cha nges without cal cification or
LV end-systolic dia meter �40 mm (class I ) stenosis)

Moderate (class Ila) or severe (class I ) mitral M itral valve replacement


regurgitation at time of other cardiac surgery
Pu l monary hypertension ( PA systo l i c pressure
�SO mm Hg) (class I l a )

New-onset atrial fi bril lation (class Ila)

Tricuspid regurgitation Severe tricuspid regurgitation at time of surgery Tricuspid valve repair if anatomy favo ra ble
for left-sided valve (class I)
Tricuspid valve replacement ( b i oprosthetic)
Symptoms due to severe, primary tricuspid
regurgitation not responsive to medical therapy
(class I l a )

LV = left ventricle; LVEF = left ventricular ejection fraction; MVA = mitral valve area; PA = pulmonary artery.

iJStrength of recommendation: class l: procedure should be performed; class I l a : procedure i s reasonable.

bAll patients considered for percutaneous balloon mitral valvotomy should undergo transesophageal echocardiography to assess for left atrial appendage clot and mitral regurgi­
tation severity regardless of whether patient has sinus rhythm or atrial fibrillation.

Recommendations from Nishimura RA, Otto CM, Bonow RO, et al; American College of Cardiology/American Heart Association Task Force on Practice Guidelines. 2 0 1 4 AHA/ACC
guideline for the management of patients with valvular heart disease: executive summary: a report of the American College of Cardiology/American Heart Association Task Force
on Practice Guidelines. J Am Coll Cardiol. 20 1 4 Jun 1 0;63(22):2438-88. Erratum in: J Am Coll Cardiol. 2 0 1 4 Jun 1 0;63(22):2489. [PMIO: 24603 1 92]

76
Va lvular H e a rt D i s e a s e

TABLE Serial Evaluation o f Asymptomatic Patiehts with Left-Sided Valvular Conditions


35.

Factors Considered Lesion Severity Frequency


Aortic Stenosis

Stenosis severity; rate of progression; LV At risk (Vm ax <2 mis)


systolic function; ascending aorta d i l ation if
bicuspid aortic valve M i l d (Vrnax 2.0-2.9 mis or mean g rad ient Cli nica l eval yearly; echo every 3-5 y
<20 mm Hg)
Moderate (Vmax 3.0-3.9 mis or mean gradient Clinical eval yearly; echo every 1 -2 y
20-39 mm Hg)
Severe (Vrn ax �4 mis or mean g radi ent C l i nical eval yearly; echo every 6-1 2 mo
�40 m m Hg, AVA S:1 .0 cm2)
Very severe (Vrnax �5 mis or mean gradient C l i n ical eval yearly; echo every 6-1 2 mo
�60 m m H g )

Mitral Stenosis
Stenosis severity M i l d and mod erate ( MVA > 1 .5 cm 2 , Cli nical eval yearly; echo every 3-5 y
d iasto l ic pressure half-time < 1 50 msec)

Severe ( MVA S:1 .5 cm 2 , d iasto l i c pressure Clinical eval yearly; echo every 1 -2 y for
ha lf-time � 1 50 msec or �220 msec with MVA 1 .0- 1 .5 cm 2 , every year for MVA
very severe stenosis, PASP >30 m m H g ) < 1 .0 cm 2

Aortic Regurgitation
Regurgitation severity; rate of progression; M i l d (VC <0.3 cm, E R O <0. 1 0 cm 2 , C l i nical eval yearly; echo every 3-5 y
LV ejection fraction; LV chamber size; RV <30 m Ubeat, RF <30%); normal EF
ascending aorta d i l ation if bicuspid aortic
Moderate (VC 0.3-0.6 cm, ERO 0. 1 0- C l i nical eval yearly; echo every 1 -2 y
valve
0.29 cm 2 , RV 30-59 m Ubeat, RF 30%-49%)
Severe (VC >0.6 cm, ERO >0.3 cm 2 ,
RV �60 m Ubeat, RF �50%)
EF �50%; LVESD S:50 mm C l i nical eval every 6- 1 2 mo; echo every
6- 1 2 mo, more frequently for d i l ating LV
EF <50%; LVESD >50 mm C l i nical eval every 6- 1 2 mo; echo every
6- 1 2 mo, more frequently for d i l ating LV

Mitra! Regurgitation
Reg u rgitation severity; rate of progression; At risk (VC <0.3 cm) Clin ica l eval yearly; echo only if
EF; LV cham ber size symptomatic

M i l d and mod erate (VC <0.7 cm, ERO Clini cal eval yearly; echo every 3-5 y for
<0.40 cm 2 , RV <60 m Ubeat, R F <50%) mild severity, every 1 -2 y for moderate
severity

Severe (VC �O. 7 cm, ERO �0.4 cm 2 , Clin ical eval every 6- 1 2 mo; echo every
RV �60 mUbeat, RF �50%) 6-1 2 mo, more frequently for d i lating LV

AVA = aortic valve area; echo = echocardiography; EF = ejection fraction; ERO = effective regurgitant orifice; eval = evaluation; LV = left ventricle; LVESD = left ventricular end-sys-
tolic dimension; MVA = mitral valve area; PASP = pulmonary artery systolic pressure; R F = regurgitant fraction; RV = regurgitant volume; VC = vena contrada width; Vmax = maxi-
mum aortic jet velocity.

Recommendations based on Nishimura RA, Otto CM, Bonow RO, et al; American College of Cardiology/American Heart Association Task Force on Practice Guidelines. 2 0 1 4 AHA/
ACC g uideline for the management of patients with valvular heart disease: executive summary: a report of the American College of Cardiology/American Heart Association Task
Force on Practice Guidelines. J Am Coll Cardiol. 2 0 1 4 Jun 1 0;63(22):2438-88. Erratum in: J Am Coll Cardiol. 2 0 1 4 Jun 1 0;63(22):2489. [PMID: 24603 1 92]

KEY POI NTS Aortic Sten osis


• Medical t herapy does not stall the progression o f valvu­ Clinical Presentation and Evaluation
lar heart disease but is indicated for patients with
Aortic stenosis is the most common type o f valv u l a r
symptoms and left ventricular dysfunction who are
heart di sease in the United States a n d is typi cally caused
awaiting valve repair or replacement, as well as for
by calcific degeneration o f an ot herwise normal aortic
those who are not operative candidates.
valve. Other causes i nclude bicuspid aortic valve and
• The mainstay of t herapy for patients with severe valvu­
rheumatic valve d isease, the latter o f which remains
lar heart disease is either valve repair or valve replace­
common worldwide a nd is a l most always accompanied
ment with bioprosthetic or mechanical valves.
by m i t ral valve disease.

77
Va lvular H e a rt Disease

Calcific aortic stenosis is a disease of the elderly, and coro­ aortic jet velocity (V ma.l below 4 mis have moderate aortic
nary artery disease and hypertension are common in these stenosis and medical therapy is appropriate. Some patients
patients. Patients may present with heart failure symptoms, with true aortic stenosis do not have contractile reserve and no
usually associated with preserved ejection fraction. Some change in LV function or aortic valve gradients is seen with
patients present with decompensated heart failure associated dobutamine stress. These patients have a worse prognosis. A
with reduced cardiac output. The chronic increase in LV sys­ lack of contractile reserve is an indicator of higher surgical
tolic pressure results in a compensatory increase in myocardial mortality but is not in itself a contraindication to surgery.
cell mass; interstitial fibrosis may follow, leading first to dias­ Paradoxic low-gradient aortic stenosis is a more recently
tolic dysfunction and then to systolic dysfunction. Increased described entity in which the measured aortic valve area is
LV end-diastolic pressure results in pulmonary congestion. reduced in a setting of preserved ejection fraction (>50%) but
These and other adaptive and pathologic processes contribute reduced stroke volume (<35 mL/m2) . Typically encountered i n
to the three cardinal symptoms of aortic stenosis: angina, t h e elderly, i t i s associated with factors related t o reduced
dyspnea, and syncope. Identification of the symptomatic stroke volume, such as small LV size and marked LV hypertro­
patient is crucial; in the absence of symptoms, patients with phy. This may cause the severity of a011ic stenosis to be under­
aortic stenosis- even severe aortic stenosis- have a low risk of estimated by echocard iography owing to the low gradient.
mortality, with a risk of sudden death estimated to be less than Identifying these patients-who are usually symptomatic-is
2
1 %. The rate of progression is approximately 0.1 cm annually. important, as these are patients who may benefit from aortic
Although variable, symptoms of heart failure, angina, or syn­ valve replacement.
2
cope generally begin once the valve area is below 1 . 0 cm . Once
symptoms develop, prognosis is poor without valve replace­ Management
ment, with an average survival of less than 10% over the I n asymptomatic patients with aortic stenosis, identifying
next 2 to 3 years. those who are in higher risk subgroups is important, as these
TIE is essential in the initial diagnosis and follow-up of patients may benefit from earlier intervention. Exercise tread­
aortic stenosis, permitting quantification of valve area and gra­ mill testing is reasonable in asymptomatic patients to identify
dients (see Table 35) , chamber size, wail thickness, ventricular those who actually do have symptoms with exercise, have
function, and coexistent valve disease. Echocardiographic find­ ST-segment changes on ECG, or have an abnormal blood pres­
ings for severe aortic stenosis include a heavily calcified aortic sure response (lack of i ncrease in systolic blood pressure by at
valve with restrictive leaflets, possibly a bicuspid aortic valve, least 20 mm Hg above baseline) , as these patients may benefit
and associated aortic regurgitation. TEE can provide additional from earlier surgery. Patients must be closely monitored dur­
information regarding leaflet anatomy (bicuspid, unicuspid) ing exercise testing. Additional factors that may predict more
when TIE images are inadequate as well as information about rapid progression and thus indicate closer follow-up include
concomitant aortic root abnormalities. If echocardiographic elevated B-type natriuretic peptide level, LV ejection fraction
data conflict with clinical data or echocardiographic images are below 50%, pulmonary hypertension, and moderate to severe
suboptimal, CMR imaging may be useful. It can assess leaflet valve calcification. Appropriate follow-up of asymptomatic
anatomy, measure gradients, and, in patients with a bicuspid patients is based on lesion severity (see Table 35).
aortic valve, assess aorta and aortic root anatomy. Cardiac CT Aortic valve replacement i s indicated for asymptomatic CJ
can be used to measure valve area when echocardiographic patients with severe aortic stenosis and LV systolic dysfunction
images are inadequate and to quantify calcification, a marker ( LV ejection fraction <50°1..) as well as for those pat i ents with
for risk of disease progression. severe aortic stenosis who are undergoing coro n a ry artery
Some patients with calcific aortic stenosis have severe bypass grafti ng or surgery on t he aorta or other heart valves.
aortic stenosis based on valve area but a gradient that is less Surgical aortic valve replacement is the t rea tment of
than 30 mm Hg. Whether symptoms in this "low-flow/low­ choice for most patients with symptomatic severe aortic steno­
gradient aortic stenosis" are caused primarily by aortic valve sis. It is associated with low mortality for pat ients younger
disease with resultant LV dysfunction or the effective valve is t ha n 70 years ( I %-3%) a nd for selected patients who are older
reduced owing to poor leaflet excursion ("pseudosevere aortic than 80 years. Balloon aortic valvuloplasty ( BAV) has a limited
stenosis") can be best determined with dobutamine stress role in t he t rea t ment of adult ao rt i c stenosis. Whereas BAV
echocardiography. Key to this evaluation is the understanding does result in reduction in gradient and increase in valve area.
of contractile reserve. Contractile reserve is defined as an t hese results typically last only for a few months. Recen tly. BAV
i ncrease in transaortic stroke volume of greater than 20% with in the adult has been used successfully as a bridge to definitive
dobutamine infusion. In patients with severe aortic stenosis t reatment (surgical or transcatheter aortic valve replacement
and resultant LV dysfunction but with contractile reserve, the [TAVR]) or to di fferentiate symptoms in high- risk patients
mean gradient will increase with dobutamine stress and the with comorbid conditions such as COPD.
aortic valve area will remain below 1.0 cm2; these patients will Medical therapy has limited benefit in the treatment of
benefit from aortic valve replacement. Those with an increase aortic stenosis. Statins do not delay the progression of aort ic
in aortic valve area to greater than 1.0 cm2 or with a maximum stenosis bu t should be prescribed as indicated to patients with

78
Va lvu l a r Heart D isease

Cl heart failure, ACE i n hibitors. diuretics, angiotensi n receptor


hyperlipidem ia or a t herosclerotic risk factors. I n patients wi t h Symptoms from acute aortic regurgitation result from the
sudden large volume i n a previously norma l LV. resul t i ng i n a
CONT.
blockers, and d igoxi n may be used as appropriate but have not rapid increase in LV end-diastolic pressure and compensatory
been shown to be helpfu l in changi ng the natural h istory of increase in heart rate and contracti lity to maintain cardiac
aortic stenosis. output. When this compensation becomes i nadequate, pul­
It is esti mated that fewer t han h a l f of pat ients with monary edema, myocardial ischemia (due to decreased dias­
symptomatic, severe aortic stenosis undergo surgical aort ic tolic perfusion) . and cardiogenic shock ensue. Conversely,
valve replacement because of comorbid medical condi tions chronic aortic regurgitation perm i ts a compensatory i ncrease
t hat increase surgical risk. This population has become the in LV size wi t h increased LV diastolic volume and LV hypertro­
target for TAVR. The 3 -year fol low-u p resu lts of the PA RTN E R phy (eccentric and concentric) clue to i ncreased volume and
trial i ndicate that TAVR has a survival benefit similar to that a fterload . These compensatory mechan isms help mainta i n
of surgical replacement for h igh -risk patients ( PA RT N E R A stroke volume and cardiac output, but eventually, the i ncrease
cohot1) . TAVR is superior to medical therapy in patients in volume leads to cardiac dysfunction and heart failure symp­
thought not to be surgical candidates ( PART N E R B cohort ) . as toms due to i ncreased LV fil l i ng pressure as well as angi na clue
1 -year morta li ty was 30 .7% i n the TAVR arm versus 50 .7% i n to decreased coronary perfusion pressure and marked
the medica l therapy arm . Compl ications i nclude stroke and LV hypertrophy.
paravalvular aortic regurgitation. Currently, TAV R is i nd i­ Q ua n t i fying t he severity of aortic regurgitation is most
cated for patients who have been determ ined by a cardiac o ften clone w i t h TTE. Markers o f severe aortic regurgitation
surgeon to be i noperable or at high risk for dea t h or major i nclude wide vena contracta (or d i ameter of the regurgitant
morbidity with open aort i c valve replacement and in whom jet or i fice) . rapid equil ibration o f aortic a n d LV pressures
existing comorbidit ies would not preclude the expected ben­ during d iastole. early m i t ra l valve closure. a n d d iastol i c
e fi t from correct ion of the aortic stenosis. For high-risk m i t ra l regurgitation . TEE is usefu l i n ident i fy i ng leaflet a nat­
patients who are surgical ca ndidates. risk should be deter­ omy. in fu rther assessment of severity of aorti c regurgitation
m i ned by a team of cardiologists and cardiac surgeons. with a n d aort ic root pathology. and i n evaluation o f subvalvular
the use o f objective measurements, such as t he Society of ( for example. subaortic membrane) or supravalvular steno­
Thoracic Surgeons adult cardiac risk score (STS score) ( http: // sis. and can aid i n i n t raoperative repai r and rep lacement.
riskcalc.sts.org/STSWebRiskCalc273 /cle. aspx) . Pat ients w i t h C M R i magi ng. i n c l u d i ng magnetic resonance angiography
a n STS r i s k score of greater than or equal t o S 'Yo may b e can­ ( M RA ) . can q u a n t i fy severity of' aortic regurgitation a nd
d idates for TAVR. TAVR is not approved i n patients with a assess aortic root a n d ascending aorta d i la ti on . Cardiac CT
bicuspid aortic valve, sign i ficant aort ic regurgi tation . or may provide i n formation such as coronary a nomalies and
m i t ra l valve d isease. Cl leaflet anatomy.

KEY POI NTS


Management
• Once symptoms of aortic stenosis develop, prognosis is Urgent or emergent surgical valve replacement is indicated for
poor without intervention (valve replacement) , with an acute severe aortic regurgi tation. For chronic severe ao11ic
average survival of only 2 to 3 years and a high risk of regurgitation, valve replacement is indicated for symptomatic
sudden death. patients regardless of LV systolic function (see Table 34) . Valve
• Transcatheter aortic valve replacement is a treatment replacement also is indicated lor asymptomatic patients with
option for patients with aortic stenosis who are inoper­ chronic severe aortic regurgitation and LV systo l ic dysfu nction
able or at high risk for death or major morbidity with and for patients with chronic severe aortic regurgitation
open aortic valve replacement. u ndergoing coronary a 11ery bypass graft surgery or surgery on
the aorta or other heart valves. Combined aortic root replace­
ment with aortic valve replacement is used when t here is a n
associated aortic root aneurysm. Fol low-up of asymptomatic
CJ Aortic Reg u rg itation patients is based on severity of regurgitation and other factors
Clinical Presentation and Eval uation (see Table 35) .
Acute severe aorti c regurgitation usual ly i s caused by i n fec­ Vasod i lators a nd i notropic agents may benefit patients
t ive endocard itis or aortic d issect i o n (typica l ly i n a setting o f with acute severe ao11ic regurgitation and concomitant heart
hypertension o r M a r fa n syndrome) . Chro n ic severe aortic failure as short term therapy before proceeding with surgery.
regurgitation is most com monly associated with a d i l a ted ACE i n h ib itors or a ngiotensin receptor blockers may be used
ascending aorta from hypertension or primary aortic dis­ i n patients with chro n ic severe aortic regurgitation and heart
ease, calc i fic aortic sclerosis. bicuspid aortic valve, or rheu­ fai l u re and in patients with hypertension; however. t hese
m a t i c disease. Calcifi c aortic sclerosis can progress to agents, as wel l as di hydropyri d i ne calciu m channel blockers.
significan t stenosis a n d is associated w i t h a m i l d degree of have not been shown to delay need for surgery in asympto­
aortic regu rgi tat io n. matic patients without hypertension . Cl

79
Va lvu l a r Heart D isease

KEY POI NTS degree and rate of aortic dilation and by family history. Annual
evaluation should occur if the aortic diameter is greater than
• Urgent o r emergent surgical valve replacement is indi­
4.5 cm. Multidetector CT or CMR imaging is appropriate for
cated for patients with acute severe aortic regurgitation.
further assessment of aortic pathology when TIE or TEE is not
• Valve replacement is indicated for symptomatic patients conclusive and for long-term follow up of aortic dimensions
with chronic severe aortic regurgitation regardless of when TEE images are inadequate. A bicuspid aortic valve may
left ventricular systolic function. be present in patients with other thoracic aneurysm syn­
dromes as well, such as Loeys-Dietz syndrome (hypertelorism,
split uvula or cleft palate, and aortic aneurysms) .
B icuspid Aortic Va lve Echocardiography of first-degree relatives of patients with a
Bicuspid aortic valve is the most common congenital heart bicuspid aortic valve is indicated to screen for bicuspid aortic
abnormality, affecting 0.5% to 2% of the total population. valve or aortic aneurysms.
Although a bicuspid aortic valve may be suspected in patients As with calcific aortic stenosis, medical therapies for
with classic auscultatory findings (systolic ejection click at the bicuspid aortic valve stenosis are limited. Aortic valve replace­
left lower sternal border: murmur of aortic stenosis or aortic ment is the only therapeutic option for adult patients with a
regurgitation in a young patient) , it is most often diagnosed by stenotic bicuspid aortic valve, and recommendations regard­
TIE performed for some other indication. A bicuspid aortic ing when to intervene are the same as for tricuspid aortic
valve may be diagnosed initially in a patient presenting with a valves (see Table 34) .
complication, such as bacterial endocarditis or aortic dissec­ For patients with a regurgitant bicuspid aortic valve, valve
tion. Features that may raise suspicion include eccentric aortic replacement is the treatment of choice when regurgitation is
regurgitation, enlarged sinuses ofValsalva, a dilated ascending clinically significant; however, repair may be performed in
aorta, or an elliptical valve opening. Calcification may obscure selected patients by experienced surgeons.
valve leaflets and make the diagnosis difficult. TEE may pro­ Surgery to repair the aortic root or replace t he ascending CJ
vide confirmatory information as to leaflet anatomy as well as aorta is i ndicated i n patients wi th a bi cu sp id aortic valve when
the presence of associated conditions, including aortic aneu­ the aort ic root diameter is greater than 5 .5 cm . Surgery is rea­
rysms and aortic coarctation, and is appropriate when the sonable if' t he dia meter of the ascending aorta or aortic root is
diagnosis is uncertain. greater t ha n 5 . 0 cm and a r i sk factor for d issection is present
The clinical course of patients with a bicuspid aortic valve ( fa m i ly history of aortic d issection or if' the rate of i ncrease i n
varies widely. Although overall survival is similar to that of diameter i s 2:0.5 c m per ye a r) . Ascending aorta re pl ace m en t is
age-matched controls, approximately one in three patients reasonable in patients who are u nd ergo i ng aortic valve surgery
with a bicuspid aortic valve may eventually require valve sur­ because of severe aortic stenos is or aortic regurgitation if t he
gery for either stenosis or regurgitation. Stenosis proceeds at a diameter of the ascen d i ng aorta is greater than 4 . 5 cm. CJ
faster rate when the aortic valve is bicuspid, and valve replace­
KEY P O I N TS
ment may be required in the fifth or sixth decade of life. Some
degree of regurgitation is common with a bicuspid aortic valve • I n general, older, asymptomatic patients with a bicuspid HVC
but is less likely than stenosis to lead to valve replacement. The aortic valve and severe aortic valve stenosis or regurgita-
presence of a bicuspid aortic valve carries an increased risk of tion require yearly echocardiography; those with mild
infective endocarditis, and good dental care is important in aortic stenosis or regurgitation require echocardiogra-
these patients. Current guidelines no longer recommend anti­ phy every 3 to 5 years.
biotic prophylaxis for this patient population (see Infective • Echocardiography of first-degree relatives of patients
Endocarditis, later) . The timing of follow-up TTE depends on with a bicuspid aortic valve is indicated to screen for
many factors, including patient age and severity of the valve bicuspid aortic valve or aortic aneurysms.
lesion. In general, older, asymptomatic patients with a bicus­
• Patients with a bicuspid aortic valve are predisposed to
pid aortic valve and severe aortic valve stenosis or regurgitation
aortopathy, and aortic surgery is indicated when the
require yearly echocardiography; those with mild stenosis or
aortic root diameter is greater than 5 . 5 cm.
regurgitation should have echocardiography every 3 to 5 years.
A bicuspid aortic valve may occur with other cardiovas­
cular and systemic abnormalities, such as aortic coarctation,
aneurysm of the sinuses of Valsalva. and patent ductus arterio­ M itra l Ste nosis
sus. Patients with a bicuspid aortic valve are predisposed to Clinical Presentation and Evaluation
aortic aneurysm and dissection owing to aortic connective The primary cause of mitral stenosis is rheumatic carditis.
tissue abnormalities. For this reason, the ascending aortic Improved hygiene and the routine use of antibiotics for group
diameter should be reassessed by echocardiography if the A streptococcal pharyngitis have greatly reduced the incidence
aortic root or ascending aorta dimension is greater than or of rheumatic heart disease in the United States and developed
equal to 4.0 cm, with the evaluation interval determined by countries. Rarely, functional mitral stenosis occurs as a result

80
Va lvular Heart D isease

of outflow obstruction from tumor (such as myxoma) or left valvotomy is unava i la b le , unsuccessfu l , or contra indicated or
atrial thrombus. Other causes of mitral stenosis include con­ when the valve morphology is un favorable.
genital disease (such as parachute mitral valve, wherein the Medical therapy for mitral stenos is co n si s t s of d iuretics or
mitral chordae insert into one instead of two papillary mus­ long-act i ng ni trates, which may help i mprove symptoms such
cles) and mitral annular calcification. Symptoms can be indo­ as dyspnea. I n add i t ion. �-blockers or n o ndi hyd ropyr id i ne
lent, with patients remaining asymptomatic for years and then calciu m channel blockers can lower heart rate and improve LV
presenting with a gradual decrease in activity. Other symp­ diastoli c filling t i me. c:J
toms include dyspnea, orthopnea, fatigue, and, less com­
KEY POINT
monly, hemoptysis or systemic embolization. Symptoms typi­
cally are not present until the mitral valve area is less than • Mitra! valvotomy or surgery is indicated for sympto­

1 . 5 cm 2 , although tachycardia may precipitate symptoms at matic patients (New York Heart Association functional
larger valve areas. The decrease in mitral valve area results in a class II-IV) with severe mitral stenosis and favorable
diastolic pressure gradient between the left atrium and ventri­ valve morphology.

cle. This increase in left atrial pressure causes left atrial


enlargement, with development of atrial fibrillation and a
M itra l Reg u rgitation
significantly increased risk of left atrial/left atrial appendage
thrombus, as well as exertional dyspnea and eventual pulmo­ Clinical Presentation and Eva luation
nary edema owing to increased pulmonary venous pressure. Abnormalities in any of the structures of the mitral valve appa­
In addition, chronic pulmonary venous pressure elevation ratus, including anterior and posterior mitral leaflets, the
may result in pulmonary hypertension. The risk of thrombo­ annulus, the papillary muscles, and the chordae tendineae, can
embolism increases with age and the presence of atrial fibril­ result in mitral regurgitation. Organic, or primary, mitral
lation. Warfarin remains the anticoagulant of choice for these regurgitation refers to processes involving the leaflets, such as
patients with concomitant atrial fibrillation as they were mitral valve prolapse, myxomatous degeneration (the abnormal
excluded from trials of novel oral anticoagulants. accumulation of proteoglycans) , collagen vascular disease, and
TIE is used to assess disease severity of mitral stenosis by infective endocarditis. Processes that affect the support struc­
measuring valve area and transvalvular gradient (see Table 35) . tures, such as coronary artery disease and LV remodeling in the
Typical echocardiographic findings of rheumatic mitral steno­ setting of LV dysfunction, result in functional, or secondary,
sis include a domed or "hockey stick" appearance to the mitral mitral regurgitation.

Cl
valve, with thickened, calcified subvalvular apparatus and Acute severe m i l ral regurgi t a t i o n is associated w i t h
marked left atrial enlargement. Gradients are heart-rate papi l la ry muscle ru p t ur e fo llowing acute myocard i a l
dependent and can vary greatly in patients who are tachy­ i n fa rc t i o n . fla i l m i t ral valve (d issoc i a t i o n o f t h e valve l e a f­
cardic or have atrial fibrillation. Echocardiography is essential l e t fro m t h e c horclae) . a n d i n fe c t ive endocard i ti s w i t h
for assessment of valve morphology, including factors such as leaflet p e r fo ra t ion . The sudden l a rge vol u m e i n the l e ft
valve calcification and thickening, leaflet motion, and iJ l r i u m and ven t ricle results i n ra pid i n c reases i n LV e n d ­
subchordal thickening. These factors, taken together, help d i as t o l i c pressure a n d left a t rial pressure, which leads t o
predict the likelihood of successful percutaneous mitral bal­ e l evated pu l m o n a ry a rtery p ressure a n d p u l m o n a ry
loon valvuloplasty. TEE provides better visualization for edema. The d i m i n i s h ed LV s t ro ke vo l u m e leads t o hypo­
assessment of mitral regurgitation severity as well as the pres­ tension and shock. c:J
ence of left atrial appendage thrombus. Multidetector CT may Patients with chronic mild to moderate mitral regurgita­
provide additional information by valve planimetry as well as tion may remain asymptomatic for many years. Progression
visualization of the coronary anatomy. This may be helpful for is variable and caused by either progression of lesions or
patients whose valve is not well imaged on TIE and ar,e unable increasing mitral annulus size. The increase in volume and
to tolerate TEE. subsequent increase in left atrial pressure lead to compensa­
tory dilation of the left atrium and LV. Left atrial dilation

Cl Management
Percutaneous m i t ral balloon valvotomy is indicated f"or symp­
predisposes to atrial fibrillation. A chronic increase in
preload and resultant eccentric hypertrophy, i n the setting of
tomatic patients (New York Heart Association [NYJ -IA] func­ increased stroke volume (normal forward stroke volume and
tional class I I , I l l or I V) with severe mitral stenosis (see Table 34)
, regurgitant volume) lead to contractile dysfunction and
a nd favorable valve morphology. TEE plays an important role increased end-diastolic volume, which lead to pulmonary
in assessment of patients being cons id e red for percutaneous congestion and pulmonary hypertension. Appropriate fol­
bal loon valvotomy to evaluate for potential contraindications. low-up of asymptomatic patients with mitral regurgitation is
i ncluding left atrial appendage clot or signi ficant (moderate to outlined in Table 35.
severe) m itral regurgitat ion . M i t ra l valve su rgery (repai r if pos­ TTE serves as the main imaging modality in the evalua­
sible) is indica ted in patients with symptomatic (NYHA func­ tion and management of mitral regurgitation. TIE allows diag­
tional class I l l-IV) severe m itral stenosis when balloon nosis of the mechanism of mitral regurgitation, qualitative and

81
Va lvular H e a rt Disease

TABLE 37. Empiric Therapy for Infective Endocarditis which changed significantly in recent years, may contradict
long-standing expectations of patients and practice patterns of
Condition Therapy
health care providers. However, infective endocarditis is more
Com m u n ity-acq u i red native Consider vancomycin + likely to result from frequent exposure to random bacteremia
valve I E genta m i ci n
associated with daily activities than from bacteremia caused
Nosocomia l-associated I E Consider vancomycin + by dental, gastrointestinal tract, or genitourinary procedures;
genta micin + rifampin or
vancomycin + gentamicin +
additionally, the risk of antibiotic-associated adverse
a carbapenem or cefepime effects may exceed the benefit (if any) from prophylactic
Prosthetic valve I E Consider vancomycin + antibiotic therapy.
gentamicin + rifampin Infective endocarditis prophylaxis is currently recom­
I E = infective endocarditis.
mended for patients with cardiac conditions with the highest
risk for adverse outcomes from infective endocarditis (rather
than those with an increased lifetime risk for infective endo­

Cl within 60 days of a hospi tal admission that vvas associated wi th carditis) . Prophylaxis is recommended for patients with (1) a
prosthetic cardiac valve; (2) a previous episode of infective
_ _
a risk for bacterernia or mfecllve _
endocard1t1s. For nosocorn1al
CONT. endocarditis; (3) congenital heart disease, including unre­
infections, coverage for multidrug-resistant bacteria, particu­ paired cyanotic congenital heart disease, a completely repaired
larly coagulase-negative staphylococci, is recom mended. congenital heart defect with prosthetic material or device dur­
Surgery for native valve endocarditis usually entails resec­ ing the first 6 months after the procedure, and repaired con­
tion of the vegetation and valve repair or replacement if appro­ genital heart disease with residual defects; or (4) valvulopathy
priate. Early surgery (during i nitial hospitalization and before following cardiac transplantation.
completion of a ful l course of antibiotics) is indicated for Unless there is a history of infective endocarditis, prophy­
patients with acute infective endocarditis presenting with laxis is not recommended for patients with native valve dis­
valve stenosis or regurgitation resu lting i n heart fail ure: left­ ease, including rheumatic heart disease, mitral valve prolapse
sided infective endocarditis caused by Stap h y lococcus a u reus, with regurgitation, or a bicuspid aortic valve.
fungal, or other highly resistant organisms; infective endocar­ In patients who meet the criteria, prophylaxis should be
ditis complicated by heart block. annular or aortic abscess, or administered prior to dental procedures that involve manipu­
destructive penetrating lesion; and infective endocarditis with lation of gingival tissue or the periapical region of the teeth or
persistent bacteremia or fever lasting longer t han 5 to 7 days perforation of the oral mucosa (Table 38 ) . Infective endocardi­
after sta rting antibiotic therapy. Surgery is recommended for tis prophylaxis is not recommended prior to nondental
patients with relapsing prosthetic valve endocarditis. l n procedures, such as TEE, genitourinary procedures, or gastro­
patients with infective endocarditis who have documented intestinal procedures. Antimicrobial prophylaxis is recom­
i n fection of pacemaker or defibril lator systems. complete mended for procedures involving incision or biopsy of the
removal of these systems (all leads and generator) is indicated. respiratory tract mucosa, such as bronchial biopsy, tonsillec­
Early surgery is reasonable in patients with infective endocar­ tomy, and adenoidectomy.
ditis who h ave recurrent emboli and persistent vegetations on
antibiotic therapy, and may be considered in patients with KEY P O I NTS
native valve endocarditis who have mobile vegetations greater • In patients with a cardiac murmur suggestive o f organic
than 10 mm in length . The Early Surgery versus Conventional valvular or congenital heart disease or patients with a
Treatment in infective Endocarditis (EASE) trial compared the prosthetic heart valve, infective endocarditis should be
clinical outcomes of early surgery with a conventional treat­ suspected in the presence of fever, anemia, hematuria,
ment strategy in patients with left-sided i n fective endocarditis and physical findings suggestive of embolization.
and a high risk of embolism . In this study. early surgery • Early surgery is indicated for patients with acute infec­
(within 48 hours a fter diagnosis and random ization) in tive endocarditis presenting with valve stenosis or regur­
patients with large vegetations (>10 mm) significantly reduced gitation resulting in heart failure; left-sided infective
embolic events with similar in-hospital and 6 -rnonth mortal­ endocarditis caused by Staphy lococcus aureus, fungal,
i ty rates compared with delayed surgery. CJ or other highly resistant organisms; infective endocardi­
tis complicated by heart block, annular or aortic abscess,
Prophylaxis or destructive penetrating lesion; and infective endocar­
The significant morbidity and mortality associated with infec­ ditis with persistent bacteremia or fever lasting longer
tive endocarditis underscore the importance of appropriate than 5 to 7 days after starting antibiotic therapy.
administration of antibiotics before procedures expected to
• Surgery is recommended for patients with prosthetic
produce bacteremia. The American College of Cardiology/
valve endocarditis and relapsing infection.
American Heart Association (ACC/AHA) recommendations and
the European Society of Cardiology (ESC) recommendations, (Continued)

84
Va lvu l a r Heart D isease

TABLE 38. Prophylactic Infective Endocarditis Regiinens for Adults Before a Dental Procedure
Situation Agent" Dosage
Oral Amoxici l l i n 2g
Una ble to take oral medication Ampici l l i n 2 g I M or IV
or

Cefazo l i n or ceftriaxone 1 g I M o r IV
Allerg i c to penicillin or a m picillin - oral Cephalexin 2g
or

C l i nd a mycin 600 mg
or

Azithromycin or cla rithromycin SOO m g


Alle rgic to penic i l l i n or ampicillin and u na b l e Cefazolin or ceftriaxone 1 g I M or IV
t o take oral medication
or

Clindamyci n 600 mg IM or IV

IM = intramuscular; I V = intravenous.

aRegimen consists of a single dose 30 to 60 minutes before the dental procedure, or, if inadvertently not administered, drug may be given up to 2 hours after the procedure.

Adapted from Wilson W, Taubert KA, Gewitz M, et al; American Heart Association. Prevention of infective endocarditis: guidelines from the American Heart Association: a guide­
line from the American Heart Association Rheumatic Fever. Endocarditis and Kawasaki Disease Committee, Council on Cardiovascular Disease in the Young, and the Council on
Clinical Cardiology, Council on Cardiovascular Surgery and Anesthesia, and the Quality of Care and Outcomes Research Interdisciplinary Working Group. J Am Dent Assoc. 2008
Jan; 1 39 Suppl:3S-24S. Review. Erratum in: J Am Dent Assoc. 2008 Mar;1 39(3):253. IPMID: 1 8 1 673941. Copyright 2008 American Dental Association.

K E Y P 0 I NT S (continued) valve selection (Table 39) . In general, patient factors that favor a
mechanical prosthesis include younger age, an absence of con­
HVC • Infective endocarditis prophylaxis should be limited to
t raindications to long-term anticoagulation. a risk of acceler­
those with a prosthetic cardiac valve; a history of infec­
ated bioprosthetic structural valve deterioration (for example,
tive endocarditis; unrepaired cyanotic congenital heart
with pregnancy or kidney failure) . and a reasonable l i fe expec­
disease or repaired congenital heart defect with prosthe­
tancy in patients for whom future redo valve surgery wou ld be
sis or shunt (::;6 months post-procedure) or residual
h igh risk. Conversely, factors that would favor a bioprosthelic
defect; or valvulopathy following cardiac transplantation.
valve i nclude older age; patients in whom adequate long-term
anticoagu lation is unlikely (for example, owing to adherence
problems or ant icoagu lation agents not being readily ava ilable)
Prosthetic Va lves or contraindicated because of' high bleeding risk (prior major

Cl ment must accoLU1t for various considerations, including the


The choice of prosthesis for patients requiring valve replace­ bleed. li festyle or occupation issues) ; patients requiri ng reop­
eration for mechanical valve thrombosis despite good long-term
patient's age. a ny comorbid factors, and the patient's preference anticoagu lant control; and young women contemplating preg­
and ability to adhere to medical therapy post-i mplantation. The nancy (although a mechanical prosthesis requiring anticoagula­
ACC/AHA and ESC have published recommendations regarding tion is not a contraindication to pregnancy) .

TABLE 39. Summary of Recommendations for Pro�hetic Valve Choice


Type of Valve Recommendation

Bio p rosthesis Recommended: patients of any age for whom anticoag u lant therapy is contra i nd icated,
cannot be ma naged appro p riately, or is not desired

Reasonable: patients > 70 y of age

Mechanical prosthesis Reasonable: AVR or MVR i n patients <60 y of age who do not have a contra i n d ication to
anticoagulation
Either bio- or mechanical prosthesis Reasonable: patients between 60 y and 70 y of age

AVR = aortic valve replacement; MVR = mitral valve replacement_

NOTE: The guideline recommends that the choice of valve intervention and prosthetic valve type should be a shared decision-making process.

Recommendations from Nishimura RA, Otto CM, Bonow RO, et al; American College of Cardiology/American Heart Association Task Force on Practice Guidelines. 201 4 AHA/ACC
guideline for the management of patients with valvular heart disease: executive summary: a report of the American College of Cardiology/American Heart Association Task Force
on Practice Guidelines. J Am Coll Cardiel. 201 4 Jun 1 0;63(22):2438-88. Erratum in: J Am Coll Cardiel. 201 4 Jun 1 0;63(22):2489. IPMID: 24603 1 921

85
Ad u lt Cong e n ita l H e a rt D isease

Cl mended for all patients with a mechanical prosthesis and those


Lifelong oral anticoagulation with warfarin is recom­ odic informed follow-up by a team consisting ideally of ilie
internist and an adult or pediatric cardiologist trained in adult
CONT.
with bioprostheses with other indications for anticoagu lation. congenital heart disease. Specialized care is particularly impor­
The target I N R is based upon prosthesis location, with a target of tant for patients wiili complex and cyanotic congenital cardiac
2.5 for patients with a mechanical aortic prosU1etic valve and 3.0 disease. Congenital heart conditions may be fust identified dm­
for patients with a mechanical mHral prosthetic valve and those ing pregnancy or symptom status may be worsened by the phys­
patients with a mechanical aortic prosthetic valve and risk iologic changes of pregnancy. Decisions regarding the safety of
factors (atrial fibrillation, LY dysfunction, previous thromboem­ pregnancy in patients with congenital caTdiac disease and man­
bolism, hypercoagulable condition, or older-generation agement during pregnancy should be made on an individual
mechanical ao11ic valve replacement) . The addition of aspirin, 75 basis in consultation with a congenital heart disease specialist.
to 1 00 mg/d, is recommended as well. Ongoing studies aTe
evaluating various anticoagulation regimens for newer and
Patent Fora m en Ova le
potentially less thrombogenic valves.
The need for anticoagulation for those with bioprosthetic The foramen ovale allows transfer of oxygenated placental
valves is less clear. Oral anticoagulation should be considered blood to the fetal circulation and usually closes within the first
for the first 3 months after implantation of a mitral or tricuspid few weeks of life. In 25% to 30% of the population, however,
bioprosthesis and for the first 3 months a fter bioprosthetic the foramen ovale remains patent ( Figure 27) . Patent foramen
mitral valve repair. Low-dose aspirin or anticoagulation should ovale (PFO) is usually asymptomatic and identified inciden­
be considered for the first 3 months after implantation of an tally. No treatment or follow-up is needed for the PFO found
aortic bioprosthesis. Long-term aspirin use is reasonable for incidentally in an asymptomatic patient.
all patients with bioprost hetic valves. c:J PFO is diagnosed by visualizing the interatrial septum by
Patients with prosthetic valves require lifelong cardiology echocardiography and demonstrating shunting of blood across
follow-up, including annual clinical examination. For baseline ilie defect by color flow Doppler imaging or by using agitated
assessment, all patients undergoing valve replacement surgery saline injected intravenously and subsequent transfer ilirough ilie
should have TIE performed 2 to 3 months a fter implantation PFO from right to left atrium. If transthoracic echocardiography
as a baseline study. Patients with bioprosthetic valves may be (TIE) is nondiagnostic, transesophageal echocardiography (TEE)
considered for annual echocardiograms after the first 10 years usually provides improved visualization of the atrial septum.
in the absence of a change in clinical status. Routine echocar­ Antiplatelet therapy is recommended as first-line therapy
diography is not recommended for patients with mechanical for patients with PFO and cryptogenic stroke. Results of rand­
valves or those with recently implanted biologic valves in the omized controlled trials of device closure for crytogenic stroke
absence of any change in symptoms or examination findings. and PFO have not demonstrated benefit compared with medi­
cal therapy for secondary stroke prevention. Patients with
KEY P O INTS cryptogenic stroke and PFO may be treated with antiplatelet or
• Lifelong oral anticoagulation with warfarin i s recom­ anticoagulant therapy and should be encouraged to participate
mended for all patients with a mechanical prosthetic in one of the ongoing trials comparing closure with medical
heart valve. therapy. There is no indication for PFO closure or for anti plate­
• Oral anticoagulation should be considered for the frrst let therapy in asymptomatic patients.
3 months a fter implantation for patients with a Observational studies have suggested an association
bioprosthetic m itral or tricuspid valve and for the first between PFO and migraine. However, randomized studies of
3 months a fter bioprosthetic mitral valve repair. PFO closure to prevent migraine recurrence have not shown
beneficial effect, and PFO closure should not be performed for
• Routine echocardiography is not recommended for
the prophylaxis of migraine.
patients with mechanical valves or those with recently
implanted biologic valves in the absence of any change
i n symptoms or examination findings.

Ad u lt Congenita l
H ea rt Disease
I ntroduction
Advances i n caTe of patients with congenital heart disease over
the past six decades have resulted in more adults U1an children F I G U R E 2 7 . Patent fora men ovale. The arrows demonstrate the mecha nism of
rig ht-to-l eft s h u nt through the patent foramen ova le. LA = left atri um; LV = left ven·
living in North America with these conditions. CardiovasculaT
tricle; RA = right atri u m ; RV = right ventricle.
residua are common in patients wiili previous intervention for
Redrawn from original supplied courtesy of Dr. William 0. Edwards, Department of Laboratory Medicine and
congenital cardiac lesions, underscoring ilie importance of peri- Pathology, Mayo Clinic, Rochester, MN.

86
A d u lt Congen ita l H e a rt D i sease

Rarely, a PFO may be associated with the platypnea-orthode­


oxia syndrome. Platypnea-orthodeoxia syndrome is an acquired
disorder characterized by cyanosis and dyspnea in the upright
position. These symptoms are related to right-to-left shunting
across a PFO or atrial septal defect and are caused by a transient
increase in right atrial pressure, which occurs as a complication of
right ventricular myocardial infarction, pulmonary embolism,
tricuspid regurgitation, or acute right heart failure. Device PFO
closure in these patients may decrease hypoxemia and cyanosis.
Atrial septa) aneurysm is redundant and mobile atrial
septa! tissue. No treatment is needed when atrial septa! aneu­
rysm is discovered incidentally. Atrial septal aneurysm in con­
junction with a PFO reportedly increases the risk of stroke
compared with a PFO alone. Antiplatelet therapy is recom­
mended for patients with cryptogenic stroke and an isolated
atrial septa) aneurysm. In patients with atrial septa! aneurysm
and recurrent stroke on antiplatelet therapy, anticoagulant
therapy is recommended if no other cause of stroke is identi­ F I G U R E 2 8. Positions of various atrial septa I defects viewed from the right side
of the heart. (1) ostium secu ndum; (2) ostium prim u m; (3) sinus venosus; (4) infe·
fied. Rarely, surgical excision of an atrial septa! aneurysm and
rior vena cava. PT = pul monary trunk; RV = right ventricle.
defect closure is considered if antiplatelet or warfarin therapy
Redrawn from original supplied courtesy of Or. William D. Edwards, Department of Laboratory Medicine and
fails to prevent stroke recurrence. Pathology, Mayo Clinic, Rochester, MN.

KEY P O I NTS
• Antiplatelet therapy i s recommended a s first-line ther­ abnormalities and congenital heart defects, most commonly
apy for patients with patent foramen ovale and crypto­ an ASD. Familial ostium secundum ASDs have also been
genic stroke. described and may be autosomal dominant or are linked to
HVC • There is no indication for patent foramen ovale closure chromosome 5. Down syndrome is commonly associated with
or for antiplatelet therapy in asymptomatic patients. congenital heart disease, most commonly a form of atrioven­
tricular septa! defect, including ostium primum ASD.
HVC • Randomized trials do not support patent foramen ovale
closure to reduce risk of recurrent stroke or migraine.
Clinical Presentation
ASDs may initially be identified in adulthood; the age of presen­
tation depends on the shunt size and associated defects.
Atrial Septa l Defect Common presenting teatures include fatigue; exertional dysp­
Pathophysiology and Genetics nea; atrial fibrillation; paradoxical embolism; and abnormalities
An atrial septal defect (ASD) is a communication between the on the physical examination, including a pulmonary outflow
atria. Left-to-right shunt occurs and, over time, causes right­ murmur or fixed splitting of S 2 or, less commonly, findings that
sided cardiac chamber dilatation. Defects are classified accord­ are consistent with a genetic syndrome involving an ASD.
ing to location (Figure 28) . Ostium secundum ASDs are the most Occasionally, an ASD is identified as the cause for right heart
common type (75% of cases) . These are located in the middle enlargement found incidentally on an echocardiogram. Rarely,
portion of the atrial septum and are usually isolated anomalies. patients with isolated ASDs present with pulmonary arterial
Ostiurn primum defects (15%-20% of cases) are located in the hypertension (PAH); this usually occurs in young women, sug­
lowest portion of the atrial septum; they are commonly associ­ gesting the coexistence of idiopathic PAH .
ated with a cleft mitral valve, ventricular septal defect, and sub­ Clinical findings in ASD include jugular venous disten­
aortic stenosis, a collection of abnormalities tenned endocardial tion, a parasternal impulse, and a systolic flow murmur at the
cushion detect. Sinus venosus defects (5%-10% of cases) are second left intercostal space. Large shunts may cause a dias­
usually located near the superior vena cava or, rarely, near the tolic flow rumble across the tricuspid valve owing to a large
inferior vena cava. More than 90% of patients with sinus venosus left-to-right shunt. Fixed splitting of the S 2 is the characteristic
ASD have associated anomalies of pulmonary venous connec­ auscultatory finding in patients with an ASD.
tion. A coronary sinus ASD (<l % of cases) is a communication
between the coronary sinus and the left atrium. These detects Diagnostic Eval uation
are commonly associated with complex congenital heart lesions The electrocardiogram (ECG) and chest radiograph findings in
or a persistent left-sided superior vena cava. patients with ASDs are outlined in Table 40. Complete heart
Most ASDs occur sporadically; however, several genetic block may occur in familial ASD.
syndromes associated with ASDs are recognized. The Holt­ TI'E is the diagnostic imaging modality of choice for iden­
Oram syndrome involves bilateral upper extremity bone tification of ostium primum and secundum ASDs. Additional
87
Adult Con g e n ital Heart D i sease

TABLE 40. Imaging Findings and Late Complications in Adult Congenital Heart Disease
Lesion ECG and CXR Findings Late Complications

Patent foramen ova le Normal Paradoxical embolism, platypnea-orthodeoxia


syndrome

Ostium secu ndum ASD ECG : I n complete RBBB, RA enl argement, right Right heart enlargement, atrial fibril lation, PAH
axis deviation (rare)

CXR: Right heart enlargement, prominent Post repair: residual s h u nt (rare)


p u l monary artery, i ncreased p u l monary
vascula rity

Ostium primum ASD ECG : Left axis deviation, 1 st-deg ree Right heart enlargement, atrial fibrillation, m itral
atrioventricular block regurgitation (from mitral valve cleft), PAH (rare)

CXR: Right heart enlargement, prominent Post repair: residual s h u nt (rare), m itral
p u l monary artery, i ncreased pulmonary regurgitation (from mitral valve c left), left
vascularity ventricular outflow tract obstruction

Sinus venosus ASD ECG : Abnormal P axis Right heart enlargement, atria l fi bri l l ation, PAH
(rare)
CXR: Right heart enlargement, prominent
pulmonary a rtery, i ncreased p u l monary Post repair: residual s h u nt (rare), residual
vascularity anomalous pul monary venous co n nection

Small VSD Normal Endocarditis

Large VSD ECG : RV or RV/LV hypertrophy Left heart enlargem ent, PAH , Eisenmenger
syndrome
CXR: LA a n d LV enlargement, increased
pulmonary vascu lar markings; with PAH : Post repair: residual VSD, residual s h u nt (rare)
prominent central pul monary a rteries, reduced
peripheral p u l monary vascular markings

Small PDA Norma l Endocarditis

Large PDA ECG: LA enl argement, LV hypertrophy; with PAH : Endocarditis, heart fa i l u re, PAH, Eisenmenger
RV hypertrophy syndrome

CXR: Cardiomega ly, i ncreased pulmonary Post repair: residual shunt (rare)
vascular markings; calcification of PDA
(occasional); with PAH : prominent central
pulmonary arteries, reduced peripheral
pulmonary vascu l a r markings
Pulmonary valve stenosis ECG : Normal when RV systolic pressure Post repair: Severe p u l monary valve regurgitation
<60 mm Hg; if RV systol i c pressure >60 m m Hg: after pul monary valvotomy or valvuloplasty
RA enlargement, right axis deviation, RV
hypertrophy

CXR: Pulmonary artery d i l atation, cal cification of


pulmonary valve (rare); RA enlargement may be
noted

Aortic coa rctation ECG: LV hypertrophy and ST-T wave Hypertension ( 7 5% of cases), bicuspid aortic
a b normal ities valve (> 50% of cases), increased risk of aortic
aneurysm and intracra n i a l aneurysm
CXR: Dilated ascending aorta, "figure 3 sign"
beneath aortic arch, rib notching from col lateral Post repair: Recoarctation, hypertension, aortic
vessels aneurysm
Repaired tetra logy of Fa I lot ECG: RBBB, i ncreased ORS d u ration (ORS Post repair: Increased atrial and ventricular
d u ration reflects degree of RV d i l atation) arrhyth mia risk, pulmonary valve reg u rgitation or
stenosis; tricuspid regurgitation
CXR: Ca rd iomegaly with pul monary or tricuspid
valve regurgitation; right aortic arch i n 25% of ORS > 1 80 msec i ncreases risk of ventricular
cases tachycardia a n d sudden death
Eisenmenger syndrome ECG : Right axis deviation, RA enl argement, RV Right heart fa i l u re, hemoptysis, stroke
hypertrophy

CXR: RV d i l atati on, prominent pulmonary a rtery,


reduced pulm onary vascu la rity

ASD = atrial septal defect; CXR = chest radiograph; ECG = electrocardiogram; LA = left atrium; LV = left ventricle; PAH = pulmonary arterial hypertension; PDA = patent ductus arte­
riosus; RA = right atrium; RBBB = right bundle branch block; RV = right ventricle; VSD = ventricular septa I defect.

88
Ad ult Con g e n ita l H e a rt D i s ease

findings on TIE include right-sided cardiac chamber enlarge­ Patients with small ASDs do not need a ny limitation
ment, tricuspid regurgitation related to annular dilatation, and of physical activity. I n patients with large left-to-right
increased right ventricular systolic pressure. Agitated saline shunts, exercise is often self-limited owing to decreased
contrast injection may help identify a right-to-left atrial shunt cardiopulmonary function. If pulmonary vascular disease
if Eisenmenger syndrome is suspected (see Adults with is present, patients should b e advised against isometric or
Cyanotic Congenital Heart Disease, later) . Sinus venosus and competitive exercise.
coronary sinus ASDs are less readily diagnosed by TIE in Pregnancy in patients with ASD is generally well tolerated
adults and often require TEE, MRI, or CT imaging. in the absence of PAH. The risk of congenital heart disease
MRI and CT can be used to quantify right ventricular transmission in patients with sporadic ASD is estimated to be
volumes and ejection fraction. These studies are rarely used as less than 10%. Genetic syndromes have variable inheritance; a
the primary imaging modality when ASD is suspected but may family history should be taken. Holt-Oram syndrome is inher­
help quantify right heart enlargement in a patient with ASD. ited in an autosomal dominant fashion.
In addition, a CT or MRI performed for another reason may be
the first imaging study to demonstrate the ASD. MRI, CT, and Follow-up After Atrial Septal Defect Closure
TEE are useful for identifying anomalous pulmonary veins. Clinical follow-up is recommended for all adult patients after
Cardiac catheterization is the only reliable method to surgical or device ASD closure; the frequency of follow-up
calculate the pulmonary-to-systemic blood flow ratio should be individualized. TIE imaging is generally recom­
(Qp:Qs) but is rarely required for uncomplicated ASDs. mended within the first year after closure and then periodi­
Cardiac catheterization may be recommended in the patient cally after that. Pre- and post-closure atrial fibrillation occurs
with an ASD and PAH to aid in determining whether ASD more frequently the older the patient is at the time of ASD
closure is indicated. closure. Rare complications after device closure include device
migration, erosion into the aorta or pericardium, and sudden
Treatment
death. Chest pain or syncope after device closure warrants

Cl the main indications for ASD closure. Closure should also be


Sym pt oms a nd righ t- sided cardiac chamber enlargement are
urgent evaluation for device erosion.

co nsi d ere d for pa t i e n ts w i t h p la typnea-o rL hodeox ia syn ­ KEY POI NTS


drome and patients with i n t racardiac shunt before pacemaker • Fixed splitting o f the S 2 is the characteristic auscultatory
placement because of the i ncreased risk of' systemic t h rombo­ finding in patients with an atrial septa! defect.
embolism. Other co n si de ra t ions i nclude patient age. delecl • A small atrial septa! defect (pulmonary-to-systemic HVC
size and l oca t i o n and associated abnorma li ties such as right
,
blood flow ratio [Qp:Qs] <l.5:1 ) with no associated
heart enl a rgem ent moderate (or less) hypertension. and t ri ­
,
symptoms or right heart enlargement can be followed
c usp i d reg urgit a t i o n . A small ASD (Qp: Qs < l . 5 : 1 . 0) w i t h no clinically.
associated symptoms or right hean enlargement can be fo l ­

• Symptoms and right-sided cardiac chamber enlarge­


lowed clinically.
ment are the main indications for atrial septa! defect
Percutaneous intervention involves closure of the ASD
closure.
using a device delivered via the venous system in t he cathcter­
ization laboratory. This is t he pref erred treatment lor pa tients
with an isolated ostium secundum ASD and righ t-sided car­
diac chamber e n la rgemen t but no other associated card iovas­
Ventricu lar Septa l Defect
c ul a r abno rma l i ty t ha t requ i res operative i n tervention . Pathophysiology
S u rgi c al closure is ind icated for very large ost ium secundum Ventricular septa! defects (VSDs) are common at birth, but the
ASDs or i nsu ffic i e n t se p ta! anatomy for device closure: for all frequency decreases by adulthood because of spontaneous
other types of' A SD s ; and for patients wi t h any type of ASD closure of small defects. VSDs are defined by their location on
when t here i s coex istent cardiovascular d i sease t hat req u i res the ventricular septum (Figure 29) . The most common type of
operat ive intervention, such as coronary artery disease or t ri ­ VSD is perimembranous, comprising 80% of cases; these are
cuspid valve regurgitation. usually isolated abnormalities. Subpulmonary VSDs (also
Pat ienrs with severe PAI-I and an ASD may be considered called outlet or supracristal VSDs) account for approximately
for closure providing t here is persistent left- to- righ t shunt and 6% of defects in the non-Asian population and 33% in Asians.
no evidence of fixed pulmonary vascu lar disease. Standard Spontaneous closure of these defects is rare, and progressive
medical therapy for PAI-I should also be considered. Cl aortic valve regurgitation is common owing to aortic cusp
Patients with an isolated anomalous pulmonary venous distortion. Muscular VSDs (10% of cases) can be located any­
connection may present with clinical and TTE features similar where in the ventricular septum and may be single or multi­
to an ASD, but no atrial-level shunt during agitated saline con­ ple; these defects usually close spontaneously. Inlet defects
trast study. A high clinical index of suspicion should prompt (4%) occur in the superior-posterior portion of the ventricular
focused imaging with TEE, CT, or MRI. septum adjacent to the tricuspid valve. These defects occur as

89
A d u lt Congenital Heart D isease

in most patients. along with the presence or absence of associ­


ated va lve regurgi tation, PA H . and lef'l heart enlargement. M R I
o r CT imaging may be used in select patients for delineation 0 1·
cardiac anatomy in patients with lim ited echocarcliographic
images and measurement of ventricular vol u mes. Cardiac
catheterization is rarely performed to confirm the diagnosis
but is helpful in delineating pulmonary vascular disease and
the Qp:Qs ratio. CJ

Treatment
Although percutaneous device closure is possible for select c::J
VSDs, most patients a re treated surgically. Closure of' a VSD is
generally indicated when there is a significant shunt (Qp:Qs
ratio is 2 . 0 or greater) , and there is evidence of left ventricular
volume overload: an aclclitional ind ication for closure is a his-
tory of enclocarclitis. CJ
For small VSDs with a small left-to-right shunt and no
F I G U R E 2 9 . Positions of various ventricular septal defects viewed from the left left heart enlargement or valve disease, observation is
side of the heart. ( 1 ) perimembranous; ( 2 ) subpulmonary; (3) muscular; (4) in let. appropriate with periodic clinical evaluation and imaging.
Ao = aorta; LA= left atri u m . Large VSDs with right-to-left shunt reversal and PAH
Redrawn from original supplied courtesy o f Dr. William D. Edwards, Department o f laboratory Medicine and (Eisenmenger syndrome) should not be closed ; closure
Pathology. Mayo Clinic. Rochester, MN.
will result in clinical deterioration clue to reduction in
cardiac output.
Patients with a small VSD require no activity restrictions.
part of the atrioventricular septal defect complex and are
If pulmonary vascular disease is present (pulmonary artery
characteristically seen in patients with Down syndrome.
systolic pressure >50 mm Hg) , patients should be advised
against isometric or competitive exercise.
Clinical Presentation
In the absence of PAH , pregnancy in women with VSDs is
Clinical presentation of an isolated VSD depends on the defect
generally well tolerated. Women with VSDs and associated
size and pulmonary vascular resistance. Patients with a small
PAH should be counseled against pregnancy.
VSD and no PAH present with a loud holosystolic murmur
located at the left sternal border that often obliterates the S 1
Follow-up After Ventricular
and may be palpable. Small VSDs do not cause left heart
Septal Defect Closure
enlargement or PAH.
Complications following VSD repair include residual or recur­
A moderate-sized VSD with a moderate left-to-right
rent VSD, arrhythmias, PAH , enclocarclitis, and aortic or tri­
shunt may cause left ventricular volume overload and PAH.
cuspicl valve regurgitation. Cardiovascular evaluation with
Patients remain asymptomatic for many years but eventually
TIE imaging is recommended within 1 year of VSD closure.
present with symptoms of heart failure. The left ventricular
Subsequent clinical and TIE follow-up frequency depends on
impulse may be displaced, suggesting volume overload. A
clinical and cardiac status. Anticoagulation is not routinely
holosystolic murmur is noted at the left sternal border; the
recommended following VSD closure.
duration and quality depend on the pressure gradient between
the left and right ventricles. Progressive PAH results in short­ KEY POI NTS
ening of the murmur. • For small ventricular septal defects with a small left-to- HVC
Large VSDs associated with moderate or large left-to­ right shunt and no left heart enlargement or valve dis-
right shunts are usually detected in chilclhoocl by the presence ease, observation is appropriate with periodic clinical
ofa murmur, heart failure, and failure to thrive. Unless closure evaluation and imaging.
is performed early in life, fixed PAH will ensue within several
• Closure of a ventricular septal defect is indicated when
years, resulting in Eisenmenger syndrome and right-to-left
the pulmonary-to-systemic blood flow ratio (Qp:Qs) is
shunt reversal.
2.0 or greater, and there is evidence of left ventricular

c:::J Diagnostic Evaluation volume overload or a history of enclocarclitis.


• Large ventricular septal defects with right-to-left
The ECG and chest racliograph findings in patients with VSDs
shunt reversal and pulmonary arterial hypertension
are outlined in Table 4 0 . TIE is the primary diagnostic test for
(Eisenmenger syndrome) should not be closed as clo­
patients in whom a VSD is suspected. TIE a llows determina­
sure will result in clinical deterioration.
tion of the location, size. and hemoclynamic impact of the VSD

90
Adult Congenital Heart D isease

Patent Ductus Arteriosus KEY P O I NTS


Pathophysiology • Closure o f a patent ductus arteriosus i s indicated for
Patent ductus arteriosus (PDA) is the persistence of the arterial left-sided cardiac chamber enlargement in the absence
duct that connects the aorta and the pulmonary artery in the of severe pulmonary arterial hypertension.
fetus. Maternal rubella and neonatal prematurity predispose to • A large patent ductus arteriosus with severe pulmonary
PDA. The PDA may be an isolated abnormality or associated arterial hypertension and shunt reversal should be
with other congenital cardiac defects. observed as closure may be detrimental.

Clinical Presentation
A PDA produces an arteriovenous fistula, usually resulting in a
Pu l monary Va lve Stenosis
continuous murmur that envelops the S2• A tiny PDA is gener­ Pathophysiology
ally asymptomatic and inaudible. Patients with a moderate­ Pulmonary valve stenosis is usually an isolated congenital
sized PDA may present with symptoms of dyspnea and heart cardiac lesion, causing obstruction to right ventricular out­
failure. A continuous "machinery" murmur is heard beneath flow. Noonan syndrome, an autosomal dominant disorder, is
the left clavicle. Bounding pulses and a wide pulse pressure often associated with isolated pulmonary valve stenosis or
may also be noted. other congenital cardiac defects. Additional features of
A large PDA causes a large left-to-right shunt and, if unre­ Noonan syndrome include short stature, variable intellectual
paired, may cause PAH with eventual shunt reversal from impairment, unique facial features, neck webbing, and
right-to-left (Eisenmenger syndrome) . A characteristic feature hypertelorism.
of an Eisenmenger PDA is clubbing and oxygen desaturation
that affects the feet but not the hands owing to desaturated Clinical Presentation
blood reaching the lower part of the body preferentially Patients with mild or moderate pulmonary valve stenosis are
(differential cyanosis) . generally asymptomatic, whereas those with severe stenosis
may have exertional dyspnea. On physical examination, mild
CJ Diagnostic Eval uation pulmonary valve stenosis is characterized by a normal jugular
The ECG and c hest ra d iogra p h f i n d i ngs i n patients wi t h PDJ\ venous waveform and precordial impulse. A pulmonary ejec­
a re o u t l i ned in Tab l e 4 0. TTE w i t h color llow Doppler i m a g i ng tion click decreases with inspiration. Moderate or severe pul­
u sually confirms the presence o f a PDA. I n 1xl l i e n t s w i t h severe monary valve stenosis results in right ventricular hypertrophy
PAH , the PDA m ay be d i fficu l t to visua l ize ow i ng to e q u a l iza with a resultant prominent a wave on the jugular venous pres­
lion of pressures i n t he aorta a n d p u l m ona ry artery. I n pa t i e n t s sure waveform and a right ventricular lift. An ejection click
w i t h a PDA a n d e l eva t ed p u lm on a ry a rt e ry pressures . cardiac may be audible, but as the severity of pulmonary valve stenosis
cat heteriza t ion is used to determ ine reversi b i l i ty and shu n t progresses, the click disappears owing to loss of valve pliabil­
size. Angiography confi rms t h e size a n d s h a pe o r t he P DA and ity. An ejection systolic murmur, heard at the left sternal bor­
helps to determ ine whether pe rc u t a n eou s closure i s fe a s i bl e . der, increases in intensity and duration as the severity of pul­
Cardiac CT and MRI may i d e n t i fy a P DA b u t are not used as monary valve stenosis worsens. The pulmonary valve
p r i m a ry diagnostic tec h n iques. Cl component of S2 is delayed and eventually disappears with
increasing severity. A right ventricular S,1 is heard in severe
Treatment pulmonary valve stenosis.
C lo s u re of a PDA is i n d ica te d for lert-sided c ard i a c chamber
Cl e n l a rge me n t i n t he absence or severe PA H . C l osure may be Diagnostic Evaluation
Cl
performed su rg i ca l ly or percutaneously : h owever. su rgical The l·:CG and chest rndiogra p h fi n d i ngs i n patients wi t h p u l ­
closure of a calci fied PDA may be chal le ngi ng. Referral to a monary \·al\ c st enosi s :1rc o u t l i ned in Ta ble "1 0 . TTE w i t h
co n ge n i t a l cardiac center for considera t ion or c los u re options l )o ppl er co n f"irms t h e presence o r pulmonary valve ste n o s i s
is appropriate for t h ese p<1 t ients. Cl and a l lows assessment o f ' i ts se ver i t y. P u l monary valve stenos is
A tiny PDA requires no intervention. In patients with a i s considered severe if' t h e peak gra d i e n t is 60 mm Hg or
small PDA and prior endocarditis, closure is suggested. A gre<ller. Pulmonary cusp mobi l i ty. calcifka t i o n . and t h e effects
moderate-sized PDA is generally closed percutaneously. A or obs t ru c t i o n on t he rig h t ven t ricle may a f 'IC.'ct t rea t me n t
large PDA with severe PAH and shunt reversal should be o p t i on s. Right ,·c n t ricu la r hy pe rt ro phy is expected in pa t i e n t s
observed, as closure may be detrimental. In these patients, wi t h p u l m o n a ry \"<live ste nos is. b u t when right heart e n l a rge­
medical therapy for PAH should be considered. m e n t occ ur s . <1 11 ;1ssociated lesi o n . suc h as pul monary regurgi ­
Patients with a small PDA without PAH do not need any t a t i o n or an ASD. should be su spected . Cardiac C<ll he teriza t i on
limitation of physical activity. Anticoagulation is not rou­ is pri1mir i ly used when percut<rneous i n t erve n t ion is co nsid­
tinely recommended for patients with PDA or following ered . M R I ;1 11d CT <i re n o t rou t i nely used i n patients with pul­
PDA closure. mon a ry val\·e stcnosis. Cl

91
Ad u lt C o n g e nital H ea rt Disease

Treatment The characteristic murmur of aortic coarctation is a sys­


tolic murmur heard in the left infraclavicular region or over
Cl
Pul monary b<1l loon valvuloplasty is the treatment of choice for
pu lmonary valve stenosis and is indicated for asymptomatic the back. When severe, the murmur may be continuous, and a
patients with appropriate pulmonary valve morphology who murmur from collateral intercostal vessels may also be audible
have a peak instantaneous Doppler gradient of at least 60 mm and palpable over the chest wall. In patients with aortic coarc­
Hg or a mean gradient greater than 4 0 mm Hg and pulmonary tation and a bicuspid aortic valve, an ejection click or a systolic
valve regu rgi tation that is less than moderate. Balloon valvulo­ murmur may be heard. An S,1 is often audible.
plasty is also recommended for symptomatic pat ients with
appropriate valve morphology who have a peak instantaneous
Doppler gradient or greater t han so mm Hg or a mean gradient
Diagnostic Evaluation
The ECG and chest radiograph findings in patients with aortic
c:J
grea ter than 30 mm Hg. Operative in tervention is recom­ coarctation are outlined in Table 40. The "figure 3 sign" on chest
mended for pulmonary valve stenosis associated with a small radiograph (Figure 30) is caused by dilatation of the aorta above
pulmonary annulus. more t han moderate pul monaty regurgi­ and below the area of coarctation. Dilatation ofintercostal arter-
tation. severe subvalvar or supravalvar pulmonary stenosis, or ies may result in the radiographic appearance of "rib notching. "
anot her cardiac lesion that requires operative in tervent ion . CJ TIE is usually the initial diagnostic test in patients with
Patients with previous pulmonary balloon or surgical suspected aortic coarctation and aJ!ows identification of asso­
valvuloplasty rarely have residual pulmonary stenosis but are ciated features. such as bicuspid aortic valve and left ventricu­
at increased risk for pulmonary valve regurgitation. Long-term lar hypertrophy. M R I and CT are recommended to identify
clinical and TIE follow-up is recommended after intervention; coarctation severity, the presence of collateral vessels. and
the frequency depends on the severity of pulmonary valve associated abnormalities such as aortic aneurysm . Cardiac
regurgitation. catheterization is reserved for patients who are being consid­
Patients with pulmonary valve stenosis and a peak gradi­ ered for percutaneous intervention. CJ
ent below SO mm Hg do not require exercise restriction.
Patients with more severe stenosis should participate only in Treatment
low-intensity sports. Severe aortic coarctation is associated with reduced survival.
Pregnancy is generally well tolerated in women with pul­ Common causes of morbidity and mortality include systemic
monary valve stenosis unless the lesion is severe. Percutaneous
valvotomy has been performed during pregnancy.
KEY P O I NT
• Pulmonary balloon valvuloplasty is the treatment of
choice for severe pulmonary valve stenosis.

Ao rtic Coa rctation


Pathophysiology
Aortic coarctation is a discrete aortic narrowing, usually
located just beyond the left subclavian artery, causing hyper­
tension proximal and reduced blood pressure distal to the
aortic narrowing.

Clinical Presentation
Patients with aortic coarctation may be asymptomatic or pre­
sent with hypertension, symptoms of exertional leg fatigue, or
headaches. Upper extremity hypertension and reduced blood
pressure and pulse amplitude in the lower extremities are
characteristic findings and cause a radial artery-to-femoral
artery pulse delay. More than SO% of patients with aortic
coarctation have a bicuspid aortic valve.
Turner syndrome is a chromosomal abnormality (4S,X)
characterized by a female with short stature, a broad chest
with widely spaced nipples, webbed neck, and aortic coarcta­
tion. Aortic coarctation is also associated with bicuspid aortic
F I G U R E 3 0 . Chest radiograph of a patient with aortic coarctation exh ibiting
valve, aortic valve and subaortic stenosis, parachute mitral the "fi g u re 3 sig n," caused by d i latation of the aorta above and below the area of
valve, VSD, and cerebral artery aneurysms. coarctation (arrow).

92
Ad ult Co n g e n ital Heart D isease

hypertension, coronary artery disease, stroke, aortic dissec­


tion, and heart failure.

Cl
I ntervention klr coarctalion is recom mended when the
coarctat1on systolic peak (peak-to-peak) pressure gradient is
20 mm Hg or h igher (measured by TTE Doppler or cardiac
catheterization) or if" there is radiologic evidence or severe
coarctat ion with collateral flow. Surgical and percu taneous
in tervent ion options are ava i lable. and selection depends on
the length, location. and severi ty of the coarctalion as well as
t he presence or associated lesions. c:J
Patients with severe residual or unrepaired coarctation,
aortic stenosis, or a dilated aorta should be counseled to avoid
contact sports and isometric exercise.
Women with repaired aortic coarctation and no significant
residua generally tolerate pregnancy well. A comprehensive
prepregnancy evaluation is warranted to evaluate for residua.
Women with severe unoperated coarctation should avoid preg­
nancy prior to intervention. Patients with mild or moderate
residual or unoperated coarctation will generally tolerate preg­
F I G U R E 31 Tetra logy of Fa I lot. A subarterial ventricular septa I defect (asterisk)
.

nancy well but should undergo blood pressure monitoring and pulmonary stenosis (arrow) are associated with secondary aortic override and
during pregnancy and receive cardiovascular follow-up. right ventricular hypertrophy. Ao = aorta; lA = left atrium; LV = l eft ventricle;
RA = right atrium; RV = right ventricle.
Follow-up After Aortic Coarctation Repair Redrawn from original supplied courtesy of Dr. William 0. Edwards, Department of Laboratory Medicine and
Pathology, Mayo Clinic, Rochester, MN.
Hypertension occurs in up to 75% of patients following coarc­
tation repair. Blood pressure control is recommended to
reduce hypertension-related morbidity. I ntervention is often Approximately 15% of patients with tetralogy of Fallot
required for bicuspid aortic valve, aortic aneurysm, aortic dis­ have the 22qll.2 chromosome microdeletion. This increases
section, recoarctation, coronary artery disease, systolic or the chance of congenital heart disease inheritance to approxi­
diastolic heart failure, and intracranial aneurysm. Age at the mately 50% compared with 5% in patients without the micro­
time of repair is the most important predictor of long-term deletion. Genetic testing is recommended for all patients with
survival. Regular follow-up should include evaluation with a tetralogy of Fallot who are planning reproduction. Tetralogy of
congenital cardiologist as well as TIE and aortic imaging. Fallot is also common in persons with Down syndrome.
Surgical repair of tetralogy of Fallot involves patch closure
KEY POI NTS
of the VSD and relief of right ventricular outflow tract obstruc­
• Upper extremity hypertension and reduced blood pres­ tion by patch enlargement. The transannular patch procedure
sure and pulse amplitude in the lower extremities are invariably disrupts the integrity of the pulmonary valve, caus­
characteristic findings of aortic coarctation and cause a ing severe pulmonary valve regurgitation, the most common
radial artery-to-femoral artery pulse delay. reason for reoperation in patients after repair of tetralogy of
• Intervention for aortic coarctation is recommended Fallot. Over many years, pulmonary regurgitation causes pro­
when the coarctation systolic peak (peak-to-peak) pres­ gressive right heart enlargement, tricuspid regurgitation, exer­
sure gradient is 20 mm Hg or higher or if there is radio­ cise limitation, and increased risk for arrhythmias. Annual
logic evidence of severe coarctation with collateral flow. follow-up by a congenital cardiologist is recommended to
monitor these sequelae and determine optimal timing for
• Patients with severe residual or unrepaired aortic coarc­
intervention. Current surgical techniques include attempted
tation, aortic stenosis, or a dilated aorta should be
relief of pulmonary valve stenosis with preservation of native
counseled to avoid pregnancy, contact sports, and iso­
pulmonary valve function.
metric exercise.

Diagnostic Evaluation After Repai r


of Tetralogy of Fallot CJ
Tetra logy of Fa l l ot The r:: cc a n cl chest radiograph ri n d i ngs in pa t i e n t s w i t h
Tetralogy of Fallot is the most common cyanotic congenital repai red tetralogy o r Fal lot are ou t l i ned in Table 40 . Presence
cardiac lesion and includes a large subaortic VSD, infundibu­ of"an a rrhy t hmia should prompt <l search !Or right heart ch<Jm ­
lar or valvular pulmonary stenosis, aortic override, and right ber enlargement f"rom pulmonary valve regurgi tation. The
ventricular hypertrophy (Figure 31) . Unoperated adult patients QRS duration on the l�CG ref'lccts the degree or right ventricu­
are rarely encountered. lar cl i l<Jta l ion . A QRS duration or rno msec or longer and non-

93
Adult Cong e n ital H e a rt Disease

CJ sustained ven tricular tachyca rdia a re risk factors for left intracardiac shunt, filters on intravenous lines should be
sudden cardiac death. used to preven t paradoxical air embolism. Early ambulation,
CONT.
TI'E can confirm t he presence of pulmonary or t ricuspid pneumat ic compression devices, or anticoagu lation is also
valve regurgitation. right ventricular outflow tract obstruction. recommended for prevention of' venous stasis and potential
residual VSD. aortic regurgitation. and aortic dilatation. M R I is venous thrombosis and paradoxical embolism. Because perio­
used to assess right ventricular size and function . which helps perative cardiac complications a re common in t hese patients,
determine appropriate t iming for pulmonary valve replace­ elective operations should be performed at centers that care
ment . Diagnostic catheterizal ion may be requi red to assess for such patients with a coordinated multidisciplinary team
hemodynamics and residual shunts and deli neate coronary approach. Consu ltat ion with a congenital cardiac special ist is
artery and pulmonary artery ;rnatomy. Cl recommended when patients are hospitalized. Cl
Most patients with cyanosis have compensated erythrocy­
Treatment of Tetralogy of Fallot Residua tosis and stable hemoglobin levels. Occasionally, hyperviscos­
Pulmonary valve replacement is recommended in patients ity symptoms occur, including headaches and reduced
with repaired tetralogy of Fallot who have severe pulmonary concentration. Phlebotomy is recommended for a hemoglobin
valve regurgitation with symptoms, decreased exercise tol­ level greater than 20 g/dL (200 g/L) and a hematocrit level
erance, more than moderate right heart enlargement or greater than 65% associated with hyperviscosity symptoms in
dysfunction, or arrhythmias. Long-standing pulmonary the absence of dehydration. Phlebotomy should be performed
valve regurgitation may result in tricuspid regurgitation, no more than two to three times each year. Dehydration should
and repair of t he tricuspid valve may also be needed. be excluded before considering the procedure, and it should
Percutaneous pulmonary valve replacement is now availa­ be followed by intravenous fluid administration. Repeated
ble for select patients with previous operative intervention phlebotomies deplete iron stores and may result in the
for tetralogy of Fallot. production of iron-deficient erythrocytes or microcytosis,
Patients with repaired tetralogy of Fallot and residual increasing the risk of stroke. Iron deficiency in a patient with
sequelae should be cautioned regarding participation in con­ destabilized erythropoiesis is treated with oral iron therapy for
tact sports and heavy isometric exercise. a short time. Iron therapy is discontinued when serum ferritin
and transferrin saturation values are within the normal range.
KEY POINT
Maternal cyanosis impairs normal fetal growth and devel­
• Pulmonary valve replacement is recommended in
opment and increases the risk of intrauterine growth retarda­
patients with repaired tetralogy of Fallot who have
tion and miscarriage. Maternal and fetal morbidity and
severe pulmonary valve regurgitation with symptoms,
mortality are increased, related to the degree of cyanosis,
decreased exercise tolerance, more than moderate right
ventricular function, and pulmonary pressures. If pregnancy
heart enlargement or dysfunction, or arrhythmias.
occurs in a patient with cyanotic heart disease, physical activ­
ity should be curtailed and supplemental oxygen is recom­
Adu lts with Cya n otic Congenita l mended. Preventive measures to decrease the risk of venous
thrombosis and paradoxical embolism also are recommended.
Heart Disease
General Management Eisenmenger Syndrome
Right-to-left cardiac shunts, such as unrepaired or palliated Eisenmenger syndrome is severe PAH and reversal of a con­
tetralogy of Fallot, truncus arteriosus, tricuspid atresia, and genital cardiac shunt caused by VSD, PDA, or, less commonly,
Eisenmenger syndrome, result in hypoxemia, erythrocytosis, ASD. Eisenmenger syndrome has become increasingly rare
and cyanosis. Erythrocyte mass is i ncreased in patients with owing to medical screening, including TIE and appropriate
cyanosis as a compensatory response to improve oxygen intervention.
transport. Conservative medical measures are recommended in the
Physical features include central cyanosis and digital management of Eisenmenger syndrome. Persons with
clubbing. Patients with cyanotic congenital heart disease are Eisenmenger syndrome should be cautioned regarding iron
predisposed to scoliosis. arthropathy, gallstones, pulmonary deficiency, dehydration, acute exposure to excess heat, and
hemorrhage or thrombus, paradoxical cerebral emboli or cer­ moderate or severe strenuous or isometric exercise. Routine
ebral abscess, kidney dysfunction, and hemostatic problems. phlebotomy based on hemoglobin or hematocrit level is not
Because of these problems, a congenital cardiac specialist recommended because iron deficiency and microcytosis may
should evaluate patients with cyanotic congenital heart dis­ lead to increased symptoms; rather, phlebotomy should be
ease at least annually. performed only when symptoms of hyperviscosity occur and

Cl
Patients w i t h cyanotic or complex congenital heart dis­ with saline volume replacement. In addition, chronic high
ease are at increased risk for endocard i t is: therefore, antim i­ altitude exposure should be avoided, as it further reduces oxy­
crobia l prophylaxis is recommended prior to certain nonsterile gen saturation. Women should be cautioned to avoid preg­
procedures. In patients who are hospitalized with a right-to- nancy because of the high risk for maternal mortality.

94
D iseases of the Aorta

All patients with Eisenmenger syndrome should be evalu­


ated annually by a congenital cardiac specialjst. Noncardiac sur­
gery should be performed at centers with expertise in the care of
patients with complex congenital cardiac disease whenever pos­
sible. Meticulous care of intravenous lines with filters to avoid
paradoxjcal air embolism is imperative. Patients with progressive
symptoms may benefit from pulmonary vasodilator therapy.
Long-distance air travel should be approached with cau­
tion and occur in pressurized aircrafts. Select patients may
benefit from supplemental oxygen during prolonged air travel .

KEY P O I N T
• Jn patients with Eisenmenger syndrome, meticulous
care of intravenous lines with filters to avoid paradoxi­
cal air embolism is imperative.

Diseases of the Aorta


I ntrod uction
Aortic disease includes acute life-threatening conditions such
F I G U R E 3 2 . Chest radiograph shows a widened superior mediastinum and a
as aortic dissection, chronic conditions such as aortic ather­ right pleural effusion consistent with aortic dissection.
oma that may lead to embolism, and aortic aneurysms that
carry the risk of rupture. Appropriate surveillance and treat­
ment of aortic disease are crucial to preventing catastrophic
vascular events.
Echocardiography, CT, and MRI have similar sensitivity CJ
and speci fici ty for diagnosis of acute thoracic aoiia disease,
although each has advantages and disadvantages. Transthoracic
I maging of the Thoracic Aorta echocardiography (TTE) a nd t ransesoph ageal echocardiog - ra

Using imaging to screen asymptomatic patients for abnor­


phy (TEE) have the advantages of eing performed at the
b

malities of the thoracic aorta is not recommended, except in


patient's bedside and providing results within several minutes.
patients with underlying vascular pathology (such as Marfan
However. although TrE may be useful in identifying an intimal
or Ehlers-Danlos syndrome), a bicuspid aortic valve, or a fam­
flap. enlarged aort ic root. or aortic insufficiency it is limited in
,

ily history of aortic disease. Abnormalities of the thoracic


evaluating the mid-distal ascending aorta, transverse aorta. or
aorta are sometimes discovered incidentally on chest radiog­
descending thoracic aorta. TEE overcomes the shortcomings of
raphy performed for other purposes. An acute aortic syn­
TI'E bu t requires conscious sedation. The primary advantages
of TEE compared with Cr imaging of suspected aortic dissec-
drome may produce a widening of the mediastinal silhouette,
enlargement of the aortic knob, or displacement of the tra­
tion include its portab i l i ty in the unstable patient and lack of
chea from midline (Figure 32) . !fan abnormality is identified,
contrast injection.
additional noninvasive imaging of the aorta may be useful to
Invasive imaging of the aorta angi ography is rarely
by

determine aortic cross-sectional area. which may predict the


ind ica ted for the diagnosis of acute disease. It may be helpful
risk of aneurysm rupture or dissection. Echocardiography, CT,
in determi ning the length of an aneuiysm and involvement of
and MRI can be used to cross-sectionally image the aorta
branch vessels and should be reserved for patients in whom a
(Table 41) . There is substantial variance in measured aortic
percutaneous intervention is planned. CJ
dimension according to image technique based on measure­ KEY POI NTS
ment of either internal or external aortic diameter. Care must • Screening o f asymptomatic patients for abnormalities HVC
be taken to measure the dimension perpendicular to the long of the thoracic aorta should be reserved for patients
axis of the aorta because oblique or tangential measurements with underlying vascular pathology (such as Marfan
may overestimate the true aortic diameter. or Ehlers-Danlos syndrome), a bicuspid aortic valve,
Maximum aortic diameter at a specific location is gener­ or a family history of amiic disease.
ally reported by imaging results. Dilation above the upper
• Invasive imaging of the aorta by angiography is rarely HVC
limits for normal may represent an aneurysm, but a patient
necessary for the diagnosis of acute disease; it should be
with a larger body size may have a larger absolute aortic diam­
reserved for patients in whom a percutaneous interven-
eter. The maximum ascending aortic diameter may be indexed
tion is planned.
to body surface area (z-score) .

95
D iseases of the Aorta

TABLE 41 . Comparison of Thoracic Aortic Imaging Modalities . - . _ . " .

Modality Advantages Disadvantages

Tra nsthoracic Good visual ization of aortic root/proximal Limited visualization of the d i stal ascend i n g aorta
echocardiography (TIE) ascending aorta and aortic arch with branches of the great vessels

No exposure to radiation or contrast dye Req u i res experienced operator

A l l ows definition of valvular pathol ogy, myocardial A negative TIE does not rule out aortic dissection,
function, pericardia! disease and other imaging techniques m ust be considered.

Bedside d ia g nosis

Tra nsesophageal TEE has su perior imaging q uality versus TIE Req u i res experienced operator
echocardiography (TEE)
Exce l l ent visual ization of the aorta from its root to I nvasive proced u re
the descending aorta

No exposure to radiation or contrast dye

Allows defi n ition of valvular pathology, myocard i a l


function, pericardia! disease

Bedside d iagnosis

CT Visual ization of entire aorta and side branches Exposes patient to radiation, iodi nated contrast
dye
Rapid imaging

M u ltiplanar reconstruction

MRI Vi sual ization of entire aorta and s i d e branches For acute disease, prolonged i mage acquisition
away from acute care area
No exposure to radiation or iodi nated contrast dye
Contra indicated i n patients with implanted
pacemaker or defi brillator
Gadolinium contrast dye contraindicated in
patients with kid ney d i sease

disease. The vast majority of aneurysms affecting the descend­


Tho racic Aortic Ane u rysm
ing thoracic aorta are associated with atherosclerosis. Other
Thoracic aortic aneurysms may involve the aortic root, ascend­
causes of thoracic aortic aneurysm include acquired infection
ing aorta, aortic arch, or descending aorta. Aneurysms of the
and inflammatory conditions, such as syphilis, Takayasu arte­
aortic root and ascending aorta are most common and usually
ritis, and giant cell arteritis.
occur as a consequence of underlying medial degeneration.
The leading cause of death in patients with thoracic aortic
Thoracic aortic aneurysms often are asymptomatic and are
aneurysm is rupture (60%) . Several studies have shown an
frequently detected incidentally during evaluation for another
increasing risk of rupture after the aneurysm has surpassed
problem. Rarely, thoracic aortic aneurysms may be discovered
5 . 0 cm in diameter (4.0-5.0 cm in patients with genetically
because of compressive symptoms, such as hoarseness, stri­
mediated disorders), and a rapid rate of expansion is an inde­
dor, or dysphagia. If rupture occurs, patients may have severe
chest pain, back pain, sudden shortness of breath, or sudden pendent risk factor for rupture. Pregnancy also is associated
death. A diastolic heart murmur or symptoms of heart failure with an increased risk of aortic dissection, particularly in
may occur with aneurysmal dilatation of the aortic root and women with M arfan syndrome, in whom dissection may
subsequent aortic valve regurgitation. occur at aortic diameters that are smaller than would usually
Causes of thoracic aortic aneurysms are listed in Table 42. be considered for elective repair.
These aneurysms typically result from cystic medial degenera­ Smaller thoracic aortic aneurysms can be medically man­
tion that leads to loss of smooth muscle cells and elastic fiber aged with aggressive blood pressure control. 13 -Blockers may
degeneration, resulting in a weakening of the aortic wall. be of particular benefit for reducing the rate of aortic growth
Cystic medial degeneration occurs normally with aging and is in patients with Marfan syndrome, although their benefit in
exacerbated by hypertension. In younger patients, aneurysm treating aneurysms of other etiologies has not been proved.
is most often related to a connective tissue disorder (such as Losartan, an angiotensin receptor blocker, has also been asso­
Marfan or Ehlers-Danlos syndrome). Bicuspid aortic valve is ciated with slower progression of aortic root dilation in
an important risk factor for aneurysm involving the aortic root patients with Marfan syndrome.
and ascending thoracic aorta. Approximately 50% of patients In any patient with a thoracic aortic aneurysm that does
with bicuspid aortic valves have enlargement of the proximal not require immediate intervention, regular surveillance is
aorta that may be independent of the severity of aortic valve important for identifying the development of signs and

96
D iseases of the Aorta

TAB LE 42. Causes of Thoracic Aortic Aneurysms


Category Syndrome
Atherosclerosis

Connective tissue d isorders Ma rfa n syndrome

E h l e rs-D a n l os syndrome, type IV

Loeys-Dietz syndrome
Other genetic a n d/or congenital conditions Fa m i l i a l thoracic aortic aneurysms and aortic dissections (TAAD) syndrome
Bicuspid aortic valve
Tu rner syndrome
Vascu l itis Ta kayasu a rteritis

Giant cell arteritis

Nonspecific (id iopathic) aortitis

Other autoimmune conditions (Behi;et syndrome, syste mic l u pus erythematosus)


I nfectious Septic em bolism

Direct bacterial i n oculation

Bacterial seed ing


Contiguous infection

Syp h i l i s
Aortic injury Prior acute aortic syndrome

Chest trauma

symptoms. Annual echocardiography should be performed to transthoracic ultrasound is the preferred imaging modality in

c::J
monitor aortic growth. Earlier re-evaluation is indicated for these patients.
changes in symptoms or physical examination findings and for The repa i r ol' a t horacic aortic aneurysm is often recom-
hemodynamic assessment related to pregnancy. When the mended prophylacl ical ly lo preve n t lhe morbid i ty and mortal-
aortic dimension nears the threshold for intervention, patients i ty associa ted w i l h a neurysm rupture. In asymptoma t i c
should be referred to an appropriate specialist for evaluation. patients. elective t ho racic a o r l i c repa i r is recom m ended i f t h e
Because patients with a bicuspid aortic valve have an increased a o r t i c root or ascendi ng aorta is greater t ha n 5 .5 cm (5 .5 -
risk of aortic aneurysm and dissection, these patients should 6 . 0 cm for t he descendi ng aorta) or has rapid growth (>0. 5 cm /
undergo echocardiography of the aorta annually if the aortic year) . For genet ical ly media led d isorders (such as Marian syn­
root or ascending aorta dimension is greater than 4.5 cm. In drome) . a lower t h reshold or 5 . 0 cm (4 .0 -5 .0 c m in certain
those with an aortic diameter between 4.0 and 4.5 cm, the pat ients) may be used for repa i r. For pat ients w i t h a bicuspid
examination interval depends on the rate of progression of aort ic valve. repa i r is indica ted if t he aortic d i a meter is greater
dilation and the family history. t ha n 5 . 5 cm <1 11CI is reasonable i f" t he diameter is grea ter than
Familial thoracic aortic aneurysms and aortic dissections 5 . 0 c m and Lhe patient has an increased risk of d issection
(TAAD) is an inherited autosomal dominant condition. ( fam i ly h istory or d i ssection or rapid growth) .
Screening is recommended for first-degree relatives of persons Repai r or ascend i ng aortic and aortic a rch aneurysms
with TAAD once a year or at least every few years. If the muta­ requ i res surgery and may include aort ic valve replacement or
tion is known, genetic testing can identify those relatives who repair in pa t ients w i t h sign i ficant a n n u l a r d i l a tation or associ­
should be screened with aortic imaging. ated aort ic valve pathology. /I. conservative procedure whereby
In patients with Marfan syndrome, follow-up imaging is t h e a neurysm is replaced w i t h a Dacron graft and t h e aortic
recommended 6 months after diagnosis with annual surveil­ valve is preserved has ga i ned widespread use. I f t h e aortic
lance thereafter if the aortic root is less than 4.5 cm in diam­ valve n e e d s rep l acement and the pa t i e n t has a d i la ted
eter and otherwise stable. If the aortic diameter is 4.5 cm or aortic roo t . a composite aortic va lve and aortic root and
greater or shows significant growth over time, then more fre­ ascending aorta gran replacemen t ( Bental l operat ion) may be
quent surveillance is suggested (for example, twice yearly) . performed. The Ben tall operation i ncludes re-implan tation or
Most patients with Marfan syndrome present with enlarge­ t he coronary arteries into t h e ascend i ng aortic graft. Thoracic
ment of the ascending aorta; therefore, serial examination is endovascu l.:ir aort ic repai r (TEVAR) w i l h stent grafting has
focused mainly on assessing this portion of aorta, and emerged as a promising a l terna t ive lo open repa i r for

97
D iseases of the Aorta
------------�--

Cl an e u rysm or the descending t h oracic aorta . TCVAR has been Acute aortic d issection
associated \\'i t h shorter h ospi t <1 I stays and lower hosp i t a l mor
CONT.
b id i ry a n d has t he potcn t i<l l <ld\'<l n tages o r <1\'0 id i ng t h o rncot
omy. aortic cross c l <l m p i ng. and e x t rncorporea l su p port .
Adverse c\'e n t s fo l \ o\\·i ng TE\'A R i n c l u d e s t ro ke . s p i na l
isch e m i a . access com pl i ca t i o ns . and cnd o l ea ks. Cl

K EY P O I N T S
• Patients with a thoracic aortic aneurysm should
undergo annual echocardiography to monitor aortic
aneurysm growth.
• Patients with a bicuspid aortic valve should undergo Acute i ntra m u ra l hematoma
annual echocardiography of the aorta if the aortic root
or ascending aorta dimension is greater than 4.5 cm.
• In asymptomatic patients, elective thoracic aortic repair
is recommended if the aortic root or ascending aorta is
greater than 5 . 5 cm (5.5-6.0 cm for the descending
aorta) or has rapid growth (>0. 5 cm/year) ; for patients
with genetically mediated disorders, the threshold for
repair is lower.

Acute Aortic Synd ro mes Penetrating atheroscl erotic u l cer


c:::J Acute aort ic synclro111cs incl ude aortic d issec t ion . i n t r<l murnl
hc111ato111d. pcne t rn t i ng <l l h erosclerotic u l cer and t rauma
.
i n d uced aort i c rupt ure (Figure 33) . ,\cute aort ic syndromes
t h re;lten central ;1ort ic pressu re. cri t irn l organ pcrrusion . and
su rvi\·<l l . Prompt recogn i t ion <llld cle\ i\'ery or;1ppropriatc 111edica\
;1ncl in tervent ion.ii C<lrC arc crit ical deter111in;111ts o r outcome. Cl

Pathophysiology
In aortic dissection, blood passes through a tear in the aortic
intima, creating a false lumen that separates layers of the aorta. F I G U R E 3 3. Cross·sectional representation of acute aortic syndromes. Acute
aortic dissection: interruption of intima (blue) with creation of an intimal flap and
Propagation of the dissection can proceed in an anterograde or
false lumen formation with i n the media (red). Color flow by Doppler echocardiog­
retrograde fashion from the initial tear, involving side branches raphy or intravenous (IV) contrast by CT is present with i n the false lumen i n the
and causing complications such as tamponade, aortic valve acute phase. Acute intramural hematoma: crescent-shaped hematoma contained
insufficiency, or malperfusion syndromes. An intramural with i n the media without interruption of the intima (blue). No color flow by
hematoma may result from rupture of the vasa vasorum or Doppler echocardiography or IV contrast by CT within crescent. Penetrating athero·
sclerotic ulcer: atheroma (yellow) with plaque rupture disrupting intimal integ rity;
"microtears" in the intima, resulting in a crescent of hema­
blood pool contained with i n inti ma-medial layer (pseudoaneurysm). Color flow by
toma within the media without identifiable interruption of the Doppler echocardiography or IV contrast by CT enters the ulcer crater.
intima. Penetrating atherosclerotic ulcers are most likely
caused by atherosclerosis with subsequent erosion across the
internal elastic membrane of the aorta, allowing for a blood­ co m p l i ca t ion s. The c lassic prese n t a t i o n cons i s t s of .. aort i c
filled false space within the wall of the aorta. chest P<l in ·· described a s severe ripping o r tearing pa i n t ha t
The Stanford classification describes type A dissections as may rc1 d i a t c t o t he c1 ntcrior chest or back, jaw. or abdomen.
originating within the ascending aorta or arch, whereas type B clepen d i ng o n vvh i c h segm e n t o f t he aorta is i nvolved.
dissections originate distal to the left subclavian artery. This Hypert e nsion i s t he most i m porta n t risk factor: o t her risk fac­
nomenclature has been generalized to all of the acute aortic tors i nclude smoki ng a nd atherosclerosis. In t he setting of an
syndromes, although most intramural hematomas and pene­ acute <lO rt ic d i ssec t i o n . hypertension a n d an aort ic regurgi t a ­
trating ulcers are type B lesions. t i o n m u r m u r t ha t is fa i n t . s h o rt in d u ra t i on . a n d low i n pi t c h

Cl Diagnosis and Eval uation


may be prese n t . O t h e r fi n d i ngs o n physical exa m i n a t i o n t h a t
may i ncrease t he i ndex of suspicion i n c l u d e p u lsus pa ra ­
The d i agnosis o r a n acute aortic syndrome req u i res a h igh doxus. asym m e t ri c b l ood pressure i n t h e u pper ex t rem i t i es,
i n dex or SUSpi c i o n bCC<l USC o f" its \ i re t h rc;1 t e n i ng a n d an asym m e t ri c p u lse exa m i n a t ion . A l ow D - cl i mer l e ve l

98
D iseases of the Aorta

c:J (<0 . 5 p g/ml [0: 5 �g/L]) .suggests against an acute aortic syn­ Aortic Atheroma
drome. In aortic d1ssect1on, chest radiography may demon-
CONT. Aortic atherosclerotic plaques are a manifestation of systemic
strate a widening of the mediastinum. An electrocardiogram is
atherosclerosis. Aortic atheroma may be detected incidentally
often abnormal. but nondiagnostic. Clinical suspicion should
during imaging (Figure 34) . The risk of embolism and stroke
be high, and CT, M R I . or TEE for confirmation of the diagnosis
in patients with aortic atheroma is significantly increased for
should not be delayed.
plaques that are mobile or protruding, particularly if the
plaque is greater than 4 mm in size. Thromboembolism may
Treatment
also result from dislodgment of debris from the aortic wall
Patients with a suspected acute aortic syndrome who are not
occurring as a complication of an invasive cardiovascular pro­
in cardiogenic shock should receive medical therapy to control
cedure, such as catheterization, intra-aortic balloon pump
heart rate and blood pressure. Intravenous �-blockers should
placement, or vascular surgery.
be used to target a heart rate of below 70/min. In patients
Noncoronary atherosclerotic disease, including aortic
requiring additional blood pressure control. a rapidly titratable
atheroma, is a coronary heart disease risk equivalent and,
antihypertensive medication, such as sodium nitroprusside,
therefore, should be aggressively treated to reduce the risk of
labetalol. enalaprilat, hydralazine, or nicardipine, should be
future cardiovascular events, including treatment with anti­
given intravenously, with a goal of decreasing the mean arte­
platelet agents and statins. In an observational study, statin
rial pressure to the lowest level that still allows visceral and
therapy was associated with a 17% absolute reduction in
cerebral perfusion.
thromboembolic events in patients with aortic atheroma.
Emergency surgery is recommended for all patients with
type A aortic dissection as well as for type A intramural hema­ KEY POINT
toma. Any delays to surgery should be avoided or addressee!. as • Aortic atheroma is a coronary heart disease risk equiva­
type A dissection has a very high short-term mortality rate. lent, and patients should be aggressively treated with
Concomitant aortic arch reconstruction. coronary artery antiplatelet agents and statins to reduce their cardiovas­
reimplantation, aortic valve repair or replacement, or branch cular risk.
vessel repair may be required depending on the anatomy and
pathology of the lesion.
Uncomplicated type B aortic syndromes may be treated
Abd o m i n a l Aortic Ane u rysm
medically. The INSTEAD trial, which enrolled subjects with
Screening and Surveillance
uncomplicated chroni c type B dissection, showed no dif­
An abdominal aortic aneurysm (AAA) is considered to be
ference in clinical or aortic outcomes at 2 years for patients
present when the minimum anteroposterior diameter of the
treated with TEVAR versus medical therapy alone. Among
aorta reaches 3.0 cm. The most important risk factors for AAA
patients who survived at 2 years. additional long-term fol­
are increasing age, smoking, and male sex (men with AAA
low-up of the I NSTEAD trial demonstrated that TEVA R in
outnumber women by up to 6:1) . Other risk factors include
addition to optimal medical treatment was associated with
atherosclerosis, hypertension. and family history of AAA.
i m proved long-term aorta-speci fic survival and delayed
disease progression. S u rgery is i ndicated for compl icated
type B aortic dissection defined by refractory pain or
hypertension . rapid aneurysmal expansion, rupture, o r
malperfusion syndrome. c:J

KEY P O I NTS
• The classic presentation o f a n acute aortic syndrome
consists of "aortic chest pain" -severe ripping or tearing
pain that may radiate to the anterior chest or back, jaw,
or abdomen, depending on which segment of the aorta
is involved.
• Findings that increase the index of suspicion for an
acute aortic syndrome include pulsus paradoxus, asym­
metric blood pressure in the upper extremities, and an
asymmetric pulse examination.
• Type A aortic dissection has a very high short-term
mortality rate, and emergency surgery is recommended
for all patients without delay. F I G U R E 3 4 . Transesophageal echocardiogram demonstrating a n aortic ather·
oma in the descending aorta (arrows). Left panel: short axis view; right panel: long
axis view.

99
Pe r i p h eral Arte rial Disease

Based on randomized data, the U.S. Preventive Services Task and aort ic anatomy. Random ized tri a ls comparing open repair
Force guidelines recommend a one-time ultrasonographic wi lh F.VA R have found sign i fica n t ly i mp roved shon -term
screening in men aged 65 to 75 years who are active or former (30 clay) morbidity and mortal i ty for E\IAR. but no sign ificant
smokers (see MKSAP 17 General Internal Medicine). The sen­ d ifferences in long- term mortal ity. Moreover. in the long term.
sitivity and specificity of ultrasonography for detection of E\IA R was associated with increased complication rates (includ­
AAA are excellent. ing endovascular leaks. de\·ice migra t ion and fail ure. and post­
AAA rupture has an exceedingly high mortality rate, yet implan tat ion syndrome) and the need [or re intervent ions.
most AAAs never rupture. Thus, deciding when and in whom Because of these potent i a l complications. patients who have
to intervene and electively repair an AAA is of major impor­ undergone E\IAR of' AAA req ui re dil igent fo l low-up wi t h imag­
tance. The strongest risk factor for the rupture of an AAA is ing studies performed annually to eva l uate the status or t he
maximal aortic diameter; this measurement is the dominant graft. Additional long-term data from prospect ive randomized
indication for repair. Estimated annual rupture risk according trials are needed t o fu l ly eva luate t he benefit risk prof1le of' open
to AAA diameter is shown in Table 43. AAA repair versus E\IAR. In patients with severe comorbid d is­
After AAA has been identified, surveillance imaging ease considered not eligible for open surgical correction. a trial
results determine the timing of repair. The frequency of sur­ o r E\IA R versus medical t herapy demonstrated no d if'!erence in
veillance is dependent on baseline aneurysm size; larger aneu­ al l-cause mort a l i ty a t 8 years. but greater cost and i ncreased
rysms expand faster than smaller ones and may require more complicat ions were associated wi t h endovascular repair. Cl
frequent surveillance. If AAA maximum diameter is 3.5 to
KEY P O INTS
4.4 cm, repeat ultrasonography is recommended annually; if
maximum diameter is 4.5 to 5.4 cm, repeat ultrasonography • The most important risk factors for abdominal aortic
should be performed every 6 to 12 months. Elective repair aneurysm are increasing age, smoking, and male sex;
should be considered for AAA of 5.5 cm in diameter, for those one-time ultrasonographic screening is recommended
that increase in diameter by more than 0.5 cm within a for men ages 65 to 75 years who have ever smoked.
6-month interval, and for those that are symptomatic (tender­ • Elective repair should be considered for an abdominal
ness or abdominal or back pain). For women, elective repair aortic aneurysm of 5.5 cm in diameter, for those that
may be considered for an AAA that reaches 5.0 cm in diameter. increase in diameter by more than 0.5 cm within a
6-month interval, and for those who are symptomatic.
Treatment • Medical therapies for abdominal aortic aneurysm focus
Medical therapies for AAA focus on targeting modifiable risk on targeting modifiable risk factors for abdominal aortic
factors for AAA and cardiovascular disease with the goals of aneurysm and cardiovascular disease; smoking cessa­
reducing aneurysm expansion or rupture, reducing morbidity tion is the cornerstone of therapy for active smokers.
and mortality associated with repair, and reducing cardiovas­
cular morbidity and mortality. Smoking cessation is the cor­
nerstone of therapy for active smokers.

Cl gical or enclovascular aneurysm repair (E\IA R ) is individual ized


I f repai r is being considered. t he choice between open sur­
Periphera l Arterial Disease
based on t he patient's age. comorbidities. palient preterence. Epidemiology and Screening
Lower extremity peripheral arterial disease (PAD) i s a disor­
der of the aortic bifurcation or arteries of the lower limbs.
TABLE 43. Annual Rupture Risk of Abdominal Aortic The most common cause of PAD is atherosclerosis, and PAD
Aneurysm by Diameter
shares the primary atherosclerotic risk factors of smoking,
Aneurysm Diameter Annual Rupture Risk diabetes mellitus, and increasing age. However, unlike other
<4.0 cm <0.5% types of arterial disease, the population burden of PAD is
higher in women than in men. PAD is defined noninvasively
4.0-4.9 cm 0.5%-5%
by calculation of the ankle-brachia! index (AB!) . PAD is com­
5.0-5.9 cm 3%- 1 5%
mon, with an estimated unselected adult prevalence in the
6.0-6.9 cm 1 0%-20% United States of approximately 6% in persons aged 40 years
7.0-7.9 cm 20%-40% and older.
2'.8 .0 cm 30%-50% There is a wide spectrum of clinical manifestations for
PAD:
Adapted with permission of Elsevier Science and Technology Journals, from
Brewster DC, Cronenwett JL, Hallett JW Jr, et al; Joint Council of the American
Association for Vascular Surgery and Society for Vascular Surgery. Guidelines for
• Asymptomatic patients found to have PAD from a screen­
the treatment of abdominal aortic aneurysms. Report of a subcommittee of the ing AB! (20%-50%)
Joint Council of the American Association for Vascular S u rgery and Society for
Vascular Surgery. J Vase Surg. 2003 May;37(5):1 1 06· 1 7. [PMID:l 27563631; • Atypical symptoms associated with exercise limitation
permission conveyed through Copyright Clearance Center, Inc.
(40%-50%)

1 00
Pe ri p heral Arterial D isease

• Classic intermittent claudication (10%-35%) symptoms of leg ischemia, but many are asymptomatic.
• Ischemic pain and ulceration in a lower extremity from Among symptomatic patients, the perception of symptoms
chronic limb ischemia (1%-2%) can vary from minimal to severe and may not necessarily cor­
relate with severity of disease. Discriminating claudication
PAD is associated with reduced exercise capacity and func­
from pseudoclaudication (a result of narrowing of the lumbar
tional status regardless of the symptomatic state. Limb-specific
spinal canal) is important (Table 44) . In severe PAD or critical
outcomes (such as limb ischemia or amputation) occur less
limb ischemia, patients may describe discomfort that worsens
frequently than systemic manifestations of atherosclerosis
with elevation of the leg and improves with dependent posi­
(myocardial infarction or stroke) . Both asymptomatic and
tioning, such as dangling the leg off the foot of the bed.
symptomatic patients with PAD are at increased risk of cardio­
Components of the physical examination of patients with
vascular morbidity and mortality, and PAD is considered a coro­
suspected PAD are shown in Table 45. Patients at risk for lower
nary heart disease risk equivalent. Patients with PAD are likely
extremity PAD should undergo comprehensive pulse exami­
to have concomitant vascular disease in other arterial beds and
nation and inspection of the feet. Shoes and socks are removed
have an annual cardiovascular event rate of 5% to 7%. Thus,
and the feet inspected for skin changes. Physical examination
early recognition of PAD provides a unique opportunity to iden­
may reveal diminished, absent, or asymmetric pulses below
tify persons at early increased risk for a cardiovascular event
the level of stenosis, with occasional bruits over stenotic
and to modify risk factors. However, the U.S. Preventive Services
lesions and evidence of poor wound healing. Other physical
Task Force concluded that the current evidence is insufficient to
findings seen in PAD include a unilaterally cool extremity; a
assess the balance of benefits and harms of screening for PAD
prolonged venous filling time (>20 seconds); shiny atrophied
and cardiovascular disease risk assessment with the AB! in
skin; ulceration; and thickened, ridged, and brittle nails. Most
adults. Initial testing may be considered in persons with the fol­
patients with upper extremity PAD are asymptomatic, and
lowing characteristics that signify high risk:
PAD may be detected only by the finding of asymmetric arm
• Exertional leg discomfort blood pressures, with a typical differential in systolic blood
• Nonhealing wounds pressures of greater than 15 mm Hg.
• Age 50 years or older with a history of smoking or diabetes
Diagnostic Testing
KEY P O I NTS There are several diagnostic modalities to choose from to
• Patients with peripheral arterial disease are likely to have assess for PAD. Measurement of the AB! is a simple, inexpen­
concomitant vascular disease in other arterial beds and sive, and noninvasive technique that correlates well with
have an annual cardiovascular event rate of 5% to 7%. angiographic disease severity and functional symptoms.
• Initial testing for peripheral arterial disease may be Patients should rest for 5 to 10 minutes before measuring the
considered in persons with exertional leg discomfort, ankle pressure and should be lying flat for an accurate ABI
nonhealing wounds, or aged 50 years or older with a measurement, with the head and heels fully supported. The
history of smoking or diabetes mellitus. study is performed by applying a blood pressure cuff to the
calf and measuring the systolic blood pressure by palpation or
Doppler at the ankle. The blood pressure is recorded for both
the dorsalis pedis (DP) and the posterior tibial (PT) arteries,
Eva l u ation and the higher of the two is used as the ankle pressure. This
H istory and Physical Examination procedure is repeated for the opposite ankle. The brachia!
Patients at risk for PAD should undergo a vascular review of artery systolic blood pressure is measured in a similar fashion
symptoms to assess walking impairment, claudication, and in both arms. For the measurement of AB!, measuring the
ischemic pain while at rest. Patients with PAD may have limb pressures in the following order is recommended: first

TABLE 44. Discriminating Claudication from Pseudoclaudication


Characteristic Claudication Pseudoclaudication

Nature of d iscomfort Cra m ping, tig htness, aching, fatigue Same as claudication plus tingl ing,
burning, n u m bness, weakness

Location of discomfort B uttock, h i p, thigh, calf, foot Same as claud ication; most often bilateral
Exercise-induced Yes Variable

Wa lking d i stance at onset of symptoms Consistent Variable


Discomfort occurs with sta n d i n g sti l l No Yes
Action for relief Stand or sit Sit, flexion at the waist
Time to relief <5 m i n utes $30 minutes

1 01
Pe riph eral Arterial Disease

TABLE 45. Clinical Examination of Patients for Peripheral repetitive active pedal plantar flexion ("toe ups") while stand­
Arterial Disease ing has been proposed for an office-based assessment of pos­
texercise ABI. In patients with even mild PAD, the ankle
Measure blood pressure in both a rms (systo lic blood pressure
d ifference > 1 5 mm Hg is suggestive of subclavian ste nosis) pressure decreases more during treadmill exercise compared
with healthy patients, and the recovery time to the pre-exer­
Auscu ltate for presence of arterial bruits (e.g., femoral artery)
cise value after exercise cessation is prolonged, proportional to
Pa l pate for presence of an abdominal aortic aneurysm
the severity of PAD. A decrease of the AB! by 20% after exercise
Pal pate and record pulses ( radial, brachia!, carotid, femoral, suggests significant PAD.
popliteal, posterior tibial, dorsa l i s pedis)
A common procedure in many vascular laboratories is
Evaluate for elevation pallor and dependent rubor
measurement of multiple or segmental pressures in the lower
I n spect feet for ulcers, fissures, call uses, tin ea, and tendin ous extremities along with pulse volume recordings, which meas­
xanthoma; evalu ate overa l l skin care
ure the magnitude and contour of the blood pulse volume.
Once a patient is placed in a supine position for 5 to 10 min­
arm, first PT artery, first DP artery, other PT artery, other DP utes, blood pressures are obtained at successive levels of the
artery, and other arm. If the systolic blood pressure of the first extremity, localizing the level of disease fairly accurately.
arm exceeds the systolic blood pressure of the other arm by Noninvasive angiography may be performed for ana- �
more than 10 mm Hg, measurement of the blood pressure of tomic delineation of PAD in patients requiring surgical or W
the first arm should be repeated (to temper the "white coat endovascular intervention. CT angiography (CTA) is rapid
effect" of the first measurement) , and the first measurement and easily available but requires the administration of intra­
of the first arm should be disregarded. For each leg, the AB! is venous contrast dye. The risk of dye-induced nephropathy
calculated by dividing the ankle pressure by the higher of the must be considered in patients with chronic kidney disease,
two brachia! pressures. I n healthy persons, the ankle and arm especially if an endovascular repair is being contemplated,
systolic pressures are approximately the same or slightly because this would entail repeat administration of iodinated
higher in the ankle. Thus, a normal resting AB! is between contrast dye. Magnetic resonance angiography (MRA)
1.00 and 1 .40 (Table 46). However, ifa fixed obstruction of the requires intravenous gadolinium for PAD definHion; gado­
arterial lumen supplying the lower extremity is present, a linium has been associated with nephrogenic systemic fibro-
pressure gradient occurs, resulting in a reduced downstream sis in patients with severe kidney disease. Additionally, MRA
pressure and concomitant reduction in the AB! (AB! <0.90). may be contraindicated in patients with implanted pacemak-
AB! values between 0.91 and 0.99 are considered borderline, ers or cardioverter-defibrillators. Both CTA and MRA com-
and while equivocal for PAD, these patients are at increased pare favorably with digital subtraction (invasive) angiography
risk for adverse cardiovascular events. An AB! greater than for the detection of occlusive arterial disease. CTA has addi­
1 .40 is associated with calcification of the arterial wall and tional benefits of demonstrating vascular calcification, has
may occur in patients with medial calcinosis, diabetes melli­ higher spatial resolution than M RA, and aJ!ows visualization
tus, or end-stage kidney disease. This finding is uninterpret­ of adjacent soft tissues and of endovascular stent grafts.
able, and therefore a toe-brachia! index may be useful for Invasive angiography. in most instances, is used only as part
diagnosing PAD in patients who are at risk for PAD and have of an interventional procedure. CJ
an ABI above 1.40. A toe-brachia! index below 0.70 is consid­
K E V P O I N TS
ered diagnostic of PAD. This test is typically performed in a
vascular laboratory. • An ankle-brachia! index of 0.90 o r below i s diagnostic
The sensitivity of the ABI is increased when it is measured for peripheral arterial disease.
after exercise. A common exercise protocol involves walking • An ankle-brachia! index greater than 1.40 is uninter­
on a treadmill at 2 mph at a 12% incline for 5 minutes or until pretable, and a toe-brachia! index should be obtained
the patient is forced to stop because of leg pain. Alternatively, for diagnosing peripheral arterial disease.
• An exercise ankle-brachia! index (ABI) may help estab­
TAB L E 46. Interpretation ofthe Ankle·Brachial Index lish the diagnosis of peripheral arterial disease (PAD)
Ankle·Brachial Index Interpretation in patients in whom the resting ABI is equivocal; a
> 1 .40 decrease of the AB! by 20% after exercise suggests
Noncompressible (calcified)
vessel ( u n i nterp reta ble result) significant PAD.
1 .00- 1 .40 Normal
0.91 -0.99 Borderline M ed ica l Thera py
0.41 -0.90 Mild to moderate PAD The treatment of PAD has evolved over the past decade to
0. 00-0 . 4 0 Severe PAD include a broad approach, focusing on the reduction of adverse
cardiovascular events, improving symptoms in claudication,
PAD = peripheral arterial disease.
and preventing tissue loss in critical limb ischemia.

1 02
Periph eral Arterial D isease

Cardiovascular Risk Reduction Diabetes is one of the strongest risk factors for PAD. Although
Cigarette smoking is the strongest risk factor for PAD and for intensive diabetic control (hemoglobin A1c �7%) can be effective
subsequent complications, and smoking cessation is impera­ to reduce microvascular complications, no prospective studies
tive for patients with PAD. Smokers with PAD who quit have have demonstrated that this strategy reduces macrovascular com­
lower risks for myocardial infarction and stroke and an plications, including PAD. In general, diabetes treatment of
improved long-term survival versus those who continue to patients with PAD should follow current national treatment
smoke. Cessation of cigarette smoking is also associated with a guidelines, with meticulous attention paid to foot care.
lower amputation rate, lower incidence of rest ischemia,
improvement in maximal treadmill walking distance, and Symptom Relief
improved bypass graft patency rates. Exercise rehabilitation is a class I, level of evidence A, recom­
A moderate- or high-intensity statin is indicated in all mendation for the treatment of claudication in patients with
patients with PAD. Although no prospective statin trials have PAD. Either treadmill exercise or resistance training improves
focused singularly on PAD, the Heart Protection Study (HPS) functional performance, but treadmill exercise has shown
studied 6748 patients with PAD. Use ofsimvastatin was associ­ greater increases in the 6-minute walk distance and in the
ated with relative reduction in overall mortality by 12%, vascu­ maximum treadmill walking time compared with resistance
lar mortality by 17%, and cardiovascular events by 24% at a training. Although supervised exercise therapy has statistically
mean follow-up of 5 years. significant benefit on treadmill walking distance compared with
Antihypertensive therapy is effective at reducing cardio­ non-supervised regimens, a randomized trial found benefit in
vascular events in patients with PAD. Current guidelines rec­ both a supervised exercise and a home-based exercise program
in patients with claudication. The National Institutes of Health­
ommend a brachia! blood pressure goal of less than
140/90 mm Hg for patients with PAD. Whereas cardiovascular funded CLEVER trial evaluated the relative efficacy, safety, and
groups have advocated a lower target of 130/80 mm Hg for health economic impact of noninvasive and endovascular treat­
patients with PAD and diabetes or kidney disease, the recently ment strategies for patients with aortoiliac PAD and claudication.
Whereas supervised exercise provided a superior improvement
released report of the Eighth Joint National Committee (JNC 8)
in walking outcomes, patients undergoing endovascular repair
suggests a therapeutic target of 140/90 mm Hg with or with­
had greater improvements in self-reported physical function.
out diabetes or kidney disease. Data are insufficient to recom­
Cilostazol is a phosphodiesterase inhibitor with antiplate­
mend use of one class of antihypertensive agents over another.
let activity and vasodilatory properties. A meta-analysis of
Concern has been raised regarding the use of �-blockers in
eight randomized trials that included 2702 patients with PAD
the treatment of hypertension in the setting of intermittent
with claudication found that cilostazol improved maximum
claudication; however, data have not supported this concern.
and pain-free treadmill walking distance and quality-of-life
Thus, �-blockers are not contraindicated in patients with PAD
measures. Cilostazol may have an additional benefit of reduc­
and should be used when appropriate. Nonetheless, thiazide
ing restenosis and repeat revascularization following endovas­
diuretics, ACE inhibitors or angiotensin receptor blockers,
cular therapy. Because milrinone, another phosphodiesterase
and calcium channel blockers remain first-line therapy in
inhibitor, increased mortality in patients with heart failure,
patients with hypertension. In the Heart Outcomes Prevention
cilostazol should not be used in this population. In the absence
Evaluation (HOPE) trial, ramipril (5 to 10 mg/d) decreased
of heart failure, a therapeutic trial of cilostazol should be con­
cardiovascular events in subjects with PAD with or without
sidered in all patients with lifestyle-limiting claudication.
hypertension. The use of an ACE inhibitor is reasonable in
Pentoxifylline is a xanthine derivative that affects platelet
patients with lower-extremity PAD to reduce the risk of
adhesiveness and whole-blood viscosity. A meta-analysis
adverse cardiovascular events.
demonstrated a modestly improved walking distance with
Current guidelines recommend antiplatelet therapy in
pentoxifylline, but it was substantially less effective than either
patients with PAD. Because most patients with PAD have con­
cilostazol or a supervised exercise program.
comitant coronary artery disease or cerebrovascular disease, aspi­
Some data have shown that therapy with statins and ACE
rin is an acceptable antiplatelet agent in this setting despite the
inhibitors leads to substantial improvements in pain-free
lack of good-quality evidence supporting improved outcomes in walking time and maximal walking time.
PAD. In addition to aspirin, clopidogrel (as monotherapy, not with
aspirin) is a reasonable alternative strategy, and some data suggest KEY P O I NTS
clopidogrel may be particularly effective in patients with PAD. • Smoking cessation, lipid control, antiplatelet medica­
Oral anticoagulation with warfarin has not been estab­ tions, and antihypertensive therapy are important com­
lished to reduce cardiovascular events in patients with PAD ponents of cardiovascular risk reduction in patients
because it is no more effective than antiplatelet therapy and with peripheral arterial disease.
confers a higher risk of bleeding. There are no data to suggest • Exercise therapy and cilostazol are effective treatments
using newer anticoagulants in patients with PAD for the for claudication.
reduction of cardiovascular events.

1 03
Per i p h e ra l Arteria l Disease
- --- --------

Pa t i e n t s w i t h c r i t i c a l l i m b ische m i a ( A B ! <0 . 4 0 , a nat


l nterventional Thera py
wave lo rm o n pu lse vo l u me record i ng, a n d low o r absen t
Endovascular or surgical revascularization is indicated for
p e d a l flow on d u plex u l t raso nography) shou l d be co n s i d ­
patients with significant disability due to claudication
ered f'or i m m e d i a t e reva scu l a r i z a t i o n . e i t her s u rg i c a l o r
who have had an inadequate response to exercise or phar­ Cl
e n d ovascu l a r.
macologic therapy, provided the benefits outweigh the
risks. Revascularization should be considered in the fol­ KEY POI NTS
lowing settings: • Endovascular o r surgical revascularization i s indicated
• The patient is significantly disabled by claudication, re­ for patients with significant disability due to claudica­
sulting in an inability to perform normal work or other tion who have had an inadequate response to exercise
activities that are important to the patient. or pharmacologic therapy.

• The patient has not had an adequate response in symptom • Patients with critical limb ischemia should be consid­
alleviation to exercise rehabilitation and pharmacologic ered for immediate revascularization, either surgical or
therapy. endovascular.

• The patient is able to benefit from an improvement in


claudication; that is, exercise is not limited by another Acute Li m b lschemia Cl
cause, such as angina, heart failure, lung disease, or or­ Acute l i m b ischemia i s a n u ncommon m a n ifestation o f ' PA D
thopedic problems. t h a t ca rries sign i f'i c a n t r i s k o f morb i d i ty a n d mo rt a l i ty.
In addition, the characteristics of the lesion should per­ Physical f'i n d ings a re characterized by t he " 6 Ps" - Parest hesia,
mit appropriate intervention at low risk with a high likeli­ Pa i n . Pa l l or. P u l se lessness, Poi ki lothermia (cool ness) . and
hood of initial and long-term success. Inflow (aortoiliac l'a ra lys is . Acute l i m b ischemia is caused by acute t h rombos i s
artery) and outflow (infrapopliteal artery) should always be of' a l i m b a rtery or bypass graft, embolism. d i ssec t i o n , o r
assessed before revascularization. Inflow lesions should be t ra u m a . Li m b v i a b i l i ty is determ i ned by c l i n ica l findi ngs a n d
revascularized first followed by outflow lesions if bothersome b y t h e viabi l i ty o f t i ssues a t t h e t i me o f po tent i a l revasculari ­
symptoms persist. zation (Table 47) . The onset of' acute l i m b ischem i a represen t s

Cl fer a l ower r i s k of periopera t ive adverse even ts t h a n open


E n dovasc u l a r o p t i o n s a re m i n i m a l ly i nvasive a n d con ­ a t rue emergen cy. Co m p l ication rates of' acute l i m b ischemia
are h i g h ; despi te u rge n t revascu l a ri za t i o n , amputation occurs
su rgery. The most com mon endovascu l a r opt ions i n c l u d e in 10% Lo 1 5% of patients d u r i ng h os p i t a l i za t io n .
bal loon <1 11gi o p l a s ty w i t h o r w i t ho u t sten t i ng : o t h e r opt ions Approx i mately l S 'Y., t o 20'Y., o f p a t i e n ts d i e w i t h i n 1 year a f'ler
i n c l u d e d r ug-coa ted ba l l oons . l a sers, a t herectomy devi ces . presentat io n .
a n d t h e r m a l dev ices . End ovascu l a r t rea t m e n t has become A n t i coagu l a t i o n s h o u l d be sta rted as soon as t h e d i ag­
the revascu l a ri z a t i o n m od a l i ty o f' c h o i ce for a o rt o i l i ac d i s ­ n o s i s h a s been m a d e . E a rl y consu l ta t i o n w i t h a vasc u l a r
ease i n a l l b u t t h e m o s t c o m p l e x l es i o n s . E n dovascu l a r spec i a l i s t i s a pp ro p r i a te . Pa t i e n t s with a c u t e l i m b i s c h e m i a
procedures have h i g h rates o f ' procedu ra l success, low res te­ a n d a sa lvageab l e e x t re m i t y s h o u l d u n d e rgo e m erge n t
nosis ra tes. a n d decreased m o rb i d ity a n d mort a l i ty co m ­ a n gi ography to d e f i n e L h e a n a t o m i c level o f o cc l u s i o n a n d
pared w i t h open s u rgical rep a i r. I n pLl l i e n ts w i t h aorto i l i ac assess t rea t m e n t o p t ions . Trea t m e n t c o n s i s t s o f s u rgi c a l o r
d i sease . p r i m a ry sten t i ng i s associ a ted w i t h a 43'7., red u c t i o n perc u t a n e o u s reva s c u \ a r izat i o n o r, i n s e l e c t e d pa l i en t s .
i n l ong - term fa i l u re c o m pare d wi t h a n g i o p l asty a l o n e . L h ro m b o l y t i c t h e rapy. Cat heter-d i rected t h ro m b o lys i s i s
l n f'ra i ng u i n a l ( fe m o ra l . po p l i t e a l . a n d i n f'rapo p l i t e a l a r ter­ a n a dj u n c t ive a p p roach t h a t m ay b e usefu l i n p a t i e n t s w i t h
i es ) revascu l a ri za t i o n o f't'e rs less robust c l i n i c a l benefi t t h a n a v i a b l e o r m a rg i n a l ly t h reatened l i m b . re cent occ l us i o n
aort o i l i a c revascu l a ri z a t i o n . As a ge nera l ru l e , excel l e n t ( n o m o re t h a n 2 weeks' d ur a t i o n ) , t h ro m b o s i s o f a sy n ­
l o n g- term resu l t s c a n b e a c h i eved w i t h i n terve n t ion fo r t h e t i c graft or an occluded s te n t . a nd a t l e a s t o n e i de n t i fi ­
s h o rt f'o cal l es i o n s , whereas l o nger l es i o n s are asso c i a t e d a b l e d i s l a l ru n o ff vesse l . A l t hough m a ny p a t i e n t s w i l l
w i t h less d u ra b l e p r i m a ry pate ncy rates fo l lo w i ng i n terve n ­ req u i re reva s c u l a ri z a t i o n fo l l owing t h ro rn b o lysis, t h e p ro ­
t io n . I n pat i e n t s w i t h in f'ra i ngu i n a l d i sea:e. bal loon a ngi o­ ced u re i s freq uen t ly less c o m p l ex t h a n i f t h ro m b o lys i s was
p l as ty is usua l ly perfo r med w i t h o u t ste n t i ng. as t h e l a t ter not perform ed .
p roced ure has s ign i f'i c a n t reoccl u s i o n a n d restenosis rates Fol lowi n g l i m b reperfusion . close m o n i tori ng is requ i red
a n d does not appear to i m p rove outcomes a fter i n t erven ­ for l i m b edema and t issue swe l l i ng t hat may cause a compart ­
t i o n . Recent t ri a l s suggest t h at use of d rug-coated ba l l oo n s ment syndrome. Symptoms a n d signs i nclude severe pai n .
may b e a n e ffe c t ive endovasc u l a r a pproa ch : however. l o ng­ hypoest hesia, and weakness o f t h e a ffected l i m b : myoglob i n u ­
t e r m d a t a a re need e d . Les i o n s vv i t h complex a n a t o my ( fo r ria a n d i n creased seru m c rea t i n e k i nase leve l s often occur. I r
exa m p l e . long-segm e n t stenos i s . m u l t i f o c a l stenosis. ecce n ­ t h e compartment syndrome occurs, s u rgical fasciotomy i s
t ri c/ca l c i f'ied s tenosis . or l ong-segmen t occ l u s i o ns ) m a y b e i nd i cated t o prevent i rreversible neurologic a n d soft- t issue
better t reated s u rgica I ly. damage. Cl

1 04
Card iovascu l a r D isease in Ca ncer S u rvivors

TABLE 47. Categories and Prognosis of Acute Limb lschemia


Category Sensory Loss Muscle Weakness Arterial Doppler Prognosis
Signals
Viable None None Audible Not i m med iately
threatened

M a rginally threatened None to minimal (toes) None Inaudible Salvagea ble with
prompt treatment

Immed iately threatened More than toes M i l d to moderate Inaudible Salvagea ble only
with immed iate
revascula rization
I rreversi ble Profound anesthesia Profound/paralysis Inaud i ble Not viable; major tissue
loss inevitable

Adapted from Hirsch AT, Ha ska I ZJ, Hertzer N R , eta!; American Association for Vascular Surgery; Society for Vascular Surgery; Society for Cardiovascular Angiography and
Interventions; Society for Vascular Medicine and Biology; Society of lnterventional Radiology; ACC/AHA Task Force on Practice Guidelines Writing Committee to Develop
Guidelines for the Management of Patients With Peripheral Arterial Disease; American Association of Cardiovascular and Pulmonary Rehabilitation; National Heart, Lung, and
Blood Institute; Society for Vascular Nursing; TransAtlantic Inter-Society Consensus; Vascular Disease Foundation. ACC/AHA 2005 Practice Guidelines for the management of
patients with peripheral arterial disease (lower extremity, renal, mesenteric, and abdominal aortic): a collaborative report from the American Association for Vascular Surgery/
Society for Vascular Surgery, Society for Cardiovascular Angiography and Interventions, Society for Vascular Medicine and Biology, Society of lnterventional Radiology, and the
ACC/AHA Task Force on Practice Guidelines (Writing Committee to Develop Guidelines for the Management of Patients With Peripheral Arterial Disease): endorsed by the
American Association of Cardiovascular and Pulmonary Rehabilitation; National Heart, Lung, and Blood Institute; Society for Vascular Nursing; TransAtlantic Inter-Society
Consensus; and Vascular Disease Foundation. Circulation. 2006 Mar 21 ; 1 1 3( 1 1 ):e488. IPMID: 1 6549646]

KEY POI NTS Although pericarditis may occur acutely with chest radia­
tion, manifestations of radiation-induced cardiotoxicity fre­
• Physical findings of acute limb ischemia are character­
quently develop after a long indolent period (5 to 20 years or
ized by the "6Ps" - Paresthesia, Pain, Pallor,
later) owing to the chronic nature of the pathology and there­
Pulselessness, Poikilothermia (coolness) , and Paralysis.
fore require a high index of suspicion in at-risk patients. The
• Patients with acute limb ischemia should receive imme­ clinical manifestations are related to the affected portion of
diate anticoagulation therapy; those with a salvageable the cardiovascular system. Myocardial fibrosis leads to a
extremity should undergo an emergent evaluation that restrictive cardiomyopathy, resulting in poor chamber compli­
defines the anatomic level of occlusion and leads to ance and diastolic heart failure (see Myocardial Disease) . Signs
prompt endovascular or surgical revascularization. of restrictive cardiomyopathy have been reported in 15% to
• Following limb reperfusion in patients with acute limb SO% of patients with previously treated Hodgkin lymphoma
ischemia, close monitoring is required for limb edema and are more evident in those who have received therapy with
and tissue swelling that may cause a compartment cardiotoxic drugs. Constrictive pericarditis also frequently is
syndrome. present. The potential for coexistent constrictive pericarditis
and restrictive cardiomyopathy presents significant challenges
in the management of these patients, as the clinical manifesta­
tions frequently overlap while the treatments for these two
Ca rd iovascu lar Disease disorders vary considerably (that is, pericardiectomy versus
cardiac transplantation) . Pericardia! effusion is particularly
in Ca ncer S u rvivors common in patients treated for esophageal cancer (approxi­
mately 25%) , with a median presentation time of 5 months
Ca rd iotoxicity of Radiation after therapy in one report. Any cardiac valve can be affected
Thera py to the Thorax by radiation injury (5%-40% of patients) , although left-sided
Chest radiation therapy is associated with significant cardio­ lesions predominate and frequently occur with mixed stenosis
toxicity, which can manifest as a complex, life-threatening and regurgitation. Coronary artery disease typically is ostial or
disorder. Radiation-induced toxicity can affect nearly every proximal in location; microvasculopathy (disease involving
component of the cardiovascular system, with manifestations vessels that are not epicardial in location) also can occur.
such as pericarditis (acute or chronic) , cardiomyopathy. aorti­ Recognition of the potential for cardiotoxicity has led to
tis, conduction system disease, valvulopathy, and coronary reductions in radiation exposure in the treatment of chest
artery disease (Table 48) . These pathologic alterations are malignancy. Most contemporary studies have demonstrated
likely caused by the generation of reactive oxygen species from significant decreases in cardiac mortality in comparison with
irradiation, which leads to blood vessel injury and a cascade of historical studies. However, the increased risk of cardiotoxicity
inflammation, ischemia with loss of capillary density, and and death due to vascular complications attributable to radia­
fibrosis of the cardiovascular structures. tion therapy remains. Thus, lifetime clinical monitoring for

1 05
Cardiovascu l a r Disease i n Cancer S u rvivors

TABLE 48. Cardiotoxicity of Radiation Therapy to the Thorax


Manifestations Clinical Onset After Comments
Radiation Therapy

Acute pericarditis or pericardia! As early as 2 months, but A common cardiac compl ication historically ( 25%), less
effusion 5 months on average com mo n now ( 2% ) owing to methods m i n i m izing mediastinal
irradiation

Pericardia! fibrosis and As early as 1 .5 years, but often Risk persists for >25 years
constriction 1 0 to 1 5 years or more
RV more exte nsively involved, leading to ma rked fi nd ings of RV
fa i l u re

Acce lerated coronary Average o nset 7 years Pred i l ection for involvement of ostia or proximal segments of
atherosclerosis coronary arteries

Patients with traditional risk factors for CAD are at hig her risk

May manifest as myocardial infarction

S u dden death may occur rarely

Valvu l a r fibrosis and 1 0 to 25 years or more Frequency greater i n left- vs. right-sided valves
reg u rg itati on
Clini ca lly significant aortic regurgitation may occur i n 2:25% of
lo ng-term survivors

Slowly progressive and requires lifelong monitoring

Concom itant anth racycl ine use increases risk

Myocardial fibrosis, diasto l i c Years Concomitant anthracycline use i ncreases risk for heart fa ilure
dysfu nction, and restrictive
cardiomyopathy
Fibrosis of conduction pathways Years or decades
lead ing to bradycard ia,
dysrhythmias, or heart block

CAD = coronary artery disease; RV = right ventricle.

radiation-induced cardiotoxicity is warranted owing to the toward the predominant pathology, although it is recognized
chronic, lethal nature of this complication. Propensity for that concomitant abnormalities increase the surgical proce­
radiation-induced injury is related to younger age at treat­ dural risk. For example, patients undergoing pericardiec­
ment, female sex, radiation exposure (total dose, dose per frac­ tomy will be at significant operative risk owing to the
tion, and cardiac chamber affected) , the use of concomitant propensity for increased surgical bleeding and myopathy in
cardiotoxic drugs (such as anthracyclines) , and the presence of these patients. Thus, a high degree of individualization of the
cardiac risk factors (hypertension, smoking, hyperlipidemia) . treatment plan for patients with radiation-induced heart
There is no clearly defined threshold of radiation exposure for disease is recommended.
cardiotoxicity risk. In all patients with a history of significant
K E Y P O I N TS
chest radiation, aggressive management of risk factors for ath­
erosclerosis is warranted owing to the heightened risk of • I n all patients with a history o f significant chest radia­
ischemic heart disease in these patients. tion, aggressive management of risk factors for athero­
Although contemporary studies have shown a lower inci­ sclerosis is warranted owing to the heightened risk of
dence of radiation-induced cardiotoxicity, the follow-up in ischemic heart disease in these patients.
these studies has been relatively short (frequently 5 to 10 years) • Cardiotoxicity should be considered in any patient with
and thus may be inadequate to ascertain the indolent effects of a history of chest radiation who develops symptoms or
radiation-induced cardiotoxicity. Recent analyses frequently signs of cardiovascular disease.
have focused on mortality rates without detailed examinations
of other complications, such as myopathy, valvular disease,
and constrictive pericarditis. The timing and clinical methods Ca rd iotoxicity of Chemothera py
for serial monitoring have not been defined, but cardiotoxicity Cardiotoxicity from chemotherapy can result from traditional
should be considered in any patient with symptoms or signs of cytotoxic chemotherapy agents, such as anthracyclines (doxo­
cardiovascular disease and a history of chest radiation. rubicin, daunorubicin, mitoxantrone), as well as from newer
Owing to the frequent multiple cardiac pathologies in agents, such as monoclonal antibodies (trastuzumab) and
these patients, management of radiation-induced heart dis­ tyrosine kinase inhibitors. Cardiotoxicity can occur in patients
ease can be challenging. Treatment is directed primarily with normal hearts but is more common in patients with

1 06
Cardiovasc u l a r D isease in Cancer S u rvivors

preexisting cardiac disease. Cardiotoxicity from these agents occurring within 1 week after presentation. Chronic cardio­
can manifest as dilated cardiomyopathy, myocardial ischemia toxicity due to anthracyclines, which begins with a subclinical
from coronary vasospasm, or arrhythmias (Table 49) . decline in systolic and diastolic function, manifests with
The cardiotoxic effects of chemotherapy can be short­ symptoms usually within months after completion of chemo­
term, intermediate, or long-term. 5-Fluorouracil is associated therapy. However, cardiotoxicity from anthracyclines can have
with a high incidence of acute chest pain and electrocardio­ long latency periods (10 years or more) .
graphic changes (70% within 72 hours of the first treatment The strongest risk factor for cardiotoxicity related to
cycle) , resulting in death in 2% to 8% of patients affected by anthracycline agents is cumulative dose. The incidence of car­
5-fluorouracil toxicity. diotoxicity for doxorubicin or daunorubicin has been reported
Early manifestations of anthracycline toxicity are rela­ to be less than 1 % for cumulative dose of less than 400 mg/m2,
tively uncommon (3%) and include high-grade heart block, but 26% for cumulative doses of 550 mg/m2 or more. It is gen­
supraventricular and ventricular arrhythmias, heart failure, erally accepted that maximum cumulative doses for these
myocarditis, and pericarditis, with resolution in many patients drugs should be limited to 450 to 500 mg/m2 , but the doses

TABLE 49. Late-Onset Cardiotoxicity of Chemotherapeutic Agents•


Drug Cardiotoxicity

Anthracycli nes Synergistic toxicity when administered with nonanthracycline agents

Doxorubicin Heart fai l u re ( 1 . 6%-5%)


H i g her i n elderly women
Typically i rreversible

I m proved by aggressive treatment (resynchronization therapy)

Left ventricular dysfu nction ( progression slowed by sta ndard treatment)

Dilated cardio myopathy (odds ratio: 6.25b)


Cardiac death (odds ratio: 4.94b)
Daunorubicin Presu mably similar to doxoru bicin

Epirubicin Heart fa i l u re ( <5% with appropriate dosing)

Mitoxantrone Heart fa i l u re

I ncidence sign ificantly i ncreased with dose > 1 60 mg/m 2

Survival i m p roved by sta ndard hea rt fa i l u re treatment

Left ventricular dysfu nction

Alkylating agents

Cyclophosphamide Heart fa i l u re (dose dependent)

Left ventri cular dysfunction (3%-5% with dose > 1 00 mg/kg)

Cisplatin Heart fa i l u re

Mitomycin Cardiomyopathy

5-Fluorouracil Vasospasm (common) and heart fa i l u re (rare) from myocard ial i nfarctio n occurring d u ring treatment

Paclitaxel Heart failure (if given with doxoru bicin)

Trastuzumab Heart fa i l u re

0.6% (NYHA class I l l or IV)


1 . 5% (NYHA class II at 2 y)

Left ventricu lar dysfunction (3%; asymptomatic and reversib le)

Not dose dependent

lnterleukin-2 Heart fai l u re from previous cardiomyopathy, myocarditis, or myocard ial infarction occurring during
treatment (rare)

I nterferon-a Heart fai l u re from previous myocard i a l infarction during treatment (rare)

NYHA = New York Heart Association.

acardiotoxicity emerging 1 year or more after chemotherapy.

bWhen an anthracydine is added to chemotherapy regimen.

1 07
Pregna ncy a n d Cardiovascu l a r D isease

that lead to toxic responses vary considerably among individ­ for serial monitoring of patients who have undergone chemo­
ual patients. Other risk factors for cardiotoxicity include age at therapy, the thresholds for these markers as well as the appro­
treatment, concomitant therapy with other cardiotoxic agents, priate timing for their measurement remain uncertain.
chest radiation, and preexisting cardiac disease. The toxic Patients who have left ventricular dysfunction should receive
responses to anthracyclines can be modified by liposome appropriate therapy with �-blockers, vasodilators, and diuret­
encapsulation of the molecule, infusional rather than bolus ics as in patients with heart failure disorders not related to
administration, use of structural analogues (epirubicin and chemotherapy toxicity.
mitoxantrone) , and adjunctive cardioprotective agents.
KEY POINTS
Dexrazoxane is an EDTA chelator that reduces the risk of
chronic cardiotoxicity associated with doxorubicin and epiru­ • Hypertension is a potential adverse effect o f kinase
bicin and may be considered in patients being treated with _inhibitors that may require dose adjustment or, in
high anthracycline doses (>300 mg/m2) . patients with severe hypertension, discontinuation of
Cardiotoxicity due t o trastuzumab typically causes a the kinase inhibitor.
chronic, asymptomatic decline in ventricular function with a • Cardiotoxicity related to trastuzumab is not dose related
low frequency of overt heart failure (3%-7%) . Older patients and is reversible.
(age >50 years) and those with prior or concomitant exposure • Chronic cardiotoxicity with anthracyclines is dose
to anthracyclines are at increased risk of trastuzumab-induced related and is not reversible.
cardiotoxicity. In most patients, cardiotoxicity due to trastu­
• In patients who have undergone chemotherapy, base­
zumab is related to changes in contractility and is reversible.
line evaluation and routine surveillance of cardiac func­
Unlike anthracyclines, trastuzumab-related cardiotoxicity is
tion using echocardiography should be performed with
not dose related and patients can be successfully rechallenged
assessment of left ventricular ejection fraction as well
after recovery of ventricular function.
as indices of diastolic function.
Kinase inhibitors, such as tyrosine kinase inhibitors, are
a relatively new approach to tumor receptor-targeted therapy.
Hypertension is a potential adverse effect that may require
dose adjustment or, in patients with severe hypertension, dis­ Pregnancy and
continuation of the kinase inhibitor.
In adult patients being considered for chemotherapy with Ca rdiovascu lar Disease
anthracyclines, baseline evaluation of left ventricular function
should be considered before initiation of therapy, although the
Ca rd iovascu lar Changes
need for this assessment is controversial in patients with no D u ring Preg nancy
symptoms or signs of abnormal left ventricular function and Understanding the normal physiologic changes of pregnancy
in whom the cumulative dose is expected to be low (<300 mg/ is important in the interpretation of signs and symptoms in
m 2) . For patients who receive treatment with trastuzumab, a the pregnant patient (Table 50) . During a normal pregnancy,
baseline evaluation of left ventricular function should be per­ there is an increase in plasma volume and a lesser increase in
formed, particularly if there is a history of anthracycline use. erythrocyte mass, resulting in increased total blood volume
Routine surveillance of cardiac function using echocardiogra­ and relative anemia. The systemic vascular resistance
phy should be performed in all patients with assessment of left decreases during pregnancy, but the heart rate and cardiac
ventricular ejection fraction as well as indices of diastolic output rise; as a result, there is generally a slight reduction in
function. The timing intervals for these assessments are indi­ mean arterial pressure. Maternal cardiac output peaks at
vidualized based on the patient's baseline function, chemo­ approximately 40% above the prepregnancy level by the 32nd
therapeutic regimen, risk profile, and evidence of change in week of pregnancy and then plateaus until delivery. During
function in serial evaluations. delivery, heart rate and blood pressure increase, leading to a
In adults undergoing doxorubicin therapy, the drug rise in cardiac output to as much as 80% above the prepreg­
should be discontinued if there is evidence of heart failure, a nancy level.
10% or greater decline in left ventricular ejection fraction to
below the lower limit of normal, an absolute left ventricular
ejection fraction of less than 45%, or a 20% decline in left ven­ Prepreg nancy Eva l uation
tricular ejection fraction to any level. Owing to the reversible Prepregnancy evaluation i s recommended for all women with
nature of cardiotoxicity related to trastuzumab, this therapy cardiovascular disease who are anticipating pregnancy.
can be resumed after recovery of left ventricular function in Patients with congenital heart disease should consult with a
selected patients. cardiologist specializing in congenital conditions and a high­
Although other echocardiographic indices (such as strain risk obstetrician to discuss the need for genetic counseling,
imaging or volume measurements) and serum markers (car­ evaluate the risks of future pregnancy, and develop a plan for
diac troponin, B-type natriuretic peptide) have been proposed management during labor and the postpartum period.

1 08
Pregnancy a n d Ca rd i ovascu l a r D isease

TABLE 50. Normal Versus Abnormal Cardiac Symptoms and Signs in Pregnancy
Symptom or Sign Normal Pathologic
Shortness of b reath M i l d , with exerti on. Orthopnea, PND, cough
Pa lpitations Atrial and ventricu lar premature beats Atrial fi bril lation or fl utter; ventricular tachycard ia
Chest pain No Chest pressure, heaviness, or pain
M u rmur Basal systolic m u rm u r grade 1 /6 or 2/6 present i n Systo l i c m u rm u r grade '2:.3/6; any d iastolic m u rm ur
8 0 % o f pregnant women
Tachycard ia Heart rate in creased by 20%-30% Heart rate > 1 00/min
Low blood pressure Blood pressure typica l ly is modestly decreased Low blood pressure associated with symptoms
( - 1 0 m m Hg)

Edema M i l d peripheral Pulmonary edema


G a l l op S3 S4

PND = paroxysmal nocturnal dyspnea.

The CARPREG index is used to estimate risk for cardio­ KEV POI NTS
vascular complications during pregnancy in women with
• Women with severe pulmonary hypertension are at
cardiovascular disease (Table 51) . Women with severe pul­
high risk for pregnancy-related death.
monary hypertension (pulmonary artery pressure 2'. two­
thirds systemic pressure) are at high risk during pregnancy, • Systolic ventricular dysfunction (ejection fraction <40%)
with an estimated maternal mortality rate between 30% and with New York Heart Association (NYHA) functional
50%. Systolic ventricular dysfunction (ejection fraction class III or IV heart failure is considered a contraindica­
<40%) with New York Heart Association (NYHA) functional tion to pregnancy.
class Ill or IV heart failure confers a high risk of maternaI and
fetal complications and is considered a contraindication to
Management of Ca rd iovascu lar
pregnancy.
Patients with severe obstructive cardiac disease, such as Disease D u ring Pregnancy
mitral or aortic valve stenosis, are generally considered for Because pregnancy involves an increase in blood volume and
intervention before pregnancy, even if asymptomatic. cardiac output, women with severe obstructive cardiac lesions

TABLE 5 1 . Predictors of Maternal Cardiac Events in Women with Congenital or Acquired Cardiac Disease (CARPREG Index)
Risk Factor (Predictor) Operational Definition

Previous cardiac event or arrhyth m ia Heart fa i l u re, transient ischemic attack, stroke, arrhythm ia

Base l i ne NYHA functional class I l l or IV or cyanosis M i l d symptoms ( m i l d sh ortness of breath and/or angina) and sli g ht l i m itation
during ord i nary activity
2
Left-sided heart obstruction Mitra I valve area <2 cm ; aortic valve a rea < 1 .5 cm2 or resting peak left ventricular
outflow tract g ra d ient >30 m m Hg

Reduced system ic ventricular systolic function Ejection fraction <40%

Estimated Risk for Cardiac Events•


No. of Predictors Estimated Risk (%) Recommendation

0 4 Consider preconception cardiac i ntervention for specific lesions; increase


frequency of follow-up; del ivery at com m u n ity hospital

1 31 Co nsider preconception cardiac i ntervention for specific lesions; refer to regional


center for ongoing care
>1 69 Consider preconception cardiac intervention for specific lesions; refer to reg ional
center for ongoing care

NYHA = New York Heart Association.

3Pulmonary edema, tachyarrhythmia, embolic stroke, cardiac death.

Data and recommendations from Siu SC, Sermer M, Colman JM, et al. Cardiac Disease in Pregnancy (CARPREG) Investigators. Prospective multicenter study of pregnancy
outcomes in women with heart disease. Circulation. 200 1 ; 1 04(5): 5 1 5-52 1 . [PMID: 1 1 4792461

1 09
Preg nancy a n d Cardiovascu l a r Disease

generally develop symptoms during pregnancy, whereas function, which can result in clinical deterioration or even
women with regurgitant valve lesions tolerate pregnancy rea­ death.
sonably well.
Vaginal delivery is generally preferred for patients with Other Cardiovascular Disorders
cardiovascular disease because it results in a shorter and less Women with Marfan syndrome have been reported to have an
marked hemodynamic derangement than cesarean delivery. increased risk of aortic dissection during pregnancy. Women
To reduce the risk of fetal intracranial hemorrhage, cesarean with Marfan syndrome and an ascending aortic diameter of
delivery is recommended in women receiving warfarin antico­ 4.5 cm or greater are recommended to have aortic repair sur­
agulation therapy. Cesarean delivery is also recommended for gery before considering pregnancy to reduce this risk. Some
obstetric reasons and in select patients with severe pulmonary women with Marfan syndrome and aortic diameter less than
hypertension or a markedly dilated aorta. 4.5 cm are at high risk for dissection during pregnancy and are
counseled to have aortic valve replacement before pregnancy;
Peripartum Cardiomyopathy these include patients with rapid dilatation of the ascending
Left ventricular systolic dysfunction identified toward the end of aorta or a family history of aortic dissection.
pregnancy or in the months following delivery in the absence of Spontaneous coronary artery dissections in women with­
another identifiable cause is known as peripartum cardiomyo­ out risk factors for coronary artery disease may occur in the
pathy. This occurs with increased frequency in women who are peripartum setting. Spontaneous healing may occur with con­
multiparous, older (age >30 years), and black; in those with servative medical therapy, but revascularization, either perCL1-
multifetal pregnancy, gestational hypertension, or preeclamp­ taneous or bypass surgery, has been utilized as well.
sia; and in those treated with tocolytic agents.
The leading cause of pregnancy-related maternal death Cardiovascular Medication Use
in North America is peripartum cardiomyopathy. Death in During Pregnancy
women with peripartum cardiomyopathy results from Limited data are available on the safety of cardiovascular
heart failure, thromboembolic events, and arrhythmias. medications administered during pregnancy. The FDA catego­
Half of women who develop peripartum cardiomyopathy rizes drugs by their fetal effects during pregnancy (see MKSAP
show improvement in left ventricular function within 6 17 General Internal Medicine, Women's Health). Most cardio­
months of delivery, and 20% to 40% have normalization of vascular drugs are not FDA-approved for use during preg­
ventricular function. nancy. General guidelines for the use of several cardiovascular
Prompt initiation of medical therapy is recommended for drugs during pregnancy are provided in Table 52.
women with peripartum cardiomyopathy and includes Cardiovascular medications should be used only when needed
P-blockers, digoxin, hydralazine, nitrates, and diuretics. ACE and at the lowest possible dose, and the desired therapeutic
inhibitors, angiotensin receptor blockers, and aldosterone effect should outweigh the risk.
antagonists should be avoided until after delivery owing to When P-blockers are used during pregnancy or lactation,
teratogenicity. Anticoagulation with warfarin is recommended periodic fetal and newborn heart rate monitoring and initial
for women with peripartum cardiomyopathy with left ven­ newborn blood glucose assessment are indicated because
tricular ejection fraction below 35%, owing to the increased P-blockers cross the placenta and are present in human breast
risk ofthromboembolism related to this disorder; the duration milk. Atenolol is usually avoided during pregnancy because it
of anticoagulation is individualized, and anticoagulation is has been reported to cause small fetal gestational size, early
discontinued when the ejection fraction improves. delivery, and low birth weight. Patients taking atenolol are
In women with acute severe periparturn cardiornyopa­ usually transitioned to a different P-blocker.
thy, brornocriptine, which blocks prolactin secretion, has The treatment of choice for acute symptomatic supraven­
been shown to improve left ventricular ejection fraction and tricular tachycardia during pregnancy is adenosine. Recurrent
clinical outcomes when added to periparturn-related heart tachycardia symptoms are often treated with P-blockers and
failure therapy. Bromocriptine inhibits lactation and may digoxin; sotalol and flecainide have also been safely used.
increase risk of thromboembolism; therefore, anticoagula­ Amiodarone is rarely used owing to toxicity concerns.
tion is suggested in conjunction with brornocriptine. For Spironolactone is considered compatible with breast­
these reasons, discussion with the patient is an important feeding; although spironolactone and its active metabolite,
precursor to initiating therapy. Referral for ventricular assist canrenone, appear in breast milk, the concentrations are
device or heart transplantation should be considered for pharmacologically insignificant. ACE inhibitors, angiotensin
women with refractory severe heart failure related to peri­ receptor blockers, and aldosterone antagonists are terato­
partum cardiomyopathy. genic and should be avoided during pregnancy. Some ACE
Women with peripartum cardiomyopathy with persistent inhibitors are safe to use while breastfeeding. Angiotensin
left ventricular dysfunction should be counseled to avoid sub­ receptor blockers are generally avoided during lactation
sequent pregnancy since another pregnancy is often associ­ because data are inconclusive regarding infant risk when
ated with recurrent or further reduction of left ventricular used during breastfeeding.

110
Pregna ncy a nd Cardiovascular D isease

TABLE 52. Drugs for Cardiac Disorders in Pregnancy


Drug Use in Compatibility with Comments
Pregnancy Breastfeeding

ACE i n h i b itors

Captopril, enalapril No Yes Teratogenic in first trimester; cause feta l/neonatal kidney
fai l u re with second or third trimester exposure; scleroderma
renal crisis is only ind ication
Lisinopril No ? Same as above
ARBs No ? Teratogenic i n first trim ester; cause feta l/neonatal kid ney
fai l ure with second or third tri mester exposure
Adenosine Yes ? No change in fetal heart rate when used for su praventricu lar
tachycard ia
Amiodarone No No Fetal hypothyro idism, prematurity
Anti platelet agents

D i pyridamole, Yes ? Second-line a gent; no evidence of harm in a n i m a l


clopidogrel clopidogrel stud i es; no human data

Aspirin (:581 m g ) Yes Yes


f3-Bl ockers
Atenolol Yes No Second-line agent; low birth weight, intrauterine g rowth
restriction
Esmolol Yes ? Second-line agent; more pronounced bradycardia
Labetalol Yes Yes Preferred drug i n class

Metoprolol Yes Yes Shortened ha lf-life

Propranolol Yes Yes Second-line agent; i ntrauterine growth restriction

Sotalol Yes ? Second-line agent; insufficient data; reserve use for


arrhyth mia not responding to alternative agent

Ca lcium channel b lockers

Diltiazem, verapamil Yes Yes Second-line agent; maternal hypotension with rapid
intravenous infusion; used for fetal supraventricu lar
tachycardia

Digoxin Yes Yes Second-line agent; shortened half- l ife

Disopyramide Yes Yes Second-line agent; case reports of preterm labor

Diuretics Yes Yes Second-line agent; use when needed for maternal volume
overload only

Flecainide Yes ? Second-line agent; i nadeq uate data but used for feta l
arrhyth mia; case report of fetal hyperbilirubinemia

Hydralazine Yes Yes Vasodilator of choice

Lidocaine Yes Yes Treatment of choice for ventricu lar a rrhyth mias

Sodium nitroprusside No No Potential fetal thiocyanate toxicity

O rganic nitrates Yes ? No apparent i ncreased risk

Phenytoi n No Yes Known te ratogenicity and bleeding risk; last resort for
arrhyth mia

Procainamide Yes Yes Used for fetal arrhyth mia

Propafenone Yes ? Second-line age nt; used for fetal arrhyth mia

Quinidine Yes Yes Preferred d rug i n class; i ncreases d igoxin levels

ARB = angiotensin receptor blocker; ? = unknown.

Adapted from Rosene-Montella K, Keely EJ, Lee RV, Barbour LA. Medical Care of the Pregnant Patient. 2nd Edition. Philadelphia, PA: American College of Physicians; 2008.
p 356-357. Copyright 2008, American College of Physicians.

111
Pre g n a ncy a n d Cardiovascu l a r Disease

Anticoagu lation Therapy During Pregnancy adjustment are recommended for all anticoagulation regi­
Pregnancy is a hypercoagulable state, and anticoagulation is mens. Warfarin is stopped before delivery owing to the risk
often indicated during pregnancy; regimens and levels of anti­ of fetal intracranial hemorrhage if spontaneous labor occurs
coagulation depend on the specific indication (Table 53) . while the mother, and thus the fetus, is anticoagulated with
Prepregnancy counseling is recommended for all women warfarin.
receiving chronic warfarin anticoagulation to help them Women with mechanical valve prostheses represent a
understand maternal and fetal risk and to make an informed high-risk subset of patients during pregnancy, with excess risk
decision regarding which anticoagulation regimen to use dur­ of valve thrombosis, bleeding, and fetal morbidity and mortal­
ing their pregnancy. ity. The optimal anticoagulation regimen for this patient group
Unfractionated heparin, low-molecular-weight hepa­ has not been established. Warfarin anticoagulation during
rin (LMWH) , and warfarin can all be used for anticoagula­ pregnancy may be the safest agent for prevention of maternal
tion during pregnancy. Meticulous monitoring and dose prosthetic valve thrombosis; however, warfarin poses an

TABLE 53. Anticoagulation Regimens During Pregnancy


Weeks of Gestation Recommended Regimen
Venous Thromboembolism
Weeks 6-1 2 Warfarin (if dose to attai n I N R 2-3 is S:S mg)

U F H (IV or SQ; a PTT 2 x control)

Weight-based LMWH

Weeks 1 3-37 U F H (SQ; a PTT 2 x control)

Weight-based LMWH
Warfarin ( I N R 2-3)

Weeks 3 7 to term UFH ( IV; aPTT 2 x control)

Atrial Fibrillation

Weeks 6-1 2 Warfarin (if dose to atta i n I N R 2-3 is S:S mg)

U F H (IV or SQ; a PTT 2 x contro l )

Weig ht-based LMWH

Weeks 1 3-37 UFH (SQ; a PTT 2 x control)

Weight-based LMWH

Warfarin ( I N R 2-3)

Weeks 37 to term UFH ( IV; a PTT 2 x control)

Mechanical Valve Prosthesis

Weeks 6-1 2 Warfarin dose S:S mg for therapeutic I N R

Continue warfarin (class I l a )

U F H : IV; a P TT 2 x control (class l i b )

Anti-factor X a adj usted LMWH (class l i b )

Warfarin dose >5 mg for therapeutic I N R

U F H : IV; a P TT 2 x control (class I l a )


Anti-factor X a a dj u sted LMWH (class I l a )
Weeks 1 3-37 Warfa rin (therapeutic INR)
Weeks 3 7 to term UFH (IV; a PTT 2 x control)

aPTI = activated partial thromboplastin time; IV= intravenous; LMWH = low-molecular-weight heparin; SQ = subcutaneous; UFH = unfractionated heparin.

Recommendations from Nishimura RA, Otto CM, Bonow RO, et al; American College of Cardiology/American Heart Association Task Force on Practice Guidelines . 2 0 1 4 AHA/ACC
guideline for the management of patients with valvular heart disease: executive summary: a report of the American College of Cardiology/American Heart Association Task Force
on Practice Guidelines. J Am Coll Cardiol. 2 0 1 4 Jun 1 0;63(22):2438·88. Erratum in: J Am Coll Cardiol. 20 1 4 Jun 1 0;63(22):2489. ( P M I D : 24603 1 92] and Bates SM, Greer IA,
Middeldorp S, Veenstra DL, Prabulos AM, Vandvik PO; American College of Chest Physicians. VTE, thrombophilia, antithrombotic therapy, and pregnancy: Antithrombotic Therapy
and Prevention of Thrombosis (9th edition): American College of Chest Physicians Evidence-Based Clinical Practice Guidelines. Chest. 20 1 2 Feb; 1 4 1 (2 Suppl):e691 S-7365. [ P M I D :
2231 5276] a n d Furie K L , Kasner SE, Adams RJ, e t a l ; American Heart Association Stroke Council, Council on Cardiovascular Nursing, Council on Clinical Cardiology. and
Interdisciplinary Council on Quality of Care and Outcomes Research. Guidelines for the prevention of stroke in patients with stroke or transient ischemic attack: a guideline for
healthcare professionals from the American Heart Association/American Stroke Association. Stroke. 201 1 Jan;42(1 ):227-276. ( P M I D : 2096642 1 }

112
B i b l i og ra phy

increased fetal risk, with possible teratogenicity, miscarriage, Diagnostic Testing in Cardiology

and fetal loss due to intracranial hemorrhage. Data suggest that American College of Cardiology Foundation Task Force on Expert Consensus
Documents. Hundley WG, Bluemke DA, Finn JP, et al. ACCF/ACR/AHA/
LMWH and unfractionated heparin are safer for the fetus than NASCl/SCMR 2010 expert consensus document on cardiovascular mag­
warfarin, but these therapies appear to increase the risk of netic resonance: a report of the American College of Cardiology Foundation
Task Force on Expert Consensus Documents. J Am Coll Cardiol. 2010 Jun
maternal prosthetic valve thrombosis. 8;55(23):2614-62. [PMID: 20513610]
Guidelines from the American College of Cardiology/ American College of Cardiology Foundation Task Force on Expert Consensus
American Heart Association on the management of anticoagu­ Documents. Mark DB. Berman DS. Budoff MJ, et al. ACCF/ACR/AHA/
NASCl/SAlP/SCAl/SCCT 2010 expert consensus document on coronary
lation during pregnancy conclude that intravenous unfrac­ computed tomographic angiography: a report of the American College of
tionated heparin, LMWH, or warfarin may be used for Cardiology Foundation Task Force on Expert Consensus Documents. J Am
Coll Cardiol. 2010 Jun 8;55(23):2663-99. [PMID: 20513611]
anticoagulation of pregnant women with mechanical heart American College of Cardiology Foundation Appropriate Use Criteria Task
valves. Intravenous unfractionated heparin is the drug of Force; American Society ofEchocardiography; American Heart Association;
American Society of Nuclear Cardiology; Heart Failure Society of America;
choice for patients with mechanical valve prostheses around Heart Rhythm Society: Society for Cardiovascular Angiography and
the time of delivery. Interventions: Society of Critical Care Medicine: Society of Cardiovascular
Computed Tomography: Society for Cardiovascular Magnetic Resonance,
K EY P O I NT S Douglas PS, Garcia MJ, Haines DE, et al. ACCF/ASE/AHA/ASNC/HFSA/
HRS/SCAl/SCCM/SCCT/SCv!R 2 0 1 1 Appropriate Use Criteria for
• Women with Marfan syndrome and an ascending aortic Echocardiography. A Report of the American College of Cardiology
Foundation Appropriate Use Criteria Task Force, American Society of
diameter of 4.5 cm or greater are recommended to have Echocardiography, American Heart Association, American Society of
aortic repair surgery before considering pregnancy to Nuclear Cardiology. Heart Failure Society of America, Heart Rhythm
Society, Society for Cardiovascular Angiography and Interventions, Society
reduce the risk of aortic dissection. of Critical Care Medicine, Society of Cardiovascular Computed Tomography,
• ACE inhibitors, angiotensin receptor blockers, and and Society for Cardiovascular Magnetic Resonance Endorsed by the
American College of Chest Physicians. J Am Coll Cardiol. 2011 Mar
aldosterone antagonists are teratogenic and should be 1;57(9):1126-66. [PMID: 21349406)
avoided during pregnancy. Fihn SD, Blankenship JC, Alexander KP, et al. 2014 ACC/AHAiAATSiPCNA/
SCAl/STS Focused Update of the Guideline for the Diagnosis and
• Unfractionated heparin, low-molecular-weight heparin, Management of Patients With Stable lschemic Heart Disease: A Report of
or warfarin may be used for anticoagulation of pregnant the American College of Cardiology/American Heart Association Task
Force on Practice Guidelines. and the American Association for Thoracic
women with mechanical heart valves; unfractionated Surgery, Preventive Cardiovascular Nurses Association, Society for
heparin is the drug of choice for patients with mechani­ Cardiovascular Angiography and Interventions, and Society of Thoracic
Surgeons. Circulation. 2014 Nov 4:130(19):1749-67. [PMID: 25070666]
cal valve prostheses around the time of delivery. Greenland P, Alpert JS, Beller GA, et al; American College of Cardiology
Foundation; American Heart Association. 2010 ACCF/AHA guideline for
assessment of cardiovascular risk in asymptomatic adults: a report of the
Bibliogra phy American College of Cardiology Foundation/American Heart Association
Task Force on Practice Guidelines. J Am Coll Cardiol. 2010 Dec
Epidemiology and Risk Factors 14:56(25):e50-103. [PMID: 21144964)
Dhawan SS. Quyyumi AA. Rheumatoid arthritis and cardiovascular disease. Greenland P, Bonow RO, Brundage BH, et al; American College of Cardiology
Curr Atheroscler Rep. 2008 Apr;l0(2) :128-33. [PMID: 18417067] Foundation Clinical Expert Consensus Task Force (ACCF/AH A Writing
Go AS. Mozaffarian D, Roger \IL, et al: American Heart Association Statistics Committee to Update the 2000 Expert Consensus Document on Electron
Committee and Stroke Statistics Subcommittee. Heart disease and stroke Beam Computed Tomography) ; Society of Atherosclerosis Imaging and
statistics--2013 update: a report from the American Heart Association. Prevention; Society of Cardiovascular Computed Tomography. ACCF/ AHA
Circulation. 2013 Jan 1 :127(1):e6-e245. Erratum in: Circulation. 2013 Jan 1 ; 127 2007 clinical expert consensus document on coronary artery calcium scor­
(l). Erratum in: Circulation. 2013 Jun 11;127(23):e84l. [PMID: 23239837] ing by computed tomography in global cardiovascular risk assessment and
in evaluation of patients with chest pain: a report of the American College
Greenland P, Alpert JS, Beller GA, et al; American College of Cardiology of Cardiology Foundation Clinical Expert Consensus Task Force (ACCF/
Foundation; American Heart Association. 2010 ACCF/AHA guideline for AHA Writing Committee to Update the 2000 Expert Consensus Document
assessment of cardiovascular risk in asymptomatic adults: a report of the on Electron Beam Computed Tomography) developed in collaboration with
American College of Cardiology Foundation/ American Heart Association the Society of Atherosclerosis I maging and Prevention and the Society of
Task Force on Practice Guidelines. J Am Coll Cardiol. 2010 Dec Cardiovascular Computed Tomography. J Am Coll Cardiol. 2007 Jan
14;56(25) :e50-103. [PMID: 21144964] 23;49(3):378-402. [PM I D : 17239724]
Pignone M , Alberts MJ, Colwell JA, et al. Aspirin for primary prevention of Hendel RC, Berman DS, Di Carli M F. et al; American College of Cardiology
cardiovascular events in people with diabetes: a position statement of the Foundation Appropriate Use Criteria Task Force; American Society of
American Diabetes Association, a scientific statement of the American Nuclear Cardiology; American College of Radiology; American Heart
Heart Association, and an expert consensus document of the American Association: American Society of Echocardiology; Society of Cardiovascular
College of Cardiology Foundation. Circulation. 2010 Jun 22:121 (24):2694- Computed Tomography; Society for Cardiovascular Magnetic Resonance;
701. [PM I D: 20508178] Society of Nuclear Medicine. ACCF/ASNC/ACR/AHA/ASE/SCCT/SCMR/
Schoenfeld SR. Kasturi S, Costenbaker KH. The epidemiology of atheroscle­ SNM 2009 Appropriate Use Criteria for Cardiac Radionuclide Imaging: A
rotic cardiovascular disease among patients with SLE: a systematic review. Report of the American College of Cardiology Foundation Appropriate Use
Semin Arthritis Rheum. 2013 Aug;43(1) :77-95. [PM I D : 23422269] Criteria Task Force, the American Society of Nuclear Cardiology,
the American College of Radiology. the American Heart Association, the
U.S. Preventive Services Task Force. Aspirin for the Prevention of
Cardiovascular Disease. vvww.uspreventiveservicestaskforce.org/uspstf/ American Society of Echocardiography. the Society of Cardiovascular
uspsasmi.htm. Updated October 2013. Accessed October l, 2014.
Computed Tomography, the Society for Cardiovascular Magnetic
Resonance, and the Society of Nuclear Medicine. J Am Coll Cardiol. 2009
Yusuf S, Hawken S, Ounpuu S. et al; lNTERHEART Study Investigators. Effect Jun 9:53(23):2201-29. [PMID: 1 9497454]
of potentially modifiable risk factors associated with myocardial infarction
in 52 countries (the INTERHEA RT study): case-control study. Lancet. 2004 Coronary Artery Disease
Sep 11-17:364(9438):937-52. [PMID: 15364185] Amsterdam EA, Wenger NK. Blind is RG, et al: 2014 AHA/ACC Guideline for the
Yusuf S, Reddy S, Ounpuu S, et al. Global burden of cardiovascular diseases. Management of Patients With Non-ST-Elevation Acute Coronary
Part I I : Variations in cardiovascular disease by specific ethnic groups and Syndromes: Executive Summary: A Report of the American College of
geographic regions and prevention strategies. Circulation. 2001 Dec Cardiology/American Heart Association Task Force on Practice Guidelines.
4;104(23):2855-64. [PMID: ll733407] Circulation. 2014 Sep 23. [PMID: 25249586]

113
B i b l i o g ra p h y

Antithrombotic Trialists' (ATT) Collaboration, Baigent C , Blackwell L , Collins Heart Failure


R, et al. Aspirin in the primary and secondary prevention of vascular dis­ Felker GM, Lee KL, Bull DA. et al: NHLBI Heart Failure Clinical Research
ease: collaborative meta-analysis of individual participant data from ran­ Network. Diuretic strategies in patients with acute decompensated heart
domised trials. Lancet. 2009 May 30;373(9678):1849-60. [PMID: 19482214] failure. N Engl J Med. 2011 Mar 3;364(9) :797-805. [PMID: 21366472]
BARI 20 Study Group, Frye RL, August P, Brooks MM, et al. A randomized trial Heart Failure Society of America, Lindenfeld J, Albert NM, Boehmer JP, et al.
of therapies for type 2 diabetes and coronary artery disease. N Engl J Med. HFSA 2010 Comprehensive Heart Failure Practice Guideline. J Card Fail.
2009 Jun 1 1 ; 360(24):2503-15. [PM!D: 19502645] 2010 Jun:l6(6):el-194. [PMID: 20610207]
Farkouh ME, Domanski M, Sleeper LA, et al; FREEDOM Trial Investigators.
Mehra MR, Kobashigawa J, Starling R, et al. Listing criteria for heart transplan­
Strategies for multivessel revascularization in patients with diabetes. N
tation: International Society for Heart and Lung Transplantation guidelines
Engl J Med. 2012 Dec 20;367(25):2375-84. [PMID: 23121323]
for the care ofcardiac transplant candidates-2006. J Heart Lung Transplant.
Giugliano RP, White JA, Bode C. et al; EARLY ACS Investigators. Early versus 2006 Sep:25(9):1024-42. [PM!D: 16962464]
delayed, provisional eptiflbatide in acute coronary syndromes. N Engl J Russo AM, Stainback RF, Bailey SR. et al. ACCF/HRS/AHA/ASE/H FSA/SCAI/
Med. 2009 May 2 1;360(21) :2176-90. [PMID: 19332455]
SCCT/SCMR 2013 appropriate use criteria for implantable cardioverter­
Gulati M. Cooper-DeHoff RM, McClure C. et al. Adverse cardiovascular out­ deflbrillators and cardiac resynchronization t herapy: a report of the
comes in women with nonobstructive coronary artery disease: a report American College of Cardiology Foundation appropriate use criteria task
from the Women's Ischemia Syndrome Evaluation Study and the St James force, Heart Rhythm Society, American Heart Association, American
Women Take Heart Project. Arch Intern Med. 2009 May 11: 169(9):843-50. Society of Echocardiography, Heart Failure Society of America, Society for
[PM!D: 19433695] Cardiovascular Angiography and Interventions, Society of Cardiovascular
Kaul S, Bolger AF, Herrington D. et al: American Heart Association: American Computed Tomography, and Society for Cardiovascular Magnetic
College of Cardiology Foundation. Thiazolidinedione drugs and cardiovas­ Resonance. J Am Coll Cardiol. 2013 Mar 26:61(12):1318-68. [PM!D:
cular risks: a science advisory from the American Heart Association and 23453819]
the American College of Cardiology Foundation. J Am Coll Cardiol. 2010 Yancy CW, Jessup M. Bozkurt B, et al: American College of Cardiology
Apr 27;55(17) :1885-94. [PMID: 20413044] Foundation: American Heart Association Task Force on Practice Guidelines.
Mehta SR, Granger CB, Boden WE. et al: TIMACS Investigators. Early versus 2013 ACCF/AHA guideline for the management of heart failure: a report of
delayed invasive intervention in acute coronary syndromes. N Engl J Med. the American College of Cardiology Foundation/ American Heart
2009 May 21:360 (21) :2165-75. [PMID: 1 9458363] Association Task Force on Practice Guidelines. J Am Coll Cardiol. 2013 Oct
15;62(16):el47-239. [PMID: 23747642]
O'Gara PT, Kushner FG, Ascheim DD, et al: American College of Cardiology
Foundation/American Heart Association Task Force on Practice Guidelines. Myocardial Disease
2013 ACCF/AHA guideline for the management of ST-elevation myocardial
Ammash NM. Seward JB, Bailey KR, Edwards WO, Taji k AJ. Clinical proflle and
infarction: a report of the American College of Cardiology Foundation/
outcome of idiopathic restrictive cardiomyopathy. Circulation. 2000 May
American Heart Association Task Force on Practice Guidelines. Circulation.
30;101 (21):2490-6. [PMID: 10831523]
2013 Jan 29:127(4) :e362-425. Erratum in: Circulation. 2013 Dec
24:128(25):e481. [PMID: 23247304] Gersh BJ, Maron BJ, Bonow RO, et al. 2011 ACCF/AHA guideline for the diag­
nosis and treatment of hypertrophic cardiomyopathy: executive summary:
Patel MR, Dehmer GJ, Hirshfeld JW, et al; ACCF/SCAJ/STS/AATS/AHA/ASNC
a report of the American College of Cardiology Foundation/American Heart
2009 Appropriateness Criteria for Coronary Revascularization: A Report of
Association Task Force on Practice Guidelines. J Am Coll Cardiol. 2011 Dec
the American College of Cardiology Foundation Appropriateness Criteria
13;58(25) :2703-38. [PMID: 22075468]
Task Force, Society for Cardiovascular Angiography and Interventions,
Society of Thoracic Surgeons, American Association for Thoracic Surgery, Lee VH, Connolly HM, Brown RD Jr. Central nervous system manifestations of
American Heart Association, and the American Society of Nuclear cardiac myxoma. Arch Neurol. 2007 Aug;64(8) :lll5-20. [PM!D: 17698701]
Cardiology: Endorsed by the American Society of Echocardiography, the Maron BJ. Hypertrophic cardiomyopathy and other causes of sudden cardiac
Heart Failure Society of America, and the Society of Cardiovascular death in young competitive athletes. with considerations for preparticipa­
Computed Tomography. Circulation. 2009 Mar 1 0 ; 119(9) :1330-52. Erratum tion screening and criteria for disqualification. Cardiol Clin. 2007
in: Circulation. 2009 Apr 21:119(15) :e488. [PMID: 19131581] Aug;25 (3):399-414, vi. [PMID: 17961794]
Qaseem A, Fihn SD. Williams S, et al; Clinical Guidelines Committee of the Maron BJ, Casey SA. Poliac LC. Gollman TE, Almquist AK. Aeppli DM. Clinical
American College of Physicians. Diagnosis of stable ischemic heart disease: course of hypertrophic cardiomyopathy in a regional United States cohort.
summary of a clinical practice guideline from the American College of JAMA. 1999 Feb 17:281(7):650-5. Erratum in: JAMA 1999 Jun 23-30:
Physicians/American College of Cardiology Foundation/American Heart 281(24):2288. [PM!D: 10029128]
Association/ American Association for Thoracic Surgery / Preventive
Cardiovascular Nurses Association/Society of Thoracic Surgeons. Ann Maron BJ, Maron MS. Hypertrophjc cardiomyopathy. Lancet. 2013 Jan 19;
Intern Med. 2012 Nov 20:157(10) :729-34. [PMID: 23165664] 381 (9862):242-55. [PMID: 22874472]
Maron BJ, Spirito P, Shen WK, et al. Implantable carilloverter-deflbrillators and
Qaseem A, Fihn SD, Dallas P, et al; Clinical Guidelines Committee of the
American College of Physicians. Management of stable ischemic heart dis­ prevention of sudden cardiac death in hypertrophic cardiomyopathy.
ease: summary of a clinical practice guideline from the American College JAMA. 2007 Jul 25;298(4):405-12. Erratum in: JAMA. 2007 Oct
of Physicians/American College of Cardiology Foundation/ American Heart 3;298(13):1516. [PMID: 17652294]
Association/American Association for Thoracic Surgery/Preventive Sorajja P, Ommen SR, Holmes DR Jr, et al. Survival after alcohol septa! ablation
Cardiovascular Nurses Association/Society of Thoracic Surgeons. Ann for obstructive hypertrophi<;: cardiomyopathy. Circulation. 2012 Nov
Intern Med. 2012 Nov 20;157(10):735-43. [PMID: 23165665] 13;126(20):2374-80. [PMID: 23076968]
Serruys PW, Morice MC, Kappetein AP, et al; SYNTAX Investigators. Sorajja P. Invasive hemodynamics of constrictive pericarditis, restrictive car­
Percutaneous coronary intervention versus coronary-artery bypass graft­ diomyopathy, and cardiac tamponade. Cardiol Clin. 2011 May;29(2) :19l-9.
ing for severe coronary artery disease. N Engl J Med. 2009 Mar [PMID: 21459242]
5:360(10):961-72. Erratum in: N Engl J Med. 2013 Feb 7;368(6):584 [PMID: Truong PT, Jones SO, Martens B, et al. Treatment and outcomes in adult
19228612] patients with primary cardiac sarcoma: the British Columbia Cancer
Shaw U, Bugiardini R. Merz CN. Women and ischemic heart disease: evolving Agency experience. Ann Surg Oneal. 2009 Dec;l6(12):3358-65. [PMID:
knowledge. J Am Coll Cardiol. 2009 Oct 20:54(17):1561-75. [PM!D: 1 9833255] 19830494]
Sobel BE. Coronary revascularization in patients with type 2 diabetes and Arrhythmias
results of the BARI 20 trial. Coron Artery Dis. 2010 May:2l (3):189-98.
[PM!D: 20308880] Baddour LM, Epstein AE, Erickson CC. et al: American Heart Association
Rheumatic Fever, Endocarditis, and Kawasaki Disease Committee: Council
Stone GW, Witzenbichler B, Guagliumi G, et al: HORIZONS-AMI Trial on Cardiovascular Disease in Young; Council on Cardiovascular Surgery
Investigators. Bivalirudin during primary PCI in acute myocardial infarc­ and Anesthesia; Council on Cardiovascular Nursing; Council on Clinical
tion. N Engl J Med. 2008 May 22:358:2218-30. [PMID: 18499566] Cardiology; Interdisciplinary Council on Quality of Care; American Heart
Wallentin L. Becker RC. Budaj A, et al; PLATO Investigators, Freij A, Thorsen M . Association. Update on cardiovascular implantable electronic device infec­
Ticagrelor versus clopidogrel in patients with acute coronary syndromes. tions and their management: a scientific statement from the American
N Engl J Med. 2009 Sep 10;361:1045-57. [PMID: 19717846] Heart Association. Circulation. 2010 Jan 26;121(3):458-77. [PMID: 20048212]

Wiviott SD, Braunwald E, McCabe CH. et al; TRITON-TI M I 38 Investigators. Conen D, Adam M, Roche F, et al. Premature atrial contractions in the general
Prasugrel versus clopidogrel in patients with acute coronary syndromes. population: frequency and risk factors. Circulation. 2012 Nov
N Engl J Med. 2007 Nov 15: 357(20):2001 -15. [PMID: 17982182] 6 : 126(19) :2302-8. [PMID: 23048073]

1 14
B i b l i og ra p h y

ConnoUy SJ, Camm AJ, Halperin JL, et al; PALLAS Investigators. Dronedarone in Spodick DH. Acute cardiac tarnponade. N Engl J Med. 2003 Aug 14;349(7):684-
high-risk pem1anent atrial fibriUation. N Engl J Med. 2011 Dec15;365(24):2268- 90. [PM ID: 12917306]
76. Erratum in: N Engl J Med. 2012 Feb 16;366(7):672. [PMID: 22082198]
Syed FE Ntsekhe M, Mayosi BM. Oh JK. Effusive-constrictive pericarditis.
ConnoLiy SJ, Eikelboom J, Joyner C, et al; AVERROES Steering Committee and Heart Fail Rev. 2013 May:I8(3):277-87. [PMID: 22422296]
I nvestigators. Apixaban in patients with atrial fibrillation. N Engl J Med.
Talreja DR. Nishimura RA. Oh JK. Holmes DR. Constrictive pericarditis in the
2011 Mar 3:364(9):806-17. [PM!D: 2 1309657]
modern era: novel criteria for diagnosis in the cardiac catheterization labo­
Connolly SJ, Ezekowitz MD, Yusuf S, et al; RE-LY Steering Committee and rato1y. J Am Coll Cardiol. 2008 Jan 22:51(3):315-9. [PMID: 18206742]
I nvestigators. Dabigatran versus warfarin in patients with atrial fibrillation.
Tsang TS. l'reeman WK. Sinak U, Seward JB. Echocardiographically guided
N Engl J Med. 2009 Sep 17:361(12):1139-51. Erratum in: N Engl J Med. 20!0
pericardiocentesis: evolution and state-of-the-art technique. Mayo Clin
Nov 4;363(19):1877. [PMID: 19717844]
Proc. 1998 Jul:73(7):647-52. [PMID: 9663193]
Epstein AE. DiMarco JP, Ellenbogen KA, et al. 2012 ACCF/AHA/HRS focused
update incorporated into the ACCF/AHA/HRS 2008 guidelines for device­ Valvular Heart Disease
based therapy of cardiac rhythm abnormalities: a report of the American David TE. Surgical treatment of aortic valve disease. Nat Rev Cardiol. 2013
College of Cardiology Foundation/American Heart Association Task Force Jul:l0(7):375-86. [PMID: 23670613]
on Practice Guidelines and the Heart Rhythm Society. Circulation. 2013 Jan
22;127(3):e283-352. [PMID: 23255456] Joint Task force on the Management of Valvular Heart Disease of the European
Society of Cardiology (ESC): European Association for Cardio-Thoracic
Friberg L. Rosenqvist M. Lip GY. Evaluation of risk stratification schemes for Surgery (EACTS). Vahanian A. Alfieri 0, Andreotti F. et al. Guidelines on the
ischaemic stroke and bleeding in 182 678 patients with atrial fibrillation: management of valvular heart disease (version 2012). Eur Heart J. 2012
the Swedish Atrial Fibrillation cohort study. Eur Heart J. 2012 Oct:33(19):2451 -96. [PMID: 22922415]
Jun;33(12):1500-IO. [PM ! D : 22246443]
Kang DH, Kim YJ. Kim SH, et al. Early surgery versus conventional treatment
Fuster V. Ryden LE. Cannom DS, et al; American College of Cardiology: for infective endocarditis. N Engl J Med. 2012 Jun 28;366(26):2466-73.
American Heart Association Task Force; European Society of Cardiology [PMID: 22738096]
Committee for Practice Guidelines; European Heart Rhythm Association:
Heart Rhythm Society. ACC/AHA/ESC 2006 guidelines for the management Leong DP, Joseph MX, Selvanayagam JB. The evolving role of multimodality
of patients with atrial fibrillation: full text: a report oft he American College imaging in valvular heart disease. Heart. 2014 l'eb;J00(4):336-46. [PM I D :
of Cardiology/American Heart Association Task Force on practice guide­ 23574967]
lines and the European Society of Cardiology Committee for Practice Lindman BR. Bonow RO. Otto CM. Current management of calcillc aortic
Guidelines (Writing Committee to Revise the 2001 guidelines for the man­ stenosis. Circ Res. 2013 Jul 5:113(2):223-37. [PMID: 23833296]
agement of patients with atrial fibrillation) developed in collaboration with
Mordi I, Tzemos N. Bicuspid aortic valve disease: a comprehensive review.
the European Heart Rhythm Association and the Heart Rhythm Society. Cardiol Res Pract. 2012:2012:1 96037. [PMID: 22685681]
Europace. 2006 Sep:8(9):651-745. Erratum in: Europace. 2007 Sep:9(9):856.
[PMID: 16987906] Nishimura RA, Otto CM, Bonow RO, et al; American Col lege of Cardiology/
American Heart Association Task force on Practice Guidelines. 2014 AHA/
Goldschlager N. Epstein AE, Naccarelli G. et al. Practical guidelines for clini­
ACC guideline for the management of patients with valvular heart disease:
cians who treat patients with amiodarone. Practice Guidelines
executive summary: a report of the American College of Cardiology/
Subcommittee, North American Society of Pacing and Electrophysiology.
American Heart Association Task force on Practice Guidelines. J Am Coll
Arch Intern Med. 2000 June 26 ; 160(12):1741-8. [PMID: 10871966]
Cardiol. 2014 Jun 10:63(22) :2438-88. Erratum in: J Am Coll Cardiol. 2014
Greenspon AJ, Patel JD. Lau E. et al. 16-year trends in the infection burden for Jun 10:63 (22):2489. [PMID: 24603192]
pacemakers and implantable cardioverter-dellbrillators in the United
Rogers JH, Franzen 0. Percutaneous edge-to-edge MitraClip therapy in the
States 1993 to 2008. J Am Coll Cardiol. 2011 Aug 30:58(10):1001-6. [PMID:
management of mitral regurgitation. Eur Heart J. 2011 Oct:32(19):2350-7.
2 1 867833]
[PMID: 21606080]
Hart RG, Pearce LA. Current status of stroke risk stratification in patients with
Wilson W. Taubert KA, Gewitz M. el al: American Heart Association Rheumatic
atrial fibrillation. Stroke. 2009 Jul:40 (7) :2607-10. [PMID: 19461030]
!'ever. Endocarditis, and Kawasaki Disease Committee: American Heart
January CT. Wann LS, Alpert JS. et al. 2014 AHA/ACC/HRS Guideline for the Association Council on Cardiovascular Disease in the Young; American
Management of Patients With Atrial Fibrillation: A Report of the American Heart Association Council on Clinical Cardiology: American Heart
College of Cardiology/American Heart Association Task Force on Practice Association Council on Cardiovascular Surgery and Anesthesia; Quality of
Guidelines and the Heart Rhythm Society. Circulation. 2014 Apr 10. [PMID: Care and Outcomes Research Interdisciplinary Working Group. Prevention
24682347] of infective endocarditis: guidelines from the American Heart Association:
Lee GK. Klarich KW, Grogan M, et al. Premature ventricular contraction­ a guideline from the American Heart Association Rheumatic Fever.
induced cardiomyopathy: a treatable condition. Circ Arrhythm Endocarditis. and Kawasaki Disease Committee. Council on Cardiovascular
Electrophysiol. 2012 Feb:5(1):229-36. [PM I D : 22334430] Disease in the Young, and the Council on Clinical Cardiology, Council on
Cardiovascular Surge1y and Anesthesia, and the Quality of Care and
Nieuwlaat R, Connolly BJ. Hubers LM, et al: ACTIVE Investigators. Quality of
Outcomes Research Interdisciplinary Working Group. Circulation. 2007
individual I N R control and the risk of stroke and bleeding events in atrial
Oct 9 : 1 1 6 (15): 1736-54. Erratum in: Circulation. 2007 Oct 9 ;116(15) :e376-7.
fibrillation patients: a nested case control analysis of the ACTIVE W study.
[PMID: 17446442]
Thromb Res. 2012 Jun;l29(6):715-9. [PMID: 21 924760]
Patel MR, Mahaffey KW. Garg J, et al; ROCKET Al' Investigators. Rivaroxaban Adult Congenital Heart Disease
versus warfarin in nonvalvular atrial fibrillation. N Engl J Med. 2011 Sep Brown ML. Burkhart HM. Connolly HM. et al. Coarctation of the aorta: lifelong
8;365(10):883-91. [PMID: 21830957] surveillance is mandatory following surgical repair. J Arn Coll Cardiol. 2013
Sep 10:62(1 1 ) : 1020-5. [PMID: 23850909]
Pericardia! Disease
D'Alto M. Romeo E. Argiento P. et al. Bosentan-sildenalll association in
Antman EM, Cargill V, Grossman W. Low-pressure cardiac tamponade. Ann
patients with congenital heart disease-related pulmonary arterial hyper­
Intern Med. 1979 Sep:91(3):403-6. [PMID: 475168]
tension and Eisenmenger physiology. Int J Cardiol. 2012 Mar 22:155(3):378-
Bertog SC, Thambidorai SK, Parakh K, et al. Constrictive pericarditis: etiology 82. [PMID: 21081251]
and cause-specific survival after pericardiectomy. J Am Coll Cardiol. 2004
DeSimone CV, l'riedman PA. Noheria A. et al. Stroke or transient ischemic
Apr 2 1 :43(8) : 1445-52. [PMID: 15093882]
attack in patients with transvenous pacemaker or defibrillator and echo­
Burstow DJ. Oh JK, Bailey KR, Seward JB, Tajik AJ. Cardiac tamponade: char­ cardiographically detected patent foramen ovale. Circulation. 2013 Sep
acteristic Doppler observations. Mayo Clin Proc. 1989 Mar:64(3):312-24. 24;128(13): 1433-4 1 . [PM ID: 23946264]
[PMID: 2704254]
European Society of Gynecology (F.SG) : Association for European Paediatric
Ha JW. Oh JK. Schaff I-IV, et al. Impact of left ventricular function on immedi­ Cardiology (AEPC): German Society for Gender Medicine (DGesGM ) ,
ate and long-term outcomes after pericardiectomy in constrictive pericar­ Regi tz-Zagrosek V. Blomstrom Lundqvist C , Borghi C . e t a l : ESC Committee
ditis. J Thorac Cardiovasc Surg. 2008 Nov;l36(5) :1136-4 1 . [PMID: 19026793] for Practice Guidelines. ESC Guidelines on the management of cardiovas­
Haley JI-I, Tajik AJ, Danielson GK, Schaff I-IV, Mulvagh SL. Oh JK. Transient cular diseases during pregnancy: the Task Force on the Management of
constrictive pericarditis: causes and natural history. J Am Coll Cardiol. 2004 Cardiovascular Diseases during Pregnancy of the European Society of
Jan 21;43(2):271-5. [PMID: 14736448] Cardiology (ESC). Eur Heart J. 2011 Dec;32(24):3l47-97. [PMID: 21873418]
lmazio M, Brucato A, Cemin R. et al: ICAP I nvestigators. A randomized trial of Furie KL. Kasner SE. Adams RJ. et al: American Heart Association Stroke
colchicine for acute pericarditis. N Engl J Med. 2013 Oct 17;369(16):1522-8. Council, Council on Cardiovascular Nursing, Council on Clinical Cardiology,
[PMID: 23992557] and I nterdisciplinary Council on Quality of Care and Outcomes Research.

115
B i b l i o g raphy

Guidelines for the prevention o f stroke in patients with stroke o r transient Anderson JL, Halperin JL, Albert NM, et al. Management of patients with
ischemic attack: a guideline for healthcare professionals from the American peripheral artery disease (compilation of2005 and 2011 ACCF/AHA guide­
Heart Association/American Stroke Association. Stroke. 2011 Jan:42(1):227- line recommendations): a report of the American ColJege of Cardiology
76. [PMID: 20966421] Foundation/American Heart Association Task Force on Practice Guidelines.
Warnes CA, Williams RG, Bashore TM, et al; American ColJege of Cardiology; Circulation. 2013 Apr 2;12703 ) : 1425-43. [PMID: 23457117]
American Heart Association Task Force on Practice Guidelines (Writing Ankle Brachia! I ndex ColJaboration, Fowkes FG, Murray GD. Butcher I, et al.
Committee to Develop Guidelines on the Management of Adults With Ankle brachia] index combined with Framingham Risk Score to predict
Congenital Heart Disease); American Society of Echocardiography: Heart cardiovascular events and mortality: a meta-analysis. JAMA. 2008 Jul
Rhythm Society; International Society for Adult Congenital Heart Disease; 9;300(2):197-208. [PMID: 18612117]
Society for Cardiovascular Angiography and Interventions: Society of Berger JS, Hiatt WR. Medical therapy in peripheral artery disease. Circulation.
Thoracic Surgeons. ACCIAHA 2008 guidelines for the management 2012 Jul 24:126(4):491-500. [PMID: 22825411]
of adults with congenital heart disease: a report of the American ColJege of
Cardiology/American Heart Association Task Force on Practice Guidelines Berger JS, Hochman J. Lobach I. Adelman MA, Riles TS, Rockman CB.
(Writing Committee to Develop Guidelines on the Management of Adults Modifiable risk factor burden and the prevalence of peripheral artery dis­
With Congenital Heart Disease). Developed in colJaboration with the ease in different vascular territories. J Vase Surg. 2013 Sep:S8(3):673-81 .el.
American Society ofEchocardiography, Heart Rhythm Society, International [PM ID: 23642926]
Society for Adult Congenital Heart Disease, Society for Cardiovascular Creager MA, Kaufman JA, Conte MS. Clinical practice. Acute limb ischemia.
Angiography and Interventions, and Society of Thoracic Surgeons. J Am N Engl J Med. 2012 Jun 7;366(23):2198-206. [PMID: 22670905]
ColJ Cardiol. 2008 Dec 2;52(23):el43-263. [PMID: 19038677] McDermott MM, Ades P, Guralnik JM, et al. Treadmill exercise and resistance
Diseases of the Aorta
training in patients with peripheral arterial disease with and without inter­
mittent claudication: a randomized controlled trial. JAMA. 2009 Jan
Baxter BT, Terrin MC. Dalman RL. Medical management of smalJ abdominal 14 ; 301 (2):165 -74. Erratum in: JAMA. 2012 Apr 25:307 (16): 1694. [ PM I D:
aortic aneurysms. Circulation. 2008 Apr 8:117(14) : 1883-9. [PMID: 18391122] 19141764]
Chaikof EL, Brewster DC. Dalman RL, et al. SYS practice guidelines for the care Murphy TP, Cutlip DE, Regensteiner JG, et al; CLEVER Study Investigators.
of patients with an abdominal aortic aneurysm: executive summary. J Vase Supervised exercise versus primary stenting for claudication resulting from
Surg. 2009 Oct;50(4):880-96. [PMID: 19786241] aortoiliac peripheral artery disease: six-month outcomes from the claudi­
De Bruin JL, Baas AF, Buth J, et al: DREAM Study Group. Long-term outcome cation: exercise versus endoluminal revascularization (CLEVER) study.
of open or endovascular repair of abdominal aortic aneurysm. N Engl J Circulation. 2012 Jan 3 ; 125(1):130-9. [ PMID: 22090168]
Med. 2010 May 20:362(20): 1881 -9. [PMID: 20484396) Norgren L, Hiatt WR, Dormandy JA, et al: TASC II Working Group. Inter­
Hiratzka LF, Bakris GL, Beckman JA, et al; American ColJege of Cardiology Society Consensus for the Management of Peripheral Arterial Disease
Foundation/American Heart Association Task Force on Practice (TASC I I ) . J Vase Surg. 2007 Jan;45 Suppl S:S5-67. [PM ID: 17223489]
Guidelines: American Association for Thoracic Surgery; American College Rooke TW, Hirsch AT, Misra S, et al; American ColJege of Cardiology
of Radiology: American Stroke Association: Society of Cardiovascular Foundation/American Heart Association Task Force on Practice Guidelines;
Anesthesiologists; Society for Cardiovascular Angiography and Society for Cardiovascular Angiography and Interventions; Society of
Interventions; Society of lnterventional Radiology; Society of Thoracic lnterventional Radiology: Society for Vascular Medicine; Society for
Surgeons: Society for Vascular Medicine. 2010 ACCF/AHA/AATS/ACR/ Vascular Surgery. 2011 ACCF/AHA focused update of the guideline for the
ASA/SCA!SCAl/SIR/STS/SVM Guidelines for the diagnosis and manage­ management of patients with peripheral artery disease (updating the 2005
ment of patients with thoracic aortic disease. A Report of the American guideline): a report of the American ColJege of Cardiology Foundation/
College of Cardiology Foundation/American Heart Association Task Force American Heart Association Task Force on Practice Guidelines: developed
on Practice Guidelines, American Association for Thoracic Surgery, in colJaboration with the Society for Cardiovascular Angiography and
American ColJege of Radiology, American Stroke Association, Society of Interventions, Society of lnterventional Radiology, Society for Vascular
Cardiovascular Anesthesiologists, Society for Cardiovascular Angiography Medicine, and Society for Vascular Surgery. J Vase Su rg. 2011 Nov;
and Interventions, Society of lnterventional Radiology, Society ofThoracic 54(5):e32-58. [PMID: 21958560]
Surgeons, and Society for Vascular Medicine. J Am ColJ Cardiol. 2010 Apr
6:55(14) :e27-el29. Erratum in: J Am ColJ Cardiol. 2013 Sep 10:62(1 1): 1039 - Cardiovascular Disease in Cancer Survivors
40. [PMID: 20359588] Fiuza M. Cardiotoxicity associated with trastuzumab treatment of H ER2+
Jondeau G, Detaint D. Tubach F, et al. Aortic event rate in the Marfan popula­ breast cancer. Adv Ther. 2009 Jul;26 Suppl l:S9-17. [ PM I D : 19669637]
tion: a cohort study. Circulation. 2012 Jan 17:125(2 ) : 226-32. [PMID: Perez EA, Suman VJ, Davidson NE, et al. Cardiac safety analysis of doxorubicin
22133496] and cyclophosphamide followed by paclitaxel with or without trastuzumab
Nienaber CA, Kische S, Rousseau H, et al: INSTEAD-XL trial. Endovascular in the North Central Cancer Treatment Group N9831 adjuvant breast cancer
repair of type B aortic dissection: long-term results of the randomized trial. J Clin Oncol. 2008 Mar 10;26(8):1231-8. [PM ID: 18250349]
investigation ofstent grafts in aortic dissection trial. Circ Cardiovasc Jnterv. Slamon D, Eiermann W, Robert N, et al; Breast Cancer I nternational Research
2013 Aug;6(4):407-16. [PMID: 23922146] Group. Adjuvant trastuzumab in HER2-positive breast cancer. N Engl J
Nienaber CA, Rousseau H, Eggebrecht H , et al; INSTEAD Trial. Randomized Med. 2011 Oct 6;365(14) : 1273-83. [PMID: 21991949]
comparison of strategies for type B aortic dissection: the INvestigation of Swain SM, Whaley FS. Ewer MS. Congestive heart failure in patients treated
STEnt Grafts in Aortic Dissection ( INSTEAD) trial. Circulation. 2009 Dec with doxorubicin: a retrospective analysis of three trials. Cancer. 2003 Jun
22;120(25):2519-28. [PMID: 19996018] 1;97(11):2869-79. [PM I D : 12767102]
United Kingdom EVAR Trial Investigators, Greenhalgh RM, Brown LC, Powell Yeh ET, Tong AT, Lenihan DJ, et al. Cardiovascular complications of cancer
JT, Thompson SG, Epstein D. Endovascular repair of aortic aneurysm in therapy: diagnosis, pathogenesis, and management. Circulation. 2004 Jun
patients physically ineligible for open repair. N Engl J Med. 2010 May 29:!09(25):3122-31. [PM I D : 15226229]
20:362(20): 1872-80. [PMID: 20382982]
Pregnancy and Cardiovascular Disease
United Kingdom EVAR Trial Investigators, Greenhalgh RM. Brown LC, Powel l
JT, Thompson SG, Epstein D . Sculpher MJ. Endovascular versus open repair De Santo LS, Romano G, Della Corte A, et al. Mechanical aortic valve replace­
of abdominal aortic aneurysm. N Engl J Med. 20!0 May 20:362(20) :1863-71. ment in young women planning on pregnancy: maternal and fetal out­
[PMID: 20382983] comes under low oral anticoagulation, a pilot observational study on a
comprehensive pre-operat ive counseling protocol. J Am Coll Cardiol. 2012
Peripheral Arterial Disease Mar 20:59(12) : 1110-5. [PMID: 22421305]
Aboyans V, Criqui MH, Abraham P, et al: American Heart Association Council European Society of Gynecology (ESG): Association for European Paediatric
on Peripheral Vascular Disease: Council on Epidemiology and Prevention; Cardiology (AEPC); German Society for Gender Medicine (DGesGM),
Council on Clinical Cardiology; Council on Cardiovascular Nursing; Regitz-Zagrosek V, Blomstrom Lundqvist C, Borghi C, et al; ESC Committee
Council on Cardiovascular Radiology and Intervention, and Council on for Practice Guidelines. ESC Guidelines on the management of cardiovas­
Cardiovascular Surgery and Anesthesia. Measurement and interpretation of cular diseases during pregnancy: the Task Force on the Management of
the ankle-brachia! index: a scientific statement from the American Heart Cardiovascular Diseases during Pregnancy of the European Society of
Association. Circulation. 2012 Dec 11: 126(24):2890-909. Erratum in: Cardiology (ESC). Eur Heart J. 20ll Dec:32(24):3147-97. [PM I D : 21873418]
Circulation. 2013 Jan 1: 127(l):e264. [PMID: 23159553] Siu SC. Sermer M. Colman JM, et al: Cardiac Disease in Pregnancy (CARPREG)
Adam DJ, Beard JD, Cleveland T, et al: BASIL trial participants. Bypass versus Investigators. Prospective multicenter study of pregnancy outcomes in
angioplasty in severe ischaemia of the leg (BASIL): multicentre, randomised women with heart disease. Circulation. 2001 Jul 3 1 ;104(5):515-21. [PM I D :
controlJed trial. Lancet. 2005 Dec 3:366(9501):1925-34. [PMID: 16325694) 1 1479246]

116
Bib liog ra p h y

Sliwa K, Blauwet L, Tibazarwa K, et al. Evaluation of bromocript ine in the Committee to Develop Guidelines on the Management o f Adults With
treatment of acute severe peripartum cardiomyopathy: a proof-of-concept Congenital Heart Disease); American Society of Echocardiography; Heart
pilot study. Circulation. 2010 Apr 6 ; 12 1 (13) : 1465-73. [PM I D : 20308616] Rhythm Society: International Society for Adult Congenital Heart Disease:
Sliwa K, Hilfiker-Kleiner D, Petrie MC, et al; Heart Failure Association of the Society for Cardiovascular Angiography and Interventions; Society of
European Society of Cardiology Working Group on Peripartum Thoracic Surgeons. ACC/AHA 2008 guidelines for the management
Cardiomyopathy. Current state of knowledge on aetiology, diagnosis, man­ of adults with congenital heart disease: a report of the American College of
agement, and therapy of peripartum cardiomyopathy: a position statement Cardiology/American Heart Association Task Force on Practice Guidelines
from the Heart Failure Association of the European Society of Cardiology (Writing Committee to Develop Guidelines on the Management of Adults
Working Group on peripartum cardiomyopathy. Eur J Heart Fail. 2010 With Congenital Heart Disease). Developed in Collaboration With the
Aug:I2(8) :767-78. [PM I D : 20675664] American Society of Echocardiography, Heart Rhythm Society, International
Society for Adult Congenital Heart Disease. Society for Cardiovascular
Warnes CA, Williams RG, Bashore TM. et al; American College of Cardiology: Angiography and Interventions, and Society of Thoracic Surgeons. J Am
American Heart Association Task Force on Practice Guidelines (Writing Coll Cardiol. 2008 Dec 2:52(23):el43- 263. [ PM I D : 19038677]

117
Cardiovascular Medicine
Self-Assessment Test
This self-assessment test contains one-best-answer multiple-choice questions. Please read these directions carefully
before answering the questions. Answers, critiques, and bibliographies immediately follow these multiple-choice
questions. The American College of Physicians is accredited by the Accreditation Council for Continuing Medical
Education (ACCME) to provide continuing medical education for physicians.

The American College of Physicians designates MKSAP 1 7 Cardiovascular Medicine for a maximum of 21 AMA PRA
Category 1 CreditsrM . Physicians should claim only the credit commensurate with the extent of their participation
in the activity.

Earn "Instantaneous" CME Credits Online


Print subscribers can enter their answers online to earn Continuing Medical Education ( CME) credits instanta­
neously. You can submit your answers using online answer sheets that are provided at mksap.acponline.org, where
a record of your MKSAP 17 credits will be available. To earn CME credits, you need to answer all of the questions in
a test and earn a score of at least 50% correct ( number of correct answers divided by the total number of questions) .
Take any of the following approaches:

);;>- Use the printed answer sheet at the back of this book to record your answers. Go to mksap.acponline.org,
access the appropriate online answer sheet, transcribe your answers, and submit your test for instantaneous
CME credits. There is no additional fee for this service.

);;>- Go to mksap.acponline.org, access the appropriate online answer sheet, directly enter your answers, and
submit your test for instantaneous CME credits. There is no additional fee for this service.

);;>- Pay a $15 processing fee per answer sheet and submit the printed answer sheet at the back of this book by
mail or fax , as instructed on the answer sheet. Make sure you calculate your score and fax the answer sheet
to 215-351 -2799 or mail the answer sheet to Member and Customer Service, American College of Physicians,
190 N. Independence Mall West, Philadelphia, PA 19106-1572, using the courtesy envelope provided in your
MKSAP 17 slipcase. You will need your 10-digit order number and 8-digit ACP ID number, which are printed
on your packing slip. Please allow 4 to 6 weeks for your score report to be emailed back to you. Be sure to
include your email address for a response.

I f you do not have a 10-digit order number and 8-digit ACP ID number or if you need help creating a user name and
password to access the MKSAP 17 online answer sheets, go to mksap.acponline. org or email custserv@acponline .org.

CME credit is available from the publication date of July 31, 2015, until July 31, 2018. You may submit your answer
sheets at any time during this period.

119
..�
· .·
Directions .:c""�·
Each of the numbered items is followed by lettered answers. Select the ONE lettered answer that is BEST in each case.

Item 1 Electrocardiogram shows sinus rhythm with nonspe­


A 33-year-old woman is evaluated as an outpatient follow­ cific ST changes.
ing an episode of atrial fibrillation. 111e episode resolved Which of the following is the most appropriate diagnostic
shortly after arriving at the emergency department. She has test to perform next?
a history of tetralogy of Failor with repair performed at the
age of 4 years. (A) Chest CT
On physical examination, blood pressure is 110/70 (B) Exercise echocardiography stress testing
mm Hg, pulse rate is 62/min and regular, and respiration (C) Transthoracic echocardiography
rate is 18/min. BM! is 28. The estimated central venous (D) No further testing
pressure is normal. The apical impulse is normal; there is a
parasternal impulse at the left sternal border. S 1 is normal.
The S2 is single, and there is a soft early systolic murmur at Item 4
the second left intercostal space. A grade 2/6 decrescendo
A 58-year-old woman is evaluated during a routine physi­
diastolic murmur that increases with inspiration is noted at
cal examination. She has a history of atrial fibrillation and
the left sternal border. The remainder of the physical exam­
had an atrial fibrillation ablation procedure 6 months ago.
ination is normal.
Before her ablation, she had persistent atrial fibrillation
Which of the following is the most likely diagnosis? with palpitations and dyspnea. Since her ablation, she has
been asymptomatic with no palpitations. Ambulatory elec­
(A) Aortic valve regurgitation
trocardiographic monitoring at 3 and 6 months after the
(B) Pulmonary valve regurgitation ablation demonstrated no atrial fibrillation. Medical history
(C) Recurrent ventricular septa! defect is also significant for a transient ischemic attack, hyperten­
(D) Tricuspid valve regurgitation sion, and hyperlipidemia. Her medications are warfarin,
metoprolol, candesartan, and simvastatin.
On physical examination, the patient is afebrile, blood
Item 2 pressure is 130/80 mm Hg, pulse rate is 64/min, and respi­
A 58-year-old man is evaluated during a routine appoint­ ration rate is 16/min. BMl is 30. Heart rate and rhythm are
ment and asks for advice on cardiac risk assessment. He regular.
does not have any current cardiac symptoms, exercises An electrocardiogram shows normal sinus rhythm.
4 days per week, and has never smoked. He has no chronic Which of the following is the most appropriate manage­
health issues and takes no medications. He has no known ment?
drug allergies. Results of the physical examination are
normal. (A) Continue warfarin
Cardiovascular risk calculation using the Pooled Cohort (B) Continue warfarin and add aspirin
Equations predicts a 6% risk of a myocardial infarction or (C) Discontinue warfarin
coronary death in the next 10 years.
(D) Discontinue warfarin and start aspirin
Which of the following tests should be performed next? (E) Discontinue warfarin and start aspirin and clopidogrel
(A) Adenosine cardiac magnetic resonance (CMR) imaging
(B) Cardiac CT angiography Item 5
(C) Fractionated lipoprotein profile
A 52-year-old woman is seen in follow-up for dyspnea. Over
(D) High-sensitivity C-reactive protein assay the past 6 months, she has noticed increasing shortness of
(E) Stress echocardiography breath during her daily run, which she has had to decrease
from 2 miles to 1 mile. She is able to complete other aerobic
exercises, such as biking and tennis, with minimal limita­
Item 3 tion. She has experienced no chest pain or syncope. Medical
A 64-year-old woman is evaluated during a routine exam­ history is significant for hypertension, diagnosed 10 years
ination. She has no symptoms. Medical history is sig­ ago. Her only medication is hydrochlorothiazide.
nificant for hypertension and type 2 diabetes mellitus. On physical examination, blood pressure is 122/70 mm
Medications are amlodipine, losartan, atorvastatin, and Hg and pulse rate is 66/min. Lungs are clear to auscultation.
metformin. Cardiac examination reveals a rapid carotid upstroke and a
On physical examination, she is afebrile, blood pres­ grade 3/6 holosystolic murmur heard best at the left lower
sure is 154/77 mm Hg and equal on both sides, pulse rate is sternal border. The murmur increases during both end­
82/min, and respiration rate is 16/min. BM! is 28. Cardiac expiration and squat-to-stand maneuvers.
examination shows a grade 1/6 decrescendo diastolic mur­ Transthoracic echocardiogram shows left ventricular
mur heard best over the apex. There are no changes with a hypertrophy and dynamic left ventricular outflow tract
Valsalva maneuver or change in position. Peripheral pulses obstruction consistent with a diagnosis of hypertrophic
are normal. cardiomyopathy.

1 21
Self-Assessment Test

Which of the following is the most appropriate next step in Item 7


treatment? A 52-year-old woman is evaluated for fatigue and lower
(A) Discontinue hydrochlorothiazide extremity swelling. One year ago, she had acute idio­
pathic pericarditis treated with anti- inflammatory medi­
(B) Dual-chamber pacemaker
cations and colchicine. Symptoms initially improved and
(C) Initiate lisinopril her medications were discontinued. However, for the past
(D) Surgical myectomy 3 months, she has had worsening symptoms of exertional
fatigue and edema in both legs. She currently takes no
medications.
Cl A 71 -year-old man is eva lua ted in the emergency depart­
Item 6 On physical examination, she is afebrile, blood pres­
sure is 130/78 mm Hg, pulse rate is 88/min, and respiration
ment for severe pain in t h e chest and back t hat was a bru p t rate is 16/min. Pulsus paradoxus of 15 mm Hg is present.
i n onset and h a s persisted for 3 hours. He h a s no a bdom i n a l The estimated central venous pressure is 10 cm H p , and the
pain . leg pa in. or neurologic symptoms. His medical history jugular venous pulse contour shows diminished y descents.
is not a ble for hypertension. Med ica t ion s a re a mlod i p i ne and The lungs are clear to auscultation. Heart sounds are nor­
l isi nopril . mal, with no rubs or gallops. Hepatomegaly is present, and
On physical exam inatio n . t he patient is a feb ri le . blood peripheral edema is noted in both lower extremities up to
pressure is 1 80/ 100 mm H g in bot h arms . pulse rate is the knees.
98/min. and respiration ra te is 1 8/ m i n . Oxygen saturation A 12- lead electrocardiogram is normal. Echocardio­
is 96% on ambient air. Cardiac auscultation discloses 8 n S , gram shows the cardiac chambers to be normal in size
g8l lop but n o m urmur. Pulmonary exa m i nation i s norma l . and function with a moderate circumferential pericar­
Pu ls es a r e sym m e t ric and equal in 8 1 1 extremi ties. dia! effusion. A CT of the heart shows normal pericardia!
Laboratory s t udies show a D-dimer level of 1 . 2 pg / m L thickness.
( 1 . 2 mg/ L) a n d a serum creatinine level of 1 . 0 mg/d L A pericardiocentesis fai ls to resolve the elevated
(88.4 p mo l/ L) . I n i t ial serum cardiac tropo ni n I level i s not right atrial pressure documented on right heart cathe­
elev8ted. terization.
Electrornrdiogram shows left vent ricular hypertrophy
w i t h repolariz8tion abnormalit ies. Chest radiograp h shows Which of the following is the most likely diagnosis?
a n e n l a rge d C8rdiac s i l houette. Chest CT sca n w i t h i n t rave­ (A) Cor pulmonale
nous con t rast demonstrates 8 focal pe n e t rat i ng ulcer i n t he
(B) Effusive constrictive pericarditis
t horacic desce n d i n g aorta ( s how n ).

(C) Heart failure


(D) Recurrent acute pericarditis

Item 8
A 58-year-old man is evaluated during a routine appoint­
ment. He is asymptomatic. He was diagnosed with type 2
diabetes mellitus 4 years ago and has hypertension, dys­
lipidemia, and obesity. H is medications are enteric-coated
low-dose aspirin, lisinopril, fluvastatin (20 mg/d), and
metformin.
H is calculated 10-year risk of atherosclerotic cardiovas­
cular disease (ASCVD) using the Pooled Cohort Equations
is l0%.
On physical examination, blood pressure is 126/78 mm
Hg and pulse rate is 72/min. The remainder of the examina­
tion is normal.
Laboratory studies:
Total cholesterol 186 mg/dL (4.82 mmol/L)
LDL cholesterol 123 mg/dL (3.19 mmol/L)
HDL cholesterol 44 mg/dL (1.14 mmol/L)
Triglycerides 109 mg/dL (1.23 mmol/L)

Which of the following is the most appropriate immediate Which of the following is the most appropriate statin man­
next step in management? agement?

(A) Heparin followed by wa rfa rin (A) Increase fluvastatin to 40 mg/d


(B) Endovascular stenting (B) Switch to atorvastatin, 40 mg/d
(C) I n t ravenous P bl ocka de fol lowed by i n t ravenous
- (C) Switch to lovastatin, 20 mg/d
sodiu m n i t roprusside (D) Switch to pravastatin, 20 mg/d
(D) O pen surgica l repai r (E) Switch to simvastatin, 10 mg/ct

1 22
Self-Assessment Test

Item 9 Resting electrocardiogram (ECG) shows no evidence of


A 34-year-old woman with idiopathic heart failure diag­ left ventricular hypertrophy and no ST- or T-wave changes.
nosed 9 months ago is evaluated for a 3-month history of Exercise ECG is performed. The patient is able to exer­
nonproductive cough. She sleeps on one pillow and has no cise on the Bruce protocol for 10 minutes and 20 seconds;
paroxysmal nocturnal dyspnea. Medical history is other­ blood pressure rises appropriately, but he develops chest
wise unremarkable, and she has never smoked. Medications tightness and stops because of breathlessness. There are no
are lisinopril, carvedilol, and furosemide. ECG changes.
On physical examination, the patient is afebrile, blood Which of the following is the most appropriate manage­
pressure is 102/70 mm Hg, pulse rate is 64/min, and respira­ ment?
tion rate is 16/min. Oxygen saturation on ambient air is 98%.
The remainder of the examination, including heart and lung (A) Add diltiazem
examinations, is unremarkable. (B) Add isosorbide mononitrate
Her most recent left ventricular ejection fraction is (C) Coronary angiography
40%. Chest radiograph is unremarkable. (D) Pharmacologic nuclear stress test
Which of the following is the most appropriate manage­
ment?
Item 1 2
(A) Discontinue lisinopril and start valsartan A 32-year-old man is evaluated during an initial office visit.
(B) Echocardiography He has no symptoms and no significant medical history. He
(C) Obtain B-type natriuretic peptide level takes no medications.
(D) Pulmonary function testing On physical examination, blood pressure is 120/70
mm Hg in both arms, pulse rate is 64/min, and respiration
rate is 12/min. Cardiac examination reveals a grade 1/6
Item 1 0 decrescendo diastolic murmur heard best at the left lower
sternal border. Femoral pulses are equal.
A 48-year-old man with newly diagnosed hypertension is
referred for an echocardiogram to assess findings of left ven­ Which of the following is the most likely cause of the
tricular hypertrophy noted on the electrocardiogram. He patient's murmur?
is asymptomatic, and his medical history is unremarkable
(A) Aortic coarctation
other than hypertension. His only medication is chlortha­
lidone. (B) Atrial septa! defect
On physical examination, blood pressure is 128/70 mm (C) Bicuspid aortic valve
Hg, pulse rate is 60/min and regular, and respiration rate is (D) Mitra! stenosis
18/min. BMI is 24. The cardiac examination is normal other
than the presence of an S4 .
The transthoracic echocardiogram demonstrates nor­ Item 1 3
mal left ventricular size, function, and mass index. An atrial A 69-year-old woman is evaluated for follow-up 3 months
septa! aneurysm with a small left-to-right shunt indicative after a non-ST-elevation myocardial infarction. She was
of a patent foramen ovale is noted by color-flow Doppler assessed to be at low risk, and she was treated medically.
imaging. The right ventricular chamber size, systoUc func­ Since the acute event, the patient has done well. She has no
tion, and estimated pressures are normal. chest discomfort or shortness of breath. She has hypercho­
Which of the following is the most appropriate manage­ lesterolemia and hypertension. Medications are lisinopril,
ment based on this patient's echocardiographic findings? metoprolol, atorvastatin, aspirin, and clopidogrel. She has
modified her diet and has begun performing physical activ­
(A) Anticoagulation therapy ity 5 days a week.
(B) Closure of the defect On physical examination, the patient is afebrile, blood
(C) Transesophageal echocardiography pressure is 125/80 mm Hg, pulse rate is 60/min, and respi­
ration rate is 12/min. BMI is 26. A normal carotid upstroke
(D) No further evaluation or treatment
without carotid bruits is noted, jugular venous pulsations
are normal, and normal S 1 and S2 heart sounds are heard
without murmurs. Lung fields are clear, distal pulses are
Item 1 1
normal, and no peripheral edema is present.
A 69-year-old man is evaluated for exertional chest pain Laboratory studies show adherence to her lipid therapy
that began 2 months ago. The patient describes the chest and are otherwise normal.
pain as tightness in the left side of his chest that occurs after
walking approximately a half-mile. This chest tightness Which of the following will offer this patient the greatest
improves with rest. Two years ago, he had a myocardial reduction in her risk of future cardiovascular events?
infarction. The patient's medications are low-dose aspirin, (A) Colchicine
metoprolol, lisinopril, and rosuvastatin.
(B) Folic acid
On physical examination, blood pressure is 124/72 mm
Hg and pulse rate is 52/min. The remainder of the examina­ (C) Influenza vaccine
tion is normal. (D) Vitamin E

1 23
Self-Assessment Test

Item 1 4 Item 1 5
A 5 1 -year-o l d woman is a d m i t ted t o t h e hosp i t a l w i t h A 7LJ -yea r-old man is eva l ua ted in the emergency depa rt ­
com m u n i ty-acq u i red pneumonia a ft e r outpatient t herapy men t for a 7-day h i story of progressive exert ional dyspnea
was unsuccessfu l . She presen ted 5 clays ago w i t h cough . associated w i t h a d ry coug h . increasing ort hopnea rn·om
lever. and clyspnea . a n d she was found to have righ t lower two to lour p i llows) . and i na b i l i ty to buckle h is bel t . H e has
lobe crack les on ex<1 m i n a tion and a corresponding i n fi l t m t e a 20-year h istory of' hypert ension t reated wi t h d i l t iazem .
on chest radiography . She was started on ora l moxi f l ox­ On physical ex a m i na t ion. b lood pressure is 1 6 2/86 mm
aci n . However. she has remained febri le wi t h worsen i ng H g. pulse rate is i rregul arly irregular a t 8-1 111 i n . a n d res­
short ness or breat h and is now acl m i t tecl to t h e hospi t a l for p i ra t ion ra t e is 1 8/m i n . Est imated central venous pressure
l'u rther tre<H men t . Medical h istory is sign i ficant l'or hyper­ is J . J cm H 20 . Cardiac examination reveal s a n irregularly
tension and depression . for wh ich s h e l a kes carved i l o l and i rregu lar rhyt h m and a n S 1 • Bibas i l a r crnckles are heard on
a m i t ri p ty l i n e. Her cu rre n t medications a re mox i l loxacin. <luscu l tat ion or t he lungs. H i s l iver is e n l arged 2 c m below
carvecli l o l . <lllcl a m i tripty l i ne . t h e cos tal margin . H is e x t re m i ty exa111 i nation revea ls bi l a t ­
On physica l exa111 i na t i o n . te111perature is 38 .4 °C era l p i t t ing edema .
( 10 1 . 1 °F) . blood pressure i s 140 90 m m Hg. pulse rate is Serum e lectrolyte levels a n d k i d ney fu nction tests
88 111 in . and respi ra t ion rate is 1 8 m i n . Oxygen saturat ion a re nor111 a l . Serum B- type n a t ri uretic peptide level is 2472
brea t h i ng ambient air is 89'X. . BMI is 25 . Chest exa m i nation is pg/ m l (2472 ng L) .
consistent with right lower lobe consol idation . ·n1e re111ainder E lectrocardiogram shows a t ri a l fibri l la tion . Echocar­
of t h e physical exa111inalion is u nre111arkable. d iogram shows a l e ft ven t ricu l a r ejection rraction of 60'X..
An e l ect rocardiogra m ( ECG) at t h e t i 111e of hospi t a l septa ! wal l t h ickness ol' 1.5 c111. and posterior wall t h i c kness
ad111ission is shown . o f l .4 cm. Chest rad i ograp h shows hazy b i l a tera l i n fi l t ra tes.

Which of the following medications should be discontin­ Which of the following is the most appropriate next step
ued based on this patient's ECG findings? in management?
(A) A m i t ri p ty l i n e (A) �- B locker
(B) A m i t ri p ty l i n e and can·e d i l o l ( B) Ca rd ioversion
( C ) A m i t ri p ty l i ne and mox i f loxacin (C) Furosemide
(D ) fvl ox i f loxacin (D) Spironolactone

� I

V3

ITEM 1 4

1 24
Self-Assessment Test

Cl IAt58e myear
16 On physical examination, he is afebrile, blood pressure
- -ol d man is eva l ua ted in the hospital for weakness, is 112172 mm Hg, pulse rate is 62/min, and respiration rate
fat i gue. and hypotension . ·111e patient reports a 3-week h is­ is 12/min. Cardiac examination shows a normal S1 and S2
tory of anorexia and 2 -year h i s t o ry of non-s m a l l cell l ung without S3, S4, murmurs, or rubs. Lung examination is nor­
cancer w i t h metastatic i nvolvement in t h e liver t reated w i t h mal. He has no lower extremity edema. The remainder of the
combination cytotoxic chemotherapy. examination is normal.
On physical exa m i n a t io n . t he patient appears cachec­ Diagnostic coronary angiography reveals a 90% steno­
t ic. B lood pressure is 70/52 mm Hg and pu lse ra te is
sis in the proximal left anterior descending artery; the left
l l O/ m i n . The l un gs a re clear to a uscultation . l l1e j ugu lar circumflex artery has a diffuse 70% stenosis, and the right
venous pulse contour is ! l at . Caro t i d upstrokes a re brisk. S1 coronary artery has a 70% ostial stenosis. Left ventriculog­
a n d S" are dista n t . No rubs. ga l lops. or murmurs a re presen t . raphy shows a left ventricular ejection fraction of 50% with
N o peripheral edema is noted . T h e rema inder o f t he physical mild anterior wall hypokinesis.
exa m ination is unremarka ble. Which of the following is the most appropriate management?
Echocardiogram shows a small Jen ven t ricular ca v ­
i ty w i t h hyperdynamic funct ion (ejection frac t i o n . 75%) . (A) Change metoprolol to amlodipine
A moderately sized c i rcumfere n t i a l pericard ia! e ffusion is (B) Coronary artery bypass graft surgery
evident. Diastolic i nversion or the rig h t a t ri u m and s ig n i f­ (C) Multivessel percutaneous coronary intervention
ica n t respiratory vari a t ion in the m i tra l i n f low pattern are (D) Myocardial viability nuclear perfusion scan
presen t .
ll1e blood pressure does n o t change a n e r a d m i n is t ra­
tion o f a 1 - L bo l us of i n t ravenous f l uid . Item 1 9

In addition to continued intravenous fluid, which of the A 74-year-old woman is evaluated for a 3-week history of
following is the most appropriate next step in treatment? left shoulder pain and dyspnea on exertion. Medical history
is significant for COPD, hypertension, and coronary artery
(A) I ntra - a o rt ic b a lloon pump disease; she underwent stenting of the mid-left anterior
(B) Pericardiocentesis descending coronary artery 3 years ago. Because of her lung
(C) Phenylep hrine disease, she has limited exercise ability. Medications are
(D) Wi ndow pericarcliectomy lisinopril, hydrochlorothiazide, atorvastatin, aspirin, fluti­
casone, albuterol, and ipratropium.
On physical examination, the patient is afebrile, blood
pressure is 142/88 mm Hg, pulse rate is 82/min, and respi­
Item 1 7
ration rate is 18/min. BMI is 29. Estimated central venous
A 54-year-old man is evaluated during a routine appoint­ pressure is 8 cm H?O. Cardiac examination reveals a grade
ment. He has hypertension, dyslipidemia, obesity, and 2/6 midsystolic m1irmur heard best at the cardiac base and
erectile dysfunction. He currently drinks three alcoholic late expiratory wheezing bilaterally.
beverages daily. His mother had a nonfatal myocardial Electrocardiogram shows left ventricular hypertrophy
infarction (Ml) at age 55 years, and he is concerned about and repolarization abnormalities.
his risk of MI. Medications are hydrochlorothiazide, ator­
vastatin, and sildenafil as needed. Which of the following is the most appropriate diagnostic
test to perform next?
Which of the following components of this patient's med­
ical history is associated with the greatest risk for future (A) Adenosine single-photon emission CT myocardial
myocardial infarction? perfusion imaging
(B) Coronary catheterization
(A) Alcohol consumption
(C) Dobutamine stress echocardiogram
(B) Dyslipidemia
(D) Exercise stress echocardiogram
(C) Hypertension
(D) Obesity
Item 2 0
A 68-year-old man i s evaluated for a newly diagnosed car­
Item 1 8 diac murmur. He is active and swims and jogs regularly.
A 63-year-old man is evaluated for follow-up of coronary Medical history is otherwise unremarkable, and he takes no
artery disease that was diagnosed by exercise stress testing medications.
3 weeks ago. For his exertional chest pain, he was started On physical examination, he is afebrile, blood pressure
on a �-blocker and nitrate in addition to his baseline med­ is 140170 mm Hg, pulse rate is 82/min, and respiration rate
icatio n s . He reports that his symptoms have improved, is 16/min. Cardiac examination reveals a late-peaking sys­
although he remains limited in his activities because of tolic murmur located at the right upper sternal border with
exertional chest pain. Medical history is significant for an audible s?.
hypertension, type 2 diabetes mellitus, and hyperlipidemia. Transt horacic echocardiogram shows normal left
His current medications are aspirin, lisinopril, simvastatin, ventricular systolic function. Aortic valve area is 0.8 cm2 .
insulin, metoprolol, isosorbide mononitrate, and as-needed The mean gradient is 44 mm Hg, with a peak gradient of
sublingual nitroglycerin. 53 mm Hg.

1 25
Self-Assessment Test

Which of the following is the most appropriate management? 98/m i n. BM! is 25 . Cardiac examination shows a normal S 1
and S2 wit hout S r S � . murmur. or rnb. Lung examination is
(A) Balloon aortic valvuloplasty
normal. Serum troponin T level is 2 .0 ng/m L (2 . 0 µg/L) .
(B) Follow-up echocardiography i n 6 to 12 months Electrocardiogram is shown.
(C) Surgical aortic valve replacement T he patient is administered aspirin. clopidogrel. and
(D) Transcatheter aortic valve replacement unfractionated heparin. E mergency coronary angiography
shows normal corona1y anatomy. Diastolic ( left panel) and
systolic ( righ t pane/) i mages from left ven t riculography are
Cl A 66-year-old woman
Item 2 1
is eva lua ted a t t h e hospital for shown.
6 hours of chest pressure and shortness of breath . Earlier Which of the following is the most appropriate manage­
t h is day . the patien t 's husband was diagnosed with lung ment?
cancer. Medical h istory is otherwise u n remarkable.
On physical exa m i n a t ion . temperature is 36 .8 °C (A) Endomyocardial biopsy
(98 . 2 °F) . blood pressure is 1 10 62 m m Hg. and pulse rate is (B) I n t ra-aortic balloon pump

ITEM 2 1

1 26
Self-Assessment Test

CJ (C) Metoprolol and captopril On physical exa mi nation. the pa tient is in significant
(D ) ll1rombolytic therapy pain . He is a febrile, blood pressure is 1 18 /70 mm Hg i n both
CONT. upper extrem it ies. pu lse rate is 122/ min, and respiration
rate is 22/min . The estimated cen tral venous pressure is
Item 2 2
10 cm H.,O. The lungs are clear. The heart examination is
A 56-year-old woman is evaluated during a n appointment notable for an early diastolic decrescendo murmur heard
to establish care. She has a developmental delay, and she is loudest at the right upper sternal border. The dorsalis pedis
known to have pulmonary hypertension due to a congenital and posterior tibialis pulses are palpable and equal bilaterally.
cardiac condition. There is no history of cardiac surgery. She Serum cardiac t ropon in T level is 0. 4 ng/m L (0.4 µg/L) .
is on low-dose aspirin and thyroid replacement therapy. Electrocardiogram shows sinus tachycardia but is otherwise
On physical examination, blood pressure is 110/70 normal. Chest radiograph is normal. Chest CT with i n trave­
mm Hg, pulse rate is 68/min and regular, and respiration rate nous contrast is shown.
is 18/min. BMI is 32. The central venous pressure is elevated
with a prominent a wave. The apical impulse is normal. There
is a prominent parastemal impulse at the left sternal border.
The s, is normal; the s2 is loud. There is a grade 116 holosystolic
murmur at the left lower sternal border. The toes demonstrate
cyanosis and digital clubbing; her hands appear normal. The
remainder of the physical examination is unremarkable.

Which of the following is the most likely cause of this


patient's pulmonary hypertension?

(A) Atrial septa! defect


(B) Patent ductus arteriosus
(C) Tetralogy of Fallot
(D) Ventricular septa! defect

Cl AItem 23
56-year-old man with heart failure i s admitted t o the
hospital with a 2-week history of increasin g exertional dys ­

pnea and fat igue. He also has type 2 diabetes melli tus. Med­
ications are metform i n . l isi nopri l . carvedilol. furosem ide,
metola zo ne and digoxin .
,

On physical examination . blood p ressure is 88/ 60 mm An intravenous �-blocker is started .

Hg. pulse rate is 95/min . and respiration rate is 20 /min . He is Which of the following is the most appropriate next step
somewhat con fused and i nattentive. Jugu lar venous disten­ in management?
tion is present to the angle of the jaw wh ile sitting. Cardiac
examination reveals an S3 . There are bibasi lar crackles on (A) Conti nu ed medical t herapy alone
pulmonary examination . He has edema to the midth ighs. ( B) E me rgen cy surgical interve n t ion
Extremities appear mottled and are cool to t he touch . (C) Endovascular stenting
Serum creat inine level is 3. 1 mg/dL (274 µmol / L) : base­ (D) H eparin
l i ne value was 1 . 1 mg/dL (97 . 2 µmol/ L) . Serum sodium level
is 133 mEq/ L (133 mmol/L) . Electrocardiogram shows no
evidence of ischemia. Chest radiograph shows cardiomeg­ Item 2 5
aly and vascula r congestion .
A 46 year-old man is eva l uated in the hospital prior
-

In addition to intravenous diuresis, which of the following undergoing an elective hernia repair. Medical history is sig­
is the most appropriate management? n i ficant for a bicuspid aortic valve and a mechanical aortic
valve replacement 3 years ago for severe aortic stenosis. H is
(A) D obutam ine on ly medication is warfa ri n .
(B) Intra -aortic bal loon pump O n physical examination . blood pressure is 130 /75
(C) M i lrinone mm Hg. pulse rate is 82/m i n , and respira tion rate is 1 4 /m in .
(DJ Right hea rt catheterization Cardiac exa mination reveals a grade 1 /6 m idsystolic mur­
mur at the righ t upper sternal border.

Cl AI t e46m-year-old
24
man i s eva luated i n the emergency depa1i­
Which of the following is the most appropriate manage­
ment option for endocarditis prophylaxis?
men t for severe pain in the chest and upper back t h a t began (A) A moxici l l i n
acutely abou t l hour ago. ll1e pain is described as "deep" and
(B) Ceftriaxone
constant. H e has no other associated sym ptoms. Medical
history is significant for hypertension. and his medica tions (C) Clindamycin
are chlorthalidone and va lsartan. (D) No antibiotic prophy laxis

1 27
Se lf-Assessment Test

Item 2 6 Item 2 8
A 48-year-old man is evaluated for tightness in his calves. A 37-year-old woman is evaluated for exertional dyspnea.
His symptoms are exacerbated with walking and resolve She noticed mild shortness of breath with significant exer­
with rest. His medical history is significant for hyperten­ cise several years ago. Although she is still active, she has
sion, type 2 diabetes mellitus, and chronic kidney injury. had to progressively decrease the amount of exercise she
Medications are hydrochlorothiazide, lisinopril, met­ is able to do because of her symptoms. She has no other
formin, glyburide, and atorvastatin. He was a cigarette health problems, takes no medications, and has no known
smoker with a 30-pack-year tobacco use history but quit drug allergies.
6 months ago. On physical examination, she is afebrile, blood pres­
On physical examination, blood pressure is 138/74 mm sure is 120/70 mm Hg, pulse rate is 67 /min, and respiration
Hg, pulse rate is 68/min and regular, and respiration rate is rate is 14/min. Cardiac examination demonstrates a grade
1 6/min. BMI is 32. No abdominal or femoral bruit is present. 3/6 crescendo-decrescendo systolic murmur located at the
No skin changes are noted in the lower extremities. The right upper sternal border with delayed carotid upstrokes.
remainder of the physical examination is unremarkable. Transthoracic echocardiography demonstrates normal
Laboratory studies are significant for a serum creat­ systolic function with a left ventricular ejection fraction of
inine level of 1.9 mg/dL (168 µmol!L) , normal electrolyte 60%, mild concentric left ventricular hypertrophy, and a
levels, and a hemoglobin A 1c value of 6.4%. bicuspid aortic valve. The aortic valve has a mean gradient
Ankle-brachial index testing: of 42 mm Hg and valve area of 0.9 cm2.
Right systolic brachia! pressure 140 mm Hg Which of the following is the most appropriate manage­
Left systolic brachia! pressure 132 mm Hg ment?
Right posterior tibialis pressure 200 mm Hg
Left posterior tibialis pressure 130 mm Hg (A) Balloon aortic valvuloplasty
Right dorsalis pedis pressure Not detected (B) Start an ACE inhibitor
Left dorsalis pedis pressure 140 mm Hg (C) Surgical aortic valve replacement
Which of the following is the most appropriate diagnostic (D) Transcatheter aortic valve replacement
test to perform next?

(A) Exercise ankle-brachia! index Item 2 9


(B) Lower extremity CT angiography A 28-year-old woman i s evaluated after a 4-hour self­
(C) Lower extremity magnetic resonance angiography limited episode of palpitations. The symptoms occurred
(D) Toe-brachia! index while at work yesterday. She has no history of cardiovascular
disease and has no other cardiovascular symptoms. She is
active without limitations. She is on no medications.
Item 2 7 On physical examination, blood pressure is 110/70 mm
A n S I yea r o l d m a n is eva l u a ted i n t h e office 3 clays f'ol ­
- -
Hg, pulse rate is 60/min and regular, and respiration rate is
lo wi ng a perc u t a n eous coron a ry in terven tion wi t h place­ 15/min. BM! is 23. The estimated central venous pressure is
m e n t of a bare m e t a l s t e n t in t he left a nt e r ior descending elevated. The apical impulse is normal; there is a parasternal
artery for angina refractory to maximal medi c a l t herapy. impulse present at the left sternal border, and a soft midsys­
He i n d ica t es t h a t he feels vve l l except for pa l p i tatio n s t h a t tolic murmur is heard at the second left intercostal space.
were not presen t before t h e procedure. M edical h istory is Fixed splitting of the S? is noted throughout the cardiac
sign i fican t for hypertension and type 2 diabetes m el l i tus . cycle. The remainder of the physical examination is normal.
He has no risk factors for or h is tory o f sign i fi ca n t b leedi ng. Electrocardiogram is shown (see top of next page) .
Medica tions a re aspiri n . cl o pid o g rel . li sinopri l a t orvasta­
.
Which of the following is the most likely diagnosis?
t i n . and m e t f'orm i n .
On p hysical exa m i n a t ion. t he pat ient is af'e brile. blood (A) Atrial septa! defect
pressure is 1 1 0/60 mm H g . pulse rate is 65 m i n . and respi­ (B) Bicuspid aortic valve with aortic stenosis
rat ion rate is 1 2 m i n . B M l is 32. Est i ma ted cen t ra l venous (C) Congenital pulmonary valve stenosis
pressure is not elevated . ·111e heart has a n irregu larly i rregu­
(D) Mitra! valve prolapse with mitral regurgitation
lar rhyth m Lungs a re clea r w i t hout cra c k le s
. .

An e lec t ro c ardiog ra m shows atrial fibri l la t ion wi t h a


hea rt nue of 65 m in and no acute ischem ic cha nges. A n
Item 3 0
echoca rdiogram demons t rates a l e ft ven t ricular eject ion
fraction of' 30% . A 32-year-old woman i s evaluated fo r a prepregnancy
assessment. She has a heart murmur but is asymptomatic.
Which of the following is the most appropriate therapeu­ She has no history of atrial fibrillation. Her only medication
tic regimen for this patient? is prenatal vitamins.
On physical examination, blood pressure is 102/60
(A) Aspirin a n d c lop i do gre l
mm Hg and pulse rate is 70/min and regular. The esti­
(B) Asp ir in a n d dabigat ra n mated central venous pressure is elevated. The apical
( C ) Aspi ri n a n d warfa rin impulse is tapping, and there is a parasternal impulse at
(D) Aspi r i n . clopidogrel . and warf'arin the left sternal border. The S1 and S 2 are loud, and a grade

1 28
Self-Assessment Test

.
l���V4
.

V6

II

ITEM 29

2/6 diastolic decrescendo murmur is heard at the apex. Which of the following is the most appropriate manage­
No opening snap is appreciated. The lungs are clear, and ment of this patient's abdominal aortic aneurysm?
there is no edema.
(A) Refer for aneurysm repair
A transthoracic echocardiogram demonstrates normal
left ventricular size and function. The mitral valve is thick­ (B) Repeat abdominal ultrasonography in 6 months
ened with diastolic doming. The mitral valve mean gradient (C) Repeat abdominal ultrasonography in 12 months
is 12 mm Hg; the calculated mitral valve area is 0.9 cm2 . (D) Switch amlodipine to propranolol
There is no mitral valve regurgitation. The estimated pulmo­
nary artery systolic pressure is SS mm Hg.
Item 3 2
Which of the following is the most appropriate manage­
ment at this time? A 74-year-old man is evaluated 4 months after under­
going uncomplicated bioprosthetic surgical aortic valve
(A) Initiate an ACE inhibitor and dabigatran replacement. Within the past 2 weeks, he has developed
(B) Obtain cardiac magnetic resonance imaging exertional dyspnea, fatigue, and lower extremity edema.
(C) Proceed with mitral valve intervention Medical history is otherwise unremarkable, and he takes
no medications.
(D) Proceed with pregnancy without interventions or
On physical examination, vital signs are normal. The
testing
estimated central venous pressure is 12 cm H2 0, and the
jugular venous pulse shows prominent y descents. A peri­
cardia! knock is present. Peripheral edema is noted.
Item 3 1
An echocardiogram reveals no evidence of pericar­
A S2-year-old woman is evaluated in the office during a dia! effusion. The aortic and mitral valves are functioning
routine visit. Her medical history is significant for type 2 normally. The inferior vena cava is markedly enlarged. A
diabetes mellitus and hypertension. Medications are aspi­ Doppler ultrasound shows expiratory flow reversals in the
rin, lisinopril, amlodipine, insulin glargine, insulin aspart, hepatic veins consistent with constrictive pericarditis.
and rosuvastatin.
On physical examination, the patient is afebrile, blood Which of the following is the most appropriate next step in
pressure is 128/80 mm Hg, pulse rate is 73/min, and respi­ management?
ration rate is 18/min. BM! is 24. The lungs are clear to aus­
(A) Ibuprofen
cultation, and no cardiac murmurs are heard. Abdominal
examination reveals a pulsatile mass in the epigastrium. (B) Invasive cardiac hemodynamic evaluation
An infrarenal abdominal aortic aneurysm with maxi­ (C) Pericardiectomy
mum diameter of S.7 cm is noted on abdominal ultrasound. (0) Transesophageal echocardiography

1 29
Self-Assessment Test

Item 3 3 Which of the following is the most appropriate manage­


ment?
A 52-year-old man is evaluated during a follow-up visit.
He was initially evaluated for severe palpitations 4 months (A) Discharge and schedule fo l l ow - u p w i t h i n 7 days
ago. Evaluation at that time included 48-hour ambulatory (B) Measure B - type n a t ri u r e t i c pep t i d e
electrocardiographic monitoring that was significant for
( C ) Obtai n echocard iograp h y p r i o r to d ischarge
frequent premature ventricular contractions (PVCs) and
ventricular bigeminy. A stress echocardiogram showed no (D) U pgrade to bive n t ricu l a r i m p l a n table cardioverter­
evidence of ischemia and normal left ventricular function. deflbri l l a t o r
Cardiac magnetic resonance (CMR) imaging demonstrated
no evidence of myocardial scarring. He was started on a
�-blocker for treatment of PVCs at that time. He now reports Item 3 5
continued significant palpitations despite therapy but does A 20-year-old man i s evaluated for newly noted hyperten­
not have presyncope, syncope, or chest pain. He has no sion. He is asymptomatic and his medical history is unre­
family history of sudden cardiac death or heart failure. His markable. He takes no medications and has no family his­
only medication is metoprolol. tory of hypertension.
On physical examination, the patient is afebrile, blood On physical examination, blood pressure is 180/80 mm
pressure is 110/60 mm Hg, pulse rate is 82/min and irregu­ Hg in both upper extremities, pulse rate is 60/min and reg­
lar, and respiration rate is 12/min. BM! is 34. B-type natri­ ular, and respiration rate is 18/min. BM! is 20. The estimated
uretic peptide level is mildly elevated. central venous pressure is normal. The apical impulse is
Electrocardiogram shows frequent monomorphic displaced and sustained. The S 1 and S2 are normal. An S4
PVCs but is otherwise normal; the QRS interval on con­ is noted at the apex. A soft systolic murmur is noted over
ducted sinus beats is 110 ms. Ambulatory 24-hour electro­ the left posterior chest. An abdominal bruit is audible. The
cardiographic monitoring shows frequent monomorphic femoral pulses are difficult to palpate, and there is a radial
PVCs (21% of all beats) and continued frequent ventricu­ artery-to-femoral artery pulse delay.
lar bigeminy. An echocardiogram is significant for mild The electrocardiogram is consistent with left ventricu­
to moderate global decreased left ventricular function but lar hypertrophy.
without regional wall motion abnormalities; ejection frac­
tion is estimated at 40%. Which of the following is the most likely diagnosis?

Which of the following is the most appropriate manage­ (A) Aortic coarctation
ment? (B) Essential hypertension
(C) Hypertrophic cardiomyopathy
(A) Amiodarone
(D) Renovascular hypertension
(B) Cardiac resynchronization therapy
(C) Catheter ablation of premature ventricular contrac­
tions
Item 3 6
(D) Implantable cardioverter-defibrillator
A 29-year-old woman who i s 10 weeks pregnant i s evalu­
ated for hypertension; this is her first pregnancy. She has no
symptoms and no prior cardiovascular disease. She is taking
Item 34
no medications. She has a family history of hypertension,
A 66 -year-o l d woman is eva l ua ted prior to d isc h a rge . S h e and she does not recall when she last had her blood pressure
h a s ischemic cardiomyopa t hy a n d w a s a d m i t ted to t h e h os­ checked.
p i t a l 5 days ago for worsen i n g sym p toms of heart fa i l ure. On physical examination, blood pressure is 156/96 mm
S h e ski pped t a k i ng her d i u re t ics dur i n g a rece n t busi ness Hg and pulse rate is 80/min. BM! is 31. There is an apical S_1,
t ri p . Today. she f'eels \Ne l l a n d is able t o walk around t h e but no murmurs are detected. Pulses are normal through­
ward twice w i t h o u t a n y sym p toms . out. The remainder of the examination is unremarkable.
'Jl1is was her first hospi t a l iza t i o n in 3 years. a l t hough Serum creatinine level, plasma glucose level, and
s h e has skipped her d i u re t ics d ur i n g ot her busi ness tri ps urinalysis all are normal. An ambulatory blood pres­
d u ri n g t h is t i m e w i t h o u t a p pa re n t i l l e f fec t . S h e had a n sure monitor demonstrates an average blood pressure of
i m p l a n t a b l e card iove rt e r-defibri l l a t o r p l aced 3 years ago . 155/92 mm Hg.
An echocardiogram 1 m o n t h ago s h owed a l e f'l ven t r i c u l a r
ejection fract ion of' J S 'Y., (stable fo r t he p a s t 6 yc,: 11-s) . Medica ­ Which of the following is the most appropriate treatment?

t i ons a re cap t o p r i l . me toprolol succ i n a te. d igox i n , furose­


(A) Start labetalol
m i d e , a n d spironolact one .
(B) Start lisinopril
On physical exa m i n a t i o n . blood pressure is 1 1 0 17 2
m m Hg. p u l se ra t e i s 5 6/ m i n . a n d respira t i o n ra te i s 1 4 / m i n . (C) Start losartan
S h e has no j u g u l a r venous d i s t e n t i o n a n d no S"' Lungs a re (D) No intervention is necessary
clear. a n d s h e has no edema .
Electrocardiogram s h ows s i n u s rhy t h m . a Q R S i n t e r­
val o f' 90 m s . a n d Q waves i n V1 t h rough V , . T here a re n o Item 3 7
c h a nges com p.i red wi t h t h e adm ission electrocardiogram A 28-year-old pregnant woman i s evaluated for a cardiac
recorded 3 years ago. murmur identified on examination by her obstetrician. She

1 30
Self-Assessment Test

is asymptomatic. She is in her 24th week of pregnancy. An exercise electrocardiographic treadmill test is per­
Medical history is unremarkable, and there is no family formed. The patient is able to exercise for 4 minutes to a
history of heart disease. She takes prenatal vitamins and no heart rate of 82% of the maximum predicted and energy
other medications. expenditure of 4 metabolic equivalents until the study is
On physical examination, she is afebrile, blood pres­ discontinued because of fatigue. Testing did not reproduce
sure is 120/70 mm Hg, pulse rate is 86/min, and respiration her symptoms, and there were no significant electrocardio­
rate is 18/min. Cardiac examination reveals a midsystolic graphic changes with exercise.
ejection click followed by a grade 3/6 early peaking, cre­
Which of the following is the most appropriate next step in
scendo-decrescendo murmur at the right upper sternal bor­
management?
der. The murmur radiates toward the apex and decreases
slightly with the Valsalva maneuver. No diastolic murmur (A) Cardiac catheterization
is heard. (B) Pharmacologic stress testing
Which of the following is the most likely diagnosis? (C) Switch lisinopril to metoprolol
(A) Bicuspid aortic valve (D) Clinical observation
(B) Hypertrophic obstructive cardiomyopathy
(C) Mammary souffie
Item 4 0
(D) Mitra! valve prolapse
A 72-year-old woman i s evaluated fo r sharp chest pain that
(E) Physiologic murmur of pregnancy occurs randomly. She walks 3 to 4 miles daily, and her
symptoms have never occurred with exertion. She has never
smoked. Medical history is significant for hypertension, type
Item 3 8 2 diabetes mellitus, and hyperlipidemia. Medications are
A 64-year-old man i s evaluated for chest discomfort that low-dose aspirin, metformin, lisinopril, and simvastatin
he has had over the past year. It does not always occur (10 mg/d). She has no known drug allergies.
with exercise. There is no associated nausea or diaphoresis. On physical examination, blood pressure is 122/76
Medical history is significant for hypertension and hyperlip­ mm Hg, pulse rate is 76/min, and respiration rate is 12/min.
idemia. Medications are metoprolol, hydrochlorothiazide, Cardiac examination shows a normal S1 and S2 ; there is no
and lisinopril. S3 , S4, murmur, or rub. The remainder of the examination
On physical examination, vital signs are normal, as is is normal.
the remainder of the physical examination. Electrocardio­ Laboratory findings include a serum total cholesterol
gram is normal. level of 200 mg/dL (S.18 mmol/L), LDL cholesterol level
The patient is scheduled for exercise stress testing. of 126 mg/dL (3.26 mmol/L), and HDL cholesterol level of
SO mg/dL (1.30 mmol/L) .
Which of the following should be done prior to the stress
An exercise treadmill test is administered for 8 min­
test?
utes, 40 seconds. There are no electrocardiogram changes
(A) Stop hydrochlorothiazide at rest or with exercise. She does not have chest pain during
(B) Stop lisinopril exercise or recovery.
(C) Stop metoprolol Which of the following is the most appropriate manage­
(D) Stop all medications ment?

(A) Increase simvastatin to 80 mg/d


(B) Continue current therapy
Item 3 9
(C) Start atorvastatin, discontinue simvastatin
A 43-year-old woman is evaluated for a 1-month history of
(D) Start clopidogrel
chest discomfort. She states that she experiences a vague
pressure-like sensation in her chest that occurs intermit­
tently, with each episode lasting less than S minutes. She
has had approximately two episodes each week, and several Item 4 1
have seemed to be associated with exertion but also appear A 57-year-old man is evaluated i n follow-up 1 month after
to have resolved after taking antacids. Her medical history being diagnosed with peripheral arterial disease. He ini­
is significant for hypertension. Her only medication is lisin­ tially presented with left calf pain and was diagnosed by an
opril. She is a current smoker with a 15-pack-year history. abnormal ankle-brachia! index. An exercise rehabilitation
Family history is negative for coronary artery disease. program was completed, but he continues to feel lower
On physical examination, the patient is afebrile, blood extremity discomfort in his left leg that limits his walking
pressure is 132/78 mm Hg, pulse rate is 85/min, and respi­ ability. He is a former smoker who quit 1 year ago. His
ration rate is 12/min. BMI is 32. Cardiopulmonary examina­ medical history is otherwise notable for hypertension and
tion is unremarkable, as is the remainder of her physical dyslipidemia. Medications are enalapril, amlodipine, rosu­
examination. vastatin, and aspirin.
An electrocardiogram shows sinus rhythm, normal PR On physical examination, blood pressure is 124/72 mm
and QRS intervals, and no ST-segment or T-wave abnor­ Hg, pulse rate is 78/min and regular, and respiration rate is
malities or Q waves. 16/min. Peripheral examination reveals a left femoral bruit.

1 31
Self-Assessment Test

Item 43
c:J
The left lower extremity is warm and without tenderness
or skin changes. The remainder of the examination is unre­ A 68-yea r-old m a n is eval ua ted i n t h e e mergency depart­
markable. ment l'or a 24 -hour history of persisten t chest pai n. H e had
Which of the following is the most appropriate addition to a non -ST-elevation myocardial i n fa rction 1 week ago t ha t
his current therapy? was managed medically w i t h complete symptom recovery.
Yesterday, he developed recurrent chest pain that dilTers
(A) �-Blocker from his previous a ngina pai n. TI1e pain is constant but
(B) Cilostazol exacerbated when lea n i ng for.Nard a n d not associated with
(C) Clopidogrel o ther symptoms. Medications are low-dose aspi rin. clopido­
(D) Warfarin grel , metoprolol. and a torvastat in .
On physical exami nation. vital signs are normal. There
is no j ugular venous distention . TI1e lungs are clear to a us­
cultat ion. S 1 and S 2 are norma l , and there is no S,1 or S,1 • A

c:J A 75-year-old woman is eval ua ted i n t he hosp i t a l 4 hours


Item 42 two-componen t friction rub is presen t at the left lower s ter­
nal border, and a grade 2/6 holosystolic m urmur is heard
a fter onset of c hest pain with A nd i ngs or a n ST-eleva ­ al t h e apex . The remai nder of t h e physical exa m i nation is
t ion myoca rd ial i n farction . S h e was taken emergen t ly to unremarkable.
t h e ca theterization laboratory and u nderwen t e mergency Electrocardiogram shows di ffuse. concave u pward
percuta neous coronary i n terven tion for a total ly occluded ST-segment eleva l ions and PR-segme n t depression mosL
vessel. H e r post-in terven t i o n ventricu l ogra m demon ­ prominent in leads v i t h rough v ,,.
s t ra ted a l e ft ven tricular ejection fraction of 30%. One
Which of the following is the most appropriate primary
hour after the proced u re. she developed a n acute arrhy t h ­
treatment?
m i a . Medications are aspirin . metopro l o l . a to rvastat in . and
clopidogre l . (A) 1-1 igh-dose aspirin
On physical examination, the patient is a febrile. blood (B) Ibuprofen
pressure is 1 00/60 mm Hg, pulse rate is 92/ m i n . and respi ­
(C) N i troglycer i n
rat ion rat e is 1 21 m i n. B M I is 25 . Neck exa m ination demon­
stra tes cannon a waves. Cardiac exa m i n a tion demonstra tes ( D) Precl n isone
regular rhyt h m w i t h a variable S 1 • Lungs are clear to aus­
cultat ion .
c:J
Electrocardiogram is shown . I t e m 44
A 56 -year-old m a n is admit ted to t h e coronary care u n i t
Which o f the following is t h e most appropriate manage­ with recen t-onset subslernal ches t discom fort and dyspnea.
ment? Upon adm issi o n . he was given aspiri n . t icagrelor. metopro-
(A) Amiodarone lol. and enoxapari n . He has hyperlipidem i a . Regular medi­
(B) Cardioversion cations are low-dose aspirin and si mvastatin .
On p hysica l exa m i na t io n . tempera t ure is 36 .5 °C
(C) I m plantable cardioverler-deAbril lator (97.7 ° F) . blood pressure is 134/82 m m Hg. and pulse rate is
(D) Lidocaine 82/m i n . Cardiac and p u l monary examina tions are norm a l .
( E) No i n tervent ion as is t h e remainder of t h e exa m inat ion.

H--0--/�f\r\��
� ���
���
ITEM 42

1 32
Self-Assessment Test

Cl shows normal sinus rhyt hm and heart rate of' 80/ m in . There
Seru 111 t ropon in level is elevated . Elect roca rel iogram rate is 16/min. Oxygen saturation breathing ambient air is 98%.
There is no jugular venous distention. Lungs are clear. Cardiac
CONT.
are nonspec i fic ST-T wave abnorma lities but no ST-segment examination reveals a regular rate and a grade 3/6 apical holo­
eleva tion or depression. systolic murmur that radiates to the axilla. There is no lower
Cardiac ca lheterization is sign i f ican t for preserved left extremity edema.
ventricular systolic fu nction and two-vessel coronary artery Electrocardiogram shows normal sinus rhythm and
disease. Percuta neous coronary i n tervention of t h e micl left evidence of left atrial enlargement. Echocardiogram shows
an terior clescencling a rtery ancl p rox i m a l righ t coronary severe eccentric mitral regurgitation with marked calcifi­
a rt e ry is performed w i t h p lacement of drug - eluting stents. cation of the valve leaflets; left ventricular systolic function
is normal.
In addition to continuing aspirin, which of the fo llowing
is the most appropriate management of this patient's anti ­ Which of the following is the most appropriate treatment?
platelet regimen?
(A) Bioprosthetic mitral valve replacement
(A) Continue t icagrelor f'or 30 days (B) Mechanical mitral valve replacement
(B) Continue t icagrelor f'or l yea r (C) Oral vasodilator therapy
( C) Con t i nue t icagrelor inclefi n i lely (D) Percutaneous mitral valvuloplasty
(0) Stop ticagrelor. s tart clopiclogrel

Item 47
Item 45 A 7S-year-old woman i s evaluated for a 3-month history
A 42-year-old woman is evaluated for a routine outpatient of progressive exertional dyspnea and decreased exercise
medical assessment. She was diagnosed with a ventricular tolerance. She does not have chest pain. She has a history
septa! defect at age 6 months. Evaluation was performed of hypertension and COPD. She has a SS-pack-year tobacco
early in life and observation was recommended. She has no use history but quit 3 years ago. She has no history of alcohol
symptoms and is taking no medications. use. Medications are lisinopril, tiotropium, and as-needed
On physical examination, blood pressure is 100/60 albuterol.
mm Hg, pulse rate is 70/min and regular, and respiration On physical examination, blood pressure is 136/78 mm
rate is lS/min. BM! is 28. The estimated central venous Hg, pulse rate is 88/min, and respiration rate is 16/min. The
pressure is normal. The apical impulse is normal. There is central venous pressure is estimated at 9 cm H20. There are
no parasternal impulse. S1 and S2 are masked by a loud holo­ decreased breath sounds throughout both lung fields, but
systolic murmur noted at the left lower sternal border. The no crackles are detected. An S4 is heard on cardiac examina­
rest of the examination is unremarkable. tion. There is trace bilateral lower extremity edema.
An electrocardiogram is normal. The heart size is nor­ Laboratory studies, including thyroid function studies,
mal on the chest radiograph. An echocardiogram demon­ are normal. Electrocardiogram is shown (see top of next
strates normal left ventricular size and function with an page) . A chest radiograph shows changes consistent with
ejection fraction of 60%. A membranous ventricular septa! COPD, mild vascular congestion, and blunting of the costo­
defect is noted with a small left-to-right shunt. The right phrenic angles bilaterally. Echocardiogram shows a left ven­
heart chambers and valve function are normal. The esti­ tricular ejection fraction of30% and an a kinetic anterior wall.
mated pulmonary artery pressure is normal. The patient is started on furosemide.

Which of the following is the most appropriate manage­ Which of the following is the most appropriate diagnostic
test to perform next?
ment?

(A) Cardiac catheterization (A) Cardiac magnetic resonance (CMR) imaging


(B) Cardiac magnetic resonance (CMR) imaging (B) Coronary artery calcium scoring
(C) Endocarditis prophylaxis (C) Myocardial perfusion imaging stress test
(D) Follow-up in 3 to S years (D) Endomyocardial biopsy
(E) Stress testing to determine exercise capacity
Item 48
A S9-year-old woman is evaluated for continued subster­
I t e m 46 nal chest pain. She presented with exertional chest pain
A 72-year-old woman is evaluated in the hospital for a 6 months ago that occurred with minimal ambulation. She
3-month history of increasing shortness of breath. Although was evaluated with a stress nuclear medicine myocardial
she had previously been physically active, her ambulation is perfusion study that showed no ST-segment changes but a
now limited to about so feet because of shortness of breath. small area of inducible ischemia in the lateral area of the left
Medical history is significant for rheumatic fever as a child, ventricle and an ejection fraction of 45%. She was initially
diverticulosis with gastrointestinal bleeding that required treated medically but has continued to have chest pain
blood transfusions, hypertension, and hyperlipidemia. with exertion despite the addition of multiple antiangi­
Medications are chlorthalidone and atorvastatin. nal agents. Medical history is significant for hypertension,
On physical examination, the patient is afebrile, blood hypercholesterolemia, and type 2 diabetes mellitus. She
pressure is 140/70 mm Hg, pulse rate is 83/min, and respiratory has a 30-pack-year smoking history but quit 1 year ago.

1 33
Self-Assessment Test

.
n�� I - . . _ aVR _ • -

n� �11 - . . -- aVL
• - - ' ·-

n ·- :------
Ill
- .iv------�--"'l!--
.

- .-,11 aVF
-

�--:-- � �
_
_ f\ _ r (I- - · 1 -A- A
-rN . _ .. :-:-: � �A�
V6 . _
-

n_�0· __
(L,\/-�
:� \/ J_ L �
__
�·
- IJ0\/(L,·
-- J_ L(L,11���-
� ____
rt;,�, -t-
11 \L\� �-=-=--
__ -�
- � _� ___

ITEM 47

Medications are aspirin, lisinopril, simvastatin, metformin, Which of the following is the most appropriate manage­
metoprolol, and long-acting nitroglycerin. ment?
On physical examination, the patient is afebrile, blood
(A) Mitra! valve repair
pressure is 132/72 mm Hg, pulse rate is 68/min, and respira­
tion rate is 16/min. BM! is 28. The remainder of her physical (B) Repeat TIE in 6 months
examination is normal. (C) Start lisinopril
Electrocardiogram is unchanged from the time of her (D) Transesophageal echocardiography
stress test.

Which of the following is the most appropriate next step in


Item 5 0
management?
A 35 -year-old man i s evaluated for a 6 -month history
(A) Cardiac catheterization of intermittent palpitations. His symptoms occur about
(B) CT angiography once a week with no consistent pattern. He occasion­
(C) Dobutamine stress echocardiography ally becomes lightheaded with the palpitations but
has no syncope. H e exercises three times weekly and
(D) Continued medical therapy
does not notice symptoms during exercise. He has a
history of migraine, for which he takes naproxen and
Item 49 sumatriptan as needed. He has no family history of
sudden cardiac death.
A 26-year-old woman is evaluated during a visit to estab­ On physical examination, the patient is afebrile, blood
lish care. She has noted shortness of breath for the past pressure is 138/68 mm Hg, pulse rate is 75/min, and respira­
18 months that is progressively worsening. She has no sig­ tion rate is 16/min. BMI is 24. Cardiac examination and the
nificant medical history. She is active and exercises regu­ remainder of the examination are unremarkable.
larly. She takes no medications. Electrocardiogram is normal.
On physical examination, blood pressure is 115/70 mm
Hg and pulse rate is 62/min. BM! is 45. Cardiac examination Which of the following is the most appropriate testing
reveals a midsystolic click with a grade 3/6 late systolic mur­ option?
mur heard over the apex and radiating toward the axilla.
(A) 30- Day event recorder
Transthoracic echocardiogram (TIE) shows moderate
to severe mitral regurgitation with marked prolapse of the (B) Echocardiogram
anterior leaflet, normal left ventricular systolic function (C) Exercise stress test
with an ejection fraction of 55%, and normal chamber sizes; (D) 24-Hour continuous ambulatory electrocardiographic
the regurgitant jet is not well visualized. monitor

1 34
Self-Assessment Test

c::J I t e m 5 1 Which ofthe following is the most appropriate management?


A 76-year-old man is admitted to the hospital for recur­ (A) Initiate atorvastatin
rent palpitations and dyspnea that began 4 days ago. He (B) Initiate cilostazol
has hypertension and coronary artery disease. which was (C) Initiate warfarin
treated with percutaneous intervention 8 years ago. Medi­ (D) Obtain CT angiography
cations are aspirin, atorvastatin, and lisinopril.
On physical examination, temperature is 36.8 °C (E) Obtain segmental limb pressures
(98.2 °F). and blood pressure is 115/62 mm Hg. 111e resting
heart rate is 110/min with intermittent irregularity. The esti­
mated central venous pressure is not elevated. S1 and S2 are Item 5 3 c::J
unremarkable. The lung fields are clear, and the extremities A 58-year-old man is ready for hospital discharge following
are without edema. a non-ST-elevation myocardial infarction. He was treated
An electrocardiogram obtained after the physical with ticagrelor and underwent percutaneous coronary
examination is shown. intervention with drug-eluting stent implantation. He has
remained free of chest pain since admission to the hospital.
Which of the following is the most appropriate next step The patient's medical history is significant for hyper­
in treatment? tension and hyperlipidemia. Medications are low-dose
(A) Emergent cardioversion aspirin, ticagrelor, metoprolol, lisinopril, atorvastatin. and
(B) Initiate �-blocker therapy sublingual nitroglycerin as needed.
(C) Intravenous amiodarone On physical examination, blood pressure is 124/78 mm
Hg and pulse rate is 54/min. BM! is 26. Lungs are clear to
(D) Intravenous procainamide auscultation. Cardiac examination shows a normal S1 and
S2 ; there is no S3 , S,l ' murmur. or rub. The remainder of the

Item 5 2
examination is normal. His left ventricular systolic function
is normal. as measured on transthoracic echocardiography
A 74-year-old woman is evaluated during a routine exam­ on the day after hospital admission.
ination. Her medical history is significant for hypertension
and obesity. She is a former smoker, stopping 5 years ago. Which of the following is the most appropriate adjustment
Medications are amlodipine, lisinopril, and aspirin. to his discharge medications?
On physical examination, she is afebrile, blood pres­ (A) Add diltiazem
sure is 136/78 mm Hg, pulse rate is 68/min, and respiration
rate is 1 5/min. BMI is 32.
The lungs are clear to auscultation,
(B) Discontinue ticagrelor, start clopidogrel
and no murmurs are noted. A bruit is heard over the left (C) Increase dose of metoprolol
femoral artery. (D) Start eplerenone
The right ankle-brachia! index is 1.2
and the left is 0.81. (E) Make no changes to his medications

II
f t . I
l-

Ill

1 :�1
t
I

· 11 t ! ,_ '

ITEM 5 1

1 35
Self-Assessment Test

Item 5 4 Item 5 6
A 54-year-old m a n is evaluated after a recent diagnosis A 77-year-old man with a 5-year history o f idiopathic
of systolic heart fa ilure. He initially presented with a cardiomyopathy is evaluated for progressive exertional
4 - month history of exertional dyspnea. He has not had fatigue and dyspnea. He has recently stopped carrying
prior regular medical care and had no known medical groceries in from the car because of his exertional dys­
problems. His blood pressure was 164/96 mm Hg at the pnea. He had an implantable cardioverter-defibrillator
time of diagnosis. Echocardiography showed evidence placed 3 years ago. Medical history is also significant
of hypertensive cardiomyopathy with no regional wall for hypertension. Medications are lisinopril, 40 mgld;
motion abnormalities and a left ventricular ejection metoprolol succinate, 25 mg/d; furosemide, 40 mg/d; and
fraction of 30%. Cardiac stress testing showed no evi­ spironolactone, 25 mg/d.
dence of ischemia, and he exercised for 7 minutes and On physical examination, blood pressure is 94/60 mm
1 0 seconds to a peak heart rate of 142/min. He was Hg and pulse rate is 70/min. Estimated central venous pres­
started on lisinopril and is now able to walk 6 blocks sure is 5 cm H20. There is no edema.
before experiencing dyspnea. Serum electrolyte levels and kidney function are nor­
On physical examination, blood pressure is 110/72 mal. Electrocardiogram shows normal sinus rhythm, a PR
mm Hg, pulse rate is 84/min, and respiration rate is 14/min. interval of 210 ms, QRS duration of 160 ms, and a new left
Estimated central venous pressure is 6 cm H p . The lungs bundle branch block. His left ventricular ejection fraction 3
are clear. Cardiac examination shows the point of maximal months ago was 25%.
impulse is shifted to the left anterior axillary line. There is no
Which of the following is the most appropriate next step in
lower extremity edema.
management?
Laboratory studies, including electrolytes and kidney
function, are normal. (A) Cardiac resynchronization therapy
Which ofthe following medications is the most appropriate (B) Dobutamine therapy
addition to this patient's treatment regimen? (C) Increase furosemide dose
(A) Amlodipine (D) Left ventricular assist device placement
(B) Carvedilol
(C) Furosemide
Item 5 7
(D) Spironolactone
(E) No added therapy A 53-year-old man is evaluated for a 6-week history of
epigastric and chest discomfort. The onset of the pain has
a variable relationship to stress and exercise and spicy
food. The discomfort is relieved at times with antacids and
Cl IAt e63m-year-o
55
l d m a n is hosp i t a l ized fo l l owing a rece n t
with rest. He has hypertension and is a former smoker
(quit 2 years ago) . Medications are lisinopril and hydro­
i n ferior myoca rd i a l i n fa rc t i o n . Perc u t a n eous coro n a ry chlorothiazide.
i n t e rve n t i o n was n o t su ccessfu l . A n echocardiogram On physical examination, he is afebrile, blood pres­
o b t a i n ed fo l l owi ng t h e a t t e m pted coro n a ry i n t erve n t io n sure is 140/92 mm Hg, pulse rate is 78/min, and respiration
demons t ra ted a left ven t ri c u l a r eject i o n fra c t i o n o f 55% rate is 12/min. BM! is 29. Funduscopic examination is nor­
w i t h i n ferior w a l l a k i n es i s a n d a d i l a ted and dysfu n c­ mal. Results of the cardiac examination are normal, with
t i o n a l righ t vent ric le . On t h e t h i rd clay a ft e r a d m issi o n , no S3 or S4.
t h e pat i e n t develops p rog ress ive o xygen desa t ur a t i o n Electrocardiogram is shown (see top of next page) .
a n d clyspnea despi te o xygen t he ra py w h i le u prigh t t h a t
i m proves w h e n s u p i n e . Which of the following is the most appropriate diagnostic
On physica l exa m i n a tio n . h i s blood pressure is 90/70 test to perform next?
m m Hg. pu lse ra te is 86 / m i n a n d reg u l a r, and res p i ra ­ (A) Adenosine myocardial perfusion study
t ion ra te is 25 / m i n . Est i m a ted cen t ra l venous pressure is
(B) Cardiac magnetic resonance (CMR) imaging
markedly eleva t ed . The apical i m p u lse is norm a l ; t here is
a para s t e rn a l i m pu l se a t the left s t e rn a l border. The hea rt (C) CT angiography
sounds are d i s t a n t . There is a soft h o l osysto l i c m u r m ur a t (D) Dobutamine stress echocardiography
t he left s t e rn a l border t ha t i ncreases w i t h i nspira t i o n . The (E) Exercise stress test
oxygen saturation is 90% on oxygen acl m i nisterecl by mask
w h i le t h e pat ien t is s i t t i n g a n d i m p roves to 94% on re t u rn
t o h i s bed . ·n1e rem a i n der 0 1· t h e physical e x a m i n a t i o n
is norm a I .
Item 5 8
A 58-year-old m a n is eva luated for a 2-wee k history o f
CJ
Which o f t h e following i s the most likely diagnosis? mala ise and subjective fever. Medical h istory is sign i ficant
(A) Patent fora men ova le with right - to-lert s h u n t for wel l -con t rol led type 2 d i a betes mel l i tus a n d sinus node
dysfunction . A d u a l -cha m ber pacema ker was i m p l a n ted
M i t ra! regurgitat ion
( 13)
5 years ago. He does not have clyspnea or weig h t loss . None
(C) Severe left ventricular systolic dysfunction of h is fa m i ly mem bers have had a recen t v i ra l or febri le
(D) Ven tricu lar septa I defect i l lness.

1 36
Self-Assessment Test

III VF
a V3 V6

nn---r-Tl��r-1--r
flLi��-J�J.�J��L��_j�Lr
ll

nJ � V5

ITEM 57

Cl ( 10 0.4 °F) , blood pressure is 1 32 1 75 mm Hg. pulse ra te is


O n physical exam i n a t i o n , tempera l u re i s 38 . 0 °C Which of the following is the most appropriate manage­
ment?
CONT.
6 2 / m i n . ancl respi ra t ion rate i s 1 6/ m i n . BM! is 3 2 . His con­
(A) Aortic valve replacement
j u nc t ivae. o ro p h a ry n x , a n d tympa n i c m e m b ra n es a re
normal in appearance. There is no head. neck, axi llary , or (B) Clinical reassessment in 1 year
i ng u i n a l lymphaclenopat hy. The s k i n overlying Lhe patien l 's (C) Endocarditis prophylaxis
pacemaker pocket is mi Ici ly ery t hemalous a ncl warm . TI1ere (D) Start an ACE inhibitor
is no Ouctuance, b u t t here is m i l e! Lenderness LO pa l patio n . (E) Start a calcium channel blocker
The precord ial cadence is regu lar. a nd t here is no evidence
of cardiopu l monary congestion . Res u l ts o f' a complete blood
co u n t with d i fferential a nd a n e ry t h rocyle sed i m e n t a t i on
ra te are pendi ng. Item 60

Which of the fo ll owing is the most appropria te manage ­ A 26-year-old woman with a mechanical mitral valve
me nt ? prosthesis visits to discuss anticoagulation management
during pregnancy. Her last menstrual period was 6 weeks
(A) B l ood c u l tures ago and her pregnancy was confirmed by laboratory test­
(B) Pacemaker pocket aspira t i o n ing in the office. Her mitral valve was replaced 5 years ago.
(C) U l t rasonography of t h e pacemaker pocket Her medications are low-dose aspirin, metoprolol, and
warfarin (4 mg/d) .
(D) Repeat eva l u a L io n in 1 week
On physical examination, vital signs are normal. Car­
diac auscultation demonstrates a normal mechanical S 1 •
There are no murmurs or added sounds. Her INR is 2.6.
Item 5 9
A 47-year-old man is evaluated during a routine examina­ Which of the following anticoagulation regimens will pro­
tion. He has no symptoms. Medical history is significant for vide the greatest protection against thromboembolism
a bicuspid aortic valve. He is not taking any medications. during her pregnancy?
On physical examination, he is afebrile, blood pressure (A) Continue warfarin and aspirin
is 130170 mm Hg, pulse rate is 56/min, and respiration rate
is 15/min. Cardiac examination reveals a grade 1/6 diastolic (B) Stop warfarin and start dabigatran
murmur at the left lower sternal border. (C) Stop warfarin and start subcutaneous fixed-dose
Echocardiogram shows a bicuspid aortic valve with unfractionated heparin
moderate aortic regurgitation, normal left ventricular sys­ (D) Stop warfarin and start weight-based low-molecular­
tolic function, and normal left ventricular chamber size. weight heparin

1 37
Self-Assessment Test

Cl A 72-year-old woman i s evaluated i n the emergency depart­


Item 6 1 athy (HCM ) . The patient has had no symptoms, including
with physical activity such as golfing or playing tennis.
Medical history is unremarkable, and a review of family
ment for progressive chest pain that began 2 hours ago. She
history is negative for other relatives with HCM, sudden car­
has not had recent surgery or stroke. She takes amlodipine
diac death, or tachyarrhythmias. He takes no medications.
for hypertension.
On physical examination. blood pressure is 154 /88 mm Fi ndi ngs of a comprehensive physical examination are
Hg. and pulse rate is 88/ min . Cardiac and pul monary exam­ unremarkable.
inations are normal. An electrocardiogram and echocardiogram are normal,
I nitial electrocardiogram shows 2-mm ST-segment ele­ with no evidence of HCM.
vation in leads V 1 t h rough V5 with reciprocal ST-segment When should this patient next b e screened for HCM?
depression in leads l l . l l l. and aVF. Chest radiograph shows
no cardiomegaly and no evidence of pulmonary edema. (A) In 6 months
111e patient is given aspirin. clopidogrel, unfract ionated (B) In 1 to 2 years
hepari n, and a P-blocker. Because the nearest hospital with ( C) In 5 years
primary percutaneous coronary intervention capabi l ities is (D) No further screening is necessary
more than 120 m inutes away, she is also given a bolus dose
of tenecteplase.
Thirty minutes l ater. the patient's blood pressure has
dropped to 85/58 mm Hg. Her chest pain persists, and she
rates the pain as 8 out of 10 . Pulmonary crackles are aus­
Item 63
A 73-year-old m a n is evaluated i n the emergency depart­
Cl
cultated to the scapulae. Electrocardiogram shows 3-mm ment for chest pain of 2 hours· duration . He is bradycardic.
ST-segment elevation i n leads V1 through Vs with reciprocal He does not have dyspnea, lightheadedness, or loss of con­
ST-segment depression in leads I I , I I I , and a VF. sciousness. Medical history is significan t for type 2 diabetes
mellitus. hypertension, and hyperlipidemia. Medications
Which ofthe following is the most appropriate management? are aspiri n , metformin. lisinopril , and hydrochlorothiazide.
On physical examination. the patient is afebrile, blood
(A) Continued medica l therapy
pressure is 120/60 mm Hg, pulse rate is 47 /m i n , and respira­
(B) Glycoprotein Ilbi llla inh ibitor t ion rate is 12/m i n. BM! is 34. He is warm and well-perfused.
(C) Repeat tenecteplase Trace bibasil ar crackles are heard in the l ungs.
(D) Transfer for emergency percutaneous coronary i n ter­ 111e electrocardiogram is shown .
vention
Which of the foIJowing is the most appropriate treatment?

Item 6 2 (A) Aminophylline

A 56-year-old man is being evaluated after his 18-year-old (B) Low-dose dopamine
son had a syncopal episode during a high school basketball (C) Percutaneous coronary intervention
game and was diagnosed with hypertrophic cardiomyop- (D) Temporary pacing

aVR

l
i �
l

.v V2

l
I ,
I . '

--�---'''�' '
I j ; �
ITEM 63

1 38
363

Self-Assessment Test

CJ I t e m 64 of right-sided claudica tion . T h ree days ago he developed


. . severe rest pain in t h e right foot t h a t is starting to subside.
A S3-year-old woman with Eisenmenger syndrome related
to a ventricular sep t a ! defect is evaluated i n the emergency I-le has hypertension and a 1 0-year h istory of type 2 dia­
department for fever and c h i l ls over the past 1 2 hours. She betes mel l i tus. He has a SS- pack-year history of cigarette
has dysmia and urinary frequency. Her only outpatient smoking but stopped 2 years ago. Medications are aspirin.
medication is bosentan. metformi n , losartan, and am lodipine.
On physical exami nation, temperature is 38. 2 °C On physical exa mination, vital signs a re stable. The
rig h t foot is cool and pale with slow capil la ry refi l l i n t h e
(100.8 °F) , blood pressure is U0 /70 mm Hg, pulse rate is
n a i l beds. The dorsa l i s pedis and posterior tibialis pulses
86/min and regular. and respiration rate is 20/rnin. Oxygen
saturation on ambient air is 8S%. B M I is 24 . Digital clubbing a re not pa l pabl e. A rterial Doppler u l t rasound signal over
and cyanosis are evident. The estimated central venous pres­ the dorsalis pedis is present but markedly d i m inished.
sure is elevated with a p ro minent a wave. The apical impulse Sensation of light touch is present but decreased over the
dorsum of' the right foot .
is normal. A prominent parasternal i mpulse is present at the
left sternal border. Tlle S1 is normal : the S.,. is loud. A soft holo­ Laboratory studies are significant for normal complete
� blood cou n t . electrolytes. and kid ney function. Electrocar­
systolic murmur is heard at the left lowet sternal border. M i ld
right flank tenderness is noted. diogram demonstrates sinus rhyt h m.
A urinary Gram stain is positive: urine and blood cul­ A continuous heparin i n fusion is started.
ture resul ts are pending. The hemoglobin level is 18 g/d L Which of the following is the most appropriate next step
(180 g! L) , hema tocrit i s SS%, a n d t he leukocyte cou n t is in management?
20.000/µL (20 x 109/L) .
(A) Ca t het er -di rect ed t hrombolytic therapy
In addition to intravenous antibiotic administration, which
( B) Emergent surgica l amputa tion
of t he following is the most appropriate management?
(C) I n itiation of warfarin
(A) A i r fi lters on i n t ravenous l ines (D) Urgent a ngiography
( B ) Oxygen by close-fit t i ng mask
(C) Phlebotomy
(D) Transthoracic echocardiograrn Item 6 7
A 72-year-old man is evaluated during a routine examina­
tion. He has no symptoms or significant medical history.
Item 65 He is active and exercises regularly. He does not take any
A 62-year-old man i s evaluated during a routine visit. H e medications.
i s asymptomatic and walks 1 mile most days o f the week. On physical examination, blood pressure is 13S/70 mm
Medical history is significant for aortic stenosis, type 2 dia­ Hg, pulse rate is 82/min, and respiration rate is 17/min.
betes mellitus, hypertension, and hyperlipidemia. Medi­ Cardiac examination reveals a grade 3/6 apical holosystolic
cations are aspirin, metformin, lisinopril, metoprolol, and murmur.
rosuvastatin. Echocardiogram shows severe mitral regurgitation and
On physical examination, the patient is afebrile, blood a left ventricular ejection fraction of 4S% without evidence
pressure is 130/66 mm Hg, pulse rate is 68/min, and respi­ of regional wall motion abnormalities.
ration rate is 14/min. BMI is 29. Cardiac examination reveals Which of the following is the most appropriate manage­
a grade 2/6 early-peaking systolic murmur at the cardiac ment?
base. Carotid upstrokes are normal. The remainder of the
examination is unremarkable. (A) Vasodilator therapy
Laboratory studies demonstrate a total serum choles­ (B) Percutaneous mitral balloon valvuloplasty
terol level of lSO mg/dL (3.89 mmol/L) . Electrocardiogram (C) Repeat echocardiogram in 6 months
is within normal limits. Echocardiogram from 1 year ago
(D) Surgical mitral valve repair
shows a peak velocity of 2.0 mis, mean transaortic gradient
of 13 mm Hg, aortic valve area of l.S cm2 , and preserved
ejection fraction.
Item 68
Which of the following is the most appropriate manage­
A 40-year-old man i s evaluated in the emergency department
ment?
for syncope. He was attending a baseball game when he expe­
(A) Echocardiogram rienced a witnessed, abrupt episode of syncope while seated.
(B) Exercise perfusion study He did not experience any prodromal symptoms. He sus­
tained some facial trauma when he struck the railing in front
(C) Exercise stress test
of him. He has experienced near-syncope on several occa­
(D) No additional testing sions in the past. His father died suddenly at age SO years.
The patient takes no medications and has no drug allergies.
On physical examination, the patient is afebrile, blood

Cl IAt e78-year-old
m 66
m a n is evaluated in t h e emergency depart­
pressure is 12S/64 mm Hg, pulse rate is 64/min, and respi­
ration rate is 16/min. BMI is 26. Estimated central venous
ment because of a painful righ t foot. He h as a 1 -year h istory pressure is normal. Cardiac examination shows a regular

1 39
Self-Assessment Test

ITEM 68

rate with normal heart sounds and no murmurs. H e has no (C) Evaluation for left ventricular assist device placement
peripheral edema. (D) Home inotropic therapy
Pertinent laboratory findings include a negative tro­
ponin test, a normal metabolic profile, and normal kidney
function studies. Item 7 0
Electrocardiogram is shown. Echocardiogram demon­
A 7S-year-old woman is evaluated during a follow-up visit
strates no structural heart disease and shows normal left
for recently diagnosed atrial fibrillation that is adequately
ventricular function.
rate controlled on medication. Medical history is significant
Which of the following is the most appropriate management? for hypertension and end-stage kidney disease; she is on
hemodialysis. Medications are metoprolol, digoxin, lisino­
(A) Cardiac magnetic resonance (CMR) imaging pril, and amlodipine. She has not yet been started on stroke
(B) Exercise treadmill stress test prevention therapy.
(C) Implantable cardioverter-defibrillator placement On physical examination, temperature is 3 6 . 8 °C
(D) Tilt-table test (98. 2 ° F) , blood pressure is 120/6S mm Hg, pulse rate is
72/min, and respiration rate is 16/min. BM! is 29. The pre­
cordial cadence is irregularly irregular. There is no evidence
of pulmonary or peripheral congestion.
Item 6 9
A 68-year-old man is evaluated a t a follow-up appointment. Which of the following is the most appropriate treat­
He has a 7-year history of heart failure secondary to isch­ ment?
emic cardiomyopathy. Over the past 6 months, he has had
(A) Apixaban
three hospitalizations for exacerbations of his heart failure.
He currently has exertional dyspnea while getting dressed, (B) Aspirin and clopidogrel
and his maximal activity level is limited to riding to the store (C) Dabigatran
with his wife but staying in the car. Medical history is signif­ (D) Dose-adjusted warfarin
icant for disseminated prostate cancer treated with andro­ (E) Rivaroxaban
gen deprivation therapy. Medications are aspirin, lisinopril,
carvedilol, furosemide, digoxin, spironolactone, rosuvasta­

Cl
tin, and leuprolide. He is stable on his current medications. Item 7 1
On physical examination, blood pressure is 92/60 mm
A 62-year-old woman i s eval uated i n the emergency depart ­
Hg and pulse rate is 80/min. There is no jugular venous dis­ men t for sudden onset of severe chest . u pper abdom ina l .
tention. An S3 is heard on cardiac examination. The legs are
a n d back pai n of 2 h ours' duration . S h e h a s not had s i m i l a r
cool to the touch; there is no edema.
sym ptorns previously a n d notes no o t h e r sym p toms. Med­
Laboratory studies are significant for a serum sodium
ical history is sign i fica n t for hypertension. She is a curren t
level of 132 mEq/L (132 mmol!L) and serum creatinine level
smoker w i t h a SS-pack-year h istory . Her medications are
ofl.8 mg/dL (1S9 µmol/L) .
a rn lodipine and benazepri l .
Which of the following is the most appropriate manage­ O n p hysical exa rn i n a tion . s h e is afebri le. b lood pres­
ment? sure is 1 6S 1 00 m m Hg i n both arms, pu lse rate is 1 02 1111 i n .
and respira t ion ra te is 20/ m i n . Oxygen saturation is 98% on
(A) Add metolazone ambient a i r. Cardiac auscul tation reveals a n S , gallop but
(B) Cardiac transplantation evaluation no m u rrnu rs. Pulmona ry exa m ination is norm a l . Pulses are

1 40
Self-Assessment Test

Cl sym metric a ncl equa I in a l l e x l remi t ics. ·1 he rem a i ncler or l he tral venous pressure is elevated with a prominent a wave.
p hysical examinat ion is unrema rkable. The apical impulse is normal. There is a prominent paraster­
CONT.
Laboratory studies reveal a D-di mer level o f 0 .8 pgi m L nal impulse at the left sternal border. The sl is normal; the s2
(0 . 8 mg/ L) < 111cl a serum crea t i n i n e level o r 2.'l mgtcl L is soft. A grade 4/6 late-peaking systolic murmur is heard at
(2 1 2 �t mol/ L) (baseline is < l mg/d L [ 88.4 p mol / L]) . I n i t ia l the left sternal border and second left intercostal space. An
carcl i <:ic l roponin T level is 0. 4 ng/ m L (0 .4 �tg/ L) . ejection click is not audible.
Elec l rocarcliogram shows Jell ven t ricular hypert ro ­ An echocardiogram demonstrates a dysplastic pulmo­
phy wi t h repol a riza l ion abnorma l i t ies. Chest radiograph nary valve with a peak instantaneous systolic gradient of 62
demonstrates an e n l a rged cmcl iac sil houe l l e. J\ magnetic mm Hg and mean systolic gradient of 45 mm Hg. There is
resona nce a ngiography s tudy demonstra tes aort ic dissec­ moderate pulmonary valve regurgitation. The right ventric­
t ion origin<:i l i ng distal to t he Jen subclavi:rn artery extending ular size and function are normal, but there is right ventricu­
to t he aortoi l iac bifurcat ion (max i m u m d i a meter 63 m m ) . lar hypertrophy. The left heart size and function are normal.
Bila teral renal arteries arise from the fa lse lumen.
Which of the following is the most appropriate manage­
Trea t ment wi t h ana lgesics. a �-blocker. and sodium
ment for this patient?
ni lroprusside is start ed .
(A) Endocarditis prophylaxis
Which of the following is the most appropriate next step
in managem en t? (B) Exercise testing
(C) Pulmonary valve replacement
(/\) Aort ic repa i r
(D) Observation
( B) Coronary angiogra phy

Cl
(C) Con t i nue current med ica l t herapy
(D) I n t ravenous heparin Item 7 3
A 62-year-old m a n i s evaluated in t h e emergency depart ­
ment (ED) f'or a 3- hour h istory or d u l l . substernal chest clis­
Item 7 2 com f'orl. He has type 2 diabetes mel l i t us. hypertension . and
An 18-year-old woman with Noonan syndrome is evaluated clysl i pidemia. H e does not smoke cigare ttes. Medica t ions are
for a heart murmur noted on a sports physical examination. low-close aspi rin. l isinopri l . and pravasta ti n . H i s younger
She is asymptomatic and her medical history is unremark­ sister was diagnosed w i t h coronary artery disease at the age
able. She takes no medications. of 50 years.
On physical examination, blood pressure is 120/70 mm Electroca rdiogram obta ined upon h is a rrival lo t h e ED
Hg, pulse rate is 70/min and regular, and respiration rate is shown . He is a d m i n istered aspirin and sublingual n i t ro­
is 18/min. BM! is 18. The patient is of short stature and has glycerin . H is chest disco m lorl is rel ieved w i t h i n 1 5 m i n utes
hypertelorism, neck webbing, and a low hairline. The cen- or arriva l .

ITEM 73

141
Self-Assessment Test

Cl mm Hg. pulse rate is 62/min and regular. and oxygen satu ­


On physical examination, blood pressure is 1 4 8/86 at the second right intercostal space. The femoral pulses are
easily palpated, and there is no radial artery-to-femoral
CONT. ration is 94% on ambient air. B M l is 28. Cardiac examination artery pulse delay.
is significant only for the presence of a grade 1 /6 holosystolic
Which of the following is the most likely cause of the sys­
murmur at the left lower sternal border. 1l1e murmur does
tolic murmur?
not radiate. Lungs are clear. 1l1e reminder of the examina­
tion is normal. (A) Aortic valve stenosis
Initial serum t roponin I concentration is 0.94 ng/mL (B) Ascending aortic aneurysm
(0. 94 �tg/ L) .
(C) Mitra! valve regurgitation
Portable chest radiograph demonstrates a normal car­
diac sil houette and no evidence of pulmonary edema. (D) Recurrent aortic coarctation

Which of the fo l lo wi ng is the most appropriate initial


management?
Item 7 6
(A) Administer clopidogrel and enoxaparin A SS-year-old man with a 6-month history o f heart failure
(B) Emergent coronary angiography is evaluated during a follow-up appointment. At diagnosis,
(C) Exercise stress testing his left ventricular ejection fraction was 1S%, and he had
moderate mitral and tricuspid regurgitation. Cardiac cathe­
(D) Monitor in the emergency department until a second
terization at that time revealed normal coronary arteries. He
set of cardiac enzyme markers is available
was started on the appropriate medications and is now back
to working at a desk job. He has dyspnea walking up a flight
of stairs. Medical history is also significant for hypertension.
Item 7 4 Medications are enalapril, carvedilol (2S mg twice daily),
A SS-year-old woman i s evaluated for a 3-week history furosemide, and spironolactone.
of progressive shortness of breath. She now has difficulty On physical examination, blood pressure is 100/6S mm
walking up two flights of stairs. Medical h istory is signifi­ Hg, and pulse rate is S6/min. Weight is 72 kg (1S9 lb) . On
cant for asthma and mitral valve prolapse with moderate cardiovascular examination, there is a grade 3/6 holosys­
mitral regurgitation. Her last echocardiogram was 4 months tolic murmur at the apex radiating to axilla and no S3 gallop.
ago and showed mild left atrial enlargement and normal left The remainder of the examination is normal.
ventricular size and thickness, with an ejection fraction of Serum electrolyte levels and kidney function tests are
6S%. Her only medication is an albuterol inhaler as needed. normal. Recent echocardiogram shows a left ventricular
On physical examination, the patient is afebrile, blood ejection fraction of 20% and moderate rnitral regurgitation.
pressure is 132/S6 mm Hg, pulse rate is 88/min, and respira­ Electrocardiogram demonstrates normal sinus rhythm with
tion rate is 16/rnin. BM! is 27. Cardiac examination reveals a a QRS width of lOO ms.
grade 3/6 holosystolic murmur radiating to the axilla. Lungs
Which of the following is the most appropriate manage­
are clear to auscultation.
ment?
Electrocardiogram shows normal sinus rhythm.
Which of the following is the most appropriate diagnostic
(A) Add an angiotensin receptor blocker
test to perform next? (B) Implantable cardioverter-defibrillator placement
(C) I ncrease carvedilol to 37.S mg twice daily
(A) Exercise stress testing
(D) Mitra! valve replacement
(B) 24-Hour continuous ambulatory electrocardiographic
monitoring
(C) Spirometry
Cl
Item 7 7
(D) Transesophageal echocardiogram
A 4S-year-old woman i s evaluated in the emergency depart­
(E) Transthoracic echocardiogram
ment for a 1-week history of dyspnea and fatigue. 1l1e patient
is being treated with systemic chemotherapy for a diagnosis
of breast cancer. She has received two cycles of adjuvant
Item 7 5 chemotherapy with doxornbicin (60 mg/m2) and cyclophos­
A 38-year-old man i s evaluated for a n insurance physical phamide (600 mg/m2) ; her first treatment occurred 3 months
examination. He has a history of aortic coarctation with an ago a nd her most recen t treatment occurred 2 weeks ago. She
end-to-end anastomosis performed at the age of 3 years. has had no chest pain, and medical history is otherwise unre­
He is asymptomatic, works full time, and performs regular markable. She takes no other medications.
exercise without limitation. He takes chlorthalidone once On physical examination, blood pressure is 120/72 m m
daily for blood pressure control. H g and pulse rate is 88/min; B M ! i s 25 . Carotid upstrokes
On physical examination, blood pressure is 120/80 mm are normal. The estimated central venous pressure is 10 cm
Hg, pulse rate is 78/min and regular, and respiration rate is H20. Crackles are present in bot h lung fields. S1 is norma l.
18/min. BM! is 24. The estimated central venous pressure the pulmonic component ofS2 is increased. an S" is present,
is normal. The apical impulse is normal. The S1 and S2 are and there is no S,1• A soft holosystolic murmur at the left
normal. An ejection click is noted at the left lower sternal lower sternal border is audible. Lower extremity edema to
border. A grade 3/6 midpeaking systolic murmur is noted the midshins is present. The mastectomy site is healing well.

1 42
Se lf-Assessment Test

Cl with a normal cardiac silhouette. Echocardiogram shows


A chest radiograph shows infiltrates in both lung fields Transthoracic echocardiogram shows myocardial
hypertrophy (maximal septa! wall thickness, 32 mm) and
CONT.
a left ventricular ejection fraction of 30% . The estimated mild left ventricular outflow tract obstruction at rest (gra­
pulmonary artery systolic pressure is SO mm Hg. No other dient, 31 mm Hg) . On a 24-hour ambulatory electrocardi­
significant echocardiographic findings are present. ography study, a four-beat run of nonsustained ventricular
The patient is started on lisinopril and furosem ide. tachycardia is present.

Which of the following is the most appropriate next step Which of the following is the most appropriate next step in
in the management of her doxorubicin chemotherapy? treatment?

(A) Continue doxorubicin (A) Alcohol septa! ablation


(B) Discontinue doxorubicin (B) �-Blocker therapy
(C) Reduce the dose of doxorubicin (C) Implantable cardioverter-defibrillator
(D) Substitute daunorubicin (D) Surgical myectomy

Item 7 8 Item 80
A 60-year-old man is evaluated fo r increasing shortness A 65-year-old man i s evaluated fo r a routine examination.
of breath. He noticed progressive exertional intolerance He is asymptomatic and is active, walking 2 miles on a
1 month ago. His symptoms have worsened, and he is now treadmill three times a week. He has hypertension and
short of breath with walking mild inclines. He does not dyslipidemia. He has a 15-pack-year smoking history but
have chest pain, orthopnea, paroxysmal nocturnal dyspnea, has not smoked since age 30 years. Current medications are
cough, wheezing, or lower extremity edema. He has a his­ hydrochlorothiazide, atorvastatin, and aspirin.
tory of atrial fibrillation but remains in sinus rhythm after On physical examination, his blood pressure is 134/76
his second catheter ablation procedure for atrial fibrillation mm Hg in the right upper extremity and 146/80 mm Hg in
1 year ago. Medical history also includes hypertension and the left. His pulse rate is 72/min. BM! is 23. He has a grade
hyperlipidemia but is negative for heart failure or left ven­ 2/6 midsystolic murmur heard loudest at his left sternal
tricular dysfunction. Medications are warfarin, metoprolol, border. Abdominal examination reveals a pulsatile mass in
ramipril, and atorvastatin. the epigastrium.
On physical examination, the patient is afebrile, blood An abdominal ultrasound reveals an aneurysm with
pressure is 132/78 mm Hg, pulse rate is 70/min, and res­ a maximum diameter of 4.7 cm not involving the renal
piration rate is 18/min. Pulse oximetry demonstrates 98% arteries.
oxygen saturation on ambient air. BM! is 30. Cardiac rate
Which of the following is the most appropriate manage­
and rhythm are regular. He has bilateral breath sounds but
ment of this patient's abdominal aortic aneurysm?
no wheezes, crackles, or rhonchi. There is no prolongation
of the expiratory phase. (A) Refer for aneurysm repair
The electrocardiogram shows normal sinus rhythm. (B) Repeat abdominal ultrasonography in 6 to 12 months
A plain chest radiograph is normal, and pulmonary func­
(C) Repeat abdominal ultrasonography in 24 to 36
tion tests demonstrate no obstruction. An echocardiogram
months
demonstrates normal left ventricular function with a left
ventricular ejection fraction above 55% and evidence of mild (D) No follow-up management is needed
diastolic dysfunction.

CJ
Which of the following is the most likely cause of this Item 8 1
patient's dyspnea? A 54-year-old man is evaluated in the emergency depart­
ment for an episode of crushing substernal chest pain and
(A) Chronic thromboembolic disease
discomfort that began 30 minutes ago. He is obese and
(B) Intracardiac shunting
currently smokes 1 to 2 packs of cigarettes daily. He has
(C) Phrenic nerve injury dyslipidemia. TI1e patient"s medications are enteric-coated
(D) Pulmonary vein stenosis low-dose aspirin and simvastatin .
On physical examination, he is afebrile, blood pres­
sure is 146/88 mm Hg. pulse rate is 88/min and symmetric
Item 7 9 bilaterally, and respiration rate is 18/ min . B M ! is 32. Cardiac
A 41-year-old man comes t o the office t o discuss man­ examination reveals a normal S, and S 2 and no S 3 : there is
agement of hypertrophic cardiomyopathy (HCM), which an S_, . TI1ere are no murmurs or rubs. The remainder of the
was diagnosed 2 weeks ago after a murmur was detected examination is normal.
incidentally on examination for another medical condition. Serum troponin levels are e levated . Hematocrit is 42%
HCM has since been diagnosed in his father and brother and platelet count is 220.000/µL (220 x 1 09/ L) . Electrocar­
during family screening. There is no family history of sud­ diogram shows changes consistent with an i nferior ST­
den cardiac death. He is asymptomatic. elevation myoca rdial infarction. Portable chest radiograph
On physical examination, vital signs are normal. A soft shows a normal cardiac silhouette and no infiltrate.
holosystolic murmur is heard, which decreases during both TI1e patient is treated wiU1 enteric-coated aspirin, nitrates,
handgrip and stand-to-squat maneuvers. and a �-blocker. The hospital does not have capabilities to

1 43
Self-Assessm ent Test

Cl and t he nearest pri 111ary PC! center is 111ore than 2 hours away.
perfor111 pri111ary percutaneous coronary in tervention (PC! ) , Which of the following is the most appropriate manage­
ment?
CONT. ll1e patient is ad111 inisterecl in travenous tenecteplase .

(A) Coronary artery calcium scoring


Which of t he following is the most appropriate treatment? (B) Exercise stress testing
(A) A bci xi 111 ab (C) Increase dose of lisinopril
( B) Clopiclogrel (D) Recommend antioxidant vitamin therapy
(C) Prasugrel
( D ) Ticagrelor I t e m 84
A 6 1 -year-old man is evaluated for a 3 -month history
of progressive aching pain in the left lower extremity;
Item 8 2
the pain is present during walking and is absent with
A S6-year-old man with Eisenmenger syndrome related rest. He has a 3 0-pack-year history of smoking and quit
to a ventricular septa! defect is evaluated for recent fatigue 1 year ago . H is medical history is significant for New
and dyspnea. He had an elective cholecystectomy for symp­ York Heart Association functional class II heart fail­
tomatic cholelithiasis 4 weeks ago and has had persistent ure, hypercholesterolemia, and type 2 diabetes mellitus.
fatigue and exertional dyspnea since his operation. He has Medications are aspirin, lisinopril, simvastatin, met­
no other symptoms. His current medications are sildenafil formin, and metoprolol.
and bosentan. On physical examination, vital signs are stable. BM!
On physical examination, vital signs are normal. BMI is 2S. Femoral, popliteal, and foot pulses are diminished.
is 2S. The estimated central venous pressure is elevated There is no distal ulceration o r skin breakdown. The
with a prominent a wave. The apical impulse is normal. ankle-brachia! index is 0.70 on the left and 0 .8S on the
There is a prominent parasternal impulse at the left sternal right .
border. The s , is normal; the S 2 is loud. No murmur is appre­
ciated. Digital clubbing and central cyanosis are noted. Which o f the following i s the most appropriate manage­
The abdominal wound is healing well with no evidence of ment?
infection. (A) Initiate cilostazol
Laboratory testing reveals a hemoglobin level of
(B) Refer for endovascular repair
1 1 . 8 g/dL (118 g/L) and hematocrit of 4S%. A review of the
patient's recent laboratory results shows a hemoglobin (C) Refer for vascular surgery
level of 1 8 . 6 g/dL (186 g/L) and hematocrit of S6% before (D) Start a supervised exercise program
the cholecystectomy, and a hemoglobin level of 12 g/dL
(120 g/L) and hematocrit of 47% at the time of hospital
discharge. Item 8 5
A 19-year-old woman i s evaluated during a routine exam­
Which of the following is the most appropriate manage­
ination. She has no symptoms or significant medical history.
ment?
She takes no medications.
(A) Erythrocyte transfusion On physical examination, blood pressure is 130170 mm
(B) Erythropoietin Hg and pulse rate is 72/min. Cardiac examination reveals a
grade 116 diastolic murmur at the base of the heart. Pulses
(C) Initiate intravenous epoprostenol
are equal in the upper and lower extremities.
(D) Initiate short-course iron therapy
Transthoracic echocardiogram shows a bicuspid aortic
(E) Refer for heart-lung transplantation valve with mild aortic regurgitation. The aortic dimension
is 4 . 2 cm at the sinuses of Valsalva. Chest CT scan with con­
trast confirms the aortic root measurements, and no other
Item 8 3 pathology is noted.
A SS-year-old man i s evaluated for cardiovascular risk
Which of the following is the most appropriate manage­
assessment. He has osteoarthritis and hypertension. He is a
ment?
construction worker. His brother had a myocardial infarc­
tion at the age of S3 years. The patient's medications are (A) Annual cardiac magnetic resonance (CMR) imaging
l isinopri l and naproxen. He is a nonsmoker. (B) Annual multidetector CT
On physical examination, the patient is afebrile, blood (C) Annual transesophageal echocardiography
pressure is 138/70 mm Hg, pulse rate is 78/min, and respi­
(D) Annual transthoracic echocardiography
ration rate is 14/min. BM! is 2S. The remainder of the exam­
ination is unremarkable. (E) Reassurance and clinical observation
Laboratory studies are significant for a serum LDL cho­
lesterol level of l3S mg/dL (3.SO mmol/L) and an HDL cho­
lesterol level of 38 mg/dL (0.98 mmol/L). Item 86
His American College of Cardiology I American Heart A 66-year-old woman i s evaluated for a 3 -week history
Association 10-year atherosclerotic cardiovascular disease of worsening dyspnea on exertion. Medical history is sig­
risk based on the Pooled Cohort Equations is 6%. nificant for type 2 diabetes mellitus and hypertension.

1 44
Se lf-Assessment Test

fl,
+-'

�-:rn'±J
.:i::.: · · · ..

•. 1:::: :,,:1:•· 1 T

,\E,!f�j! . , .

�: 'i:' : I : · .

.
•,
,, . . !

ITEM 86

Medications are metformin, lisinopril, and hydrochloro­ Which of the following is the most appropriate diagnostic
thiazide. test to perform next?
On physical examination, the patient is afebrile, blood
(A) Diagnostic coronary angiography
pressure is 132/78 mm Hg, pulse rate is 78/min, and respi­
ration rate is 14/min. BM! is 28. The remainder of the exam­ (B) Exercise electrocardiography
ination is unremarkable. (C) Exercise nuclear perfusion study
Electrocardiogram is shown. (D) Pharmacologic nuclear perfusion study
Which of the following is the most appropriate diagnostic
test to perform next?
Item 88
(A) Coronary artery calcium scoring A 48-year-old woman i s evaluated during a follow-up visit.
(B) Exercise nuclear perfusion imaging She has a 5-year history of type 2 diabetes mellitus. She has
(C) Exercise treadmill stress testing no other significant medical history. Medications are ator­
vastatin, metformin, and a multivitamin. She works as a
(D) Vasodilator nuclear perfusion imaging
mail carrier and has a walking route that takes 3 hours each
day. She consumes a diet high in fruits and vegetables and
does not smoke.
Item 8 7 On physical examination, the patient is afebrile, blood
A 49-year-old woman i s evaluated for intermittent sharp, pressure is 128/80 mm Hg, pulse rate is 70/min, and respi­
nonradiating, substernal chest pain for the past 2 weeks. ration rate is 12/min. BM! is 26. The remainder of the exam­
The pain occurs more frequently in the morning and is ination is unremarkable.
not associated with meals or exertion but may be ini­ Laboratory studies are significant for a serum LDL cho­
tiated with emotional stress. The pain does not include lesterol level of 135 mg/dL (3.50 mmol/L) and serum HDL
any pleuritic or positional components, and she states cholesterol level of 37 mg/dL (0.96 mmol/L). Urinalysis is
that there are no aggravating factors. The pain often negative for albuminuria.
lasts for 10 minutes and subsides spontaneously. She has Her estimated 10-year cardiovascular risk by the Pooled
hyperlipidemia treated with pravastatin. Her mother had Cohort Equations is 2.7%.
a myocardial infarction and heart failure starting at the
age of 5 2 years. Which of the following is the most appropriate cardiovas­
cular disease risk management?
On physical examination, blood pressure is 132/82 mm
Hg and pulse rate is 78/min. BM! is 28. Lungs are clear to (A) Aspirin
auscultation. Cardiac examination shows a normal S1 and S2 ; (B) Coronary artery calcium scoring
there is no S3, S4 , murmurs, rubs, or gallops. She has no lower
(C) Exercise stress testing
extremity edema. The remainder of the examination is normal.
Electrocardiogram shows a heart rate of 80/min. The (D) Folic acid supplementation
QRS axis is normal, and there are no ST-T wave changes. (E) No further testing or therapy

1 45
Self-Assessment Test

Item 9 1
Cl A 57 -yea r-o ld woman is evaluated i n t he hos p i t a l for
Item 8 9
A 64-year-old woman is evaluated for a 3-month history
c h ro n ic sys t o l i c heart fa i l ure. She was ad m i t ted w i t h of sharp chest discomfort that she experiences during gar­
progressive clyspnea of 2 weeks' d ur a t ion . A fter 3 clays dening. Medical history is significant for hypertension and
o f aggressive d i ur e t i c t herapy w i t h weight loss o f 5 kg hyperlipidemia. The patient's father had a coronary artery
(11 l b) . she rem a i ned very dyspneic. and right heart ca t h ­ bypass graft at the age of 68 years. Medications are losar­
e teriza t ion w a s performed . Medica t ions a re l isinopri l , tan, hydrochlorothiazide, and atorvastatin, and she recently
digox i n . spironolactone. a n d i n t e rm i t tent furosemide started taking low-dose aspirin daily.
i n t ravenously. On physical examination, the patient is afebrile, blood
On physical examina tion . b lood pressure is 9 6/74 m m pressure is 136/84 mm Hg, pulse rate is 78/min, and respira­
Hg. pulse rate i s 1 1 8/ min . a n d respiration rate i s 20/m i n . TI1e tion rate is 16/min. BM! is 26. The remainder of the physical
i n ternal jugular vein is not visible when the patient is in an examination is unremarkable.
upright posi tion . Lungs a re clear. An S.i is heard on cardiac Baseline electrocardiogram shows left ventricular
examination. There is b i lateral edema to the knees. Her hypertrophy with ST-segment depressions less than
serum creatinine level is 1 .7 m g/ell ( 1 5 0.3 pmol / L) . 0.5 mm in the lateral leads. During exercise stress test­
Hemodynamic measurements: ing, the patient develops 1-mm ST-segment depressions
Right atrium pressure 4 mm Hg in leads II, III, and aVF. She exercised 5 minutes and
Pul monary capi llary 16 mm Hg 30 seconds of a Bruce protocol; her peak heart rate was
wedge pressure 129/min (85% predicted maximum) , and blood pressure
Cardiac output 3.1 U m in (normal, was 186/76 mm Hg.
4. 0- 8. 0 U m i n ) Which of the following is the most appropriate next step in
Cardiac i ndex 1 .8 U m i n/ m1 the management of this patient?
Systemic vascular 2050 dyne/s/cm2 (normal,
resistance 800- 1200 dynels/cm") (A) Add a �-blocker
(B) Cardiac catheterization
Which of the following is the most appropriate change in
(C) Cardiac magnetic resonance (CMR) imaging
this patient's thera py?
(D) Stress echocardiography
(A) Con t i nuous i n t ravenous furosem icle
(B) Dopamine i n fusion

Cl
(C) Esmolol drip Item 9 2
( D) N i t roprusside A 66 -year-old man i s evalua ted i n the emergency depart­
ment for 45 m i n u tes ofsubsternal chest pain that radiates to
the left shoulder. The patient's medical histo ry is signi Acant
Item 9 0 for hy pert e nsi o n . type 2 diabetes mellitus. and hy pe rli pid ­
A 47-year-old man i s evaluated fo r a 3-month history of em ia . He has never had abnormal bleeding. Med icat ions are
fatigue, abdominal fullness, and lower extremity edema. low-close aspiri n . gli m e p i ride. l isinopr i l . and simvasta t i n .
Ten years ago, the patient had acute pericarditis with car­ He h a s no known drug al lergies.
diac tamponade; the tamponade was treated successfully On physical exami na tion. bl o od pressure is 174/ 92 m m
with pericardiocentesis, and the pericarditis resolved fol­ Hg a n d p ulse rate is 82/m i n. Cardiac examination shows a
lowing a course ofan anti-inflammatory medication. He has normal S1 and S2; there is no S,l' S4, m u rmur, or rubs. The
no history of significant alcohol consumption, hepatitis, or remainder of the physical exam ination is norm a l .
autoimmune disease, and takes no medications. Hemoglobin concent ra t ion is 13.4 g/d L ( 1 34 g/L) and
On physical examination, the patient is icteric. Vital serum crea t i n i ne level is 1.0 m g/d L (88.4 pmol 1 L) . Results
signs are normal; BM! is 30. The estimated central venous of serum troponin levels are pending. Electrocardiogram is
pressure is 12 cm Hp, and the jugular venous pulse shows shown (see top of next page) .
a prominent y descent. S1 and S2 are normal, and no mur­ Tiie pa t ie n t is given aspiri n . clopidogrel. u n fractionatecl
murs, rubs, or gallops are heard. There is dullness to percus­ hepari n , and a �-blocker. Transport to the nearest hos p i ­
sion at the right lung base. The remainder of the pulmonary tal w i t h primary percutaneous coronary intervention (PC!)
examination is normal. Both ascites and lower extremity capabi lities would take approximately 1 35 minutes.
edema are present.
Which of the following is the most appropriate manage­
Transthoracic echocardiography is technically chal­
ment?
lenging, and limited information is obtained. Fluid obtained
from abdominal paracentesis is transudative. (A) Administer tenecteplase and t ra ns fer to a PCI-capable
center
Which of the following is the most appropriate next step in
management? (B) A d m i t to the hospital and awa i t ca rdiac biomarker
results
(A) Hemodynamic cardiac catheterization (C) I n i tiate abciximab and transfer for u rgent coronary
(B) Liver biopsy angiography
(C) Measurement of B-type natriuretic peptide (D) Transfe r for primary percutaneous coronary i n ter­
(D) Vigorous diuresis v e n t i on

1 46
Self-Assessment Test

• i ! ; l

v: l l '. l ! I __.._:

ITEM 92

Item 9 3 palpable bilaterally. The resting ankle-brachia! index is 1 .1


A 60-year-old man is evaluated for a murmur that was on both sides.
noted on a routine examination. He reports exertional dys­ Which of the following is most likely to confirm the diagnosis?
pnea when walking up stairs. He has hypertension and
takes lisinopril. (A) Exercise ankle-brachia! index
On physical examination, the patient is afebrile, blood (B) MRI of the lumbosacral spine
pressure is 1 60/60 mm Hg, pulse rate is 90/min, and respi­ (C) Segmental limb plethysmography
ration rate is 16/min. Cardiac examination reveals a laterally (D) Toe-brachia! index
displaced cardiac apex, soft S I ' and absent S 2 • There is a grade
216 high-pitched blowing diastolic murmur at the left third
intercostal space and a grade 1 1 6 rumbling mid-diastolic Item 9 5
murmur at the apex. A 35-year-old woman with recently diagnosed nonischemic
systolic heart failure and a left ventricular ejection fraction
Which of the following is the most likely diagnosis?
of 30% presents 1 week after hospital discharge with a new
(A) Aortic regurgitation cough, increased exertional dyspnea, and peripheral edema.
(B) Mitra! stenosis Medications are lisinopril (5 mg/d) and furosemide (40 mg/d).
(C) Patent ductus arteriosus On physical examination, blood pressure is 1 00/70
mm Hg, pulse rate is 98/min and regular, respiration rate is
(D) Ruptured sinus of Valsalva aneurysm
13/min, and oxygen saturation on ambient air is 96%. Esti­
mated central venous pressure is 15 cm H p . Her weight
has increased by 2.3 kg (5 lb). Cardfac exammation reveals a
Item 9 4
grade 2/6 holosystolic murmur at the apex and an S3. Extrem­
A 72-year-old woman is evaluated for cramping pain i n the ity examination reveals bilateral peripheral pitting edema.
buttocks and thighs \Ni th standing and walking. Symptoms are Her electrocardiogram shows sinus rhythm and is
exacerbated after standing at work for several hours and are unchanged from baseline. Serum electrolyte levels and kid­
relieved by sitting. Her symptoms have been present for 6 to ney function tests are normal.
8 months and have been stable. Her medical history is signif­
icant for hypertension. She has a history of smoking but quit In addition to a low-sodium diet, which of the following is
12 years ago. Her medications are amJodipine and Lisinopril. the most appropriate management?
On physical examination, vital signs are normal. BM!
(A) Increase furosemide
is 22. Deep tendon reflexes are decreased at the ankles but
(B) Increase lisinopril
normal at the knees. Lower extremity muscle strength is
normal. No abdominal or femoral bruit is present. No skin (C) Start carvedilol
changes are noted in the lower extremities. Distal pulses are (D) Start spironolactone

1 47
Self-Assessment Test

loss of consciousness. The episodes are not precipitated by


Item 96
any particular activity, including exercise. She takes no
A 46-year-old m a n is evaluated i n the emergency depart­
medications.
ment for an episode of left facial and left upper extremity
On physical examination, the patient is afebrile, blood
numbness and weakness that began just over 1 hour ago.
pressure is 110/68 mm Hg, pulse rate is 72/min, and respira­
The motor symptoms have resolved fully, but some numb­
tion rate is 16/min. BMI is 29. Cardiac examination reveals
ness persists. He has no other medical problems and takes
physiologic splitting of S,, regular rate and rhythm, and no
no medications. -
gallop. Estimated centra 1 venous pressure is normal. She
On physical examination, vital signs are normal. BM!
has no edema. Serum thyroid-stimulating hormone level
is 35. There are no carotid bruits or heart murmurs. 1he
is normal.
neurologic examination is now normal, with the exception
A 12-lead electrocardiogram shows normal sinus
of facial numbness.
rhythm.
Laboratory testing and electrocardiogram are normal.
Carotid ultrasonography is normal. Magnetic resonance Which of the following is the most appropriate diagnostic
angiography demonstrates a small right-sided ischemic testing option?
stroke but no other lesions. Ultrasonography of the lower
extremities is normal. (A) 30-Day wearable loop recorder
A transesophageal echocardiogram is unremarkable (B) Echocardiogram
with the exception of a patent foramen ova le with right-to­ (C) Exercise treadmill stress test
left shunt noted with cough and Valsalva release. (D) 48-Hour ambulatory electrocardiographic monitor
Which of the following is the most appropriate treatment? (E) Implantable loop recorder

(A) Aspirin
(B) Patent foramen ovale device closure
(C) Warfarin
Item 99
A 56-year-old man is eval uated i n t h e hosp i t a l for a 2-week
Cl
(D) No therapy history of' fevers and mala ise. M ed ical h ist ory is sign incant
for a bicuspid aortic valve . The patient t a kes no medica t ions.
On p hysica l exa m i na t ion . tem pera ture is 38 . 5 °C
Item 97 ( l O l . 3 ° F) . blood pressure is 1 4 0 1 50 mm Hg. p u l se ra te is
A 46-year-old man is evaluated i n follow-up for a bicus­ 9 8/ m i n . a n d res p i ra t ion rate is 16 m i n . ·n1ere is n o j ugu lar
pid aortic valve. He exercises regularly without any activi­ venous distent ion . ll1e l u n gs are clear. Cardiac exam i na ­
ty-limiting symptoms and feels well. His medical history is t ion revea ls a grade I 6 diastolic m u rmur. There a rc no
otherwise negative and he takes no medications. s igns of' periphera l embolic d isease . No lower e x t re m i t y
On physical examination, blood pressure is 138/85 mm edema is presen t .
Hg. BMI is 28. A systolic ejection click followed by a crescendo­ Electrocardiogram shows normal sinus rhy t h m . a P R
decrescendo murmur are noted at the left sternal border. interval of' 230 ms. and nonspecific T-wave changes. Except
No diastolic murmur is appreciated. 1he lower extremity for t he increased PR i n terval . t here a re no cha nges com ­
pulses are normal. The remainder of the examination is pared w i t h a prior t racing. A t ransthoracic echocardiogram
unremarkable. shows a 6 -m m vegetation on the aort ic valve wi t h m i l d to
Transthoracic echocardiogram shows a bicuspid aortic moderate aort ic regurgi ta t ion . A t ransesophageal echoca r­
valve with systolic doming of the aortic valve and a valve cliogram con fi rm s t he va lve f i n d i ngs and suggests t he pres
area of 1.7 cm2• l11e mean gradient across the aortic valve ence o l ' a n area of' r l uicl around t h e aortic a n n u l us posterior
is 22 mm Hg. The ascending aorta is dilated at 4.5 cm; the to t he yegetation . i ndicat ive of a n aortic root abscess.
descending thoracic aorta is incompletely visualized. Chest Blood cul tures are posit i ve for Staphy lococcus a u reus
CT demonstrates a 4.6-cm aneurysm of the ascending aorta sensit ive to met h ici l l i n . Appropria te a n t ibiot ics are st a rt ed .
with no evidence of coarctation and no enlargement of the
Which of the following is the most ap prop ria te treatment?
descending aorta .
(A) A n t ibiotic t h erapy f'or 6 weeks and t hen reassess
Which of the following is the most appropriate next step in
management? ( B) A n t i b iotic t herapy for 3 mont hs and t hen reassess
(C) Aortic v;:i lve replacement a fter 6 w eeks of' a n t ibiotic
(A) Aortic valve replacemen t t h erapy
(B) Aortic valve replacement and ascending aortic repair
(D ) U rgent aort ic valve replacement
(C) Ascending aortic repair
(D) Repeat echocardiogram in 1 year
Item 1 0 0
A 26-yea r-o ld \Noman is evaluated i n t he emergency depart­
CJ
Item 98 ment for palpitations and pounding in her neck. She of'ten
A 42-year-old woman i s evaluated for episodes of palpi­ gets t hese episodes and t hey typica l l y last sewral m i n u tes:
tations that last several seconds in duration. They occur however. t h i s episode has been going on for 30 m i n u tes.
once or twice a month and are accompanied by light­ She can usual ly stop t h e episodes by beari ng clown . but on
headedness and mild dyspnea. She has not experienced t h i s occasion t h i s has not· worked . She reports f'eel i ng a l i t t le

1 48
Self-Assessment Test

�JlUTilllln�1nrrrrUillill
)lliWJ��rtt1111t�llilu
���lJ��tfWrKthillJJ-u
!i _,UJJil
JJL lllUJlLUJJl.
'\JI
VI
rrrrrr
rmr rr
rnrmnr rr
"' lJilllJJJJJilLil JJJJJJJlillUlL
ITEM 1 00

Cl s hort of brea t h , but she docs not have chest pain or loss o f' to t h e hospi t a l . he began receiving metoprolol, clopidogre l .
consciousness. She has no other sign i fie a n t med ica I history. and i n t ravenous hepari n .
CONT.
and her only medicat ion is an oral contracept ive. V i t a l signs are norm a l . and h is physical exa m i n a t ion is
On physica l exam i na t i o n , t h e p a t i e n t is af'e bri le. b l ood u nremarkable.
pressure is 1 20 /80 mm H g. pu lse ra te is 1 45 / m i n . a n d
In addition to continuing aspirin indefinitely, how long
respi r a t i o n r a t e is 1 8/ m i n . B M I is 25. Card iac e x a m i n a t i o n
should this patient's clopidogrel therapy be continued?
shows tachyca rdia b u t regu l a r r hy t h m . Lungs a re c l e a r t o
a uscu l ta ti o n . (/\) 2 weeks
The elec trocardiogram is shown. (B) I month
Which of the following is the most appropriate treatment? (C) I yea r
( D) Li f'c long
( A ) Adenosine
( 13) /\ m iodarone
(C) Cardioversion
(D) l b u t i l ide
Item 1 0 2
A 68-yea r-old man is eva luated for progressive shortness of
CJ
brea t h . He underwen t heart t ra nsplantation 10 years ago for

Cl Item 1 0 1 ischemic ca rd iom yo pa t hy <1 11d has had no l i m i ta tions or his


activi t ies si nce t hen until developing dyspnea with exertion
A 64 -yea r-o l d man is eva luated before discharge rrom t he over t he past 3 weeks. Medical h istory is ot herwise significant
hospital fol lowing a non -ST-elevat ion myocardi a l i n fa rc­ ror hypertension. hyperlipidem ia. and a 45-pack-year smok-
t ion t reated w i t h p lacement o f' a bare metal s t e n t . He is ing history: he stopped smoking before his t ransplant. Med ­
curre n t ly pain free and tolera t ing h is med ications wel l . H e icat ions a re aspirin . l isi nopri l . atorvast a l i n . and tacro l i m us.
notes no new symp to ms. Medicat ions prior to adm ission On physical exa m i na t i o n , b l ood pressure is 1 4 0/ 78 m m
were aspiri n , l isinopri l . and a torvastatin : upon adm ission I l g. pu lse ra te is 1 02/ mi n . a n d respira t ion rate is 1 6 / m i n .

1 49
Se lf-Assess ment Test

IT'I TI1ere is no jugular venous disten t i o n. ·n1e l u ngs are clear. Which of the following is the most appropriate preopera­
LI.I a n d t h e heart e x a m i n a t ion is unremarkable . T he rem a i nder tive device management?
CONT.
or the exam i n a ti on is norm a l . (A) Disable shock i ng function
Elect rocardiogram demons t ra tes s i n us tachycard i a.
( B) Proceed with surgery a n d i n terrogate t he device post ­
right bundle branch block. and no Q waves. Echocard io­
opera tively
gram shows a left ven tricular eject ion fraction of 5 5 % . evi ­
dence or m i l d d iastolic dysfu nct ion . septa I wa l l t h ickness (C) Reprogram to async h ronous pacing and disable
of 0.9 cm . posterior wall t h ickness or 1 . 0 cm . and moderate shoc k i ng function
t ricuspid regu rgi t a ti on. (D) Advise aga i n s t surgery
Which of the following is the most appropriate diagnostic
test to perform next?
Item 1 0 4
(A) oro n a ry angiography A 38-year-old man is evaluated for gradualJy progressive
( B) Endomyocardial biopsy exertional dyspnea. He had one episode of atrial fibrillation
(C) Pulmonary function test 1 year ago but converted spontaneously in the emergency
department. No additional testing was performed at that
( D) Ven t i la t ion -perfusion lung scan
time, and no medical therapy was initiated. He is otherwise
healthy and has been active. His medical history is other­
wise unremarkable. He takes no medications and has no

Cl A 69-year-old woman scheduled to undergo shou Icier surgery


Item 1 03 al lergies.
On physical examination, blood pressure is 120/70 mm
is evaluated in the hosp i ta l for perioperat ive management of Hg, pulse rate is 68/min and regular, and respiration rate is
her cardiac device. She v.ras diagnosed with heart failure l year 16/min. BM! is 26. The estimated central venous pressure is
ago. and a dual-chamber i mp lantabl e carclioverter - defibrillator elevated. There is a parasternal impulse at the left sternal
was placed. Her last device check was 3 mon t hs ago. Medi ­ border. Persistent splitting of the S, is noted. There is a soft
cal hi story is sign i ficant for previous myocardial i n farct ion. midsystolic murmur at the second le ft intercostal space and
corona1y arte1y bypass graft su rgery 1 year ago. ven t ricu lar a separate holosystolic murmur at the apex. The rest of the
tachycardia, and complete heart block. Medicat ions are aspi­ examination is normal.
rin. metoprolol. lisinopri l . rurosem icle. and spironolactone. The electrocardiogram is shown.
On physical exa m i n a t io n . the pat ient is a febrile. b lood
pressure is 95/50 mm Hg, pu lse rate is 64 / m i n . and respi
Whlch of the folJowing is the most likely diagnosis?
rat ion rnte is l 6/ m i n . B M I is 32. Est i m a ted cen tral venous (A) Coronary sinus atrial septa! defect (ASD)
pressure is norm a l . T here is no carotid bru i t . Cardiac a us­
(B) Ostium primum ASD
c u l t a t ion reveals a norm a l rat e and a fi xed spl it S ,. Lungs a re
- (C) Ostium secundum ASD
clear to auscu l t a t io n . She has no edema.
An elect rocardiogram shows sequen t i a l a t rioven tric­ (D) Patent foramen ova le
ular pacing. (E) Sinus venosus ASD

I i ll r
.
II

) ,, v

ITEM 1 04

1 50
Self-Assessment Test

Cl An
Item 1 05
82-year-old man was admWed to the coronary care unit
(C) Add warfarin
(D) Cardiac resynchronization therapy
(CCU) 48 hours ago after a late presentation with anterior
ST-elevation myocardial infarction. -n1e patient underwent
. 11. 1 fect1ve
. endocard1t1s
. . CJ
coronary angiography and was found to have an occluded Item 1 0 7
proxi.rnaJ left anterior descending coronary artery but did not .
A 45-year-old man be111g treated for
undergo an attempt at revasculari.zation at the time of coro­ is seen for a follow-up examination. He was diagnosed with
nary angiography because of his late presentation and symp­ endocarditis 1 week ago after presenting with fatigue and
tomatic improvement. Today. he felt faint and lost conscious­ fever. Initial transthoracic echocardiogram showed a bicus-
ness while visiting with his family in the CCU. Medications pid aortic valve with a small vegetation but was otherwise
are aspirin. ticagrelor, metoprolol. lisinopril, and atorvastatin. normal. Blood cultures were positive for methicillin-sensitive
On physical examination. blood pressure is 72/54 mm Staphy lococcus a u reus, and intravenous nafcillin was ini­
Hg and pulse rate is 108/min. Cardiac examination shows tiated. Blood cultures obtained 48 hours and 72 hours after
tachycardia with a normal S1 and S,. new holosystolic mur­ starting antibiotic therapy showed no growth.
mur heard best at the left lower sternal border that radiates On physical examination, temperature is 37.8 °C
to the apex, and a right ventricular heave. Crackles are (100.0 °F), blood pressure is 128/78 mm Hg, pulse rate is
heard at the bases of both lungs, one third of the way up. 88/min. and respiration rate is 16/min. BM! is 25. Physical
He has no lower extremity edema. ll1e remainder of the examination reveals no cutaneous or ocular stigmata or
examination is normal. bacterial endocarditis. Cardiac examination reveals a grade
Electrocardiogram shows persistent ST-segment eleva­ 2/6 early systolic murmur al the base of"the heart, unchanged
tion and Q waves in leads V1 through V,. Heart rate is UO/min. Crom previous examinations. -n1e remainder of the physical
Emergency transthoracic echocardiogram shows a lef't ven­ examination is normal.
tricular ejection fraction of 35'Yu with severe anterior-apical Electrocardiogram is unchanged from the time of diag­
akinesis, a small pericardiaJ effusion, and a color flow jet nosis except for an increase in the PR interval from 120 to
across the ventricular septum. suggestive of"left-to-right flow. 210 ms.
Which of the following is the most appropriate manage­ Which of the following is the most appropriate next step
ment? in management?

(A) Emergency cardiac surgery (A) Cardiac CT


(B) Emergency pericardiocentesis (B) Continued antibiotic therapy without additional testing
(C) Percutaneous coronary intervention to left anterior (C) Repeat transthoracic echocardiogram
descending artery (D) Transesophageal echocardiogram
(D) Right heart catheterization and dopamine infusion

Item 1 06 Item 1 08

A 72-year-old woman is evaluated for progressive heart A 58-year-old man is evaluated for a 3-month history of left
failure symptoms. She has a 10-year history of nonischemic upper extremity symptoms and dizziness. He is left-handed
heart failure. She currently experiences exertional dyspnea and works as a carpenter. He describes an aching sensation
with climbing one flight of stairs, which she was able to do and feeling of fatigue in his arm and occasional dizziness
without sho1iness of breath 3 months ago. Medical history that occur within 2 to 3 minutes of using a hand saw; these
is significant for hype1iension, and her medications are symptoms resolve several minutes after stopping activity. He
lisinopril, carvedilol, furosemide, digoxin, and spironolac­ is otherwise asymptomatic. Medical history is significant for
tone. The patient is black. hypertension, hyperlipidernia, and type 2 diabetes mellitus.
On physical examination, blood pressure is 134/72 mm He has a 40-pack-year smoking history but quit 1 year ago.
Hg and pulse rate is 66/min. BM! is 35. She has no jugular Medications are lisinopril, atorvastatin, and metformin.
venous distention. Cardiac examination reveals a grade 116 On physical examination, he is afebrile, left arm blood
holosystolic murmur but is otherwise normal. The1:e is . no pressure is 135/76 mm Hg, pulse rate is 68/min, and respi­
lower extremity edema. The remainder of her exam111at10n ration rate is 16/min. BM! is 29. ·nie carotid upstrokes are
is unremarkable. normal. The chest is clear and the cardiac examination is
Laboratory studies are significant for normal elec­ normal. Examination of the left upper extremity is unre­
trolyte levels and a serum creatinine level of 1.5 mg/dL markable, with palpable distal pulses and no evidence of
(133 µmol/L). distal ulceration or skin breakdown. 'llie remainder of his
Electrocardiogram shows normal sinus rhythm, a QRS physical and neurologic examination is unremarkable.
duration of 110 ms, and nonspecific ST-T wave changes. Which of the following elements of the physical examina­
Echocardiogram shows a left ventricular ejection fraction tion would be most helpful in establishing the diagnosis?
of 38% and trace mitral regurgitation. (A) Ankle-brachia! index
Which of the following is the most appropriate treatment?
(B) Bilateral blood pressure measurement
(A) Add hydralazine and isosorbide di.nitrate ( C) Evaluation for pulsus paradoxus
(B) Add losartan (D) 111oracic outlet maneuvers

1 51
Self-Assessment Test

l��
J aVR VJ V4

1-J
JI
�J�'\�\�·�

ITEM 1 09

Item 1 0 9 On physical examination, blood pressure is 146/94


mm, and pulse rate is 52/min. BM! is 28. The remainder of
An 86-year-old man is evaluated in the emergency depart­
the examination is normal.
ment after a fall. He reports tripping over a rug. He did
not experience presyncope or other symptoms. He other­ Laboratory studies:
wise feels well and is active. Medical history is significant Total cholesterol 152 mg/dL (3.94 mmol/L)
for hypertension. Family history is notable for pacemaker LDL cholesterol 72 mg/dL (1.86 mmol/L) (pre-treatment
implantation in his sister. His only medication is lisino­ baseline: 150 mg/dL [3.89 mmol!L])
pril. A perfusion stress test 1 year ago obtained because of HDL cholesterol 46 mg/dL (1.19 mmol/L)
chest discomfort demonstrated normal left ventricular wall Triglycerides 84 mg/dL (0. 95 mmol/L)
motion and ejection fraction.
On physical examination, temperature is 3 6 . 8 °C One year ago, she underwent transthoracic echocar­
(98.2 °F) . blood pressure is 135/80 mm Hg, pulse rate diography that was significant for normal left ventricular
is 67/min, and respiration rate is 1 2/min. BM! is 24. His systolic function and no valvular abnormalities.
neck veins are flat. The point of maximal impulse is in
Which of the following interventions offers the greatest
the mid-clavicular line without heave or lift . There is
cardiovascular risk reduction to this patient?
mild bruising over his left hip. He has no lower extremity
edema . (A) Add fish oil
Plain radiographs of the left hip are negative for frac­ (B) Add niacin
ture. Laboratory evaluation demonstrates a normal met­
(C) Reduce alcohol consumption
abolic profile and preserved kidney function. His 12-lead
electrocardiogram is shown. (D) Start an ACE inhibitor

Which of the following is the most appropriate manage­


ment? Item 1 1 1

(A) Adenosine nuclear stress test A 52-year-olcl m a n is eval ua ted in t h e hosp i t a l for pro­
(B) Dual-chamber pacemaker gressive chest pressure over t he past 3 wee ks. He has a
35-pack-year history of cigarette smoking. M ed ica l h i s tory
(C) Single-chamber pacemaker
is sign i f i c:rn t for h y p e rt ensi on a n d hy per li p i clem i a t rea ted
(D) No intervention is indicated w i t h aspi rin, hyd roc h l o ro t h iaz i d e . l is in op ri l a n d pravasta­
.

t i n . His brother h::id a my oc ard i a l i n farction a t age 48 years.


On p hys i ca l exami nation . he is a febri le. b lood pressure
Item 1 1 0 is 148 82 m m Hg. a ncl pulse rate is 98 m i n . Card iac a ncl lung
A 58-year-old woman is evaluated during a routine exam­ exa m i na t ions a re normal.
ination. She is asymptomatic. She had a myocardial infarc­ Cardiac biomarkers are el e v a t ed I n i t ial electrocardio­
.

tion 4 years ago and has hypertension and dyslipidemia. She gra m s hows 2 - m m ST-segment d epress i on i n l ea ds I. a\IL.
is a former smoker and consumes one alcoholic beverage a ncl \I 1 th rough v( ,.
daily. She has no limitations with physical activity and He is a d m i t ted to the coronary care u n i t a nd gi ve n a sp i ­

is able to exercise periodically. Medications are low-dose r i n . metopro l o l . n i t roglycerin paste. a n d e n o x a p a r i n . Over
aspirin, hydrochlorothiazide, metoprolol, and high-dose t he course of' the first 12 hours. h is chest pressure worsens.
atorvastatin. requ i ring i n t ravenous n i t roglycerin in fusion . Subsequently.

1 52
Self-Assessment Test

rr1 h is chest pressure i mproves a n d he undergoes coronary


and the myocardium both appear normal. No pericardia!
L&JI a ngiography .
effusion or significant disease involving the cardiac valves
CO NT.
.
Co 1·o n a ry a ng1ograp .
. · ri1ca n t I'or a 70'X, left m a i n
I1y IS s1g111 is present.
coronary artery s te nosis . 80'X, m i d left a n t e r i o r desc en d i n g
stenosis, a nd 90% proximal r ig h t coronary art ery s tenosis . Which of the following tumors is the most likely cause of
Left ven t ricular ejection fraction is .:J S % w i t h mild a n t erior the patient's symptoms and clinical findings?
hypo kines is . M i l d m i tral regur gi ta t ion i s not ed . (A) Adenocarcinoma
He is c u rre n t ly he m o d y na m ic a l ly stable and pa in Cree .
(B) Angiosarcoma
Which of the following is the most appropriate manage­ (C) Lipoma
ment? (D) Myxoma
(A) Coro n a ry a r t e ry bypass g r<oi ft surgery (E) Papillary fibroelastoma
( B) I n t ra - a o r t ic bal loon pu m p placement
(C) Perc u t a n eo u s coronary i n t e rvention
Item 1 1 4
( 0) Con t i n ue cu rrent t h erapy
A 38-year-old woman is evaluated for a 1-week history of
substernal chest pain. Symptoms are aggravated by lying in
c:J Item 1 1 2 the supine position or taking a deep breath. The patient is
otherwise healthy and takes no medications.
A 6 0- yea r- o ld woman is evalua ted i n t he hosp i t a l for a On physical examination, temperature is 37.5 ° C
3-wee k history of worse n i n g dyspnea a n d chest pain . Med­ (99.5 ° F) , blood pressure is 1 12/70 mm Hg, and pulse rate
i c a l h i s t ory is sign i ficant for hypert ension, hyper l ipidemia . is 84/min. Systolic pressure variation with respiration
a n d previous coronary artery bypass g r a ft surgery . Medica­ is 5 mm Hg. The lungs are clear to auscultation. A loud,
t i ons are aspirin , rurosemide. me l o prolo l . a nd a t orvas ta l i n . three-component cardiac rub is heard at the apex. S 1 and
On p hys ical exami n at ion . s h e is atebri le . blood pres­ S 2 are normal. The remainder of the physical examination
sure is l l O 70 mm H g, pu ls e ra te is 9 2 1 m i n . a nd respi rn ­ is unremarkable.
t i on rnte is 1 6 / m i n . Esti mated centra l venous pressure i s Laboratory studies are significant for a leukocyte count
e levated . Exa m i n a t io n at t h e card iac base demonstra t es of 7000/µL (7.0 x 109/L) . Echocardiogram shows normal left
a grade 3 1 6 late-pea k i ng systolic m u r m u r <1 11d a grade l 6 and right ventricles. A moderately sized circumferential
d i a s t o l ic m u rm u r. An S.1 is not he<1 rd . pericardia! effusion is noted. The inferior vena cava is nor­
Echocardiogram s li o w s <oi markedly calc i f ied bicuspid mal in size. A Doppler ultrasound shows minimal change in
aortic valve with severe aort i c stenosis a n d moderate ao rt i c mitral inflow with respiration.
regurgitation . High-dose ibuprofen, a proton pump inhibitor, and
colchicine are started.
Which ofthe following is the most appropriate treatment?
Which of the following is the most appropriate next step in
(A) Aortic va l ve repai r
management?
(B) Ba ll oon aortic valvuloplas ly
(C) Surgical aortic valve replacem e n t (A) Cardiac CT
(D) Transcat heter a o r t i c v<oilve replacement (B) Clinical follow-up
(C) Glucocorticoid administration
(D) Inpatient monitoring
Item 1 1 3 (E) Pericardiocentesis
A 56-year-old woman is evaluated for a 1 -year history of
exertional dyspnea. She also occasionally experiences light­
headedness during intense aerobic activity, such as run­ Item 1 1 5
ning. Symptoms are not associated with chest pain and A 31-year-old man is evaluated for follow-up 2 days after
have been relatively stable since onset. She also reports a an emergency department visit for palpitations. He reports
6.8-kg (15-lb) weight loss and intermittent fevers over the intermittent palpitations and occasional episodes of short­
past 4 months. She had a colonoscopy and mammography ness of breath. 1hese episodes have increased in frequency
6 months ago with normal results. Medical history is other­ and are often accompanied by light-headedness. He expe­
wise unremarkable, and she takes no medications. rienced loss of consciousness on one occasion. He does not
On physical examination, blood pressure is 120/72 mm have chest discomfort or jaw pain. His medical history is
Hg, and pulse rate is 62/min; BM! is 18. The jugular venous unremarkable except for a previous emergency department
pulse, carotid upstroke, and lung examination findings visit several years ago for palpitations. He has no significant
are all normal. The p rec ordiu m is quiet, and S1 and S2 are family history.
normal. A soft grade 2/6 diastolic rumble is heard at the On physical examination, the patient is afebrile, blood
apex and is preceded by an early diastolic filling sound. The pressure is 105/68 mm Hg, pulse rate is 67/min, and respi­
remainder of the examination is unremarkable. ration rate is 12/min. BM! is 24. His neck veins are flat, and
Echocardiogram shows a 2.5 x 2.2 cm mass in the left the point of maximal impulse is in the midclavicular line
atrium; the mass is moderately mobile with a pedunculated without heave or lift. He has no lower extremity edema.
stalk adherent to the fossa ovalis. Left ventricular function Serum thyroid-stimulating hormone level is normal.

1 53
Self-Assessment Test

i 1

VI
I I Y4
i '
. r

j I
�� �-f:-
a YL V2 �5 .
II

n
I I

n
n
n VS

ITEM 1 1 5

Electrocardiogram is shown. Transthoracic echocar­ hypertrophy. Echocardiogram shows a left ven tricular ejec­
diogram shows no structural abnormalities. t ion fraction or 1 0% and no valvular regurgi tation.

Which of the following is the most appropriate next step in Which of t h e following i s t h e most appropriate di ag n ostic
management? test to perform next?

(A) Antiarrhythmic drug therapy (A) Antinuclear a n t i body level


( B) Diltiazem (B) Endomyocardial biopsy
(C) Electrophysiology study (C) TI1yroid stud ies
(D) Metoprolol (D) V ir<l l t iters

Cl A 48-year-old woman is eval uated for a 3 -week history of


Item 1 1 6 Item 1 1 7
A 31 -year-old woman is evaluated during a follow-up
progressive dyspnea. palpitat ions. and peripheral edema. examination. Marfan syndrome was diagnosed 6 months
S he has loose stools and a recent 2 . 3- kg (S- lb) weight loss. ago and was confirmed by significant family history and the
Medical h istory is unrema rka ble. and she does not use injec­ presence of ectopia lentis. She has been in good health and
t ion drugs. She does not take any medications and has no physically active. She does not report any chest discomfort,
known drug a l lergies. shortness of breath, syncope, or presyncope.
On physical exam ination. tempera t u re is 37. 3 °C On physical examination, the patient is tall and slen­
(99 .2 °F). blood pressure is 92/ 60 mm Hg, pulse rate is der. Blood pressure is 100/62 mm Hg and equal in both
1 1 8/ m i n , and respirat ion rate is 22/ m in . BMI is 23 . J ugular arms. Significant findings include a high arched palate,
venous distention extends to t he jaw . The t hy roid is pal ­ pectus excavatum. and arachnodactyly. The jugular and
pable but without ident iAable nodules. The precordiurn is carotid examinations are normal. There is a grade 116 blow­
hyperdy n a m ic. and a n S.i is heard on cardiac examinat ion. ing diastolic murmur best heard at the left sternal border.
She has severe bi lateral lower extremity edema ex tending to The remainder of the examination is unremarkable.
the midcalf. Examination of the joints is normal , and there Except for pectus excavatum, a chest radiograph is
is no skin rash . unremarkable. Transthoracic echocardiography shows
Leukocyte count is 6000 /µ L (6. 0 x 1 0"/ L) w i t h a normal enlargement of the aortic root, measuring 3. 9 cm with mild
d ifferential. Results or complete blood coun t are norma l . aortic regurgitation, unchanged from previous imaging
Electrocardiogram shows sinus tachyca rdia . no Q waves studies. The remainder of the echocardiographic examina­
or T-wave abnorm a l i t ies. and no signs of left ven tricular tion is unremarkable.

1 54
Self-Assessment Test

How frequently should this patient undergo surveillance est i m a ted cen t ra l venous pressure is eleva ted . The apical
imaging? i m pulse is di ffuse. T he S 1 and S 2 are soft . A n S 1 and S , a re
(A) Every 6 months prese n t . A soft holosystolic murmur is heard a t t he apex.
Crackles are heard over bot h l u ng fields. P i t t i ng edema is
(B) Every 12 months
noted to t he knees.
(C) Every 24 months An elect rocardiogram is shown. An echocardiogram
(D) Every 3 to 5 years reveals a global red uction in con trac t i l i ty and left ven t ricu­
lar enlargement wit hout hypertrophy.

Item 1 1 8 Which of the following is the most likely diagnosis?

A 59-year-old man is evaluated for a 3-month history of (A) Acu te pulmonary embolism
intermittent exertional chest discomfort. He has hyperten­ (B) lschemic ca rd iomyopa t hy
sion treated with lisinopril and amlodipine. (CJ Peri part um cardiornyopat hy
On physical examination, the patient is afebrile, blood
( D ) S t ress-induced cardiomyopa t hy
pressure is 138/92 mm Hg, pulse rate is 82/min, and respi­
ration rate is 14/min. BM! is 27. The remainder of the exam­
ination is unremarkable. Item 1 2 0
Exercise electrocardiographic stress testing shows
A 37-year-old man is evaluated for a 6-month history of
1.5-mm ST-segment depressions in leads II, III, and a VF; in
exercise intolerance and shortness of breath when walking
addition, the patient developed chest pressure during this
up stairs. He has no significant medical history and takes no
test. He exercised 4 minutes and stopped because of chest
medications.
discomfort. Heart rate and blood pressure increased appro­
On physical examination, blood pressure is 140/70 mm
priately. Duke treadmill score is - 11.5.
Hg, pulse rate is 62/ min, and respiration rate is 16/min. Car­
Which o f the following i s t h e most appropriate next step in diac examination reveals an irregularly irregular rhythm.
management? An opening snap is heard after S 2 , followed by a grade 1/6
diastolic rumble at the apex.
(A) Begin aspirin, �-blocker, and statin and re-evaluate in
Electrocardiogram shows atrial fibrillation. Transtho­
2 weeks
racic echocardiographic findings are consistent with rheu­
(B) Cardiac catheterization matic valve disease, showing a mildly thickened mitral
(C) Dobutamine stress echocardiography valve with minimal calcification and mild restriction in
(D) Exercise myocardial perfusion imaging leaflet motion. ·n1e subchordal apparatus is mildly thick­
ened, and there is mild mitral regurgitation and marked left
atrial enlargement. Mean gradient across the rnitral valve
CJ I t e m 1 1 9 is 13 mm Hg. Mitra! valve area is 1.2 cm2. Transesophageal
echocardiogram shows no left atrial appendage thrombus
A 26-year-old woman is evaluated i n t h e emergency depart ­ and confirms transthoracic echocardiographic findings.
ment for progressive dyspnea . She is 2 weeks postpa rtum .
T he pregna ncy was com p l icated by preec l a m ps i a but In addition to anticoagulation therapy, which of the follow­
resul ted in a norm a l del ivery. The i n fa n t is hea l t hy. She has ing is the most appropriate management?
no h istory of cardiovascular d isease. Her o n ly medicat ion is (A) Medical management; repeat echocardiogram in
prena t a l vitamins. 6 months
On physical exa m i n a t i o n . t h e patient is alebri le.
(B) Mitra! valve replacement
Blood pressure is 100/70 mm Hg i n bot h arms. pu lse rate is
! OS/ m i n and regular. and respi ra t ion rate is 25/ m i n . BMI (C) Percutaneous mitral balloon valvuloplasty
is 29. T h e oxygen satura tion on ambient a i r is 96%. ·n1e (D) Surgical mitral valve repair

�J,:-�tv-4 � �

�;;:-- ��
��"i���

ITEM 1 1 9

1 55
Answers and Critiques

Item 1 Answer: B The most appropriate management of this patient is to obtain


Educational Objective: Diagnose pulmonary valve high-sensitivity C-reactive protein (hsCRP) levels. He has an
regurgitation as a late consequence oftetralogy ofFallot intermediate risk of myocardial infarction and coronary death
repair. (5% to below 7.5% as defined by the Pooled Cohort Equations) .
The measurement o f hsCRP has been proved t o b e useful for
The most likely diagnosis is pulmonary valve regurgitation. guiding primary prevention strategies in intermediate-risk
Tetralogy of Fallot comprises a large subaortic ventricular sep­ patients, with as many as 30% of patients being reclassified
ta! defect, infundibular or valvular pulmonary stenosis, aortic as either low risk or high risk based on hsCRP measurement.
override, and right ventricular hypertrophy. Pulmonary regur­ When used for this purpose, the CRP assay should be able
gitation is the most common structural disorder that occurs to detect levels to at least 0.03 mg/L (high sensitivity); a single
following tetralogy ofFallot repair. The clinical findings include test is appropriate in patients with levels below 1.0 mg/L, but
features of right heart volume overload with a parasternal testing should be repeated in 2 weeks for values of 1.0 mg/L or
(right ventricular) lift and a soft systolic pulmonary outflow higher to assess for persistent elevation. Patients with hsCRP
murmur. There is a single S2 because the function of the pul­ measurement below 1.0 mg/L are considered at a low relative
monary valve is sacrificed during repair. The diastolic murmur risk for coronary heart disease and those with levels of3.0 mg/L
heard at the left sternal border that increases in intensity with or higher are considered at a high relative risk. A meta-analysis
inspiration is caused by pulmonary valve regurgitation. from the Emerging Risk Factors Collaboration in 2010 found that
Aortic valve regurgitation can occur late in patients fol­ hsCRP levels have a strong linear association with both ischemic
lowing tetralogy of Fallot repair owing to progressive aortic stroke and vascular mortality. Although evidence is not strong
enlargement. The aortic diastolic murmur is generally heard that modification of risk can occur with treatment after hsCRP
at the left sternal border, but it decreases in intensity with measurement, the JUPITER study randomized patients with
inspiration. A right ventricular prominence would not be serum LDL cholesterol levels below 130 mg/dL (3.37 mmol/L)
expected in a patient with aortic regurgitation. and hsCRP levels greater than or equal to 2.0 mg/L to rosu­
Recurrent ventricular septa! defect also can occur in vastatin or placebo. Patients were followed for the occurrence
patients following tetralogy ofFallot repair. However, the phys­ of death, myocardial infarction, stroke, or a composite of first
ical examination findings would include a systolic murmur major cardiovascular event for 5 years. In addition to lowering
heard at the left sternal border, which often obliterates the S1 serum LDL cholesterol levels from 108 to 55 mg/dL (2.80 to
and S2• A right ventricular prominence would not be expected 1.42 mmol!L) and hsCRP from 4.2 to 2.2 mg/L, rosuvastatin sig­
in a patient with a recurrent ventricular septal defect. nificantly reduced the incidence of major cardiovascular events.
Tricuspid valve regurgitation may occur late in patients Because this patient is asymptomatic, adenosine cardiac
following tetralogy of Fallot repair owing to progressive magnetic resonance (CMR) imaging, cardiac CT angiography,
right ventricular and annular dilatation from long-standing and stress echocardiography are not indicated and have not
severe pulmonary valve regurgitation. The physical exam­ been associated with reduction in cardiovascular events.
ination findings include a holosystolic murmur located at There is currently no role for the evaluation of lipid
the lower left sternal border that increases with inspiration. particle size and number (fractionated lipoprotein profiling) .
The systolic murmur in the patient presented here is soft and No studies to date have shown that treatment targeted to
heard over the pulmonary valve region. lipoprotein particle size and number affects outcomes, and
the use of these tests is not addressed in current cholesterol
KEY POINT
management guidelines.
• Pulmonary regurgitation is the most common structural
disorder that occurs following tetralogy of Failot repair. KEY POINT
• I n patients with an intermediate risk of cardiovascular
Bibliography disease, the measurement of high-sensitivity C-reactive
Ferraz Cavalcanti PE. Sa M P. Santos CA. et al. Pulmonary valve replacement protein (hsCRP) has been proved to be useful for guid­
after operative repair of tetralogy of Fallo!: meta-analysis and meta­
regression of 3,118 patients from 48 studies. J Am Coll Cardiol. 2013 Dec ing primary prevention strategies, with as many as 30%
10;62(23) :2227-43. [PMID: 24080109] of patients being reclassified as either low risk or high
risk based on the hsCRP measurement.

Item 2 Answer: D
Bibliography
Educational Objective: Use high-sensitivity C-reactive
Ridker PM, Danielson E, Fonseca FA, el al: JUPITER Study Group. Rosuvastatin
protein level to guide treabnent and cardiac risk stratification to prevent vascular events in men and women with elevated (-reactive
in a patient at intermediate risk of cardiovascular disease. protein. N Engl J Med. 2008 Nov 20:359(21):2195-207. [PMID: 18997196]

1 57
Answers a n d C ritiq u e s

c a 58-year-old woman with a prior transient ischemic attack


ltem 3 Answer:
(TIA) and hypertension. Accordingly, her CHA2DS2-VASc risk
Educational Objective: Evaluate a patient with a
score is 4 (2 points for TIA, 1 point for hypertension, and 1
diastolic murmur.
point for female sex) . Current guidelines advocate oral anti­
TI1is asymptomatic patient has a diastolic murmur and coagulation for a ny patient with nonvalvular atrial fibrillation
should undergo transthoracic echocardiography (TIE) . '!TE is and a CHA2DS2-VASc score greater than 1 . She has a history of
indicated for asymptomatic patients with a systolic murmur a central nervous system event, and her annual risk of stroke
that is grade 3/6 or higher, a late or holosystolic murmur, or is high (greater than 5% a1mually) . Therefore, she should be
a diastolic or continuous murmur; TIE is also indicated for continued on anticoagulation with warfarin.
patients with a murmur and accompanying symptoms. Concomitant aspirin therapy with warfarin is reserved
Chest CT may be helpful in the further evaluation of patients for patients with active coronary a rtery disease. This patient
with documented aortic valve pathology (such as identification has risk factors for atherosclerosis, but she does not have a
of a bicuspid valve or screening for concomitant aortopathy in a history of coronary artery disease or acute coronary syn­
patient with a bicuspid aortic valve), but it would not be the first dromes. The addition of aspirin to warfarin significantly
step in identifying the cause of this patient's mmmur and would increases the risk of bleeding.
expose the patient to wmecessary radiation. Discontinuation of warfarin and substitution with aspi­
Exercise echocardiography stress testing is most com­ rin or dual antiplatelet therapy is not correct. Aspirin is insuf­
monly used to evaluate for coronary heart disease in those ficient therapy for a patient at high risk of stroke. The Atrial
ineligible for electrocardiographic stress testing, although in fibrillation Clopidogrel Trial with lrbesartan for prevention
selected patients it may be helpful in evaluating the hemo­ of Vascular Events (ACTIVE-W) trial compared warfarin with
dynamic significance of valvular disease. Diastolic murmurs therapy with aspirin and clopidogrel and fow1d that aspirin
tend not to vary with exercise; therefore, exercise echocar­ with clopidogrel was inferior to warfarin for stroke preven­
diography stress testing would not be an appropriate initial tion with no statistically significant diflerence in bleeding.
test to help clarify the cause of the murmur. Anticoagulation with a novel oral anticoagulant (such as
Although there are multiple benign causes of systolic dabigatran, rivaroxaban , or apixaban) could be considered;
murmurs, diastolic murmurs are virtually always considered however, these agents have been associated with i ncreased
abnormal. TI1erefore, further evaluation of this patient's car­ gastrointestinal bleeding compared with warfarin.
diac findings is needed.
KEY POINT
KEY POINT
• Stroke prevention therapy after catheter ablation of
• Transthoracic echocardiography is indicated for atrial fibrillation should be based upon risk stratifica­
asymptomatic patients with a systolic murmur that is tion, not heart rhythm status.
grade 3 / 6 or higher, a late or holosystolic murmur, or
a diastolic or continuous murmur and for patients Bibliography
with a murmur and accompanying symptoms. Calkins H. Kuck KH. Cappa to R. et al: Heart Rhythm Society Task Force on
Catheter and Surgical Ablation of Atrial Fibrillation. 2012 HRS/EHRA/
ECAS expert consensus statement on catheter and surgical ablation of
Bibliography atrial fibrillation: recommendations for patient selection. procedural
Salazar SA, Borrero JL, Harris DM. On systolic murmurs and cardiovascular techniques. patient management and follow-up. definitions. endpoints,
physiological maneuvers. Adv Physiol Educ. 2012 Dec:36(4):251 -6 . and research trial design: a report of the Heart Rhythm Society (HRS)
[PMID: 23209004] Task Force on Catheter and Surgical Ablation of Atrial Fibrillation.
Developed in partnership with the European Heart Rhythm Association
(EHRA). a registered branch of the European Society of Cardiology (ESC)
and the European Cardiac Arrhythmia Society (ECAS): and in collabora­
tion with the American College of Cardiology (ACC), American Hean
ltem 4 Answer: A Association (AHA), the Asia Pacific Heart Rhythm Society (APHRS), and
Educational Objective: Manage thromboembolic risk the Society ofThoracic Surgeons (STS). Endorsed by the governing bod­
ies of the American College of Cardiology Foundation, the American
following atrial fibrillation ablation. Heart Association, the European Cardiac Arrhythmia Society, the
European Heart Rhythm Association. the Society of Thoracic Surgeons.
This patient should continue taking warfarin. She has a history the Asia Pacific Hearl Rhythm Society, and the Heart Rhythm Society.
Hean Rhythm. 2012 Apr; 9(4):632-696.e21. [PMID: 22386883]
of symptomatic atrial fibrillation and is now symptom-free
without evidence of recurrent atrial fibrillation after catheter
ablation. However, patients with successful ablation and elim­
Item 5 Answer: A
ination of symptoms may have transient asymptomatic atrial
Educational Objective: Treat a patient with hypertro­
fibrillation with continued risk tor atrial fibrillation-associ­
phic cardiomyopathy.
ated thromboembolic disease. '01erefore, current consensus
recommendations counsel that stroke prevention therapy Hydrochlorothiazide should be discontinued in this patient.
following atrial fibrillation ablation be based on risk factors She has mild symptoms of obstructive hypertrophic cardiomy­
and not rhythm status, with the preferred risk stratification opathy (HCM ) . Dynamic left ventricular outflow tract (LVOT)
tool being the CHA2DS2-VASc risk score, which has improved obstruction affects 70% of patients with HCM and is exac­
predictive ability relative to the CHADS2 score. The patient is erbated by decreases in preload (for example, from diuretic

1 58
Answers a n d Critiques

use or squat-to-stand maneuvers) and afterload (for example,


from expiration or vasodilator therapy) and by increases in
myocardial contractility (for example, from digoxin therapy) .
For patients with symptoms of HCM, the initial treatment is
medjcal therapy that addresses the factors that predispose
toward LVOT obstruction. Use of hydrochlorothiazide and
other iliuretics, particularly in rugh doses, will exacerbate the
propensity towards dynamic LVOT obstruction and, therefore,
should be avoided in patients with HCM. Similarly, vasodilator
therapy also should not be used. Negative inotropic agents,
such as �-receptor antagonists, calcium channel blockers, and
djsopyrarude, are the cornerstone of meilical therapy in these
patients. Th_is patient was previously stable on hydrochlorotru­
azide; however, HCM and its associated pathophysiology can
manifest at any age and result in symptoms of heart failure.
Dual-chamber pacemaker implantation previously was
considered to be a therapeutic option but was found to be rel­
atively ineffective in randomjzed clirucal trials of patients with
HCM. These trials showed a significant placebo effect (40%-
50%) from dual-chamber pacing without clear clinical benefit.
com m o n ly i n t he descending aorta, which may reflect a
ACE inhibitors, such as lisinopril. reduce afterload and
grea ter burden of a t heromatous d i sease at t his s i te. Eleva t ion
can exacerbate LVOT obstruction, and therefore they should
of the D-dimer level frequen t ly accompanies acute aort ic
be avoided in patients with HCM.
synd romes. Uncomplirnted type B acute aortic i n j u ry is best
For patients vvith drug-refractory, severely symptomatic
t reated med ical ly. i n i t i a l ly w i t h �-bl ockade to decrease t he
obstructive HCM, septal reduction therapy with surgical myec­
heart rate to below 6 0/m i n fol lowed by a p a re n t e ral arterial
tomy or alcohol septal ablation may be considered. Neither
vasod i lator as needed to con t rol b lood pressure.
myectomy nor alcohol ablation is recommended for patients
Chest pa i n is a com mon presen t i ng symptom i n t he
with the mmimal symptoms that tills patient has or for those
emergency depart ment. and t h ree potentially lethal causes
without an adequate trial of appropriate pharmacotherapy
must be rapi d ly excluded: acute myocardial i n fa rct ion . pul ­
KEY P O I NT monary embolism, and an acute aortic syndrome. An ticoagu­
• Diuretics, particularly in rugh doses, will exacerbate lalion t herapy would be considered standard of care if an acute
the propensity towards dynaruc left ventricular out­ corona1y syndrome were suspected . I n t h is palient. a lthough

flow tract obstruction and, therefore, should be a non-ST-eleva t ion myocardial in farction can not be excluded
based on t he electrocardiogram, it is less l i kely given his nor­
avoided in patients with hypertrophic cariliomyopathy.
mal tropon i n I level : the t ropon i n level wou l d l i kely be elevated
arter 3 hours of ongoing cardiac ischemfa. TI1e patient also has
Bibliography
a low pretest probabi l i ty or pul monary embolism by Wel l s
Gersh BJ. Maron, BJ. Bonow. RO, et al: American College of Cardiology
Foundation/American Heart Association Task Force on Practice Guidelines. scoring. Given h i s c l i n ical a n d rad iograph ic presentation. nei­
20ll ACCF/ AHA Guideline for the Diagnosis and Treatment of Hypertrophic t her al Lerna l ive d iagnosis to an acute aortic syndrome is l ikely,
Cardiomyopathy: a report of the American CoUege ofCardiology Foundation/
American Heart Association Task Force on Practice Guidelines. Developed a nd a n ticoagulation is not indicated in acute aortic syndrome.
in collaboration with the American Association for Thoracic Surgery, Mal perfusion syndromes a re uncommon w i t h an aor­
American Society of Echocardiography, American Society of Nuclear
Cardiology, Heart Failure Society of America, Heart R hythm Society. Society tic penet rating a t herosclerotic u l cer but cou l d occur w i t h
for Cardiovascular Angiography and Interventions. and Society of Thoracic transformation to classic cl issecl ion. In such cases, emergency
Surgeons. J Am CoU Cardiol. 20U Dec l3;58(25):e212-60. [PMID: 22075469]
enclovascu lar ste n t i ng with or wit hout fenestration may be
necessary. This patient shows no evidence of l ower l i m b or

Cl Educational Objective:
Item 6 Answer: c
Treat a penetrating atheroscle­
viscernl mal perfusion . and cat heterizal ion with endovascular
repa i r is not ind icated. Emergency open surge1y is rarely indi­
rotic ulcer in the descending aorta. cated i n t h is setting w i t h t he advent ofendovascular therapies.

TI1e most appropriate management for t h is patient is i n t rave­ KEY P O I NT


nous �-blockade fol l owed by i n t ravenous sodi u m n i t roprus­ • Uncomplicated type B acute aortic injury is best treated
side. The CT scan (shown. see top of' next col u m n ) reve;ils <J
meilically, iilltially with �-blockade followed by a par­
focal penetra t i ng a t h erosclero t i c u lcer ( PAU) i n t h e prox i m a l enteral arterial vasodilator to control blood pressure.
descend i ng aorta ( a rrow) . a type B acute aortic syndrome.
PAU is a focal defect or lesion occurri ng at t he site of' an
Bibliography
i n t i ma l atherosclerotic plaque. Pa t ie n ts tend to be older a n d
Nienaber CA. Powell JT. Management of acute aortic syndromes. Eur Heart
w i t h greater cardiovascu lar comorbidity. PAU occurs most J. 2012 Jan :33 (1 ) :26-3Sb. [PMID: 21810861]

1 59
Answers a n d Critiques

Item 7 Answer: B that statin therapy be initiated in patients at high risk for
Educational Objective: Diagnose effusive constrictive
CHO. The intensity of the statin therapy should be tailored
to the CHO risk. Candidates for high-intensity statin therapy
pericarditis.
include:
The most likely diagnosis in this patient is effusive constric­
tive pericarditis. Effusive constrictive pericarditis is a clini­ • Patients with known atherosclerotic disease (clinical CHO,

cal entity in which patients who have a pericardia! effusion, cerebrovascular disease, or petipheral artetial disease)
with or without cardiac tamponade, experience persistent • Patients with an LDL cholesterol level 190 mg/dL
symptoms and hemodynamic derangements after treat­ (4. 92 mmol/L) or greater
ment and relief of the pericardia! effusion. In some patients
• Patients with diabetes mellitus, an LDL cholesterol level
with constrictive pericarditis, pericardia! inflammation
below 190 mg/dL (4.92 mmol/L) , and calculated
results in an effusion, which is placed under pressure by
10 -year CHO risk of7.5% or higher
the inelastic pericardium. Symptoms of low cardiac output,
systemic congestion, and an elevated jugular venous pulse • Some patients without diabetes with an LDL cholesterol
are seen, as in constrictive pericarditis; however, in patients level below 190 mg/dL (4. 92 mmol/L) and calculated
with effusive constrictive pericarditis, a pericardia! knock 10-year CHO risk of7.5% or higher
is absent and the y descent of the jugular venous pulse may
Moderate-intensity statin therapy can be considered
be less prominent. Additionally, pulsus paradoxus may be
for:
present, which is not a typical finding of constrictive peri­
carditis. T11is disorder is caused by pericarditis involving • Patients with diabetes who are not receiving high­
the visceral layer of the pericardium. Thickening of the intensity therapy
visceral layer of the pericardium can be difficult to detect • Most patients without diabetes with an LDL cholesterol
with CT and other noninvasive imaging, and a high index
level below 190 mg/dL (4. 92 mmol!L) and calculated
of clinical suspicion is necessary to establish the diagnosis.
10-year CHO risk of7.5% or higher
In this patient, effusive constrictive pericarditis is suggested
by the persistently elevated right atrial pressure following • Some patients without diabetes with an LDL choles­
pericardiocentesis. terol level below 190 mg/dL (4.92 mmol/L) and cal­
Similar findings in the jugular venous pulse can occur culated 10-year CHO risk of 5 % or higher but lower
in patients with cor pulmonale or heart failure. With cor than 7 . 5 %
pulmonale, however, evidence of right ventricular dysfunc­
This patient has diabetes, an LDL cholesterol level less
tion or chamber enlargement is usually seen on imaging.
than 190 mg/dL (4.92 mmol/L), and a calculated 10-year
The clear lungs on auscultation do not support the presence
CHO of 10%, and, therefore, should be considered for
of heart failure.
high-intensity statin therapy. Drugs and doses that consti­
The presence of a normal electrocardiogram and the
tute high-intensity statin therapy include atorvastatin, 40
absence of the typical symptom of chest pain argue against
to 80 mg/d; rosuvastatin, 20 to 40 mg/d ; and simvastatin,
recurrent acute pericarditis as a diagnosis. Additionally,
80 mg/d. (The FDA has issued a warning regarding the inci­
acute pericarditis cannot explain this patient's pulsus par­
dence of muscle injury with products that contain 80 mg of
adoxus and elevated jugular venous pulse.
simvastatin and recommends that patients be switched to a
KEY POINT different statin rather than increasing the dosage of simvas­
tatin to 80 mg/d.)
• Effusive constrictive pericarditis is a clinical entity in
Fluvastatin, 40 mgld; lovastatin, 20 mgld; p ravasta­
which patients who have a pericardia! effusion expe­
tin, 10 mg/d; and simvastatin, 10 mg/d, are all classified
rience persistent symptoms and hemodynamic
as low-intensity dosing and are inadequate to reduce this
derangements after relief of the pericardia! effusion.
patient's CHO risk.

Bibliography KEY POINT


Syed FF. Ntsekhe M. Mayosi BM, Oh JK. Effusive-constrictive pericarditis. • Patients with diabetes mellitus should receive moder­
Heart Fail Rev. 2013 May:I8(3):277-87. [PMID: 22422296]
ate- or high-intensity statin therapy to reduce their
risk of coronary heart disease.
Item s Answer: B
Educational Objective: Manage high LDL cholesterol Bibliography

level in a patient with diabetes mellitus and an elevated Stone NJ. Robinson JG, Lichtenstein AH. et al: American College of
Cardiology/American Heart Association Task Force on Practice
risk for coronary artery disease. Guidelines. 2013 ACC/AHA guideline on the treatment of blood choles­
terol to reduce atherosclerotic cardiovascular risk in adults: a report of
The most appropriate management in this patient with a the American College of Cardiology/American Heare Association Task
Force on Practice Guidelines. Circulation. 2014 Jun 24;129(25 Suppl 2):5 1 -
coronary heart disease (CHO) risk equivalent is to switch
45. Erratum i n : Circulation. 2014 Jun 24:129(25 Suppl 2):S46-8. [PMID:
to atorvastatin, 40 mg/d. Current guidelines recommend 24222016]

1 60
Answers a n d Criti q u es

ltem 9 Answer: A Bibliography


Educational Objective: Manage ACE inhibitor side Heran BS. Musini VM, Bassett K. Taylor RS. Wright JM. Angiotensin receptor
blockers for heart failure. Cochrane Database Syst Rev. 2012 Apr
effects in a patient with heart failure. 18:4 :CD003040. [PMID: 22513909]

The most appropriate treatment for this patient is to


switch from her ACE inhibitor ( lisinopril) to an angioten­
Item 1 0 Answer: D
sin receptor blocker (ARB) , such as valsartan, and observe
her closely for several weeks to see if her cough resolves. Ed ucational Objective: Manage incidental finding of

For patients with heart failure, the two most common atrial septa) aneurysm with patent foramen ovale.

causes of a nonproductive cough are volume overload No further evaluation or treatment is the appropriate manage­
and ACE inhibitors. An ACE inh ibitor-induced cough ment approach for this patient with an incidentally discov­
may occur at any time after an ACE in hibitor has been ered atrial septa! aneurysm. Atrial septa! aneurysm is redun­
initiated, and it would not be surprising that she could dant atrial septa! tissue that is often associated with a patent
develop an ACE inhibitor cough after 9 months of therapy. foramen ovale. When atrial septa! aneurysm is identified inci­
The most appropriate course of therapy, therefore, would dentally, no medical treatment or intervention is needed.
be to switch to an ARB. Although data regarding mortality Antiplatelet therapy is recommended for patients with cryp­
outcomes in patients with heart failure taking ARBs are togenic stroke and an isolated atrial septal aneurysm.
limited, results thus far demonstrate equivalent mortality In patients with an atrial septa! aneurysm and recurrent
outcomes and fewer medication-related adverse events in stroke while taking antiplatelet therapy. anticoagulant ther­
this setting. apy is recommended if no other cause of stroke is identified.
An echocardiogram evaluates left ventricular function Rarely, surgical excision of an atrial septa] aneurysm
and valvular abnormalities. It would not be helpful in a and defect closure is considered in patients in whom anti­
patient with diagnosed heart failure to assess for volume platelet or warfarin therapy fails to prevent stroke recurrence
overload. Indications for repeating echocardiograms in or in patients with a large left-to-right shunt causing right
patients with heart failure include a decline in functional heart enlargement. Percutaneous device closure is rarely
status and to reassess function after uptitrating medications. performed in patients with atrial septal aneurysms, because
Additionally, in patients followed over time, repeating an a large device is required to plicate the atrial septa! aneurysm
echocardiogram every 2 to 3 years is indicated to assess for and close multiple fenestrations.
further left ventricular dilation and evaluate left ventric­ Atrial septa! aneurysms are most commonly detected by
ular ejection fraction for further decline. It is unlikely to transesophageal echocardiogram. However, when an inci­
be helpful in this patient, who has no evidence of volume dental atrial septa! aneurysm is well visualized by trans­
overload on examination and no pulmonary edema on chest thoracic echocardiogram, additional imaging with a trans­
radiograph. esophageal echocardiogram is not needed.
B-type natriuretic peptide (BNP) level is useful for
KEY POINT
the assessment of acute dyspnea. Studies have shown that
levels of BNP are elevated in patients with heart failure. • When an atrial septa! aneurysm is identified inciden­
Additionally, higher levels are associated with an increase tally, no further evaluation, medical treatment, or
in mortality. BNP has not been demonstrated to be useful intervention is needed.
to guide diuresis in patients with heart failure. In a patient
with a history of heart failure, a random BNP level would Bibliography
not help in the assessment of fluid overload being the cause Burger AJ. Sherman H B, Charlamb Ml. Low incidence of embolic strokes
with atrial septal aneurysms: A prospective. long-term study. Am Heart
of a cough.
J. 2000 Jan: 139(1 Pt 1 ) : 149-52. [PMID: 10618576]
Pulmonary function testing is useful primarily in the
evaluation of dyspnea or to assess for the presence of under­
lying lung disease that might contribute to cough, such as
Item 1 1 Answer: B
cough-variant asthma. However, this patient has no clinical
history or increased risk for lung disease, is a never-smoker, Ed ucational Objective: Manage a patient with stable

and has a normal lung examination. 111erefore, pulmonary angina pectoris and a low-risk stress test result.

function testing would not be appropriate prior to a trial of In this patient with ongoing stable angina pectoris and a
medication adjustment. low-risk exercise stress test result (that is, a Duke tread­
KEY POINT mil l score of +6) , initiation of a long-acting nHrate such as
isosorbide mononitrate is recommended by current guide­
• An ACE inhibitor-induced cough may occur at any
lines. �-Blockers and nitrates improve functional capacity,
time after an ACE inhibitor has been initiated; the
delay onset of exercise-induced myocardial ischernia, and
best course of therapy is to switch to an angiotensin
decrease the frequency and severity of anginal episodes. Most
receptor blocker. patients with stable angina will require combination therapy
with these two classes of dmgs to achieve effective control of

1 61
Answers a n d Critiq u es

angina1 symptoms. Although this patient is at low risk, initi­ with congenital lesions such as aortic coarctation, interrupted
ation of medical therapy is also appropriate in patients who aortic arch, and Turner syndrome. More than 70% of patients
are at intermediate risk and high risk based on clinical risk with a bicuspid aortic valve will require surgical intervention
factors, symptom burden, and/or stress testing results. Those for a stenotic or regurgitant valve or aortic pathology over
patients with intermediate-risk stress test findings (Duke the course of a lifetime. The presence of a bicuspid aortic
treadmill score of -10 to +4) and high-risk stress testing find­ valve increases the risk for aortic stenosis or regurgitation,
ings (Duke treadmill score of less than - 11) have a 1 % to 3% and stenosis proceeds at a faster rate when the aortic valve is
cardiovascular mortality per year and a 3% or higher cardio­ bicuspid. The risk for infective endocarditis also is increased in
vascular mortality rate per year, respectively. these patients. In addition, bicuspid aortic valve is associated
Calcium channel blockers, such as diltiazem, are sec­ with aortopathy and a predisposition to aneurysm formation
ond-line therapy in patients with stable angina pectoris who and thoracic aortic dissection.
are intolerant of �-blockers or who have continued symp­ Adults with previously undiagnosed aortic coarcta­
toms on �-blockers and nitrates. This patient is tolerating his tion may present with hypertension or a murmur. Pal­
�-blocker well and is not yet taking a nitrate for his angina. pation of reduced femoral pulses and measurement of
Therefore, diltiazem is not indicated at this time. discrepant blood pressures during routine examination are
Because of the invasive nature of coronary angiography helpful in raising suspicion for the diagnosis. The murmur
and the inherent risks of vascular complications, it should associated with coarctation may be nonspecific but is usu­
be reserved for patients with lifestyle-limiting angina despite ally a systolic murmur in the left infraclavicular area and
optimal medical therapy or high-risk criteria on noninvasive under the left scapula.
stress testing such as significant ST-segment depression at a The murmur associated with an atrial septa1 defect is a
low work load, ST-segment elevation. or hypotension. midsystolic flow murmur caused by the ejection of increased
Pharmacologic nuclear stress testing is not indicated right-sided volume, owing to the left-to-right shunt that
in this patient owing to the presence of stable symptoms, occurs initially with this defect. This murmur is best heard
lack of optimal medical therapy, and low-risk findings on over the pulmonic area of the chest and may radiate toward
exercise stress testing. the back, as with the murmur of pulmonary stenosis. The
most characteristic finding on auscultation in patients with
KEY POINT
an atrial septa! defect is a fixed split S 2 .
• First-line antiangina1 therapy for patients with stable The murmur of mitra1 stenosis is a diastolic low-pitched
angina pectoris is �-blockers and nitrates. decrescendo murmur heard best in the left lateral decubitus
position. With mitral stenosis. S 1 has increased intensity
Bibliography and S 2 is normal. The opening snap, which is due to forceful
Qaseem A. Fihn SD, Dallas P. Williams S. Owens DK, Shekelle P; Clinical opening of the mitral valve, occurs 1..vhen the pressure in the
Guidelines Committee of the American College of Physicians.
Management of stable ischemic heart disease: summary of a clinical left atrium is greater than the pressure in the left ventricle.
practice guideline from the American College of Physicians/ American As the severity of the mitra1 stenosis increases, the pressure
College of Cardiology Foundation /American Heart Association/American
Association for Thoracic Surgery/ Preventive Cardiovascular Nurses
in the left atrium increases, and the mitra1 valve opens ear­
Association/Society of Thoracic Surgeons. Ann Intern Med. 2012 Nov lier in ventricular diastole.
20;157(10) :735-43. [PMID : 231656651
KEY POINT
• A bicuspid aortic valve is often discovered inciden­
Item 1 2 Answer: C
tally; the murmur depends on the degree of valve dys­
Educational Objective: Diagnose bicuspid aortic valve. function, with a systolic ejection murmur that may
This patient most likely has a bicuspid aortic valve. Bicuspid range from a minimal flow disturbance to findings
aortic valve is the most common congenital heart lesion, consistent with the murmur of aortic stenosis as the
occurring in approximately 0.5% to 2% of the general popu­ degree of outflow obstruction increases.
lation. It is the second most common cause of aortic stenosis
after ca1cific degeneration of a tricuspid aortic valve and the Bibliography
second most common cause of aortic regurgitation after aor­ Siu SC, Silversides CK. Bicuspid aortic valve disease. J Am Coll Cardiol. 2010
Jun 22:55(25):2789-800. [PMID: 20579534]
tic root dilation. Many patients with a bicuspid aortic valve
are asymptomatic, with the diagnosis being suggested based
on incidentally noted auscultatory findings. The presenting Item 1 3 Answer: c
murmur depends on the degree of valve dysfunction, with a
Educational Objective: Recommend influenza vaccina­
systolic ejection murmur that varies in intensity, ranging from
tion for the secondary prevention ofischemic heart disease.
minimal flow disturbance to findings consistent with the
murmur of aortic stenosis as the degree of outflow obstruc­ Providing this patient with an annual influenza vaccination
tion increases. A diastolic murmur may occur if aortic valve would significantly reduce her risk of future cardiovascular
incompetence with regurgitation is present, as in this patient. events. A meta-analysis of randomized trials demonstrated
Bicuspid aortic valve has an increased prevalence associated that use of the influenza vaccine was associated with a 36%

1 62
Answers a n d Critiq u es

lower risk of major adverse cardiovascular events compared interval of 630 ms: a normal QTc is defined as 460 ms or less
with nonimmunized patients. Based on these data, the Amer­ in women and 440 ms or less in men. A QTc greater than 500
ican Heart Association and American College of Cardiology ms is associated wit h increased risk of torsades de pointes.
recommend influenza vaccination for the secondary preven­ ll1e cause of t his patient's QT prolongation is l i kely
tion of ischemic heart disease. In addition to influenza vac­ her pharmacot herapy. Many drugs have been impl icated in
cination as a preventive measure for cardiovascular disease, QT-interval prolongation . including antiarrhythmic agents.
this patient also qualifies for influenza vaccination according antibiotics (including some macrol ides and fluoroquino­
to Advisory Committee on Immunization Practices (ACIP) lones ) , ant ipsychotic d rugs. and ant idepressants. A Ust of
guidelines that recommend all persons aged 6 months or QT-prolonging drugs is avai lable at http: //crediblemeds.org/.
older receive the influenza vaccination. The large degree of" QTc prolongation in t h is patient may be
Colchicine has anti-inflammatory properties, and caused by either the presence of two QT-prolonging med­
observational studies of patients taking colchicine for gout ications or an underlying ion channel variant t hat is only
or familial Mediterranean fever suggest a decreased risk of evident in the setting of a QT-prolonging medication.
cardiovascular disease associated with treatment. However, A l t hough bot h medications may lead to her ECG find­
its use for secondary prevention of cardiovascular disease ings. discontinuing either the moxifloxacin or amitriptyline
has not been established. i n isolation is insu fficient treatment given the sign ificant
Folic acid lowers homocysteine levels, which have degree of QT prolongation on t his patient's ECG.
been associated with increased cardiovascular disease in Carvedi lol does not prolong t he QT interval directly.
observational studies. However, clinical trials examining although it may increase t he risk of bradycardia-related
the effectiveness o f lowering homocysteine levels by folic arrhythmias in patients with acquired QT- i n terval prolon­
acid supplementation have failed to show a reduction in ga tion by slowing t he heart rate. H owever, the most import­
adverse cardiovascular events. Folic acid supplementa­ ant i ntervention in this patient is to reduce the risk of
tion for this purpose is therefore not recommended as torsades de pointes by d iscont i nu ing the oflending QT- pro­
secondary prevention. longing agents.
Because inflammation and oxidative stress are involved
KEY POINT
in atherosclerosis, the use of antioxidant agents, including
vitamins E and C and P-carotene, has been proposed as both • A corrected QT interval greater than 500 ms is associ­
a primary and secondary preventive intervention for cardio­ ated with increased risk of torsades de pointes;
vascular disease. Although supported by some basic science patients with this degree of QT-interval prolongation
and observational data, several large, randomized controlled should discontinue any potentially QT-interval pro­
trials have failed to document benefit of antioxidant therapy longing drugs.
for either purpose. Therefore, the use of vitamin E supple­
mentation would not be appropriate in this patient. Bibliography
Isbister GK, Page CB. Drug induced QT prolongation: the measurement and
KEY P O I NT assessment of the QT interval in clinical practice. Br J Clin Pharrnacol.
2013 Jul;76(1}:48-S7. [PMID: 231675781
• Influenza vaccine should be administered to patients
with established cardiovascular disease to reduce the
risk of future cardiovascular events.
Cl
Item 1 5 Answer: c

Bibliography
Educational Objective: Manage heart failure with

Udell JA. Zawi R, Bhatt DL. et al. Association between innuenza vaccination preserved ejection fraction with diuretics.
and cardiovascular outcomes in high-risk patients: a meta-analysis.
JAMA. 2013 Oct 23;310(16):1711-20. [PMID: 241504671 ll1is patient should be admitted to the hospital and given
intravenous furosemide. H is presentation is characteristic for
heart failure with preserved ejection fraction ( H FpEF) . He has
volume overload manifested by i ncreasing abdominal girth,
Cl Educational Objective:
Item 1 4 Answer: c
i ncreased exertional dyspnea, and progressive orthopnea. His
Treat acquired QT-interval
left ventricular ejection fraction is normal, but he has mild left
prolongation.
ventricular hypertrophy and a long histo1y of hypertension.
Both amitriptyline and rnoxifloxacin should be discontinued Additionally, he has a markedly elevated B-type natriuretic
in t his patient. Her electrocardiogram (ECG) demonstrates peptide level. ll1e etiology of h is acute exacerbation i nto heart
QT- i nterval prolongat ion. Many medications may prolong the failure is most l i kely acute atrial fibrillation, but because he is

QT i nterval. including amitriptyline and moxifloxacin: QT al ready on diltiazem and has a normal heart rate. he may have
prolongation may be markedly increased in patients taking been in atrial fibrillation for some t i me and not noticed it.
more than one medication with this effect. I n con trast to patients with a reduced ejection fraction,
The corrected QT i nterval (QTc) is most often deter­ no drugs have been shown to reduce mortal i ty rates in
mined using the Bazett formula (QT interval I v'R-R i nter­ patients with H Fp EF. I nstead, guidelines emphasize con­
val ) . This patient's ECG at t he time of admission shows a QTc t ro l l i ng blood pressure and volume. Patients with H FpEF a re

1 63
Answers a n d Criti q u es

of volume contraction. the intracardiac fil l i ng pressures are


Cl
often volu111e sensitive. and careful use of diuretics to 111ain­
tain euvole111ia is i 111portant. Th is patient is not al ready lak­ low. and tamponade does not result in an elevation of esti­
CONT. mated central venous pressure. ·nierefore. several physical
ing a diuretic. and starting with a low dose of furose111 ide is
a reasonable approach. If the patient were already on a n oral examination findings usually associated with pericardia! ta111-
d i u retic. giving at least t he equivalent dose intravenously ponade. such as jugular venous distention and pulsus para­
would be suggested. Patients with 1-1 FpEF should be encour­ doxus. may not be evident in many patients. Low-pressure
aged to 111onitor t heir weight closely. as s111all d i fferences in cardiac tamponade may be caused by malignancy. tuberculo­
volu111e can quickly cause volume overload and subsequent sis. or other severe chronic i l lnesses that result in both dehy­
hospital admissions. dration and pericardia! effusions. with metastatic involve­
�-Blocker therapy is relatively contraindicated in t h is ment of the pericardium l i kely present in this patient with
pat ient with acute decompensated heart failure as it may known dissem i nated cancer.
exacerbate his heart fai l u re. Once his heart failure is success­ In cardiac tamponade. t he pericardia! effusion causes
fu l ly treated with diuretics. this pat ient may benefit fro111 i n t rapericardial pressure to exceed ven tricular d iastolic
�-blocker t herapy to help 111anage h i s heart rate and blood pressures. which leads to impair111ent in ventricular fi l ling
pressure. but this should be avoided in the setting of acute and stroke volu111e. Sinus tachycardia. as evident in this
volume overload. whenever possibl e. patient. is a compensatory response to maintain forward
Despite the fact that the patient is currently in a t rial cardiac output. Treatment should consist of acute i nt rave­
fibri llation . cardioversion at this point is incorrect . Because nous hydration to augment ventricular preload and stroke
he is hemodynam ically stable with good rate control. t here volume and. most i mportant ly. procedu res to relieve the
is no indica tion for i 111111ediate cardioversion. I n add i t ion. tamponade. speci fically pericardiocentesi s.
because it is unclear how long he has been in atrial fibri l la­ An intra -aortic balloon pump is used in patients with
t ion . cardioversion without a t ransesophageal echocardio­ hemodynamic instability. usually as a bridging device until
gra111 to rule out t h ro111bus or i n i t iation of prophylactic anti­ defi n i t ive treatment can be undertaken. Because removal of
coagulation would place t he patient at risk for embolization pericardia! fluid in this patient would be expected to mark­
a t the l i me of t he procedure. edly improve his hernodynamic status. pericardiocentesis is
Several small trials have suggested that alclosterone antag­ indicated prior to pursuing additional support ive t herapy.
onists may improve diastolic function in patients with H FpEF Phenylephrine is a potent vasoconstrictor. A lthough
However. a recent t rial co111pa1ing spironolactone with placebo phenylephrine may i mprove t h is pat ient's b lood pressure
shm,ved a reduction in hea1i failure hospitalizations but no by increasing his systemic vascu lar resistance. t he i ncrease
dif ference in mortality rates or all-cause hospitalizations in in resistance w i l l reduce stroke volume. which is a l ready
patients with H FpEF. and spironolactone was associated with sign i fica n t ly i mpai red by tarnponade and hypovolemia .
significant increases in serum creatinine and potassium levels. Phenyleph ri ne. t he refore. wou l d not be an appropriate
Given this minimal benefit but substantial i ncrease in risk of i ntervention prior to t reat i ng t he tamponade and ensuring
adverse efTects. the addition of spironolactone for his current adequate volume expa nsion.
symptoms is not appropriate in t h is patient with H FpEF. A window pericardiectomy procedure is performed in
t he operating room either as a n open procedure or using
KEY POINT
video-assisted t horacoscopy by cardiac surgery and would
• Patients with heart failure with preserved ejection be indicated only i f this patient's tamponade was unrespon­
fraction are often volume sensitive, and careful use of sive to pericardiocentesis. a less i nvasive bedside procedure.
diuretics to maintain euvolemia is important.
KEY P O I N T
Bibliography • Treatment of low-pressure cardiac tamponade should
Heart Failure Society of America; Lindenfeld J. Albert NM. Boehmer J P. et al. include acute intravenous hydration to augment ven­
H FSA 2010 Comprehensive Heart Failure Practice Guideline. J Card Fail. tricular preload and stroke volume and pericardio­
2010 Jun:L6(6) :el-el94. [PMID: 20610207)
centesis to relieve the tamponade.

Bibliography
Cl Educational
Item 1 6 Answer:
Objective:
B
Treat low-pressure cardiac tam ­
Sagrista-Sauleda J. Angel J. Sambola A. Alguersuari J. Permanyer-Miralda G.
Soler-Soler J. Low-pressure cardiac tamponade: clinical and hemody­
ponade. namic profile. Circulation. 2006 Aug 29:114(9):945-52. [PMID: 16923755]

ll1is patient should u ndergo pericardiocentesis. He has


low-pressure cardiac tamponade. which is tamponade occur­ Item 1 7 Answer: B
ring in the setting of clin ical dehydrat ion. Evidence of cardiac
Educational Objective: Identify the importance of indi­
tamponade on echocardiography includes diastolic inversion
vidual risk factors in the risk of myocardial infarction.
of the right-sided chambers and respirawry va1iation in the
mitral inflow pattern : a ventricular septaI shift and plethora of The presence of dyslipidemia places this patient at high­
t he i n ferior vena cava also may be present. However. because est risk for future myocardial infarction (MI), accounting for

1 64
Answers a n d Critiques

approximately 50% of the risk of developing acute Ml. The reasonable second-line therapy in patients who are
INTERHEART study assessed the prevalence of nine poten­ intolerant of �-blockers or who have continued symp­
tially modifiable risk factors in more than 15,000 patients toms on �-blockers and nitrates. However, it would not
with first acute MI and almost 15,000 asymptomatic age- and be appropriate to switch to a calcium channel blocker,
sex-matched controls. Nine risk factors were strongly associ­ such as amlodipine, in this patient who currently toler­
ated with acute MI in the 52 countries included in the trial. ates an effective dose o f a �-blocker.
In descending order, these are: dyslipidemia, smoking, psy­ Myocardial viability testing is performed with a radio­
chosocial stressors, diabetes mellitus, hypertension, obesity, nuclide radiotracer that is taken up by viable myocardial tis­
alcohol consumption, physical inactivity, and diet low in fruits sue. Viability testing may demonstrate hypoperfused regions
and vegetables. Results of the INTERHEART study suggest that of the heart that might show functional improvement if
these modifiable risk factors account for more than 90% of the revascularization is performed. However, information from
risk for acute Ml. a substudy of the Surgical Treatment of Ischemic Heart Fail­
KEY POINT ure (STICH) trial demonstrated no relationship between the
results of viability imaging and the effectiveness of bypass
• Nine modifiable risk factors account for more than
surgery. Therefore, in this patient who remains symptomatic
90% of the risk for acute myocardial infarction; in
despite optimal medical therapy and is a reasonable surgical
descending order, these are: dyslipidemia, smoking, candidate, revascularization is indicated, and myocardial
psychosocial stressors, diabetes mellitus, hyperten­ perfusion testing would not contribute significant informa­
sion, obesity, alcohol consumption, physical inactivity, tion regarding medical decision making. Myocardial perfu­
and diet low in fruits and vegetables. sion testing is typically limited to use in patients at high risk
for revascularization surgery in whom assessing the degree
Bibliography of viable myocardium present may influence the risk-benefit
Yusuf S, Hawken S, Ounpuu S. et al: INTERHEART Study Investigators. ratio of surgical treatment.
Effect of potentially modifiable risk factors associated with myocardial
infarction in 52 countries (the INTER H EART Study) : case-control study. KEY POINT
Lancet. 2004 Sep ll- 17;364(9438):937-52. [PM I D : 15364185]
• Coronary artery bypass grafting is recommended for
patients who remain symptomatic with optimal med­
ical therapy and have specific angiographic findings
Item 1 8 Answer: B
(either left main disease or multivessel disease with
Educational Objective: Manage a patient with diabetes
involvement of the proximal left anterior descending
mellitus and three-vessel coronary artery disease.
artery) , concomitant reduced systolic function, or dia­
The appropriate management of this patient is coronary artery betes mellitus.
bypass grafting (CABG). He has type 2 diabetes mellitus and
multivessel coronary artery disease (CAD) with moderate to Bibliography
severe symptoms despite optimal medical therapy. In multi­ Farkouh ME. Domanski M, Sleeper LA. et al: FREEDOM Trial Investigators.
ple observational studies and randomized controlled trials, Strategies for multivessel revascularization in patients with diabetes.
N Engl J Med. 2012 Dec 20:367(25):2375-84. [PMID: 23121323]
performing CABG compared with percutaneous coronary
intervention (PC!) as the initial revascularization strategy in
patients with a clear indication was associated with improved Item 1 9 Answer: C
outcomes, including reduced rates of death, myocardial
Educational Objective: Evaluate coronary artery disease
infarction (Ml) , and stroke. The FREEDOM trial evaluating
in a patient with COPD.
management of multivessel CAD in patients with diabetes
showed that the composite endpoint of death, MI, and stroke This patient should undergo dobutamine stress echocardi­
was significantly lower in patients treated with CABG versus ography. She has a history of coronary artery disease (CAD)
PC!. This difference was driven by a statistically significant with new atypical, but exertional, symptoms suggestive of
reduction in the occurrence of death and MI in CABG patients, cardiac ischemia. Because she has baseline electrocardiogram
although stroke rates were higher in the CABG group than the abnormalities (left ventricular hypertrophy with repolariza­
PC! group. CABG is recommended for patients who remain tion abnormalities) that will make interpretation of ST-seg­
symptomatic despite optimal medical therapy and have spe­ ment changes difficult, she should undergo stress testing with
cific angiographic findings (either left main disease or mul­ imaging, with either stress echocardiography with dobuta­
tivessel disease with involvement of the proximal left anterior mine or myocardial perfusion imaging with a vasodilator.
descending artery) , concomitant reduced systolic function, or With stress echocardiography, regional myocardial
diabetes mellitus. function is assessed in real time. Stress images are obtained
�-Blockers are first-line antianginal agents because at peak or immediately after stress, before cardiac func­
of their ability to reduce heart rate, myocardial contrac­ tion returns to baseline. Wall motion abnormalities indicate
tility, and blood pressure, resulting in reduced myo­ either infarction (seen on stress and rest images) or isch­
cardial oxygen demand. Calcium channel blockers are emia (seen on stress images only) . For patients who cannot

1 65
Answers and Critiques

exercise, such as this patient, pharmacologic stressors such associated with the procedure. Additionally, this patient
as dobutamine in combination with imaging can be used in is asymptomatic, so there is no indication for intervention
place of exercise and imaging. at present.
Because of her COPD with active wheezing on exam­ Surgical aortic valve replacement is indicated for
ination, pharmacologic testing with vasodilators should be symptomatic patients with severe aortic stenosis, asymp­
avoided. Pharmacologic vasodilators, such as dipyridamole, tomatic patients with severe aortic stenosis and LV systolic
adenosine, and regadenoson, can cause bronchospasm and dysfunction (LV ejection fraction <50%), and patients with
are therefore contraindicated in a patient who is actively severe aortic stenosis who are undergoing coronary artery
wheezing. These agents can be used with caution in a patient bypass graft or surgery on the aorta or other heart valves.
with a history of bronchospastic airways disease, but the This patient is asymptomatic with normal LV systolic func­
presence of active wheezing in this patient precludes the use tion, and he does not have any other cardiac procedures
of a vasodilator. Therefore, stress testing with dobutamine is planned.
the correct choice. Transcatheter aortic valve replacement (TAVR) is indi­
This patient has no symptoms to indicate an acute cor­ cated for patients with symptomatic severe aortic steno­
onary syndrome that would prompt cardiac catheterization sis who are considered unsuitable for conventional surgery
as the initial diagnostic test. Evaluation of the extent and because of severe comorbidities. Currently, TAVR should not
severity of disease would be the first step in deciding man­ be performed in patients with intermediate or low surgical
agement in this patient. If she has a small perfusion defect, risk, and no therapeutic intervention is currently indicated
she could be treated medically with more intensive antiangi­ in this asymptomatic patient.
nal therapies.
KEY POINT
KEY P O I NT • I n patients with asymptomatic severe aortic stenosis,
• Pharmacologic vasodilators, such as dipyridamole, close clinical follow-up with echocardiography every
adenosine, and regadenoson, can cause broncho­ 6 to 12 months is appropriate.
spasm during cardiac stress testing; these agents can
be used with caution in a patient with a history of Bibliography
COPD but are contraindicated i n a patient who is Manning WJ. Asymptomatic aortic stenosis in the elderly: a clinical review.
JAMA. 2013 Oct 9;310(14):1490-7. [PMID: 241043731
actively wheezing.

Cl
Bibliography Item 2 1 Answer: c
Qaseem A. Fihn SD, Williams S. Dallas P. Owens DK. Shekelle P; Clinical
Guidelines Committee of the American College of Physicians. Diagnosis Educational Objective: Diagnose stress cardiomyopathy
of stable ischemic heart disease: summary of a clinical practice guideline (takotsubo cardiomyopathy) .
from the American College of Physicians/American College of Cardiology
Foundation/American Heart Association/American Association for ·n1is patient's clinical history and presentation are consis­
Thoracic Surgery/Preventive Cardiovascular Nurses Association/Society
of Thoracic Surgeons. Ann Intern Med. 2012 Nov 20;157(10) :729-34. tent with stress cardiomyopathy (takotsubo cardiomyopa­
[PMID: 23165664] thy) . ll1e absence of coronary artery stenosis and t he pres­
ence of hypokinesis of t he mid and apical left ventricle on
Item 2 0 Answer: B ventriculography confirm this diagnosis. ll1is patient with
takotsubo cardiomyopathy without evidence of cardiogenic
Educational Objective: Manage asymptomatic severe
shock should be administered metoprolol and captopril. ll1e
aortic stenosis.
treatment of stress cardiomyopat hy is supportive. including
Although this patient has severe aortic stenosis based on t he use of �-blockers and ACE inhibitors. and most patients
quantitative echocardiographic findings, he is asymptom­ have resolution of symptoms and recovery of left ventricular
atic with normal left ventricular (LV) systolic function; function within 7 days.
therefore, follow-up echocardiography in 6 to 12 months Takotsubo cardiomyopat hy often mimics non-ST­
is the most appropriate management. Appropriate fol­ elevation myocardial infarction (NSTEMl) or ST-elevation
low-up in patients with asymptomatic severe aortic ste­ myocardial infarction (STEM!) . Patients present with chest
nosis includes a clinical evaluation and echocardiography pai n or short ness of breat h, e lectrocardiographic cha nges
every 6 to 12 months. Patients should also be educated to consistent with anterior and/or lateral ST-segment eleva­
identify and report possible aortic stenosis-related symp­ tion . and elevated cardiac biomarkers. Although not required
toms, such as dyspnea, reduced exercise tolerance, exer­ for diagnosis, many patients develop symptoms following a
tional chest pain, lightheadedness, and syncope, before stressful or emotional event. The diagnosis of stress cardiomy­
scheduled follow-up. opathy requires (1) ST-segment elevation on electrocardiogra­
Balloon valvuloplasty, although important in the phy. (2) transient wall motion abnormalities of the mid and
treatment of the pediatric patient with severe aortic ste­ apical left ventricle. (3) t h e absence of significant obstrnctive
nosis, has a more limited role in adults owing primarily coronary artery disease. and (4) the absence of other causes
to its limited efficacy and the high rate of complications of transient left ventricular dysfunction. such as myocarditis.

1 66
A nswers a n d Critiques

Cl
CONT.
E nd omyocard ial biopsy is genera l ly not indicated for
t he i n i t ial evaluation of heart failure unless a speciAc diag­
feet equally and a loud early systolic murmur heard at the
left sternal border related to the right ventricular outflow
nosis that wou ld i nfluence ma nagement or prognosis is tract obstruction. The pulmonic component of the S 2 is
suspected based on clinical data or noninvasive testing. This generally soft or absent depending on the degree of pul­
patient's presentation, with acute onset following a st ress fu l monary valve stenosis. Right ventricular hypertension but
event, ST-segment elevation, and hypokinesis of the cardiac not pulmonary hypertension is present; the pulmonary
apex. is cha racteristic of takotsubo cardiomyopathy. a nd an vasculature is protected by the presence of pulmonary
en domyo ca rd ia l biopsy is not indicated as a n i nitial d iag­ valve stenosis.
nostic test. Myocarditis has a variable presentation , but focal A ventricular septa! defect can cause pulmonary
ST-segment changes and apical hypokinesis are not typical. hypertension if it remains open beyond age 2 years. Clin­
More than 95% of patients with stress cardiomyopathy ical features include cyanosis and clubbing that affect the
recover ventricular function with conservative supportive hands and feet equally, a parasternal lift, and increased S 2 .
care (�-blockers and ACE inhibitors) . This patient does not Differential clubbing and cyanosis would not occur in a
have evidence of hemodynamic compromise. and intra-aor­ patient with pulmonary hypertension related to a ventric­
tic balloon pump implantation is not indicated. ular septa! defect.
In t h is patient, the lack of coronary artery obstructive
K EV P O I N T
disease i n the presence of ST- segment elevation and elevated
cardiac biomarkers eliminates STEM! or NSTEM I as poss i ­ • A patent ductus arteriosus with Eisenmenger syn­
ble etiologies for this presentation . Therefore. t h rombolytic drome is characterized by differential cyanosis and
therapy is not indicated. clubbing affecting the lower body.

KEV POINT
Bibliography
• Stress cardiomyopathy presents similarly to myocar­ Srinivas SK. Manjunath CN. Differential clubbing and cyanosis: classic signs
dial infarction, with ST-segment elevation and, often, of patent ductus arteriosus with Eisenmenger syndrome. Mayo Clin Proc.
2013 Sep;88(9):el05-6. [PMID: 24001503]
elevated cardiac biomarkers; however, coronary angi­
ography demonstrates an absence of significant
obstructive coronary artery disease.
Item 23 Answer: A
Educational Objective: Treat a patient with acute
Bibliography
decompensated heart failure and evidence of low cardiac
Prasad A, Lerman A. Riha! CS. Apical ballooning syndrome (Tako-Tsubo or
stress cardiomyopathy): a mimic of acute myocardial infarction. Am output.
Heart J. 2008 Mar:l55(3):408-17. [PMID: 18294473]
This patient should be started on dobulamine for proba­
ble cardiogenic shock. Cardiogenic shock is present when

Item 2 2 Answer: B t here is systemic hypotension and evidence for end-organ


hypoperfusion, primarily due to inadequate cardiac output.
Educational Objective: Diagnose Eisenmenger syn­
Cardiogenic shock usually requires treatment with intrave­
drome due to a patent ductus arteriosus.
nous vasoactive medications and. in severe cases, device­
The patient has a patent ductus arteriosus (PDA) with based hemodynamic support . Manifestations of' encl-orga n
secondary pulmonary hypertension (Eisenmenger syn - hypoperfusion may include acute kid ney fai l ure, elevated
drome) . Clinical features of an Eisenmenger PDA include serum aminotransferase levels or hyperbi l i rubinemia, cool
clubbing and oxygen desaturation affecting the lower body. extremities, and decreased mental status. In this patient.
This differential cyanosis and clubbing are caused by desat­ initiating inotrop ic t herapy is reasonable. Both clobutamine
urated blood reaching the lower part of the body prefer­ and milri none are used to increase cardiac output; however.
entially. in the setting of kidney dysfunction, dobutamine would be
Pulmonary hypertension occurs infrequently in the appropriate choice because m i l rinone is metabolized by
patients with unrepaired atrial septa! defects owing to the the kidneys. Also. m i l rinone is a vasodilator. which could
relatively small shunt size. Clinical features of unrepaired exacerbate his hypotension .
atrial septa! defect with pulmonary hypertension include Mechan ical t h e rapy f'or ca rdiogenic shock s h o u l d be
cyanosis and clubbing that affects the hands and feet equally. considered in p a t i e n t s w i t h e n d - o rgan dysfu n c t i o n that
The clinical findings are otherwise similar to findings in d o e s not ra pidly s how s i g n s o f i m p rove m e n t (wi t h i n t h e
patients with idiopathic pulmonary arterial hypertension, fi r s t 1 2- 24 hours) w i t h i n t rave n o u s va soact ive m e d i c a ­
with a parasternal lift and increased s2. t i o ns a n d correc tion of volume overload . O p t i o ns for
Unrepaired tetralogy of Fallot consists of right ven­ m e c h a n i c a l t he rapy i nclude pl acement of an i n t ra - aortic
tricular outflow tract obstruction and a large ventricular b a l l o o n p u m p and p e rcutaneous or surgica l ly i m p l a n ted
septa! defect with secondary features of right ventric­ ven t ricu l a r ass ist devices (VADs) . An i n t ra - aortic b a l ­
ular hypertrophy and aortic override. Clinical features loon p u m p i s t i m ed to i n fl a t e d u ring d i astole, augment ­
include cyanosis and clubbing that affect the hands and ing coro n a ry a n d systemic perfu s i o n . a n d d e fl a te during

1 67
Answers a n d Critiq u e s

rate or pat ients w i t h acute d issection who survive i n i t i a l


Cl
systole. red u c i ng left ve n t r i c u l a r a fterload . I L i s p rema­
t u re to consider mechanical t h e ra py fo r t h i s p a t i e n t . hospi talization i s reported t o b e 3 0 % t o 6 0% .
CONT. Endovascular t reatment of dissect ion is used as an
R ight heart catheterization can b e helpfu l t o guide ther­
apy if' volume status or cardiac output is u ncertain. However. a l termit ive to open su rgery primarily in complicated type
it has not been shown to improve outcomes in patients hos­ B dissect ions. A l t hough endovascular procedures for type A
pital ized with heart fai l u re. T h is pat ient has cli nical evidence d issections have shown some promise, they are not rou t i nely
of' volume overload. includ i ng j ugular venous d isten t ion . used, particularly in pat ients w i t h valve dysfunction requir­
p u lmonary crackles, edema to t he mid t h ighs. pulmonary ing surgical repai r.
edema on chest radiography, and an S.i . Addit iona l ly, he A lt hough hepa ri n is commonly used i n the initial treat­
has evidence of low cardiac output (narrow pulse pressure, ment of' acu te coronary syndromes and p u lmonary embo­
hypotension. acute kid ney injury, mot tled and cool extrem ­ l ism. heparin is not indicated in the setting of an acute type
ities) . Placement of a right heart cat heter is not necessary A aortic dissection . The use of hepa rin in t h is setting can
prior to in itiat i ng i notropic t herapy. be complicated by major bleedi ng and cardiac tamponade.
Moreover, heparin can i ncrease t he risk of l i fe- t h reaten i ng
K EY P O I N T
major bleeding when used i n patients with very elevated
• Cardiogenic shock usually requires treatment with blood pressu res (commonly seen d u ri ng a dissection) .
intravenous vasoactive medications and, in severe
KEY POINT
cases, device-based hemodynamic support.
• A dissection involving the ascending aorta (Stanford
Bibliography type A) is a surgical emergency.
Nativi-Nicolau J, Selzman CH, Fang JC. Stehlik J. Pharmacologic therapies
for acute cardiogenic shock. Curr Opin Cardiol. 2014 May;29(3):250-7.
Bibliography
[PMID: 24686400]
Braverman AC. Aortic dissection: prompt diagnosis and emergency treat­
ment are critical. Cleve Clin J Med. 2011 Oct:78 (l0):685-96. [PMID:
21968475]

Cl Educational Objective:
Item 24 Answer: B
Manage type A aortic dissection.

Cl
Item 2 5 Answer: D
The most appropriate management for this patient with an
Educational Objective: Manage antibiotic prophylaxis
acute type A aortic dissection is emergency surgical inter­
in a patient with a mechanical aortic valve prosthesis.
vention. The abrupt onset of severe chest and back pain is
typical of an acute aortic syndrome. A diastolic murmur con­ For t h is patient with a mechanical valve preparing for hernia
sistent w i l h aortic valvu lar i nsufficiency increases the clin ical repai r su rgery. antibiotic prophylaxis to prevent bacterial
suspicion for a prox imal (type A) aortic dissection t hat has endocarditis is not i ndicated. Prophylaxis to prevent bacte­
d isrupted normal valve leaflet coapta l ion. Acute aortic dissec­ rial endocard i t is is appropriate before certain dental proce­
t ion is the most common l i fe-threatening disorder aflecting d u res for patients with specific i ndica tions placing them at
the aorta. In the Stan ford classi ficat ion. type A dissections h igh ris k for an adverse outcome from i n fective endocarditis
involve t he ascending aorta. and type B d issections are those (class I la recommendation) . These i nd ications include pre­
t hat do not involve the ascending aorta. Type A dissections vious endocard i t is. a h i story of cardiac transplantation . a
require emergency surgical repair, whereas medical therapy, prosthetic valve, and specific forms of complex congenital
consisting of a �-blocker to decrease t he heart rate to below heart d isease. However. prophylaxis is not recom mended for
6 0/min plus add i t ional medications as needed to control nondental proced u res. i ncluding t ra nsesophageal echocar­
hypertension, is usually t he initial st rategy for acute type B cliography and genitourinaty or gastrointestinal procedures
dissections. Therefore. pursuing medical managemen t alone (such as upper endoscopy. colonoscopy, or hernia repai r) .
wou ld not be appropriate i n this pat ient. ·n1e immediate mor­ i n the absence o f active i n fection (class I l l recommenda­
tality rate in aortic dissection is as high as l 'Y., per hour over tion) . Dental procedures for which a n t i biotic prophylaxis
the First several hours, making early diagnosis and treatment is reasonable i nclude t hose t hat i nvolve manipulation o f
crit ical for survival. gi ngival t i ssue or t he periapical region of tee t h or perfora­
A l t hough most p at i e n t s w i t h d i ssec t i o n have underly i n g t i o n of' t h e oral m ucosa. Prophylaxis is n o t reco m m en ded
hypertension. only a t i ny fraction of all persons with hyper­ fo r rou t i ne dental procedu res, inclu d i ng radiographs a nd
tension ever have a d issect ion . Syncope occurs in approx i ­ ort hodon t ics.
mately IO'X. of patients w i t h an acute aortic dissection a n d is When antibiotic prophylaxis is indicated. it shou ld be
more commonly associated with prox imal dissect ion . Pulse given as a single dose 30 to 60 minutes before t h e dental
deficits occur in less than 20% of type A d issections. Abnor­ procedure. I f' t he prophylactic medica tion is inadvertently
mal aortic contour or widening of the aortic s i l houette may not admin istered, it may be given u p to 2 hours a fter the
be an important clue to the d iagnosis of' aortic d issect ion . procedu re. Options include amoxici l l i n , 2 g oral ly. o r ampi­
However. a normal chest radiograph is seen in nearly I S'Y.. c i l l i n . 2 g i ntravenously. For patients allergic to pen ici l l i n or
of patients with acute aortic dissection . The 1 0-year survival amoxicill i n, alternatives include clindamycin. 600 mg orally;

1 68
Answers and Criti q u es

CJ azit h romycin. 500 mg orally; or cetazo l i n /ceftriaxone, 1 g not be used for the diagnosis of PAD. Moreover, this patient
i n t ram uscu larly or i n t ravenously.
CONT. has chronic kidney injury, and use of contrast for either
KEY POINT study should be avoided if possible.

• Antibiotic prophylaxis to prevent bacterial endocardi­ KEY POINT


tis is not recommended for nondental procedures, • In a patient with suspected peripheral arterial disease,
including transesophageal echocardiography and gen­ an ankle-brachia! index above 1 .40 is uninterpretable;
itourinary or gastrointestinal procedures, in the measurement of the great toe systolic pressure can
absence of active infection. establish the diagnosis.

Bibliography Bibliography
Bonow RO, Carabello BA, Chatterjee K, et al; American College of Cardiology/ Aboyans V, Criqui M H , Abraham P. et al: American Heart Association
American Heart Association Task Force on Practice Guidelines. 2008 Council on Peripheral Vascular Disease; Council on Epidemiology and
focused update incorporated into the ACC/AHA 2006 guidelines for the Prevention; Council on Clinical Cardiology; Council on Cardiovascular
management of patients with valvular heart disease: a report of the Nursing; Council on Cardiovascular Radiology and Intervention, and
American College of Cardiology/American Heart Association Task Force Council on Cardiovascular Surgery and Anesthesia. Measurement and
on Practice Guidelines (Writing Comminee to revise the 1998 guidelines interpretation of the ankle-brachia! index: a scientific statement from
for the management of patients with valvular heart disease). Endorsed the American Heart Association. Circulation. 2012 Dec 11;126(24):2890-
by the Society of Cardiovascular Anesthesiologists, Society for 909. Erratum in: Circulation. 2013 Jan 1;127(l):e264. [PM I D : 23159553]
Cardiovascular Angiography and Interventions. and Society of Thoracic
Surgeons. J Am Coll Cardiol. 2008 Sep 23;52(13):el-142. [PMID: 18848134]

Cl
Item 2 7 Answer: D
Educational Objective: Manage atrial fibrillation in a
Item 2 6 Answer: D
patient following percutaneous coronary intervention for
Educational Objective: Diagnose peripheral arterial treatment of refractory angina.
disease in a patient with uninterpretable ankle-brachia}
This pal ient should be treated w i t h aspirin. clopiclogrel, and
index testing.
warfarin ("lriple therapy") . He has new-onset atrial fibrillation
The most appropriate diagnostic test to perform next in this in the set t i ng of' recent bare metal slent placement for medi­
patient is a toe-brachia! index. The ankle-brachia! index (AB!) cally rcf'raclory a ngi na. Palienls with a bare metal stent should
is obtained by measuring the systolic pressures in the dorsalis be lrealed w i t h dual anliplatelet t herapy for at least l month
pedis and posterior tibialis arteries on both sides. The AB! for to al low endothelial iz<Jtion of t he stenl; with clrug-eluling
each leg is the highest ankle pressure for that side divided slents. t he requ i rement for dual antiplalelet t herapy is longer
by the highest brachia! pressure (regardless of side) . An AB! and depends upon the lype of' stcnt implanted. ·1 11is patient is
of 0. 90 or lower establishes a diagnosis of peripheral arterial also a t h igh risk or t h romboembolic d isease associated w i l h
disease (PAD) . In this patient, the right AB! is greater than alrial fibril lalion . He has a CH1\DS1-VASc score of 5 (2 points
1.40. An AB! above 1.40 suggests noncompressible vessels, for age >75 years . I poin t each for diabetes melli t us, hyper­
which may reflect medial calcification but is not diagnostic tension. and vascular disease) . ll1erefore, oral <Jnticoagu lanl
of flow-limiting atherosclerotic disease. An AB! greater than t herapy is also i ndicated. Although l riple t herapy w i t h two
1 .40 is associated with worse cardiovascular outcomes than antiplalelel agents and systemic a n t i coagulation is associated
a normal AB!. In such patients, an appropriate next step is to wilh a significant increase in bleeding risk, t h is regimen is
either measure great toe pressure or calculate a toe-brachia! appropriate t reatment i n this pat ient !b r at leasl 1 mo nt h u n t i l
index (systolic great toe pressure divided by systolic brachia! stent endothelialization can b e assured. at which t i m e h e can
pressure) . Vessels within the great toe rarely become noncom­ be transitioned to only aspirin ancl an oral anlicoagulant to
pressible, and a great toe systolic pressure below 40 mm Hg or decrease bleeding risk but provide adequate t hromboembolic
a toe-brachia! index of less than 0.70 is consistent with PAD. prophylaxis. I f' warfarin is used as an anticoagulant during
An AB! obtained immediately following symptom­ triple t herapy. careful maintenance of' t he I N R w i t h i n lhe rec­
limited exercise is useful when a high clinical suspicion ommended range of 2 .0 lo 2.5 in patients without mechanical
for PAD remains despite a normal (1.00-1 .40) or borderline valves may reduce the overall bleeding risk.
resting AB!. A decrease of the AB! by 20% compared with Aspirin and clopiclogrel are i n ferior to oral an ticoagula­
the resting AB! is consistent with significant PAD. This lion for the prevenlion of stroke in patients w i t h an i n d ica­
patient's resting AB! is above the normal range; therefore, tion for a n t icoagu lat ion for th romboembolism prophylaxis
exercise AB! would not help to establish the diagnosis. i n a lrial fibri l lat ion .
In patients with an established diagnosis of PAD and Treatment with aspirin and dabigatran is not opti m a l
indications for revascularization, further vascular anatomic f C i r two reasons. Firsl. i n the Random ized Eva luation of
data can be obtained noninvasively using gadolinium­ Long Term A n t i coagu lant ·1 herapy ( R E-LY) trial, t here was
enhanced magnetic resonance angiography or contrast­ a n u meric excess of' myocard ial i n farclions observed w i t h
enhanced multi-detector CT angiography. However, these dabigatran . More i m portanl ly, no dala are ava i lable regard­
tests should be reserved for planning intervention in patients ing l he e f ficacy of aspi rin and dabigal ran for l h e p revention
who have not benefited from medical therapy; they should of' stenl t h rombosis fol low i ng an acute coronary syndrome.

1 69
Answers a n d Critiques

Cl all patients with a coronary stent, with the recommended


Treatment with dual antiplatelet therapy is indicated in K EY P O I N T
• Surgical aortic valve replacement is the treatment of
CONT.
duration based on the underlying condition and type or stent choice for most patients with symptomatic severe aor­
placed. Therefore. treatment with aspirin and warfarin does
tic stenosis.
not optimally prevent acute stent occlusion in a patient with
stent placement.
Bibliography
KEY POINT Joint Task Force on the Management of Valvular Heart Disease of the
European Society of Cardiology (ESC); European Association for Cardio­
• Patients with atrial fibrillation and recent stent Thoracic Surgery (EACTS), Vahanian A, Alfieri 0, Andreotti F, et al.
Guidelines on the management of valvular heart disease (version 2012).
placement should be treated with appropriate sys­
Eur Heart J. 2012 Oct;33(19):2451-96. [PMID: 22922415]
temic anticoagulation and antiplatelet therapy as
determined by risk scoring and the type of stent
placed. Item 2 9 Answer: A
Educational Objective: Diagnose atrial septa! defect.
Bibliography
Lip GY, Huber K, Andreotti F. et al: Consensus Document of European This patient has features of an ostium secundum atrial septa!
Society of Cardiology Working Group on Thrombosis. Antithrombotic
management of atrial fibrillation patients presenting with acute coro­
defect. Adults with atrial septa! defects often present with
nary syndrome and/or undergoing coronary stenting: executive sum­ atrial arrhythmias. The characteristic physical examination
mary-a Consensus Document of the European Society of Cardiology
findings in atrial septal defect are fixed splitting of the S2 and
Working Group on Thrombosis. endorsed by the European Heart
Rhythm Association (EHRA) and the European Association of a right ventricular heave. A pulmonary midsystolic flow mur­
Percutaneous Cardiovascular Interventions (EAPCI). Eur Heart J. 2010 mur and a tricuspid diastolic flow rumble caused by increased
Jun:31 (11):1311-8. [PMID: 20447945]
flow through the right-sided valves from a large left-to-right
shunt may be heard. In ostium secundum atrial septal defect,
Item 28 Answer: c the electrocardiogram (ECG) demonstrates right axis devi­
ation and incomplete right bundle branch block. Ostium
Educational Objective: Manage symptomatic severe
primum atrial septa! defects are nearly always associated with
aortic stenosis.
anomalies of the atrioventricular valves, particularly a cleft in
The patient should be referred for surgical aortic valve the anterior mitral valve leaflet or defects of the ventricular
replacement. She has symptomatic severe aortic stenosis, a septum. The ECG in ostium primum atrial septa! defect char­
class I indication for valve replacement. Surgical aortic valve acteristically demonstrates first-degree atrioventricular block,
replacement is the treatment of choice for most patients left axis deviation, and right bundle branch block.
with symptomatic severe aortic stenosis and is associated Bicuspid aortic valve with aortic stenosis causes a sys­
with low mortality rates (1 %-3%) in patients younger than tolic murmur at the second right intercostal space. The cen­
70 years. tral venous pressure is normal in aortic stenosis, and a right
Balloon valvuloplasty of the aortic valve has an ventricular impulse would not be expected. Fixed splitting of
extremely limited role in treating aortic stenosis owing to the S 2 is not heard in patients with bicuspid aortic valve with
the high risk of complications associated with the procedure aortic stenosis. The ECG typically demonstrates a normal
(10%-20%), a short-lived clinical benefit, and poor long-term axis and features of left ventricular hypertrophy.
outcomes. It would not be an appropriate treatment option Patients with pulmonary valve stenosis may demonstrate
in this patient. jugular venous pressure elevation with prominence of the
Medical therapy for aortic stenosis is limited. Multiple a wave and a parastemal impulse from increased right ven­
agents have been evaluated for treatment of aortic steno­ tricular pressure. The ejection murmur of pulmonary valve
sis, including ACE inhibitors, angiotensin receptor block­ stenosis is heard at the second left intercostal space, and the
ers, digoxin, diuretics, and statins. None have been shown timing of the peak of the murmur is related to stenosis sever­
to alter the progression of disease and valve replacement ity. An ejection click is often heard; the proximity of the click
remains the indicated therapy. These agents are appropriate to the S2 varies depending on the severity of stenosis. Splitting
if there are other indications for their use, none of which are of the S2 results from prolonged ejection delay in the pulmo­
present in this patient. nary valve component and may become fixed in severe pul­
Transcatheter aortic valve replacement (TAVR) is an monary valve stenosis. The ECG typically demonstrates right
option for patients with an indication for aortic valve axis deviation and features of right ventricular hypertrophy.
replacement but who are not operative candidates or are Mitra! valve prolapse with mitral regurgitation may
at high risk for death or major morbidity with open aor­ cause symptoms of palpitations. The murmur of mitral
tic valve replacement. However, TAVR is not currently regurgitation is generally heard best at the apex. The central
approved in patients with a bicuspid aortic valve, and venous pressure is generally normal, and a right ventricular
given this patient's age and significant lack of comorbidi­ impulse would not be expected. Fixed splitting of the S2 is
ties, she would appear to be a good candidate for surgical not heard in patients with mitral valve prolapse. The ECG is
replacement. typically normal.

1 70
Answers and Criti ques

KEY POINT . ·
Management of Cardiovascular Diseases during Pregnancy of the
European Society of Cardiology (ESC). Eur Heart J. 2011 Dec-32(24):3147-
'
• Fixed splitting of the S2 throughout the cardiac cycle 97. [PM l D : 21873418)
and a right ventricular heave are characteristic clinical
features of atrial septal defect.
Item 3 1 Answer: A
Bibliography
Educational Objective: Manage abdominal aortic aneu­
Baumgartner H. Bonhoeffer P, De Groot N M , et al; Task Force on the
rysm with referral for repair.
Management of Grown-up Congenital Heart Disease of the European
Society of Cardiology (ESC); Association for European Paediatric
Cardiology (AEPC); ESC Committee for Practice Guidelines (CPG) ESC The most appropriate management is to refer this patient
Guidelines for the management of grown-up congenital heart di�ease for abdominal aortic aneurysm (AAA) repair. AAA is a
(new version 2010). Eur Heart J. 2010 Dec·31(23):291S-S
' 7 . [PMlD·. common and potentially life-threatening condition, and
20801927)
management of detected aneurysms is based on size or rate
of expansion. Elective repair to prevent rupture in asymp­
Item 3 0 tomatic patients is optimal management in those meet­
Answer: C
ing criteria for intervention. Once an aneurysm reaches
Educational Objective: Manage a patient with mitral
5 . 5 cm in men and 5 . 0 cm in women, repair is generally
valve stenosis considering pregnancy.
warranted. Repair may be performed by an open approach
This patient has clinical and echocardiographic features of or an endovascular approach, if the anatomy of the aneu­
severe mitral valve stenosis as indicated by mitral valve mean rysm is amenable; the mode of therapy should be decided
gradient, valve area, and pulmonary artery systolic pressure by the surgeon, the internist, and the patient after a com­
greater than SO mm Hg. Planned pregnancy is a class I indi­ prehensive discussion of risks and long-term benefits. Ran­
cation for intervention in patients with severe rnitral steno­ domized trials show that endovascular aneurysm repair
sis despite the absence of baseline symptoms. Most young (EVAR) is associated with lower perioperative morbidity
patients will be candidates for mitral balloon valvuloplasty. and mortality compared with open AAA repair, but EVAR
In severe rnitral valve stenosis, negative chronotropic does not completely eliminate the future risk of AAA rup­
drugs such as �-blockers allow increased diastolic filling ture. Open repair is associated with higher perioperative
time of the left ventricle and may improve symptoms. If morbidity and mortality than EVAR, but it provides a more
atrial fibrillation develops (even if paroxysmal), chronic definitive repair.
anticoagulation therapy with warfarin is indicated to reduce The optimal surveillance schedule for patients once
the risk of thromboembolism, which is much higher than an AAA has been identified has not been clearly defined.
in nonvalvular atrial fibrillation. ACE inhibitors provide no Annual surveillance is recommended, but larger aneu­
particular benefit to patients with mitral stenosis and are rysms expand faster than small ones and may require
contraindicated in pregnant patients. Dabigatran is approved more frequent surveillance. Aneurysm diameter is the
for prevention of systemic embolism in adults with nonval­ most important factor predisposing to rupture, with risk
vular atrial fibrillation; however, its effectiveness in pre­ increasing markedly at aneurysm diameters greater than
S . S cm. For asymptomatic patients, the risk of AAA rup­
venting embolism in patients with valvular heart disease is
unknown, and it is not recommended in this setting. ture generally exceeds the risk associated with elective
Cardiac magnetic resonance imaging will not add AAA repair when aneurysm diameter exceeds 5 . 0 cm in a
incremental information to determine therapeutic strategy woman and 5 . 5 cm in a man. This patient's AAA is 5 . 7 cm
for this patient with rheumatic mitral stenosis considering in diameter; therefore, she should be referred for repair,
pregnancy. rather than continuing surveillance.
Pregnancy is associated with a marked increase in blood Although controlling risk factors for cardiovascular
volume and cardiac output. Patients with severe mitral ste­ disease is essential in patients with AAA , there is little com­
nosis and moderate pulmonary hypertension often develop pelling evidence for treating hypertension in these patients
symptoms during pregnancy and should receive interven­ with a specific agent, including �-blockers, to prevent aneu­
tion prior to pregnancy. rysm expansion. As this patient's blood pressure is well con­
trolled, no change in antihypertensive therapy is indicated.
KEY POINT
K E Y P O I NT
• Planned pregnancY. iS a class I indication for mitral
• An abdominal aortic aneurysm larger than 5.5 cm in
valve intervention in patients with severe mitral valve
men and 5 . 0 cm in women is an indication for referral
stenosis despite the absence of baseline symptoms.
for repair.

Bibliography
European Society of Gynecology (ESG); Association for European Paediatric Bibliography
Cardiology (AEPC); German Society for Gender Medicine (DGesGM). Buck DB. van Herwaarden JA, Schermerhorn ML, Moll FL. Endovascular
Regitz-Zagrosek V, Blomstrom Lundqvist C, Borghi C, et al; ESC treatment of abdominal aortic aneurysms. Nat Rev Cardiol. 2014
Committee for Practice Guidelines. ESC Guidelines on the management Feb;ll(2) :112- 23. Erratum in: Nat Rev Cardiol. 2014 Feb;l l (2 ) : i. [PMl D :
of cardiovascular diseases during pregnancy: the Task Force on the 24343568]

1 71
Answers a n d Critiq u es

Answer: A should be referred to a cardiac electrophysiologist for catheter


Item 3 2
ablation of his PVCs.
Educational Objective: Treat a patient with potentially
PVCs are common and are usually benign. However,
transient constrictive pericarditis.
symptomatic or frequent PVCs (>10,000 PVCs/24 hours or
This patient has evidence of constrictive pericarditis and >10% of all beats) require treatment. Up to one third of
should be treated with an anti-inflammatory medication, patients with frequent PVCs develop PVC-induced cardio­
such as a high-dose NSAID or prednisone. Supportive myopathy and progressive left ventricular dysfunction. First­
findings are symptoms and signs of right heart failure and line therapy for symptomatic or frequent PVCs is 13-blocker
congestion, with hemodynamic evidence of constriction on or calcium channel blocker therapy. Patients with medically
echocardiography. In some patients with constrictive peri­ refractory frequent PVCs or who develop left ventricular
carditis, the constriction can be transient and either sponta­ dysfunction should undergo catheter ablation of the PVCs.
neously resolve or respond to medical therapy. This subtype of Catheter ablation leads to resolution of PVC-induced cardio­
constrictive pericarditis more frequently has idiopathic, viral, myopathy in most patients.
or postsurgical causes. Although a minority of patients will Amiodarone could be used for PVC suppression; how­
have this transient constrictive pericarditis, a trial of medical ever, it has many long-term risks, including thyroid, liver,
therapy with an anti-inflammatory medication is reasonable. pulmonary, and neurologic toxicity. Given this patient's rel­
If medical therapy is successful, then surgical pericardiectomy atively young age, amiodarone would not be the most appro­
can be avoided. Anti-inflammatory medication regimens priate therapy, particularly if long-term control is needed.
for potentially transient constrictive pericarditis are similar Cardiac resynchronization therapy involves the simul­
to those for acute pericarditis, with relatively high doses of taneous pacing of both ventricles in patients with advanced
NSA!Ds used (for example, ibuprofen, 800 mg three times heart failure and evidence of intraventricular conduction
daily; indomethacin, SO mg three times daily; aspirin, 650 mg delay (QRS interval 2':150 ms) to reduce dyssynchrony to
three times daily), with a slow taper over 2 to 3 weeks. improve pump performance. This patient does not have
Cardiac catheterization for hemodynamic assessment severe heart failure or evidence of dyssynchrony. As his
of possible constriction is only indicated when diagnos­ symptoms are related more to his ventricular ectopy, cardiac
tic information cannot be obtained with echocardiography, resynchronization therapy would not be appropriate.
which is not the case in this patient. An implantable cardioverter-defibrillator (!CD) is
Pericardiectomy is inappropriate before a 2- to 3-month indicated in patients at risk for ventricular tachycardia or
trial of anti-inflammatory medication in this patient. ventricular fibrillation to prevent sudden cardiac death.
Although pericardiectomy is the definitive treatment for Although this patient has very frequent PVCs, including
relief of heart failure in patients with constrictive pericar­ bigeminy, he does not have sustained tachyarrhythrnias of
ditis, it is a complex, invasive procedure that should not be potential hemodynamic significance. Therefore, ICD place­
used in patients with transient constriction. ment would not be appropriate in this patient.
Transesophageal echocardiography is only indicated
KEY POINT
when data from other noninvasive imaging studies (such as
transthoracic echocardiography) are inconclusive. • Patients with frequent premature ventricular contrac­
tions and subsequent cardiomyopathy should be
KEY POINT
treated with catheter ablation.
• I n some patients with constrictive pericarditis, the
constriction may be transient and either spontane­ Bibliography
ously resolve or respond to medical therapy, which Yokokawa M, Good E, Crawford T, et al. Recovery from left ventricular dys­
obviates the need for surgical pericardiectomy. function after ablation of frequent premature ventricular complexes.
Heart Rhythm. 2013 Feb;l0(2):172-5. [PM I D : 23099051]

Bibliography
Haley JH, Tajik AJ, Danielson GK, Schaff HV, Mulvagh SL, Oh JK. Transient
constrictive pericarditis: causes and natural history. J Am Coll Cardiol. Item 34 Answer: A
2004 Jan 21;43(2):271-5. [PMID: 147364481
Educational Objective: Manage discharge of a patient
with heart failure to prevent read m iss ion .

Item 33 Answer: C ·n1is patient should be discharged home. with a follow-up


appo i n t ment scheduled w i t h i n 7 clays. S he has had one heart
Educational Objective: Manage symptomatic prema­
fai lure hospi ta lization i n t he past 3 years and her nonad her­
ture ventricular contractions.
ence with her diuretic medica t ion was the most l i kely cause
This patient has frequent, symptomatic premature ventricular of t he adm ission . 'v\lith a ny heart fai lu re hospitalization. it is
contractions (PVCs) with decline in his left ventricular func­ important to reassess several factors before discharge. First.
tion despite treatment with 13-blocker therapy. As there is no patiems must be adequately cli u resecl prior to discharge. It i s
clear underlying myocardial process, it is Likely that his PVCs important t o know t hat measuring a seru m B-type natriuretic
may represent a reversible cause of his cardiomyopathy. He peptide level w i l l not help with that assessment. Patients

1 72
Answers a n d Critiq u es

Cl shou l d be exam i ned for flat neck veins, resolution of periph­


the collateral blood flow. The chest radiograph (shown)
eral or abdom i nal edema (if possible) . and resolution of t he
CONT. . demonstrates "rib notching" affecting several of the pos­
.
signs and symptoms of acute heart fai l u re (51. cxertional clys- terior ribs; rib notching results from exaggerated collateral
pnea and fatigue, orthopnea. pa roxysmal nocturnal dyspnea) . blood flow diverting blood around the area of obstruction.
Second, patients should be on appropriate medical t herapy Also present on the chest radiograph is the "figure 3 sign"
for their stage of heart fa i l u re. For this pat ient. appropriate caused by dilatation of the aorta above and below the area
medications include an ACE i n h ibitor or angiotcnsin receptor of coarctation.
blocker, P- blocker, aldosterone antagonist. and an adequate
dosage of d i u retic to prevent readm ission. Third . i t has been
demonstrated that a patient being seen within I week after
discharge is associated with a red uction of fu ture heart failure
hosp ital izations. Th is patient is adequately d i u resed and is on
appropriate med ications. Reducing her risk of readmission
requi res a follow- up visit w i t h i n 7 days o f d ischarge and
appropriate pat ient education .
An echocarcliogram performed I month ago demon­
st rated that the pat ient's left vent ricular fu nction is sta­
ble. There is no suggestion o f ischemia or cha nge i n valvu­
lar function as a precipitant of t h i s hospita l ization . If t h i s
patient h a d n o t h a d an echocardiogram i n at least 6 mont hs,
i t would be reasonable to repeat the echocard iogram d ur i ng
t he hospi tal izat ion : otherwise u n less t here is a suspicion of
a change. there i s no reason to do so.
Pa t i en t s a re c a n d i d ates fo r a biven t r i c u l a r pace­
maker i f t h ey have a l l o f t he fo l l owi ng i n d i cat i o ns : o n
gu i del i ne -d i rected medical t h erapy, a reduced ejec t i o n
fract ion (::;35 % ) . a wide QRS i n terval ( � 1 5 0 ms) o r a left
b u n d l e branch b l ock, a n d New Yo rk Heart Assoc i a t i o n
fu n c t i o n a l c l ass I I I or J V sy mptom s . T h i s pa t i e n t h a s
a na rrow Q R S i n t erval a n d t herefo re wo u l d not b e a
c a n d i d a t e fo r upgra d i ng to a bive n t ricu l a r i m p l a n table Essential hypertension is the most common cause of
card iove r ter-def"i b r i I l a tor. hypertension in adults. A family history of hypertension is
common. The physical examination in a patient with severe
KEY POINT essential hypertension often includes an 54, but a pulse and
• Patients hospitalized for heart failure who are sched­ blood pressure differential between the upper and lower
uled for a follow-up appointment within 1 week after extremities is not expected.
discharge have a reduced risk of future heart failure Patients with hypertrophic cardiomyopathy do not usu­
hospitalization. ally present with hypertension. The systolic murmurs in
hypertrophic cardiomyopathy characteristically include an

Bibliography ejection quality murmur at the left sternal border related to


Fleming LM, Kociol RD. Interventions for heart failure readmissions: suc­ outflow obstruction and a late systolic murmur at the apex
cesses and failures. Curr Heart Fail Rep. 2014 Jun;J1(2): 178-87. [PMID: related to mitral valve regurgitation.
24578234]
Renovascular hypertension is a common cause of hyper­
tension occurring primarily in patients with diffuse athero­
sclerosis. An epigastric bruit may be audible. The rest of the
Item 3 5 Answer: A
physical examination in a patient with renovascular hyperten­
Educational Objective: Diagnose aortic coarctation in sion is usually normal. A pulse and blood pressure differential
an adult. between an upper and a lower extremity is not expected.

This patient presents with newly diagnosed hyperten­ KEY POINT


sion and clinical features of aortic coarctation, which
• Aortic coarctation is characterized clinically by upper
include upper extremity hypertension and a radial artery­
extremity hypertension and a radial artery-to-femoral
to-femoral artery pulse delay suggesting a mechanical
artery pulse delay.
obstruction between the radial and femoral arteries; lower
extremity blood pressure determinations may be low or
Bibliography
difficult to obtain. A systolic murmur over the left poste­
Tanous D, Benson LN, Horlick EM. Coarctation of the aorta: evaluation and
rior chest is common in patients with severe aortic coarc­ management. Curr Opin Cardiol. 2009 Nov;24(6):509-15. [PMID:
tation; these murmurs can arise from the obstruction or 196679801

1 73
Answers and Critiques

Answer: A of a nonstenotic bicuspid aortic valve may radiate to the apex.


Item 36
Fetal echocardiography is indicated if there is maternal cardiac
Educational Objective: Treat hypertension during preg­
structural disease because of the increased risk of fetal cardiac
nancy.
abnormalities, which is estimated to be in the range of3% to 7%.
The most appropriate treatment for this patient is labetalol. The altered hemodynamics of pregnancy may bring
When hypertension is noted before the 20th week of gesta­ out murmurs not previously heard or may bring the preg­
tion, it is most consistent with a new diagnosis of chronic nant patient to medical attention for the first time owing
hypertension. There is no evidence that tight control of hyper­ to the increase in systemic blood volume and cardiac out­
tension during pregnancy will prevent preeclampsia; instead, put. Systolic murmurs are common during pregnancy. Most
antihypertensive therapy is warranted only to limit mater­ often these are ejection murmurs caused by increased flow
nal end-organ damage in those with severe hypertension. through the right and left ventricular outflow tracts. The
Hypertension of the magnitude demonstrated by this patient murmurs tend to be grade 1/6 or 2/6 midsystolic murmurs
(systolic blood pressure >150 mm Hg) is a class I indication for that do not radiate. Diastolic murmurs are not common.
pharmacologic treatment during pregnancy. The systolic murmur associated with hypertrophic
If treatment is necessary, it is important to note that all obstructive cardiomyopathy generally increases with maneu­
antihypertensive agents cross the placenta. Methyldopa and vers that decrease preload, such as the Valsalva maneuver or
labetalol appear to be the safest choices, whereas ACE inhib­ change in position from squatting to standing.
itors, angiotensin receptor blockers (ARBs), and likely renin A mammary soulle is a continuous murmur with a
inhibitors are not safe. soft, humming quality. It is typically heard over the breast
ACE inhibitors (such as lisinopril ) , ARBs (such as during late pregnancy and lactation and is thought to result
losartan), and direct renin inhibitors (aliskiren) are con­ from increased blood flow to the breast.
traindicated during pregnancy owing to fetal toxicity. Fetal While the murmur of mitral valve prolapse is often late
exposure to these agents during the first trimester can systolic and can be associated with a click, this click would
cause central nervous system and cardiovascular malfor­ not be described as an ejection click. The click associated
mations, and exposure during the second trimester can with mitral valve prolapse is mid- to late systolic and is
cause urogenital and renal developmental malformations. related to tensing of the chordae tendineae or valve leaflets.
Diet and weight loss should be used in conjunction with The click is responsive to changes in ventricular volume
pharmacologic therapy in this patient, but nonpharmaco­ induced by posture or pharmacologic agents.
logic strategies alone are not sufficient in a patient with this Functional murmurs, or physiologic murmurs, can
magnitude of hypertension. occur in the absence of valvular pathology. An increase in
Not treating this patient's chronic hypertension may cardiac output, as occurs in pregnancy, can result in physio­
result in maternal end-organ damage and is not an accept­ logic ejection murmurs.
able option. Given the findings on examination, it would be appropri­
ate to obtain transthoracic echocardiography for this patient
KEY POINT
to identify the etiology of the murmur and the potential need
• Labetalol has been demonstrated to be safe and effective
for adjuvant screening to evaluate for associated aortopathy.
for the treatment of hypertension during pregnancy
and is the preferred �-blocker for use during pregnancy. KEY POINT
• The characteristic finding of a bicuspid aortic valve is
Bibliography an aortic ejection sound associated with either a sys­
European Society of Gynecology (ESG); Association for European Paediatric tolic or diastolic murmur.
Cardiology (AEPC): German Society for Gender Medicine (DGesGM) .
Regitz-Zagrosek V. Blomstrom Lundqvist C . Borghi C . e t a l ; ESC
Committee for Practice Guidelines. ESC Guidelines on the management Bibliography
of cardiovascular diseases during pregnancy: the Task Force on the
Franklin WJ, Gandhi M. Congenital heart disease in pregnancy. Cardiel Clin.
Management of Cardiovascular Diseases during Pregnancy of the
2012 Aug;30(3):383-94. [PM I D : 22813364]
European Society of Cardiology (ESC). Eur Heart J. 2011 Dec;32(24):3147-
97. [PMID: 21873418]

Item 38 Answer: C
Item 37 Answer: A Educational Objective: Manage medication use prior to
Educational Objective: Diagnose a murmur heard in a stress testing.
pregnant woman.
This patient's metoprolol should be withheld for 48 hours
The most likely cause of this woman' s murmur is a bicus­ before stress testing. In this intermediate-probability patient
pid aortic valve, the most common congenital heart abnor­ with a normal electrocardiogram, exercise stress testing is
mality. The characteristic finding of a bicuspid aortic valve is appropriate. Exercise stress is preferred to pharmacologic
an aortic ejection sound associated with either a systolic or stressors because it provides a gauge of functional capacity
diastolic murmur. While the murmur associated with aortic and a contextual understanding of symptoms, and it records
stenosis usually radiates to the carotid arteries, the murmur hemodynamic response to exercise. The sensitivity of the

1 74
Answers and Criti ques

study to detect obstructive coronary artery disease (CAD) is studies in this clinical setting and would allow for further
lowered, however, if patients are taking certain medications. stratification of this patient's risk for ischemic heart disease.
P-Blockers and nondihydropyridine calcium channel block­ Proceeding directly to cardiac catheterization may be
ers can blunt the maximal heart rate that can be achieved appropriate in patients at very high risk for ischemic heart
with exercise and may limit a patient's ability to reach 85% of disease in whom noninvasive testing would not be expected
the maximal predicted heart rate. However, dihydropyridine to significantly change the pretest probability of disease.
calcium channel blockers do not need to be withheld prior However, in this patient with an intermediate risk of disease,
to testing. Similarly, digoxin can limit the maximal heart rate adequate noninvasive testing would be helpful in evaluating
and should be withheld. Nitrates are effective antianginal for the presence of ischemic heart disease, and coronary
agents but may minimize the ischemic response on stress angiography would not be indicated as a next diagnostic test.
testing; therefore, they should also be withheld. If, however, Switching the patient's antihypertensive medication to
a patient has known CAD and the goal of testing is to deter­ an agent with antianginal properties would not be indicated
mine whether symptoms are related to ischemia or to assess without establishing the presence of ischemic heart disease,
adequacy of antianginal therapy, there is no need to stop any particularly with adequate control of her blood pressure on
of the medications. her current regimen.
There is no evidence that ACE inhibitors, such as lisin­ Because an inadequate exercise ECG exercise test is
opril, or angiotensin receptor blockers alter the sensitivity unable to assess for the presence of ischemic heart disease in
of exercise stress testing in the diagnosis of CAD, and these this patient at intermediate risk, clinical observation without
agents do not need to be discontinued before testing. For this further evaluation would not be appropriate.
patient on several medications for hypertension, an appro­
KEY POINT
priate strategy would be to continue hydrochlorothiazide
and lisinopril and discontinue metoprolol. • Exercise stress testing is recommended as the initial
test of choice for patients with intermediate risk of
KEY POINT
ischemic heart disease who are capable of exercising
• In patients undergoing stress testing to diagnose coro­ and have a normal resting electrocardiogram,
nary artery disease, P-blockers should be withheld for although advanced imaging is indicated if the exercise
24 to 48 hours before testing. stress test is inadequate or indeterminate.

Bibliography Bibliography
Fihn SD, Gardin JM. Abrams J, et al; American College of Cardiology Fihn SD. Gardin JM, Abrams J. et al: American College of Cardiology
Foundation; American Heart Association Task Force on Practice Foundation/American Heart Association Task Force. 2012 ACCF/AHA/
Guidelines; American College of Physicians; American Association for ACP/ AATS/PCNA/SCAl/STS guideline for the diagnosis and management
Thoracic Surgery; Preventive Cardiovascular Nurses Association: Society of patients with stable ischemic heart disease: a report of the American
for Cardiovascular Angiography and Interventions: Society of Thoracic College of Cardiology Foundation/American Heart Association task force
Surgeons. 2012 CCF/AHA/ACP/AATS/PCNA/SCAI/STS Guideline for the on practice guidelines. and the American College of Physicians. American
diagnosis and management of patients with stable ischemic heart dis­ Association for Thoracic Surgery. Preventive Cardiovascular Nurses
ease: a report of the American College of Cardiology Foundation / Association. Society for Cardiovascular Angiography and Interventions.
American Heart Association Task Force on Practice Guidelines, and the and Society of Thoracic Surgeons. Circulation. 2012 Dec 18;126 (25) :e354-
American College of Physicians. American Association for Thoracic 47l. Erratum in: Circulation. 2014 Apr 22;129(16) :e463. [PMI D : 23166211]
Surgery. Preventive Cardiovascular Nurses Association. Society for
Cardiovascular Angiography and Interventions. and Society of Thoracic
Surgeons. J Am Coll Cardiol. 2012 Dec 18;60(24):e44-el64. [PMID:
23182125]
Item 40 Answer: c
Educational Objective: Manage cardiovascular risk in
an older woman with diabetes mellitus.
Item 39 Answer: B
This patient should start atorvastatin, discontinue simvas­
Educational Objective: Manage a patient with new­
tatin, and continue her other medications. The most recent
onset exertional angina pectoris.
cholesterol guidelines recommend a moderate- or high­
This patient with an intermediate risk of ischemic heart dis­ intensity statin, such as atorvastatin, in patients aged 40 to
ease and an inadequate exercise electrocardiographic (ECG) 75 years with diabetes mellitus who have a 10-year cardio­
stress test should undergo pharmacologic stress testing. vascular risk greater than or equal to 7.5%. This patient's
Although exercise ECG stress testing is the preferred diagnos­ 10-year cardiovascular risk is above 10%. A cardiovascu­
tic study in patients with an indication for testing who have an lar risk calculator based on the Pooled Cohort Equations
interpretable resting ECG and are able to exercise, in patients for the purpose of managing cholesterol levels is ava ilable
who are unable to meet the minimal criteria for adequacy on at http: //my.americanheart.org/professional/Statements­
this study (achievement of at least 85% of the age-predicted Guidelines/PreventionGuidelines/Prevention-Guidelines_
maximal heart rate and maximal metabolic demand) , addi­ UCM_ 457698_SubHomePage.jsp. In patients with diabetes
tional testing is indicated to appropriately evaluate for isch­ in this age group with a 10-year risk below 7.5%, a moder­
emic heart disease. Dobutamine stress echocardiography or a ate-intensity statin (such as simvastatin 20-40 mg/d) would
vasoclilator nuclear medicine stress test would be appropriate be recommended. While there are multiple options for a

1 75
Answers and Critiques

high-intensity statin, the fact that atorvastatin has a generic tension in patients with PAD because of the possibility of
alternative makes it a more attractive choice. loss of p-receptor-mediated vasodilation causing worsening
I ncreasing simvastatin from 10 mg/d to 80 mg/d is claudication, this has not been supported by study data.
incorrect, as the FDA issued a black box warning against the Therefore, p-blockers may be used in patients with PAD for
use of simvastatin 80 mgld because of a heightened risk of blood pressure control. However, this patient's hypertension
muscle adverse effects. is wel l controlled, and P-blockade is not indicated as therapy
Continuing the patient's current medications is incor­ for claudication symptoms.
rect because this patient's cardiovascular risk warrants Clopidogrel or another thienopyridine should be added
change to a moderate- or high-intensity statin. to aspirin therapy in all patients following an acute coronary
The addition of clopidogrel to this patient's drug regi­ syndrome and in those undergoing coronary stent place­
men is incorrect because dual antiplatelet therapy (such as ment. However, there is no benefit in adding clopidogrel to
aspirin plus clopidogrel) increases the risk of bleeding and is aspirin in patients with PAD for treatment of the vascular
not routinely recommended for patients for primary preven­ occlusion or reducing the risk of cardiovascular events.
tion of cardiovascular events. In the Warfarin Antiplatelet Vascular Evaluation (WAVE)
Despite a negative exercise stress test, aspirin therapy trial among patients with PAD, the combination of an oral
would be recommended in this woman who has several risk anticoagulant and antiplatelet therapy was not more effec­
factors for cardiovascular events and stroke, including hyper­ tive than antiplatelet therapy alone in preventing major
tension, type 2 diabetes me!Jitus, hyperlipidemia, and her age. cardiovascular complications and was associated with an
In the Women's Health Study of 40,000 healthy women, 100 increase in life-threatening bleeding.
mg/d of aspirin decreased the risk of stroke, myocardial infarc­
KEY POINT
tion, and cardiovascular death in patients older than 65 years.
• Cilostazol has been shown to be effective at improving
KEY POINT pain-free walking and overall walking distance in
• A moderate- or high-intensity statin is recommended patients with claudication.
for patients aged 40 to 75 years with diabetes mellitus
who have a 10-year cardiovascular risk greater than or Bibliography
equal to 7.5%. Pande RL. Hiatt WR. Zhang P, Hittel N. Creager MA. A pooled analysis of the
durability and predictors of treatment response of cilostawl in patients with
intermittent claudication. Vase Med. 2010 Jun:l5(3):181-8. [PMID: 20385711]
Bibliography
Stone NJ, Robinson JG, Lichtenstein AH. et al: American College of
Cardiology/American Heart Association Task Force on Practice

Cl
Guidelines. 2013 ACC/AHA guideline on the treatment of blood choles­ Item 42 Answer: E
terol to reduce atherosclerotic cardiovascular risk in adults: a report of
the American College of Cardiology/American Heart Association Task Educational Objective: Manage an accelerated idioven­
Force on Practice Guidelines. J Am Coll Cardiol. 2014 Jul 1:63(25 Pt tricular rhythm following myocardial infarction.
8):2889-934. Erratum in: J Am Coll Cardiol. 2014 Jul 1:63(25 Pt 8):3024-
3025. [PMID: 24239923]
111is patient requires no further intervention at this time. She
developed a wide complex rhythm shortly after percutaneous
Item 4 1 Answer: B coronary intervention and reperfusion of her infarct-related
artery. The electrocardiogram (ECG) shows a regular wide
Educational Objective: Manage claudication with phar­
complex rhythm at 92 min with no clearly discernible atrial
macologic therapy in a patient with peripheral arterial
activity findings consistent \.\'ith accelerated idioventricular
disease.
rhythm (AIVR) . A l \I R is postulated to result from abnor­
The most appropriate treatment is the addition of cilostazol. mal automaticity in the subendocardial Purldnje fibers. It is
Cilostazol is an oral phosphodiesterase-3 inhibitor that has observed in up to 1 5°!.. of patients who undergo reperfusion
demonstrated increases in pain-free walking and overall of an infarct-related artery. 111e rate is almost always less
walking distance in patients with claudication in randomized than 120 min and usually less than 1001 min. �ost studies
clinical trials. Cilostazol is contraindicated in patients with have shown that it is a benign rhythm when it occurs within
heart failure or a left ventricular ejection fraction below 40%. 24 hours of reperfusion. ·n1is patient is tolerating the rhythm
This contraindication exists because cilostazol has a similar vvell and is already on a �- blocker for post-myocardial i n farc­
pharmacologic action to the inotropic drugs milrinone and tion care: therefore. no intervention is required.
amrinone, which demonstrated increased mortality rates with Neither amiodarone nor lidocaine is indicated because
long-term use in patients with heart failure. In the absence of A IVR is a benign ventricular arrhythmia and usually does
heart failure, a therapeutic trial of cilostazol should be consid­ not recur. Studies of prophylactic lidocaine after acute cor­
ered in all patients with lifestyle-1.imiting claudication. onary syndromes have demonstrated potential harm. and
Antihypertensive therapy is recommended for reduc­ amiodarone has been associated with decreased survival
tion of cardiovascular events in patients with peripheral after myocardial infarction.
arterial disease (PAD) . Although concern has been raised in Cardioversion is not indicated because Al\IR is a tran­
the past regarding use of P-blockers for treatment of hyper- sient rhythm and. in this patient. it is well-tolerated.

1 76
Answers a n d Critiq u es

CJ reperfusion
CONT.
A I VR usua l ly ind icates successful (or at least part i a l )
and i s co nsidered a reversi bl e arrhyt h m ia .
K EY P O I N T

I mplantable cardioverter-defibri l l a tor ( !CD) placement is not • Anti-inflammatory therapy with aspirin or other
indicated a l t h is time given t he patienl·s recent revascular­ NSAIDs, such as ibuprofen, is indicated in patients with
ization and nature of t he arrhy t h m ia . I f t h e left ventricular acute pericarditis; when the pericarditis is associated
ejection fract ion remains low despite med ical t herapy. ! CD with myocardial infarction, only aspirin should be used
placement m ight be i n dicated in the fu ture. because other NSA!Ds can impair myocardial healing
and increase the risk of mechanical complications.
K EY P O I NT
• Accelerated idioventricular rhythm is a common Bibliography
complication following coronary reperfusion a n d Lotrionte M. Biondi-Zoccai G, lmazio M, et al. I nternational collaborative
does not require intervention when it occurs within systematic review of controlled clinical trials on pharmacologic treat­
ments for acute pericarditis and its recurrences. Am Heart J. 2010
24 hours of reperfusion. Oct;l60(4):662-70. [PMID: 209345601

Bibliography Item 44 Answer: B


Bonnemeier H, Ortak J. Wiegand UK. et al. Accelerated idioventricular
rhythm in the posHhrombolytic era: incidence, prognostic implications.
Educational Objective: Manage dual antiplatelet CJ
and modulating mechanisms after direct percutaneous coronary inter­ therapy after drug-eluting stent placement following
vention. Ann Noninvasive Electrocardiol. 2005 Apr; 10(2): 179-87. [PMID: non-ST-elevation myocardial infarction.
15842430]
This pat ient. who has u ndergone drug-elut ing stent place­
ment follovvi ng a non - ST-elevat ion myocardial i nf arct ion .

CJ Educational Objective:
Item 43 Answer: A should co ntin ue taking ticagrelor for l year. He also should
Treat a patient with acute cont i nue taking aspirin i ndeftni tely. T he American College of
pericarditis with high-dose aspirin. Cardiology American Hean Associat ion percutaneous coro­
nary intervention ( PC ! ) guideli nes recommend at least l year
This patient should receive high-dose aspi ri n . He has acute
of dual anti platelet t herapy for patients undergoing PCI with
pericard i t is in the setting of a recent myoca rdial i n farct ion .
a drug-elut i ng stent ( DES) .
ll1e typical chest pa in , physical exa m i nation fi ndi ngs. and
A n t i pla telet d rugs i nd icated for patients who h ave
abnormal electrocardiogram (ECG) are a l l consistent with this
received a d rug- e l u t i ng stenl following an acute coronary
diagnosis. especially t he fi ndi ngs of concave upward ST-seg­
syndrome are clopidogre l . t icagre lor. and prasugre l . In the
ment elevation and PR- segment depression in a l l leads. except
PLATe let i n h i b i t ion and pat ient Outcomes (PLATO) t r i a l .
aVR. on t he ECG. Anti-inflam matory t herapy with aspi1in or
t i cagrelor was found to b e superior Lo clopi dogrel i n reduc­
other N SA ! Ds. such as ibuprofen. is indicated in patients with
ing the i ncidence of cardiovascular dea t h . myocardial in farc­
acute pericard i tis. I n those whose pericarditis is associated
t io n . and st roke fol lowing an acute coronary syndrome.
with myocardi a l i n farction. such as t his pat ient. only aspirin
Dual a n t iplatelet t herapy for patients who have under­
should be used because ibuprofen and other NSAI Os can
gone pl acement of a bare metal sten t fol lowing a n acute
i mpair myocardia l healing and increase t he risk or mechani­
coronary syndrome i s ind icated for a t least 4 weeks and up
cal compl ications. ll1e a n t i-inflammatory medication should
to l yea r. Fol lowing placement o f a DES. h owever, a 30-day
be given i n relatively high doses to ach ieve an a n t i -in flamma­
duration of t icagre lor i s insu fficient.
tory e f fect and t hen tapered slowly over 2 to 4 weeks lo reduce
Despite numerous studies about t he risk o f very late
t he risk of recurrent pericard i tis. Colchicine (0. 5- t .2 mg d)
stent t h rombosis. defined as the occurrence of stent t h rom­
also has been shown to be effective as adjunctive t herapy to
bosis greater than I ye<Jr after placemen t . t here is no recom ­
anti-inflammatory agents i n patients with acute pericarditis.
mendation for l i fe l ong dual a n l iplatelet therapy in current
fw 1her red ucing t he risk of recurrent pericarditis and t reat­
trea t ment guidel i nes.
ment fai lure. Colch icine is not recommended for pat ients
Stopping ticagrelor and starting clopidogrel is i nconect
with post-i n farction pericardi tis. Colchicine may be associ­
because there is no i ndication to stop t icagrelor. an agent that
ated with gast rointestinal side effects. l iver tox icity. and bone
appears to be superior to clopidogrel i n i mproving cardiovascu­
ma rrow suppression but is genera l ly well tolerated .
lar outcomes following an acute coronary syndrome. Cost and
N i t roglycerin is an e f fect ive t herapy for chest pain
adverse events (i ncluding bleeding. dyspnea. and bradycardia)
caused by myocardial ischem ia but is not e ffec t ive for symp­
are t he most common reasons for discontinuation of ticagrelor.
toms caused by pericard i t i s.
G l u cocorticoids. such as pred n isone. are reserved for KEY POINT
patients w i t h con t ra i ndications to N S A I Ds or t hose w i t h • At least 1 year of dual antiplatelet therapy is recom­
refractory acute pericarditis. primarily because t here is evi ­ mended for patients undergoing percutaneous coro­
dence t hat t heir use i s associated with an increased risk of
nary intervention with a drug-eluting stent (DES) ;
recurrent pericarditi s. As t h is pa tient has no apparent con­
therapeutic options for agents to be taken with aspirin
t ra i nd ication to aspir i n u se. t reatment w i t h glucocorticoids
are clopidogrel, ticagrelor, and prasugrel.
i s not in d icated.

1 77
Answers a n d Critiq u es

Bibliography echocardiogram, consistent with rheumatic heart disease.


Wallentin L, Becker RC, Budai. A, et al. Ticagrelor versus clopidogrel in Indications for mitral valve surgery include symptomatic
patients with acute coronary syndromes. N Engl J Med. 2009 Sep severe rnitral regurgitation with left ventricular (LV) ejection
10;361(11):1045-57. [PMID: 19717846]
fraction greater than 30% and asymptomatic severe mitral
regurgitation with mild to moderate LV dysfunction (ejection
ltem 45 Answer: D fraction of 30%-60% and/or LV end-systolic diameter 2'.40
mm). Although surgical valve repair is generally preferred
Educational Objective: Manage a patient with ventricu­
to valve replacement, it may not be possible with extensive
lar septal defect.
calcification of the valve leaflet or annulus, prolapse of more
The most appropriate management for this patient with than one third of the leaflet tissue, or extensive destruction
a small uncomplicated ventricular septa! defect (VSD) is of the chordal apparatus. The choice of valve intervention
follow-up in 3 to s years. She has clinical features of a and prosthetic valve type should be a shared decision-mak­
small membranous VSD without associated volume over­ ing process. When valve replacement is indicated, as in this
load of the left heart, pulmonary hypertension, or valve patient, a bioprosthesis is preferred in patients of any age
regurgitation. for whom anticoagulant therapy is contraindicated, cannot
Cardiac catheterization is not indicated for this patient be managed appropriately, or is not desired. In addition, a
because the echocardiogram does not demonstrate left heart bioprosthesis is a reasonable choice for patients older than
enlargement or features that suggest pulmonary hypertension. 70 years. This patient's history of gastrointestinal bleeding
Cardiac magnetic resonance (CMR) imaging will usu­ and her advanced age make a bioprosthetic valve the more
ally demonstrate a membranous VSD and can quantitate reasonable choice.
the impact of the VSD on the left heart; however, it is not Intravenous vasodilator therapy improves forward
indicated in this patient because the clinical assessment blood flow and symptoms acutely in patients with severe
and echocardiogram suggest that observation is appropriate. mitral regurgitation. However, there is little evidence that
CMR imaging would be a reasonable test if the transthoracic long-term oral vasodilator therapy is beneficial for symp­
echocardiogram images were not diagnostic. tomatic mitral regurgitation, and it is used primarily in
Endocarditis prophylaxis is recommended for patients patients unable to undergo surgical correction. Because this
with congenital heart disease characterized by unrepaired patient has no clear contraindication to surgical therapy,
cyanotic congenital heart disease, including palliative shunts medical therapy alone would not be appropriate.
and conduits; a completely repaired congenital heart defect Percutaneous mitral valve repair procedures include
with prosthetic material or device during the first 6 months percutaneous valvuloplasty or implantation of a clip to
after the procedure; and repaired congenital heart disease reduce regurgitant flow. However, these procedures are
with residual defects. Patients with uncomplicated VSDs either under investigation or their use is limited to a small
without a prior history of endocarditis do not require endo­ group of patients.
carditis prophylaxis.
Functional aerobic capacity measured by stress testing KEY POINT
is not used to determine management in patients with small • When valve replacement is indicated, a bioprosthesis
VSDs. A small VSD does not impact exercise capacity and is preferred in patients of any age for whom anticoag­
thus assessment of exercise capacity will not change man­ ulant therapy is contraindicated, cannot be managed
agement in this patient. appropriately, or is not desired.
K EY P O I NT
Bibliography
• A small membranous ventricular septa! defect with­
Nishimura RA. Otto CM. Bonow RO. et al ; American College of Cardiology/
out left heart enlargement, pulmonary hypertension, American Heart Association Task Force on Practice Guidelines. 2014
AHA/ACC guideline for the management of patients with valvular heart
recurrent endocarditis, or valve regurgitation can be
disease: executive summary: a report of the American College of
observed clinically. Cardiology I American Heart Association Task Force on Practice
Guidelines. J Am Coll Cardiol. 2014 Jun 10:63(22):2438-88. Erratum in: J
Am Coll Cardiol. 2014 Jun 10;63(22):2489. [PMID: 246031921
Bibliography
Penny DJ, Vick GW 3rd. Ventricular septa I defect. Lancet. 2011 Mar
26:377(9771):1103-12. [PMID: 21349577]
Item 47 Answer: c
Educational Objective: Evaluate a patient with new-onset
Item 46 Answer: A heart failure who has evidence of coronary artery disease.
Educational Objective: Treat mitral regurgitation with
The most appropriate diagnostic test to perform next in this
bioprosthetic mitral valve replacement.
patient is myocardial perfusion imaging stress testing. This
The most appropriate treatment for this patient is mitral patient has evidence of new-onset heart failure as evidenced
valve replacement (MVR) with a bioprosthetic valve. She by her clinical presentation (decreased exercise tolerance,
has a history of rheumatic fever and extensive degenera­ jugular venous distention, crackles on lung examination,
tion of the mitral valve with severe rnitral regurgitation on and lower extremity edema) . A diagnosis of new-onset heart

1 78
Answers a n d Critiques

failure should be confirmed by echocardiography, which Bibliography


has both high sensitivity and specificity for heart failure and Hunt SA. Abraham WT, Chin MH, et al; American College of Cardiology
may be useful in evaluating for specific possible causes of Foundation: American Heart Association. 2009 Focused update incorpo­
rated into the ACC/AHA 2005 Guidelines for the Diagnosis and
heart failure, such as valve dysfunction. Treatable causes of Management of Heart failure in Adults. A Report of the American College
heart failure include coronary artery disease (CAD), thyroid of Cardiology Foundation/ American Heart Association Task Force on
Practice Guidelines Developed in Collaboration With the International
disease, alcohol abuse, and some valvular diseases (such as Society for Heart and Lung Transplantation. J Am Coll Cardiol. 2009 Apr
aortic stenosis, if repaired early) and should be looked for 14;53(15):el-e90. Erratum in: J Am Coll Cardiol. 2009 Dec 15;54(25):2464.
[PMID: 19358937]
during the initial evaluation.
As many as two thirds of cases of heart failure are
caused by CAD. This patient has several risk factors for car­
Item 48 Answer: A
diovascular disease, including hypertension and a history
of smoking. Her electrocardiogram (ECG) demonstrates left Educational Objective: Manage a patient with diabetes

bundle branch block and her echocardiogram demonstrates mellitus presenting with stable angina pectoris not con­

an akinetic left wall, both of which suggest CAD. Patients trolled with optimal medical therapy.

with heart failure and multiple risk factors or symptoms This patient with stable angina pectoris with symptoms that
of CAD should be evaluated by either a stress test or car­ are not adequately controlled on optimal medical therapy
diac catheterization. The reason to evaluate for CAD is that should undergo left heart catheterization for further evalu­
revascularization by either percutaneous coronary inter­ ation and potential revascularization. Her myocardial perfu­
vention (PCI) or coronary artery bypass graft surgery may sion imaging results are consistent with ischemic coronary
improve her left ventricular ejection fraction and reduce her artery disease (CAD) ; however, these findings alone would
symptoms of heart failure. Noninvasive exercise testing is not be an indication for left heart catheterization. In patients
often performed initially to provide information about the with stable angina pectoris, coronary revascularization has
possible presence of ischemic heart disease but also to assist not been shown to improve morbidity or mortality, and thus
in risk stratification and prognosis. Cardiac catheterization is not indicated in patients whose symptoms are able to
may be helpful in patients with suggestive findings on non­ be controlled with optimal medical therapy. However, in
invasive testing or may be an appropriate initial study in patients with coronary ischemia who fail to respond to ade­
selected patients. quate antianginal therapy, such as this patient, coronary
Cardiac magnetic resonance (CMR) imaging is not part angiography is indicated to evaluate for possible revascular­
of the routine evaluation of new-onset heart failure but ization to control her angina symptoms. Catheterization may
may be used if an infiltrative or an inflammatory process is allow for percutaneous intervention to address a coronary
suspected, such as myocarditis, hemochromatosis, Wilson occlusion leading to her angina symptoms, or assessment for
disease, or sarcoidosis. If the patient's evaluation for CAD the need for surgical revascularization if extensive or com­
as a cause of her heart failure is normal and myocarditis is a plex CAD is present.
consideration, CMR imaging may be a reasonable test. CT angiography is an emerging technology for the non­
Coronary artery calcium scoring is a method of mea­ invasive evaluation of the coronary arteries. Although it may
suring vascular calcification in the coronary arteries, be able to confirm the diagnosis of CAD in this patient, it
with increased levels of calcium being associated with an would not allow the opportunity for percutaneous coronary
increased burden of atherosclerotic plaque and cardiac intervention, if possible. The use of CT angiography to esti­
events. Its optimal use may be in providing additional infor­ mate the need or benefit of coronary artery bypass grafting
mation for making therapeutic decisions in asymptomatic also has not been established. Therefore, this study would
patients at intermediate risk for atherosclerotic cardiovas­ not be indicated in this clinical setting.
cular disease. However, its role in evaluating patients with Dobutamine stress echocardiography is typically used
heart failure believed to be caused by CAD has not been to evaluate for ischemic CAD in patients who are unable to
established. exercise. However, in this patient with documented coro­
Endomyocardial biopsy is indicated in patients with nary ischemia established by a nuclear medicine myocardial
heart failure that progresses despite medical therapy and perfusion study, there would be no benefit to performing
those with malignant arrhythmias to evaluate for giant cell this alternative diagnostic study for ischemia.
myocarditis, as well as in those in whom amyloidosis or Because this patient remains symptomatic with restric­
hemochromatosis is suspected. Endomyocardial biopsy is tions on her quality of life, continuing her current medi­
not indicated in this patient with evidence of heart failure in cal therapy without additional intervention would not be
whom CAD has not been evaluated. appropriate.

KEY POINT KEY P O I NT


• Patients with new-onset heart failure with multiple • Patients with stable angina not adequately controlled
risk factors or symptoms of coronary artery disease with optimal medical therapy should undergo coro­
should be evaluated by either a stress test or cardiac nary angiography to evaluate for possible revasculari­
catheterization. zation.

1 79
Answers and Critiques

Bibliography KEY P O I NT
Fihn SD, Gardin JM, Abrams J, et al ; American College of Cardiology
• In patients with mitral regurgitation, transthoracic
Foundation/American Heart Association Task Force. 2012 ACCF/AHA/
ACP/AATS/PCNA/SCAl/STS guideline for the diagnosis and manage­ echocardiography (TTE) is used to evaluate the
ment of patients with stable ischemic heart disease: a report of the
degree of m i t ral regurgitation and assess the causa­
American College of Cardiology Foundation /American Heart
Association task force on practice guidelines. and the American College t ive valve abnormalities, t hereby providing essential
of Physicians. American Association for Thoracic Surgery, Preventive
information for guiding therapy; if visualization is
Cardiovascular Nurses Association. Society for Cardiovascular
Angiography and I nterventions. and Society of Thoracic Surgeons. inadequate with TTE, transesophageal echocardiog­
Circulation. 2012 Dec 18: 126(25) :e354-471. Erramm in: Circulation. 2014
raphy is indicated.
Apr 22;129(16) :e463. lPM ID: 231 66211]

Bibliography
Item 49 Answer: D Solis J, Piro V, Vazquez de Prada JA. Loughlin G. Echocardiographic assess­
ment of mitral regurgitation: general considerations. Cardiol Clin. 2013
Educational Objective: Use appropriate imaging to May:3l(2):165-8. [PMID: 23743069]
evaluate mitral regurgitation.

Transesophageal echocard i ography (TEE) would be Item 50 Answer: A


t h e most appropriate next step i n management of t h i s
Educational Objective: Evaluate infrequent palpitations
patient . Evaluating the degree o f m i tral regurgi tation
in a patient without syncope.
a n d assessing the causative valve abnormalities provides
essential informat ion fo r guiding therapy, particularly An event recorder would be appropriate to evaluate this
whether surgical repair is possible or indicated. Although patient's episodic symptoms of palpitations and lightheaded­
t h i s information is usually obtai ned w i t h transthoracic ness. The evaluation and identification of arrhythmias can be
echocardiography, in situations in which full evaluation challenging because of their intermittent nature; the modality
o f the valve and degree o f m itral regurgitation i s not pos­ used depends on the frequency and nature of the symptoms.
sible, such as in this very obese patient i n whom accurate In patients with infrequent symptoms, event recorders are
Doppler parameters cannot be obtained, TEE i s indi­ the most effective means of obtaining rhythm information
cated. Because the transducer in T E E is able to be closely around the time of the symptoms. Event recorders record
approximated to the left atrium and m i t ral valve, struc­ electrocardiographic (ECG) tracings only when activated by
tural details, including integrity of the valve leaflets and the patient and are more useful for infrequent symptoms. Two
chordae, and the regu rgitant jet, are a l most always well types of event recorders are used. A patient-triggered recorder
visu a l ized, allowing assessment of va lvular abnorma l i t ies is an external device without leads that is held to the chest and
and accurate measurement of Doppler parameters. Chest triggered by the patient when symptoms occur. The advantage
CT angiography and cardiac magnetic resonance i maging of this device is the lack of electrode leads, making it more
may also be useful adjuvants, allowing assessment of comfortable and convenient; however, no preceding rhythm
coronary anatomy, valve structure, and coexisting struc­ is saved when the device is triggered. When the patient acti­
tural heart disease. Three-d imensional echocardiography, vates a looping event recorder, a permanent recording is cre­
both transthoracic and transesophagea l , can fu rther help ated that includes several seconds of the presymptom rhythm,
identify l e a flet and scal lop involve ment, like l i hood o f which is useful in patients with syncope.
operative repa i r, and additi onal quantification o f m i t ral In patients in whom pathologic structural heart disease
regurgi tation severi ty. is suspected, transthoracic echocardiography provides non­
The decision to proceed with mitral valve surgery, invasive, quantitative evaluation of cardiac size and function,
including mitral valve repair, would be premature in this valve morphology and function, pericardia! disease, pulmo­
patient without adequate knowledge of the severity of mitral nary a rtery pressures, and proximal great vessels. Physical
regurgitation and better definition of the underlying valvular examination findings in patients with structural disease
abnormalities. include signs of heart failure and abnormal cardiac auscul­
Although clinical follow-up with serial echocardiogra­ tation. I n this patient, there is no evidence of structural heart
phy may be appropriate for this patient, the appropriateness disease by either history or examination, so an echocardio­
of this management strategy depends upon adequate i n i t i a l gram is not warranted.
evaluation of her mitral valve disease. Exercise ECG stress testing allows diagnosis of exer­
Medical therapy for patients with asymptomatic m itral cise-related arrhythmias, as well as assessment of the impact
regurgitation is limited. To date, no studies have shown of the arrhythmia on blood pressure. The patient does not
benefit of ACE inh ibitors, angiotensin receptor blockers, describe exercise-related symptoms, and an exercise stress
or diuretics in the absence of another specific indication, test is not indicated.
such as hypertension, left ventricular systolic dysfunction, Ambulatory ECG monitors record continuous rhythms
or evidence of volume overload. This patient has none of and can be useful when patients have frequent, usually daily,
those features. symptoms. Monitors are usually worn for 24 or 48 hours.

1 80
Answers a n d Critiq u es

Patients are encouraged to perform their normal activities a x i s 1 23 d egrees ) : howeve r. t h ere is a lso evidence or le f'l pos­
while wearing the monitor. Patients keep a symptom diary terior l�1 scic u l a r bloc k (sm a l l r w:wes a n d deep S waves i n
or trigger a marker on the continuous reading that correlates leads I a n d ci \/ L: q R complexes in leads 1 1 . 1 1 1 . ci nd a \/ F) . T h us .
with symptoms. Ambulatory ECG monitors can also be use­ t hese fe<i t u res a re most consistent w i t h aberra n t cond u c t i o n
ful to detect asymptomatic arrhythmias, such as asymptom­ i n t he set t i ng of' a t ri a l f i bri l l a t ion ra t h e r t h a n ven tricu l a r
atic atrial fibrillation. This patient's symptoms occur about tac hyca rd ia .
once a week, and a 24- or 48-hour ambulatory monitoring I n t ravenous proc a i n a m i d e wou l d be t h e <1gen t ol' choice
period is likely to miss the symptomatic episodes. i f t h i s tachycard i a were pre-excited (Wol ff'- Pa rki nson -Wh i te

KEY POINT syn d ro m e ) . IJre-exc i ta t i o n is evide nced by t he presence


o f a d e l t a wave. T h i s pa t i e n t' s elect roca rdiogram does not
• In patients with infrequent symptoms, cardiac event
demonst ra te p re- exci ta t ion in e i t her the s i n u s beat o r t h e
recorders are the most effective means of obtaining tachycard i a .
rhythm information around the time of the symptoms.
KEY POINT
Bibliography • Paroxysms of an irregularly irregular rhythm with a
Zimetbaum P, Goldman A. Ambulatory arrhythmia monitoring: choosing typical right bundle branch block appearance on elec­
the right device. Circulation. 2010 Oct 19:122(16):1629-36. [PMID:
trocardiogram most likely represent atrial fibrillation
20956237]
with aberrant conduction.

Item 5 1 Answer: B Bibliography


Educational Objective: Treat wide-complex tachycardia Goldberger ZD. Rho RW. Page RL. Approach to the diagnosis and initial
management of the stable adult patient with a wide complex tachycar­
due to atrial fibrillation with aberrant conduction.
dia. Am J Cardiol. 2008 May 15;101 (10) : 1456-66. Erratum in: Am J
Cardiol. 2008 Aug I :1 02(3):374. [PMID: 18471458)
The mosl appropriale next slep in t re<H m e n t or t h is pal ienl is
to i n i ti a te �- b locker t herapy. He has paroxysmal at rial A bril ­
l a t ion w i t h aberra n t conduc tion . res u l t i n g in a wide-comp lex Item 5 2 Answer: A
tachyca rd ia . The electroca rdiogram demonslrates a normal
Educational Objective: Manage cardiovascular risk
sinus beat fol lowed by a run of' a t rial A b ri l la tion with righl
reduction in a patient with peripheral arterial disease.
bu n d le branch block. Note the i rregularly i rregu l a r n a t u re
of t he tachyca rdia and t h e QRS morp h o l ogy con sistent w i t h The most appropriate management is to start a moderate­
typical r i g h t bundle branch bl ock. Given h i s rapid ven t ricu ­ or high-intensity statin. Peripheral arterial disease (PAD) is
l a r response a n d h i s symptoms or p a l p i t a l i o n s and dyspnea . strongly associated with smoking, diabetes mellitus, and
t he a t ri a l Abri l la t ion req u i res t re<1 t m e n l . � Blocker t herapy is aging. PAD is defined noninvasively by calculation of the
t he preferred a t riovent ricu l a r nodal block i ng age n t given t h e ankle-brachia) index (ABI) . An AB! of 0.90 or below is diag­
patient's h istory of coronary artery d i sease. nostic of PAD. Most patients with PAD are asymptomatic;
Assessment o f' t h e need for a n t icoagu l a t i o n t herapy is approximately 25% have symptoms referable to circulatory
a l so i n d icated i n t h i s p a t i e nt w i t h a t r i ;:i l Ab ri l l;:i t i o n . Cu rre n t compromise. PAD is considered a coronary artery disease risk
guidel i nes reco m mend the u s e o f t he
CH ApS2 -VASc score equivalent and statin therapy has been demonstrated to lower
for t h i s purpose. rep l a c i ng t he CHADS2 score because or i t s cardiovascular events in patients with PAD.
a b i l i ty to more clearly d iscri m i n a t e stroke risk . T h is p a t i e n t Exercise and cilostazol are effective therapies for
has a C H .1\ DS2-VASc score o r 4 (J p o i n t f o r hyperten sion . 2 patients with stable symptomatic PAD. Cilostazol signifi­
p o i n t s for age. a n d J poi n t for coron;:iry a rtery d i sease) . p l ac­ cantly increases pain-free walking time and maximal walk­
i ng h i m a t moderale risk for stroke. T herefo re. i n i t i a t ion of' ing time, although the gains with structured exercise are
oral a n t icoagu l ;:i t i o n a l so is ;:i p p ropri<lte. two- to three-fold greater than with cilostazol alone. Since
Emergent card i overs i o n is not necessary becrnse t he this patient is asymptomatic, cilostazol is not indicated.
p a t i e n t is hemocly n a m i ca l ly stable a n d appears to be havi ng Antiplatelet therapy is indicated for all patients with
sel f- te r m i nating paroxysms o f' tachyca rd ia . I f' the pa t ie n t symptomatic PAD, previous lower extremity revascularization,
h a d a sustai ned a rrhyt h m i a accom pa nied by hemody n a m i c or amputation clue to PAD. Antiplatelet therapy is reasonable
i n stabi l i ty. e m e rge n t card i overs i o n wou ld be i n d i ca ted in patients with asymptomatic PAD, particularly if they have
reg<.ml less o f the spec i fic e t i ology of' the <1tThy t h m i a ( t hat is . evidence of atherosclerosis elsewhere (coronary or cerebral
sup rave n t ri c u l a r versus ven t ric u l a r) . arteries). Combination treatment with an antiplatelet agent
I n t ravenous a m iodarone wou l d be an appropri<lle t rea t ­ and warfarin, and warfarin monotherapy (adjusted to an lNR
ment for recur re n t ven t ri c u l a r tachyca rd ia . l l1e el ectrocar­ of 2.0-3.0) , is no more effective than antiplatelet therapy alone
d iogram appearance is co n s i s t e n t w i t h right b u n d l e b ra n c h and carries a higher risk of life-threatening bleeding.
b loc k. There i s a n rSR pa ttern i n l e a d V, a n d a term i n a l S Noninvasive angiography is performed for anatomic delin­
wave in leads I and \/6 . Right a x i s dev i a t i o n is presen t (QRS eation of PAD in patients requiring surgical or endovascular

1 81
Answers a n d Critiques

intervention. CT angiography (CTA) i s rapid and easily available support t he use of an aldosterone a ntago nist such as epler­
but requires the administration of intravenous contrast dye. enone. Based on the EPH ESUS ( Eplereno ne Post-A M I Heart
While CTA compares favorably with digital subtraction (inva­ Fai lure Effi cacy and Surviva l ) t r i a l . the 2007 American Col ­
sive) angiography for the detection of occlusive arterial disease, lege of Cardiology /American Heart Association gu idel i nes
imaging is not needed at this time because the patient does not recommend the a d m i nistra t ion of an aldosterone an tago­
require surgical inte1vention. nist to all pat ien t s fol l owing a non- ST-elevation myoca rd i a l
Lower extremity segmental pressure measurement can i n fa rction (NSTEM I ) w h o a re receivi ng a n A C E i n h i bi tor,
help determine the level and extent of PAD. Using special­ have a left ven tricu lar ejection fraction of 40'Y., or below. and
ized equipment in the vascular laboratory, blood pressures have e i t her heart fai lure sym ptoms or d iabetes mel l itus.
are obtained at successive levels of the extremity, localizing ACE i nh i b i tors i nh i b i t postinfarction remodel i ng. hel p­
the level of disease. Many vascular laboratories use air pleth­ i ng to preserve ven t ricular function . ACE i n h i bi tors shou l d
ysmography to measure volume changes within the limb, in b e cont i n ued indefi n itely.
conjunction with segmental limb pressure measurement.
KEY POINT
Lower extremity segmental pressure measurement is not
needed at this time because localization of disease is not • Long-term therapy following myocardial infarction
needed to guide therapy. such as would be required if surgi­ includes aspirin, a �-blocker, an ACE inhibitor, and a
cal intervention were being planned. statin; a P2Y 12 inhibitor (clopidogrel, prasugrel, tica­
grelor) should be continued for at least 1 year for
KEY POINT
patients undergoing coronary percutaneous interven­
• Patients with peripheral arterial disease should be tion with stent placement.
treated with a moderate- or high-intensity statin.
Bibliography
Bibliography Makki N, Brennan TM, Girotra S. /\cute coronary syndrome. J Intensive Care
Berger JS. Hiatt WR. Medical t herapy in peripheral artery disease. Med. 2013 Sep 18. [PMID: 24047692]
Circulation. 2012 Jul 24:126(4):491-500. [PMID: 228254 11]

Item 54 Answer: B

Cl Educational Objective:
Item 53 Answer: E Educational Objective: Treat heart failure with reduced
Choose appropriate antiplatelet ejection fraction with a �-blocker.
agents following acute coronary syndrome and percutane­
This patient with a recent diagnosis of heart failure with
ous coronary intervention.
reduced ejection fraction (HFrEF) should be started on a
No changes should be made to this pat ient"s medicat ions at �-blocker, such as carvedilol. Standard therapy for patients
t he t i me o f hospital discharge. with HFrEF includes an ACE inhibitor and a �-blocker. This
Calcium chan nel blockers . wi t h t he except ion o f n i fed ­ patient is already on an ACE inhibitor for treatment of his
i p i ne. can be used in patients with co n t ra i nd icat ions to blood pressure and for afterload reduction for his heart fail­
�- blockers and i n t hose wi t h cont i n ued angina despi te opti ­ ure. ACE inhibitors are typically started first in patients with
mal doses of �- blockers and n i t ra tes. l l 1 is patient has no heart failure because of their positive hemodynamic effects.
i n d i cations for a calcium channel blocker such as cl i l t iazem. An angiotensin receptor blocker (ARB) would be another
ll 1ere is no evidence t o support a change from t icagre­ treatment option, particularly if an ACE inhibitor were not
lor to clopidogrel a fter percutaneous coro n a ry i n tervention tolerated. A �-blocker should then be started in stable, euvole­
(PCI) for acute coronary synd rome. In the PLATO ( PLATelet mic patients with heart failure, either at the time of diagnosis
i n h i bi t ion and pat ient Outcomes) t r i a l . t h e use of t i cagrelor or after acute decompensation is treated. �-Blockers have sev­
was associated w i t h a 1 . 9"/., absolute risk reduct ion i n the eral beneficial effects and have been shown to prolong overall
occurrence o r cardiovascu lar dea t h . myocardial i n farct i o n . and event-free survival.
�nd stroke w h e n comp<Jrecl w i t h clopidogre l . A P2Y1:1 i n h i b­ The P-blockers that have been shown to provide benefit
itor (clopidogre l . prasugre l . t i cagrelor) should be con l i n ued in patients with HFrEF are metoprolol succinate, ca1vedilol,
for <Jt least I year for pat ients undergoi ng PCI with stent and bisoprolol. The P-blocker dosage should be increased
placement . slowly-at 1- to 2-week intervals- to the maximal dose.
Oral � blockers should be given to a l l pat ients w i t h acute Like ACE inhibitors, there are data that suggest improved
coronary syndrome w i t hout a con t ra i ndic<Jtion (decom ­ outcomes on higher doses of �-blockers (increased ejection
pensated heart fai l ure, advanced a triovent ricular block. or fraction, reduced symptoms, lower mortality rates) ; there­
severe reactive a i 1-vvays disease) and con t i nued incleAni tely. fore, attempting to up-titrate to maximally tolerated doses
ll1 i s pat ient is a l ready bradyca rd ic. and an i n c rease i n t h e is important.
dosage o f metoprolol may b e associated w i t h sym ptomatic Although dihydropyridine calcium channel block­
bradyca rd ia. ers, such as amlodipine, are effective antihypertensive and
I n t h i s patient w i t h an acute coronary syndrome and antianginal medications, they do not provide the same ben­
preserved left ven t ricular function. t here i s no evidence to efits as ACE inhibitors, ARBs, or P-blockers, and would not

1 82
Answers and Critiq u es

be appropriate add-on therapy in this patient who is not tion: however. the presentation generally is characterized
currently on a P-blocker and has controlled blood pressure by acute dyspnea and pulmonary edema rather than platy
without angina. pnea-ort hodeox ia . Physical exa mination fi ndi ngs include a
This patient has clear lungs, no significant jugular systolic murmur at t he apex that increases during expiration
venous distention, and no peripheral edema. He has no rather t han inspiration.
evidence of volume overload and therefore does not need Severe left ventricular systolic dysfunction generally
a diuretic, such as furosemide. Diuretics have no mortality does not cause arterial oxygen desaturation. In addition. the
benefit and are only used for symptom relief in the setting initial assessment of left ventricular function was normal.
of volume overload. ·n1e clin ic<Jl picture in this patient is more compatible with
Spironolactone has been demonstrated to decrease right ventricu lar dysfunction.
mortality rates in patients with New York Heart Associa­ Ven t ricular septa! defect is a recognized complication
tion (NYHA) functional class II to IV heart failure (dyspnea a fter transmural myoca rd ial in farction; however, the pre­
with activities of daily Living) . 1his patient has good exer­ senta t ion generally includes acute dyspnea and pulmonary
cise capacity and has NYHA class II heart failure. However, edemJ rather than ox')'gen desaturation. The left-to-right
candidates for spironolactone therapy should already be on shunt associated with the ventricular septa! defect causes
standard medical therapy, including an ACE inhibitor and a left heart volume overload. rather than the right heart vol­
P-blocker. ume overload caused by right-to-left shunting seen in this
Making no changes in this patient's treatment regimen patient. Physical examination findi ngs in patients with ven ­
would not be appropriate because he is not being treated t ricular septa! defect following myocardial i n farction include
with medications associated with improved outcomes in a holosystolic murmur at the left sternal border that does not
patients with systolic heart failure. change with respirat ion .

KEV POINT KEY P O I NT


• Initial treatment for patients with heart failure with • Right-to-left shunting across an atrial septa! defect or
reduced ejection fraction (HFrEF) includes an ACE patent foramen ovaJe may rarely cause cyanosis and
inhibitor and a P-blocker; the P-blockers that have dyspnea due to deformation of the atrial septum and
been shown to provide benefit in patients with HFrEF redirection of shunt tlow that result from increased
are metoprolol succinate, carvedilol, and bisoprolol. right atrial pressure in the upright position.

Bibliography Bibliography
Parikh R, Kadowilz PJ. A review of current therapies used in the treatment Kubler P, Gibbs H. Garrahy P Platypnoea- orthodeoxia syndrome. Heart.
of congestive heart failure. Expert Rev Cardiovasc Ther. 2013 2000 Feb;83(2):221 -3. [PMID: 10648502]
Sep;ll(9):ll71-8. [ PMID: 23980607]

Item 5 6 Answer: A

Cl Ed ucational Objective:
Item 55 Answer: A Educational Objective: Manage heart failure with car­
Diagnose platypnea­ diac resynduonization therapy.
orthodeoxia syndrome.
1his patient with symptomatic heart failure and a reduced
ll1e most likely diagnosis in this patient is a patent foramen left ventricular ejection fraction with evidence of significant
ovale with right-to-left shunt. He presents with features of conduction system disease should undergo placement of a
platypnea-orthodeoxia syndrome. characterized by posilionaJ biventricular pacemaker (cardiac resynchronization therapy
symptoms of cyanosis and dyspnea that generally occur when [CRT]) . He has progressive heart failure symptoms while on
the patient is sitting and resolve in the supine position. Right­ appropriate medical therapy and has New York Heart Associ­
to-left shunting across an atrial septa! defect or patent fora­ ation (NYHA) functional class III symptoms. With his ejection
men ovale may rarely cause cyanosis and dyspnea owing to fraction less than 35% and left bundle branch block (LBBB) ,
defom1ation oft he atrial septum and redirection of shunt flow he is a candidate for a biventricular pacemaker, which has
that result from increased right atrial pressure in the upright been demonstrated to reduce mortality and symptoms in
position. ll1is patient had an i n ferior and right ventricular patients with NYHA functional class III and IV heart failme
myocardial infarction with associated right heart enlargement by improving cardiac hernodynamics. The 2013 American
and dysfunction and clinical features of hypotension. T he College of Cardiology Foundation/American Heart Associa­
right hean enlargement causes a n nular d i latation and t ricus­ tion/Heart Rhythm Society(ACCF/AHA/HRS) guideline rec­
pid regurgitation . The foramen ovale stretches and becomes ommends CRT therapy in patients with an ejection fraction
patent. ll1e preferential cyanosis is caused by the hemody­ of 35% or below, NYHA functional class III to IV symptoms
namic alterations and preferential t ransfer of right atrial blood on guideline-directed medical therapy, and LBBB with QRS
across the patent foramen in the upright position. duration greater than or equal to 150 ms. ·n1is patient already
Mitra! regurgitation due to papillary muscle injury or has an implantable cardioverter-defibrillator, which is indi­
rupture is a recognized complication after myocardial i n fa rc- cated for patients with NYHA functional class II to Ill heart

1 83
Answers a n d Critiques

failure and an ejection fraction less than 3 5 % . Now that he has are present (left bundle branch block [LBBB] , left ventric­
developed a LBBB and an increase in symptoms, it is reason­ ular hypertrophy, paced rhythm, Wolff-Parkinson-White
able to proceed with placement of a biventricular pacemaker pattern) , results may be indeterminate. This patient has
as well. none of these conditions, and therefore exercise stress
lnotropic therapy, such as dobutamine, is reserved for testing is a reasonable option. In patients who can exercise,
patients with end-stage heart failure, either as a bridge to exercise stress is preferred to phannacologic stress because
transplantation or for palliative care. Patients in this cate­ of the functional and prognostic information exercise stress
gmy often have recurrent hospitalizations for heart failure, provides. Persons who can exercise have a better prognosis
have evidence of end-organ compromise such as worsening than those who are unable to exercise and require pharma­
kidney and liver function, and have very poor exercise tol­ cologic stress testing.
erance. Although this patient has progressive symptoms, Among patients with resting ECG abnormalities that
he has not reached this stage yet, and has no indication for limit ST-segment analysis, the addition of imaging aids diag­
inotropic therapy. nostic accuracy and provides improvement in localizing the
The patient has no evidence of volume overload on site and extent of ischemia. In patients with LBBB, exercise
examination and a borderline low blood pressure; there­ stress may result in abnormal septa! motion due to conduc­
fore, increasing his diuretic dose would not be expected to tion delay with falsely positive septa! abnormalities; this
improve his symptoms and may worsen them by lowering abnormality is lessened with use of vasodilator (such as
his cardiac filling pressures and cardiac output. adenosine) stress imaging. This patient does not have ECG
The patient is fairly symptomatic but has not yet had abnormalities that warrant adenosine myocardial imaging
optimal therapy, as he has an indication for CRT and has not study and the added expense and radiation exposure that
yet received it. Left ventricular assist devices (LVADs) are this procedure would require.
reserved for patients with end-stage refractory heart failure Cardiac magnetic resonance (CMR) imaging can be
as a bridge to heart transplantation or as destination therapy used to evaluate aortic pathology, pericardia! diseases, and
in selected patients who are not candidates for transplan­ myocardial diseases, as well as to evaluate the extent of myo­
tation. However, prior to being considered for either an cardial fibrosis. CMR imaging may be useful in determining
LVAD or heart transplantation, a patient must be on optimal the extent of myocardial infarction and potential viability.
medical therapy. This patient is asymptomatic; therefore, CMR imaging is not
indicated.
KEY POINT
CT angiography a llows determination of the pres­
• Cardiac resynchronization therapy is recommended ence and extent of coronary artery d isease. I f this
in patients with an ejection fraction of 35% or below, i ntermediate-risk patient was unable to exercise or
New York Heart Association functional class III to IV the ECG was uninterpretable, CT a ngiography could be
symptoms on guideline-directed medical therapy, and performed. I f, however, obstructive disease was found,
left bundle branch block or QRS duration of 150 ms or the patient would then need to undergo coronary angi­
greater. ography to perform a percutaneous intervention, thus
performing two procedures that require contrast agents
Bibliography and radiation exposure.
Epstein AE. Di Marco JP. Ellenbogen KA, et al; American College of Cardiology For patients unable to exercise because of physical lim­
Foundation; American Heart Association Task Force on Practice
itations or physical deconditioning, pharmacologic stressors,
Guidelines; Heart Rhythm Society. 2012 ACCF/AHA/HRS focused update
incorporated into the ACCF/AHA/HRS 2008 guidelines for device-based such as dobutamine, can be used. These agents, which are
t herapy of cardiac rhythm abnormalities: a report of the American recommended if the patient cannot achieve at least five
College of Cardiology Foundation/American Heart Association Task
Force on Practice Guidelines and the Heart Rhythm Society. J Am Coll metabolic equivalents, increase myocardial contractility and
Cardiol. 2013 Jan 22:61 (3):e6-75. [PMID: 23265327] oxygen demand. This patient can exercise, and dobutamine
stress echocardiography is not indicated.
Item 5 7 Answer: E KEY POINT
Educational Objective: Evaluate a patient with cardio­ • Exercise electrocardiographic testing is the standard
vascular risk factors and atypical chest pain and a normal stress test for the diagnosis of coronary artery disease
resting electrocardiogram.
in the absence of conditions that limit ST-segment
1he most appropriate diagnostic test for this patient is analysis.
exercise stress testing. He has an intermediate pretest prob­
ability of coronary artery disease (CAD) based on his age, Bibliography
sex, and symptoms. He should undergo stress testing to American College of Cardiology Foundation Appropriate Use Criteria Task
Force: American Society o f Echocardiography; American Heart
determine if his symptoms are related to obstructive CAD. Association: American Society of Nuclear Cardiology; Heart Failure
Exercise electrocardiographic (ECG) testing is the standard Society of America; Hean Rhythm Society; Society for Cardiovascular
Angiography and Interventions; Society of Critical Care Medicine;
stress test for CAD diagnosis in patients with a normal Society of Cardiovascular Computed Tomography; Society for
baseline ECG. If abnormalities limiting ST-segment analysis Cardiovascular Magnetic Resonance, Douglas PS. Garcia MJ. Haines DE.

1 84
Answers a n d C ritiques

et al. ACCF/ASE/AHA/ASNC/HFSA/l-IRS/SCAl/SCCM/SCCT/SCM R 2011 d iagnos is may a l low m o re sign i f i ca n t c o m p l i c a t i o n s


Appropriate Use Criteria for Echocardiography. A Report of the to
American College of Cardiology Foundation Appropriate Use Criteria d e\ e l o p. ·n1erefore. observa t ion i n a h igh - ri s k p a t i e n t wou l d
Task Force, American Society of Echocardiography, American Heart n o t b e <lppropriate.
Association, American Society of Nuclear Cardiology. Heart Failure
Society of America. Heart Rhythm Society. Society for Cardiovascular KEY POINT
Angiography and Interventions. Society of' Critical Care Medicine.
Society of Cardiovascular Computed Tomography. and Society for • Patients with possible implanted cardiac device infec­
Cardiovascular Magnetic Resonance Endorsed by the American College
of Chest Physicians. J Am Coll Cardiol. 20tl Mar 1 :57(9): t l26- 66. [ PM I D : tion (with or without fever) should have a minimum
21349406] of two blood cultures drawn from separate sites.

Bibliography
Cl Ed ucational Objective:
Item 58 Answer: A
Baddour LM. Cha YM. Wilson WR. Clinical practice. Infections of' cardiovas­
Manage infection of an cular implantable electronic devices. N Engl J Mee!. 2012 Aug 30:
implanted electronic cardiac device. 367(9):842- 9. Erratum in: N Engl J Med. 2012 Oct I I ; 367(15): 1474. N Engl
J Med. 2012 Sep 27:367(13): 1272. [PMID: 22931318]
T l 1 i s pa t i ent has signs a nd symptoms conce rn i n g !or possi ­
ble i n fect ion of an i m p l a n ted cardiac elec t ro n i c device. He
should undergo laboratory eva l u a t ion i nclud i ng assess ment
Item 59 Answer: B
of' a complete blood cou n t with d i f frre n t i a l . two peripheral Ed ucational Objective: Manage asymptomatic moder­
blood cul tures from separate ph lebotomy sites . and an e ryth ate aortic regurgitation.
rocyte sed i mentat ion rate to assess for t he possibi l i ty or a
This patient with moderate aortic regurgitation should be
device- related i n fect io n .
reassessed clinically in 1 year. Patients with moderate aortic
Pa t i e n t s w i t h a n i m p l a nte d ca rd i a c dev ice can deve l o p
regurgitation should be evaluated on a yearly basis and echo­
e i t h er a l o ca l i zed t i ssue i n fe c t i o n a t t he i m p l a n t s i t e
cardiography performed every 1 to 2 years.
( pocket i n fect i o n ) o r a system i c i n fe c t i o n w i t h bactere m i <l
Aortic valve replacement is indicated for symptomatic
( to r ex a m p l e. e n d oca rcl i t i s ) . T h ese i n f e c t i o n s can occ u r
patients with chronic severe aortic regurgitation irrespec­
a ft e r i n i t i a l i m p l <rn ta t i o n . l a t e <lfter i m p l a n t a t i o n . o r fol ­
tive of left ventricular ( LV) systolic function, asymptomatic
l ow i ng a device b a t tery re p l acement or rev i s ion . I f' l e f't
patients with chronic severe aortic regurgitation and LV
u n t re a ted . i m p l a n ted ca rd i ac device i n fec t i o n s w i l l prog­
systolic dysfunction (LV ejection fraction :<:;SO%) , and patients
ress to e n doca rd i t is a ncl seps i s a n d u l t i ma t e l y dea t h . 'J h e
with chronic severe aortic regurgitation undergoing coro­
fa t a l i ty ra t e fo r an u n t reated device i n fe c t i o n a p p roaches
nary artery bypass graft (CABG) or surgery on the aorta or
75% to 100% . A n l i b i o t i c t h erapy a l o n e is i n su f f i c i e n t .
other heart valves. TI1is patient is not a candidate for aortic
C ura t i ve t he rapy req u i res a n t i b i o t ic t h erapy a n d comp lete
valve replacement.
h a rdware remova l .
Endocarditis prophylaxis is not recommended for
T h e most common pat hogens a re coagulase n eg<l l i ve
patients with bicuspid a011ic valves in the absence of another
Staphy lococcus species a n d S. a u re us. Pat ie n t s m,1y prese n t
speciAc indication such as a prior episode of infective endo­
w i t h fever or m <1 l aise: many a lso have loC<ll f- i n d i ngs sugges­
carditis, previous valve replacement, prior cardiac trans­
t i ve of' i n fect i o n . such as ery t h em<l or vva rmt h a t t h e i m p l a n t
plantation with valvulopathy, and certain forms of complex
s i t e. 'J hese p a t i e n t s s hou lcl u n dergo a laboratory eva l u a t i o n
congenital heart disease.
for s i g n s of' i n lec t i o n . E l evated ery t h rocyte sed i me n t a t i o n
Medical therapy for chronic aortic regurgitation is
rate. Jeu kocytosis w i t h a left s h i r t . a n d <rne m i a a rc suggest ive
limited. ACE inhibitors or angiotensin receptor blockers
of' i n fect ion . All pa t ie n t s w i t h poss i b l e device-related i n fec­
may be used in patients with chronic severe aortic regur­
t i o n ( w i t h or w i t hout fever) s h o u l d have a m i n i m u m of two
gitation and heart failure as wel l as in patients with aortic
b lood c u l t u res d rawn f'rom separate s i tes. Once t here is sus
regurgitation and concomitant hypertension, but these
p i c i o n tor a device i n lec t i o n , re lerral to the patient ' s electro
agents, as well as dihydropyridine calcium channel block­
physiologist or an i n lecrious d i sease spec i a l i s t is manda to ry.
ers, have not been shown to delay surgery in asymptomatic
Pacem<1ker pocket aspi ra t i on s h o u lcl never be per
patients without hypertension. There is no established
lormed . as it can seed a steri le pocket a n d lead to i n fect i o n .
benefit in medical therapy for this patient with moderate
espec i a l ly i f . t here i s superAci<l l cel l u l i t i s w i t hout deeper
aortic regurgitation without other specific indications for
t i ssue i nvolveme n t .
treatment.
U l t rns o n ogra p h i c ex a m i n a t io n or <I pace m a ker o r
d e fi b r i l l a t o r pocket mJy be h e l p f'ul in p a t i e n t s w i t h sus ­ KEY POINT
pected i rn p Lrnted card i::ic device i n lect ion s. I Jowevcr. pocket • Asymptomatic patients with moderate aortic regurgi­
f l u i d may not a lways represe nt i n lect i o n . <ll1cl steri l e sero mas tation should be evaluated on a yearly basis and have
a re someti mes en cou n tered . Tl1erefore. pocket u l t rnsou n d echocardiography performed every 1 to 2 years.
h < l S I i m i ted - i r a ny -d iagnost ic va l ue.
::ven
1 t h ough t h e prese n t i ng sy m pt o m s or a dev i ce ­
Bibliography
re l a ted i n le c t i o n may be n o n spec i l i c a nd d ifficult to
Helms AS, Bach DS. Heart valve disease. Prim Care. 2013 Mar:40(t) : 9 1 - 108.
d i s t i ng u i s h rro m ot her com m o n . ben ign i n le c t i o n s . d e l ayed [PMID: 23402463]

1 85
Answers and Crit i q u es

Item 60 Answer: A emergency PCI. ·n1 rombolytic t hernpy l�1 i l ure. w h ich occurs
in up to 30% or patienrs. remains d i l f ic u l t to diagnose. Chest
Educational Objective: Manage anticoagulation therapy
pain resol u t i on . ST- segme n t elevat ion i m p roveme n t . a n d
in a pregnant woman with a mechanical valve prosthesis.
reperrusion a rrhyt h m ias (most co m mo n ly a n acce lerated
The anticoagulation regimen that will provide the greatest i d i ove n t ri c u l a r rhy t h m ) i n d icate success fu l L h rombo ly ­
protection against thromboembolism in this patient is warfa­ sis. A l t hough complete ST- segment elev•ll i o n resol u t ion is
rin therapy. Low-dose aspitin therapy should also be contin­ associated w i t h coron<iry p<itency. il occurs in a m i nority
ued. Women with mechanical valve prostheses cany a high o f P•l l ien ts. I m provement i n ST- segment elevation greater
tisk of valve thrombosis, bleeding, and fetal morbidity and t h a n SO% on a n electrocardiogram ( ECG) obta i ned 60 m i n
mortality duting pregnancy, and the optimal anticoagulation utes alter t h e :.id rn i n is l ra t ion or t h romboly t ic t herapy i s t h e
strategy has not been established. Options i nclude warfa1in , most com mo n ly used criterion l o ind icate successfu l reper
unfractionated heparin (UFH) , a n d low-molecular-weight fusio n. Cont i nued chest pain . lack of i m p rovement in ST-seg
heparin (LMWH). Although warfarin poses an increased risk menl eleva t i o n . hemodyna m i c i nstabi l i ty. a n d t he absence
of teratogenicily and fetal loss, it appears to be the most effec­ o r reperfusion a rrhy t h m ias most l i kely i n d icate l�1 i l ure or
tive option for reducing thromboembolism tisk in the mother. L h romboly t i c t herapy and ind icate the need !or rescue PC I .
1he current dose ofwarfatin (4 mg/cl), used to achieve a thera­ 1 h is pa t ient has clear evidence or t a i led reperfusion o r reoc
peutic INR, is associated vvith a low risk of warfarin embryop­ clusion (worseni ng or ST - seg me n t eleva t i o n . persistence or
athy and a low risk of fetal complications. sympto ms) and now has signs o f cardiogenic shock ( low
TI1e novel oral anticoagu lants, such as dabigatran, blood pressure. p u l monary edema ) . I n pat ients with t h rom ­
bivalirudin, rivaroxaban, and apixaban, do not adequately bolyl ic l herapy f a i l u re. guideli nes recom mend i m mediate
protect patients with mechanical valve prostheses against t ransfer tor rescue P C I . In m u l t iple t ri a ls or t h rornbolytic
thromboembolism and should not be used in pregnant or t he rapy fa i l u re. patients who underwent rescue PCI had a
nonpregnant patients with mechanical valve prostheses. sign i ficant i m p rovement in t h e rate or rei n f a rction when
I ntravenous UFH is the anticoagulant treatment of co mpared with t hose receiving conservat ive care. b u l no
choice around the time of del ivery. Intravenous UFH can i m p rovement i n mort a l i ty.
also be used during the first trimester. TI1e dose effect must 'n1e use or g lycopro t e in l i b I l la i n h i bi tors has been
be measured by activated partial thromboplastin time and tested i n m u l ti p l e scenarios i n pa t ients w i t h STE M ! . Based on
the dose adjusted to a therapeutic level. Fixed-dose subcuta­ t hese stud ies. t he i r use has been l i m i ted ow i ng lo excessive
neous UFH may not provide adequate anticoagulation. bleed i ng events. In pat ients i n whom t h romboly t i c t herapy
LMWH can be used as an anticoagulant during preg­ has !ai led. rescue PCI w i t hout the use of a glycoprolein I l b
nancy, but for patients with a mechanical valve prosthesis, Illa in h i bitor o r addi t i onal L h romboly t i c agents i s prelerred.
a weight-based regimen has been demonstrated to be inad­ A meta -a nalysis p u b l i shed i n 2007 co mpared repeat
equate. The LMWH dose must be adjusted to anti -factor Xa t h rombolytic t herapy w i t h con ervative t h e rapy in pa t i ents
activity in order to provide adequate anticoagulation. i n whom i n i t ia l t h rombolyt ic t he rapy fa i l ed. T h is analysis
KEY POINT showed no sign i ficant difference i n mort a l i ty rates or re i n ­
t a rc t ion between the two groups, a n d ou tcomes in t h ese
• Anticoagulation strategies for pregnant women with a
groups were i n ferior to rescue PC! .
mechanical valve prosthesis include warfarin, dose­
adjusted unfractionated heparin, and dose-adjusted K EY P O I N T
low-molecular-weight heparin; of these options, war­ • Patients with thrombolytic therapy failure following
farin poses a lesser risk of maternal thromboembo­ an ST-elevation myocardial infarction should be
lism but a greater risk of fetal embryopathy. immedia tely transferred for rescue percutaneous cor­
onary intervention.
Bibliography
Nishimura RA, Otto CM. Bonow RO. et al: American College of Cardiology/ Bibliography
American Heart Association Task Force on Practice Guidelines. 2014
Sutton AG. Campbell PG. Graham R, et al. A randomized trial of rescue
AHA/ACC guideline for the management of patients with valvular heart
angioplasty versus a conservative approach for failed fibrinolysis in
disease: executive summa1y: a report of the American College of
ST-segment elevation myocardial infarction: the Middlesbrough Early
Cardiology/A merica n Heart Association Task Force on Practice
Guidelines. J Am Coll Cardiol.2014 Jun 10;63(22):2438-88. Erratum in: J
Revascu la rizat i on to Limit lNfarction (MERLI N) trial. J Am Coll Cardiol.
2004 Jul 21 :44(2):287-96. [PM ID: 152619201
Am Coll Cardiol. 2014 Jun 10:63(22):2489. [ PMID: 24603192]

Cl Item 61 Answer: D
Educational Objective: Treat a patient with thrombo­
Item 62 Answer:
Educational Objective: Screen patients with a family
c

lytic failure following an ST-elevation myocardial infarction. history of hypertrophic cardiomyopathy for the disease at
appropriate intervals.
1 l1 is patient w i t h ST-elevation myocard ial in f a rction (STEM I )
should b e t nrn s ferrecl to t h e nearest hospital \Ni t h p r i m a ry TI1 is patient should again be screened for hypertro­
percutaneous coronary i n tervent ion ( PC I ) capabi l i t ies !or phic cardiomyopathy (HCM) i n 5 years. All first-degree

1 86
Answers and Criti q u es

relatives of patients with HCM should undergo screening Item 63


Cl
Answer: c
for the disorder with a comprehensive physical exam­
Educational Objective: Treat atrioventricular block
ination, electrocardiogram, and echocardiogram. Because
complicating acute myocardial infarction.
HCM can manifest at any age, lifetime screening offtrst-de­
gree relatives in whom the disorder has not yet been ·1 h i s p a t i e n t has evidence o f' ::in i n ferio r- posterior ST­
diagnosed is indicated. l11e recommended HCM screening eleva t i o n myocard i a l i n E:irction �rnd shou l d u n dergo u rgen t
intervals, which are based on clinical suspicion, patient percutaneous coronary i n terve n t i o n ( PC I ) . H e is braclyca rdic
age, family history, and participation in competitive ath­ with Mobitz type 1 second -degree heart block, also known
letics, are shown. as Wenckeb,ich block. T h is type or <l l rioven t ricular block is
a l most always w i t h i n L he compact at rioven t ricu l a r node (a ncl
not i n l'r::i 1 l isia n ) a nd in t h i s pa t ient is l i kely c<J usecl by right
Recommended Hypertrophic Cardiomyopathy coronary <l rtery occlusion . · 1 11e righl corona ry a rtery suppl ies
Screening Intervals the a t rioven tricu lar nodal a rtery in 90% o r patients. ·1 11e most
Age Group Recommendation i m port a n t i n terve n t ion tor t h is patient is urgent PC! a nd
reperf'u sion or t h e i n l�irct- related a rt e ry. Alt hough t he pres­
< 1 2 years Screening optional except i n the
followin g situations: ( 1 ) presence of ence or al riovenl ricul<i r block is usua l ly t ra nsient an cl resolves
symptoms; (2) family history of w i t h reperf'usion . it is associated w i t h worse prognosis and
malignant ventricu l a r tachyarrhythmias;
i n - hosp i t;i l survival.
(3) patient is com petitive athl ete i n a n
intense training program; or (4 ) A m i nophy l l i ne i ncre;1ses cyc l i c <lclenosine monophos­
presence of othe; clinical suspicion phate (cA M P) a nd cci n be used to promote a t riove n l ri c u l a r
of early left ventricula; hypertrophy con d u c t i o n i n pa t ients w i t h hemoclyn a m i c a l ly u nstable bra ­
1 2 to 1 8-21 years Every 1 2 to 1 8 months clyca rd i <l or aclvancecl a t rioven t r i c u l a r block ( M o b i t z type 2
second degree or t h i rd -degree a l riove n l ricu l a r block) d ue t o
> 1 8-2 1 years At symptom onset or at least every
5 years ( more frequently in fa m i l ies coro n a ry ischem i a . T h is pat i e n t is hemocly n a rn ica l ly stable
with m a l ig nant tachyarrhythmias or and h is a t r i oven t ricu l a r block is not aclva ncecl ; t herel'ore.
late onset)
a m i nophyl l i ne is not i n d ica ted .
Because l he patient is not expe r i e n c i ng hemocly n a rn i c
seq uel ae, l o w close dopa m i ne i n rusion is not i n d icated. I f

l11ese recommendations are for relatives of patients t h e p a t i e n t develops he mocly n a m i c a l ly signi fica n t brady­

with HCM in whom genetic testing is negative, inconclusive, c a rd ia . dop<1 111 i n e i n fusion cou l d be used to stabi l i ze h i m

or not performed. Genetic testing of probands can be used u n t i l coro n a ry reperfusion a n d tem porary paci n g cou l d be

to identify pathologic mutations, which can then be used acco mpl ished .

to screen family members and, if negative, may obviate the Aclva ncecl a l riove n t ricu l a r block i n t h e se t t i ng o f' a n

need for continued imaging. l11e yield of genetic testing, acute coronary syndrome of'Len req u i res tempora ry or per­

which can be costly, varies according to the phenotypic m a n e n t paci ng. I n t h is patient. tem porary paci ng is not i ncl i ­

expression and familial nature of HCM. Thus, referral to a cated because he is hemody n a m i ca l ly stable a n d h i s block

cardiovascular specialist or a genetic counselor is recom­ is not advanced . Decisions o n perm a ne n t paci ng should be

mended for clinical decision-making based on genetic test­ de layed u n t i l a patient has been revascu la rized a n d st<lb i l i zecl

ing in patients with HCM. to deter m i n e whether the a rr hy t h m i a persists.

KEY POINT KEY POINT

• All first-degree relatives of patients with hypertrophic • Patients with acute coronary syndrome and related
cardiomyopathy should u ndergo screening for the Mobitz type 1 second-degree atrioventricular block ·
disorder with a comprehensive physical examination, should undergo urgent reperfusion therapy as the
electrocardiogram, and echocardiogram; lifetime treatment of choice for this conduction block.
screening of those in whom the disorder has not yet
been diagnosed is indicated. Bibliography
Epstein AE, DiMarco JP, Ellenbogen KA, et al; American College of
Cardiology/American Heart Association Task Force on Practice
Bibliography Guidelines (Writing Committee to Revise the ACC/AHA/NASPE 2002
Gersh BJ, Maron. BJ, Bonow. RO, et al; American College of Cardiology Guideline Update for Implantation of Cardiac Pacemakers and
foundation/ American Heart Association Task Force on Practice Antiarrhythmia Devices): American Association for Thoracic Surgery:
Guidelines. 2011 ACCF/AHA Guideline for the Diagnosis and Treatment Society of Thoracic Surgeons. ACC/AHA/HRS 2008 Guidelines for
of Hypertrophic Cardiomyopathy: a report of the American College of Device-Based Therapy of Cardiac Rhythm Abnormalities: a report of the
Cardiology Foundation/American Heart Association Task Force on American College of Cardiology/American Heart Association Task Force
Practice Guidelines. Developed in collaboration with the American on Practice Guidelines (Writing Committee to Revise the ACC/AHA/
Association for Thoracic Surgery, American Society of Echocardiography, NASPE 2002 Guideline Update for Implantation of Cardiac Pacemakers
American Society of Nuclear Cardiology. Hearl Failure Society of and Antiarrhythmia Devices) : developed in collaboration with the
America. Heart Rhythm Society, Society for Cardiovascular Angiography American Association for Thoracic Surgery and Society of Thoracic
and I nterventions, and Society of Thoracic Surgeons. J Am Coll Cardiol. Surgeons. Circulation. 2008 May 27;117(2l) :e350-408. Erratum in:
2011 Dec l3;58(25):e212-60. [PMID: 22075469] Circulation. 2009 Aug 4: 120(5):e34-5. [PMID: 18483207]

1 87
Answers and Critiques
-'--
�� � ����������
� ��

death in patients with diabetes mellitus is cardiovascular


Cl
Item 64 Answer: A
disease, but routine testing for coronary artery disease (CAD)
Educational Objective: Manage intravenous lines in a
in asymptomatic patients with diabetes does not reduce mor­
patient with cyanotic congenital heart di seas e .
tality. Aggressive treatment of cardiovascular risk factors,
In t h i s pat ient w i t h Eisen menger syndrome. a i r fi l ters on however, does improve outcomes and reduce mortality as
i n t rnvenous l i n es are recom mended to reduce the risk of para ­ seen in the Steno-2 study. In this study, intensive interven­
doxical a i r embol ism. She h<lS cli nical l ea tu res of pyeloneph ri tion with behavior modification and multiple pharrnacologic
tis. and i n t ravenous a n t i b i o t ic t herapy is warranted. Patients interventions aimed at achieving hemoglobin A 1e levels below
with Eisen menger syndrome have an i n t racardiac right-to- left 6.5%, blood pressure below 130/80 mm Hg, and serum total
s h u n t . and t herelore are a t risk !or paradm:ical air embol ism . cholesterol levels below 175 mg/dL (4.53 mmol/L) resulted in
These A l ters can be easily and rapidly applied to any i n t ra­ a 53% reduction of cardiovascular disease in a nearly 8-year
venous l i ne prior LO a d m i n istering i n t ravenous t herapy and follow-up. Continued risk factor management in this patient
l i ke ly reduce t he risk of a i r embol i sm . Meticu lous care of i n t ra ­ is, therefore, the most appropriate choice.
venous l i nes and cat heters is very i m portant i n pal ients vv i t h This patient does not need an echocardiogram. He is
Ei senmenger syndrome owing lo t he poten t i a l for paradox ical asymptomatic, and the murmur described is consistent with
embol is m . which may cause st roke and deb i l i ty. mild aortic stenosis as supported by his echocardiogram
ll1ere a re no data to support i m p roved survival w i t h 1 year ago. He should undergo an annual clinical evaluation
chronic oxygen t herapy ! o r patients w i t h Eisen menger syn ­ and echocardiography every 3 to 5 years. Echocardiography
drome and assoc i a ted C)"<lllOsis. Sa t u ration may i ncrease m i n i ­ at this time in the absence of a clinical change is unnecessary.
ma lly w i t h oxygen t herapy. b u t i t genera l ly does not nonrn1 l ize. lf a screening test were to be performed prior to exercise,
l h e hemoglob i n a n d hemalocrit levels in t h i s patient an exercise stress test would be the most appropriate test;
are p hy s i ol ogic fOr a P<l t i en L with Ei sen menger syndrome. exercise perfusion imaging provides no additional informa­
Ph lebotomy is n o t reco m mended . tion. ln routine screening of patients with diabetes in the
A t ra ns t h o racic echocardiogram is not reco m m e nded DIAD study, despite 22% of patients having evidence of per­
al t h i s l i me because i l is u n l i kely lo c h a nge management in fusion detects on single-photon emission CT, most of which
t h i s p a l i e n l who has known heart d isease . ·111e p a t ie n t has were small, mortality rates were not changed compared with
f e a t u res or i n fect io n . but the data suggest a u r i n a ry tract patients who did not undergo screening. ll1e event rates were
i n lect ion ra t her t ha n endoca rd i t i s. Ir the patient presented low in both groups, at about 3% over nearly 5 years.
\1· i t h or develops fea t u res of end oca rd i t is . such as a new The 2012 U.S. Preventive Services Task Force statement
m u rm u r. sep t i c embo l i . or persisten t bactcre m i a . a t rn nse­ on screening for CAD with electrocardiography (ECG) rec­
sop hageal echocardiogram vvo u l d be favored over <1 l ra ns­ ommended against screening with resting or exercise ECG
t h o racic echocardiogram . for the prediction of CAD events in asymptomatic adults
K EY P O I NT at low risk for CAD events, and stated that the evidence is
insufficient to assess the balance of benefits and harms of
• In patients with Eisenmenger syndrome, air filters screening in asymptomatic adults at intermediate or high
and meticulous care of all intravenous lines should be risk for CAD events. ll1e 2002 American College of Cardi­
instituted to prevent paradoxical air embolism. ology/ American Heart Association (ACC/AHA) guidelines
also concluded that there is no evidence to support routine
Bibliography testing in asymptomatic adults but concluded that it is rea­
Warnes CA. Williams RG. Bashore TM, et al; American College of Cardiology; sonable to screen for CAD in asymptomatic patients with
American Heart Association Task Force on Practice Guidelines (Writing
Committee to Develop Guidelines on the Management of Adults With diabetes who plan to begin a vigorous exercise program.
Congenital Heart Disease) ; American Society of Echocardiography; Heart
Rhythm Society; International Society for Adult Congenital Heart Disease; KEY POINT
Society for Cardiovascular Angiography and Interventions; Society of
Thoracic Surgeons. ACC/AHA 2008 guidelines for the management • Routine screening for coronary artery disease in
of adults with congenital heart disease: a report of the American College of asymptomatic patients with diabetes mellitus does
Cardiology/American Heart Association Task Force on Practice Guidelines
(Writing Committee to Develop Guidelines on the Management of Adults not reduce mortality.
With Congenital Heart Disease). Developed in Collaboration With the
American Society ofEchocardiography, Heart Rhythm Society, International
Society for Adult Congenital Heart Disease, Society for Cardiovascular Bibliography
Angiography and Interventions, and Society of Thoracic Surgeons. J Am Gaede P. Lund-Andersen H. Parving H H , Pedersen 0. Effect ofa multifacto­
Coll Cardiol. 2008 Dec 2:52(23):el43-263. [PMID: 19038677] rial intervention on mortality in type 2 diabetes. N Engl J Med. 2008 Feb
7:358(6):580-91. [PMID: 18256393]

Item 65 Answer: D

Cl
Educational Objective: Manage coronary artery disease Item 66 Answer: D
risk in an asymptomatic patient with diabetes mellitus. Educational Objective : Manage acute limb ischemia.

This patient should continue his current therapy; no addi­ ll1c most appropriate ma nagem ent !or t his pa t i ent with an
tional testing is indicated at this time. The leading cause of ische m ic but viable e x t re m i ty (severe acute l i mb ische m i a )

1 88
Answers a n d Critiques

c::J is u rgent a ngiograp hy to defi ne t he anatomic level o f oc lu c to moderate LY dysfunction (ejection fraction of 30%-60%
s 1 o n and assess appropriate treat ment options. wh ich may and/or LY end-systolic dimension ;:::40 mm). Mitral valve repair
CONT
include su rgica l or percutaneous revascularizal ion or t hrom is the operation of choice when the valve is suitable for repair
bolytic therapy in se l ected pat ients. He has several risk factors and appropriate surgical skill is available and is recommended
lor at herosclerotic peripheral arterial d isease ( P A D) . a nd the for most patients. Mitra] valve replacement, especially with
claudicat ion t hat he has experienced for t he past year has chordal preservation, is appropriate for patients with severe
progressed to severe rest ing l i mb pain. ll1e limb i s v ia bl e as mitral regurgitation in whom the valve is not repairable or a less
indicated by t he presence or pa in . slow but present capi l la ry than optimal result would be obtained. Recently, a percutane­
refi l l. and t he presence o f Dopple r vascular s ig nals. Acute
'
ously placed mitral valve clip has been introduced that is placed
ischemia can be caused by remote embol izat ion but may also to better approximate the edges of the anterior and posterior
resul t from in-situ t h rombosis. Because of' t h is. ant icoagu la­ leaflets of the valve and may be a therapeutic option in patients
t ion is crucial once a d i agn o s is of acute a rterial occlusion has who are at a prohibitive risk for m.itral valve surgery.
been made by h i s tory and physica l exam ination. The next step Yasodilator therapy, such as with ACE inhibitors or
in management is to fu r t her eva l uate t he l i mb ischemia and angiotensin receptor blockers, has not been shown to
plan fOr t reatment. Digi tal subt raction angiogra p hy provides improve outcomes in patients with severe mitral regurgita­
the most hel p fu l i n formation and is t he p re f erred i maging tion who are asymptomatic. Additionally, vasodilator ther­
modal i ty for acute l i m b i sc h e m i a : d e l ayi ng angiography could apy may mask the development of more severe left ventric­
lead to l i m b necrosis a nd loss of' limb funct ion ing. ular dysfunction due to regurgitant volume. Therefore, these
Ca t he te r- d i re ct e d t h ro m bo ly t i c therapy may be a n agents should not be used as a substitute therapy for surgery
option i n some p a t i e nt s with acute l i m b ischem ia w i t h a when the patient is thought to be a surgical candidate.
viable o r ma rg i na l ly t h reatened l i m b as an a l ternat ive to a Mitra! balloon valvuloplasty or valvotomy is indicated
surgical approach . part icularly i r t he duration or acute l i m b for patients with severe mitral stenosis in whom there is a
ischem ia is less t ha n I day. However. ini t i a t i ng t h ro mbo reasonable likelihood of success and in whom there are no
l y l i c t he ra py in t h is patient before furt her evaluat ion of' t h e contraindications (such as moderate to severe mitral regurgi­
nature o f the occ l u s i o n wou l d not be appropria te. tation or left atrial thrombus) . TI1is patient has severe mitral
for a nonviable extrem i ty. su rg ica l a m putation wit hout regurgitation, and repair, not valvotomy, is indicated.
a ngiography is i n clica t ecl because or t h e i ncreased risk or Serial echocardiography may be helpful in follow-up of
t issue necrosis and i n fect i o n . I l owevc r. t h is pat ien t's loot the asymptomatic patient in whom worsening of LY systolic
shows evidence of' viabi l i ty. mak i ng i m med i,1te amputation function or increase in chamber size may help facilitate deci­
i na pp ro p r i a t e
. sion for surgery. This patient's LY function is compromised
Warfarin has not been shown to be an ef lecl ive t herapy and intervention is indicated.
lor managing stable Pr\ D. and a l t hough a n ticoagu lat ion is
K EV P O I N T
ind icated in ma naging acute l i mb ischemia pend ing fur­
t he r eva l ua t i o n . i n i tiat ion of' long-term ant icoagulation with • Mitra! valve repair is the operation of choice for severe
wa rfarin i n t h is p<1 l iem with a viable but t h reatened l i m b mitral regurgitation when the valve is suitable for
w i t hout fu rt her intervention wou l d not b e a p pro pria te .
repair.

KEV POINT
Bibliography
• Patients with an ischemic but viable extremity on clin­ Nishimura RA, Oito CM, Bonow RO. et al: American College of Cardiology/
ical examination should undergo urgent angiography American Heart Association Task Force on Practice Guidelines. 2014
AHA/ACC guideline for the management of patients with valvular heart
to plan surgical or percutaneous revascularization. disease: executive summary: a report of Ihe American College of
Cardiology/American Hean Association Task Force on Practice
Guidelines. J Am Coll Cardiol. 2014 Jun 10;63(22):2438-88. Erratum in: J
Bibliography Am Coll Cardiol. 2014 Jun 10:63(22):2489. [PMID: 24603192)
Creager MA. Kaufman JA. Conte MS. Clinical practice. Acute limb ischemia.
N Engl J Med. 2012 Jun 7:366(23):2198-206. [PMID: 2267090S l

Item 68 Answer: c
Item 6 7 Answer: D Educational Objective: Manage Brugada syndrome.
Educational Objective: Manage asymptomatic severe
This patient should undergo placement of an implantable
mitral regurgitation with reduced left ventricular function.
cardioverter-defibrillator (!CD). He had an episode of abrupt
Mitra] valve repair is the most appropriate option for this patient syncope that is concerning for a cardiac etiology, specifically
with asymptomatic severe mitral regurgitation and moder­ an arrhythmia. He has a structurally normal heart on echo­
ate left ventricular (LY) dysfunction. Surgery is indicated for cardiogram, but his electrocardiogram (ECG) shows right
patients with symptomatic acute severe mitral regurgitation, bundle branch block and 2-rnm ST-segment elevation in the
those with symptomatic chronic severe mitral regurgitation precordial leads. These findings are consistent with a type 1
with LY ejection fraction greater than 30%, and asymptom­ Brugada pattern (coved or descendant ST-segment elevation
atic patients with chronic severe mitral regurgitation and mild followed by negative T waves) on ECG. The presence of a type

1 89
Answers and Crit i q u es

1 Brugada pattern and symptoms (cardiac syncope) or vent1ic­ pitalizations, poor kidney function, diuretic dependence to
ular arrhythmia is diagnostic for Brugada syndrome. Brugada maintain fluid balance, and hypotension. Tiie two possible
syndrome can be genetically heterogeneous, but it is often options for therapy in a patient with this degree of heart
caused by mutations in the SCNSa sodium channel, which failure are placement of an LVAD and heart transplantation.
are believed to cause alterations in the ventricular refracto1y Because of his diagnosis of disseminated prostate cancer,
period and are responsible for the characteristic ECG findings however, the patient is not a candidate for transplantation.
and predisposition to sudden cardiac death. LVADs are indicated either as a b1idge to hea11 transplantation
Because this patient with Brugada syndrome is at risk or as destination therapy in selected patients who are not can­
for sudden cardiac death, particularly given his recurrent didates for transplantation. Newer LVAD devices are smaller
episodes of near-syncope and syncope, an !CD should be and easier to maintain than earlier versions. making their
implanted. Patients with precordial ST-segment abnormal­ long-tenn use as destination therapy possible. Although this
ities should be referred to a cardiologist or electrophysiolo­ patient might otherwise be a candidate for transplantation,
gist. Once Brugada syndrome is diagnosed, first-degree fam­ his diagnosis of disseminated prostate cancer is an absolute
ily members should be referred to an inherited arrhythmia contraindication because of the required long- term post­
clinic (electrophysiology clinic specializing in genetic disor­ transplant immunosuppression. However, placement of an
ders) for counseling and screening. Patients who have a Bru­ LVAD would be an appropriate consideration in this patient if
gada pattern but are asymptomatic often do not require !CD he is expected to survive for longer than 1 year.
placement. Tiie incidence of B rugada syndrome is higher in Other contraindications to cardiac transplantation
patients of Asian ethnicity. include medical problems associated with a reduced life
Cardiac magnetic resonance (CMR) imaging would not expectancy (rheumatologic disease, severe pulrnona1y dis­
be helpful given the nonnal echocardiogram and diagnosis ease, liver failure) , fixed severe pulmona1y hypertension,
of Bmgada syndrome. CMR imaging would be helpful if diabetes mellitus with end-organ manifestations, age greater
occult structural heart disease was suspected, such as car­ than 65 to 70 years, severe pe1ipheral arterial or cerebro­
diac sarcoid. amyloidosis, or arrhythmogenic right vent1ic­ vascular disease, and advanced kidney disease. Although
ular dysplasia. several of these factors are also associated with poorer out­
An exercise treadmill stress test can be valuable for comes with LVAD use (such as advanced age and degree
identifying an exercise-induced arrhythmia. Brugada syn­ of comorbid disease), assist devices are a viable option for
drome often presents with nocturnal arrhythmias and is treatment in patients who are clearly not candidates for
usually not adrenergically driven. A stress test would not aid transplantation.
in this patient's diagnosis. Metolazone inhibits sodium reabsorption in the distal
Tilt-table testing should be reserved for patients with tubule and may be particularly effective in inducing diuresis
recurrent syncope without known heart disease or those when used in combination with a loop diuretic in patients
with heart disease in whom a cardiac cause of the syncope with volume overload who have not responded adequately
has been excluded. Tilt-table testing may also be helpful in to high doses of a loop diuretic. However, this patient does
evaluating patients in whom documenting neurocardiogenic not have signs of volume overload (no jugular venous dis­
syncope is important (such as in high-risk occupational tention or edema) and therefore would not be expected
settings) . In this patient, the ECG is diagnostic for Brugada to benefit from the addition of metolazone to his current
syndrome and tilt-table testing is not needed. regimen.
Home inotropic therapy is associated with a mortality
KEY POINT
rate of approximately 90% at 1 year and should be consid­
• The presence of 2-mm or more coved precordial ered as a palliative care option only. Use of this therapy is
ST-segment elevation (leads V1 through V ) on electro­ associated with worsening hea11: failure, infection, and
cardiogram and symptoms (cardiac syncope) or ven­ arrhythmias. In a patient who is a candidate for either
tricular arrhytlunia indicates the presence of Brugada LVAD or heart transplantation, this should not be con­
syndrome. sidered as an alternative therapy. Occasionally, patients
require supportive inotropic therapy until t hey receive a
Bibliography transplant. Tiiis should be managed by their transplant
Mizusawa Y. Wilde AA. Brugada syndrome. Circ Arrylhm Electrophysiol. cardiologist.
2012 Jun 1 :5(3):606- 16. [PM!D: 22715240]
K EY P O I N T
• Placement of a left ventricular assist device is an
Item 69 Answer: c
option for patients with end-stage heart failure who
Educational Objective: Manage end-stage heart failme are not candidates for hea11: transplantation.
with a left ventricular assist device.

This patient should be evaluated for placement of a left ven­ Bibliography


Garbade J, Barten MJ. Bittner HB. Mohr FW. Heart transplantation and lefl
t1icular assist device (LVAD) . He has end-stage heait failure
vent1icular assist device therapy: two comparable options in end-stage
manifested by ell.1:reme limitations of activity. multiple hos- heart failure? Clin Cardiol. 2013 Jul;36(7):378-82. [PM!D: 23595910]

1 90
An swers and Criti q u es

Item 70 Answer: D end-organ ischemia. persistent severe hypertension or pain.


Educational Objective: Prevent stroke in a patient with propagation of the dissection (which may be manifested by
atrial fibrillation and chronic kidney disease. persistent or recurrent pai n) . a neurysmal expansion. and
rupture. This patient has new acute kidney failure because
TI1is patient with atrial fibrillation should be prescribed the dissection occludes t he renal arteries. Emergency su rgery
dose-adjusted warfarin for stroke prevention, with a goal lNR is recommended for acute type A a nd complicated type B
of 2 to 3. Patients with atrial fibrillation and kidney disease dissect ions: initial medical management is recommended for
are at increased risk for stroke and are also at increased risk uncomplicated type B dissections. Because this patient has a
for bleeding events with oral anticoagulation. In addition to complicated type B dissection. medical management alone
her kidney disease, the patient has three CHA2 DS 2 -VASc risk would not be appropriate.
factors for stroke, including her female sex (1 point), age (2 Because pa tients with aortic dissection have a s ign i fi ­
points), and the presence of hypertension (1 point). Based cant prevalence of coronary artery d isease. coronary angiog­
upon her risk profile (CHA2 DS 2 -VASc score = 4) , her adjusted raphy is somet i mes performed prior to surgical intervention
risk of stroke is 4% per year. The presence of end-stage kidney to allow for repa i r prior to or at the t ime of operation for
disease is also associated with an increased risk of stroke. treat ment of t he aort ic dissection. However. the benefit of
Accordingly, she is at moderate to high risk of stroke and t h is approach has not been establ ished, and the requ ired
requires oral anticoagulation. contrast adm i nistration would be potentially harm ful in
In the past several years, several novel oral anticoagu­ t h is pat ient with acute kidney failure caused by her aortic
lants have been approved for stroke prevention in patients dissection . It would therefore not be an appropriate next step
with nonvalvular atrial fibrillation. Dabigatran is a direct in the t reat ment of' this patient.
thrombin inhibitor that is dosed twice daily. Rivaroxaban is Although some variants of ao11ic dissection are associ­
a factor Xa inhibitor dosed once daily. Apixaban is a factor ated with i n t ramural th rombus. antithrombotic or t h rombo­
Xa inhibitor that is dosed twice daily. All three agents exhibit lyt ic t herapy is contraindicated a nd may be poten tially fatal
partial clearance in the kidneys and are contraindicated in in pa t ients with aortic dissection . ll1is h ighlights the need
patients with end-stage kidney disease. None of the ran­ to exclude aort ic dissect ion prior to ini tiating e ither t herapy,
domized controlled trials evaluating these three new agents which may be indicated in patients with evidence of cardiac
included patients with creatinine clearance less than 30 ml/ ischem ia.
min/1 .73 m 2 .
KEY POINT
Aspirin and clopidogrel (dual antiplatelet therapy) can
be considered an alternative therapy for patients who cannot • Complicated type B aortic dissection is an indication
tolerate oral anticoagulation; however, they provide infe­ for immediate aortic repair.
rior stroke prevention with similar rates of bleeding when
compared with dose-adjusted warfarin. The best option for Bibliography
stroke prevention therapy in this patient with atrial fibrilla­ Braverman AC. Acute aortic dissection: clinician update. Circulation. 2010
Jul 13; 122(2):184-8. [PMID: 20625143)
tion and end-stage kidney disease at high risk for stroke is
dose-adjusted warfarin.

KEY POINT Item 7 2 Answer: C


• The best option for stroke prevention therapy in a Educational Objective: Treat severe asymptomatic pul­
patient with atrial fibrillation and end-stage kidney dis­ monary valve stenosis.
ease at high risk for stroke is dose-adjusted warfarin.
This patient should undergo replacement of the pulmonary
valve. She has Noonan syndrome, which is an autosomal
Bibliography dominant disorder commonly associated with congenital car­
Steinberg BA, Beckley PD. Deering TF, et al; Society of Cardiovascular
Patient Care. Evaluation and management of the atrial fibrillation
diac lesions, one of which is pulmonary valve stenosis. The
patient: a report from the Society of Cardiovascular Patient Care. Crit pulmonary valve is usually dysplastic in patients with Noonan
Pathw Cardiol. 2013 Sep;l2(3) : 1 07- 15 [PMID: 23892939) syndrome and thus is not amenable to balloon intervention.
This patient has severe pulmonary valve stenosis demon­
strated by physical examination; features include a late­
Cl Educational Objective:
Item 7 1 Answer: A
peal<lng palpable systolic ejection murmur located at the sec­
Manage complicated type B aor­
ond left intercostal space, absence of an ejection click, and fea­
tic dissection.
tures of right ventricular pressure overload. An ejection click
The most appropriate management for t h is patient is emer­ may be audible in patients with pulmonary valve stenosis, but
gency aortic repair. She has a complicated type B aortic dis­ as the severity progresses, the click disappears owing to loss of
section. Type A dissections or i ntramural hematomas i nvolve valve pliability. TI1is patient's echocardiogram confirms severe
t he ascending aorta or aortic arch, whereas type B syndromes pulmonary valve stenosis with a peak systolic gradient of
begin distal to the left subclavian artery. Complicated dissec­ 62 mm Hg and mean systolic gradient of 45 mm Hg. Inter­
tions include occlusion of' a major aortic branch leadi ng to vention is recommended owing to the severity of obstruction

1 91
Answers a n d Criti ques

despite the absence of symptoms. Surgical valve replacement i n i t ial ECG. A l though an early i nvasive strategy (defi ned as
is recommended given the dysplastic valve features and pres­ w i t h i n 2-1 hours o f hospital adm ission) has been proved lo
ence of coexisting moderate pulmonary valve regurgitation. be efTect ive in t reatment o f NSTE M I . t here is no evidence
Most patients with severe pulmonary valve stenosis are treated t hat earlier angiography (<6 hours or at hospi tal admission)
with balloon valvuloplasty, but this patient has a dysplastic ofTers incremental beneAt lo these patients.
valve related to Noonan syndrome and moderate pulmonary J n pat ients with a low TI M I risk score ( 0- 2) . i n d ica t i ng
valve regurgitation; thus, pulmonary valve replacement is a low i n - h o sp ital risk of de<1 t h or recurrent ischem ia 'in farc­
recommended. tion. predischarge st ress testing may be warran ted to further
Patients with pulmonary valve stenosis without a his­ define a large ischemic burden and guide revascul a rizat ion
tory of endocarditis or pulmonary valve replacement do not decisions. ll1 is patient has a T I M I risk score o r 4 (:2:3 t ra­
require endocarditis prophylaxis. d i t ional card i ovascu lar risk factors. ST-segment de\·i a t ion .
Exercise testing could help determine whether the d a i ly aspirin use. elevated cardiac b iomarkers) . p laci ng h i m
patient has exercise limitation related to her pulmonary a t i ntermediate risk. 'l l1ese patients have i mproved c l i nical

valve stenosis but would not change the recommendation for outco mes w i t h an early i nvasive strategy. Exercise stress
intervention; thus, it is not required. test ing is not appropriate and may b e da nge rous .

Clinical observation, with continued participation in Cl i n ical decisio n - making should not be affected by t h e
competitive sports, is not advised in patients with severe resu lts of t he second set o f card iac b i o m arkers ·n1e pres­.

pulmonary valve stenosis regardless of symptom status ence of an elevated t ropo n i n level drawn i n t he emergency
because of the risk of progressive right heart failure. department is p rognost i cal ly sign i ficant and warrants hospi ­
tal adm ission . t rea t ment of NSTE M I . and risk stra l i ficat i on .

KEY POINT
• In patients with severe pulmonary valve stenosis, KEY POINT

valve intervention is recommended regardless of the • Initial therapy o f non-ST-elevation myocardial infarc­
presence or absence of symptoms. tion is medical, with antiplatelet and anticoagulant
medications, antianginal medications, and cardiopro­
Bibliography tective medications; early angiography (<6 hours or at
Burch M. Sharland M. Shinebourne E. Smith G. Patton M. McKenna W. hospital admission) does not provide incremental ben­
Cardiologic abnormalities in Noonan syndrome: phenotypic diagnosis
and echocardiographic assessment of ! l 8 patients. J Am Coll Cardiol.
efit but an early invasive strategy (defined as within
1993 Oct:22(4) : l l89- 92. [PMlD: 8409059] 24 hours of hospital admission) may improve outcomes.

Bibliography
Cl Educational Objective:
Item 73 Answer: A
Amsterdam EA, Wenger NK, Brindis RG, et al; ACC/AHA Task Force
Manage a patient with a non­ Members. 2014 AHA/ACC guideline for the management of patients with
ST-elevation myocardial infarction with antiplatelet and non-ST-elevation acute coronary syndromes: executive summary: a
report oft he American College of Cardiology/American Heart Association
anticoagulant medications. Task Force on Practice Guidelines. Circulation. 2014 Dec 3:130(25) :2354-
94. [PMID: 25249586]
ll1 is patient should be given a P2Y 1 2 i n h i b ito r. such as clopido­
grel. and an a n ticoagulant. such as the low-molecu la r-weigh t
heparin enoxapari n . a nd b e adm itted to I he hospital. Once Item 74 Answer: E
the d iagnosis of a non -ST-elevation myocardial i n farction
Educational Objective: Evaluate change in clinical sta­
(NSTEM I ) has been con Armed by the presence or ischemic
tus of a patient with mitral regurgitation.
chest pai n. ST segm en t de p ressi o n on the electrocardiogram
-

( ECG) . a nd or abnormal cardiac biomarkers. the use of' a n t i­ Transthoracic echocardiography (TIE) in a patient with val­
platelet a nd ant icoagu lant medications. a n tianginal medica­ vular heart disease is appropriate when there is a change in
t ions. a nd cardioprotective medications is imperative. ll1is clinical symptoms. This patient has had worsening dyspnea
patient was given aspi rin a nd n i t roglycerin prior to the diag­ on exertion for the past 3 weeks that may be a result of
nosis of NSTEM I. and he takes daily ACE i n hibitor a nd statin worsening mitral regurgitation. In patients with myxomatous
med icat ions. T he addi t ional t herapies t hat are warra nted in mitral valve disease, rupture of a primary or secondary chor­
t h i s situat i on i ncl ude a P2Y12 i n h i bi to r ( c l op i cl ogre l . prasu­ dae may cause an acute change i n the degree of mHra l regur­
grel. t icagrelor) . an ant icoagu lant (unfract ionated hepa rin o r gitation and change in clinical status; this is the likely cause
l ow- m ol ecu l a r we i ght h e pa ri n ) . a nd a �- b locke r. ·n1e use of'
- of this patient's worsening symptoms. Other causes of her
clopidogrel. in addi t ion lo aspirin. is the besl -stucliecl combi progressive shortness of breath could also be evaluated with
nation of antiplatelet medications. an echocardiogram. For example, new wall motion abnor­
I n patients w i t h an ST-elevation myocardial i n farction malities could signal recent silent myocardial infarction, and
(STEM I ) . repe rfusion . preferably ,·ia percuta neous coronary changes in overall ejection fraction would prompt evaluation
i rnervernion. should be performed as quickly as poss i b l e for new cardiomyopathies.
from symptom onset. T h is patient does n o t have evidence Exercise stress testing may be appropriate if TTE does
or ST-segment elevation or lef't bundle bra nch block on t he not reveal a structural cause of her shortness of breath.

1 92
Answers and Critiq u es

In addition to evaluating for obstructive coronary artery patients with mitrai valve regurgitation include a holosys­
disease, stress echocardiography could be used to evaluate tolic murmur, heard best at the apex, that radiates to the
changes in mitral regurgitation and pulmonary pressures axilla.
with exercise. However, a TTE at rest should be obtained Recurrent aortic coarctation occurs in about 20% of
before deciding whether a stress test is warranted. patients with previous coarctation repair. Clinical features
111is patient' s mitral regurgitation increases her risk for include hypertension that is difficult to control with medical
atrial fibrillation. However, her baseline electrocardiogram therapy. lll i s patient has hypertension that is controlled with
demonstrates sinus rhythm, so paroxysmal atrial fibrillation medical therapy, which occurs in up to 75% of patients with
as a cause of her dyspnea is less likely. In addition, she has repaired coarctation. Other features of recurrent coarctation
had progressive dyspnea on exertion that has not waxed not demonstrated in this patient include a radial artery-to­
and waned, as might be expected if she was in and out of femoral artery pulse delay and a systolic murmur over the
atrial fibrillation. 111e fact that she presents with worsening left anterior or posterior chest.
symptoms and is in sinus rhythm makes paroxysmal atrial
KEY POINT
fibrillation less likely. 111erefore, 24-hour ambulatory elec­
trocardiographic monitoring would not be the best choice • A bicuspid aortic valve is present in more than 50% of
for this patient. patients with aortic coarctation; more than 70% of
lll i s patient does not appear to have active asthma patients with a bicuspid aortic valve will require car­
symptoms or changes in pulmonary status that would indi­ diac surgical intervention for valve dysfunction or
cate a need for spirometry at this time. aortic pathology over the course of a lifetime.
Transesophageal echocardiography may be helpful in
further defining the anatomy, particularly if surgical inter­ Bibliography
vention is planned, but should not be the first diagnostic Tanous D. Benson LN, Horlick EM. Coarctation of the aorta: evaluation and
study. If the cause of the valvular disorder or degree of management. Curr Opin Cardiol. 2009 Nov;24 (6):509- 15. [PMID:
19667980]
regurgitation is unclear from the TIE, then transesophageal
echocardiography may be appropriate.

KEY POINT Item 76 Answer: B


• Transthoracic echocardiography in a patient with val­ Educational Objective: Manage risk of sudden cardiac
vular heart disease is appropriate when there is a death in a patient with heart failure with placement of an
change in clinical symptoms. implantable cardioverter-defibrillator.

lll is patient with heart failure and a low left ventricular


Bibliography ejection fraction should be referred for placement of an
Matulevicius SA. Rohatgi A. Das SR. et al. Appropriate use and clinical
implantable cardioverter-defibrillator (!CD) . For patients
impact of transthoracic echocardiography. JAMA Intern Med. 2013 Sep
23:173(17):1600-7. [PMID: 2:3877630] with an ejection fraction less than or equal to 35% and New
York Heart Association (NYHA) functional class JI or III heart
failure on optimal medical therapy, placement of an !CD is
Item 7 5 Answer: A a class I indication. Patients with new-onset heart failure
should not undergo placement of an !CD because ventricular
Educational Objective: Diagnose aortic valve stenosis in
function often recovers to above 35%. lll i s patient, however,
an asymptomatic adult with repaired aortic coarctation.
is on appropriate medical therapy, has had heart failure for
lllis patient most likely has aortic valve stenosis. 111e systolic at least 6 months, and is still symptomatic. For patients with
ejection cUck at the left sternal border suggests a bicuspid NYHA functional class IV heart failure symptoms, an !CD
aortic valve. 111e quality and location of the systolic ejection is not warranted unless the patient is a cardiac transplant
murmur suggest associated aortic stenosis. A bicuspid aor­ candidate.
tic valve is present in more than 50% of patients with aortic Cardiac resynchronization therapy (CRT) with a biven­
coarctation, and more than 70% of patients with a bicuspid tricular pacemaker to improve hemodynamic function of
valve will require surgical intervention for a stenotic or regur­ the heart may also be considered in patients with persistent
gitant valve or aortic pathology over the course of a lifetime. heart failure but is reserved for patients with evidence of
Ascending aortic aneurysms occur in up to 50% of conduction system disease. llle 2013 American College of
patients with aortic coarctation. Aneurysms are generally Cardiology Foundation/American Heart Association/Heart
clinically silent and require imaging to confirm their pres­ Rhythm Society (ACCF/AHA/HRS) guideline recommends
ence. Ascending aortic aneurysm that results in dilation of CRT in patients with an ejection fraction of 35% or below,
the annulus will be associated with the murmur of aortic NYHA functional class III to IV symptoms on guideline­
regurgitation (diastoUc murmur), not aortic valve stenosis as directed medical therapy, and left bundle branch block with
seen in this patient. QRS duration greater than or equal to 150 ms. With a QRS
Mitra! valve regurgitation is not an expected sequeia width of 100 ms, this patient is not a candidate for a biven­
of coarctation repair. In addition, the clinical features in tricular pacemaker in addition to an ICD.

1 93
Answers a n d Criti q u es

Adding an angiotensin receptor blocker to a heart fail­ is ge n e ra l ly l i m i ted to 4 5 0 to 5 0 0 mg m 2• b u t t h ere is con ­

ure regimen that already includes an ACE inhibitor would s i derable i nc l i v i c l u a l va r i a t i o n i n closes t h a t lead to tox ic

not be indicated in this patient as it would not provide addi­ responses.

tional benefit, and this medication combination has been 'l h i s pa t i e n t developed carclio tox i c i ty despi t e a low

shown to increase risk of hyperkalemia and kidney injury. r i s k ( I 'X, ) . ll1erelore. n e i t her co n t i n u i ng doxoru b i c i n nor

Because this patient's heart rate is 56/min, indicating decrea s i n g t he dose i s an appropriate t reatmen t . Aclcl it ion ­

adequate p-blockade, and his blood pressure is at a desired a l ly. bec<i u se C<ircl iotoxici ty is associated wi t h t h i s class o f

level, no benefit would be expected by increasing his dose agents. subst i t u t i ng a n o t h e r a n t h racyc l i ne for t h e cloxoru­

of carvedilol. b i c i n . such as clau noru b i c i n . wou ld a lso n o t be a p p ro p r i a te.

Indications for mitral valve replacement, an invasive I n t h i s pa t i e n t . <rn t h rncyc l i ne t he rapy m u st be d i scon t i n ued

procedure that carries risks, include the presence of severe i ndefi n i tely to reverse the carcli otox ic eflect s and p reve n t

mitral regurgitation and NYHA class III or IV symptoms t hem from recurri ng.

attributed to the valve disease. None of these are present ACE i n h i b i tors are i n d icated i n p<:l t i e n t s w i t h asymp­
in this patient. This patient's mitral regurgitation is "func­ toma t i c or sym p toma t ic syst o l i c dysfun ct io n . ll1ere a re l i m­

tional, " meaning it is more likely to be a result of his dilated i ted d a ta on t he use of t hese age n t s to h e l p preve n t ca r­

cardiomyopathy and not the underlying cause. d i otox i c i t y re l a t ed to chemot herapy. b u t t h e i r use for t h is
i nd i cation i s not rou t i nely recom mended.
KEY POINT
KEY POINT
• Implantable cardioverter-defibrillator placement is
indicated for patients with heart failure and a left ven­ • Anthracyclines, such as doxornbicin, can cause carcli­
tricular ejection fraction less than or equal to 35% and otoxicity in patients being treated with chemotherapy.
New York Heart Association functional class II or III
heart failure while on optimal medical therapy. Bibliography
Swain SM, Whaley FS, Ewer MS. Congestive heart failure in patients treated
with doxorubicin: a retrospective analysis of three trials. Cancer. 2003
Bibliography Jun 1 ;97(11):2869-79. [PMID: 12767102)
Bardy GH. Lee KL. Mark DB, et al; Sudden Cardiac Death in Heart Failure
Trial (SCD-HeFT) I nvestigators. Amiodarone or an implantable cardio­
verter-defibrillator for congestive heart failure. N Engl J Med. 2005 Jan
20;352(3):225-37. Erratum in: N Engl J Med. 2005 May 19;352(20):2146. Item 78 Answer: D
[PM ID: 156597221
Educational Objective: Evaluate a patient with dyspnea
with a history of catheter ablation for atrial fibrillation.
Cl Educational Objective: Treat cardiotoxicity due to che­
Item 7 7 Answer: B
lhe most likely cause of dyspnea in this patient is pulmonary
vein stenosis. He has progressive, unexplained dyspnea and
motherapy with an anthracycline.
a history of multiple catheter ablation procedures for atrial
-n1e doxoru b i c i n s h ou l d b e d i sco n t i n u ed i n d e fi n i t e ly i n L h i s fibrillation. During catheter ablation of atrial fibrillation,
pat i e n t . T h e e f'fi cacy o f a n t h racyc l i n e based c h e m o t herapy the tissue around each of the pulmonary veins is cauter­
reg i m e n s lor breast a d e n oca rci no m a has been. docu men led ized to achieve electrical isolation and prevent ectopic foci
by m u l t ip le s t u d ies. H ovveve r. doxoru b i c i n . d a u n o ru b i c i n . from triggering recurrent atrial fibrillation. Following this
a n d ot h e r a n t h racyc l i nes have k n ov; n c a rd io t ox i c i ty. w h i c h procedure, approximately l % to 3% of patients can develop
i s depe n d e n t o n dosage . age . conco m i t a n t t rea t m e n t w i t h symptomatic pulmonary vein stenosis. The risk is higher
o t h e r carcl i o t o x i c age n ts. chest rad i a t i o n . a n d p re e x i s t i ng after multiple procedures, but it can occur after a single
c a r d i a c d i sease . ll1e i nc i d e nce of' card i o t ox i c i t y l ( i r doxo ­ procedure. Patients present with progressive dyspnea, but
ru b i c i n or d a u n o r u b i c i n is less t h a n 1 'X, k i r c u m u l a t ive more severe pulmonary vein stenosis can be accompanied by
dosages of' less t h a n 400 m g m 2 bu t 26'X, lor c u m u l a t ive cough, hemoptysis, or chest pain. The image (shown on top
dosages grea t e r t h a n or eq u a l to 550 mg ' 111 2 . Cll"cl i o tox ­ of next page) depicts a three-dimensional cardiac magnetic
i c i t y from a n t h racyc l i nes usu<J l ly develops w i t h i n seve rn l resonance angiogram showing the pulmonary venous return
m o n t h s a ft er i n i t i a t i on or chemo t h erapy. m a n i test i n g as to the left atrium (LA) , with stenosis of a pulmonary vein at
e i t her systo l i c o r d i a s t o l i c h e a r t l'a i l u re. a l t hough t h ere ca n its ostium where it empties into the LA ( a rrow) .
be a long l a te n cy period . O n ce h e a r t f a i l ur e is prese n t . a s lhe most important step in the diagnosis of pulmonary
in t h i s pat i e n t . a n t h racyc l i n e s s h o u l d be clisco n t i n u ecl. I n vein stenosis is maintaining a high degree of suspicion when
t h e absence o r h e a r t fa i l u re. a n t h racyc l i nes a l s o s h ou l d encountering a patient with dyspnea and prior atrial fibril­
be cl isco n t i n u ecl w h e n new left ve n t ricu l a r dys fu nc t i o n lation ablation. Several diagnostic modalities can be used to
is cl e t ec t ecl . Ro u t i n e su rvei I l a n e e vvi t h s e r i a l echoca rel i ­ make the diagnosis noninvasively, including CT angiography,
ography is reco m me n d e d i n p a t i e n ts b e i n g t re<l l ecl w i t h magnetic resonance angiography, and nuclear lung perfu­
a n t h rncyc l i nes. w i t h t i m i n g i n terva l s based on t h e pa t i e n t ' s sion scanning. Each method has advantages and disadvan­
base l i n e fu n c t i o n . r i s k p ro l l le. dose. ;rnd c l i n i cal s u s p i c i o n tages, and the preferred method of diagnosis often differs
lo r tox ici t y. T h e m a x i m u m c u m u l a t ive d o s e !O r t h ese c l rugs from institution to institution.

1 94
Answers and Critiques

from therapy with an !CD. Patients should be considered for


!CD therapy if they have any of the following risk factors for
sudden cardiac death: (1) massive myocardial hypertrophy
(wall thickness �30 mm) ; (2) previous carcliac arrest due to
ventricular arrhythmia; (3) blunted blood pressure response
or hypotension during exercise; (4) unexplained syncope; (5)
nonsustained ventricular tachycardia on ambulatory elec­
trocard.iography; and (6) family h.istory of sudden death due
to HCM. In patients who have ICD placement for second­
ary prevention of sudden death (that is, patients with prior
carcliac arrest due to ventricular arrhythmia) , in whom the
indications for implantation are most compelling, the rate of
appropriate device discharge is approximately 11% per year.
For primary prevention, the annual rate of appropriate !CD
ctischarge is approximately 4%. Although older stucties from
tertiary referral centers suggested an adverse prognosis for
all patients with HCM, multiple contemporary investigations
from relatively unselected cohorts have shown that HCM is
compatible with normal longevity in most patients, partic­
Chronic thromboembolic disease can cause progressive
ularly those without risk factors for sudden death, with the
dyspnea, but it would be unlikely in a patient without prior
overall incidence of death ranging from 0.5% to 0.8% per year.
venous thromboembolism and on chronic anticoagulation.
Septa) reduction therapy, with either alcohol septa!
Although intermittent atrial fibrillation could cause
ablation or surgical myectomy. is indicated only for patients
symptoms after ablation, it is not likely to cause progressive
with drug-refractory, severe symptoms and not the mild
dyspnea in the setting of sinus rhythm. Intracarcliac shunt­
symptoms that this patient has. Survival has been shown
ing is rare after an ablation for atrial fibrillation. Although
to be favorable among selected patients who undergo sep­
trans-septa! catheterization is performed during ablation
ta] reduction therapy in experienced centers, but neither
procedures, the puncture site usually seals off. The normal
ablation nor surgical myectomy is advocated as a means of
oxygen saturation in this patient makes the possibility of a
preventing sudden cardiac death.
symptomatic shunt unlikely.
�-B locker therapy is reserved principally for patients
Phrenic nerve injury can result from delivery of radio­
with symptoms and has not been associated with reduction
frequency energy in the vicinity of the right superior pulmo­
in risk of sudden cardiac death.
nary vein with bystander injury of the right phrenic nerve;
however, this injury is observed immediately after ablation KEY P O I NT
and is accompanied by an elevated hemi-diaphragm on • Placement of an implantable cardioverter-defibrillator
chest radiograph. is an appropriate treatment for patients with hyper­
KEY POINT trophic cardiomyopathy who have one or more risk
factors for sudden cardiac death.
• Pulmonary vein stenosis is a known complication of
catheter ablation of atrial fibrillation and may present
Bibliography
with unexplained dyspnea.
Gersh BJ. Maron BJ. Bonow RO. et al: American College of Cardiology
Foundation / American Heart Association Task Force on Practice
Bibliography Guidelines. 201 1 ACCF/AHA Guideline for the Diagnosis and Treatment
of Hypertrophic Cardiomyopathy: a report of the American College of
Holmes DR Jr, Monahan KH, Packer D. Pulmonary vein stenosis complicat­ Cardiology Fou ndation/American Hearl Association Task Force on
ing ablation for atrial flbrillation: clinical spectrum and interventional Practice Guidelines. Developed in collaboration with the American
considerations. JACC Cardiovasc lnterv. 2009 Apr; 2(4):267-76. [PMID: Association for Thoracic Surgery. American Society of Echocardiography,
194634361 American Society of Nuclear Cardiology. Heart Failure Society of
America, Heart Rhythm Society. Society for Cardiovascular Angiography
and Interventions. and Society of Thoracic Surgeons. J Am Coll Cardiol.
20J t Dec 13;.58(25):e212-60. ! PM ID: 22075469]
Item 7 9 Answer: C
Educational Objective: Identify risk factors for sudden
death in a patient with obstructive hypertrophic cardio­ Item 80 Answer: B
myopathy. Educational Objective: Manage a small abdominal aor­
tic aneurysm with surveillance.
The patient should be treated by placement of an implant­
able cardioverter-defibrillator (!CD). He has asymptomatic The most appropriate management for thfa man with a 4.7-cm
mild obstructive hypertrophic cardiomyopathy (HCM). An abdominal aortic aneurysm (AAA ) is surveillance by abdom­
essential goal in the management of patients with HCM is risk inal u ltrasonography every 6 to 12 months. Aneurysm diame­
stratification and identification of patients who may benefit ter is the most important factor predjsposing to rupture, with

1 95
Answers a n d C ritiques

risk increasing with progressive increases in aneurysm size. for adm i n istration during PCI . rat her t ha n "up front" in the
For aneurysms smaller than 4.0 cm in diameter, the annual emergency departrnenl. Large studies have shown no clear
rupture risk is below 0.5%, whereas for aneurysms between mortal i ty benefit and sign i fi can t ly h igher rates o f' major
5.0 and 5.9 cm in diameter, the annual rupture risk is 3% to bleeding in patients u ndergoing fibrinolysis treated w i t h
15%. For AAAs with a maximum diameter less than 3.5 cm, abcixi mab versus placebo . U s e o f ' platelet glycoprotein l i b/
repeat surveillance with ultrasonography may be repeated I l l a i n h ibi tors in pat ien ts u ndergo i ng t h rombolysis is not
every 3 to 5 years. However, larger aneurysms require more cu rrently recom mended.
frequent surveillance because of their tendency to expand For STE M I patients u n dergo i ng primary PCI . both pras­
faster than smaller anemysms and their increased risk of ugrel and ticagrelor have shown superior efficacy compared
rupture, with reevaluation on a 6- to 12-month basis recom­ with clopidogre l . However. use of t hese agents in patients
mended. Because this patient's aneurysm is 4.7 cm in diam­ t reated with t h rombo lytic t herapy has not been wel l studied.
eter, he should undergo surveillance every 6 to 12 months. and l ittle evidence exists to reco mmend t he use or either of
Elective repair should be considered for AAAs of t hese agents i n patien ts receiving t h rombolyt ic t herapy.
5 . 5 cm in diameter in men and 5.0 cm in women, for AAAs
KEY POINT
that increase in diameter by more than 0.5 cm within a
6-month interval, and for those that are symptomatic (ten­ • Patients with ST-elevation myocardial infarction
derness or abdominal or back pain) . undergoing thrombolysis should be given adjunctive
AAA rupture has an exceedingly high mortality rate, antiplatelet therapy with clopidogrel.
and the risk of rupture is 0.5% to 5% annually for aneurysms
between 4.0 cm and 4.9 cm in diameter. Therefore, no fur­ Bibliography
ther management would not be appropriate in this patient Sabatine MS. Cannon CP. Gibson CM, et al; CLARITY-TIMI 28 Investigators.
Addition of clopidogrel to aspirin and fibrinolytic therapy for myocardial
with a 4.7-cm aneurysm. infarction with ST-segment elevation. Engl J Med. 2005 Mar
24:352(12):1179-89. [PM ID: 15758000]
KEY POINT
• An abdominal aortic aneurysm smaller than 5 . 5 cm in
Item 82 Answer: D
men and 5 . 0 cm in women is managed conservatively,
with routine surveillance by abdominal ultrasonogra­ Educational Objective: Manage iron-deficiency anemia
phy every 6 to 12 months. in a patient with cyanotic congenital heart disease.

ll1is patient with Eisenmenger syndrome has symptomatic


Bibliography relative anemia, and short-course iron therapy should be ini­
Chaikof EL, Brewster DC, Dalman RL, et al. SVS practice guidelines for the tiated. Most patients with cyanosis have compensated eryth­
care of patients with an abdominal aortic aneu1ysm: executive summary.
J Vase Surg. 2009 Oct: S 0 (4 ) :880- 96. [PMID: 19786241] rocytosis with stable hemoglobin levels. Iron deficiency and
resultant microcytosis in patients with cyanosis are often
caused by unnecessary phlebotomy or blood loss. Microcytes

CJ Item 81 Answer: B

Educational Objective: Treat a patient undergoing


are more rigid than norn1al erythrocytes; thus, microcytosis
increases the risk of stroke. The treatment of iron deficiency
is challenging. Oral iron often causes a rapid and dramatic
thrombolysis for an ST-elevation myocardial infarction
increase in erythrocyte mass. Therefore, administration of
with clopidogrel.
1 tablet daily of ferrous sulfate or ferrous gluconate is recom­
ll1is pat ient vvho is receiving thrombolyt ic t herapy for an mended, with repeat hemoglobin assessment in 7 to 10 days.
ST-elevation myocardial in farction (STEM ! ) shou ld be given This patient's usual hemoglobin level would be in the range of
clopidogrel. 300 mg ora l ly. Clopidogrel is the most widely 18 g/dL (180 g/L) . He developed perioperative anemia that has
studied antiplatelet agent in pat ients undergoing reperrusion persisted and caused dyspnea and fatigue. Once the hemoglo­
for STEM I . Clopidogrel is associated with improved outcomes bin and hematocrit levels begin to increase, iron therapy can
and no apparent i ncrease in the risk of' b leeding when used be discontinued. Erythrocyte transfusion is considered only
with concomitant t h rombolytic t herapy and with pri mary for severe symptoms.
percutaneous coronary in tervention ( PCI) . I n the CLA R ITY­ Erythropoietin is used in patients with anemia related
TI M I 28 study exa m i n i ng clopidogrel as adjunct ive t herapy i n to kidney disease and some malignancies but is not recom­
patients undergoing t h rombolysis. there was a 6.7% absolute mended for patients with heart disease and cyanosis who
risk reduction in the i ncidence of occluded in farct-related develop postoperative anemia.
artery or recurrent myocardial i nfarction or death in pat ients Data are limited regarding the efficacy of epoprostenol
assigned to clopidogrel when compared with placebo. in patients with Eisenmenger syndrome and pulmonary
Abciximab is a platelet glycoprotein l lb t l l la i n h i bitor hypertension. When used, oral, subcutaneous, and inhaled
t hat fu rther i n h i bi ts platelet aggregation and impairs p l a te­ therapies are preferred. Intravenous epoprostenol is typically
let activation . G lycoprotei n l lb I l l a i n hibitors are used i n avoided because of the risk for paradoxical embolism with
patients u ndergoing PCI. Owi ng to t h e risk of i ncreased continuous intravenous therapy in patients with an intra­
bleedi ng with t hese agents. t heir use should be reserved cardiac right-to-left shunt.

1 96
Answers and Critiq u es

Although heart-lung transplantation is an option for In this active construction worker who is asymptom­
end-stage cardiopulmonary disease in patients with Eisen­ atic, the pretest probability of finding ASCVD is low; because
menger syndrome, this option is not indicated without fur­ of the increased rate of false-positive test results in low-risk
ther trial of standard medical therapy with iron. patients, stress testing is not recommended. The use of stress
testing to diagnose ASCVD in asymptomatic persons does
KEY POINT
not reduce mortality. Appropriate risk factor modification,
• Most patients with cyanotic heart disease have com­ however, does have the potential to reduce cardiovascular
pensated erythrocytosis with stable hemoglobin levels. risk and mortality.
This patient's blood pressure is adequately treated, and
Bibliography there is no evidence that further lowering of blood pres­
Tay EL, Peset A, Papaphylactou M, et al. Replacement therapy for iron defi­ sure results in decreased cardiovascular risk. Therefore, an
ciency improves exercise capacity and quality of life in patients with
cyanotic congenital heart disease and/or the Eisenmenger syndrome. Int increase in his lisinopril dosage would not be beneficial.
J Cardiel. 2011 Sep 15;151(3):307-12. [PMID: 20580108] Although studies of the mechanisms of atherosclerosis
suggest that antioxidant therapy might be protective against
development of ASCVD, studies have failed to show a benefit
Item 83 Answer: A of antioxidants as a primary prevention intervention. The
Educational Objective: Use coronary artery calcium use of vitamins A, C, or E, alone or in combination, is there­
score to clarify cardiovascular risk in an intermediate-risk fore not recommended to decrease cardiovascular risk.
patient.
KEY POINT
The most appropriate option for this patient with interme­ • Coronary artery calcium scoring can improve cardio­
diate cardiovascular risk is to obtain an additional factor to vascular risk assessment in intermediate-risk patients
help clarify risk to guide therapy, such as a coronary artery in whom therapy may be affected by a reclassification
calcium (CAC) score. Assessment for risk of atherosclerotic of risk.
cardiovascular disease (ASCVD) is an important component
of primary prevention. Several risk assessment tools for
Bibliography
ASCVD are available, with the Framingham risk score being Greenland P, Alpert JS, Beller GA, et al; American CoUege of Cardiology
the most commonly used. The Pooled Cohort Equations are Foundation; American Heart Association. 2010 ACCF/AHA guideline for
a new risk assessment instrument developed from multiple assessment of cardiovascular risk in asymptomatic adults: a report of the
American College of Cardiology Foundation/American Heart Association
community-based cohorts (including the Framingham study) Task Force on Practice Guidelines. J Am Coll Cardiel. 2010 Dec
that includes a broader range of variables and endpoints than 14;56(2S):e50-103. [PMID: 21144964]

the Framingham score when evaluating 10-year ASCVD risk.


Its use as a primary risk assessment tool was recommended
in the 2013 American College of CardiologyIAmerican Heart Item 84 Answer: D
Association Guideline on the Assessment of Cardiovascular Educational Objective: Treat symptomatic peripheral
Risk. Using this method, a 10-year risk of ASCVD of below arterial disease with a supervised exercise program.
S% is considered low risk, S% to below 7.S% is classified as
The most appropriate management for this patient with
intermediate risk, and 7.S% or above is designated as high risk.
symptomatic peripheral arterial disease is to start a supervised
For patients with intermediate risk, such as this patient with a
exercise program. Supervised exercise therapy can effectively
risk of 6%, additional factors may be helpful in further refin­
treat claudication, with increases in pain-free walking time
ing risk assessment by identifying patients who may benefit
and maximal walking time, and is recommended as part of
from a more aggressive prevention strategy. Factors that can
the initial treatment regimen for intermittent claudication.
help further define risk include:
The CLEVER study demonstrated superior improvement in
• CAC score greater than 300 or greater than 7S% for age walking distance with supervised exercise for patients with
aortoiliac disease, as compared with stent revascularization
• High-sensitivity C-reactive protein level (hsCRP) above
or medical therapy alone.
2 mg/L
Cilostazol is an oral phosphodiesterase-3 inhibitor that
• Ankle-brachia! index below 0. 90 has demonstrated increases in pain-free walking and over­
• LDL cholesterol level 160 mg/dL (4.14 mmol/L) or all walking distance in patients with claudication in ran­
higher or other evidence of genetic hyperlipidernia domized clinical trials, although the gains with exercise
are two- to three-fold greater than with cilostazol alone.
• Family history of premature ASCVD with onset younger
However, cilostazol is contraindicated in patients with heart
than SS years in a first-degree male relative or younger
failure or a left ventricular ejection fraction below 40%.
than 6S years in a first-degree female relative
This contraindication exists because cilostazol has a similar
The results of the CAC score may help inform the indi­ pharmacologic action to the inotropic drugs milrinone and
vidual patient-physician discussion in this intermediate-risk amrinone, which demonstrated increased mortality rates
patient. with long-term use in patients with heart failure.

1 97
Answers a n d Critiques

Patients with stable claudication progress to critical this patient, CMR imaging and multidetector CT scanning
limb ischemia and limb loss at a rate of less than 5% annu­ are not indicated because the aortic root and ascending aorta
ally. For most symptomatic patients, therefore, noninvasive are adequately visualized by transthoracic echocardiogra­
therapy with exercise and medication is appropriate. If con­ phy. In addition, annual multidetector CT scanning would
servative therapy fails or patients have symptoms limiting needlessly expose the patient to excessive radiation.
their lifestyle or employment, revascularization by an endo­ Transesophageal echocardiography is an invasive pro­
vascular or surgical approach should be considered. cedure and would be considered if transthoracic imaging
was inadequate to measure aortic root size and evaluate the
KEY POINT
ascending aorta.
• Supervised exercise therapy can effectively treat clau­ Because of the risk of aortic dissection and rupture,
dication, with increases in pain-free walking time reassurance and clinical observation are inadequate fol­
and maximal walking time, and is recommended as low-up for bicuspid valve-related aortopathy.
part of the initial treatment regimen for intermittent
KEY POINT
claudication.
• I n patients with a bicuspid aortic valve, serial evalua­
Bibliography tion of ascending aortic diameter should be per­
Murphy TP, Cutlip DE, Regensteiner JG, et al; CLEVER Study Investigators. formed by transthoracic echocardiography (or by CT
Supervised exercise versus primary stenting for claudication resulting angiography or magnetic resonance angiography if
from aortoiliac peripheral artery disease: six-month outcomes from the
claudication: exercise versus endoluminal revascularization (CLEVER) not adequately visualized by echocardiography) .
study. Circulation. 2012 Jan 3:125(1):130-9. [ PM I D : 220901681

Bibliography
Morcli I, Tzemos N. Bicuspid aortic valve disease: a comprehensive review.
Item 85 Answer: D
Cardiol Res Pract. 2012;2012:1 96037. [PM I D : 22685681]
Educational Objective: Select proper imaging surveil­
lance for a patient with a bicuspid aortic valve and aor­
topathy. Item 86 Answer: D
Educational Objective: Diagnose obstructive coronary
This patient with a bicuspid aortic valve should have annual
artery disease in a patient with left bundle branch block.
transthoracic echocardiography. Bicuspid aortic valve occurs
with other cardiovascular and systemic abnormalities. Specif­ This patient with a left bundle branch block (LBBB) on her
ically, ascending aortic dilation may occur in persons with a baseline electrocardiogram (ECG) should undergo vasodilator
bicuspid aortic valve, in combination with aortic valve disease nuclear perfusion imaging. In patients with baseline ECG
or as an independent condition. Previously considered a sec­ abnormalities such as pre-excitation, LBBB, a paced rhythm,
ondary event caused by abnormal aortic valve function, the or baseline ST-segment depression greater than 1 mm, ECGs
aortopathy associated with a bicuspid aortic valve is now rec­ obtained during stress testing cannot be appropriately inter­
ognized to result from intrinsically abnormal connective tis­ preted, and standard exercise treadmill testing is therefore
sue. As a result, serial evaluation of ascending aortic diameter not appropriate. Instead, these patients must undergo stress
should be performed by transthoracic echocardiography (or testing with additional imaging, such as nuclear perfusion
by CT angiography or magnetic resonance angiography if not imaging or stress echocardiography.
adequately visualized by echocardiography) . The frequency When myocardial perfusion imaging is used to evaluate
of surveillance depends upon aortic root and ascending aorta patients with LBBB, a vasodilator study using an agent such
size. Expert consensus guidelines recommend reassessment as adenosine or dipyridamole is necessary instead of a n
of the aorta if the aortic root or ascending aorta dimension is exercise study. Th i s i s because perfusion defects that are not
greater than or equal to 4.0 cm, with the evaluation interval related to obstructive coronary artery disease (CAD) can be
determined by degree and rate of aortic dilation and by family seen in the septum with exercise. Radiotracers are distrib­
history. Annual evaluation should occur if the aortic diame­ uted with blood flow, and when the coronary arteries fill
ter is greater than 4.5 cm. New or changing symptoms and during diastole, the delay in contraction of the septum with
pregnancy are indications for earlier imaging of the aorta. For LBBB can impair filling and create a defect in the septum in
this patient, in view of the similar findings on transthoracic the absence of obstructive CAD. However, vasodilators pro­
echocardiogram and chest CT scan, serial transthoracic echo­ duce hyperemia and a flow disparity between myocardium
cardiography is reasonable. Moreover, transthoracic echo­ supplied by the stenotic vessel as compared with the unob­
cardiography is more cost effective than both CT and cardiac structed vessel that is not affected by the delay in septa! con­
magnetic resonance (CMR) imaging, and serial CT scans can traction related to LBBB. For this reason, an exercise nuclear
result in significant cumulative doses of radiation in this perfusion study would not be appropriate in this patient.
young patient. Coronary artery calcium scoring quantifies the amount
CMR imaging and multidetector CT are appropriate for of calcium in the walls of the coronary arteries and correlates
further assessment of aortic pathology when transthoracic well with plaque burden in the coronary arteries. It is an ana­
or transesophageal echocardiography is not conclusive. In tomic study with fairly high sensitivity for detecting occlusive

1 98
Answers a n d Criti q u es

CAD, although the frequency of false-negative results (signif­ to warrant immediate coronary angiography as the initial
icant CAD with a low CAC score) is not known. Therefore, diagnostic test.
CAC scoring is more frequently used for risk stratification in The sensitivity and specificity of noninvasive stress test­
patients with intermediate risk for atherosclerotic cardiovas­ ing for the evaluation of chest pain are lower in women
cular disease. Another anatomic study, coronary er angiog­ than in men. However, the routine use of exercise testing
raphy, is an emerging technology that has high correlation with either nuclear perfusion imaging or echocardiography
with findings on invasive coronary arteriography and may be to assess left ventricular regional wall motion or perfusion
increasingly useful in evaluating for occlusive CAD. imaging is not recommended for women or men in the
absence of baseline ECG abnormalities. Although the addi­
KEV POINT
tion of noninvasive imaging increases diagnostic sensitivity
• In patients with suspected coronary artery disease tor coronary artery disease, use of exercise nuclear perfusion
with baseline electrocardiographic (ECG) abnormali­ testing as the initial test has not been found to reduce cardio­
ties such as pre-excitation, left bundle branch block, a vascular events compared with exercise ECG testing alone.
paced rhythm, or ST-segment depression greater than Pharmacologic stress testing with imaging is indicated
1 mm, ECGs obtained during stress testing cannot be for patients who are unable to exercise. In addition, patients
interpreted; therefore, stress testing with additional with. left bundle branch block undergoing nuclear stress
imaging is required. testing should be administered a pharmacologic stressor
even if they are able to exercise because of the potential for
Bibliography a false-positive test owing to a septa] perfusion abnormality
Fihn SD, Gardin J M , Abrams J. et al: American College of Cardiology that may occur with exercise. Pharmacologic stress testing is
Foundation: American Heart Association Task Force on Practice not indicated because this patient is physically able to exer­
Guidelines: American College of Physicians: American Association lor
Thoracic Surgery: Preventive Cardiovascular Nurses Association: Society cise and does not have a left bundle branch block.
for Cardiovascular Angiography and Interventions: Society of Thoracic
Surgeons. 2012 CCF/AHA/ACP/AATS/PCNA1SCAl/STS Guideline for the K EV P O I N T
diagnosis and management of patients with stable ischemic heart dis­
ease: a report of the American College of Cardiology Foundation/ • Exercise electrocardiographic testing is recommended
American Heart Association Task Force on Practice Guidelines, and the as the initial test of choice in patients with a normal
American College of Physicians. American Association lor Thoracic
Surgery, Preventive Cardiovascular Nurses Association. Society for baseline electrocardiogram and an intermediate pre­
Cardiovascular Angiography and Interventions, and Society of Thoracic test probability of coronary artery disease based on
Surgeons. J Am Coll Cardiol. 2012 Dec 18:60(24):e44-el64. lPMID:
23182125] age, sex, and symptoms.

Bibliography
Qaseem A. Fihn SD, Williams S. Dallas P. Owens DK, Shekelle P; Clinical
Item 8 7 Answer: B Guidelines Committee of the American College of Physicians. Diagnosis
of stable ischemic heart disease: summary ofa clinical practice guideline
Educational Objective: Evaluate a woman with atypical
from the American College of Physicians/American College of Cardiology
chest pain with exercise electrocardiography. Foundation/ American Heart Association/American Association for
Thoracic Surgery/Preventive Cardiovascular Nurses Association/Society
This patient should undergo exercise electrocardiography of Thoracic Surgeons. Ann Intern Med. 2012 Nov 20; 157(10):729-34.
[PMID: 23165664]
(ECG). Although she has several risk factors for coronary
artery disease (CAD) , including hyperlipidemia and a family
history of premature CAD, her symptoms are not typical for Item 88 Answer: E
angina, which requires the presence of pain precipitated by Educational Objective: Manage aspirin use for primary
exercise or emotion, a substernal location of the pain, and prevention in a patient with diabetes mellitus and a low
relief with rest or nitroglycerin. Because she has only two cardiovascular risk.
of the three diagnostic criteria for angina, she is classified as
having atypical angina. Women in her age group with atyp­ No further testing or therapy would be most appropriate
ical angina have an intermediate pretest probability of CAD in this patient. Although she has diabetes mellitus, she has
(approximately 22%) . For patients with an intermediate pre­ no other major cardiovascular risk factors. Risk assessment
test probability of disease and a normal resting ECG, exercise for atherosclerotic cardiovascular disease (ASCVD) has tradi­
ECG testing is recommended as the initial test of choice. tionally been with the Framingham risk score, although the
Conventional coronary angiography identifies the loca­ American College of CardiologyIAmerican Heart Association
tion and severity of blockages and allows vascular access for Pooled Cohort Equations, a new method for assessment that
percutaneous intervention. Because of the invasive nature includes additional variables for risk stratification, is increas­
of coronary angiography and the inherent risks of vascular ingly being used. With this method, a 10-year risk of ASCVD
complications, it should be reserved for patients with acute of less than 5% is considered low risk, 5% to below 7.5% is
coronary syndrome requiring immediate intervention, life­ considered intermediate risk, and 7.5% and above is desig­
style-limiting angina despite medical therapy, or high-risk nated as high risk. This patient has a calculated 10-year risk of
criteria on noninvasive stress testing. This patient's pretest 2.7%, making her low risk for ASCVD. Therefore, no additional
probability of CAD is intermediate, which is not high enough testing is indicated at present.

1 99
Answers and Critiques

It is reasonable to give low-dose aspirin to adults with card i a c o u t p u t is a very h igh systemic vasc u l a r resista nce.
diabetes and no previous history of vascular disease who as t he syste m i c c i rc u l a t i o n in creases a f'terload to m a i n t a i n
are at increased cardiovascular risk and without increased bl ood pressure i n t he sett i ng o f low stroke vo l u m e. W i t h cor­
risk for bleeding. However, aspirin should not routinely be rection of t h e vo l u me overload s t a te. t h e next step in l herapy
given to patients with diabetes who are at low risk (men is to red uce a fterloacl w i t h n i t ropruss icle.
younger than SO years and women younger than 60 years N i l ro p ru s s i d e is an i n t ravenously a d m i n istered vaso ­
without other major risk factors such as hypertension or c l i l a t o r t h a t lowers sys t e m i c vascu l a r res i s ta n ce a n d . t h e re­
tobacco use) . The risks of gastrointestinal bleeding or hem­ fore . i n creases c a rd i ac o u t p u t . ll1 is t h e ra py should be used
orrhagic stroke outweigh the benefits of aspirin in this low­ o n ly i n the s e t t i ng o f i nvasive mo n i to r i n g. i n c l u d i ng a righ t
risk patient. h e a rt ca t h e t e r a n d pos s i b ly an a r t e ri a l l i n e lo c l osely mea ­
Coronary artery calcium scoring is reasonable to fur­ s u re syst e m i c p ressu re . C o u n t e ri n t u i t ive lo w h a t wou ld
ther define cardiovascular risk in patients with intermediate be expected. t h e b l ood pressu re u s u a l ly rises w i t h n i t ro ­
risk as determined by the Pooled Cohort Equations (S"lo to prusside because or t he i m p roved c a r d i a c pe rfo r ma n ce.
<7.S%). However, this patient's risk of ASCVD is considered N i t roprussi de is assoc i a ted w i t h pos s i b l e rebo u n d vaso ­
low; therefore, good adherence to lifestyle factors and mon­ con s l r i c t i o n fol lowi ng d i scon t i n ua t i o n a n d poten t i a l tox­
itoring of cardiovascular risk factors are most appropriate in i c i ty cl ue lo i t s m e t abol i s m to cya n i d e w i t h l onger term
this patient. use: t he re fore. t h e rapy i s genera l ly l i m i ted to no more t h a n
There is no role for routine exercise testing in an asymp­ 24 l o 48 hours i n most p a t i e n t s . Pat i e n t w i t h card i oge n i c
tomatic patient. In patients with low coronary artery disease s h o c k may a l so be t reated w i t h an i no t ro p i c agent s u c h as
pretest probability, false-positive results will be more com­ d o b u l a 111 i ne.
mon than true-positive results and may lead to unnecessary C h a n g i n g lo cont i n uous i n t ravenous furose m i d e is not
downstream testing and treatment. i n d icated because t he patient h a s normal fi l l i ng p ressures
Although elevated homocysteine levels are associated m a n i fested by t h e p u l m o n a ry ca p i l l a ry wedge pressure
with cardiovascular risk, no data support the use o f folic acid or l 6 m m Hg and right a t r i a l pressure of 4 mm Hg. More
supplementation. which can lower homocysteine levels, to aggressive d i uresis will not i m pact the p r i n c i p a l p ro b l e m .
reduce the risk. w h i c h is low card i ac o u t p u t a n d a h i g h sys t e m i c vasc u l a r
res ista nce. S t u d ies have eva l ua ted t h e e f fi c acy of co n t i n u ­
KEY POINT
ous versus i n te r m i l le n t b o l u ses o f i n t ravenous d i ur e t i c s i n
• Aspirin should not routinely be given to patients with pa t i e n t s hosp i t a l ized w i t h <lcute heart fa i l u re. ll1ere W<lS
diabetes mellitus who are at low cardiovascular risk no d i f ference d e m o n s t ra ted in p a t i e n t s· sy m p toms . k i d ney
(men younger than SO years and women younger !'u nct ion . or l e n g t h of s t ay between t he two s t ra t egies .
than 60 years without other major risk factors such as H igh - versus l ow-d ose d i u re t ics a l so have been eva l ua ted .
hypertension or tobacco use). P<J t i ents t a k i ng h i g h dosages e x h i b i ted a t rend toward m o re
d i u resis a n d s l i g h t \.vo rs eni n g of k i d ney fu nct io n. D i u resis
Bibliography s h o u l d be performed using w h a tever s t ra t egy i s necessa ry
Pignone M. Alberts MJ. Colwell JA, et al. Aspirin for primary prevention of to remove t he f l u i d .
cardiovascular events in people with diabetes: a position statement of the
Dopa m i ne was rece n t ly compared vvi l h nes i r i t i d e a nd
American D iabetes Association. a scientific statement of the American
Heart Association, and an expert consensus document of the American placebo in pat ients w i t h acute heart f a i l u re a n d m i ld k i d ­
College of Cardiology Foundation. Circulation. 2010 Jun 22;121 (24):2694 - ney dysfu nc t ion . No bene f i t was demonstrated w i t h e i t her
701 . IPMID: 20508178]
dopa m i n e o r nes i r i t icle compared with p l acebo fbr either
u ri ne output or protec t i o n o f k i d ney func t ion . I n ge neral .

t he res u l t s of s t u d i es eva l u a t i ng t h e use of i n o t ropic t herapy

Cl Ed ucational Objective:
Item 89 Answer: D for the t reat me n t of patients hos p i t a l ized w i t h acute heart
Treat low cardiac output in fa i l u re have been negative. For t h e rou t i ne care of p a t i e n t s
heart failure by reducing afterload. hosp i t a l ized w i t h h e a r t f a i l u re . dopam i ne. clob u l a m i ne . a n d
m i l ri none h ave not been shown lo be h e l p fu l a n d m a y b e
Th e most a p p ropriate add i t io n a l t re a t m e n t fbr t h i s p a t i e n t
associated w i t h adverse outcomes.
i s n i t ropru s s i de. A rter several days o r d i u resis . t h i s p a t i e n t
Esmolol i s <111 i n t rnvenous ()-b locker. Li ke a l l (}-blockers.
h<lS <l n o r m a l r i g h t a t ri a l pressure ( 0 -5 111111 Hg) a n d p u l ­
il has some negat ive i n o t ropic activi ty. and use of t h is
m o n a ry cap i l l a ry wedge pressure above normal but w i t h i n
d rug m ig h t worsen t he p a t i e n t ' s hemody n a m i c status. not
t h e accept a b l e range f b r p a t i e n t s w i t h heart fa i l u re (<1 8 111111
i m p rove i t .
Hg) lo provide opt i m a l ven t ri c u l a r fi l l i ng. These hemocly­
n a m i c parameters suggest t ha t t h e card iac outpul is very low K EY P O I N T
a n d is t h e major explanat ion fbr t h e p a t i e n t ' s heart f�1 i l u re • In patients with low-output heart failure, nitroprus­
sym p to ms . Acu t e heart ta i l u re is typica l ly m a rked by a com ­
side can reduce afterload and increase cardiac output;
b i n a t i o n of vo l u m e overload ( m a n i fested by an i n c reased
nitroprusside should be used only in the setting of
p u l m o n a ry capi l l a ry wedge pressure . usual ly 2'.1 8 111 111 Hg)
invasive cardiac monitoring.
a n d reduced card i a c o u t p u t . Part o f t he reason for reduced

200
Answers a n d Critiques

Bibliography Item 9 1 Answer: D


Binanay C. Califf RM. Hasselblad V. et al: ESCAPE Investigators and ESCAPE
Study Coordinators. Evaluation study of congestive heart failure and Educational Objective: Evaluate a patient with atypical
pulmonary artery catheterization effectiveness: the ESCAPE trial. JAMA. chest pain and indeterminate findings on exercise stress
2005 Oct 5;294(13): 1625-33. [PMID: 16204662]
testing.

This patient with atypical chest pain and indeterminate results


on exercise stress testing should undergo stress echocardiog­
Item 90 Answer: A
raphy. Baseline abnormalities on the resting electrocardio­
Educational Objective: Manage liver disease resulting gram (ECG) can limit the ability to interpret the ECG during
from constrictive pericarditis. exercise. Specific abnormalities on the resting ECG preclude
The patient should undergo hemodynamic cardiac catheter­ the use of ECG stress testing because of difficulty in accurately
ization. He has evidence of liver disease most likely caused by interpreting changes that may occur with stress. These abnor­
constrictive pericarditis. Patients with constrictive pericar­ malities include the presence of left ventricular hypertrophy
ditis can develop severe symptoms of right-sided congestion with repolarization abnormalities (ST-segment depressions)
that can lead to chronic hepatic congestion and hepatopathy. greater than 0.5 mm, pre-excitation, left bundle branch block,
When detected early, cirrhosis can be reversible if constrictive and a paced rhythm. [n these settings, exercise stress testing
pericarditis is identified and treated in a timely manner with with imaging, either with echocardiography or perfusion
surgical pericardiectomy. An important clue that suggests imaging, is required. Although patients without these exclu­
constrictive pericarditis as a contributor to hepatopathy is sionary findings may undergo stress ECG, baseline changes
an elevated jugular venous pulse, which demonstrates the need to be considered in interpreting the test. This patient had
presence of elevated right atrial pressures. The central venous ST-segment depressions less than 0.5 mm at baseline; if there
pressure is rarely abnormal in patients with primary liver dis­ were no further changes with exercise testing, these findings
ease. A hemodynamic study with an echocardiogram is suf­ could be interpreted as normal. However, a positive stress test
ficient to diagnose constrictive pericarditis in most patients. is defined as greater than 1-mm ST-segment depression in two
For patients such as this one with an indeterminate echocar­ contiguous leads during exercise testing. In this patient, the
diogram, the most appropriate next step is an invasive hemo­ 1-mm ST-segment depression that developed during exercise
dynamic study with cardiac catheterization, which provides is a less reliable predictor of ischemia given the ST-segment
the greatest sensitivity and specificity for the diagnosis of baseline abnormalities. Therefore, these changes on stress
constrictive pericarditis. testing are not definitive and her test result is considered inde­
A liver biopsy may be helpful in patients with liver dis­ terminate. She should therefore undergo repeat stress testing
ease of unknown etiology or to gauge the type and degree of with imaging in order to establish a diagnosis of coronary
liver injury or fibrosis. A liver biopsy is not performed prior artery disease (CAD).
to the completion of less invasive tests, such as a liver chem­ The patient does not yet have a diagnosis of CAD and her
istry profile and ultrasonography. In this patient, the elevated hypertension is adequately treated on her current regimen;
central venous pressure suggests cardiac rather than primary adding treatment with a �-blocker to treat myocardial isch­
liver disease, and cardiac catheterization with hemodynamic emia would not be indicated.
measurements is more likely to prove diagnostic. Cardiac catheterization should be reserved for patients
B-type natriuretic peptide level is often normal or min­ with high-risk features on exercise stress testing, such as a
imally elevated in patients with constrictive pericarditis high-risk Duke treadmill score (below - 11), hypotension,
despite the elevated central venous pressure. However, mea­ severe ST-segment depression, and early-onset angina. 1his
surement of B-type natriuretic peptide will not establish the patient has atypical chest pain, and cardiac catheterization
diagnosis of constrictive pericarditis or guide therapy. should be deferred until a diagnosis of CAD is made and the
Judicious use of loop diuretics is important in patients extent and severity ofdisease are evaluated with stress imaging.
with constrictive pericarditis to reduce dyspnea and edema, Cardiac magnetic resonance (CMR) imaging is best uti­
but because higher filling pressures are needed to maintain lized for diagnoses of the aorta, pericardium, and myocar­
stroke volume, overly aggressive diuresis can reduce cardiac dium, including viability and extent of myocardial fibrosis.
output, causing dizziness and orthostatic hypotension. Stress CMR imaging can be performed, but its availability is
limited.
KEY POINT
KEY P O I NT
• For patients with suspected constrictive pericarditis
• In a patient with indeterminate results on exercise
who have an indeterminate echocardiogram, the most
stress testing, a repeat stress test with imaging should
appropriate next step is an invasive hemodynamic
be performed to elucidate the diagnosis.
study with cardiac catheterization.

Bibliography Bibliography
Khand aker MH, Espinosa RE, Nishimura RA. et al. Pericardia! disease: diag­ Fihn SD. Gardin JM. Abrams J, et al: American College of Cardiology
_ and management. Mayo Clin Proc. 2010 Jun;85(6) :572-93. [PMID:
nosis Foundation: American Heart Association Task Force on Practice
20511488] Guidelines: American College of Physicians; American Association for

201
Answers a n d Critiq ues

Thoracic Surgery: Preventive Cardiovascular Nurses Association; Society PCl -c<lp<lble hospital c a n be achieved ( f i rst medical contact­
for Cardiovascular Angiography and Interventions: Society of Thoracic
to- device time of 120 minutes or less) . I n studies of patients
Surgeons. 2012 CCF/AHA/ACP/AATS/PCNA/SCAl/STS Guideline for the
diagnosis and management of patients with stable ischemic heart dis t rans ferred from <J non - PCI faci l i ty fo r pri m<J1y PCI, more
ease: a report of the American College of Cardiology Foundation /
than half of patients w i t h STE M ! did not undergo perfusion
American Heart Association Task Force on Practice Guidelines, and the
American College of Physicians. American Association for Thoracic i n 1 20 m i n utes or Jess . In t h i s case, t ra nsfer l i me would be
Surgery. Preventive Cardiovascular Nurses Association. Society for prolonged (>120 m i n u tes) : t here fore. t h rombolytic t herapy is
Cardiovascular Angiography and Interventions, and Society of Thoracic
Surgeons. J Arn Coll Cardiol. 2012 Dec 18:60 (24):e44 -el64. [PMID: the best reperfusion strategy.
23182125]
KEY POINT
• Patients with an ST-elevation myocardial infarction
Cl Educational Objective:
Item 92 Answer: A
Manage a patient with an acute
presenting within 12 hours of symptom onset to non­
percutaneous coronary intervention (PCI)-capable
coronary syndrome with thrombolytic therapy. hospitals should receive either primary PC! (if availa­
·n1is pat ient should receive a t h rombolytic agent such as ble in <120 minutes) or thrombolytic therapy (if pri­
tenecteplase and be transferred to a center capable of' per­ mary PC! is not available within 120 minutes).
form i ng percutaneous coronary intervention (PCI) . He has
electrocardiographic changes consistent w i t h an acute i n fe­ Bibliography
rior ST-e leva t ion myoca rdial i n farction (STEM ! ) . Pa t ie n ts O'Gara p·i: Kushner FG. Ascheirn DD. et al: American College or Cardiology
Foundation/American Heart Association Task Force on Practice
w i t h a STE M ! p resen t i ng w i t h i n 12 hours of symptom onset Guidelines. 2013 ACCF/AHA guideline tor the management or
should receive reperf'u sion t herapy w i t h eit her primary PC! ST-elevation myocardial infarction: a reporl of the American College of
Cardiology Foundation/American Hearl Association Task Force on
or t h ro mbolys i s . with PCI being the preferred intervention
Practice Guidelines. Circulation. 2013 Jan 29:127(4):e362-425. Erratum
owing to increased eITT cacy. When transfer t i mes for primary in: Circulation. 2013 Dec 24:128(25):e48l. [PMID: 23247304]
PC! exceed 120 mi nutes from presen tat ion . admin istration of'
t h rombolytic t h e rapy is recom mended . such as i n this pat ient
presenting to a faci l i ty wit hout PC! capabi l i ty and a n inabi l i ty Item 93 Answer: A
to transport h i m for treat ment wit h i n t hat t i me frame.
Educational Objective: Diagnose severe aortic
' l h i s pat i e n t has no absol ute con t ra i nd i cations to
regurgitation.
t h romboly t i c therapy, which i nclude previous i n t racerebral
hemorrhage. a known cerebrovascu lar lesion (such as a n T h is patient has aortic regurgitation. The murmur of aortic
a rteriove nous mal forma t i o n ) . suspected aort ic d issection . regurgitation, described as a diastolic decrescendo murmur, is
active bleedi ng or bleed i ng diat hesis (excluding menses) . heard best at the third left intercostal space and may be better
sign i f i ca n t closed head or facia l t ra u ma w i t h i n 3 months. heard when the patient is at end-expiration, leaning forward.
a nd ischemic st roke w i t h i n t he past 3 months. A relat ive Chronic aortic regurgitation has many <Jssociated findings,
con t ra i ndication for t h rombolysis is severe hypertension including widened pulse pressure, bounding carotid and
(defi ned as a systolic blood pressure > 1 80 mm Hg) : however. peripheral pulses, and a diffuse and laterally displaced point
t h i s pat ient's systolic blood pressure does not meet t h is of maximal impulse. A low-pitched rumbling diastolic mur­
t h reshold. Even when t h rombolytic t herapy is ad m i n istered, mur ("Austin Flint murmur") can accompany aortic regurgi­
t reatment guide l i nes recom mend t hat pa t ients be t rans­ tation and is caused by premature closure of the mitral leaflets
ferred to a PCl -capable fac i l i ty because of t he poten tial for clue to the regurgitant aortic flow.
t h rombolytic fai l ure. O p t i ma l management o f patients w i t h The auscultato1y findings for mitral stenosis include
STE M ! relies heav i ly upon physician recogni t ion and rapid an opening snap with a low-pitched mid-diastolic murmur
i n i t iation o f reperf'usion t herapy, w i t h e i t her t h romboly t ic (often described <ls a rumble) that accentuates presystole and
t he ra py or PC! . Pat ients w i t h an acute coronary syndrome is heard best at the apex with the patient in the left lateral
a nd a n elect rocardiogram compatible w i t h STE M ! should be decubitus position. It most often occurs in patients with
t reated w i t h reperf'usion t herapy wit hout biomarker con f i r­ rheumatic valve disease and is frequently associated with
mation. as early bioma rker resu l ts may be normal in pat ients atrial fibrillation.
with ST E M ! . Therefore. wa i t i ng for the resu lts of' card iac bio­ A small patent ductus arteriosus in the adult produces
rna rker l eve ls wou l d delay appropriate t reatmen t . an arteriovenous fistula with a continuous murmur that
'l h e u s e o f a glycoprotein l i b / I l l a i n h i b i tor. such as envelops the S2 and is characteristically heard beneath the
abcix imab. has not been shown to improve outcomes o f left clavicle. Patients with a moderate-sized patent duc­
pat ients w i t h S T E M I p r i o r to t h e primary PC! procedure a n d tus arteriosus may present with a continuous "machinery­
s h o u l d be reserved f o r a d m i n istra t ion in t he catheteriza t ion type" murmur best heard at the left infraclavicular area and
labora to1y during primary PC! . bounding pulses with a wide pulse pressure.
Tra nsfer fo r primary PC! is a reasonable a l ternat ive The sinuses of Y<llsalva are three aortic dilatations just
lo l h rombolytic t herapy in t h e set t i ng of' absolute con t ra ­ above the aortic valve cusps. Two of the three sinuses are the
i nd ications to t hrombolylic therapy or h igh-risk c l i n ical origins of the coronary arteries. Regurgitant blood flow into
feat ures <J nd i f' a n accepl<1 b le l i me to t ra nsfer the patient to a the sinus structures fills the coronary arteries and assists

202
Answers a n d Critiques

in the closure of the aortic valve cusps. Sinus of Valsalva An ABI greater than 1 .40 is associated with calcification
aneurysm is a type of aortic root aneurysm. Rupture of the of the arterial wall and may occur in patients with medial
aneurysm will allow flow between the sinus o f Valsalva and calcinosis, diabetes mellitus, or end-stage kidney disease.
either the right atrium or right ventricle, producing a con­ This finding is uninterpretable and is associated with worse
tinuous systolic and diastolic murmur heard loudest at the cardiovascular outcomes than a normal ABI; t herefore, an
second left intercostal space. Clinical presentation can vary, appropriate next step after this finding is to either measure
ranging from asymptomatic to decompensated heart fail­ great toe pressure or calculate a toe-brachia! index (systolic
ure. Ruptured sinus of Valsalva aneurysm more frequently great toe pressure divided by systolic brachial pressure) , a
involves the left or right coronary cusps and less frequently test that is typically performed in a vascular laboratory. This
the noncoronary cusp. patient's ABI is normal, so measurement of the toe-brachia!
index is not necessary.
K EY P O I N T
• The murmur of aortic regurgitation is a diastolic KEY P O I NT
decrescendo murmur heard best at the left third • Patients with pseudoclaudication (lumbar spinal ste­
intercostal space; associated findings include widened nosis) may report bilateral leg weakness associated
pulse pressure, bounding carotid and peripheral with walking or with prolonged standing; symptoms
pulses, and a diffuse and laterally displaced point are aggravated by prolonged standing and are relieved
of maximal impulse. with bending at the waist.

Bibliography Bibliography
Choudhry NK, Etchells EE. The rational clinical examination. Does this Chou R. Qaseem A, Owens DK, Shekelle P; Clinical Guidelines Committee
patient have aortic regurgitation? JAMA. 1999;281(23):2231-8. lPMID: of the American College of Physicians. Diagnostic imaging !Or low back
103765771 pain: advice for high-value health care from the American College of
Physicians. Ann Intern Med. 2011 Feb 1 ; 154(3) :181-9. Erratum in: Ann
Intern Med. 2012 Jan 3; 156(1 Pt 1 ) :71. [PMID: 21282698]

Item 94 Answer: B
Educational Objective: Distinguish lumbar stenosis Item 95 Answer: A
from peripheral arterial disease. Educational Objective: Manage acute decompensated
systolic heart failure with diuretics.
An MRI of the lumbar spine is most likely to confirm the
diagnosis in this patient. This patient's normal ankle-brachial I n this patient with recently diagnosed heart failure, the
index (AB!) bilaterally, normal distal pulses, lack of a bruit, dosage of furosemide should be increased. She has signs of
normal skin findings, and clinical history all suggest a diagno­ volume overload (elevated central venous pressure, an S3 ,
sis other than peripheral arterial disease (PAD). Patients with peripheral edema, weight gain) .
pseudoclaudication (lumbar spinal stenosis) may report bilat­ Given the patient's relative hypotension and volume
eral leg weakness associated with walking or with prolonged overload, increasing her diuretic dose would be more appro­
standing; symptoms are aggravated by prolonged standing priate than increasing the dose of her ACE inhibitor, which
and are relieved with bending at the waist. Nearly half of might lead to low blood pressure and would not improve her
patients have absent deep tendon reflexes at the ankles, but volume overload.
reflexes at the knees and muscle strength are usually pre­ Although there is a mortality benefit to the use of
served. lhe American College of Physicians recommends that p-blockers in patients with systolic heart failure, these
advanced imaging with MRI or CT should be reserved for agents have negative inotropic activity, and initiation of
patients with a suspected serious underlying condition or P-blocker therapy is relatively contraindicated i n patients
neurologic deficits, or who are candidates for invasive inter­ with evidence of decompensated heart failure. Once the
ventions. In the absence of these indications, back imaging is patient has been appropriately diuresed, a p-blocker can
not indicated. be added. Even patients with a low systolic blood pres­
Measuring the exercise ABI can be useful in diagnosing sure, once euvolemic, can often tolerate low doses of a
PAD when the resting AB! is normal and the index of suspi­ P-blocker.
cion is high for PAD. This patient's history and examination Spironolactone is an appropriate agent to add for treat­
findings point to a diagnosis other than PAD, so measuring ment of stable patients with New York Heart Association
the exercise AB! will not add helpful information at this (NYHA) functional class II to IV heart failure. This patient,
time. however, has acute volume overload, which should be treated
Segmental limb plethysmography is useful in patients before initiation of this therapy. Although spironolactone
with an established diagnosis of PAD to help localize the site has some diuretic activity, at the usual doses prescribed for
of stenosis. In this test, blood pressures are recorded using patients with heart failure (12.5-25 mg/d) , it would not have
plethysmographic cuffs placed at the upper thigh, lower sufficient diuretic effect in this patient.
thigh, calf, and ankle. A drop in systolic pressure of 20 mm This patient's presentation demonstrates the i mpor­
Hg identifies a zone of significant disease. tance of an early (within 7 days) post-hospital clinic visit for

203
Answers and Critiques

patients after a hospitalization for heart failure. Recognizing KEY POINT


volume overload at a point when it can be treated on an
• Antiplatelet therapy, such as aspirin, is first-line ther­
outpatient basis is an example of the benefit of this visit. If
apy for patients with patent foramen ovale and initial
the patient were euvolemic, adding additional therapy, such
cryptogenic stroke.
as a P-blocker or spironolactone, would be appropriate.
This visit also allows the internist to reemphasize to the
Bibliography
patient the importance of medication adherence and fluid
Chen L, Luo S. Yan L. Zhao W. A systematic review of closure versus medical
restriction. therapy for preventing recurrent stroke in patients with patent foramen
ovale and cryptogenic stroke or transient ischemic attack. J Neural Sci.
KEY POINT 2014 Feb 15:337(1-2):3-7. lPMID: 24300230]
• I n patients with acute decompensated systolic heart
failure, the most appropriate treatment is to increase
the diuretic dosage to remove the excess fluid. Item 97 Answer: D
Educational Objective: Manage a bicuspid aortic valve
Bibliography in an asymptomatic adult.
Felker G\11 . Lee KL, Bull DA. et al: N HLBl Heart Failure Clinical Research
Network. Diuretic strategies in patients with acute decompensated The most appropriate next step in management is to repeat
heart failure. N Engl J Med. 2 0 1 1 Mar 3;364(9) :797-805. [PM ID:
2 1366472]
the echocardiogram in 1 year. This patient has a bicuspid
aortic valve and is asymptomatic. Patients with a bicus­
pid aortic valve should undergo surveillance transthoracic
echocardiogram imaging of the aortic valve, aortic root,
Item 96 Answer: A and ascending aorta to exclude aortic valve pathology and
Educational Objective: Treat a patient with patent fora­ ascending aortic aneurysm, which are commonly associated
men ovale and cryptogenic stroke with aspirin. with a bicuspid aortic valve and may be independent of the
degree of aortic valve disease. This patient has mild aortic
This patient has features of a cryptogenic stroke and patent
valve stenosis and a slightly dilated ascending aorta. If the
foramen ovale (PFO ) . Antiplatelet therapy, such as aspirin,
aortic root or ascending aortic diameter is less than 4 cm
is first-line therapy for patients with PFO and initial cryp­
togenic stroke. The foramen ovale usually closes within the
reimaging should be done approximately every 2 years. If th �
aortic root or ascending aortic diameter is greater than 4 cm,
first few weeks of life. In 25% to 30% of the population, the
reimaging should occur yearly or more often as progression
foramen ovale remains patent. This is usually an incidental
of aortic dilation warrants or whenever there is a change in
finding.
clinical symptoms or findings.
Data are insufficient to recommend PFO device clo­
Aortic valve replacement is recommended in asymp­
sure for secondary stroke prevention after a first stroke
tomatic patients with severe aortic valve stenosis, generally
unless there are exceptional circumstances, such as oxygen
defined as a valve area below 1 cm2, who may be at high risk
desaturation from right-to-left shunt or thrombus trapped
without surgery. This patient does not have severe aortic
in the PFO, or for patients with recurrent cryptogenic
valve stenosis, so aortic valve replacement is not necessary
stroke on warfarin therapy. Overall, the PFO device closure
at this time.
procedure is safe and short-term complications are rare;
Surgery to repair or replace the ascending aorta in
however, evidence of long-term follow-up and benefit
adults with a bicuspid aortic valve is recommended when
is lacking. Randomized controlled studies have failed to
the ascending aorta diameter is greater than or equal to
demonstrate superiority of closure over anti platelet ther­
5.5 cm or progressive dilatation occurs at a rate of 0 . 5 cm
apy for secondary prevention after cryptogenic stroke.
per year or greater.
Uncertainty persists regarding the best management strat­
Medical therapy has not been shown to reduce the
egy because of limitations of current randomized con­
rate of progression of aortic dilation in patients with aor­
trolled studies; therefore, a reasonable consideration is
topathy associated with bicuspid aortic valve. In patients
to encourage participation in one of the ongoing clinical
with hypertension, control of blood pressure is essen­
trials.
tial. P-Blockers and angiotensin receptor blockers have
Data are insullicient to support warfarin t herapy for
conceptual advantages to reduce the rate of aneurysm
patients with PFO and initial cryptogenic stroke. Warfarin is
progression but have not been shown to be beneficial in
the treatment of choice for patients with PFO and recurrent
clinical studies.
stroke or for patients with stroke and a documented hyper­
coagulable state. K E Y P O I NT
Surgical PFO closure, including a minimally invasive • Patients with an asymptomatic bicuspid aortic valve
approach, is another option for patients with PFO and recur­
should undergo surveillance transthoracic echocardi­
rent cryptogenic stroke but the efficacy of surgical closure in
ography yearly if the aortic root or ascending aortic
these patients has been variable and randomized trials have
diameter is greater than 4 cm.
not been performed.

204
Answers a n d Critiques

Cl
Bibliography Item 99 Answer: D
Siu SC. Silversides CK. Bicuspid aortic valve disease. J Am Coll Cardiol. 2010
Jun 22;55(25):2789-800. [PMID: 20579534] Educational Objective: Treat native valve infective
endocarditis complicated by heart block.

·1 he pat ie n t should undergo aortic valve rep l acement now.


Item 98 Answer: A A l t hough he is hcmody n a m i cal ly stable a n d does not
req u i re pacem a ker support. t he presence or a new con­
Educational Objective: Evaluate palpitations with a
duct ion defect con f i rms extension or t he i n fection i n to
loop recorder.
t he periva l v u l a r t issues as suggested by echoca rd i ogra p hy.
This patient's symptomatic episodes are intermittent and short­ When t h is occu rs. t h e e f tect iveness or cure w i t h a n t ibiot ics
lived; therefore, a 30-day wearable recorder with looping mem­ alone is decreased sign i f i ca n t ly. and early s u rgical i n ter­
ory is the best diagnostic strategy to uncover the nature of her ven t i on is i nd ica led . O t her i n d icat ions for early s u rge ry i n
palpitations. These recorders are worn continuously and record n a t ive valve i n fect ive endocard i t i s include valve stenosis
a continuous "loop" of heart rhythm. When the patient expe­ or regurgi t a t i o n resu l t i ng i n heart fa i l u re: left - sided endo­
riences symptoms, she can depress a button and the device ca rd i t is caused by Staphy lococcus a u reus, fu nga l . or ot her
captures the rhythm before, during, and after the symptoms. h ig h ly resist a n t orga n isms: endocard i t is com p l i cated by
Loop recorders are useful for recording episodes accompanied a n n u l a r or aonic a bscess; a nd endoca rd i t i s w i t h persistent
by syncope or presyncope and for episodes that are too short to bacteremia or fever last i ng longer t ha n 5 to 7 days a f'ter
be captured by a patient-triggered event recorder. start i ng a n t i b i o t i c t herapy. Add i t iona l ly, early su rgery i s
A patient-triggered event recorder (without looping reasonable i n patients w i t h i n fective endocard i t i s w h o have
memory) is useful for recording infrequent episodes that recurrent emboli <J nd persistent vegetations on a n t i bi o t ic
last long enough (1-2 minutes) for the patient to hold the t her<Jpy. and m<Jy be considered i n pa tien ts w i t h n a t i ve
device to the chest and trigger it to capture the heart rhythm. valve endoca rd i t is who have mob i l e vegetat ions great e r
A self-applied event recorder is not useful for brief episodes t h a n l O m m i n length .
because the time taken to apply the monitor may be longer Durat ion or a n l i biot ic t herapy i n patients w i t h native
than the symptomatic episode. valve i n fect ive endocard i t is is general ly 4 to 6 weeks. based
If a patient has an abnormal cardiovascular examina­ upon t he spec i f i c orga n ism, l he site or i n fection, and a ny
tion or is demonstrated to have an arrhythmia, echocardi­ associ<Jted com p l icat ions. Genera l ly. 6 -week t reat ment reg­
ography should be performed to evaluate for the presence of i mens are used in pat i e n t s wi l h more virulent or highly
structural heart disease. I n this patient, however, the cardiac resista n t orga n isms and t hose w i t h cardiac or extracardiac
examination is normal, and there is no documentation of i n fectious compl icat ions. Prolonged t rea t me n t for 3 m o n t hs
an arrhythmia at this point. Many patients with symptoms as t herapy for t h is patient would not be appropri<Jte, even
suggestive of arrhythmia are found to have causes for their w i l h surgical i n tervention. a n d wou l d not be i n d i cated i n
symptoms that are not related to heart rhythm. ot her patients w i l h nat ive valve endocarci i ti s except i n cer­
An exercise treadmill stress test would be reasonable if t a i n si tuat ions.
the episodes were precipitated by exertion or exercise, but Delay i ng i n tervention for 6 weeks of a n t ibiotic t herapy
this patient's episodes are not associated with exertion. or t rea t i ng w i t h a n t i bioti cs <Jlone w i t hout surgery would not
Given the infrequency of this patient's symptoms, l i ke ly adequately ad d ress t h i s p a t ien t ' s endocard i t is-related
48-hour ambulatory electrocardiographic monitoring is not com pl ications. J n add i t ion, t h i s approach may resu l t in fur­
likely to capture the symptomatic episodes. t her decompensation of' the pat ient's clin ical status a n d a n
In patients with very infrequent or rare episodes i ncreased operative risk for i ntervention a t a later t i me.
(>30 days between episodes) , an implanted loop recorder may Key to management of' patients w i t h i n fective endo­
be appropriate. These devices, which are approximately the card i t is req u i ri ng surgery is a m u l t i d iscipli nary approach
size of a pen cap and are implanted under the skin of the chest i nvolving t he i n ternist. cardiologist. i n fect ious d isease spe­
next to the sternum, have several years of battery life. Although cialist. a n d card iac surgeon .
they are invasive, these devices have a higher diagnostic yield
K EY P O I N T
than other forms of outpatient heart rhythm monitoring.
• The presence of a conduction block is an indication
K EY P O I N T
for surgical therapy in patients with native valve
• A looping event recorder is useful for recording epi­ infective endocarditis.
sodes of palpitations that are accompanied by syncope
or presyncope and for episodes that are too short to be Bibliography
captured by a patient-triggered event recorder. Nishimura RA. Otto CM. Bonow RO, et al; American College of Cardiology/
American Heart Association Task Force on Practice Guidelines. 2014
AHA/ ACC guideline for the management of patients with valvular heart
Bibliography disease: executive summary: a report of the American College of
Subbiah R, Gula U, Klein GJ. Skanes AC. Yee R. Krahn AD. Syncope: review Cardiology/American Heart Association Task Force on Practice
of monitoring modalities. Curr Cardiol Rev. 2008 Feb:40):41-8. [PMID: Guidelines. J Am Coll Cardiol. 2014 Jun 10;63(22):2438-88. Erratum in: J
199242761 Am Coll Cardiol. 2014 Jun 10;63(22):2489. [PMID: 24603192]

205
Answers a n d Critiques

Cl Educational Objective:
Item 1 00 Answer: A from t h e American Col l ege o f ' Cardiology a nd t h e American
Heart Associat ion state t hat all patients with an acute coro­
Treat an acute episode of supra­
nary syndrome ( u nstable angi na, NSTE M I . or ST-eleva t ion
ventricular tachycardia.
myoca rdial i n farction) treated medically or w i t h a stent (bare
This pa t i e n t should be given adenosi ne. S he has hemo ­ metal stent or drug-eluting stent) should be given P2Y " i n h i b­
dynamically stable na rrow-co mpl ex tachycard ia consistent itor therapy ( fo r example. clopidogrel. prasugrel. or ticagrelor)
with suprave n t ricular tachycard ia . ll1 e rhyt h m is regular in add i tion to aspirin for at least 12 months.
a nd no obvious P waves are visible : t herefore . a t riovent ric­ Pat i ents who receive a stent in the absence or a n acute
u l a r nodal reciprocat i ng tachycardi a (AVN RT) is the most coronary syndrome ( t hat is, for stable a ngina pectoris) a lso
l i ke ly cause. AVN RT accoun ts for u p to two t h i rds o f cases requ i re dual a n t i p latelet t herapy w i t h aspirin and clopido­
of' supravent ricular tachycardia. Pat ients often report neck grel u n t i l endot h e l i a lization o f the stent is completed and t he
pu lsations. which are caused by si m u l taneous con t ract ion risk for acute stent t h rombosis decreases. For a bare metal
o f t he atria and ven t ricl es. Because t h e patient n1 i led to stent placed u nder t hese c i rcu mstances, clopidogre l should
termi nate her tachycardia w i t h vagal maneuvers. adenosine be con t i nued for at least l mon t h : for a drug-e luting stent.
should be ad m i n istered . Adenosine is h ig h ly e f fective a t ter­ clopidogrel should be co n t i nued for at least l year. ll1ere
m i nation of nodal-dependent rhy t h ms and can help iden t i fy is no i ndication for dual a n t i p l atelet t herapy for less t h a n
t h e underlying et iology. For example. con t i n ued atrial act iv­ I month . N e i t h e r t icagre lor n o r prasugrel h a s been studied
ity ( P waves) d u ring at riove n t ricular block can help iden t i fy extensively in patients u n dergoing coronary stent i m pl a n ­
a t rial f l u tter and atrial tachyca rdia. Patients given adenosi n e tation for stable angina pectoris: t herefore. t hese patients
s h o u l d be on a cardiac monitor w i t h a running rhy t h m strip should be treated w i t h clop idogrel i n addition to aspi ri n .
on paper to document t h e resu l ts. Prior to giving adenosine.
KEY POINT
patients should be warned that t hey may experience nausea.
f l us h i ng. chest pain. or a sense o f' d read . Pa t ients w i t h bron­ • Patients with an acute coronary syndrome should be
chospastic lung disease should not receive adenosine. treated with dual antiplatelet therapy (aspirin and a
Alt hough a m iodarone would be e f lect ive for term i na t­ P2Y 12 inhibitor) for 1 year regardless of initial treat­
i ng t h is pat ient's <l rrhy t h m i a . it has many l ong- term risks, ment approach.
i ncluding t hyroid . l iver, pu l monary, and neurologic toxici ty.
I n t h is young pat ient, a m iodarone would not be an appro­ Bibliography
priate opt i on . 2012 Writing Committee Members. Jneid H, Anderson JL, Wright RS, et al;
Ca rcl ioversion is not i n d icated because t he patient is American College of Cardiology Foundation; American Heart
Association Task Force on Practice Guidelines. 2012 ACCF/AHA focused
hemodynam ical ly stable. a n d pharmacologic a t tempts at update or the guideline for the management of patients with unstable
carcl ioversion. such as adenosine. have not been at tempted . angina/ Non-ST-elevation myocardial infarction (updating the 2007
guideline and replacing the 2011 focused update): a report or the
l bu t i l icle is an intravenous Vaughan-Will iams class I l l American College or Cardiology Foundat ion/ American Heart
an t i a rrhy t h m ic d rug FDA approved for pharmacologic Association Task Force on practice guidelines. Circulation. 2012 Aug
14;126(7):875-910. [PMlD: 22800849]
cardioversion of' a t rial fibri l lati on. ll1e patient has regu lar
supravent ricular tachycardia, not atrial flbril la t ion .

Cl
KEY POINT Item 1 02 Answer: A
• Patients with hemodynamically tolerated supraven­ Educational O bjective: Recognize late complications in
tricular tachycardia refractory to vagal maneuvers a cardiac transplant patient.
should be given adenosine.
This pat ient should undergo coronary angi ogra p hy. H e
underwent heart transplantation 1 0 years ago a n d presents
Bibliography with exertional dyspnea. ll1e two most co m mon causes of
Link MS. Clinical practice. Evaluation and initial treatment or supraven­
dyspnea in post-ca rdiac t ransplant patients are rejection
tricular tachycardia. N Engl J Med. 2012 Oct 11 :367(15):1438-48. [PMID:
23050527] a nd cardiac a l l ograft vascu lopathy. The p revalence of' car­
d iac a l l ograft vascu lopa t hy is approximately sou;., by year s
post - t ra nsp la n t a nd is t h e most common cause of mortal i ty
i n patients a fter t he first year post- t ra nsplan t . Because t he
Cl Educational O bjective:
Item 1 0 1 Answer: c
transplanted heart is denervated at t h e t i m e or tra nsplant .
Manage dual antiplatelet ther­
vascu lopa t hy and subsequent ischemia may occur without
apy in a patient who had a non-ST-elevation myocardial
t h e classic symptoms o f angina . ll1eretore. t h is diagnosis
infarction treated with a bare metal stent.
must be suspected in long-term transplant pat ients present­
A f'u l l year of' clopidogrel t herapy is indicated in t h i s pat ient i ng w i t h symptoms compat ible with ischemia w i thout chest
who has sustained a non - ST-elevation myoca rdial i n farction pain . In t h is patient with exert ional dyspnea 10 years a fter
(NSTEM I ) . Clopidogrel added to aspirin i mproves outcomes transpl a n tation, t he most l i kely cause is cardiac a l l ograft
after hospital ization in pa t ients w i t h NSTE M I rega rdless of t h e vasculopat hy. a n d t here fore proceeding to coronary angiog­
i n- hosp ital t reatment approach . Current recommendations raphy to con firm the diagnosis is t he appropriate next step.

206
Answers a n d Criti q u es

Cl option in lower-risk patients (such as t hose with a relatively


Dobutamine stress echocardiography wou ld be a reasonable it ies (detection and t reat ment of a ven t ri cu l a r tachycar­
dia or ven t ricular f i brillation) . Management of the paci ng
CONT.
recent coronary angiography study) . and defibri l lator functions may d i ffer i n specific c l i n ical
If the patient had u ndergone heart t ra nsplantation si t uations. For example, t h is patient can be consiclerecl
within the past year, t he suspicion for rejection would be pacemaker clepenclent given her h istory of complete heart
h igh . However. t he incidence or rejection after the first year block and atriovent ricular sequential paci ng on her e lec­
is low un less patients are not compliant with their i m muno­ t rocarcliogra m , requ i ri ng con t i n ued pacemaker function
suppressive medications. Therefore. endomyocardial biopsy d ur i ng su rgery. However, her shoulder surgery w i l l be i n
to evaluate for rejection is not t he most appropriate step. close prox i m i ty t o her device a n d w i l l increase the l ikeli­
Because of the patient's signi ficant h istory of tobacco hood of electromagnet ic i n terference that cou ld alter bot h
use, pulmonary function testi ng might be a reasonable con­ the pacing and defibri l lator fu nctions of her device. ·n1ere­
sideration for evaluation of possible u nderlying lung disease. fore. in order to ensure adequate paci ng and avoida nce of
However. t he rapid onset of respiratory symptoms i n a pre­ shocks caused by electrical i n terference associated with
viously asymptomatic patient who is currently a nonsmoker i nstruments used during surgery. t h e device should be
would make t h i s diagnosis less l i ke ly. and test ing wou ld not reprogrammed before the procedure.
be appropriate before excludi ng a cardiac cause. Progrnm m i ng t he device to asynchronous mode ( DOO)
ll1e transplanted heart is denervated. and without t he wi l l allow con t inued pacing of t he atrium and ventricle but
normal vagal tone. a normal heart rate for t ransplant patients wit hout the device sensing the cardiac response, t hereby
is between 90/min a nd llO /m in . Because sinus tachycard ia avoiding suppression of pacing clue to electrical i nterference
may also be present i n patients with pulmonary embol ism. that t he device m ight i nterpret as an elevated heart rate ( that
it may be more cl i mcult to assess tachycardia as a possible is, oversensing) . Disabl ing the shock function will e l i m i nate
presenting sign in patients who are post- transplant. How­ false detec t ion of a tachyarrhy t h m ia due to electrical i n ter­
ever, t h is patient is not at i ncreased risk for pulmonary ference. However, not all patients with pacemaker depen­
embolism. and his heart rate of 102/min is not u nusual and dence requ i re asynchronous pacing during surgery, and
should not increase suspicion for this diagnosis. Therefore. guidelines for device management before. during, and a fter
a vent i lation-perfusion lung scan to test for t h is possibi l i ty surge1y continue to evolve. Because of t h is. it i s advisable to
would not be an appropriate next step. consul t with t he patient's outpatient electrophysiologist in
advance of surgery.
KEY POINT
D isabl i ng t he shock function of an !CD is possible
• The most common cause of reduced left ventricu tar by applying an external magnetic field to the device. The
function in heart transplant patients after the first change in function associated w i t h the application of a
year is cardiac allograft vasculopathy. magnet to an !CD d i ffers from doing so to a pacemaker.
Magnet applica t i o n i n d uces asynchronous pacing (pac­
Bibliography ing rega rdless of what is sensed) in pacemakers. whereas
Toyoda Y. Guy TS. Kashem A. Present status and future perspectives of heart magnet application i n ! CDs d isables t he shocking fu nction
transplantation. Circ J. 2013:77(5):1097- 110. [PMID: 236149631
of t he device w i t hout changing pacing p rogra m m i ng. In
this pat ient. a l though d isab l i ng t he shocki ng function is
appropriate. doing so will not change the device to an
Cl Educational Objective:
Item 1 03 Answer: c
asynchronous mode.
Manage a patient with an
Devices often need to be i nterrogated after surge1y;
i mplantable cardioverter-defibrillator undergoing surgery.
however, this option is i ncorrect because reprogramm ing is
In t his patient with an implantable cardiove11er-defibrillator needed i n this patient before she can proceed to surgery. I n
( !CD) with planned shoulder surge1y, her !CD shou ld be any patient whose device i s reprogram med before surgery,
reprogrammed i m mediately before the procedure to asyn­ the device shou ld be interrogated a fter surgery and con­
chronous pacing with disabling of tachycardia detection and firmed to be "active. "
shocking function. Advising against surgery is i n correct. Pat ients w i t h
In a ny patient with a cardiac implanted electronic i mplanted cardiac devices c a n safely undergo surgery pro­
device. three fundamental q uestions must be answered i n vided the correct precautions are taken. Patients with acute
order t o appropriately determ i ne perioperat ive device man­ arrhyth mias may require stab i l ization. but in genera l , an
agement . First, what type of device does the patient have implanted ca rdiac device in and of itself is not a contraincli­
(for example. pacemaker versus defibri l l ator)? Second. is c<J tion to su rgery or invasive proced u res .
the patient pacemaker dependent? T h ird, will t he surge1y be In summa1y, t h is pa tient is pacemaker dependent. has
performed with i nstruments t hat resu lt in electromagnetic an !CD. and is having surgery in the vici n i ty of her device.
i nterference in t h e vici n i ty of the device or its leads ( for T h erekire. the patient 's ICD requires reprogram m i ng to an
example, electrocautery)? asynchronous mode, disabl i ng of the tachytherapies, and
There i s often confusion about pacemakers and appropriate device interrogation and reprogram m i ng after
defibrillators. A n ICD is a pacemaker w i t h extra capabi l- the surgery.

207
Answers a n d C ritiq u es

KEY POINT KEY POINT


• Patients with an implantable cardioverter-defibrillator • Ostium primum atrial septa! defect is characterized
who are pacemaker dependent often require repro­ by the combination of fixed splitting of the S2, a mitral
gramming of their device before surgery. regurgitation murmur, and left axis deviation on the
electrocardiogram.
Bibliography
Crossley GH. Poole JE. Rozner MA. et al. The Heart Rhythm Society (HRS) / Bibliography
American Society of Anesthesiologists (ASA) Expert Consensus
Warnes CA, Williams RG, Bashore TM. et al: American College of Cardiology:
Statement on the perioperative management of patients with implanta­
American Heart Association Task Force on Practice Guidelines (Writing
ble defibrillators, pacemakers and arrhythmia monitors: facilities and
Committee to Develop Guidelines on the Management of Adults With
patient management. This document was developed as a joint project
Congenital Heart Disease); American Society ofEchocardiography: Heart
with the American Society of Anesthesiologists (AS). and in collabora­
Rhythm Society; International Society for Adult Congenital Heart
tion with the American Heart Association (AHA). and the Society of
Disease; Society for Cardiovascular Angiography and Interventions:
Thoracic Surgeons (STS). Heart Rhyt hm. 2011 Jul:8(7) : 1 1 1 4-54. [PMID:
Society ofThoracic Surgeons. ACC/AHA 2008 guidelines for the manage­
21 722856]
ment of adults with congenital heart disease: a report of the American
College of Cardiology/American Heart Association Task Force on Practice
Guidelines (Writing Committee to Develop Guidelines on the
Management of Adults With Congenital Heart Disease) . Developed in
Item 1 04 Answer: B Collaboration With the American Society of Echocardiography, Heart
Rhythm Society. International Society for Adult Congen ital Heart
Edu cational Objective: Diagnose ostium primum atrial Disease. Society for Cardiovascular Angiography and Interventions. and
septa) defect .
Society of Thoracic Surgeons. J Am Coll Cardiol. 2008 Dec 2;52(23):el43-
263. [PMID: 190386771
The most likely diagnosis is ostium primum atrial septa!
defect (ASD) . The patient has clinical features of an ASD,
presenting with dyspnea, previous atrial fibrillation, and Item 1 05 Answer: A
features of volume overload of the right heart with elevation
Educational Objective: Manage ventricular septa!
of the central venous pressure and a right ventricular lift.
defect complicating myocardial infarction.
A systolic murmur at the base results from increased flow
across the right ventricular outflow tract from the left-to­ T h i s patient has developed a c l i n ica l ly sign i ficant ven­
right shunt. Fixed splitting of the S2 throughout the cardiac t ri cu l a r septa! defect (VSD) and is in cardioge n ic shock:
cycle is a characteristic clinical feature of ASD. The apical sys­ he req u i res emergency surgery to repa i r the defec t . VSD
tolic murmur is from mitral regurgitation and is related to the m a n i fests a s hemodyna m ic comprom ise in l he set t i ng of a
mitral valve cleft. The electrocardiogram demonstrates left new holosystol ic murmur 3 lo 7 days a fter an i n i t ia l myo­
axis deviation, first-degree atrioventricular block, and inter­ card i a l i n fa rction ( M l ) . A l t hough su rg ical mort a l ity is h igh .
ventricular conduction delay. This combination of findings, u rgen t card i ac surgery (coronary artery bypass gra ft i ng
including fixed splitting of S2, mitral regurgitation murmur, and VSD patch repa i r) offers t h e on ly cha nce of surv iva l
and left axis deviation on the electrocardiogram, are most fo r pat ients w i t h l a rge post-i n farct ion VSDs. especia l ly i n
consistent with an ostium primum ASD. t h e presence o f cardiogen i c s hock. There is n o consensus
Patients with coronary sinus ASD have features of right on the o p t i m a l t i m i ng of' su rgery as ea rly s urgica l rep a i r is
heart volume overload but do not have mitral valve dis­ cl i f li cu l l because of i n larctecl myoca rd i u m . wh ich is often
ease and thus will not have a murmur of mitral regurgita­ friable and m a kes surgical cl osure of t h e se p t a l d e fect
tion. The electrocardiogram may be normal or demonstrate cl i flic u l l . ·n1ere have been several reports of' successfu l per­
first-degree atrioventricular block and incomplete right cuta neous closure o f' VSDs . but a natomy of t h e VSD and
bundle branch block. c l i n ical expert ise l i m i t widespread use o f' t h i s proced u re
Patients with ostium secundum ASD have features of i n a l l pat i e n ts.
right heart volume overload but do not have mitral valve Acute ven t ricular f'ree wal l rupture also occurs 3 lo
disease and thus will not have a murmur of mitral regurgita­ 7 clays a fter Ml and has a h igh m orta lit rate. Pat ients most
y

tion. The electrocardiogram may be normal or demonstrate com mo n ly presen t w i t h pericardia! tamponacle (due to
first-degree atrioventricular block and incomplete right bun­ hemopericardium ) . pulseless electrical activity, and deat h.
dle branch block. Left axis deviation on the electrocardio­ Emergen t pericarcl iocentesis and subsequent surgical recon­
gram is not found in patients with an ostium secundum ASD. st ruc t i o n cc111 i m p rove surviva l . T h e echoca rd iographic r i nd ­
Patients with patent foramen ovale have a normal echo­ i ngs in this patient did not reveal pericardia! e ffusion or
cardiogram and physical examination. These patients are tamponade. and h is presen tation is consistent with VSD.
also generally asymptomatic. ll1 is patient has an occluded left a nterior descend i ng
Patients with sinus venosus ASD have features of right artery and persistent ST- segmen t elevation : however. the
heart volume overload but do not have mitral valve disease presence o f' hemodynamic comprom ise and echocard io­
and thus will not have a murmur of mitral regurgitation. graphic evidence of VSD requi res emergency cardiac sur­
The electrocardiogram may be normal or demonstrate first­ gery. A l t hough coronary artery bypass gra fl i ng is usual ly
degree atrioventricular block and incomplete right bundle performed during an attempted repa i r of the septa I defect .
branch block. percutaneous coronary i nterve n t ion o f' t he left a n terior

208
Answers a n d Critiques

CJ d escen di ng artery is not ind icated once th e \/SD has b ee n not have evidence of dyssynchrony or an ejection fraction of
d i agnosed. 35% or less, she is not a candidate for treatment.
CONT.
The performance of right heart catheterization may aid
KEY POINT
i n the d iagnosis of \/SD. and the use ofa vasopressor such as
dopam in e may help i n i t i a l ly stab i l i ze the patien t . However, • Hydralazine and isosorbide dinitrate improve symp­
t h js patient has a severe hemodynamic impa i rment and toms and reduce mortality in patients with New York
requires emergency cardiac surgery. Heart Association class I I I or IV heart failure symp­
toms who are black and are already on maximal
K EY P O I N T
therapy.
• Urgent cardiac surgery offers the only chance o f sur­
vival for patients with large postinfarction ventricular Bibliography
septal defects, especially in the presence of cardio­ Flack JM, Sica DA, Bakris G, et al; International Society on Hypertension in
genic shock Blacks. Management of high blood pressure in Blacks: an update of the
International Society on Hypertension in Blacks consensus statement.
Hypertension. 2010 Nov;S6(5):780-800. [PMID: 20921433]
Bibliography
Van de Werf F, Bax J, Betriu, A, et al; ESC Committee for Practice Guidelines
(CPG). Management of acute myocardial infarction in patients presenting

CJ
with persistent ST-segment elevation: the Task Force on the Management Item 1 07 Answer: D
of ST-Segment Elevation Acute Myocardial Infarction of the European
Society of Cardiology. Eur Heart J. 2008 Dec;29(23):2909-45. [PMID: Educational Objective: Evaluate for suspected perival­
19004841] vular abscess in a patient with infective endocarditis.

·111 is patient should u ndergo t ra n sesophageal echocar­


Item 1 06 Answer: A d i ography (TEE) . He has evi dence of a new conduction
defect on e lectrocard iogram suggesting a possible perival­
Educational Objective: Treat a black patient with heart
vular abscess comp l icating Staphy lococcus a u reus i n fec­
failure with hydralazine and isosorbide dinitrate in addi­
t ive endocard i t i s. Perivalvul a r abscesses may be present
tion to usual therapy.
in 30% to 40% o f patients w i t h i n fective endocarditis, and
TI1is patient should have hydralazine and isosorbide dinitrate the risk 111ay be further increased in those w i t h a bicuspid
added to her medication regimen for the treatment of her aortic valve . The d iagnosis should be considered i n patients
heart failure. She has New York Heart Association (NYHA) with pers i s tent bactere111ia. fever, or evidence o f conduction
functional class III heart failure and is black Hydralazine syste111 d isorders who are being treated w i t h appropriate
and isosorbide dinitrate have been demonstrated to improve a n t i b i o t ic t herapy. TEE has a h igh sen sitivity and spec i fi c i ty
symptoms and reduce mortality in patients who are black for iden t i fying perivalvular extension of i n fection and is t h e
and who are already on maximal therapy with NYHA class III d iagnosti c study of choice w h e n a periva lvular abscess i s
or IV heart failure symptoms. Adverse effects of this therapy suspected .
include peripheral edema and headaches, but this regimen Cardiac CT has not been extensively s tu d i e d for use
should be attempted in these patients. in d i agnosing myocardi a l i n fection and is not typi ca l ly
Optimal therapy for patients with heart failure includes used for t h i s p u rpose. Cardiac m agnetic resonance ( C M R)
treatment with an ACE inhibitor, �-blocker, and an aldo­ i magi ng, however. i s e f fective in i d e n t i fying i n t ramyocar­
sterone antagonist. The addition of an angiotensin receptor d ial i n fect ion . a l t hough it is a more complex technology
blocker, such as losartan, to this combination is generally t hat may have l i m i ted ava i labi l i ty in some areas. C M R
not recommended, primarily because of concern for hyper­ i maging is o ften used in s i t u a t i ons i n which a perivalvular
kalemia. Additionally, no benefit to this treatment regimen abscess i s suspected but t ransesophageal echocard i ogra­
has been documented. It would t herefore not be appropriate phy is equ ivocal.
in this patient. Evidence of a conduction system disorder may be t he
In patients with heart failure, warfarin treatment is o n ly indicator of a perivalvular abscess i n a patient o ther­
appropriate only in those with another indication, such as wise respondi ng c l i nica l ly to treatment for i n fective endo­
atrial fibrillation meeting CHA2 DS2 -VASc criteria, but not card i tis, as in t his patient. Because of t he signi ficance o f t h i s
with heart failure alone. The routine treatment of patients possible compl ication. fai lure to further eva luate conduction
with heart failure with warfarin is not indicated. system abnormalities i n t h i s setti ng woul d be inappropria te.
Cardiac resynchronization therapy (CRT) may be an A l though t ranst horacic echocardiography is effective for
effective therapy in patients with heart failure and a pro­ i n i t i a l eva l ua tion for endocard i t i s and assessing for poten­
longed QRS duration indicating dyssynchrony. Indications tial complications once enclocard i t i s has been diagnosed
include a left ventricular ejection fraction of 35% or less i n (such as valvu lar or left ventricu lar dysfunction ) , TEE i s sig­
patients in sinus rhythm, with a left bundle branch block, n i fica n t ly more sensi t ive for detecting periva lvular abscess
and with a QRS duration of150 ms or greater in whom mod­ because of the closer proximity of the ultrasoun d probe to
erate to severe symptoms (NYHA class III or IV) are present the valve structures. It is t herefore preferred to transthoracic
despite optimal medical therapy. Because this patient does echoca rd iography if t h i s d i agnosis is a considera tion.

209
Answers a n d Critiques

KEY POINT ade and occasionally constrictive pericarditis), asthma, and


COPD. This patient, however, does not have a history sugges­
• Transesophageal echocardiography is the diagnostic
tive of these disorders, and evaluation for pulsus paradoxus
study of choice in patients with a possible perivalvular
would not be diagnostically helpful.
abscess complicating infective endocarditis.
Thoracic outlet maneuvers are used to evaluate for
neurovascular impingement at the point where the upper
Bibliography extremity neurovascular bundle exits the thoracic cavity.
American College of Cardiology Foundation Appropriate Use Criteria Task
Although thoracic outlet syndrome could be responsible
Force; American Society of Echocardiography; American Heart
Association; American Society of Nuclear Cardiology; Heart Failure for his vascular symptoms, the obstruction occurring with
Society of America; Heart Rhythm Society; Society for Cardiovascular this syndrome is distal to the vertebral artery and would not
Angiography and Interventions: Society of Critical Care Medicine;
Society of Cardiovascular Computed Tomography: Society for explain his accompanying neurologic symptoms.
Cardiovascular Magnetic Resonance, Douglas PS, Garcia MJ, Haines DE,
et al. ACCF/ASE/AHA/ASNC/HFSA/HRS/SCAl/SCCM/SCCT/SCMR 2011 KEY POINT
Appropriate Use Criteria for Echocardiography. A Report of the American
College of Cardiology Foundation Appropriate Use Criteria Task Force, • A hallmark feature of upper extremity peripheral
American Society of Echocardiography, American Heart Association, arterial disease on physical examination is a differ­
American Society of Nuclear Cardiology, Heart Failure Society of
America, Heart Rhythm Society. Society for Cardiovascular Angiography ence in systolic blood pressures between the arms
and Interventions, Society of Critical Care Medicine, Society of (typically >15 mm Hg) .
Cardiovascular Computed Tomography, and Society for Cardiovascular
Magnetic Resonance Endorsed by the American College of Chest
Physicians. J Am Coll Cardiol. 2011 Mar 1;57(9) : 1 126- 66. [PM!D: 21349406] Bibliography
Clark CE, Taylor RS, Shore AC, Ukoumunne OC, Campbell JL. Association of
a di fference in systolic blood pressure between arms v.�th vascular dis­
Item 1 08 Answer: B ease and mortality: a systematic review and meta-analysis. Lancet. 2012
Mar 10;379(9819):905-14. Erratum in: Lancet. 2012 Jul 21;380(9838) :218.
Educational Objective: Diagnose upper extremity [PM!D: 22293369]
peripheral arterial disease.

Measurement of systolic blood pressure in both arms is indi­ Item 1 09 Answer: D


cated in this patient at high risk for atherosclerotic cardio­
Educational Objective: Manage first-degree atrioven­
vascular disease (ASCVD) who likely has upper extremity
tricular block accompanied by bifascicular block.
peripheral arterial disease (PAD) as the cause of his symp­
toms. His extremity symptoms are consistent with impaired No intervention is indicated. This octogenarian has first­
arterial blood flow with exertion causing claudication in the degree atrioventricular (AV) block (PR interval >200 ms) ,
left arm, and his associated neurologic symptoms suggest the right bundle branch block, and left posterior fascicular block.
subclavian steal syndrome. In subclavian steal, a stenosis in The combination of first-degree AV block and bifascicular
the left subclavian artery proximal to the take-off of the ver­ block is often referred to as "trifascicular block, " although
tebral artery results in retrograde blood flow from the central this term is not correct because true trifascicular block would
nervous system to the arm, resulting in neurologic symp­ imply complete AV block. There is no indication for pacing in
toms. A hallmark feature of upper extremity PAD on phys­ patients with first-degree AV block and bifascicular block who
ical examination is a difference in systolic blood pressures have no symptoms due to bradycardia. The risk of this type of
between the arms, with a differential of greater than 15 mm block progressing to complete heart block is less than 2% to
Hg being typical. Differences in the distal pulses (upstroke 3% per year.
and duration) may also be noted when comparing the upper The patient is asymptomatic, has no evidence of left
extremities, and a bruit detected over the subclavian artery ventricular dysfunction, and no clinical evidence of isch­
may be present. Noninvasive vascular testing with Doppler emia. Therefore, stress testing, including adenosine perfu­
ultrasonography may be confirmatory. Upper extremity PAD, sion imaging, is not indicated. Because right coronary artery
whether symptomatic or asymptomatic, is a marker for sig­ ischemia can cause paroxysms of bradycardia, stress testing
nificant ASCVD and is associated with an increased risk for for ischemia may be indicated in patients with paroxysmal
future cardiovascular disease. Therefore, primary treatment is bradycardia or clinical findings suggestive of ischemia, nei­
aggressive therapy for ASCVD, including antiplatelet therapy. ther of which is present in this patient. In patients in whom
Treatment for clinically symptomatic upper extremity PAD, symptomatic bradycardia is a concern, treadmill stress test­
as in this patient, may include angioplasty with stenting or ing may be helpful in evaluating whether there is an appro­
surgical bypass. priate chronotropic response to exercise.
The ankle-brachia! index is used to assess for lower Permanent pacing is only indicated in two clinical situa­
extremity PAD and might be abnormal in this patient with tions. First, pacemakers are indicated in patients with symp­
likely ASCVD, although this test would not be helpful in tomatic bradycardia in the absence of a reversible cause.
evaluating his upper extremity symptoms. Second, pacemakers are indicated in patients with evidence
Pulsus paradoxus is an exaggerated drop in systol ic of AV conduction disturbances that have a high likelihood
blood pressure (>10 mm Hg) during inspiration and may be of progression to complete heart block or life-threatening
present in patients with severe pericardia! disease (tampon- sudden asystole. Second-degree Mobitz II block, alternating

210
Answe rs a n d Critiq u e s

bundle branch block, atrial fibrillation with pauses greater Association guidelines d o not currently provide recommen­
than 5 seconds, and complete heart block are all indications dations for use of fish oil after MI. Fish oil is effective in
for permanent pacing. This patient has none of these condi­ reducing triglyceride levels; however, this patient' s triglycer­
tions, and neither dual-chamber nor single-chamber pacing ide levels are normal.
is indicated. Recently released guidelines recommend treatment
of patients with established atherosclerotic disease with a
K EY P O I NT
high-intensity statin with a goal of lowering the LDL choles­
• Asymptomatic first-degree atrioventricular block with terol level to less than 50% of the baseline level but without
bifascicular block does not require pacemaker treatment to a specific LDL cholesterol level. As this patient
implantation. has had the expected decrease in LDL cholesterol level on her
present regimen, the addition of another agent for managing
Bibliography dyslipidemia, such as niacin, would not be appropriate.
Epstein AE. DiMarco J P. Ellenbogen KA. et al; American College of Although moderate alcohol consumption (approximately
Cardiology/American Heart Association Task Force on Practice
Guidelines (Writing Committee to Revise the ACC/AHA/NASPE 2002 one to three drinks daily) is associated with a lower risk of
Guideline Update for Implantation of Cardiac Pacemakers and coronary heart disease, excessive alcohol intake accounts for
Antiarrhythmia Devices): American Association for Thoracic Surgery:
approximately 4% of cases of dilated cardiomyopathy. How­
Society of Thoracic Surgeons. ACC/AHA/HRS 2008 Guidelines for
Device-Based Therapy of Cardiac Rhythm Abnormalities: a report of the ever, reducing this patient's current level of alcohol consump­
American College of Cardiology/American Heart Association Task Force tion will not reduce her risk of a future cardiovascular event.
on Practice Guidelines (Writing Committee to Revise the ACC/AHA/
NASPE 2002 Guideline Update for Implantation of Cardiac Pacemakers
KEY POINT
and Antiarrhythmia Devices): developed in collaboration with rhe
American Association for Thoracic Surgery and Society of Thoracic • Patients with prior myocardial infarction should
Surgeons. Circulation. 2008 May 27:117(2 l ) :e350 -408. Erratum in:
Circulation. 2009 Aug 4; 120(5):e34-5. [PMID: 1 84832071 receive an ACE inhibitor for secondary cardiovascular
prevention.

Bibliography
Item 1 1 0 Answer: D
Qaseem A. Fihn SD. Dallas P, Williams S, Owens DK. Shekelle P: Clinical
Educational Objective: Manage secondary risk reduc­ Guidelines Committee of the American College of Physicians.
Management of stable ischemic heart disease: summary of a clinical
tion in a patient with established coronary artery disease.
practice guideline from the American College of Physicians/American
College of Cardiology Foundation/American Heart Association/American
The intervention that offers the greatest cardiovascular Association for Thoracic Surgery/Preventive Cardiovascular Nurses
risk reduction for this patient is to start an ACE inhibitor Association/Society of Thoracic Surgeons. Ann Intern Med. 2012 Nov
20:157(10):735-43. [PMID: 231656651
for secondary prevention after myocardial infarction (Ml)
and to reduce systolic blood pressure. ACE inhibitors have

Cl
been shown to decrease both cardiovascular and all-cause Item 1 1 1 Answer: A
mortality in patients with chronic ischemic heart disease,
Educational Objective: Manage revascularization in a
especially in those patients with prior MI, left ventricular
patient with an acute coronary syndrome with a high TIMI
systolic dysfu nction, or heart failure. Guidelines from the
risk score and muitivessel disease.
American College of Physicians/American College of Cardi­
ology Foundation/ American Heart Association recommend 1l1is patient should undergo coronary artery bypass graft
a treatment goal of 140/90 mm Hg or below in patients (CABG) surgery. In patients with a non-ST-elevation acute
with stable ischemic coronary heart disease. Although pre­ coronary syndrome (unstable angina or non-ST-elevation
vious guidelines recommended treatment to reduce blood myoca rdial infarction) . the TIMI risk score is used to deter­
pressure to below 130/80 mm Hg, there is limited evidence mine whether a conservative strategy or an early invasive
to show a benefit of doing so, with the potential for adverse strategy is warranted. This patient has several TIMI risk fac­
consequences owing to overtreatment of blood pressure in tors. including aspirin use in the past week, ST-segment
these patients. The 2014 Eighth Joint National Commission deviation. elevated biomarkers. more than three traditional
(JNC-8) report does not provide specific recommendations coronary artery disease (CAD) risk factors, and documented
for treatment of patients with established atherosclerotic CAD with greater than or equal to 50% diameter stenosis;
disease, although the recommended threshold for treat­ therefore. an early invasive strategy is warranted. In this
ment for all patients younger than 60 years is also 140/90 patient, an oral P2Y 11 inhibitor (clopidogrel. prasugrel, tica­
mm Hg. Therefore, this patient would be expected to ben­ grelor) was not administered, but if this had been given, the
efit from therapy to decrease her blood pressure to at least surgery should be del ayed S days to allow discontinuation
140/90 mm Hg or below. and excretion of the antiplatelet medication.
While there is observational evidence from the GISSI Intra-aortic balloon pump placement may be considered
(Gruppo Italiano per lo Studio della Soprawivenza nell'In­ for patients with recurrent cardiac ischemia and poor left ven­
farto M iocardico) Prevention study that patients with prior tricular function. However. although this patient had recur­
MI who take fish oil have a 20% reduction in mortality rate, rent chest pain during hospitalization, his symptoms improved
the current American College of Cardiology/American Heart with intravenous nitroglycerin and medical therapy, and he

21 1
An swers a n d Criti q u es

Transca t he l er aortic valve replacem e n t (TAVR) is i n cl i ­


Cl balloon pu111p place111enl is not indicated.
has re111ained hemodynamically stable. Tl1erefore. intra-aortic
cated for patients w i t h severe symptom a t i c aortic stenosis
CONT.
Because t h i s patient has m u l t ivessel disease a nd a who are considered unsuitable for conve n t i onal surgery
reduced left ven tricular eject ion fraction . he should u ndergo because of severe com orbi d i t ies. Candidates for TAVR must
CABG rather than percu taneous coronary i nterven t ion. be carefu l ly selected . Surgical risk should be assessed
I n patients with lef'l m a i n coronary a rtery stenosis. object ively. such as by using t he Society o f' T horacic Sur­
depressed left ven tricu lar function. and an acute coronary geons adu l t card i ac risk score (STS score) ( h ttp : riskca l c.
syndrome. the optimal t reatment strategy includes revascu­ sts. org/STSWebR i s kCalc273 /de. aspx) . Patients w i t h a n STS
larization rather than medical t herapy. risk score of grea ter t ha n or equal to 8% may be candidates
for TAVR . I n addition . TAVR is not approved in pa t i e n ts
K EY P O I N T
w i t h conco m i ta n t valve d i sease (such as sign i Aca n t aortic
• Patients with multivessel coronary disease and a regurgitation o r m i t ral valve disease) a n d a bicuspid aortic
reduced left ventricular ejection fraction should valve. Tl1 is patient has a bicuspid aortic va lve and moderate
undergo coronary artery bypass graft surgery rather aortic regu rgitation : there fore. she wou l d not be a ca ndi­
than percutaneous coronary intervention. date for TAVR.
K EY P O I NT
Bibliography
Amsterdam EA. Wenger NK, Brindis RG. et al: ACC/AHA Task Force • Surgical aortic valve replacement i s the treatment of
Members. 2014 AHA/ACC guideline for the management of patients with choice for most patients with symptomatic severe aor­
non-ST-elevation acute coronary syndromes: executive summary: a
report of the American College of Cardiology/ American Heart Association tic stenosis and is associated with low mortality rates
Task Force on Practice Guidelines. Circulation. 2014 Dec 23;l30 (25) : 2354- for patients younger than 70 years (1 %-3%) .
94. [PMID: 25249586]

Bibliography

Cl Educational Objective:
Item 1 1 2 Answer: c Lindman BR, Bonow RO, Otto CM. Current management of calcific aortic
stenosis. Circ Res. 2013 Jul S;ll3(2):223-37. [PM ID: 23833296]
Treat a patient with severe cal­
cific aortic stenosis and aortic regurgitation with surgical
valve replacement. Item 1 1 3 Answer: D
Educational Objective: Diagnose a left atrial myxoma.
Tl1 is pa t i e n t s hould u n dergo surgical aortic valve replace­
men t. Surgica l aortic valve rep lacement is t he o n ly t reat­ This patient most likely has a myxoma that is causing her
ment of aortic stenos i s associ ated w i t h a survival beneft t symptoms and clinical findings. Left atrial myxomas are the
a n d d u rable sym p tom rel i e f'. Surgical aortic valve rep lace­ most common benign tumors of the heart. These lesions can
ment is the t reatm e n t of c ho ice for most pat ien t s w i t h cause constitutional symptoms, such as fatigue, dyspnea,
sym ptomal ic severe aortic stenosis a nd i s associated vvi l h fever, and weight loss, related to tumor cytokine produc­
low mort a l i ty rates for p a t i e n ts younger than 70 years tion; systemic embolization from either tumor fragments
( l'Y.,- 3 %) or associated thrombi may cause neurologic symptoms or
Aort ic valve rep a i r i s a n option i n a l i mi ted nu mber other systemic sequelae. Left atrial myxomas most com­
of adu l t patients w i t h aortic valve disease. In genera l . it is monly appear as a mass arising from the atrial septum; the
restricted lo pa tients w i t h aortic regurgitation a nd :rna­ mass can involve the mitral valve intermittently to cause
tomica l ly favorable aortic valve and root a natomy and can a "tumor plop" that may clinically mimic mitral stenosis.
range from si mple cusp plication to complex valve-sparing Myxomas may occur as part of the Carney complex, which
aortic root repl aceme n t . Tl1is pat ient has severe cal c i A c is an autosomal dominant disorder associated with pigmen­
aort i c stenosis a n d a va lve t h a t is u n l i ke ly to b e amenable tation abnormalities (such as blue nevi) , schwannomas, and
to repair. endocrine tumors.
Balloon valvu loplasty. a l though import a n t i n the t reat­ Metastatic adenocarcinoma with cardiac involvement
ment or ped iat ric patients w i t h severe aort ic stenosis. has a can manifest from direct invasion or hematogenous spread,
more l i m i ted role in adults. e i t her as a bridge to deAn i t ive with symptoms and signs dependent on the site of involve­
t rea l m c n l . to d i lTcrc n l i a l c clyspnea symptoms i n h igh - ris k ment. Metastatic adenocarcinoma with cardiac involvement,
patients with comorbid conditions such as COPD, or to treat although common in patients with this tumor type (15% of
patients w i t h calciAc aortic stenosis w i t h hemodyna m ic patient at autopsy) , would be less likely in this patient given
i nstabi l i ty or decompensation . Whi l e bal loon valvuloplasty the isolated anatomic location and recent negative malig­
is a potential considera t ion for t h i s patient. the presence nancy screening results.
of sign i Aca n t aortic regurgitation is a con t rai nd ication . Angiosarcomas are malignant tumors that can occur in
I mprovement i n aortic valve area from this procedure is the atria but are less common than myxomas and typically
modest. and many patients have residual severe aort ic stc­ inAltrate the myocardium, which is normal in this patient.
nosis i m mediately after valvu loplasty. Bal l oon valvuloplasty Lipomas typically are located in the subendocardium,
would not be the best option for t h is patient. not the atrium, and rarely cause symptoms.

21 2
Answers a n d Critiques

Papillary fibroelastomas, like myxomas, can be mobile KEY POINT


with a pedunculated stalk. However, they most commonly
• Patients with acute pericarditis who do not have high­
arise on left-sided cardiac valves and the left ventricular
risk features (fever, leukocytosis, acute trauma, abnor­
outflow tract.
mal cardiac biomarkers, immunocompromise, oral
KEY POINT anticoagulant use, large pericardial effusions, or evi­
• The most common benign tumors of the heart, left dence of cardiac tamponade) can be managed medically
atrial myxomas can cause constitutional symptoms, on an outpatient basis with close clinical follow-up.
such as fatigue, dyspnea, fever, and weight loss,
related to tumor cytokine production and neurologic Bibliography
symptoms related to systemic embolization from lmazio M, Brucato A, Cemin R, et al; ICAP Investigators. A randomized trial
ofcolchicine for acute pericarditis. N Eng!J Med. 2013 Oct 17;369(16):1522-
either tumor fragments or associated thrombi. 8. [PMID: 23992557]

Bibliography
Shapiro LM. Cardiac tumours: diagnosis and management. Heart. 2001
Feb;85 (2):218-22. [PMID: 11156679]
Item 1 1 5 Answer: C
Educational Objective: Manage a patient with
Wolff-Parkinson-White syndrome with syncope.
Item 1 1 4 Answer: B
This patient should undergo an electrophysiology study. He
Educational Objective: Manage acute pericarditis on an
has evidence of pre-excitation on his electrocardiogram with
outpatient basis.
a history of palpitations and syncope. The slurring of the
This patient should receive clinical follow-up without hos­ QRS complex (delta wave) represents early ventricular depo­
pital admission or further diagnostic testing to monitor her larization owing to conduction over the accessory pathway
response to therapy, evaluate for possible complications, and (bypass tract) . The presence of a delta wave and symptoms
assess the timing for tapering her medications. Slow tapering of tachycardia are consistent with Wolff-Parkinson-White
over 2 to 4 weeks after initial presentation with improvement syndrome. The episodes could be caused by supraventric­
in symptoms is usually performed to reduce the risk of recur­ ular tachycardia (orthodromic or antidromic reciprocating
rent inflammation. tachycardia) or pre-excited atrial fibrillation. The presence
The vast majority of patients with acute pericarditis, of syncope suggests that these episodes are hemodynami­
including the patient presented, can be managed medically cally significant. Identification of syncope in a patient with
on an outpatient basis. For a subset of patients, high-risk fea­ Wolff-Parkinson-White syndrome should prompt referral to
tures of acute pericarditis may be present and warrant hos­ a cardiologist or electrophysiologist.
pitalization for treatment and monitoring for possible com­ An electrophysiology study would allow diagnosis of the
plications; these include fever, leukocytosis, acute trauma, cause of this patient's palpitations and allow risk stratifica­
abnormal cardiac biomarkers, an immunocompromised tion for risk of sudden cardiac death. The electrophysiology
host, oral anticoagulant use, large pericardia! effusions, or procedure also affords the opportunity to ablate the acces­
evidence of cardiac tamponade. sory pathway and potentially cure his arrhythmia. Stress
CT can be used to show pericardia! thickening in testing can be an appropriate method for risk stratification
patients with acute pericarditis. However, this finding would in patients with asymptomatic pre-excitation; however, this
not change the diagnosis or appropriate management strat­ patient clearly has symptoms and therefore should undergo
egy in this patient. invasive testing and ablation.
Medical therapy with anti-inflammatory agents is Antiarrhythmic drug therapy is not indicated in this
appropriate for acute pericarditis. However, glucocorticoids patient because the type and mechanism of the arrhyth­
are reserved for patients who do not respond to NSA!Ds, such mia are not known. Catheter ablation is preferred in young
as ibuprofen, aspirin, and indomethacin, none of which has persons with Wolff-Parkinson-White syndrome in order to
been tried yet in this patient. Glucocorticoid therapy may avoid lifelong use of potentially toxic medications. Antiar­
also increase the risk of recurrent pericarditis and should rhythmic agents are reserved for second-line therapy, par­
only be considered in highly selected patients with refrac­ ticularly in patients with accessory pathways located close
tory pericarditis. to the atrioventricular (AV) node.
Pericardiocentesis is indicated only for patients with Metoprolol and diltiazem are AV nodal blockers and
tamponade or for those in whom the analysis of pericardia! may be unsafe if the patient has anterograde conduction
fluid can be of assistance in diagnosis and management. down the accessory pathway during atrial fibrillation. These
Signs of tamponade are not present in this patient whose drugs can block the AV node and promote rapid 1:1 conduc­
inferior vena cava is normal in size on echocardiography, tion from the atrium to the ventricle during atrial fibrillation
whose Doppler ultrasound shows minimal change in mitral and thus induce ventricular fibrillation. AV nodal blockers
inflow with respiration, and whose bedside maneuvers are contraindicated in patients with pre-excited atrial fibril­
reveal no pulsus paradoxus. lation, such as this patient.

21 3
Answers and Critiques

KEV P O I NT Bibliography
Fermann GJ. Collins S P. Initial management of patients with acute
• Identification of syncope in a patient with pre-excita­ heart failure. Heart Fa il Clin. 2013 J u l : 9 (3) : 2 9 1 - 3 0 1 . vi. [ P M ! D :
tion should prompt referral to a cardiologist or elec­ 23809416]

trophysiologist.

Item 1 1 7 Answer: B
Bibliography
Delacretaz E. Clinkal practice. Supraventricular tachycardia. N Engl J Med. Educational Objective: Appropriately perform surveil­
2006 Mar 9;354(10):1039-51. [PM!D: 16525141] lance imaging in a patient with Marfan syndrome and aor­
tic root dilation.

Item 1 1 6 Answer: c This patient with Marfan syndrome should undergo sur­
Educational Objective: Consider reversible causes in veillance imaging annually. Dilation of the ascending aorta
the evaluation of heart failure. is a systemic feature of Marfan syndrome, and the most
life-threatening complication of Marfan syndrome is aortic
The most appropriate diagnostic test to perform in this young
aneurysm, which can lead to an acute aortic syndrome (aor­
patient with new-onset heart failure is to obtain thyroid
tic dissection, rupture, or both) . Accordingly, examination
studies. This patient exhibits signs and symptoms consistent
of the ascending aorta and heart valves is mandatory i n
with a diagnosis of hyperthyroidism, including tachycardia,
patients with Marfan syndrome. The severity of aortic dis­
a hyperdynamic precord ium . palpitations, weight loss. and
ease is in relation to the extent of aortic dilation, the length
loose stools. Hyperthyroidism is a we l l-described, reversible
of the dilated segment, and the location of aortic involve­
cause of heart failure due to cardiac overstimulation by excess
ment. Most patients with Marfan syndrome present with
thyroid hormone that resembles sympathetic stimu lation .
enlargement of the ascending aorta; therefore, serial exam­
Hyperthyroidism causes an increase in heatt rate and myocar­
ination is focused mainly on assessing this portion of aorta.
dial contractility; systemic vascular resistance o ften decreases
American College of Cardiology Foundation/American Heart
and may result in a widened pulse pressure. Hypothyroidism
Association guidelines recommend follow-up imaging 6
is also a known cause of hea1t fai lure, alt hough it would be
months after diagnosis, with annual surveillance thereafter
less l i kely in this patient with symptoms more consistent with
if the aortic root is less than 4.5 cm in diameter and other­
excess thyroid hormone. Because thyroid function abnormal­
wise stable. This threshold is lower than for patients with an
ities are a potentially reversible cause of hearl failure, assess­
aortic aneurysm due to other causes because of the tendency
ment of t hyroid function should be considered in patients
for complications in patients with Marfan syndrome with an
with new-onset heart fail ure and clinical fi ndi ngs suggestive
aortic root diameter above this level. If the aortic root diam­
of thyroid dysfunction .
eter is 4.5 cm or greater or if the aortic root diameter shows
Evalu ation of unusual causes of heart fa i l u re s h o u l d
significant growth from baseline, more frequent imaging of
not b e performed rou tinely but shou l d be pursued when
the aorta should be considered.
there are suggestions o f specific diseases by h i story or phys­
I maging of the aortic root in patients with M arfan
ical examin ation . TI1e pa tient has no signs or symptoms
syndrome is usually performed with transthoracic ultra­
suggesting a rheumatologic d isorder, and rout ine screen­
sound because i t is able to accurately evaluate this por­
ing with an an tinuclear anti body level is not indicated .
tion of the aorta and is noninvasive. However, for aneu­
S i m i l a rly. t h is patient does not have a history of fl u - l i ke
rysms above the aortic root, CT o r MRI i s p referred a s
symptoms suggesting a viral etiology, making the poten ­
t h e y more accurately measure t h e aort ic d imensions i n
tial yield of v i ral titers quite low. Furthermore. d i rected
t h a t region.
treatment options in the presence of posit ive viral t i ters a re
quite l i m i ted . KEV POINT
Endomyocard ial biopsy is rarely indicated in t he eval ­
• In patients with Marfan syndrome and aortic root
uation of acute heart failure a s i t is i nvasive a n d is unlikely
dilation, surveillance imaging should be performed
to be helpful in ident i fy i ng a reversible cause. It may be
6 months after diagnosis and annually thereafter i f
considered i n patients whose heart failure is unresponsive
t h e aortic size remains stable.
to medical t herapy or is associated with ven tricular a rrhyth ­
m ias or conduction b lock i n order to evaluate t o r g i a n t cell
myocard itis. Bibliography
Hiratzka LF: Bakris G�. Beckman JA. et al; American College of Cardiology
KEV POINT Foundat1on/Amencan Heart Association Task Force on Practice
Guidelines: American Association for Thoracic Surgery: American
• Evaluation of unusual causes of heart failure should
College of Radiology: American Stroke Association: Society of
not be performed routinely but should be performed Cardiovascular Anesthesiologists: Society for Cardiovascular
Angwgraphy and Interventions: Society of lnterventional Radiology:
when there are suggestions of specific diseases by his­
Society of Thoracic Surgeons; Society for Vascular Medicine. 2010
tory or physical examination findings. A�CF/AHA/AATS/ACR/ASA/SCA/SCAI/SIR/STS/SVM guidelines for the
diagnosis and management of patients with Thoracic Aortic Disease: a

214
Answers and Criti ques

report of the American College of Cardiology Foundation/ American of t h i s patient's high-risk ECG stress test, he should
Heart Association Task Force on Practice Guidelines, American
Association for Thoracic Surgery, American College of Radiology,
undergo catheterization for a definitive diagnosis and
American Stroke Association, Society of Cardiovascular possible revascularization. There would be no benefit
Anesthesiologists, Society for Cardiovascular Angiography and
to a noninvasive imaging test prior to or instead of that
I nterventions, Society of l nterventional Radiology, Society of Thoracic
Surgeons, and Society for Vascular Medicine. Circulation. 2010 Apr intervention.
6 ; 1 2 1 ( 13):e266-369. Erratum in: Circulation. 2010 Jul 27;122(4):e410.
[PMID: 20233780] KEY POINT
• Cardiac catheterization is indicated in patients
with a positive electrocardiographic stress test and
Item 1 1 8 Answer: B
findings indicative of high-risk coronary artery
Educational Objective: Manage a patient with a high­ disease.
risk score on exercise treadmill testing with cardiac
catheterization.
Bibliography
This patient's exercise electrocardiographic (ECG) stress Mark DB, Hlatky MA, Harrell FE Jr, Lee KL, Califf RM, Pryor DB. Exercise
treadmill score for predicting prognosis in coronary artery disease. Ann
testing results indicate that he has coronary artery dis­ Intern Med. 1987 Jun;l06(6):793-800. [ PMID: 3579066]
ease (CAD) , and his Duke treadmill score ( - 11 . 5 ) indicates
the presence of high-risk disease. He should undergo

CJ
cardiac catheterization for diagnosis and possibly revas­ Item 1 1 9 Answer: c
cularization.
Educational Objective: Diagnose peripartum cardiomy­
Exercise ECG stress testing can be used for the diagnosis
opathy.
of CAD (as in this patient) , to evaluate adequacy of medical
therapy in patients with known CAD, and to evaluate func­ TI1e most likely d i agnosis i n t h i s wom a n who gave b i rt h
tional status. When used to evaluate chest pain, the test is 2 weeks ago is peripartum card i o rnyopathy. Peripartum
considered diagnostic of obstructive CAD (>70% obstruction) cardiomyopathy is l e ft ven t ricu l a r systolic dysfu n c t io n
if there is greater than 1-mm ST-segment depression with i dent i fied toward t he e n d o f pregnancy o r i n the months
exercise in two contiguous leads. The findings in this patient fol l owing delivery i n the absence of another iden t i fiable
are consistent with occlusive coronary disease as the cause cause. This occurs with i ncreased frequency in women
of his exertional chest pain. w i t h a h istory of preeclampsia . Pat ients may be asymp­
In addition to diagnosis, a positive treadmill study tomatic or presen t with feat u res o f heart fa i l u re. Prompt
can be used to further risk stratify obstructive CAD. The i n i tiation of medical t herapy is recom mended for women
Duke treadmill score is one method and is calculated with peripartum cardiomyopathy and i ncludes an ACE
as follows: Exercise time in minutes - (5 x ST-segment i n h ib i tor or an a ngiotensin receptor blocker (a fter del iv­
depression) - (4 x angina score) . (Angina score: O = ery) , �-blockers, d igox i n , hyd ra l az i n e . n i t rates . and
asymptomatic; 1 = nonlimiting angina; 2 = exercise­ d iuretics.
limiting angina.) Scores below - 11 are high risk, and P u l mo n a ry embolism c a n occur postpar t u m , par­
those above 5 are low risk. Patients with high-risk scores t i c u la rly if prolonged bed rest is req u i red i n t h e peri­
are likely to have left main or proximal left anterior partum period . A l t h o ugh bot h p u l m o n a ry e m b o l i s m
descending (LAD) artery disease. Other markers of a a n d h e a r t fa i l u re fre q u e n t ly a re m a rked by dy sp n e a t h i s
,

high-risk exercise study that would be suggestive of pat i e n t ' s prese n t a t ion is more i n d i c a t ive o f heart fa i l u re.
p roximal LAD artery disease or multi-vessel disease w i t h p u l m o na ry congestion and elevated central ve nous
would include a drop in blood pressure with exercise or p ressure .
severe ST-segment depression. Based on his high-risk Mosl patients with ischem ic cardiomyopathy have
Duke treadmill score, this patient should be further eval­ symptomatic coronary artery d isease, abnormal electro­
uated with coronary arteriography. cardiographic findings demonstrating previous myoc ard i a l
Although this patient should be treated with medi­ i n farction, or regional hypokinesis on echocardiography.
cal therapy including aspirin, a �-blocker, and a statin, he These fin d i n gs are absent in this patient.
should also undergo cardiac catheterization because of the Stress-induced cardiomyopathy ( ta kotsubo cardiomy­
high likelihood of severe obstructive CAD. opathy) is characterized by transient cardiac dysfu nction
The use of imaging, such as stress echocardiography with ven tricular apical bal looning. usual ly triggered by
or myocardial perfusion imaging, can localize ischemia i n tense emotional or physical stress, although in several
to a vascular territory and can be helpfu l to determine publ ished cases. no trigger was identi fiable. The presenting
affected vascular territory prior to revascularization. clinical picture may mimic an acute coronary syndrome,
Stress testing with imaging can also be helpful in making with chest pa in, mil d ly elevated cardiac enzyme levels. and
the diagnosis of CAD in patients with equivocal exercise e lect rocardiographic changes consistent wit h ischemia. TI1e
stress tests or those in whom there is a higher likelihood patient's clin ical presenlation is not consistent with stress
of a false-positive exercise stress test. However, because cardiomyopat hy.

21 5
Answers and Criti q u es

KEY POINT PBMV in the absence of left atrial thrombus or moderate to


severe mitral regurgitation. Valve morphology is evaluated
• Peripartum cardiomyopathy is left ventricular systolic
by assessing the degree of valvular calcification, valve thick­
dysfunction identified toward the end of pregnancy or
ening, degree of leaflet restriction, and extent of subchordal
in the months following delivery in the absence of
thickening. The most common cause of mitral stenosis is
another identifiable cause.
rheumatic heart disease, in which valve inflammation leads
to progressive degenerative changes including leaflet thick­
Bibliography ening, calcification, and impaired valve function, as seen in
Sliwa K. Hilfiker-Kleiner D. Petrie MC, et al: Heart Failure Association of the
this patient.
European Society of Cardiology Working Group on Peripartum
Cardiomyopathy. Current state of knowledge on aetiology, diagnosis, Given that this patient has severe symptomatic mitral
management, and therapy of peripartum cardiomyopathy: a position stenosis and that there are no obvious contraindications to
statement from the Heart Failure Association of the European Society of
Cardiology Working Group on peripartum cardiomyopathy. Eur J Heart therapy, a conservative approach would not be appropriate.
Fail. 2010 Aug;l2(8) :767-78. [PMID: 20675664] Mitra! valve replacement is indicated in patients with
symptomatic (NYHA functional class I II-IV) severe mitral
Item 1 2 0 Answer: c stenosis when PBMV is unavailable or contraindicated or
valve morphology is unfavorable.
Educational Objective: Manage severe symptomatic
Generally, rheumatic mitral valve disease is not as
mitral stenosis.
amenable to surgical repair as that from degenerative causes,
This patient has severe symptomatic mitral stenosis with a such as myxomatous mitral valve disease. Moreover, the per­
valve that by description appears amenable to percutaneous cutaneous option is preferable in a patient when available.
balloon mitral valvuloplasty (PBMV) . Classifying mitral steno­
KEY P O I NT
sis as moderate or severe can be difficult owing to variance in
heart rate and forward flow on transmitral gradient. As such, • Percutaneous balloon mitral valvuloplasty is the pre­
mean gradients are no longer included in the severity criteria. ferred treatment for severe symptomatic mitral stenosis.
However, the mean gradient is usually greater than 5 mm Hg
to 10 mm Hg in severe mitral stenosis, and the mitral valve Bibliography
area is usually less than 1.5 cm2 in severe mitral stenosis and N ishimura RA, Otto CM, Bonow RO, et al; American College of Cardiology/
American Heart Association Task Force on Practice Guidelines. 2014
1.0 cm2 or less in very severe mitral stenosis. AHA/ACC guideline for the management of patients with valvular heart
PBMV is indicated for symptomatic patients (New York disease: executive summary: a report of the American College of
Cardiology/American Heart Association Task Force on Practice
Heart Association [NYHA] functional class I I , III, or IV) with
Guidelines. J Am Coll Cardiol. 2014 Jun 10:63(22):2438-88. Erratum in:
severe mitral stenosis and valve morphology favorable for J Am Coll Cardiol. 2014 Jun 10;63(22):2489. [PMID: 24603192]

216
I ndex

Note, Page numbers followed by f and t denote figures and tables. respectively. Test Angiotensin receptor blockers (ARB), 28t
questions are indicated by Q. in angina, 17
in heart failure, 34-3S, 34t, Q9
A in STEM!, 23
Abciximab, for acute coronary syndromes, 23 Ankle-brachia! index (ABI), 100, 101 - 102, 102t
Abdominal aortic aneurysm (AAA), 99-100, Q31 Anthracycline toxicity, 106- 108, 107t, Q77
open surgical repair vs. endovascular aneurysm repair, 100 Antiarrhythmic medications, SO-Sl, SOt
risk factors for, 99 Antibiotic therapy, for infective endocarditis, 83, 84, 84t
rupture, risk of, 100, lOOt Anticoagulants
screening and surveillance for, 99-100, Q80 for acute limb ischemia, 104
therapies for, 100 for atrial fibrillation, SS, Q27
Accelerated idioventricular rhythm (AIVR). 23 for non-ST-elevation acute coronary syndromes, 26, 27f
after coronary reperfusion, Q42 for patients with prosthetic valves, 86
ACE inhibitor-induced cough, 34, Q9 for peripartum cardiomyopathy, 110
ACE inhibitors, 28t during pregnancy, 112-113, 1 1 2t
in angina, 1 6 for pregnant women with mechanical valve prosthesis, Q60
i n cardiac disorders i n pregnancy, l ! O , l l l t for STE M ! , 23
i n heart failure, 34-3S for stroke prevention in atrial fibrillation, S6-S7, S7t, Q4, Q70
in peripheral arterial disease, 103 Antihypertensive therapy, in peripheral arterial disease, 103
in restrictive cardiomyopathy, 48 Antiplatelet therapy
for secondary cardiovascular prevention, QllO for angina, 1 6
in STEM!, 23 for non-ST-elevation acute coronary syndromes, 26
Acute coronary syndrome (ACS), 20 for patent foramen ovale, 86
care after, 30 for peripheral arterial disease, 103
diagnosis of, 20f for STEM!, 23
non-ST-elevation acute coronary syndromes (NSTE-ACSs), 20, 2S-30 Aortic atheroma, 99, 99f
not associated with obstructive coronary disease, 30 Aortic coarctation. 92-93, 92f, Q3S
pathophysiology of, 20 characteristic murmur of, 92
and percutaneous coronary intervention, medications after, QS3 clinical presentation of, 92
ST-elevation myocardial infarction (STEM! ) , 20-2S diagnostic evaluation of, 92
Acute limb ischemia, 104-IOS, Q66 follow-up after repair, 93
anticoagulation therapy in, 104 pathophysiology of, 92
categories and prognosis of, l OSt treatment of, 92-93
physical findings of, 104 Aortic disease, 95-100
treatment of, 104 abdominal aortic aneurysm, 99-100, IOOt
Adenosine acute aortic syndrome, 98-99, 98f
for arrhythmias, sot, Sl aortic atheroma, 99, 99f
for cardiac disorders in pregnancy, 110, l l l t imaging of thoracic aorta, 9S, 96t
for supraventricular tachycardia, S 3 , Q!OO thoracic aortic aneurysms, 96-98, 97t
Advisory Committee on Immunization Practices (ACIP), on influenza Aortic dissection, 9Sf, 98-99, 98f
vaccination, 18 type A , Q24
African Americans, cardiovascular disease in, 1 type B, Q71
Alcohol septa! ablation, for hypertrophic cardiomyopathy, 46 Aortic override, in tetra logy of Fallot, 93, 93f
Aldosterone antagonists, in heart failure, 34t, 3S-36 Aortic regurgitation, 71, 73t. 76t, 77t, 79-80, Q93
Alteplase, in STEM!, 22t clinical presentation and evaluation of, 79
Ambulatory ECG, for arrhythmias, 12. 14t, Sl, S4 management of, 79. QS9
American College of Cardiology/American Heart Association Aortic repair, in complicated type B aortic dissection, Q71
cardiovascular risk calculator, 3 Aortic root replacement, for aortic regurgitation, 79
American Heart Association's Heart Disease and Stroke Statistics. 1 Aortic stenosis, 71, 73t, 76t, 77-79, 77t, Q20, Q28, Q7S, Q112
Amiodarone causes of, 77
for arrhythmias, sot, Sl, 60 clinical presentation and evaluation of, 77-78
for cardiac disorders in pregnancy, l i l t management of, 78-79
Amitriptyline, and QT-interval prolongation, Ql4 paradoxic low-gradient, 78
Amlodipine pseudosevere aortic stenosis, 78
for coronary artery disease, 29t Aortic valve replacement
for heart failure, 36 in aortic stenosis, 78, Q28, Q 1 1 2
Aneurysm, aortic in bicuspid aortic valve, 8 0
abdominal, 99-100, Q31 Apixaban, for stroke prevention in atrial fibrillation,
thoracic, 96-98, 97t S7, S7t
Angina pectoris. See a lso Coronary artery disease (CAD) Appropriate use criteria (AUC), 18
stable, 14-19 Arrhythmias, S0- 64
testing in, lS, 16f antiarrhythmic medications, SO-Sl, sot
treatment of, 15-19, 17f, Qll, Q48 ARYC/D and, 6lt, 62
antianginal medications, 16, 18 atrial fibrillation, SS-S8
cardioprotective medications, 16-18 atrial flutter, S8-S9
coronary revascularization, 18-19 bradycardia, Sl -S3
dual antiplatelet therapy after revascularization, in Brugada syndrome, 6 1 , 6lt, 62f
19, 19t catecholaminergic polymorphic VT, 61-62, 61 t
Angiography, in acute limb ischemia, Q66 diagnostic testing for, 12. 14t
Angioplasty, in peripheral arterial disease, 104 early repolarization syndrome and, 61t, 62
Angiotensin-converting enzyme (ACE) inhibitors. See ACE inhibitors inherited syndromes, 61-62, 6lt

21 7
I n d ex

Arrhythmias (continued) for peripheral arterial disease, L03


in long QT syndrome, 61, 61! in pregnancy, l!O, Q36
in short QT syndrome, 6 1 , 6 1 ! for penetrating atherosclerotic ulcer, Q6
STEM! and, 2 4 for premature atrial contractions, S4
sudden cardiac arrest and, 62-63 for restrictive cardiomyopathy, 48
tachycardias, S3-SS, S9-61 for stable angina pectoris, L6. 18
Arrhythmogenic right ventricular cardiomyopathy/dysplasia (ARVC/D), for STEM!, 2lf, 23
6lt, 62 for supraventricular or atrial arrhythmias, SO
Aspirin, 28t for wide-complex tachycardia, QSl
for atrial fibrillation, S7 Bicuspid aortic valve, 80, 92, 97, Ql2, Q37, Q7S, QBS, Q97
for cardiovascular disease. 4, 4t, St, 16 Bioprosthetic mitral valve replacement, Q46
for cryptogenic stroke and patent foramen ovale, Q96 Bisoprolol
for patients with prosthetic valves. 86 for arrhythmias, sot
for pericarditis, Q43 for heart failure, 3S. 3St
for peripheral arterial disease, 103 Bivalirudin. 23, 26
for pregnant women with mechanical valve prosthesis, Q60 Blacks
for STEM l, 23 cardiovascular disease in, l
Atenolol heart failure in, QL06
for arrhythmias. sot Blood cultures. in implanted cardiac device infection, QS8
for cardiac disorders in pregnancy, 111 t Blood pressure measurement, in peripheral arterial d isease, 1 0 1-102,
for coronary artery disease, 28t, 29t l02t, Ql08
Athlete ' s heart, 43. 4St Bradycardia, Sl-S3
Atorvastatin, QB, Q40 atrioventricular block, Sl
for coronary artery disease, 28t clinical presentation of, Sl
for peripheral arterial disease. QS2 diagnostic evaluation of. SI
Atrial fibrillation, SS-SS, Q4. Q70 pacemakers in, S2-S3, 52t
clinical presentation of, SS sinus bradycardia, 5 1
incidence of. SS The Breathing N o t Properly study, 3 2
long-standing, SS Bromocriptine, in peripartum cardiomyopathy, l ! O
management of Brugada syndrome, 61, 61!, 62f. Q68
acute, SS-S6 B-type natriuretic peptide (BNP) levels, in heart failure. 32-33, 38
anticoagulation t herapy, SS, S6-S7, S6t. S7t
cardioversion and acute rate control, SS-S6 c
long-term, S6-S8 CABG. See Coronary artery bypass graft surgery (CABG)
nonpharmacologic strategies, SB CAD. See Coronary artery disease (CAD)
rate or rhythm control strategy, SB Calcium channel blockers, 29t
risk stratification scores, S6, S6t for angina. 18
paroxysmal, SS, QSl for cardiac disorders in pregnancy, Ult
persistent. SS for coronary artery disease. 29t
and stent placement, Q27 for heart failure, 36
Atrial flutter, S8-S9, S9f for hypertrophic cardiomyopathy. 44
Atrial septa! aneurysm, 87, QlO for idiopathic ventricular tachycardia, 60
Atrial septa! defect (ASD). 87-89, 87f, 88t, Q29 for mitral stenosis, 81
clinical presentation of, 87 for premature atrial contractions, S4
coronary sinus, 87 for restrictive cardiomyopathy, 48
diagnostic evaluation of, 87, 88t, 89 Cancer, cardiovascular disease and, IOS-108, Q77
familial ostium secundum, 87 Candesartan. for coronary artery disease, 28t
follow-up after ASD closure, 89 Captopril
genetic syndromes associated with, 87 for coronary artery disease, 28t
ostium primum, 87, 87f, 88t, Ql04 for Takotsubo cardiomyopathy, Q21
ostium secundum, 87, 87f. 88t Cardiac allograf't vasculopathy, in heart transplant recipients. 40, Q102
pathophysiology of, 87 Cardiac catheterization, Qll8
pregnancy in patients with, 89 for atrial septa! defect, 89
sinus venosus, 87, 87[, 88t hemodynamic, Q90
treatment for, 89 in patient with stable angina pectoris not controlled with optimal
Atrial tachycardia, S4 medical therapy, Q48
Atrioventricular (AV) block. Sl for structural heart disease, 13t
first-degree, Sl for supraventricular or atrial arrhythmias, SO
with bifascicular block, Qi09 in valvular heart disease, 7S
second-degree, SL, Q63 Cardiac magnetic resonance (CMR) imaging, 75
third-degree, SL in bicuspid aortic valve, 80
Atrioventricular nodal reentrant tachycardia (AVNRT), S4, S4f in coronary artery disease, 7t. 10
Atrioventricular reciprocating tachycardia (AVRT), S4-SS in heart failure, 33
in hypertrophic cardiomyopathy, 43
B in mitral regurgitation, 82
Balloon angioplasty, in peripheral arterial disease, 104 in structural heart disease. 13t
Balloon valvuloplasty Cardiac resynchronization t herapy (CRT), 37, 37t, QS6
for aortic stenosis, 78 Cardiac resynchronization therapy-defibrillator (CRT-D). S2t
for pulmonary valve stenosis, 92 Cardiac resynchronization therapy-pacing (CRT-P), S2t
Benazepril, for coronary artery disease, 28t Cardiac surgery with pericardiectomy, in constrictive pericarditis, 68-69
Bentall operation, 97 Cardiac tamponade, 69-71. Ql6
J3-blockers, 28t, 29t causes of, 69
for arrhythmias, so, sot clinical presentation of, 69, 70f
for cardiac disorders in pregnancy, U l t evaluation of. 69. 70f
fo r heart failure, 34-3S, 34t, 3St management of, 69-71
with reduced ejection fraction, QS4 pulsus paradoxus in, 69
for idiopathic ventricular tachycardia, 60 water-bottle heart in, 69, 70f
for long QT syndrome, 6L Cardiac tumors, 48-49
for mitral stenosis, 81 clinical presentation of, 48-49
for non-ST-elevation acute coronary syndromes, 27. 27f evaluation of, 49, 49f
for peripartum cardiomyopathy, 110 management of, 49

21 8
I ndex

primary, 48 management of, 68-69


types of. 48 and restrictive cardiomyopathy, 47-4B, 66-67, 66f, 67t
Cardiogenic shock, 39-40, 40t, Q23 Contractile reserve, 7B
Cardiopulmonary resuscitation (CPR), in sudden cardiac arrest, 62-63 Coronary angiography, Ql02
Cardiotoxicity for coronary artery disease, 7t, 10-ll, llf. Q48
of chemotherapy, 106-108, 107t, Q77 for valvular heart disease, 7S
of radiation therapy to thorax, lOS-106, 106t Coronary artery bypass graft surgery (CABG), lB, 20, 26, 29, 31,
Cardiovascular disease (CVD), 1 QlB, Q l l l
aspirin for primary prevention of, 4, 4t, St in diabetes mellitus a n d multivessel coronary artery disease, Q l B
in cancer survivors, lOS-108 off-pump, 19
chronic kidney disease and, 3 in stable angina patients, l 9
diabetes mellitus and, 3, 31, Q6S Coronary artery calcium (CAC) scoring, 4, QB3
diagnostic testing for, S-14 for coronary artery disease, 71, 1 1
emerging risk factors for, 3-4 Coronary artery disease (CAD)
epidemiology of. 1-4 aspirin for prevention of, 4, 4t, St
ethnicity and, 1 diagnostic testing for, S-12, 6-7t, Bf, 9t, lOf, lS, 16f
genetics in, 2 CAC scoring, 11
history and physical examination in, S coronary anatomy, visualization of, 10-11, l l f
lifestyle risk factors for, 2-3 risks of, 11-12
elevated cholesterol levels, 2, Ql7 stress testing, S-10, 6-7t, Bf, 9t, lOf, lS
hypertension, 2 pretest likelihood of, lS, lSt
obesity, 2 stable angina pectoris in, 14-19
psychosocial stressors, 2 treatment of, lS-19, 2B-29t
sedentary lifestyle, 2 antianginal medications. 16, lB
smoking, 2 cardioprotective medications, 16-lB
mortality from, I coronary revascularization, lB-19
pregnancy and, 108-113 in women, 1 , 30-31
risk assessment, tools for, 3 Coronary CT angiography (CTA)
systemic inflammatory conditions and, 3 for coronary artery disease. 7t, 10-11, llf, lS
in women, ! for peripheral arterial disease, 102
Cardiovascular risk scores, 3 for structural heart disease, 13t
Cardioversion, for atrial fibrillation, SS-S6 Coronary vasospasm, 30
CARPREG index, 109, 109t COURAGE trial, 31
Carvedilol Critical limb ischemia, revascularization in, 104
for arrhythmias, sot CT scan
for coronary artery disease, 28t, 29t for atrial septa! defect, B9
for heart failure with reduced ejection fraction, QS4 for imaging of thoracic aorta, 9S, 96!
Catheter ablation Cyanotic congenital heart disease, 94-9S, QB2
for atrial fibrillation, S8, Q4
of premature ventricular contractions, Q33 D
for Wolff-Parkinson-White syndrome, SS Dabigatran, for stroke prevention in atrial fibrillation, S7, S7t
Catheter-directed t hrombolysis, in acute limb ischemia. 104 Dexrazoxane, 108
Catheterization, cardiac. See Cardiac catheterization Diabetes mellitus, Q40
CHAOS, score, S6, S6t and cardiovascular disease, 3, 31, QlB, Q6S, QBB
CHA2DS2-VASc score, S6-S7, S6t and peripheral arterial disease, 100, 103
Chemotherapy, cardiotoxicity from, 106-108, 107t Digoxin
Chest CT, for structural heart disease, 13t in arrhythmias, sot, Sl
Chest pain, 14-lS in cardiac disorders in pregnancy. l l l t
in acute coronary syndrome , 20 in heart failure, 3 S
angina I (see Angina pectoris) in peripartum cardiomyopathy, 1 1 0
aortic, 98 in pregnancy, 110
in pericarditis, 64 Diltiazem
in STEM!, 23 for angina, lB
Chest radiography for arrhythmias, sot
aortic coarctation, 92, 92f for atrial tachycardia, S4
in congenital heart disease, 88t for cardiac disorders in pregnancy, l l l t
in valvular heart disease, 72 for coronary artery disease, 29t
Chronic kidney disease (CKD), and risk of cardiovascular disease. 3 Disopyramide
Cigarette smoking, and peripheral arterial disease, 100, 103 for arrhythmias, sot
Cilostazol, i n peripheral arterial disease, 103, Q41 for hypertrophic cardiomyopathy, 44
CLEVER trial, 103 Disopyramide, for cardiac disorders in pregnancy, l l lt
Clopidogrel, 28t Diuretics
in angina, 16 in acute decompensated systolic heart failure, Q9S
in non-ST-elevation myocardial infarction, 26, Q73, QlOl in cardiac disorders in pregnancy, l i l t
in patients with thrombolytic therapy for STEM!, Q81 in heart failure, 3 S , QlS
in peripheral arterial disease, 103 in mitral regurgitation, B2
Colchicine, in pericarditis, 6S in mitral stenosis, Bl
Congenital heart disease, 86-9S in peripartum cardiomyopathy, 110
aortic coarctation, 92-93 Dobutamine echocardiography. for coronary artery disease, 6t,
atrial septa! defect, 87-89 9, Ql9
cyanotic. 94-9S, Q82 Dobutamine, for cardiogenic shock, 39t, Q23
imaging findings and late complications in, 88t Dobutamine nuclear perfusion. for coronary artery disease, 6t
patent ductus arteriosus. 91 Dofetilide, for arrhythmias, sot
patent foramen ovale, 86-87, B6f Dopamine, for cardiogenic shock, 39t
pulmonary valve stenosis, 91-92 Doxorubicin, cardiotoxicity due to, Q77
tetralogy of Fallot, 93-94, 93f Dronedarone, as antiarrhythmic agent, SOt, Sl
ventricular septa! defect, B9-90 Dual antiplatelet therapy
Constrictive pericarditis. 66-69, Q32, Q90 for non-ST-elevation acute coronary syndromes, 26, Q44, QlOl
causes of, 66 for STEM!, 23
chest radiation therapy and, !OS Dyslipidemia, and risk for future myocardial infarction, Ql7
clinical presentation and evaluation of. 66-6B, 66f. 6Bf Dyspnea, pulmonary vein stenosis and, Q7B

219
I ndex

E diagnosis of, 32-33


The Early Surgery versus Conventional Treatment in lnfective Endocarditis B-type natriuretic peptide (BNP) levels. 32-33
(EASE) trial, 84 cardiac magnetic resonance (CMR) imaging, 33
Echocardiography. See a lso Transesophageal echocardiography (TEE); electrocardiogram (ECG), 33
Transthoracic echocardiography (TTE) endomyocardial biopsy, 33
in aortic stenosis, 78 laboratory assessment, 33
in cardiac tamponade, 69, 70f right heart catheterization, 33
follow-up, in asymptomatic severe aortic stenosis. Q20 transthoracic echocardiography, 33
in heart failure. 32-33, 38 discharge and schedule follow-up, Q34
in hypertrophic cardiomyopathy. 43 ischemia, evaluation for, 33-34
in infective endocarditis. 83 management of, 38-40
in mitral stenosis, 8 1 cardiogenic shock and, 39-40, 39t, Q23
in pericarditis, 6 5 strategies to prevent readmission. 40
in tachycardias. 53 volume overload and, 38-39
in tricuspid regurgitation, 82 worsening kidney function and, 39
Effusive constrictive pericarditis. Q7 medical therapy for, 34-36. 34t
Eisenmenger syndrome. 90, 91, 94-95, Q22, Q64 ACE inhibitors, 34-35
Electrocardiography (ECG) aldosterone antagonists, 35-36
in atrial flutter, 58- 59. 59f angiotensin receptor blockers, 35
in Brugada syndrome, 61. 62f P-blockers, 35, 35t, Q54
in cardiac tamponade. 69 calcium channel blockers, 36
in congenital heart disease. 88t digoxin. 35
in heart failure, 33 diuretics, 35
in monomorphic ventricular tachycardia. 60, 60f isosorbide dinitrate and hydralazine, 36
in pericarditis, 64-65, 64f. 6St New York Heart Association (NYHA) functional class and,
in supraventricular tachycardias, 53 34, 34t
in tachycardias, 53 mortality in, I
in valvular heart d isease. 72 pathophysiology of, 32
Electrophysiology study. QllS with preserved ejection fraction, 32, 36, QIS
for arrhythmias, 14t prevalence of, I
Enalapril, for coronary artery disease. 28t symptoms of, 32
Endocarditis. See Infective endocarditis Heart murmurs, 1 1 , Q3
Endomyocardial biopsy, 41 of aortic coarctation, 92
in restrictive cardiomyopathy. 48 in atrial septa) defect, 87
Endovascular aneurysm repair (EVAR), 100 cardiac tumors and, 48
Endovascular revascularization. in peripheral arterial disease, 104 of obstructive hypertrophic cardiomyopathy, 43
Enoxaparin, in non-ST-elevation myocardial infarction, Q73 in patent ductus arteriosus. 91
Epinephrine, in sudden cardiac arrest, 63 in pregnancy, 109t
Eplerenone, for coronary artery disease. 28t in STEM I , 20
Eptiflbatide, for acute coronary syndromes. 23 in valvular heart disease, 71-72, 72t, 73-74t
Esmolol, for cardiac disorders in pregnancy, 1 1 1 t in ventricular septa! defect, 90
Event recorders, Q50 Heart Outcomes Prevention Evaluation (HOPE) trial,
Exercise electrocarcliography. Q87 103
for arrhythmias, 14t Heart Protection Study (HPS), 103
for coronary artery disease, 6t Heart transplantation, 40-41. Ql02
Exercise stress testing, Q38, Q57 candidates for, 40
Exercise therapy. in peripheral arterial disease, 103. Q84 complications of, 40-41
immunosuppression regimen after. 40
F Hemodynamic cardiac catheterization. Q90
Familial thoracic aortic aneurysms and aortic dissections (TAAD). 97 High-sensitivity C-reactive protein testing, 3-4, Q2
Felodipine. in heart failure. 36 Holt -Oram syndrome, 87
Flecainide Hydralazine
for arrhythmias, sot in cardiac disorders in pregnancy, l l l t
for cardiac disorders in pregnancy. l i lt a n d isosorbide dinitrate
5-Fluorouracil toxicity, 107 in heart failure, 36, Ql06
Fluvastatin, for coronary artery disease, 28t in peripartum cardiomyopathy, 1 1 0
Fosinopril. for coronary artery disease. 28t Hydrochlorothiazide, in hypertrophic cardiomyopathy, Q S
Framingham Heart Study. 1 Hypertrophic cardiomyopathy (HCM). 42-47, Q S
Framingham risk score. 3 a n d athlete ' s heart, 4 3 , 45t
Furosemide clinical course and risk stratification of. 43-44
in heart failure with preserved ejection fraction, Ql5 clinical presentation and evaluation of, 42-43, 42f-44f
in recently diagnosed heart failure. Q95 dynamic LVOT obstruction in, 42, 43
genetic testing and screening, role of. 46-47
G implantable cardioverter-defibrillators in, 44, Q79
Genetic testing. for hypertrophic cardiomyopathy. 4St, 46-47 myocardial hypertrophy in, 43
Giant cell myocarditis. 41 pathophysiologic mechanisms, 42-43, 42f, 43f
Glucocorticoids. in pericarditis, 65-66 pharmacologic treatment of, 44, 46
Glycoprotein lib/Illa in hibitors screening for, Q62
in non- ST- elevation acute coronary syndromes. 26. 27f septal reduction therapy, 44, 46
in STEM ! . 23 and sudden cardiac arrest. 43-44. 45t

H
Heart failure, 32-42. Q47, Q54, Q56, Q69 , Q76, Q89, Qll6 Ibuprofen, in constrictive pericarditis, Q32
acute decompensated, 38-39, Q95 I CDs. See Implantable cardioverter-defibrillators (ICDs)
advanced refractory. 40-41 Implantable cardiac electronic devices. 52-53. 52t
assessment of. 37-38 and infection. 63. Q58
cardiomyopathies and. 41-42 Implantable cardioverter-defibrillators (ICDs), 52, 52t
chronic. 32. 37-38 in Brugada syndrome, Q68
clinical evaluation in. 32 heart failure and, 36-37, 37t, Q76
device t herapy in, 36 in hypertrophic cardiomyopathy, 44, Q79
cardiac resynchronization therapy. 3 7 . 37t preoperative device management. Ql03
implantable cardioverter-defibrillators. 36-37. 37t Implantable loop recorder, for arrhythmias, 14t

220
I n d ex

I nfective endocarditis, 83-8S, Q l 07 in mitral regurgitation. 82


diagnosis of, 83, 83t in mitral stenosis, 81
empiric therapy for, 83, 84t Multifocal atrial tachycardia, S4
modified Duke criteria for, 83, 83t Murmurs. See Heart murmurs
prophylaxis for Myocardial disease, 42-49
for nondental procedures, 84, Q2S cardiac tumors, 48-49
surgery for, 84, Q99 hypertrophic cardiomyopathy, 42-47
I nfluenza vaccination, 18, Ql3 restrictive cardiomyopathy, 47-48
INSTEAD trial, 99 Myocardial infarction, aspirin for prevention of, 4 , 4t, St
Intra-aortic balloon pumps, 40 Myocardial perfusion imaging stress test, Q47
in mitral regurgitation, 82 Myocarditis, acute, 41
Intramural hematoma, 98-99, 98f Myxomas, 48-49, 49f. Q113
lrbesartan, for coronary artery disease, 28t
Iron therapy, in patient with cyanotic congenital heart disease, Q82 N
lsosorbide dinitrate and hydralazine, in heart failure, 36, QJ06 Nebivolol, for coronary artery disease, 28t, 29t
Isosorbide, for coronary artery disease, 29t Negative chronotropic agents, in restrictive cardiomyopathy, 48
lsosorbide mononitrate, for stable angina pectoris, Qll Negative inotropes, in hypertrophic cardiomyopathy, 44
Nesiritide, for cardiogenic shock, 39t
K Nicardipine, for coronary artery disease, 29t
Kinase inhibitors, 108 Nifedipine, for coronary artery disease, 29t
Kussmaul sign, 66 Nitrates
in angina, 18
L in coronary artery disease, 29t
Labetalol in mitral regurgitation, 82
for cardiac disorders in pregnancy, 1 1 1 t in mitral stenosis, 81
for hypertension during pregnancy, Q36 in non-ST-elevation acute coronary syndromes, 27
Left ventricular assist devices (LVADs), 40 in peripartum cardiomyopathy, 110
in end-stage heart failure, Q69 in STEM!. 23
Left ventricular free wall rupture, STEMI and, 241. 2S Nitroglycerin, for cardiogenic shock, 39t
Lidocaine N itroprusside, in low-output heart failure, Q89
for arrhythmias, sot Non-ST-elevation acute coronary syndromes (NSTE-ACSs), 20, 2S-30
for cardiac disorders in pregnancy, l l l t medical therapy for, 26-29, 28-29t
Lipomas, 48-49 antianginals, 26, 29
Lisinopril, Q9 anticoagulants, 26
for coronary artery disease, 28t antiplatelet therapy, 26
Loeys- Dietz syndrome, 80 invasive vs. ischemia-guided treatment, 29
Long QT syndrome, 61, 6lt lipid-lowering medications, 29
Loop diuretics, in restrictive cardiomyopathy, 48 non-ST-elevation myocardial infarction, 20
Looping event recorder, Q98 risk stratification in, 2S-26, 26t, 27f
for arrhythmias, 14t unstable angina (UA). 20
Losartan, for coronary artery disease, 28t Non-ST-elevation myocardial infarction (NSTEMI ) , 20, Q73
Lovastatin, for coronary artery disease, 28t Noonan syndrome, 91
Low-molecular-weight heparin (LMWH) Norepinephrine, for cardiogenic shock, 39t
for anticoagulation during pregnancy, 1 1 2 - 1 13, 112t Nosocomial endocarditis, 83-84. See a lso Infective endocarditis
in non-ST-elevation acute coronary syndromes, 26 NSAIDs, in pericarditis, 6S
in STEM!, 23 Nuclear SPECT perfusion, for coronary artery disease, 6t
Low-pressure cardiac tamponade, Ql6
0
M Obesity, and risk of cardiovascular disease, 2
Magnetic resonance angiography (MRA) , in peripheral arterial Olmesartan, for coronary artery disease, 28t
disease, 102 Organic nitrates, for cardiac disorders in pregnancy, l l l t
Malignant melanoma. 48-49 Ostium primum atrial septa! defect, 8 7 , 87f, 88t, Ql04
Marfan syndrome, Qll 7
pregnancy and, 110 p
Metabolic syndrome, and risk of cardiovascular disease, 2 Pacemaker implantation
Metoprolol for cardiac rhythm disorders, S2-53, 52t
for arrhythmias. sot indications for. 52t
for cardiac disorders in pregnancy, 111 t MRI conditional pacemakers. 52
for coronary artery disease, 281. 29t Papillary fibroelastomas, 48-49, 49f
for Takotsubo cardiomyopathy, Q21 PARTNER trial, 79
withholding of, before exercise stress testing, Q38 Patent ductus arteriosus (PDA), 91, Q22
Mexiletine, for arrhythmias, sot clinical presentation of, 91
M i lrinone, for cardiogenic shock, 39t diagnostic evaluation of, 9 1
Mitra! regurgitation, 71, 73t, 76t, 77t, 81-82, Q67, Q74 pathophysiology of. 9 1
bioprosthetic mitral valve replacement in. Q46 treatment of, 9 1
clinical presentation and evaluation of, 81 -82 Patent foramen ovale (PFO), 86-87, 86f
STEM! and, 2S antiplatelet therapy for, 86, Q96
therapy for, 82 atrial septa! aneurysm and, 87
transesophageal echocardiography in, Q49 diagnosis of, 86
Mitra! stenosis, 71, 73t, 76t, 77t, 80-81 , Ql20 and migraine, 86
causes of, 80-81 and platypnea-orthodeoxia syndrome, 87
clinical presentation and evaluation. 80-81 with right-to-left shunt, Q5S
management of, 81 Patient-triggered event recorder, for arrhythmias. 14t
pregnancy and, Q30 Penetrating a therosclerotic ulcer (PAU), 98-99, 98f
Mobile cardiac outpatient telemetry, for arrhythmias, 14t in descending aorta, Q6
Moxifloxacin, and QT-interval prolongation, Ql4 Pentoxifylline, in peripheral arterial disease, 103
MRI Percutaneous balloon mitral valvuloplasty (PBMV), Ql20
in atrial septa! defect, 89 Percutaneous coronary intervention (PCI), 23
for imaging of thoracic aorta, 9S, 96t atrioventricular block and, Q63
of lumbar spine, Q94 dual antiplatelet therapy after, 19, 19t
Multidetector CT, 7S in stable angina patients, 18-19
in bicuspid aortic valve, 80 ST-elevation myocardial infarction and, Q61, Q92

221
I nd ex

Percutaneous mitral balloon valvotomy. 76t, 81 a


Pericardia! disease, 64-71 QT-interval prolongation, Ql4
acute pericarditis, 64-66 Quinapril, for coronary artery disease, 28t
cardiac tamponade, 69-71 Quinidine
constrictive pericarditis, 66-69 for arrhythmias, SOt, 61
Pericardiocentesis, 6S, Ql6 for cardiac disorders in pregnancy, l i l t
in cardiac tamponade, 70-71
Pericarditis, acute. Q 1 14 R
causes of, 64 Radiation-induced cardiotoxicity, lOS-106, l06t
clinical presentation of, 64 Radionuclide angiography (MUGA) . for structural heart
evaluation of, 64-6S, 64[, 6St disease, 13t
management of, 6S-66, Q43 Ramipril
pericardia! friction rub in, 64 in coronary artery disease, 28t
Perindopril, for coronary artery disease. 28t in peripheral arterial disease. 103
Peripartum cardiomyopathy, 110, Qil9 Ranolazine
Peripheral arterial disease (PAD), 1 0 0 - lOS for angina, 18
and acute limb ischemia, 104-IOS for coronary artery disease, 29t
ankle-brachia! index, 101-102, l02t Resting ECG, for arrhythmias, 12, 14t, Sl, S3
causes of, 100 Restrictive cardiomyopathy, 47-48
claudication and pseudoclaudication. discrimination between. chest radiation therapy and, lOS
1 0 1 , Q94 clinical presentation, 47
clinical manifestations of, 100-101 and constrictive pericarditis, 47-48
epidemiology of, 100 diagnosis of, 47
evaluation of, 101-102 management of, 48
diagnostic testing, 1 0 1 -102, Q26 Reteplase, i n STE M ! . 22t
history and physical examination, 101, lOlt, l02t Revascularization
exercise therapy in. 103. Q84 in patients with stable angina pectoris. 18-19
interventional therapy for, 104 i n peripheral arterial disease, 104
lower extremity, 100 Reynolds risk score, 3
medical therapy for, 102-103, Q41, QS2 Rheumatoid arthritis, and risk of cardiovascular disease, 3
screening for, 100-101 Right ventricular infarction, STEM! and, 24-2S, 24t
upper extremity, 101. Q108 Rivaroxaban, for stroke prevention i n atrial fibrillation,
PET/CT, for coronary artery disease, 6t S7, S7t
Pharmacologic stress testing. Q39 Rosuvastatin, for coronary artery disease, 28t, 29
Phenytoin, for arrhythmias, SOt
Platypnea-orthodeoxia syndrome. 87, QSS s
Pooled Cohort Equations, 3 Screening
Prasugrel, 28t for abdominal aortic aneurysm, 99-100
in non-ST-elevation acute coronary syndromes. 26 for hypertrophic cardiomyopathy, 46, 47t, Q62
Pravastatin, for coronary artery disease, 28t Seattle Heart Failure model, 38
Pregnancy Septa! reduction therapy, for hypertrophic cardiomyopathy. 46
anticoagulation therapy during. l l 2 - 1 13, IJ2t Short QT syndrome, 61, 6lt
cardiovascular changes during, 108, 109t Simvastatin
congenital heart disease in, Q37 i n coronary artery disease, 28t
hypertension in. Q36 in peripheral arterial disease, 103
management of cardiovascular disease during, Sinus bradycardia, SI
109-113 Sinus tachycardia, S3
cardiovascular drugs during pregnancy, 110, l i l t Smoking cessation, for peripheral arterial disease, 103
cesarean delivery, 110 Society of Thoracic Surgeons adult cardiac risk score (STS score). 79
Marfa n syndrome. llO Sodium nitroprusside
peripartum cardiomyopathy. 110 in cardiogenic shock, 39t
vaginal delivery. 1 1 0 i n mitral regurgitation, 82
prepregnancy evaluation for women w i t h cardiovascular disease. Sotalol
108-109. l09t for arrhythmias, SOt
Premature atrial contractions (PACs). S3-S4 for cardiac disorders in pregnancy. l i l t
Premature ventricular contractions (PVCs). Spironolactone
S9-60. Q33 for coronary artery disease, 28t
Procainamide during lactation, 1 1 0
for arrhythmias, sot Stalin therapy, 2 8 t , Q40
for cardiac disorders in pregnancy, lilt for angina, 17-18
Propafenone high-intensity, QB
for arrhythmias, sot i n peripheral arterial d isease, 103, QS2
for cardiac disorders in pregnancy, l i l t ST-elevation myocardial infarction (STEM!), 20-2S
Propranolol complications of, 24-2S, 24t
for arrhythmias, sot diagnosis of, 20, 20f
for cardiac disorders in pregnancy. l i l t history and physical examination, 20
Prosthetic valves, SS-86 medical therapy for, 23
anticoagulation regimens for. 86 reperfusion therapy for. 20-23
choice of. 85. 85t t hrombolytic failure following, Q61
Pulmonary balloon valvuloplasty. for pulmonary valve STEM!. See ST-elevation myocardial infarction (STEM !)
stenosis, 92 Scents
Pulmonary-to-systemic blood flow ratio (Qp:Qs), 89 and antiplatelet therapy, 19, 191, 23, 30
Pulmonary valve regurgitation. Qi atrial fibrillation, Q27
Pulmonary valve stenosis, 91-92 bare metal, 19
clinical presentation of, 91 drug-eluting, 19, 31, S7, Q44
diagnostic evaluation of, 9 1 thrombosis. 19. 23
pathophysiology of. 91 Streptokinase, in STEM!, 22t
pregnancy and. 92 Stress cardiomyopathy, 41, Q21
pulmonary valve replacement in. Q72 Stress echocardiography, Q91
treatment of, 92 for coronary artery disease, 6t. 9
Pulmonary vein stenosis, Q78 Stress testing, for coronary artery disease. S-10. 6-7t. Sf.
Pulseless electrical activity (PEA) arrest. 63 9t. !Of

222
Index

Stroke in infective endocarditis. 83


aspirin for prevention of, 4, 4t, St in mitral regurgitation, 81-82
atrial fibrillation and, 56 in mitral stenosis, 81
Structural heart disease, diagnostic testing for, 12, 13t for structural heart disease. 13t
Subcutaneous implantable cardioverter-defibrillator, 52t in valvular heart disease, 72. Q74
Sudden cardiac arrest, 62-63 Trastuzumab-related cardiotoxicity, 108
acute management in, 62-63 Tricuspid regurgitation, 74t, 82
definition of, 62 Tricuspid stenosis, 73t. 82
device therapy for prevention of, 63 Tricuspid valve surgery, in tricuspid regurgitation. 82
epidemiology, 62 Turner syndrome. 92
risk factors for, 62 Tyrosine kinase inhibitors, 108
Supraventricular tachycardias (SVTs). 53-55, QlOO
atrial tachycardia, 54 u
atrioventricular nodal reentrant tachycardia, 54, 54f Ultrafiltration, 39
atrioventricular reciprocating tachycardia, 54-55 Unfractionated heparin (UFH)
premature atrial contractions, 53-54 for anticoagulation during pregnancy, 1 12-113, 1 1 2t
Surgical myectomy, for hypertrophic cardiomyopathy, 46, 46f in non-ST-elevation acute coronary syndromes, 26
Surveillance in STEMI. 23
in Marfan syndrome, Qll 7 Upper extremity peripheral arterial disease, 1 0 1 . Q108
for thoracic aortic aneurysms, 96-97
SYNTAX score, 18 v
Systemic inflammatory conditions. and risk of cardiovascular disease. 3 Valsartan. Q9
for coronary artery disease, 28t
T Valvular heart disease (VHD), 71 -86
Tachycardia-mediated cardiomyopathy, 42 aortic regurgitation, 79-80
Tachycardias, 53-55, 59-61 aortic stenosis, 77-79
approach to patient with, 53 bicuspid aortic valve, 80
atrial. 54 cardiac dysfunction and, 71
atrioventricular nodal reentrant, 54, S4f diagnostic evaluation of, 71-75, 72f
atrioventricular reciprocating, 54-55 clinical grading of murmurs, 71, 72t
evaluation in, 53 history and physical examination, 71-72
premature atrial contractions, 53-54, 59-60 laboratory and imaging tests, 72, 75
sinus, 53 valve lesions and heart murmurs, 73-74t
supraventricular, 53-55 infective endocarditis, 83-85
ventricular, 59, 60 management of. 75-77, 7St, 76t
wide-complex, 59-61, QSl mitral regurgitation, 81-82
Takotsubo cardiomyopathy, 30, 41, Q21 mitral stenosis, 80-81
Telmisartan, for coronary artery disease, 28t pathophysiology of. 71
Tenecteplase, Q92 and prosthetic valves, 85-86
i n STEMI. 22t serial evaluation in, 76, 77t
Tetralogy of Fallot, 92-93, 93f, Ql stages of progression of. 7St
diagnostic evaluation after repair of, 93-94 tricuspid regurgitation, 82
surgical repair of, 93 tricuspid stenosis, 82
Thoracic aorta. imaging of, 95, 96t Vasoactive medications, for cardiogenic shock, 39, 39t
Thoracic aortic aneurysms, 96-98, 97t Vasodilator nuclear perfusion imaging, Q86
causes of, 96. 97t for coronary artery disease. 6t
cystic medial degeneration and, 96 Vasopressin antagonists. for hyponatremia, 39
Marfan syndrome and, 96, 97 Vasopressin, for cardiogenic shock, 39t
repair of, 97-98 Ventricular arrhythmias, STEM! and, 24
rupture of, 96 Ventricular assist devices (VADs), 40
smaller, management of, 96 Ventricular septa! defects (VSDs), 89-90, 90f, Q45, Q105
surveillance for, 96-97 clinical presentation of, 90
Thoracic endovascular aortic repair (TEVAR), 97-98 diagnostic evaluation of, 90
Three-dimensional echocardiography, for structural heart disease, 13t follow-up after closure, 90
Thrombolytic t herapy, for ST-elevation myocardial infarction. 20-23, Q92 muscular. 89
agents for. characteristics of, 22t perimembranous, 89
contraindications to, 22t pregnancy in, 90
Thyroid studies. in heart failure, Qll6 STEM! and, 25
Ticagrelor subpulmonary, 89
after drug-eluting stent placement, Q44 treatment of, 90
for coronary artery disease, 28t Ventricular tachycardia (VT). 59
for non-ST-elevation acute coronary syndromes, 26 idiopathic. 60
TIMI risk score, for non-ST-elevation acute coronary syndromes, 25-26, with structural heart disease, 60, 60f
26t, 27f Verapamil
Tirofiban, for acute coronary syndromes, 23 for angina, 18
Tobacco use, and risk of cardiovascular disease, 2 for arrhythmias, sot
Toe-brachia! index. 102, Q26 for atrial tachycardia, 54
Trandolapril, for coronary artery disease, 28t for cardiac disorders in pregnancy, l l l t
Transcatheter aortic valve replacement (TAVR), 78, 79 for coronary artery disease. 29t
Transesophageal echocardiography (TEE) for hypertrophic cardiomyopathy, 44
for imaging of thoracic aorta, 95, 96t for idiopathic ventricular tachycardia, 60
in infective endocarditis, 83
i n mitral regurgitation, Q49 w
for perivalvular abscess, QI07 Warfarin
for structural heart disease, 12, 12f, 13t for anticoagulation during pregnancy, 1 1 2-113, 112t
i n valvular heart disease. 72 for patients with prosthetic valves, 86
Transthoracic echocardiography (TIE). 9 . 12. Q3 for peripartum cardiomyopathy, 110
i n aortic stenosis, 78 for pregnant women with mechanical valve prosthesis,
in atrial septa! defect, 87, 89 Q60
i n bicuspid aortic valve, Q85, Q97 for stroke prevention in atrial fibrillation, 57, 57t, Q4, Q70
i n heart failure, 33 Wolff-Parkinson-White (WPW) syndrome, 54-55, QllS
for imaging of thoracic aorta, 95, 96t Women, cardiovascular disease in, 1. 30-31

223

You might also like